• 7.99 MB
  • 2021-06-09 发布

2018年高考考点完全题语文考点通关练全书word

  • 520页
  • 当前文档由用户上传发布,收益归属用户
  1. 1、本文档由用户上传,淘文库整理发布,可阅读全部内容。
  2. 2、本文档内容版权归属内容提供方,所产生的收益全部归内容提供方所有。如果您对本文有版权争议,请立即联系网站客服。
  3. 3、本文档由用户上传,本站不保证质量和数量令人满意,可能有诸多瑕疵,付费之前,请仔细阅读内容确认后进行付费下载。
  4. 网站客服QQ:403074932
第一部分 语言文字运用 考点一 正确使用词语(虚词、实词)‎ 考点名片 考点内容 词语的考查,2016年全国卷首次结合语言表达的连贯考查虚词,重点考查副词、介词和关联词的正确使用。‎ 考查形式 以文段中副词、介词和关联词的辨析和使用的考查方式为主,选择题,分值3分。‎ 趋势分析 ‎2018年仍将是考查重点,题型仍以选择题为主。同时要注意实词与虚词的运用合在一起来考查的情况,所选语料也将更加注重生活化。‎ ‎1.填入下面文段空白处的词语,最恰当的一项是(  )‎ 特朗普__①__时常语出惊人、气焰嚣张,还曾更换党派,__②__正因如此,他显得更真实。卢卡斯说:“特朗普就像一个糟糕的政治爵士乐手,心里想什么就说什么。这一点什么时候会成为他的致命伤有待密切观察。”__③__,对希拉里·克林顿来说,寄希望于特朗普搬起石头砸自己的脚__④__是不够的。拉米说:“__⑤__希拉里真想有机会取胜,__⑥__她的言谈举止必须表现出一定的人性光辉,必须能够和普通的选民进行真实的交流。”‎ ‎①‎ ‎②‎ ‎③‎ ‎④‎ ‎⑤‎ ‎⑥‎ A 虽然 但也许 但是 ‎/‎ 如果 那么 B 不但 ‎/‎ 毕竟 还 可能 但 C 由于 可 虽然 仍然 ‎/‎ 于是 D ‎/‎ 或许 首先 也 倘若 不过 答案 A 解析 本题考查的是考生正确使用关联词语的能力。要分析句间的关系和前后关联词的搭配。第①处和第②处构成转折关系,应填写转折关系的关联词;第③处构成转折关系,应填写“但是”;第④处与前文衔接紧密,无须关联词;第⑤处和第⑥处构成假设关系,应该填写“如果……那么……”。故答案选A。‎ ‎2.填入下面文段空白处的词语,最恰当的一组是(  )‎ 博大精深的《红楼梦》,__①__达到了中国古代小说的顶峰,__②__堪称世界文学宝库中的一颗璀璨明珠。其内容之丰厚,思想之深邃,艺术之精妙,__③__不是轻易能够领会的。“满纸荒唐言,一把辛酸泪!都云作者痴,谁解其中味?”《红楼梦》第一回的这首诗,__④__作者对知音的期待,__⑤__对后世每一个读者的诱惑。翻开《红楼梦》吧!走进那个奇妙的艺术世界,你__⑥__会获得前所未有的精神享受。‎ ‎①‎ ‎②‎ ‎③‎ ‎④‎ ‎⑤‎ ‎⑥‎ A 不但 也 都 既是 更是 也许 B 不仅 而且 全 不是 而是 就 C 不仅 也 都 既是 也是 一定 D 不但 而且 因此 是 更是 将 答案 C 解析 本题考查正确使用关联词语的能力。答题时要根据句子之间的逻辑关系,选择合适的词语。注意常见关联词的搭配。可以从最后一处排除,最后一句是肯定红楼梦的价值,所以应选表达肯定意思的词语“一定”“就”“将”;③处填表因果关系的“因此”不通顺;④⑤两空是并列关系,但并不是否定前者。由此排除A、B、D三项,直接选C项。‎ ‎3.填入下面文段空白处的词语,最恰当的一组是(  )‎ 生存无忧,__①__谋生态——这是人民的要求。近些年来,__②__我们在生态建设方面付出了前所未有的努力,__③__连日来浓雾重锁的城市污染又警醒我们:空气污染防治__④__面临巨大挑战。政府提出“美丽中国”的概念,表明了追求生态文明的决心,__⑤__从源头上扭转了生态环境恶化趋势,我们__⑥‎ ‎__可能拥有天蓝、地绿、水净、风清的美丽中国。美丽中国,应从健康呼吸开始。‎ ‎①‎ ‎②‎ ‎③‎ ‎④‎ ‎⑤‎ ‎⑥‎ A 因而 虽然 可 ‎/‎ 只要 就 B 所以 当然 而 仍然 ‎/‎ 还 C 从而 尽管 ‎/‎ 还是 一旦 也 D 进而 ‎/‎ 但 依然 只有 才 答案 D 解析 本题考查正确使用关联词语的能力。语段第一句,分句间为递进关系。第三句是条件关系。结合所给答案,“只有”强调的是必要条件,即“才”后面的内容要实现就必须做到“只有”的条件,且条件往往是唯一的;而“只要”则强调的是充分条件,即“就”后面的内容要实现,“只要”的条件是必备之一,但不是唯一。故答案为D。‎ ‎4.填入下面文段空白处的词语,最恰当的一组是(  )‎ 艰苦奋斗__①__是一种进取的积极精神,__②__是人类生存发展的客观需要。__③__一个民族,__④__丧失了艰苦奋斗的精神,__⑤__它暂时也可能很富有,__⑥__是一个没有前途和希望的民族。‎ ‎①‎ ‎②‎ ‎③‎ ‎④‎ ‎⑤‎ ‎⑥‎ A ‎∕‎ 既然 就 只有 即使 也 B 既 也 ‎∕‎ 由于 所以 还 C 或者 或者 除非 ‎∕‎ 虽然 终究 D 不仅 而且 ‎∕‎ 如果 尽管 却 答案 D 解析 解答此题要分析句间的关系和前后关联词语的搭配。第①处和第②处之间构成递进关系;第③处不需要关联词;第④处和第⑤处之间构成假设关系;第⑤处和第⑥处构成转折关系。‎ ‎5.填入下面文段空白处的词语,最恰当的一组是(  )‎ 小说家应尽可能把人物对话写得流利自然,__①__生动活泼,__②__不能完全像实际说话。__③__讲故事或作报告,__④__又绝不能像日常说话那样支离破碎,__⑤__不写稿子,__⑥__应像一篇文章。‎ ‎①‎ ‎②‎ ‎③‎ ‎④‎ ‎⑤‎ ‎⑥‎ A ‎∕‎ 虽然 而 却 即使 也 B 尽管 可是 ‎∕‎ 而 虽然 但 C 尽管 ‎∕‎ 而 却 虽然 但 D 虽然 相反 可 即使 ‎∕‎ 也 答案 A 解析 解答此题要分析句间的关系和前后关联词语的搭配。第①处“流利自然”和“生动活泼”都做“写”的补语,不需要关联词;第②处、第③处和第④处之间构成转折关系;第⑤处和第⑥处构成假设关系。‎ ‎6.填入下面文段空白处的词语,最恰当的一组是(  )‎ 我们应该明白,__①__当前在我们的国有企业中出现的矛盾和问题,__②__有诸多方面的原因,__③__根本原因在于原有体制的弊端还没有消除。__④__,要尽快使国有企业解脱困境,__⑤__通过进一步深化改革,__⑥__能实现。‎ ‎①‎ ‎②‎ ‎③‎ ‎④‎ ‎⑤‎ ‎⑥‎ A 尽管 既然 那么 所以 只有 ‎/‎ B 如果 虽然 ‎/‎ 但是 只要 就 C ‎/‎ 虽然 但是 因此 只有 才 D 虽然 ‎/‎ 那么 因此 只要 就 答案 C 解析 要分析句间的关系和前后关联词的搭配。第①处不需要关联词语;第②、③处之间构成转折关系;第④处应该用表结论的关联词“因此”;第⑤、⑥处之间构成必要条件关系。‎ ‎7.填入下面文段空白处的词语,最恰当的一组是(  )‎ 青年人富有理想和担当,憧憬着美好的未来,这是青年的特点,__①__是优点。__②__需懂得,个人的抱负不可能孤立地实现,__③__把它同时代和人民的要求紧密结合起来,用自己的知识和本领为祖国为人民服务,__④__能使自身价值得到充分实现。__⑤__脱离时代,脱离人民,__⑥__必将一事无成。‎ ‎①‎ ‎②‎ ‎③‎ ‎④‎ ‎⑤‎ ‎⑥‎ A 更 同时 ‎/‎ 就 如若 则 B 也是 而且 除非 ‎/‎ 与其 也就 C 也 但 只有 才 如果 ‎/‎ D 更是 况且 唯有 才能 ‎/‎ 终究 答案 C 解析 本题考查的是考生正确使用关联词语的能力。要分析句间的关系和前后关联词的搭配。第①处与前句构成并列关系,应填写并列关系的关联词;第②处构成转折关系;第③处和第④处构成条件关系,前文是条件,后文是在此条件下产生的结果;第⑤处和第⑥处构成假设关系,故第⑤处应填写假设关系的关联词;第⑥处后有表肯定内容的“必”与前文照应,无需再填写关联词。故答案选C。‎ ‎8.填入下面文段空白处的词语,最恰当的一项是(  )‎ 自从英国数学家图灵提出“机器能思考吗”的命题,人工智能就开始进入人类的思想,__①__人类的全部尊严就在于思想,人工智能作为人类思想的产物,__②__将再次证明人类的伟大。因此,__③__“阿尔法围棋”与李世石谁将获胜,__④__都将是人类思想的高光时刻,为此,我们有必要重温帕斯卡尔的这句话——“人的伟大——我们对于人的灵魂具有一种如此伟大的观念,__⑤__我们不能忍受它人蔑视,或不受别的灵魂尊敬;而人的全部的幸福__⑥__在于这种尊敬。”‎ ‎①‎ ‎②‎ ‎③‎ ‎④‎ ‎⑤‎ ‎⑥‎ A 可是 就 与其 其实 因为 ‎/‎ B ‎/‎ 必 不管 实际上 尤其 就是 C 而 也 无论 ‎/‎ 以致 就 D 既然 ‎/‎ 遑论 必然 由于 必将 答案 C 解析 本题考查的是考生正确使用关联词语的能力。要分析句间的关系和前后关联词的搭配。第①处构成转折关系,应填写转折关系的关联词;第②处表同样之意,应填写“也”;第③处与后文构成条件关系,应该填写条件关系的关联词;第④处后的副词“都”与前文条件关系照应,无需填写其他关联词;第⑤处表示由前文情况所导致的后果;第⑥处只有副词“就”能与“在于”相搭配。故答案选C。‎ ‎9.填入下面文段空白处的词语,最恰当的一项是(  )‎ 我为什么开始就讲要树立信心,而且反复讲,到处讲,讲信心比货币比黄金还重要,就是__①__一个民族、一个国家在困难的时候,__②__失掉信心,那么__③__失掉了一切,有了信心,__④__才有力量的源泉,才有勇气。我要说有信心,__⑤__针对当前应对金融危机的困难,__⑥__要面向未来长远发展。我相信你们只要好好学习,就一定会有用武之地。‎ ‎①‎ ‎②‎ ‎③‎ ‎④‎ ‎⑤‎ ‎⑥‎ A 如果 ‎/‎ 也就 所以 既 又 B 假如 因此 ‎/‎ 也 之所以 是因为 C 因为 如果 就 ‎/‎ 不仅 而且 D 由于 宁可 也不 然后 既是 ‎/‎ 答案 C 解析 本题考查的是考生正确使用关联词语的能力。要分析句间的关系和前后关联词的搭配。第①处与前文构成倒置的因果关系,应填写“因为”;第②处和第③处构成假设关系,应该填写“如果……就……”;第④处后面已有关联词“才”,无须再用关联词;第⑤处和第⑥处构成递进关系,应该填写“不仅……而且……”。故答案选C。‎ ‎10.填入下面文段空白处的词语,最恰当的一项是(  )‎ 在中国文化中,龙有着重要的地位__①__影响。从距今7000多年的新石器时代,先民们对原始龙的图腾崇拜,到今天人们__②__多以带有“龙”字的成语__③__典故来形容生活中的美好事物。上下数千年,龙已渗透到中国社会的各个方面,成为一种文化的凝聚和积淀。龙成了中国的象征、中华民族的象征、中国文化的象征。__④__对每一个炎黄子孙来说,龙的形象是一种符号、一种意绪、一种血肉相连的情感。“龙的子孙”“龙的传人”这些称谓,常令我们激动、奋发、自豪。龙的文化除了在中华大地上传播承继外,还被远渡海外的华人带到了世界各地,在世界各国的华人居住区域,最多和最引人注目的饰物__⑤__是龙。__⑥__,“龙的传人”“龙的国度”也获得了世界的认同。‎ ‎①‎ ‎②‎ ‎③‎ ‎④‎ ‎⑤‎ ‎⑥‎ A 或 还 ‎/‎ 但是 依然 所以 B 和 仍然 或 ‎/‎ 仍然 因而 C ‎/‎ 还是 或者 可是 就 因此 D 及 也是 及其 这样 仅仅 ‎/‎ 答案 B 解析 本题考查的是考生正确使用关联词语的能力。要分析句间的关系和前后关联词的搭配。第①处和前面构成并列关系,应填写连词“和”或“及”;第②处表示某种情况持续不变,应填写“仍然”;第③处构成选择关系,应填写选择关系的连词“或”或“或者”;第④处“对每一个炎黄子孙来说”充当整个句子的前置状语,无需关联词;第⑤处表示某种情况持续不变,应填写“仍然”;第⑥处构成因果关系,应填写因果关系的连词。故答案选B。‎ ‎11.填入下面文段空白处的词语,最恰当的一项是(  )‎ ‎“周庄的商业气息太浓了!”听见一些游过周庄的人这么说,似乎尽兴之后还带着一种遗憾。__①__这是把周庄理解错了,__②__当成了桃花源。__③__我听到这样的感叹倒是欣慰,__④__我从受众者那里看到了世俗文化的力量。周庄__⑤__不是桃花源,“商业气息”是说对了,周庄一直就是个商业繁华的集镇,可以说,__⑥__没有商业,就没有周庄,与今天许多享誉中外的名镇、名城一样。‎ ‎①‎ ‎②‎ ‎③‎ ‎④‎ ‎⑤‎ ‎⑥‎ A 实在 而 ‎/‎ 由于 固然 既然 B 其实 ‎/‎ 不过 因为 从来 如果 C 原本 是 但是 ‎/‎ 不但 假设 D 原来 而是 然而 而且 ‎/‎ 哪怕 答案 B 解析 本题考查的是考生正确使用关联词语的能力。要分析句间的关系和前后关联词的搭配。第①处承上文转折,表示所说的是实际情况,应填写“其实”;第②处是对前文内容的补充,无需关联词;第③处与前文构成转折关系,应填写转折关系的关联词;第④处与前文构成倒置的因果关系,应填写“因为”;第⑤处表示从来如此之意,应填写“从来”;第⑥处与后面分句构成假设关系。故答案选B。‎ ‎12.填入下面文段空白处的词语,最恰当的一组是(  )‎ ‎__①__多数人不提倡以高考作文作为冒险式的想象试验,但__②__难说个别考生会不会有如此的理解和想象。万一出现类似的理解与想象,__③__该怎样给分?显然,__④__想象空间越大,作文的理解方式就会越多,评分的标准也会越复杂。__⑤__阅卷者把飞鸟般的想象缩窄为井蛙式的想象,给那些不合自己预设想象的作文以低分,__⑥__这样的评分标准呈现的是另一幅漫画——给老实理解者以香吻,给特别想象者以巴掌……‎ ‎①‎ ‎②‎ ‎③‎ ‎④‎ ‎⑤‎ ‎⑥‎ A ‎/‎ 很 又 这里 因为 所以 B 尽管 也 那么 ‎/‎ 如果 那么 C 固然 ‎/‎ 应 反而 除了 还有 D 由于 就 ‎/‎ 任凭 不但 况且 答案 B 解析 本题考查的是考生正确使用关联词语的能力。要分析句间的关系和前后关联词的搭配。第①处和后面的“但”构成转折关系,所以选“尽管”,如利用排除法正确答案已选出,但保险起见,再进行下面的筛选;第②处“也”是表转折和让步关系的副词,与前文转折关系照应;第③处问句内容与前文构成假设关系,选“那么”;第④处陈述事实,不用关联词;第⑤处和第⑥处构成假设关系。故答案选B。‎ ‎13.填入下面文段空白处的词语,最恰当的一组是(  )‎ 勇敢做自己__①__要你去做酷哥辣妹,__②__面对真实的自己,勇于表达真正的情感,有自信心,却不随波逐流。拥有自信,__③__不是去做一个和社会道德格格不入的人。我认为“我喜欢,有什么不可以”,并不代表你很“酷”,相反地,__④__表示你很“逊”,__⑤__有自信的人,也要了解有些事可以做,有些事不能做,__⑥__凡事以自我为中心,并不是高明的做法。‎ ‎①‎ ‎②‎ ‎③‎ ‎④‎ ‎⑤‎ ‎⑥‎ A 与其 不如 ‎/‎ 只是 哪怕 何况 B 不是 而是 并 却 因为 ‎/‎ C 不 但要 虽然 ‎/‎ 甚至 不管 D 既然 就 但 还 ‎/‎ 只要 答案 B 解析 本题考查的是考生正确使用关联词语的能力。要分析句间的关系和前后关联词的搭配。第①处和第②处构成并列关系,前一句否定一种意思,后一句肯定另一种意思,只能用“不是……而是”;第③处“不是”和后面内容构成否定关系,“并”放在否定词前加强这种语气;第④处与前文构成转折关系,用“却”合适;第⑤处与前文构成倒置的因果关系;第⑥处是对前文结果的补充,不用关联词。故答案选B。‎ ‎14.填入下面文段空白处的词语,最恰当的一组是(  )‎ 谈到疯狂的建筑师高迪和他的建筑,__①__要提到空间几何。虽然空间几何看上去是那样简单,__②__人们却难以用语言进行解释。__③__青年时代起,高迪就对这门学科痴迷不已,__④__影响了他的一生。尽管高迪的灵感来自于生物和自然界的形态,__⑤__他所有的作品__⑥__暗含了严格的公式、计算,将艺术与技术完美结合。‎ ‎①‎ ‎②‎ ‎③‎ ‎④‎ ‎⑤‎ ‎⑥‎ A 一定 ‎/‎ 从 以至于 但是 都 B ‎/‎ 但 首先 然后 然而 却 C 也 可是 ‎/‎ 并 不过 反而 D 或者 甚至 因为 最终 ‎/‎ 无不 答案 A 解析 本题考查的是考生正确使用关联词语的能力。要分析句间的关系和前后关联词的搭配。第①处表态度上的肯定,应填写“一定”;第②处虽然构成转折关系,但后面的句中有“却”,无须再用关联词;第③处为固定搭配“从……起”;第④处表明前文内容所述情况所达到的深度和结果,应填写“以至于”;第⑤处与前文构成转折关系;第⑥处与前文“所有的作品”照应,应填写“都”。故答案选A。‎ ‎15.填入下面文段空白处的词语,最恰当的一组是(  )‎ 幸福是什么?幸福就是自己觉得幸福。__①__这是一句人人皆知的大白话。__②__我却知道,有不少人,__③__很多人并非为自己的感觉,__④__为了他人的观瞻而去构建自己的人生与生活。__⑤‎ ‎__窥察别人的生活与家庭,__⑥__成为他们生活的另一部分。‎ ‎①‎ ‎②‎ ‎③‎ ‎④‎ ‎⑤‎ ‎⑥‎ A 因为 但是 或者 也不是 不但 而且 B 也许 而 甚至 而是 因而 便 C 毕竟 然则 或许 还 与其 不如 D 因此 不过 甚或 不是 只有 才 答案 B 解析 本题考查的是考生正确使用关联词语的能力。要分析句间的关系和前后关联词的搭配。第①处突出后文内容的可能性,应填写“也许”;第②处与前文构成转折关系,应填写“而”;第③处与前文构成递进关系,应填写递进关系的关联词;第④处与前文构成并列关系,应填写“而是”;第⑤处与前文内容构成因果关系,应填写“因而”;第⑥处根据前后文应填写副词“便”。故答案选B。‎ ‎ [3年高考真题集训]‎ ‎1.[2016·全国卷Ⅰ]填入下面文段空白处的词语,最恰当的一组是(  )‎ 我们曾说,中学生初学文言文时__①__不要依赖译文。__②__并不是说在整个学习过程中绝对不去参看译文。其实,__③__肯动脑筋,__④__不盲目机械地看待译文,__⑤__,只要译文不是太差,看看译文也无妨。有时候把译文跟注释对照起来揣摩学习,__⑥__不失为一种可行的方法。‎ ‎①‎ ‎②‎ ‎③‎ ‎④‎ ‎⑤‎ ‎⑥‎ A ‎/‎ 这 如果 而且 那么 也 B 最好 当然 一旦 ‎/‎ 而且 就 C 一定 也 如果 并且 因此 ‎/‎ D 尽量 ‎/‎ 因为 进而 所以 仍 答案 A 解析 首先通读语段,理解段意。该语段阐述的是一个辩证的观点:中学生初学文言文时,不要依赖译文,但可以参看译文。然后使用排除法确定答案。第③处使用“一旦”与语境不符,因为“一旦”一般导致不好的结果;第③处后面的内容并不是原因,使用“因为”与语境不符,排除B、D两项。第⑤处前后文不是因果关系,不能用“因此”,排除C项。‎ ‎2.[2016·全国卷Ⅱ]填入下面文段空白处的词语,最恰当的一组是(  )‎ 比尔·布莱森在他的《万物简史》里介绍了超级火山的巨大破坏性。以美国为例,__①__境内有一座超级火山喷发,__②__其产生的巨大能量将推毁数千公里范围内的所有东西,无数人会因此丧命,__③__会导致整个国家被深达6~20米的火山灰覆盖,随后__④__会出现其他许多可怕后果。__⑤__目前人类还无法预测美国超级火山会在何时喷发,__⑥__了解了它的杀伤力有利于我们制订各种减损预案。‎ ‎①‎ ‎②‎ ‎③‎ ‎④‎ ‎⑤‎ ‎⑥‎ A 一旦 则 ‎/‎ 也 即使 然而 B 倘若 那么 进而 ‎/‎ 由于 所以 C 假如 则 甚至 更 ‎/‎ 那么 D 只要 ‎/‎ 而且 还 虽然 但 答案 D 解析 本题考查正确使用关联词语的能力。解答此类试题时要注意分析句间的关系和前后关联词的搭配。第①处是表示后面情况发生的条件,应填写“只要”;第②处是结果,无须关联词语;第③处是表示更进一步,应填写表示递进的关联词;第④处,紧承上文意为还会出现许多后果;第⑤处和第⑥处构成转折关系。‎ ‎3.[2016·全国卷Ⅲ]填入下面文段空白处的词语,最恰当的一组是(  )‎ 有的人在填报高考志愿时选报热门专业,理由是能学以致用,__①__是一种误解。学以致用的真正含义是将学到的知识用于实践,__②__不是看什么东西有用才决定去学。屏弃功利性__③__使人抱着乐观的态度去学习;__④__有用才去学习会使人产生心理负担,__⑤__总要担心以后会不会真的有用。抱着功利之心去挑选专业,往往会牺牲自己真正的兴趣,__⑥__毕业后谋到了不错的职位,也不一定就工作得很开心。‎ ‎①‎ ‎②‎ ‎③‎ ‎④‎ ‎⑤‎ ‎⑥‎ A 其实这 而 要 确定 所以 ‎/‎ B 这其实 ‎/‎ 能 认为 因为 即使 C 实际上 却 会 ‎/‎ 可能 就是 D 这 当然 就是 如果 ‎/‎ 虽然 答案 B 解析 ①处所填内容中需要有一个指示代词,用来指代前面内容,充当主语,而C项的“实际上”缺少主语,所以可排除C项;②处前后是并列关系,而D项“当然”表示对前文的认可推理,所以可以排除D项;③处所在分句意在点明“屏弃功利性”的作用,而A项的“要”则使该分句指向了“屏弃功利性”的具体做法,故不当;④处表示一种主观的认识,故用表示主观态度的“认为”较好;⑤处前后有明显的因果关系,而且是前果后因,故只能用“因为”;⑥处表示假设关系,选用“即使”。综合以上分析即可得出答案。‎ ‎4.[2016·山东高考]依次选用文中括号里的词语,最恰当的一项是(  )‎ 黟县的西递和宏村,拥有蜚声海内外的徽派建筑群。两村背依青山,清流抱村穿户。数百幢明清时期的民居静静伫立。高大奇伟的马头墙有骄傲睥睨的表情、跌宕飞扬的韵致,灰白的屋壁被时间涂画出斑驳的线条。李白的“黟县小桃园,烟霞百里间。地多灵草木,人尚古衣冠”,道出了这里山水风物的(优美/幽美)、民风人情的淳厚从容。要真正(领略/领悟)徽派建筑之美,该是在西递村。在都市的喧哗之外,西递向我们呈现了一种宁静质朴的民间生活。从远处望去,西递是一片线条简洁的黑瓦铺成的屋顶和高大的白墙,黑白相间,错落有致。迈入老屋你会发现,这些老屋内部的(繁杂/繁复)精致与外部的简洁纯粹形成鲜明的对照,徽派建筑中著名的三雕——木雕、砖雕、石雕在这里体现得淋漓尽致。‎ A.优美 领略 繁杂 B.幽美 领略 繁复 C.幽美 领悟 繁杂 D.优美 领悟 繁复 答案 B 解析 本题考查正确使用词语(包括熟语)的能力。优美:美好;美妙。幽美:①形容地方很偏僻、光线暗;②使人感觉宁静、安闲的;③幽静美丽;幽雅,幽深。领略:领会,理解,理会,理睬等。领悟:体会,解悟。繁杂:事情多,繁琐而杂乱。繁复:指繁多复杂。‎ ‎5.[2015·湖北高考]依次填入下列横线处的词语,最恰当的一组是(  )‎ 研究伊始,该团队选取了华北、西北地区生产的几十种马铃薯进行分析________,从营养成分、________、硬度等方面多次试验,________确定了适合加工马铃薯面条的两个品种。随后,又从诸多面粉种类中试验选取了________的小麦粉加以调试。‎ A.鉴别 色泽 终于 适量 B.鉴别 色彩 终于 适当 C.甄别 色泽 最终 适当 D.甄别 色彩 最终 适量 答案 C 解析 本题考查近义实词、虚词的辨析。A、B、C、D四个选项由四组词语交叉组合,要比较分析每组近义词的区别,就要结合语境。鉴别:指分析事物真假,多用于古物鉴定;甄别:强调认真慎重地辨别。文段中对马铃薯研究分析,用“甄别”较好。色泽:指颜色和光泽;色彩:指物体呈现给人的一种彩色视觉。文段中与“营养成分和硬度”相并列,用“色泽”较好。终于:指到底,终究;最终:是最后,末了,与最初相对。二词比较,“终于”多用于希望达到的结果,更符合人的期望,常有明显的主观色彩,而“最终”相对立场比较客观。文段中陈述的是科学研究,用“最终”‎ 较好。适量:强调在数量上合适;适当:包括范围更广,强调恰当。文段中,强调从“种类”中选取,显然不能用“适量”而要用“适当”。‎ ‎6.[2014·重庆高考]依次填入下列横线处的词语,最恰当的一组是(  )‎ ‎①再全面的维生素补充剂对健康的弥补作用也不能________膳食结构不合理带来的损害。想要保持健康,更重要的是维持饮食的平衡以及适度运动。‎ ‎②在自然环境中怎样才能________病虫害的侵袭呢?与松树共生,就是杨树通过长期自然演化选择的一种自我保护方式。‎ ‎③有些人严重缺乏安全感,他们把说谎作为一种自我________手段,总是下意识地保护自己,不愿自己的任何行为和心思被人知道。‎ ‎④现实生活中没有法外之地,互联网同样没有。查处淫秽网站,________网络暴力,是净化网络环境的需要,更是建立法治社会的需要。‎ A.抵制 抵消 防御 抵御 B.抵消 抵御 防御 抵制 C.抵制 防御 抵御 抵消 D.抵消 抵制 抵御 防御 答案 B 解析 抵消:两种事物的作用因相反而互相消除。此词适用于第①句的语境。抵御:抵挡;抵抗。多用于外来事物的侵害等,在②句中符合语境。防御:抗击敌人的进攻,保护自己,与③中的“自我”搭配恰当。抵制:阻止某些事物,使不能侵入或发生作用,后面应与名词搭配。‎ ‎[2年全国模拟重组]‎ ‎1.[2017·衡水中学摸底]填入下面文段空白处的词语,最恰当的一组是(  )‎ ‎__①__探索半人马座α星目前是科幻,__②__不排除人类科技进一步发展后,未来可以实现这种设想,__③__那也只是将来时。从眼下的现实来看,霍金能提出这种想法完全依赖于现有科技的发展和现实水平。具体而言,__④__霍金要表达各种思想和学说,包括人类探索半人马座α星,__⑤__必须要依赖他不可须臾离开的轮椅。轮椅__⑥__是霍金的代步工具,更重要的是,有了它,霍金才能与他人交流并表达自己的思想,包括目前探索半人马座α星的科幻想法。‎ ‎①‎ ‎②‎ ‎③‎ ‎④‎ ‎⑤‎ ‎⑥‎ A 尽管 但 但 即便 也 不仅 B 虽然 但是 所以 如果 那么 不仅 C 尽管 但 因此 只要 就 就 D 因为 所以 但 即便 也 就 答案 A 解析 本题考查关联词语,可以结合句间关系考虑,同时运用排除法。“探索半人马座α星目前是科幻”“不排除人类科技进一步发展后”之间应是转折关系,由此可以排除D项;“未来可以实现这种设想”“那也只是将来时”之间也是转折关系,由此可以排除B、C两项;“霍金要表达各种思想和学说”与“必须要依赖他不可须臾离开的轮椅”之间的关系是假设关系,应用“即便……也”。由此可以排除B项。‎ ‎2.[2017·新余一中调研]填入下面文段空白处的词语,最恰当的一组是(  )‎ 怎样学习文本呢?不设计程序,不规定时间,__①__同学们自由阅读,__②__阅读要认真,每篇文章都要逐字逐句读懂,把文章的结构和思路都搞清楚,__③__把握文章的思想内容。__④__要揣摩语言运用的妙处,__⑤__重要的语句要细细领会。这样,反复玩味,就得到思想感情上的陶冶__⑥__文学欣赏上的喜悦。‎ ‎①‎ ‎②‎ ‎③‎ ‎④‎ ‎⑤‎ ‎⑥‎ A ‎/‎ 而且 接着 而且 关于 或者 B 以便 但是 进而 同时 对 和 C 因此 然而 ‎/‎ 而且 关于 以及 D 为了 同时 才能 同时 ‎/‎ 以及 答案 B 解析 本题考查关联词的运用和语言表达连贯的能力。“不设计程序,不规定时间”的目的是“同学们自由阅读”,因此①处应填表示目的关系的连词“以便”;“阅读要认真”和前面“同学们自由阅读”构成转折关系,因此②处应填表示转折关系的连词“但是”或“然而”;“把文章的结构和思路都搞清楚”和“把握文章的思想内容”是条件和结果的关系,因此③处应填“进而”;本文段,先说读懂文章内容,然后说欣赏语言,两者是并列关系,因此④处应填“同时”;“对”表示对象,“关于”表示范围,文段中“重要的词句”是“细细领会”的对象,因此⑤处应填“对”;“思想感情上的陶冶”“文学欣赏上的愉悦”是并列关系,因此⑥处应填“和”。‎ ‎3.[2017·广东四校联考]填入下面文段空白处的词语,最恰当的一组是(  )‎ 在卡尔·刘易斯一代退役之后,博尔特在近二十年里成为复兴田径运动的最大明星。他的每一次参赛,__①__有可能改写人类的极限;他的三届奥运会,已然改变了牙买加乃至世界对田径的看法。而30万美元的比赛出场费,__②__是如今所有田径运动员中最高的。不过,脚踏实地的博尔特却对自我、前途及未来都有着清晰的认识:“我觉得每一年都很重要,__③__今年是我的最后一届奥运会,__④__大家特别关注。我想去实现一些从来没有实现过的东西,为下几代人树立更高的标杆。我现在既兴奋又紧张,这也许是我留给后人遗产的最后机会。__⑤__我退役了,我认为自己不会立马被超越。人们__⑥__需要一些时间才能打破我的纪录。也许有一天,人们告诉我,我不再是世界上跑得最快的人了,但我还会是那个拿了9块奥运金牌的人。”‎ ‎①‎ ‎②‎ ‎③‎ ‎④‎ ‎⑤‎ ‎⑥‎ A 都 也 但 所以 即使 还 B 总 却 ‎/‎ 那么 虽然 也 C 也 ‎/‎ 但 所以 由于 更 D 常 仍 而且 ‎/‎ 一旦 甚至 答案 A 解析 ①处选“都”,呼应的是前面的“每一次”,强调的是博尔特体育技能的高超;③处选“但”,与“最后”相呼应,为转折关系;④处与前句为因果关系;⑤处为假设关系,因为博尔特在现实中仍未退役。‎ ‎4.[2017·山西五校联考]依次填入下面一段文字横线处的词语,最恰当的一组是(  )‎ 微信朋友圈里的很多人往往是现实生活中的朋友。私密性是微信朋友圈的基本属性,__①__,__②__互联网的开放性,朋友圈__③__具有公共性质。在朋友圈里__④__要遵守社交礼仪外,__⑤__恪守公共交往规则及秩序,个人在朋友圈发布__⑥__转发信息要慎重,不能盲目动指头。每个人在各自朋友圈发信息时,要承担一定的“注意义务”。‎ ‎①‎ ‎②‎ ‎③‎ ‎④‎ ‎⑤‎ ‎⑥‎ A 但是 鉴于 ‎/‎ 除了 仍要 和 B 因此 依据 还 既 还要 和 C 但是 基于 又 除了 还要 或 D 因此 依据 ‎/‎ 既 仍要 或 答案 C 解析 文段第二句前后句是转折关系,故①处选填“但是”。④⑤所在句间是递进关系而非并列关系,故选填“除了”“还要”。⑥前后的“发布”与“转发”又是选择性并列关系,应填“或”。故答案为C。‎ ‎5.[2017·江淮十校联考]填入下面文段空白处的词语,最恰当的一组是(  )‎ 电子服务设施的普及,方便了大多数人,提高了服务的现代化。__①__一些老年人,__②__是不识字、不懂电脑、视力不便的人,__③__难以充分享受这种便利,__④__可能成了“不便”。__⑤__这个群体的数量不大,但他们__⑥__是最需关注的困难群体。‎ ‎①‎ ‎②‎ ‎③‎ ‎④‎ ‎⑤‎ ‎⑥‎ A 但 和 又 更 不管 往往 B 然而 或者 还 更 不管 常常 C 但 或者 还 甚至 尽管 往往 D 然而 和 又 甚至 尽管 常常 答案 C 解析 本题适用排除法,根据句意,②处不能选用连词“和”,排除A、D两项。再从⑤处区别,“尽管”表示一种事实,后面不能用表示任指的词语。“不管”表示一种假设,后面用表示任指或选择的词语。可排除B项。故答案为C。‎ ‎6.[2017·河北联盟模拟]填入下面文段空白处的词语,最恰当的一组是(  )‎ 文学是心灵的抚慰剂。许多人因为孤独而爱好文学,许多人因为穷困而爱好文学,也有一些人因为富足闲适而爱好文学——__①__出于什么原因,__②__他们信仰文学,崇尚真善美,文学__③__会让他们变得安贫乐道,富而不骄,让他们的内心充实、自信、淡定、颖慧。我总是天真地认为,真正爱好文学的人,属于天底下最善良的群落,__④__坏,__⑤__坏不到哪里去。验之现实,大致不错。__⑥__以文学钻营的人,则另当别论。‎ ‎①‎ ‎②‎ ‎③‎ ‎④‎ ‎⑤‎ ‎⑥‎ A 无论 ‎/‎ 也就 虽然 但 如果 B 即便 凡是 ‎/‎ 任凭 也是 倘若 C ‎/‎ 但是 总 或者 不过 有些 D 不管 只要 就 即使 也 ‎/‎ 答案 D 解析 本题考查的是考生正确使用关联词语的能力。要分析句间的关系和前后关联词的搭配。第①处与前文构成选择关系,应填写选择关系的关联词;第②处和第③处构成条件关系,应该填写“只要……就……”;第④处和第⑤处构成假设关系,应该填写“即使……也……”;第⑥处“以文学钻营的人”是偏正短语作主语,无需关联词。故答案选D。‎ ‎7.[2017·长治模拟]填入下面文段空白处的词语,最恰当的一项是(  )‎ 语言的形式,__①__是美的,__②__它有整齐的美,抑扬的美,回环的美。这些美都是音乐所具备的,__③__语言的形式美也可以说是语言的音乐美。在音乐理论中,有所谓“音乐的语言”;在语言形式美的理论中,__④__应该有所谓“语言的音乐”。__⑤__音乐和语言不是一回事,__⑥__二者之间有一个共同点:音乐和语言都是靠声音来表现的。‎ ‎①‎ ‎②‎ ‎③‎ ‎④‎ ‎⑤‎ ‎⑥‎ A 只 不但 因为 就 虽然 可是 B 所以 不仅 除非 必然 无论 还是 C 首先 其次 为的是 况且 不管 但 D 之所以 是因为 所以 也 尽管 但是 答案 D 解析 本题考查的是考生正确使用关联词语的能力。要分析句间的关系和前后关联词的搭配。①②所在句子间是因果关系;⑤⑥所在句子间是转折关系。由此可排除其它三项。‎ ‎8.[2017·湖北黄石调研]填入下面文段空白处的词语,最恰当的一项是(  )‎ 注重学英语是好事,__①__同时也要十分重视母语的学习。汉语是世界上最优美、最富表现力、最有魅力的语言之一。__②__中国进一步改革开放,学习汉语的外国人越来越多,我们自己__③__应该学好汉语,用好汉语。某些人__④__以能讲英语为荣,说话时__⑤__夹上许多“洋文”,而母语的使用__⑥__白字连篇,真是出尽“洋相”。‎ ‎①‎ ‎②‎ ‎③‎ ‎④‎ ‎⑤‎ ‎⑥‎ A ‎/‎ 因为 也 总 还要 仍然 B 但 随着 更 ‎/‎ 总要 却是 C 尽管 ‎/‎ 还是 竟然 尤其 甚至 D 然而 由于 便 虽然 ‎/‎ 却 答案 B 解析 本题考查的是考生正确使用关联词语的能力。要分析句间的关系和前后关联词的搭配。第①处构成转折关系,应填写转折关系的连词;第②处表示后文所依赖的条件,应填写“随着”;第③处构成递进关系,应填写“更”;第④处在句中,前后内容连接顺畅,无须关联词;第⑤处表示经常发生的情况,应填写“总是”;第⑥处构成转折关系,应填写转折关系的连词。故答案应选B。‎ ‎9.[2017·长春质监]填入下面文段空白处的词语,最恰当的一项是(  )‎ 谣言在传递过程中遇到“智者”,__①__经不起仔细推敲。__②__“智者”听到谣言后的反应是运用独立思维进行理性判断,__③__进而得出结论,并不轻信谣言。__④__每个人__⑤__能将谣言封杀在自己的“嘴”中,那些不确切、不客观的信息的传播速度__⑥__会减缓,甚至停止。‎ ‎①‎ ‎②‎ ‎③‎ ‎④‎ ‎⑤‎ ‎⑥‎ A 总是 源于 并 不是 ‎/‎ 还 B 往往 因为 ‎/‎ 如果 都 就 C 必然 原因是 并且 何况 并不是 ‎/‎ D 如若 ‎/‎ 就会 凡是 时时 于是 答案 B 解析 本题考查的是考生正确使用关联词语的能力。要分析句间的关系和前后关联词的搭配。第①处表示时常之意,应填写“往往”;第②处与前文构成倒置的因果关系,应填写因果关系的连词;第③处表示继续向前,再进一步之意,已有“进而”,无需其他关联词;第④处和第⑥处构成假设关系,应填写“如果……就……”。故答案选B。‎ ‎10.[2017·湖北八校联考]填入下面文段空白处的词语,最恰当的一组是(  )‎ 在天上播种的方式和地面不同,地面上__①__是先播种后浇水,__②__由于我们带入太空的白色单元格是硬质材料,__③__吸水软化后,种子__④__能放进去,__⑤__我们是先浇水后播种。播种完后,我们会在装置里铺上一层保鲜膜,__⑥__和种庄稼的地膜一样。它的作用是保护植物,防止水分流失。‎ ‎①‎ ‎②‎ ‎③‎ ‎④‎ ‎⑤‎ ‎⑥‎ A 通常 ‎/‎ 只要 就 因此 也 B 一般 但 只有 才 所以 就 C 通常 但 只有 才 因此 也 D 一般 ‎/‎ 只要 就 所以 就 答案 B 解析 本题考查的是考生正确使用关联词语的能力。要分析句间的关系和前后关联词的搭配。第②处讲的与前面的不同,故使用转折词“但”。第③处和第④处构成必要条件关系,第⑤处与第②处的“由于”搭配使用的关联词是“所以”。依据这样的方式分析即可。‎ ‎11.[2016·冀州二模]依次填入下列横线处的词语,最恰当的一组是(  )‎ 那天夜晚,他抱着吉他即兴演唱了几支歌,脸上是________的表情,像孩子那样快乐,像农夫那样纯朴。我________一次感受到,好的男子汉本质上都是农夫,朴实、宁静,沉湎于自己的园地,________那是音乐、绘画,________书籍。‎ A.迷醉 不止 不管 还是 B.陶醉 不只 不管 还是 C.迷醉 不只 如果 或者 D.陶醉 不止 如果 或者 答案 A 解析 “迷醉”的程度要比“陶醉”深,它是指“沉迷,陶醉”。“不止”表示超出某个数目或范围,强调“数目多”;“不只”是“不但,不仅”之意,不能与“一次”搭配。“不管……还是……”是表示并列关系的关联词。“如果”与“或者”不搭配。‎ ‎12.[2016·沈阳二模]依次填入下面语段中横线处的词语,恰当的一组是(  )‎ ‎①无论是网上阅读,还是纸质稿阅读,我们都________有一字不肯放松的严谨态度。‎ ‎②________承认了山猫队的投篮是缺点,克利福德依然不打算把本·戈登调回轮换阵容。‎ ‎③今年夏天,我终于回到了魂牵梦萦的故乡——山西洪洞,再次走上熟悉的大街小巷,________想起许多童年的往事。‎ ‎④他________李白的自述和有关资料,认定李白确是出生在中亚细亚的碎叶城。‎ ‎⑤这样做________稳妥,但是太费事,太耗费时间,我就怕这个工程赶不过来。‎ A.必须 尽管 不免 根据 固然 B.必须 尽管 不免 根据 自然 C.必需 不管 难免 按照 自然 D.必需 不管 难免 按照 固然 答案 A 解析 本题考查正确使用词语的能力。①“必须”和“必需”都有必要的意思,区分在于前者强调必要,后者强调必要有,强调“需”的意思。本句有一个“有”字,表明强调必要,应选用“必须”。②“尽管”用于让步语气的转折复句,“不管”用于条件复句。本句前后分句是让步转折的关系,因此选用“尽管”。③“不免”强调的是在某种情况下很自然地会出现一种结果,是一种比较客观的陈述。“难免”强调的是某种结果不容易避免。多用于规律性的情况或有一种解释或宽慰的语气。本句表示很自然地想起往事,因此选用“不免”。④“根据”侧重指“结论或言行的凭据”;“按照”可用于上级的指示、精神、命令等外,还可用于情况、季节、次序、时间、道理等,适用范围更大是中性词。本句用于确定李白出生地的资料,因此选用“根据”。⑤“固然”表示承认某个事实,引起下文转折,“自然”表示当然,没有转折之意。本句后有“但是”,因此选用“固然”。‎ ‎13.[2016·辽宁葫芦岛二联]依次填入下列各句中的关联词语,恰当的一项是(  )‎ ‎①农村改革以前,这个地方非但没有水浇地,________吃的水都得到几十里以外去挑。‎ ‎②现代科学技术日新月异地发展,________从前一些神话中的幻想都可能成为现实。‎ ‎③他没有从上次的错误中接受教训,________这次又犯了更严重的错误。‎ A.以至 甚至 以致 B.以至 以致 甚至 C.甚至 以至 以致 D.以致 甚至 以致 答案 C 解析 “甚至”强调突出的事例;“以致”多表示不好的结果;“以至”用在下一句话的开头,表示结果。‎ ‎14.[2016·江西五校二模]依次填入下面语段中横线处的词语,恰当的一组是(  )‎ 发展向前的步伐________不能罔顾系统稳定问题或不理市场是否准备就绪,但若是过分________,亦可能会错失重要的战略良机,________是在今天这个竞争已然白热化、业内发展日新月异的环境下。‎ A.固然 保险 何况 B.固然 保守 特别 C.虽然 保险 特别 D.虽然 保守 何况 答案 B 解析 “固然”表示承认某个事实,引起下文转折;“虽然”用在上半句,下半句往往有“可是、但是”等跟它呼应,表示承认甲事为事实,但乙事并不因为甲事而不成立。“固然”比“虽然”语气更肯定,所以第一空应用“固然”。“保守”指维持原状,不求改进,跟不上形势的发展;“保险”指稳妥可靠。联系“过分”可知,第二空应用“保守”。“何况”用反问的语气表示更进一层的意思,与语境不符,故第三空应用“特别”。‎ ‎15.[2016·衡水中学模拟]‎ 在下面文字中的画线处填上适当的关联词语,使整个段落语意连贯,层次清楚,逻辑严密。‎ 人们都知道爱因斯坦创造了举世闻名的相对论学说,__①__很少有人确切地了解这种理论。跟我们所熟知的经典物理学相比,相对论学说中有关新概念的表述充满了数学公式和演算,__②__目前常见的有关相对论的科普书籍一涉及重要概念,__③__在表达上或含糊不清,或繁琐难懂。__④__这也不能全怪那些作者,__⑤__用非数学语言来表述那些新概念的确不是一件容易的事。‎ 答案 但是 以致 就 其实 因为(如有其他答案,只要能正确表达逻辑语义关系也可)‎ 解析 本题考查正确使用虚词。①为转折关系;②表示严重的后果;③“就”和“一”搭配;④是让步关系;⑤解释原因。‎ 考点二 正确使用词语(成语)‎ 考点名片 考点内容 熟语的考查,以成语为主,且考查重点是近义误用、望文生义、不合语境、褒贬误用、对象误用等错误类型。‎ 考查形式 以句中或语段中成语、熟语的使用和辨析的考查形式为主,选择题,分值3分。‎ 趋势分析 ‎2018年仍将是考查重点,题型以选择题为主。全国卷仍将以成语的辨析为考查重点;北京卷结合现代文阅读综合考查该考点。‎ ‎1.下列各句中加点成语的使用,全部正确的一项是(  )‎ ‎①中国共产党历经95年的峥嵘岁月,逐步走向成熟,始终不忘为民服务的初心,带领全国人民向着中华民族的伟大复兴前进。‎ ‎②大学不应该成为名利场,大学里真正的大师不仅学识渊博,有沉潜专注的研学态度,而且人格高尚,有光风霁月的谦谦风度。‎ ‎③尽管印尼的头号恐饰分子已于7月18日被政府军击毙,但观察家却担心他的继任者会青出于蓝,制造出更多的恐怖袭击事件。‎ ‎④在中国传统农业社会中,统治者施行重农抑商的政策,并以各种手段将人口固着于土地之上,使得安土重迁长期被公认为国人的传统习性。‎ ‎⑤连日奔波的市长一下车,就不顾心劳日拙,立即赶赴抗洪第一线指挥救灾,极大地振奋了广大军民抗洪救灾的信心。‎ ‎⑥南海仲裁案实质是披着法律外衣的政治闹剧,这种危言危行完全背离了《国际海洋公约》的宗旨,损害了《公约》的权威性。‎ A.①②④ B.②⑤⑥‎ C.①③⑤ D.③④⑥‎ 答案 A 解析 ①峥嵘岁月:形容不平凡的年月。使用正确。②光风霁月:形容雨过天晴时万物明净的景象。也比喻开阔的胸襟和心地。使用正确。③青出于蓝:比喻学生超过老师或后人胜过前人。褒义,使用错误。④安土重迁:安于本乡本土,不愿轻易迁移。指在一个地方住习惯了,不愿轻易搬迁,形容留恋故乡。符合语境。⑤心劳日拙:多指做坏事的人,虽然使尽坏心眼,到头来不但捞不到好处,处境反而一天比一天糟。多用作贬义词。使用错误。⑥危言危行:说正直的话,做正直的事。指正直的言行。褒义,使用错误。‎ ‎2.下列各句中加点的成语,使用最恰当的一项是(  )‎ ‎①让座是道德自律的行为,不能用暴力强求。当一种起码的道德被当作标准来要求甚至规定时,社会的道德基础就岌岌可危了。‎ ‎②尽管目前金融危机让部分企业减少了人才的需求量,但一些大中型企业从长计议,借机进行人才储备,有针对性地吸纳专业后备人才。‎ ‎③在朋友的帮助下,杨苡和资中筠这两位翻译家终于得偿所愿,异地重逢,虽然时隔多年,但两人仍是一见如故,相谈甚欢。‎ ‎④惠普网络中国高层换帅尘埃落定,11月15日,新任中国区总经理琴德智音沃在惠普“网络优势,尽数呈现”的媒体见面会上露面。‎ ‎⑤伴随着古典优雅的音乐,大幕缓缓拉开,十二名长袖善舞、身姿曼妙的舞蹈演员身着古装,为大家表演了经典的舞蹈《敦煌》。‎ ‎⑥本届书画交流活动中,书画名家纷纷献艺,一幅幅书法力透纸背,一幅幅丹青意境深远,博得观众的连声喝彩。‎ A.①③⑥ B.②③⑥‎ C.③④⑥ D.①④⑥‎ 答案 D 解析 ①岌岌可危:岌岌,山高陡峭,就要倒下的样子。形容非常危险,快要倾覆或灭亡。②从长计议:用较长的时间慎重考虑、仔细商量。此处不合语境。③一见如故:初次见面就很投缘,像老朋友一样。不合语境。④尘埃落定:尘埃,细小的灰尘。尘埃全部落下。比喻事情已经结束。⑤长袖善舞:袖子长,有利于起舞。原指有所依靠,事情就容易成功。后形容有财势会耍手腕的人,善于钻营,会走门路。此处望文生义。⑥力透纸背:透,穿过。形容书法刚劲有力,笔锋简直要透到纸张背面。也形容诗文立意深刻,词语精练。‎ ‎3.下列各句中加点成语的使用,全都正确的一项是(  )‎ ‎①尽管市面上销售的上海文艺出版社和作家出版社的两套莫言作品集的销售形势已成强弩之末,但百花文艺出版社还将在订货会上推出《莫言诺贝尔奖典藏文集》(全二十卷)。‎ ‎②入夏以来,天气异常,五风十雨,致使城市内涝、市民受灾的情况不断发生,有的人甚至因雨水过大而付出生命的惨重代价。‎ ‎③陶渊明看透了官场的黑暗,从此无意于仕途,转而求田问舍,过上寄情山水、恬然自适的隐居生活。‎ ‎④当今时代,许多人学艺不精,缺少刻苦钻研、务实上进的精神,以浮躁的心态对待工作,鲁鱼亥豕的低级失误经常发生。‎ ‎⑤公平分享发展成果,改革的动力就能充分释放,改革的活力就能极大迸发,共同富裕就能计日程功。‎ ‎⑥美国卫星追踪网站声称,美国一卫星曾拍到马航客机失联海域附近的照片,并以“可能为马航坠机地点”为题发布消息,这令此事件更加扑朔迷离。‎ A.①⑤⑥ B.③⑤⑥‎ C.②④⑥ D.①③④‎ 答案 A 解析 ①强弩之末:比喻强大的力量已经衰弱,起不了什么作用。②五风十雨:形容风调雨顺。此处望文生义。③求田问舍:指只知道置产业,谋求个人私利。比喻没有远大的志向。此处望文生义。④鲁鱼亥豕:指书籍在传写或刻印过程中的文字错误。此处不合语境。⑤计日程功:可以数着日子计算进度,形容在较短期间就可以成功。⑥扑朔迷离:形容事情错综复杂,难以辨别清楚。‎ ‎4.下列各句中加点成语(词语)的使用,全都不正确的一项是(  )‎ ‎①2016里约热内卢女子排球决赛,中国女排不负众望,时隔12年第三次问鼎奥运会冠军。‎ ‎②有专家指出,石油是不可忽视的战略资源,我们必须厝火积薪,未雨绸缪,进一步健全中国的石油安全体系。‎ ‎③当前的中学历史教学还存在着“重文轻图”的现象,这种现象不仅无法形成完整的知识体系,更不利于对历史知识的深化和拓展。“左图右史”,形象地说明了地图在历史学习中的重要性。‎ ‎④我们五个人要离开北大荒了,去各自最适宜的地方,快乐地生活,忘记对方,也忘记那段相濡以沫的生活。‎ ‎⑤曾经支撑起春晚半壁江山的语言类节目,近年来江河日下,整个春晚有时候连一两个优秀的语言类节目也出不来。‎ ‎⑥能够广纳意见,懂得通权达变,正是他在事业上获得成功的关键。‎ A.①②③ B.①⑤⑥‎ C.②③④ D.③④⑥‎ 答案 A 解析 ①问鼎:指夺取冠军。与后面的“冠军”重复。②厝火积薪:把火放到柴堆下面。比喻潜伏着很大的危险。此处不合语境。③左图右史:形容室内图书很多。不合语境。④相濡以沫:比喻同处困境,相互救助。不专用于夫妻。⑤半壁江山:指在敌人入侵后残存或者丧失的部分国土。后也用来指占据一半。⑥通权达变:做事能适应客观情况的变化,懂得变通,不死守常规。‎ ‎5.下列各句中加点成语的使用,全部正确的一项是(  )‎ ‎①要知道生之可贵,但不可苟且偷安;要知道死不足惧,但不轻易言死。‎ ‎②他知道自己患的是不可救药的癌症,但他以“生命不息战斗不止”的精神,鞠躬尽瘁为人民办事。‎ ‎③对于这样的意见,也要加以分析,不可以生吞活剥地完全肯定或完全否定。‎ ‎④宜春一中的老年人活动中心里,象棋、扑克、乒乓球、羽毛球、收音机、电视机、书报、画片,包罗万象。‎ ‎⑤如果不未雨绸缪,将来就要被动,就难以保证明年地方工业发展的需要,因此从现在起,就必须“今年看明年”,及早准备。‎ ‎⑥摆好后,他照例是横瞅瞅,竖看看,里磕磕,外扳扳,直到那砖横平竖直了,他自己也觉得称心如意了,这才摸起另一块。‎ A.①②④ B.①③⑤‎ C.②⑤⑥ D.③④⑥‎ 答案 C 解析 ①苟且偷安:贪图安宁,不求上进的意思,与语境不合。“苟且偷生”,跟“死”对照,本句中与“不轻易言死”对照。应该用“苟且偷生”。②不可救药:可以作定语。使用正确。③生吞活剥:比喻生硬地接受或机械搬用别人经验、言论(或文辞)或科学文化等方面的成就。本句不是“搬用”的意思。囫囵吞枣有两个意思:一是学习上不求甚解,食而不化,只知照搬照学;二是笼统含糊,不加分析。“笼统含糊,不加分析”的意思,应用“囫囵吞枣”。④“包罗万象”虽也可在前边先列举一些事物,但一般都不在前边列出具体事物。而“应有尽有”却有概括的作用,运用时可以跟总括性的词语搭配,也可以先在前边列举一些事物,而后用“应有尽有”来总括。所以此处应用“应有尽有”。⑤未雨绸缪:比喻事先做好准备。⑥称心如意:形容心满意足,事情的发展完全符合心意。使用正确。‎ ‎6.下列各句中加点词语的使用,全都正确的一项是(  )‎ ‎①北京奥运会主场馆“鸟巢”的造型美轮美奂,它是中国筑巢引凤、对外开放形象的象征。‎ ‎②当今世上追求藏书时尚者颇多,毕生致力于阅读者甚少;走马观花借书纳闲者俯拾即是,真正品出书味、读出书情者寥寥无几。‎ ‎③这些年来,越低劣的骗子越低劣的谎言越能骗人成功,越荒唐的行为越匪夷所思的举动越有市场,正是价值观疲劳的社会表现。‎ ‎④设计师要对施工工艺、材料等无所不知,只有这样,在客户面前才可以侃侃而谈,问即答来,才能赢得客户的信任,才能跟客户签下合同。‎ ‎⑤韩国对于活字印刷是做过贡献的,但是他们的这些成就与我国古代对活字印刷的贡献相比,是不能等量齐观的。‎ ‎⑥孝心是不会因时代而改变的,冬温夏凊的本质,出自孝心。子女行孝,随时皆可,给父母倒一杯水,也是在行孝道,不必等到不用做工养家时,才来行孝道。‎ A.①②④ B.①③⑤‎ C.②④⑥ D.③⑤⑥‎ 答案 D 解析 ①美轮美奂:形容新屋高大美观,也形容装饰、布置等美好漂亮。句子强调“造型”,应改为“巧妙绝伦”。巧妙绝伦:精巧奇妙到了极点,在同类事物中没有能与之相比的。②俯拾即是:形容地上的某一类东西、要找的某一类例证、文章中的错别字等很多。应改为“比比皆是”。比比皆是:到处都是,形容极其常见。③匪夷所思:指事物怪异或人的言行离奇,不是一般人按照常理所能想象的。符合语境。④“侃侃而谈”重在“有理”。语境要求“赢得客户的信任”,所以应改为“款款而谈”。“款款而谈”重在“诚恳”。⑤等量齐观:不管事物间的差异,同等看待。用于此处合适。⑥冬温夏凊 ‎:冬天使父母温暖,夏天使父母凉爽,指子女孝顺。用于此处合适。‎ ‎7.下列各句中加点成语的使用,全部正确的一项是(  )‎ ‎①《红楼梦》这部中国古典文学的巅峰之作,经曹雪芹呕心沥血,“披阅十载,增删五次”,才完成了前八十回。‎ ‎②河西走廊曾是佛教东传的要道,这里还留存着大量的石窟群,大小石窟俯拾即是,东西方文化在这里相互激荡,积淀下蔚为壮观的历史文明。‎ ‎③在对待利比亚局势的态度上,英国又一次紧跟美国顺风吹火,这纯系英美关系的惯性使然,尤其显示了它们当前特别需要相互借重的一面。‎ ‎④王校长的一席话果然起到了抛砖引玉的作用,与会的人就如何提高教学质量纷纷提出了自己的见解和建议。‎ ‎⑤随着计算机科学技术和互联网技术的快速发展,网络写手逐渐作为一个职业发展起来。一时间网络平台的文坛泥沙俱下,多数作品难称文学之名。‎ ‎⑥读古代典籍,很多时候我们总是否定古代的一些人士,其实我们没有资格对古人嗤之以鼻,因为我们现在自以为是的观念意识又有多少不属于发昏之列呢?‎ A.①②④ B.①③⑤‎ C.②③⑥ D.④⑤⑥‎ 答案 B 解析 成语题的考查,首先要明确成语的意思,然后再根据语境考虑。①呕心沥血:用尽心思。②俯拾即是:只要低下头来捡取,到处都是,形容多而易得。不符合语境。③顺风吹火:比喻乘便行事,并不费力。④抛砖引玉:敬谦错位,这是谦词,不能用于别人。⑤泥沙俱下:指泥土和沙子都跟着流下来,比喻好坏不同的人和事物混杂在一起。⑥嗤之以鼻:指用鼻子吭气,表示看不起。含贬义,用于对错误言行的蔑视。使用不当。‎ ‎8.下列各句中加点成语的运用,全部正确的一项是(  )‎ ‎①虽遭羞辱,但这位中国皇帝也很清楚,自己对于远在天边的那蕞尔岛夷其实鞭长莫及。‎ ‎②今年上半年城区二手房网上备案量达到1604套,比去年同期小幅增长。由于数据中存在部分房改房的备案量,二手房市场的交易情况依然不温不火。‎ ‎③2002年7月,农德忠从刑警学院毕业,一晃便过去14年多。大大小小办过无数的案子,和记者谈起这些,他都如数家珍。‎ ‎④政治就是一场博弈,在“总统”大选蓝绿差距拉近,王金平至少可以影响50万票的情况下,不管未来能不能续任“立法院长”,他都好整以暇。‎ ‎⑤《西方传教士眼中的厦门》以英国伦敦传道会在厦门教务的发展为线索,较为全面地讲述了该会在闽南胼手胝足开辟新教区、创立教会组织、建造教堂等故事。‎ ‎⑥黄兴家族百年变迁,他们时而被奉为座上宾,时而变为阶下囚,却都能做到不闻不问,平安地度过自己的一生。‎ A.①②④ B.③④⑤‎ C.①③⑥ D.②⑤⑥‎ 答案 A 解析 ①鞭长莫及:我国古代成语,出自《左传》,释为虽然鞭子很长,但总不能打到马肚子上,比喻距离太远而无能为力,也借指力量达不到,力所不及。这是对的。②不温不火:指性格温和或销售行情不火爆。这是对的。③如数家珍:好像数自己家藏的珍宝那样清楚,比喻对所讲的事情十分熟悉。这是错的。④好整以暇:既严整有序又从容不迫的样子,起初用于形容军队,后也可以用于个人。这是对的。⑤胼手胝足:胼、胝,老茧。皮肤等的异常变硬和增厚,一般是指长期从事体力劳动者,手脚生茧。形容十分辛勤劳动。这是对的。⑥不闻不问:人家说的不听,也不主动去问,形容对事情不关心。这里应用宠辱不惊。‎ ‎9.下列各项中,加点的成语使用,不恰当的两项是(  )‎ A.2015年,中国电影总票房不孚众望地突破了400亿元人民币大关,国产电影更是创下了票房的历史新高,但“票房高口碑差”的现象依然存在。‎ B.只有与国家和人民休戚与共,把个人荣辱置之度外的人才会得到人民的口碑,得到人民的拥戴。‎ C.李克强总理在国务院常务会议上表示,国务院绝不会发空头文件,为官也绝不能尸位素餐。‎ D.中国官方正式承认正在建造第二艘航母,但是新航母的性能和美国航母相比只能望其项背,一时难以赶超。‎ E.京剧大师梅兰芳先生不仅在舞台上风姿绰约,在日常生活中也气度不凡,无论何时何地,他总能让人为之倾倒。‎ 答案 AD 解析 A项,与语境相反,不孚众望:指不能使大家信服。B项,休戚与共:指忧喜、祸福彼此共同承担,形容关系密切,利害相同,侧重主观上的认识和态度。C项,尸位素餐:指空占着职位而不做事,白吃饭。D项,望其项背:比喻赶得上或比得上,一般用于否定句。E项,风姿绰约:形容女子风韵姿态柔美动人。这里形容梅兰芳是可以的,因为他扮演的是旦角。‎ ‎10.下列各句中,加点的词语使用不恰当的两项是(  )‎ A.微信朋友圈中有少数人常把一些不经之谈当作新闻来分享,对某些言论不加分辨就转发到朋友圈中,给社会带来了不良影响。‎ B.八达岭老虎咬人事件发生后,被老虎咬得浑身遍体鳞伤的女子的悲惨遭遇让人痛心,缺乏理性的网民暴露出来的人性的丑恶更令人痛心。‎ C.几年前,少不更事的孙杨曾因误服禁药被禁赛,在里约奥运会上,澳大利亚的霍顿借此骂孙杨为“骗子”,这种做法实在令人气愤。‎ D.新闻评论也好,学术文章也好,写作时都要有的放矢,论证严密,语言要准确简洁,不能模棱两可,也不能繁文缛节。‎ E.年轻人的一个优点就是不会由于世俗的约束而畏葸不前,而这种约束对于大多数成年人而言通常似乎又很难超越。‎ 答案 BD 解析 本题考查正确使用词语(包括熟语)的能力。A项,不经之谈:指荒唐的没有根据的言论。符合语境意思。B项,遍体鳞伤:全身都受了伤,伤痕像鱼鳞一样密,形容伤得很重,“遍体”与“浑身”重复。C项,少不更事:年纪轻,没有经历过什么事情。指经验不多。D项,望文生义。繁文缛节:指烦琐的仪式和礼节,该句把“繁文缛节”理解为语言烦琐、啰唆。E项,畏葸不前:畏惧退缩,不敢前进。‎ ‎11.下列各句中,加点的成语使用不恰当的两项是(  )‎ A.现在还有很多司机不吸取血的教训,仍然酒后驾车,对这种铤而走险的行为,公安交通部门还应加大处罚力度。‎ B.就日本名古屋市长关于“南京大屠杀”事件的危言危行,外交部发言人表示支持南京市做出暂停与名古屋市官方交往的决定。‎ C.他生前身无长物,家里连件像样的家电都没有,着实让人动容。‎ D.在开罗大学的演讲中,奥巴马表示,要和伊斯兰世界开诚布公地对话,寻找共同价值,支持穆斯林,并决心用美国的力量来解决最困扰伊斯兰世界的问题。‎ E.从被科尼法官讲述的一起案件深深触动,到把科尼的故事写成《复活》,托尔斯泰惨淡经营了整整12年之久。‎ 答案 AB 解析 A项,铤而走险:因走投无路而采取冒险行动,语意过重。B项,危言危行:说正直的话,做正直的事。指正直的言行。褒义词,望文生义。C项,身无长物:形容穷,符合语境。D项,开诚布公:指以诚心待人,坦白无私。E项,惨淡经营:费尽心思辛辛苦苦地经营筹划。后指在困难的境况中艰苦地从事某种事业。‎ ‎12.下列各句中,加点的成语使用不恰当的两项是(  )‎ A.岁月留给我们的总是推陈出新的昭示和启迪,当我们再看黄公望那幅声名远扬的《富春山居图》,除了画中令人沉醉的美景之外,自然能体会到画外人的波折坎坷。‎ B.2016年钢铁行业利润率虽悄然反弹,但日渐式微 ‎,仍创近年次低点,预计在行业整合的背景下,2017年将会继续好转,且机遇与风险并存。‎ C.微信、二维码、云报纸以及便携性手持移动终端大众传播功能的拓展,集中指向移动互联时代,将新时代下两会中的“随时、随地、随意”的精髓表现得惟妙惟肖。‎ D.现在已经进入新媒体时代,传统的纸质媒体已经四面楚歌,如若不想方设法挖掘其核心竞争力,很有可能被时代淘汰。‎ E.雅安大地震发生后,救援队员冒着余震的危险,深入原本瓦房鳞次栉比的胡同,在废墟中救出一个又一个被困群众。‎ 答案 AC 解析 A项,推陈出新:推,除掉,去除,淘汰;陈,陈旧的;新,好的、新的。除掉旧事物的糟粕,取其精华,并使它向新的方向发展(多指继承文化遗产),不合语境,改为“历久弥新”。B项,日渐式微:式微,指事物的衰落。逐渐没落,销声匿迹。泛指事物或人逐步地衰落。C项,惟妙惟肖:惟,文言语气词,这里有“真”或“特别”的意思。妙,美妙,漂亮。肖,相像,逼真。形容描写或模仿得非常逼真、传神。使用对象错误。改为“淋漓尽致”,淋漓,形容湿淋淋往下滴,比喻尽情,酣畅;尽致,达到极点。形容文章、谈话等详尽彻底,发挥充分,也形容暴露得很彻底。D项,四面楚歌:比喻陷入四面受敌、孤立无援的窘迫境地。E项,鳞次栉比:比喻像鱼鳞和梳齿那样整齐地排列着,多形容房屋等密集。这里用来形容瓦房多而密集且分布有序。‎ ‎13.下列各句中,加点的成语使用不恰当的两项是(  )‎ A.喜欢石头体现了他坚强不屈的性格,不同流俗的思想和坚若磐石的个性,当然也寄托了他对晨夕风露、阶柳庭花、餐云卧石的生活的向往。‎ B.在微博等网络舆论大力监督下,在有关部门的周密调查下,“表哥”“房叔”等人光鲜外表下掩藏的腐败本质被暴露得具体而微。‎ C.集电话、电脑、相机、信用卡等功能于一体,手机在生活中的作用被发挥得淋漓尽致。‎ D.中国需要在国际平台上做一个有相当规模的展览,在一定程度上改变西方艺术界对于中国当代艺术的管见所及。‎ E.“五四”以来,中国现代散文全面繁荣,许多评论家摒弃述而不作的陋习,纷纷提出有个人见地和研究价值的观点学说。‎ 答案 BD 解析 A项,餐云卧石:指超脱尘世的隐逸生活。B项,不合语境。具体而微:指事物的各个组成部分大体都有了,不过形状和规模比较小些。C项,淋漓尽致:形容文章或说话表达得非常充分、透彻,或非常痛快。也指表现得非常充分、透彻。D项,管见所及:指自己的见识不广,看法未必正确,用作谦辞。对象误用。E项,述而不作:述,阐述前人学说;作,创作。将古人的智慧心得加以陈述,不加入自己的思想。指只叙述和阐明前人的学说,自己不创作。‎ ‎14.下列各句中,加点的成语使用不恰当的两项是(  )‎ A.他在家里不声不响,妻子总是觉得他木讷,把他当成个闷葫芦,但在社会上他却玲珑剔透,无论做什么事总是一点就通。‎ B.为什么人言可畏?因为一次又一次的毁谤,积累下来足以致人于毁灭之地,那是一种铄石流金的力量。‎ C.新型播种机实现了“灭茬、旋耕”与深松施肥、精播一次性完成,减少了单独作业,不仅大大降低了成本,还节约了农时,秸杆焚烧的难题也迎刃而解。‎ D.现在,许多家长望子成龙、盼女成凤的心情过于急切,往往不切实际地对孩子提出过高的要求,其结果常常是弄巧成拙。‎ E.在中俄伊犁争端的交涉中,曾纪泽从国家和民族的利益出发,折冲樽俎,虎口索食,终于改订《崇约》;收回了伊犁九城的主权,博得朝野的好评和西方外交界的尊重。‎ 答案 BD 解析 A项,玲珑剔透:形容器物精致通明,结构细巧。也比喻人精明灵活。B项,铄石流金:能使金石熔化,比喻天气极热,有别于“众口铄金”‎ ‎。C项,迎刃而解:比喻解决了关键的问题,其他的问题也就顺利地解决了。D项,弄巧成拙:弄,卖弄,耍弄;巧,灵巧(指心思);拙,笨拙。想耍巧妙的手段,结果反而坏了事。含贬义,用于批评场合。E项,折冲樽俎:樽俎,古时盛酒食的器具。在酒席宴会间制敌取胜,指进行外交谈判。‎ ‎15.下列各句中,加点的成语使用不恰当的两项是(  )‎ A.“拼爹游戏”肆意上演,教育公平被踩在脚下,大学精神被无情玷污,这怎不让人痛心疾首?‎ B.吕晓萌前两次考试没进前十名,心里很郁闷,在这次期末考试中她终于力挽狂澜,考了全校第三名。‎ C.反腐败是我国当前政治生活中的大事,关系到人心向背和党的生死存亡,既要旗帜鲜明,坚决惩治,又需要瞻前顾后,综合考虑各方面的因素和影响。‎ D.民调显示,朱立伦与蔡英文分别是国民党、民进党支持度最高的人选,而两人的民意支持度不分轩轾,都获得42%的民众的支持。‎ E.调查显示,70%的网友对二手房个税增至20%的新政觉得差强人意,不满意的主要原因是这一政策对于需要改善住房条件的中低收入者不公。‎ 答案 BE 解析 A项,痛心疾首:形容痛恨到了极点。B项,力挽狂澜:指把已经倾倒的巨浪拉回来,比喻尽力挽回险恶的局势。不合语境。C项,瞻前顾后:形容做事之前考虑周密慎重;也形容顾虑太多,犹豫不决。D项,不分轩轾:不分高下、轻重。E项,差强人意:大体上还让人满意。望文生义。‎ ‎ [3年高考真题集训]‎ ‎1.[2016·全国卷Ⅰ]下列各句中加点成语的使用,全都正确的一项是(  )‎ ‎①第二展厅的文物如同一部浓缩的史书,举重若轻地展示了先民们在恶劣的自然条件下顽强抗争、繁衍生息的漫长历史。‎ ‎②这部翻译小说虽然是以家庭生活为题材的,却多侧面、多视角地展现出那个时代光怪陆离的社会生活画卷。‎ ‎③毕业后他的同学大都顺理成章地走上了音乐创作之路,而他却改换门庭,另有所爱,一头扎进中国古代文化研究中。‎ ‎④就对后世的影响来说,我们一致认为《封神演义》虽然比不上《西游记》,但和《聊斋志异》是可以并行不悖的。‎ ‎⑤在那几年的工作学习中,杨老师给了我很大的帮助,他的教导在我听来如同空谷足音,给我启示,带我走出困惑。‎ ‎⑥我国绘画史上有一个时期把王石谷等四人奉为圭臬,凡是学画,都以他们为宗,有的甚至照摹照搬。‎ A.①②④ B.①③⑤‎ C.②⑤⑥ D.③④⑥‎ 答案 C 解析 ①举重若轻:举重东西就像举轻东西那样。形容做繁难的事或处理棘手的问题轻松而不费力。②光怪陆离:形容现象奇异、色彩繁杂。③改换门庭:a.改变门第出身,提高社会地位。b.投靠新的主人或势力,以图维持、发展。④并行不悖:可以共存,而不相互违背;可同时实行,不相冲突。⑤空谷足音:在寂静的山谷里听到人的脚步声。比喻难得的音信、言论或事物。⑥奉为圭臬:圭,古代测日影的器具;臬,古代测量日影方位的标杆;圭臬,比喻事物的准则。比喻把某些言论或事物当作准则。本题适用排除法。“举重若轻”属动词性短语,一般作谓语,使用对象一般是人,①句的主语是“文物”,所以错误,排除A、B两项。“改换门庭”含贬义,与③句语境不符,同时④句中的“并行不悖”使用错误,故排除D项。‎ ‎2.[2016·全国卷Ⅱ]下列各句中加点成语的使用,全都正确的一项是(  )‎ ‎①舞台上的灯光时明时暗,快速变幻的布景令人目不交睫,随着歌手的狂歌劲舞,观众席上也一片沸腾。‎ ‎②有专家指出,石油是不可忽视的战略资源,我们必须厝火积薪,未雨绸缪,进一步健全中国的石油安全体系。‎ ‎③那些航空领域的拓荒者,很多已经离开人世,但他们筚路蓝缕的感人形象一直深深印在人们的记忆中。‎ ‎④这次会谈并没有其他人员参加,他们两个人又都一直讳莫如深,所以会谈内容就成为一个难解之谜。‎ ‎⑤正在悠闲散步的外科主任王教授,突然接到护士电话说有个病人情况危急,他立刻安步当车向医院跑去。‎ ‎⑥从用字之讲究可以看出,这首诗的作者苦心孤诣,要在这有限的篇幅中营造出一种深邃幽远的意境。‎ A.①②⑤ B.①④⑥‎ C.②③⑤ D.③④⑥‎ 答案 D 解析 ①目不交睫:形容夜间不睡觉或睡不着觉。原句是说布景快速变幻,眼睛看不过来,应该用“目不暇接”。②厝火积薪:把火放在柴堆下面,比喻潜伏着很大的危险。原句的意思是要事先做好准备,该词词义与原句要表达的意思不符。③筚路蓝缕:《左传》中有“筚路蓝缕,以启山林”,意思是说驾着柴车,穿着破旧的衣服去开辟山林。形容创业的艰苦。原句中航空领域的拓荒者创业时应该是艰苦的,所以此成语运用恰当。④讳莫如深:紧紧隐瞒。原句中的会谈只有他们两个人参加,会谈内容又成了难解之谜,所以应该是他们两人不透露会谈内容,所以此成语运用恰当。⑤安步当车:慢慢地步行,就当作是坐车。原句是说病人情况危急,应该急急忙忙地向医院跑去,“安步当车”的意思与原句句意不符。⑥苦心孤诣:费尽心思钻研或经营,达到别人达不到的境地。原句是说作者要在有限的篇幅中营造出深邃幽远的意境,故此成语运用恰当。‎ ‎3.[2016·全国卷Ⅲ]下列各句中加点成语的使用,全都正确的一项是(  )‎ ‎①这块神奇的土地上,既有浩如烟海的传统文化典籍,也有丰富多彩的民俗文化和各种流派的现代艺术,这些都深深吸引着前来参观的外国友人。‎ ‎②今年的元宵晚会上,著名豫剧演员小香玉将《谁说女子不如男》唱得字正腔圆、声情并茂,令观众刮目相看、赞叹不已。‎ ‎③最近出版的长篇小说《雪莲花开》通过对藏族姑娘卓玛的人生历程的叙述,表现了她鲜明的民族性格和一言九鼎的诚信精神。‎ ‎④经过周密的调查,公安人员终于掌握了在逃人员的行踪,然后兵分三路按图索骥,一举将他们全都缉拿归案。‎ ‎⑤这几幅书法作品笔走龙蛇、流畅飘逸,在本次春季拍卖会上甫一亮相,就引起了国内外藏家的极大兴趣。‎ ‎⑥天寒地冻、滴水成冰的季节终于过去,春天在大家的盼望中姗姗而来,到处都涣然冰释,生机勃勃。‎ A.①②④ B.①④⑤‎ C.②③⑥ D.③⑤⑥‎ 答案 B 解析 ①浩如烟海:指文献、资料等非常丰富,在此形容文化典籍恰当。②刮目相看:指别人已有进步,不能再用老眼光去看他。本句中没有新旧对比的意思,且小香玉本来就已经很著名了,所以此处使用不当。③一言九鼎:指一句话的分量像九鼎那样重,形容所说的话分量很重,作用很大。本句中用来形容诚信精神不当。④按图索骥:指按照图像寻找好马,比喻按照死规矩机械、呆板地做事,也泛指按照线索寻找目标。本句用的是后一个意思。⑤笔走龙蛇:指书法笔势雄健活泼。符合本句语境。⑥涣然冰释:指嫌隙、疑虑、误会等完全消除。这个成语现在只用其比喻义,本句误用了其字面意义。‎ ‎4.[2016·山东高考]下列各句中,加点的成语使用正确的一项是(  )‎ A.旅游业已成为当地经济发展的支柱产业,这里巧夺天工的自然美景闻名天下,每年都吸引大量游客前来观赏。‎ B.持续多日的强降雨导致部分地区山洪暴发,农田被淹,房屋倒塌,灾情扣人心弦,相关部门正全力以赴组织救灾。‎ C.两位多年未见的战友在火车上意外相逢,他们一见如故,回忆起一同出生入死的战斗经历,不禁感慨万千。‎ D.没有强大的创新设计、生产制造能力,国家实力的提升就无从谈起,民族复兴的宏伟蓝图也只能是空中楼阁。‎ 答案 D 解析 A项,巧夺天工:精巧的人工胜过天然,形容技艺极其精巧。使用对象错误。B项,扣人心弦:形容诗文、表演等有感染力,使人心情激动。用错对象。C项,一见如故:初次见面就很相投,像老朋友一样。望文生义。D项,空中楼阁:多用来比喻虚幻的事物或脱离实际的理论、计划等。‎ ‎5.[2016·浙江高考]下列各句中,加点的词语运用不正确的一项是(  )‎ A.他爱好广泛,喜欢安静的棋类运动,对热闹的纸牌游戏也来者不拒;欣赏通俗感性的流行歌曲,对庄重恢宏的交响乐也甘之如饴。‎ B.荧屏上,他沉着大方,点评时事亦庄亦谐,精辟的见解让人折服;镜头外,他开朗乐观、热心助人,是邻居、朋友心中的活雷锋。‎ C.虽然最初并不相信自己涉嫌犯罪,但由于电话那头的骗子言之凿凿,加上所谓最高检的“全国通缉公告”,信息闭塞的受害人最终成了骗子的猎物。‎ D.在媒体的长枪短炮前,明星们也许悟出了言多必失的道理,鲜有人会在聚光灯前竹筒倒豆子,少说、不说成了他们自我保护的明智选择。‎ 答案 A 解析 A项,甘之如饴:形容甘愿承受艰难、痛苦。语境中没有“艰难、痛苦”之意。B项,亦庄亦谐:(讲话或文章的内容)既庄重,又风趣。C项,言之凿凿:话说得有根有据,非常肯定。D项,竹筒倒豆子:比喻把事实全部说出来,没有隐瞒。‎ ‎6.[2015·全国卷Ⅰ]依次填入下列各句横线处的成语,最恰当的一组是(  )‎ ‎①这正是经验丰富的主教练在战术安排上的________之处:下半场比赛中想方设法消耗对方主力队员的体力,终于扭转劣势,赢得比赛。‎ ‎②经过几天的________,又和病人家属作了充分沟通,吴医生最终否定了治疗小组提出的保守治疗方案,决定尽快为病人进行肺部手术。‎ ‎③早在20世纪末,当地决策者就________,提出了从单一的小农业向大农业转移的战略措施,于是一个个生态经济园区应运而生。‎ A.老谋深算   深谋远虑   深思熟虑 B.老谋深算 深思熟虑 深谋远虑 C.深思熟虑 老谋深算 深谋远虑 D.深谋远虑 深思熟虑 老谋深算 答案 B 解析 ①老谋深算:周密的筹划、深远的打算。形容人办事精明老练。侧重做事老练,有经验。②深思熟虑:深入细致地考虑。强调思考细致。③深谋远虑:周密地计划,往长远里考虑。侧重考虑长远。‎ ‎7.[2015·全国卷Ⅱ]依次填入下列各句横线处的成语,最恰当的一组是(  )‎ ‎①他是一个心地善良的人,但性格懦弱、谨小慎微,做起事来总是________,从来不敢越雷池一步。‎ ‎②当今世界科技突飞猛进,我们更要勇于开拓,不断进取,如果________,就会落后甚至被时代潮流所淘汰。‎ ‎③要想让中国传统戏曲焕发出新的生命力,绝不能满足于现状,________,唯有创新才是弘扬戏曲文化的康庄大道。‎ A.故步自封 墨守成规 抱残守缺 B.墨守成规 故步自封 抱残守缺 C.抱残守缺 故步自封 墨守成规 D.墨守成规 抱残守缺 故步自封 答案 B 解析 ①墨守成规:形容因循守旧,不肯改进。②故步自封:比喻安于现状,不求进步。③抱残守缺:抱着残破陈旧的事物,不肯放弃。有泥古守旧,不愿革新之意。也有保存虽有残缺但仍有价值的古物之意。‎ ‎8.[2014·全国卷Ⅰ]依次填入下列各句横线处的成语,最恰当的一组是(  )‎ ‎①医疗质量是关系到病人生命安危的大事,救死扶伤是医务人员________的天职。‎ ‎②中国传统的严父慈母型的家庭关系,常令父亲们________地承担起教育子女的义务。‎ ‎③在全国比赛中屡获金奖的我省杂技团,________地承担了这次出国演出任务。‎ A.当仁不让 责无旁贷 义不容辞 B.责无旁贷 义不容辞 当仁不让 C.义不容辞 责无旁贷 当仁不让 D.义不容辞 当仁不让 责无旁贷 答案 C 解析 当仁不让:遇到应该做的事,积极主动去做,不退让。责无旁贷:自己的责任,不能推卸给别人。义不容辞:道义上不允许推辞。‎ ‎9.[2014·全国卷Ⅱ]依次填入下列各句横线处的成语,最恰当的一组是(  )‎ ‎①消防工作必须立足于________,从提高公众的防火意识做起。‎ ‎②即使现有的产品畅销,也要________,抓紧技术储备与新产品开发。‎ ‎③如果我们不从小事做起,________,那些细小的苗头最终可能酿成大祸。‎ A.防患未然 防微杜渐 未雨绸缪 B.防患未然 未雨绸缪 防微杜渐 C.未雨绸缪 防微杜渐 防患未然 D.未雨绸缪 防患未然 防微杜渐 答案 B 解析 选项所给的三个成语都有防备的意思,但三者之间又有区别。“防患未然”指在事故或灾害尚未发生时采取预防措施,重在发生前防止。“防微杜渐”指在错误或坏事萌芽的时候及时制止,不让它发展,重在在萌芽状态制止。“未雨绸缪”指趁着天没下雨,先修缮房屋门窗,比喻事先做好准备,重在事前准备。根据各句语境,不难得出答案。‎ ‎10.[2015·山东高考]下列各句中,加点的成语使用正确的一项是(  )‎ A.新闻发布会上,他讲话仅用了八分钟,简洁明了,新闻性、针对性强,没有一句穿靴戴帽的空话套话。‎ B.联合国大会曾经两次召开会议,讨论是否应该废除死刑的问题,但因各方立场南辕北辙,讨论无果而终。‎ C.本届展销会邀请到了安徽、浙江、上海等地知名企业,湖笔、宣笔、徽墨、宣纸、歙砚等文房四宝济济一堂。‎ D.写一篇小说并不太难,但要想让自己的作品在擢发难数的小说中引起读者广泛关注,就不那么容易了。‎ 答案 A 解析 A项,穿靴戴帽:比喻写文章或讲话时套用一些空洞说教。B项,南辕北辙:心里想往南去,却驾车往北走。比喻行动和目的相反。不合语境。C项,济济一堂:形容许多有才能的人聚集在一起。使用对象错误。D项,擢发难数:形容罪恶多得像头发那样,数也数不清。使用对象错误。‎ ‎11.[2015·安徽高考]下列各句中,加点的词语使用恰当的一句是(  )‎ A.于敏院士在我国首颗氢弹的成功研制上功勋卓著,然而他淡泊名利,婉拒“氢弹之父”的称号,其人品胸襟,令人高山仰止。‎ B.在东海舰队组织的此次实战演练中,我军的反水雷舰艇倾巢而出,成功扫除了“敌军”在航道上隐蔽布设的多枚新型水雷。‎ C.某些管理机构缺乏“大数据思维”,以邻为壑,不与相关机构共享信息资源,公共数据中心的建设将有助于改变这种状况。‎ D.现代舞剧《十面埋伏》,以其色彩浓重的舞台背景、风格鲜明的京剧音乐以及刚柔相济的舞者形体,一举征服了现场观众。‎ 答案 A 解析 ‎ A项,高山仰止:像仰望高山那样,对伟大的人物表示仰慕和崇敬。使用正确。B项,倾巢而出:出动全部力量(多含贬义)。褒贬不当。C项,以邻为壑:拿邻国当作大水坑,把本国洪水排泄到那里去,比喻把灾祸推给别人。不合语境。D项,刚柔相济:刚强的和柔和的互相补充,使恰到好处。多指为人处世,不能指“舞者形体”。使用对象错误。‎ ‎12.[2014·辽宁高考]下列各句中,加点的成语使用不恰当的一项是(  )‎ A.在全省经济发展座谈会上,李教授的讲话直击时弊,同时又颇具前瞻性,对于当前经济工作而言,可谓空谷足音。‎ B.他对市场发展趋势洞若观火,在市场竞争中游刃有余,这与他曾在国企和外企工作,后来又自己创业的经历有关。‎ C.张先生在这所大学从事教学和研究工作三十余年,学问炉火纯青,性格外圆内方,所以既受尊重,又有很多朋友。‎ D.这位书法家书写作品,不管十几个字还是几十个字,都倚马可待,一气呵成,并且字里行间显示出令人振奋的豪情。‎ 答案 D 解析 A项,空谷足音:比喻难得的音信、言论或事物。B项,洞若观火:形容看得清楚明白。C项,外圆内方:指人外表随和,内心却很严正。D项,倚马可待:形容文思敏捷,文章写得快。用错对象。‎ ‎[2年全国模拟重组]‎ ‎1.[2017·山西重点中学联考]下列各句中加点成语的使用,全部正确的一项是(  )‎ ‎①他闲来无事,就经常上网发一些飞短流长的帖子,结果不仅弄得与同事邻里的关系很紧张,甚至还惹上了官司。‎ ‎②在文化强省战略指引下,我省动漫产业迈入高速发展的新阶段,去年我省与沿海省份动漫原创产品的产值已经等量齐观。‎ ‎③听到张旭东升任宁江市交通局长的消息,一帮老同学、老同事纷纷发来短信,祝贺他的乔迁之喜,并要他赶快请客。‎ ‎④有人认为天才之作总是合天地之灵气,妙手偶得,据说《蓝色多瑙河》就是作者在用餐时灵感一来随手写在袖口上的。‎ ‎⑤伴着落日的余晖,诗人缓步登上了江边的这座历史名楼,极目远眺,晚霞尽染,鸿雁南飞,江河日下,诗意油然而发。‎ ‎⑥司机张师傅冒着生命危险解救乘客的事迹,一经新闻媒体报道,就被传得满城风雨,感动了无数市民。‎ A.②④⑥ B.①⑤⑥‎ C.②③⑤ D.①③④‎ 答案 D 解析 本题考查正确使用词语(包括熟语)的能力。正确识记成语的含义,注意常用成语的使用对象、范围、感情褒贬等。再结合语境判断使用是否恰当。①飞短流长:造谣生事,搬弄是非。符合语境。②等量齐观:指对有差别的事物同等看待。用来形容动漫产品的产值不恰当。排除②,同时可排除A、C项。③乔迁之喜:用于祝贺别人搬到好的地方去住或官职高升。符合语境。④妙手偶得:技术高超的人,偶然间即可得到。也用来形容文学素养很深的人,出于灵感,即可偶然间得到妙语佳作。符合语境。⑤江河日下:江河的水一天天地向下流。比喻情况一天天地坏下去。本句属于望文生义,不合语境,排除⑤,B项即可排除。⑥满城风雨:形容事情传遍各处,到处都在议论着(多指坏事情),属于贬义词,而⑥句应为褒扬,排除⑥,故选D项。‎ ‎2.[2017·安徽六校联考]下列各句中加点成语的使用,全部正确的一项是(  )‎ ‎①我们宣传《北京市控制吸烟条例》,意在增强居民的控烟意识,拒绝吸烟和“二手烟”的危害,让控烟令落地生根,不走过场,最终达到真正禁烟的目的。‎ ‎②受强冷空气影响,黑龙江多地降雪,气温持续下降,空中喷温水秒变冰雾,地面倒水瞬间结冰,人们在室内座位上坐了半天,仍然觉得席不暇暖,冰凉透骨。‎ ‎③旧时代的凄风苦雨和战争的血火考验,铸就了老一辈革命家始终不渝的理想信念。反观我们,和平与安逸好像消解了我们的精神“钙质”。‎ ‎④在市区有些只允许单向通行的小街巷,却经常遇到逆向行驶的车辆,司机对交通指示标识习焉不察,给这些单行道“添堵”。这引起市民不满。‎ ‎⑤古巴在政治上深谙左右逢源的生存之道。它积极改善古美关系,又深化与俄罗斯的战略合作关系,还与拉美几乎所有国家特别是巴西和委内瑞拉保持良好的双边关系。‎ ‎⑥家园被毁,亲人受伤,但战士何伟说:“我知道家中亲人没有生命危险就可以了,这里的群众更需要救援。”这是一种境界。无独有偶,这样的事也发生在武警四川总队中队长杨昌军身上。‎ A.①③⑤ B.①③⑥‎ C.③④⑤ D.②⑤⑥‎ 答案 A 解析 ①落地生根:比喻切切实实、一心一意地做好所从事的工作。使用正确。②席不暇暖:形容很忙,多坐一会儿的时间都没有。望文生义。③凄风苦雨:形容天气恶劣;也比喻悲惨凄凉的境遇。使用正确。④习焉不察:习惯于某种事物而觉察不到其中的问题。使用错误。⑤左右逢源:形容做事得心应手,怎样进行都很顺利;也形容办事圆滑。使用正确。⑥无独有偶:虽然罕见,但是不止一个,还有一个成对。(多用于贬义)使用错误。‎ ‎3.[2017·福建厦门一中模拟]下列各句中加点成语的使用,全都正确的一项是(  )‎ ‎①钱钟书先生用文言写成的《谈艺录》《管锥编》博大宏深,其用词运斤成风,最可表现文言词汇的活力和生命力。‎ ‎②点开广东省编办官方网的“机构编制实名制信息公开”专栏,48家省直机关及其下设机构的名称、5000多名公务员姓名等信息昭然若揭。‎ ‎③新“国八条”出台后,国务院派出八个调查组,分赴全国各地对先前出台的楼市政策产生的效果顺藤摸瓜,以便了解新变化制定新政策。‎ ‎④如果把现代化建设比作一首气势磅礴、波澜壮阔的交响乐,青年农民工就是其中最激昂、最有青春气息的乐章。‎ ‎⑤中国石油与化工联合会的资料披露,与三大石油公司的投资成本相比,三大石油公司的投资收益却很不尽人意。‎ ‎⑥有些出版社只认题材不辨真伪,致使一批并不具备写作资质的作者的文字行销书市;不只如此,更有的出版社粗制滥造。‎ A.①③④ B.③⑤⑥‎ C.①④⑥ D.②③⑥‎ 答案 C 解析 ①运斤成风:比喻手法熟练,技艺高超。②昭然若揭:真相大白。不合语境。含贬义,褒贬不当。③顺藤摸瓜:比喻沿着发现的线索追根究底。不合语境。④波澜壮阔:比喻声势雄壮浩大,多用于诗文、群众运动等。对象错误。⑤应为“不尽如人意”,指不完全使人满意。⑥粗制滥造:指写文章或做东西马虎草率,只求数量,不顾质量。‎ ‎4.[2017·广东四校联考]下列各句中加点的成语,使用不当的两项是(  )‎ A.31周岁,对于任何一名游泳运动员而言,都是一个令人望而生畏的年纪,而菲尔普斯克服了这一切,在里约奥运会上仍拿下了4枚金牌,以23枚奥运金牌总数打破前人保持的奥运奖牌纪录,也创造了奥运历史上奖牌和金牌最多的纪录。‎ B.小李是我最要好的朋友,经常对我耳提面命,使我在日常的学习与工作中能少犯错误,我非常感激他。‎ C.我跟他认识也有几年了,但除了见面点头,几句寒暄,就没有别的,因此至今还是白头如新。‎ D.今年暑假,我和几个小伙伴几经艰辛,终于登上了西岳华山绝顶。立于巅峰,放目远眺,天无涯际,我顿觉自己渺小,登高自卑之感油然而生。‎ E.卡耐基演讲口才新天地会让你通过学习增强自信,能泰然自若地当众站起来演讲,在商场或社交场合口若悬河,思路清晰又富有语言魅力。‎ 答案 BD 解析 A项,望而生畏:望,看见;畏,畏惧,害怕。表示看见了就害怕。在本句中,符合语意,是正确的。B项,耳提面命:对着耳朵告诉,表示教诲的殷勤恳切。多指(长辈对晚辈、上级对下级)恳切地教导,不能用于同辈之间。在本句中,属于用错对象。C项,白头如新:白头,头发白了;新,新交。指交朋友彼此不能了解,时间虽久,仍跟刚认识一样。在本句中,符合语意,是正确的。D项,登高自卑:自,从;卑,低。登山要从低处开始。比喻做事情要循序渐进,由浅入深。做事脚踏实地从小事做起。不能误以为看见高山而觉得自己很渺小。在本句中,“登高自卑”错在望文生义。E项,口若悬河:说话像瀑布一样,滔滔不绝。形容能言善辩。使用正确。‎ ‎5.[2016·天水检测]下列各句中加点成语的使用,全都正确的一项是(  )‎ ‎①菲律宾南海仲裁案临时仲裁庭做出的所谓最终裁决是非法无效的,南海主权归属于中国是无可辩驳的历史事实。‎ ‎②这些具有穿云裂石之功的大力士一上场,就把观众的目光吸引了过去。‎ ‎③我们一定要教育党员干部情为民所系,权为民所用,利为民所谋,只要涉及群众的事,都要无所不为,直到群众满意为止。‎ ‎④公平和正义是治愈社会心理疾病的最佳良药:只有机会公平,才不会有那么多的社会底层人士自怨自艾,感叹自己怀才不遇。‎ ‎⑤据知情人爆料,男友是音乐人的吉克隽逸没有和《中国好声音》签约,而是和男友另起炉灶。‎ ‎⑥近年来看多了电视上、舞台上一些“腕儿”们的假唱,以及出乖露丑的表演,我对国内某些所谓的专业团体逐渐失去了应有的信任、欣赏,直至尊重。‎ A.①③④ B.③⑤⑥‎ C.①⑤⑥ D.②③⑥‎ 答案 C 解析 ①无可辩驳:表示道理很明白或事实很清楚,没有可争论的。②穿云裂石:形容声音高亢嘹亮。使用错误。③褒贬失当。“无所不为”是贬义词。④自怨自艾:本义是悔恨自己的错误,自己改正。现仅指悔恨。本句语境感叹怀才不遇,是心怀幽怨,而不是悔恨,望文生义。⑤另起炉灶:比喻放弃原来的,另外从头做起。⑥出乖露丑:指在人前出丑。‎ ‎6.[2017·湖北部分重点中学联考]下列各句中,加点的词语使用不恰当的两项是(  )‎ A.作为学生和语文知识之间的一座桥梁,中学语文教师如果能够充分运用兴趣原理,提高教学过程中的趣味性,必然可以收到事半功倍的效果。‎ B.有些同学在写作议论文时,遇到理论的阐发便感到理屈词穷无话可说。‎ C.以每个人发一百元钱然后遣散的方式来应付民工集中追索欠款,这种作为只能是扬汤止沸,对解决问题毫无帮助。‎ D.独俏寒枝的梅花曾经触动多少文人墨客的审美心灵,吸引他们诗兴大发,吟风弄月。‎ E.恐怖分子多次制造自杀性爆炸事件,致使大量的无辜者死于非命,社会秩序动荡不安。‎ 答案 BD 解析 A项,事半功倍:形容用力小而收效大。B项,理屈词穷:由于理亏而无话可说。望文生义。C项,扬汤止沸:比喻办法不对头,不能从根本上解决问题。D项,吟风弄月:指喜欢用风花雪月做题材来写作。贬义词。E项,死于非命:在意外的灾祸中死亡。‎ ‎7.[2016·桂林、崇左联合调研]下列各句中,加点的成语使用不恰当的两项是(  )‎ A.购物能给女人带来幸福感,特别是面对打折等促销活动,那种以便宜的价格买到自己心仪的物品的快乐,是男人所不可理喻的。‎ B.在今年的欧洲杯中,俄罗斯队首轮比赛轻松获胜,其中有着“新沙皇”之称的扎戈耶夫的表现也可圈可点。‎ C.目前又有一股中等强度的冷空气正在“蠢蠢欲动”,预计可能会在今天开始“中国之旅”,至于它会不会给辽宁带来较大幅度降温影响,气象部门正在密切监测中。‎ D.陕西考古研究院对外消息称,上官婉儿千字铭文墓志为千字楷书,记载上官昭容世系、生平、享年、葬地等信息,行文采用春秋笔法,文学色彩浓重,溢美之词较多。‎ E.一曲《雨霖铃》,绘尽秋之凄凉,诉尽惜别之苦,二人河梁携手,凄婉至深;离愁别绪,催人泪下。一曲《雨霖铃》不知感动了多少人,占据了多少人的心扉?‎ 答案 AE 解析 A项,不可理喻:不能用道理使他明白。形容固执或蛮横,不通情理。望文生义。B项,可圈可点:指文章精彩,值得加以圈点。也形容表现好,值得肯定或赞扬。C项,蠢蠢欲动:比喻敌人准备进攻或坏人阴谋捣乱,这里是活用。D项,春秋笔法:指文字隐晦、文笔婉转而含有褒贬意思的写作风格。E项,河梁携手:河梁:桥,指送别。代指依依惜别。望文生义。‎ ‎8.[2017·沙市中学考试]下列各句中,加点的成语使用不恰当的两项是(  )‎ A.有企业家坦言,进行公益捐助的一个原因是担心消费者指责自己冷漠吝啬,细大不捐,从而影响公司产品的销路。‎ B.李鸿章主张大清国参与世界商品经济往来,利用双边贸易富强自己,制约别人,这样的认识在当时可谓空谷足音。‎ C.今年“两会”上一些代表委员的发言少了穿靴戴帽的套话,多了实实在在的内容,赢得了社会各界的好评。‎ D.《社会契约论》是法国思想家卢梭的代表作,书中对政治权利和义务等问题作了很多鞭辟入里的分析,值得一读再读。‎ E.围棋等棋类游戏能很好地培养人的统筹意识和战略眼光,因为如果目无全牛,就很可能因顾此失彼而落败。‎ 答案 AE 解析 A项,细大不捐:小的大的都不抛弃。望文生义。B项,空谷足音:比喻难得的音信、言论或事物。C项,穿靴戴帽:比喻写文章或讲话中套用一些空洞的、例行的政治说教。D项,鞭辟入里:形容做学问切实。也形容分析透彻,切中要害。E项,目无全牛:形容技艺已达到十分纯熟的地步。望文生义。‎ ‎9.[2017·江西九江一中模拟]下列各句中,加点的成语使用不恰当的两项是(  )‎ A.与散文家林非先生聊天是一件很幸福的事,他语气随和,娓娓而谈,我感到无拘无束,心情非常愉快。‎ B.打击盗版软件、保护知识产权无可非议,但采取黑屏这一方式就过了头,这样做不利于软件事业的发展。‎ C.在世界经济徘徊不前、企业间竞争异常激烈的情形下,没有什么企业联盟颠扑不破,所谓的价格联盟也不例外。‎ D.王旭明做客杭州的行程排得很满,记者的采访只能见缝插针地安排在他从宾馆前往浙江人文大讲堂的路上。‎ E.私立学校虽然缺乏教学管理经验,但可以向公办学校学习,可以在亦步亦趋的基础上,渐渐走出自己的路来。‎ 答案 CE 解析 A项,娓娓而谈:不倦地、生动地谈论。B项,无可非议:没有什么可以批评、指责的。C项,颠扑不破:比喻理论学说完全正确,不会被驳倒推翻。使用对象有误。D项,见缝插针:比喻尽量利用一切可以利用的空间、时间或机会。E项,亦步亦趋:由于缺乏主张,或为了讨好,事事模仿或追随别人。望文生义。‎ ‎10.[2017·吉林质检]下列各句中,加点的成语使用不恰当的两项是(  )‎ A.有业界专家透露,住建部拟为“奇怪建筑”设判定准则,也就是说,像“铜钱楼”这类众口一词的“奇葩建筑”,以后很可能不会获批建造了。‎ B.为探询奥巴马对朝外交政策,朝鲜不仅高调宣布其试射远程战略导弹,而且强硬中止与韩国的相关协议,美国则拒绝与其单独会谈,美朝双方僵持不下,形势如箭在弦。‎ C.《平凡的世界》这部小说以陕北黄土高原为背景,反映了从“文革”后期到改革初期广阔的社会面貌。时至今日,很多60后对那段不平凡的岁月还记忆犹新。‎ D.相比于持续火爆的住宅市场,多年来,写字楼市场一直处于不瘟不火 的状态,与同地段的住宅楼相比,写字楼的销量要小得多。‎ E.面对中国老龄化程度逐步加深的困境,民政部副部长窦玉沛日前表示,应对老龄化,发展社会养老服务事业刻不容缓。‎ 答案 BD 解析 A项,众口一词:指所有的人都说同样的话。使用正确。B项,如箭在弦:比喻势在必行。不合语境,可换为“剑拔弩张”。C项,记忆犹新:过去的事,至今印象还非常深刻,就像刚才发生的一样。使用正确。D项,不瘟不火:指表演既不沉闷,也不过火。使用对象不当,此处应用“不温不火”,形容不冷淡也不火爆,平淡适中。E项,刻不容缓:片刻也不允许拖延,形容形势紧迫。使用正确。‎ ‎考点三 辨析并修改病句 考点名片 考点内容 基本上包括6个类型的病句:搭配不当、成分残缺或赘余、不合逻辑、结构混乱、语序不当、表意不明。辨析类中,搭配不当、成分残缺或赘余、不合逻辑、结构混乱设题频率较高;修改类中,成分残缺或赘余、表意不明、表述不得体是设题重点。‎ 考查形式 ‎①全国卷重在考查辨析病句,选择题,分值3分;②修改病句,简答题,这类题地方考卷时有出现,分值2~5分。‎ 趋势分析 ‎2018年仍将重点考查辨析病句,以搭配不当、成分残缺或赘余、结构混乱、语序不当四种语病的辨析为考查重点。‎ ‎1.下列各句中,没有语病的一项是(  )‎ A.法国尼斯的恐袭事件备受关注,事件发生几小时后,各个网络媒体关于这一事件报道的新闻的点击量都超过了6万次以上。‎ B.据中国指数研究院最新发布的百城房价指数显示,2016年6月份全国100个城市(新建)住宅平均价格为10564元/平方米,环比在经历连续8个月下跌后,本月微涨0.21%。‎ C.由于最近一段时间球队成绩不佳,俱乐部经过与亚森友好协商达成一致,从2016年7月15日起,亚森不再担任石家庄永昌主教练一职。‎ D.近日,工信部发出通知,移动终端经营商要本着“谁拥有谁管理”的原则,对微视频、微记录、微电影等视听节目推进先验后播的管控制度。‎ 答案 C 解析 A项,成分赘余,去掉“以上”。B项,结构混乱,去掉“据”或“显示”。D项,搭配不当,“推进先验后播的管控制度”中的“推进”改为“推行”。‎ ‎2.下列各句中,没有语病的一项是(  )‎ A.最近,教育部正式向社会公布的高考改革方案似乎深受广大学子们所欢迎,因为它即将打破高考“唯分数论”和“一考定终身”的弊端。‎ B.根据《华尔街日报》的报道,苹果产品在华的销售每况愈下;同样,《苹果财报》显示,上季度该公司在大中华区的营收也减少了三分之一。‎ C.春节期间,网络上热传一篇一个上海女孩跟随江西男友回农村过年,在见到男方家庭准备的第一顿饭后便提出与男友分手,并以最快速度返回上海。‎ D.这次里约奥运会男子200米自由泳比赛在大家的关注下如期举行,参赛选手通过小组预赛和决赛的激烈角逐,最后孙杨以1分44秒65的成绩逆转夺冠。‎ 答案 B 解析 A项,句式杂糅,“为……所欢迎”和“深受……欢迎”糅杂在一起;C项,成分残缺,应在句末加上“的帖子”;D项,成分残缺,“参赛选手通过……角逐”缺少谓语。‎ ‎3.下列各句中,没有语病的一句是(  )‎ A.考古学家对两千多年前在长沙马王堆一号墓新出土的文物进行了多方面的研究,对墓主所处时代有了进一步了解。‎ B.读完徐志摩的《我所知道的康桥》,读者就会被这诗一般的语言所谱写的回忆梦幻曲所感染,使读者感到余味无穷,不忍释手。‎ C.由《太阳的后裔》红遍亚洲的韩国艺人宋仲基被爆过去五年一直为韩国儿童白血病基金会捐款;同时宋仲基也把近期录制《跑男》和《快乐大本营》的酬劳捐献给慈善团体。‎ D.王林呆在实验室里半个月,好像与世隔绝了,所以他回到家,强迫自己看了十天的报纸。‎ 答案 C 解析 A项,语序不当,应将“两千多年前”放在“的文物”前面。B项,句式杂糅,删掉第三个 ‎“所”;结构混乱,删掉“使读者”。D项,去掉“所以”,累赘;“十天的报纸”有歧义。所以选C。‎ ‎4.下列各句中,没有语病的一句是(  )‎ A.《人民日报》推出“创新升级中国制造”专栏,向世界展现中国制造业品牌在提升服务质量、推进创新,也为制造业做大做强品牌提供借鉴。‎ B.给学区房降温,关键在于教育公平起决定作用,要深入推进教育改革,合理分配教育资源,优化学校布局,缩小学校之间教育质量和理念的差距。‎ C.近年来,为保护候鸟,不少地区通过退耕还林还湿、改善湿地水质等措施,优化候鸟的生存环境,为人与候鸟和谐相处创造了更好的条件。‎ D.据介绍,此项改革受益面很广泛,全国240多万家餐饮食业、1400多万从业人员将从中受益,企业将减轻应付“办证”的经济成本和时间。‎ 答案 C 解析 A项,成分残缺,“展现”缺宾语,可在“推进创新”后面加“方面的成绩”。B项,结构混乱,“关键在于教育公平起决定作用”改为“关键在于教育公平”或“教育公平起决定作用”。D项,搭配不当,“减轻……成本和时间”不搭配,可以把“减轻”改为“减少”。‎ ‎5.下列各句中,没有语病的一句是(  )‎ A.火箭军全体官兵要把握火箭军的职能定位和使命任务,按照核常兼备、全域慑战,增强可信可靠的核威慑和核反击能力,加强中远程精确打击力量建设。‎ B.“一带一路”“亚投行”等名词越来越为人们所熟知,中国提出的这些倡议不仅对全球和区域发展都有利,而且对中国自身有利。‎ C.中国南沙群岛永暑礁新建机场已竣工,中国政府征用民航飞机对该机场进行校验试飞,旨在测试该机场设施是否符合民用航空标准。‎ D.中巴经济合作能很好地改善巴基斯坦的基础设施和巴中经济走廊的建设,推动两国双边贸易额的增长,开启两国合作的新征程。‎ 答案 C 解析 A项,成分残缺,“全域慑战”后面加“的战略要求”。B项,语序不当,应为“不仅对中国自身有利,而且对全球和区域发展都有利”。D项,搭配不当。“改善”和“建设”不能搭配。‎ ‎6.下列各句中,没有语病的一句是(  )‎ A.日本政府从来没有放弃,也绝不可能放弃侵略扩张的国策,安倍上台后所推行的一系列诸如“侵占钓鱼岛”“废弃《和平宪法》”以及“否认侵略历史”等政策,都充分显示出日本政府侵略他国成性的狼子野心昭然若揭。‎ B.中央自从下定反腐决心、决定老虎苍蝇一起打后,各级政府官员作风大有改观,但仍有不少贪腐分子不知收敛,依然我行我素。‎ C.据业内人士保守估计,中国每年有超过1000万人整形,女性比例为83%,男性比例约为17%,整个行业的年增速则约在30%以上。‎ D.在浙江乌镇举行的世界互联网大会开幕式上,习近平发表演讲,提出了“天下兼相爱则治,交相恶则乱”的开放合作、互信互利的主张,这得到与会者的广泛认同。‎ 答案 D 解析 A项,句式杂糅,去掉“昭然若揭”。B项,中途易辙,“中央”缺谓语。C项,“约”与“以上”重复。‎ ‎7.下列各句中,没有语病的一句是(  )‎ A.中国科学技术大学发布了我国首台具有体验交互功能的美女机器人“佳佳”,不仅有和常人一般逼真的面庞,还能像真人一般和普通人交流。‎ B.中国男足奇迹般地获得了2018年世界杯亚洲区12强赛资格,这是他们时隔14年,继2002年之后再度进入世预赛亚洲区最后阶段的比赛。‎ C.“互联网+创业创新”,就是要充分发挥互联网的创新驱动作用,引导和推动全社会大众创业、万众创新的浓厚氛围,打造经济发展新引擎。‎ D.漫游费被人形象地称为“化石费用”,就在于其是20年前2G时代的产物,可如今早已是4G时代,收取漫游费,道理上是很难说得通的。‎ 答案 D 解析 A项,偷换主语,将“发布了”改为“发布的”,或者在“不仅”前加上“佳佳”;B项,语序不当,“这是他们时隔14年,继2002年之后再度进入世预赛亚洲区最后阶段的比赛”应调整为“这是他们继2002年之后,时隔14年再度进入世预赛亚洲区最后阶段的比赛”;C项,搭配不当或成分残缺,“引导和推动”不能与“氛围”搭配,在“引导和推动全社会”之后加“形成”。‎ ‎8.下列各句中,没有语病的一句是(  )‎ A.中医药产业繁荣发展的前提是要重建文化自信,只有建立了文化自信,中医药产业的从业人员才会达到文化自觉、自律,才能获得更多百姓的认同。‎ B.路遥文学奖的发起人高玉涛表示,近日将召开路遥文学奖专家研讨会,并邀请陕西文学界、艺术界、评论界、教育界的知名人士参加,探讨路遥文学奖如何取得公信力与权威性。‎ C.“财政悬崖”问题只是表象,它的根源是美国长期积累的高债务、高赤字问题造成的,是美国式“经济结构”问题,它的化解也需要较长的过程。‎ D.成熟的市场法则强调自愿平等、公开透明,在我们的道德直觉中,“一个愿打一个愿挨”也容易获得合理性,但我们不能忽略另一个关键词——公平。‎ 答案 D 解析 A项,语序不当,“自觉、自律”应改为“自律、自觉”;B项,“探讨”缺少宾语;C项,句式杂糅,去掉“造成的”。‎ ‎9.下列各句中,没有语病的一项是(  )‎ A.部分国企中退休后收入高于退休前收入的现象将会阻碍延迟退休展开的氛围,因此,应开展与延迟退休相应的制度改革。‎ B.魏则西事件在经过众多媒体报道引发强烈关注后,相关部门调查认为百度竞价排名机制影响了搜索结果的公正性和客观性,容易误导网民,必须立即整改。‎ C.谁都不喜欢雾霾,谁都希望政府采取有效措施驱逐雾霾,可是当红色预警发布时,为何很多人仍要违反车辆限行等相关规定呢?‎ D.检察机关侦查监督部门要做好电信网络诈骗犯罪专项工作,依法稳妥办理电信网络诈骗犯罪案件,坚决遏制电信网络诈骗的高发蔓延势头。‎ 答案 C 解析 A项,“阻碍”与“氛围”搭配不当。B项,中途易辙,“魏则西事件”无谓语,而又以“相关部门”为主语重新说了一句话。D项,成分残缺,应在“做好”后面加上“打击”。‎ ‎10.下列各句中,没有语病、句意明确的一项是(  )‎ A.农业部将“镰刀弯”地区中的非优势区作为玉米结构调整的重点,涉及到河北、山西、内蒙古、辽宁、吉林、黑龙江等13个省份的部分地区。‎ B.针对社会反映强烈的限制竞争和垄断行为,工商总局决定自2016年4~10月在全国范围内集中整治公用企业限制竞争和垄断行为专项执法行动。‎ C.很多地方的建筑垃圾的资源化、产业化不理想,就是因为在其运输、处置、产生和利用等环节上,有不同程度的机制滞后或政策缺位。‎ D.当年的金沙江流域,很多林场都面临着变成荒山的危险,由于缺乏足够的森林涵养水源,洪涝、滑坡、泥石流等自然灾害经常发生。‎ 答案 D 解析 A项,成分赘余,去掉“到”;B项,成分残缺,“集中”后添加“开展”;C项,语序错误,“运输、处置、产生”改为“产生、运输、处置”。‎ ‎11.下列各句中,没有语病的一项是(  )‎ A.想让城市变得文明,靠的是专业的管理、高效的组织、科学的协调、连贯性的政策取得的,而非短时间的突击、会战,更不是将不好的东西遮掩藏匿。‎ B.在雾天,饮食时应选择清淡、易消化且富含维生素的食物,多饮水,多吃新鲜蔬菜和水果,这样不仅可以补充各种维生素和无机盐,还能起到润肺除燥、健脾补肾的作用。‎ C.陈寅恪先生的这番话,不仅是对像王国维先生这样的历史文化人物的忧伤,而且是对整个文化精神的忧伤,更是对一种即将衰落的文化的忧伤。‎ D.北京市政协提出建议:应以功能调整优化为目标,启动北京市部分行政机关及事业单位向外转移。‎ 答案 B 解析 A项,结构混乱,“靠的是……取得的”杂糅,去掉“取得的”。C项,“而且是”和“更是”后面的内容语序颠倒,不合逻辑;D项,成分残缺,在“向外转移”后加“的工作”。‎ ‎12.下列各句中,没有语病的一句是(  )‎ A.迪士尼动画影片《疯狂动物城》不仅故事有趣,动物主角可爱,情节引人入胜,而且能将哲理意义、文化内涵自然巧妙地融进精彩的故事情节中,给人以启迪。‎ B.在第53届意大利博洛尼亚国际书展上,中国儿童文学作家曹文轩获得2016年“国际安徒生奖”,开创了华人在这一世界儿童文学领域至高奖项上零的突破。‎ C.“中国诗词大会”是央视首档以诗词为主题的大型全民互动益智节目,其赛制、内容和表现形式都富有新意,力求打造一席特色鲜明的文化盛宴。‎ D.我国即将实施的首部慈善法对慈善活动进行了明确界定,同时在规范慈善组织设立运营、促进慈善事业发展、开展慈善服务等方面也做出了规定。‎ 答案 A 解析 B项,搭配不当,最后一句应改为“实现了……零的突破”;C项,偷换主语,最后一句应改为“节目组力求把该节目打造成一席特色鲜明的文化盛宴”;D项,语序不当,最后一句应改为“……在规范慈善组织设立运营、开展慈善服务、促进慈善事业发展等方面……”。‎ ‎13.下列句子中,没有语病的一句是(  )‎ A.吸烟产生的烟雾中含有上百种对人体有害的化学物质,这些物质使机体发生病变,会引发心血管、肺癌等严重疾病。‎ B.2016年春晚凸显“中国元素”,以“中国情”打动了人心,以“中国魂”凝聚人心,以“中国梦”鼓舞人心,处处体现“中国味”。‎ C.引力波的发现,为人类开启了一场新的探索旅程,传承和接收这种来自人类自身的“引力波”,更具有生命和梦想绵延不绝的实际意义。‎ D.有关统计调查显示,一、二月我市空气质量下降,可吸入颗粒物浓度严重超标。环保专家分析认为,建筑工地扬尘污染是污染源的首要因素。‎ 答案 B 解析 A项,不合逻辑,“心血管”不是疾病。C项,语序不当,将“传承”与“接收”互换。D项,句式杂糅,最后一句应为“……建筑工地扬尘是造成污染的首要因素”,或“建筑工地扬尘是首要污染源”。‎ ‎14.下列各句中,没有语病的一句是(  )‎ A.开门反腐,有利于在反腐机构和群众间建立起安全可靠且通畅无阻的联系通道,充分调动群众参与反腐的积极性,使反腐获得最有效的民意支持。‎ B.谣言的破坏力是毋庸置疑的,如不及时扑灭,会使整个群体处于一种恐慌状态,对公众造成的创伤,乃至引起社会动荡,也不是不可能的。‎ C.虽然有些城市已经开启跨境电子商务试点,但是其发展依然没有达到预期目标,跨境电子商务要实现发展,重在打破制度壁垒是关键。‎ D.中国年反映的尽管是中国人民一直抱有的强烈的大团圆意愿,还有中华民族在数千年历史长河中形成的那种强大的凝聚力与向心力。‎ 答案 A 解析 B项,成分赘余,应删去“对公众造成的创伤”中的“的”。C项,句式杂糅,删去“重在”或“是关键”。D项,关联词语“尽管”与“还”搭配不当,可以将“尽管”改为“不仅”。 ‎ ‎15.下列各句中,没有语病的一句是(  )‎ A.借助手术机器人的运动比例缩放功能,医生可减少手部的自然颤抖或无意移动现象,从而进一步提高手术操作的精确度。‎ B.《黄河颂》这幅油画作品的构图、色彩引起了美术界广泛关注,也是陈逸飞自认为最得意的作品,奠定了他在中国美术史上的地位。‎ C.以提高反应速度、方便作战指挥的需求出发,本次军改不仅调整了四大总部的管理模式,还把七大军区整合为更集约的六大战区。‎ D.中国经济步入转型期,传统行业产能过剩现象突出,创新行业还没有成型,对于众多机构鼓吹的A股牛市不能抱过高的奢望。‎ 答案 A 解析 B项,“构图、色彩”与“作品”搭配不当,也不能“奠定……地位”。C项,“以……为出发点”与“从……出发”两种句式杂糅。D项,“过高的奢望”,成分赘余。‎ ‎16.下列各句中,没有语病的一项是(  )‎ A.根据最新的尼泊尔政府草案显示,年龄在18岁以下或者75岁以上的人士将失去攀登珠峰的资格。‎ B.郭先生常戴一副黑框眼镜,穿一件质朴的外套,他除了对理论有精深研究,他的课堂也是趣味横生的。‎ C.果壳网拥有“谣言粉碎机”等15个主题站,通过粉碎社会上流传的谣言,让科学和大众需求产生联系,目前日均浏览量已超过375万。‎ D.2015年,一项利用人体上皮细胞成功制造某种干细胞的重大科研成果将他第一次推到了公众视野之中。在此之前,他并不被为人所熟知。‎ 答案 C 解析 A项,句式杂糅,首句可改为“根据最新的尼泊尔政府草案”或者“最新的尼泊尔政府草案显示”。B项,偷换主语,“他除了对理论有精深研究”一句话还没讲完,后面换成了“他的课堂”。D项,“被”与“为……所”语义重复,应把“被”去掉。‎ ‎17.下列各句中,没有语病的一句是(  )‎ A.未来十年,中国有望成为世界第一大经济体,而且其在世界500强企业中的数量也有望超过美国居世界第一。‎ B.按照布里姆的看法,卢梭在近代思想史上的主要地位在于他对现代性及其所造就的主导类型的“资产者”的激烈批评决定的。‎ C.海子、王小波的作品之所以在一般读者中引起深刻影响,是否是因为这两个人的作品,触动了有关我们这个时代脉搏中某些深层的东西?‎ D.去年8月以来,该犯罪嫌疑人在当地制造多起电动车电瓶失窃案,遭民警蹲守围堵时,还想强行冲门,却被摩托车带倒,最终成功被警方抓获。‎ 答案 A 解析 B项,句式杂糅,将“在于”改为“是由”。C项,“引起……影响”搭配不当;“有关”赘余。D项,搭配不当,应该将“失窃”改为“盗窃”;“最终成功被警方抓获”不合逻辑,应该删去“成功”。‎ ‎18.下列各句中,没有语病的一项是(  )‎ A.曹文轩获国际安徒生奖,是继莫言获得诺贝尔文学奖、刘慈欣获得雨果奖之后,中国作家再次获得国际大奖的青睐,具有里程碑式的意义。‎ B.姚明入选2016年奈史密斯篮球名人堂,是第一位中国人入选,因为名人堂认为他帮助NBA扩大了在中国的知名度。‎ C.《太阳的后裔》首次引入中韩同步播出,国内观众每周都能和韩国观众同步看到最新一集的内容,大家共同为剧情揪心期待。‎ D.旅游已成为国人常态化的生活选项,游客从过去关注景点,逐渐转向体验城乡生活,与当地居民共享常态化生活空间。‎ 答案 D 解析 A项,结构混乱。应将“的青睐”删去。B项,搭配不当。应将“是第一位中国人入选”改为“‎ 是第一位入选的中国人”,“扩大”应改为“提高”。C项,成分残缺。“播出”后面加上“模式”。‎ ‎19.下列各句中,没有语病的一句是(  )‎ A.任何一种文明的发展都是与其他文明碰撞、融合、交流的过程,完全封闭的环境不可能带来文明的进步,只会导致文明的衰落。‎ B.推行有偿使用塑料袋,主要是通过经济手段培养人们尽量减少使用塑料袋,这无疑会对减少白色污染、净化环境产生积极作用。‎ C.2015年诺贝尔医学或生理学奖获得者屠呦呦主持研究发明药物青蒿素,在帮助人类抗击疟疾方面做出了巨大贡献,挽救了无数人的生命。‎ D.文艺复兴揭开了欧洲腾飞的序幕,工业革命拉大了欧洲与中国的距离,当火车在欧洲大地高歌猛进的时候,中国的辽阔土地上,木制独轮车还在吱吱呀呀地唱着千年的凄凉。‎ 答案 D 解析 A项,语序不当,“碰撞、融合、交流”应改为“碰撞、交流、融合”;B项,成分残缺,“培养”后面缺少宾语中心语,应改为“培养人们尽量减少使用塑料袋的习惯”;C项,搭配不当,应为“屠呦呦主持研究发明的药物青蒿素”。‎ ‎20.下列各句中,没有语病的一句是(  )‎ A.为加强历史文化名城、名镇、名村的保护与管理,国务院颁布了《历史文化名城名镇名村保护条例》,该条例规范了历史文化名城、名镇、名村的申报与批准。‎ B.一般而言,衡量一次国事访问成功与否,自然离不开取得的具体成果,但更关键的是相互之间由此增强的互信以及在此基础上关系的稳定。‎ C.中华传统文化具有强烈的现实性、变异性,它无时无刻不在影响、制约着今天的中国人,为我们开创新文化奠定历史的根据和现实的基础。‎ D.为提升公众对暗物质卫星的关注度,激发全国民众对空间科学的兴趣和热爱,中国科学院主办了暗物质粒子探测卫星征名活动。‎ 答案 D 解析 A项,成分残缺,应为“规范了……申报与批准程序”。B项,一面对两面。C项,“奠定……根据”搭配不当,改为“提供……根据和基础”。‎ ‎21.下列各句中,没有语病的一项是(  )‎ A.每两年评选一次的“国际安徒生奖”2016年4月4日下午揭晓,中国儿童文学作家曹文轩获得该奖,这是首次中国作家获得该奖项。‎ B.长江野生动物资源破坏严重,濒危程度不断加剧,经渔业渔政部门调查论证,国家拟将长江刀鱼列入《国家重点保护野生动物名录》。‎ C.全面放开二孩政策后,政府要切实采取措施解决百姓有关妇女就业、孩子的养育成本、卫生及教育公共资源供给等不敢生的顾虑。‎ D.李克强总理要求相关部门彻查“问题疫苗”的流向和使用情况,依法严厉打击违法犯罪行为,相关失职渎职行为严肃问责,绝不姑息。‎ 答案 B 解析 A项,语序不当,应为“这是中国作家首次获得该奖项”。C项,“解决……顾虑”搭配不当。D项,成分残缺,“相关失职渎职行为严肃问责”前加“对”。‎ ‎22.下列各句中,没有语病、句意明确的一项是(  )‎ A.我国版权保护大环境尽管持续改善,但在今后一段时间内仍将是版权纠纷高发期,保护知识产权需要一个长期的过程。‎ B.既然环评的目的是预防环境不被污染,那么有关方面就应该将项目建设中的环境隐患纳入环评范围,全过程回应民众关切。‎ C.我刚四岁,爸爸就离开奶奶和妈妈去广东打工,一直到我七岁的时候才回来,我孤僻、不爱说话的性格,大概就与此有关吧。‎ D.园林专家指出,海芋、马蹄莲等植物体内所含的草酸钙针晶会刺激皮肤,引起瘙痒和水肿,严重的可诱发窒息,导致死亡。‎ 答案 D 解析 A项,搭配不当,“大环境”与“高发期”主宾搭配不当。B项,不合逻辑,删除“不被”。C项,表意不明,“离开奶奶和妈妈”有歧义。‎ ‎23.下列各句中,没有语病的一句是(  )‎ A.消费者习惯买涨不买落,若都等着买便宜的东西,就会导致企业产品积压,进而滞销,进而影响企业运转与员工收入,这是不良的连锁反应。‎ B.丝绸之路,这条起始于2100多年前的商旅通道,作为连接亚欧贸易的桥梁和融汇东西文明的纽带,开拓了千古传诵的壮美篇章。‎ C.我国目前已推行的涉农专业学生中职免费,意图让一些愿上中职的经济困难的学生能够上得起学。‎ D.屠呦呦的丈夫李廷钊回忆说,研究青蒿素的时候,用作溶液的是沸点较低的乙醚,屠呦呦每天回到家都满身酒精味,后来甚至患上了中毒性肝炎。‎ 答案 D 解析 A项,不合逻辑,企业产品应该是先滞销,后积压。B项,搭配不当,“开拓”应改为“抒写”。C项,成分残缺,应在“中职免费”后加“政策”。‎ ‎24.下列各句中,没有语病、句意明确的一项是(  )‎ A.白皮书说,中国人民在南海的活动已有2000多年历史。中国最早发现、命名和开发利用南海诸岛及相关海域,最早并持续、和平、有效地对南海诸岛及相关海域行使主权。‎ B.家属称,从今年3月开始,锦鹿公司虽然承担戒毒所西北区域的填土项目,但双方未签订合同。‎ C.虽然学校三令五申,要求学生自觉爱护学校环境,但是随手乱丢垃圾,人走关灯,损坏桌椅等现象依然十分严重,令人担忧。‎ D.由于《古文观止》具有特色,自问世以后近三百年来,广为传布,经久不衰,至今仍不失为一部有价值的选本。‎ 答案 A 解析 B项,语序不当,“虽然”放到“锦鹿公司”前面;C项,“人走关灯”与“十分严重”搭配不当;D项,成分残缺,删掉“由于”。‎ ‎25.下列各句中,没有语病的一句是(  )‎ A.第30届全国青少年科技创新大赛终评期间,将举办一系列开幕式暨创新论坛、科学讲坛、科学讨论会等丰富多彩的活动。‎ B.专业信息披露是消除公众对问题疫苗恐慌最好的“疫苗”,这一“针”只有及时打,才能避免这种滚雪球式的传播将更多人带入更大程度的恐慌中。‎ C.据调查,目前最适宜生育的人群介于70后父母及80后年轻父母,他们又是受中国计划生育影响最为深远的一代。‎ D.数据是未来创新社会最重要的生产,而阿里在本质上是一家扩大数据价值的公司,相信阿里在数据和技术上的投入会有巨大回报。‎ 答案 B 解析 A项语序不当,将“一系列”移到“等”之后。C项成分残缺,缺少“介于”的宾语,在“80后年轻父母”后面补上“之间”。D项搭配不当,“数据”与“生产”主宾不搭配,在“生产”后补上“资料”。‎ ‎26.下列各句中,没有语病的一句是(  )‎ A.诗歌不揭露现实,不批判恶世,而仅仅是附丽于虚张浮华之中,这样的诗歌无法走出困境;当社会走出困境之时,诗歌或许有望走出。‎ B.每一个人降生到这个世界上来,都有一个对于他最适宜的位置,这个位置对于他是否最适宜,应该去问自己的生命和灵魂,看它们感到快乐和幸福。‎ C.无论从剧本、表演和制作规模层面来讲,由原班人马四年打磨的警匪大片《寒战2》都做到了极致,可谓《无间道》之后“香港警匪片的新标杆”。‎ D.目前我国的职业学校培养了学生的职业技能,却没有培养他们职业技术的文化,即工匠精神,这是对职业技能的追求和坚守。‎ 答案 A 解析 B项,两面对一面,应在“感到”前加上“是否”。C项,搭配不当,“无论……和……”应改为“无论……还是……”。D项,表意不明,“这”指代不明。‎ ‎27.下列各句中,没有语病、句意明确的一句是(  )‎ A.由管虎执导,冯小刚、许晴、张涵予等主演的电影《老炮儿》票房突破9亿元,刷新了现实主义创作的市场纪录,更提振了业界对现实主义创作的决心。‎ B.不仅是中国、俄罗斯、韩国,就连美国的不少专家也认为,在韩国部署“萨德”系统,不仅会使矛盾更加复杂,反而无助于朝鲜半岛局势的改善。‎ C.我们要破解发展难题,厚植发展优势,实现“十三五”时期发展目标,就必须牢固树立并切实贯彻创新、协调、绿色、开放、共享的发展理念。‎ D.今年年初以来,日元升值和股市下跌使得日本经济前景蒙上迷雾,日本执政党自民党内部出现了要求动用财政措施,称应投入5万亿日元刺激经济。‎ 答案 C 解析 A项,搭配不当,“提振”与“决心”不搭配;B项,语序不当,“不仅会使矛盾更加复杂,反而无助于朝鲜半岛局势的改善”逻辑递进关系错误,交换分句顺序;D项,成分残缺,“出现了”后面没有宾语。‎ ‎28.下列各句中,没有语病的一项是(  )‎ A.经过30多年的快速发展,人们的诉求实现了从生存到生态、从温饱到环保,“会呼吸的痛”让人们渴望干净的空气和水。‎ B.“中国诗词大会”是央视首档以诗词为主题的大型全民互动益智节目,其赛制、内容和表现形式都富有新意,力求打造一席特色鲜明的文化盛宴。‎ C.利辛女子“犬口救子”诈捐一事引起了社会热议,不少专家和网友谴责这种行为是在透支社会的善意。《慈善法(草案)》规定将依法查处社会上存在的这种诈捐现象。‎ D.奥斯卡金像奖的正式名称是“电影艺术与科学学院奖”,该奖项设立于1927年,由美国电影艺术与科学学院颁发,旨在为了鼓励优秀电影的创作与发展,半个多世纪来一直享有盛誉。‎ 答案 C 解析 A项,成分残缺,应在“从温饱到环保”后加上“的转变”。B项,偷换主语,应改为“节目组力求把该节目打造成一席特色鲜明的文化盛宴”。D项,重复赘余,删去“为了”。‎ ‎29.下列各句中,没有语病的一句是(  )‎ A.2016年是“十三·五”规划的开局之年,能否做好“十三·五”期间各项工作,确保如期全面建成小康社会,需要全国各族人民共同努力。‎ B.我国军民融合发展刚进入由初步融合向深度融合的过渡阶段,还存在思想观念跟不上、顶层统筹统管体制缺乏、政策法规和运行机制滞后、工作执行力度不够等问题。‎ C.中国特色大国外交的努力目标是助力民族复兴的“中国梦”和建设人类命运共同体,战略选择是推动世界的和平发展,同时坚持自身的和平发展。‎ D.在一定意义上说,一个城市的文明程度,不仅取决于有多少高楼大厦,人文景观,而是取决于它让人感受到的舒适“温度”。‎ 答案 B 解析 A项,搭配不当,“能否”与后文两面对一面。C项,语序不当,应先“坚持自身的和平发展”再“推动世界的和平发展”。D项,关联词搭配不当,应为“不是……而是”。‎ ‎30.下列各句中,没有语病的一句是(  )‎ A.鬼斧神工的自然造化,瑰丽多姿的文化遗存,为新疆注入了无限魅力,更铸就了新疆独一无二的风土人情。‎ B.作为武汉对口支援的三峡库区县,兴山42吨蔬菜瓜果的5辆大货车将赶往武汉洪涝灾区,确保灾区人民能吃上新鲜蔬菜。‎ C.第六届国际数字地球高峰会议日前在北京开幕研讨大数据时代数字地球科学技术进展与面临的挑战问题。‎ D.首款面向云计算的高密度融合架构服务器产品——星河SDC1000,具有软件定义体系结构、模块化设计、高密度、高可扩展、高性能功耗比和高价格比等特点。‎ 答案 D 解析 A项,语序不当,应改为“铸就了新疆独一无二的风土人情,更为新疆注入无限魅力”。B项,缺少动词,在“42吨蔬菜瓜果”前加上“满载”一词。C项,句式杂糅,可改为“第六届国际数字地球高峰会议日前在北京开幕,本届峰会研讨大数据时代数字地球科学技术进展与面临的挑战问题”。‎ ‎ [3年高考真题集训]‎ ‎1.[2016·全国卷Ⅰ]下列各句中,没有语病的一句是(  )‎ A.近日刚刚建成的西红门创业大街和青年创新创业大赛同步启动,绿色设计和“互联网+农业”设计是本次赛事的两大主题。‎ B.最近几年,从中央到地方各级政府出台了一系列新能源汽车扶持政策,节能环保、经济实惠的新能源汽车逐渐进入老百姓的生活。‎ C.实时性是以互联网为载体的新媒体的重要特点,是通过图片、声音、文字对新近发生和正在发生的事件进行传播的。‎ D.广西传统文化既具有典型的本土特色,又兼有受中原文化、客家文化、湘楚文化共同影响下形成的其他特点。‎ 答案 B 解析 A项,搭配不当,“大街”与“启动”不搭配。C项,成分残缺,在“文字”后添加“等方式”。D项,结构混乱,将“受”改为“在”,或删除“下”。‎ ‎2.[2016·全国卷Ⅱ]下列各句中,没有语病的一句是(  )‎ A.自从我国第一颗人造卫星“东方红一号”成功发射,成为世界上第五个把卫星送上天的国家以来,我国的航天事业取得了巨大的突破。‎ B.国务院近日发布盐业体制改革方案,提出不再核准新增食盐定点生产批发企业,取消食盐批发企业只能在指定范围内销售,允许它们开展跨区域经营。‎ C.职业教育的意义不仅在于传授技能,更在于育人,因此有意识地把工匠精神渗透进日常的技能教学中是职业教育改革的重要课题。‎ D.面对突然发生的灾难,一个地方抗灾能力的强弱既取决于当地经济实力的雄厚,更取决于政府的应急机制和领导人的智慧。‎ 答案 C 解析 A项,偷换主语,主语由“东方红一号”偷换为“我国”。B项,成分残缺,“取消”后面缺少相应的宾语。D项,一面对两面。前面说的是“一个地方抗灾能力的强弱”,后面说的是“当地经济实力的雄厚”。语病题通常的做法是压缩后看主干有没有残缺、搭配不当等毛病,再看压缩掉的修饰成分与主干之间是否存在修饰不当的毛病,最后看句子中有没有典型的语病特征,比如表并列的词或标点,是否、能否、好坏等表两面的词,是、成为、变成等表判断的词。‎ ‎3.[2016·全国卷Ⅲ]下列各句中,没有语病的一句是(  )‎ A.随着技术的进步和经验的积累,再加上政策的扶持,使得我国自主品牌汽车进入快速发展时期,各种创新产品层出不穷。‎ B.如果有一天科技发展到人们乘宇宙飞船就像今天乘飞机一样方便的时候,银河就不再遥远,宇宙也就不再那么神秘了。‎ C.首届跨境电商论坛近日在北京举行,来自各知名电商的数十名代表齐聚一堂,分析了电商企业面临的机遇和挑战。‎ D.在第40个国际博物馆日到来之际,本市历时三年开展的第一次全国可移动文物普查工作,昨日交出了首份答卷。‎ 答案 C 解析 A项,成分残缺,滥用介词“随着”而造成主语缺失,可以删去“随着”。B项,结构混乱,“如果……”与“当……的时候”杂糅,可将“如果”改为“当”或删去“的时候”。D项,语序不当,应把“历时三年”移至“普查工作”之后。‎ ‎4.[2016·天津高考]下列各句没有语病的一句是(  )‎ A.日前,来自京津冀的近千名鸟类摄影爱好者相聚在北大港湿地,在与可爱的飞翔精灵亲密接触并拍摄了大量照片的同时,还无形中上了一堂爱鸟护鸟知识课。‎ B.“双创特区”以围绕聚集青年大学生、高校和科研院所科技人才、海外人才、企事业人员四类人才为重点,创新创业。‎ C.这场专项整治行动是为规范互联网金融在迅速发展过程中的各种乱象,经过广泛征集意见,酝酿一年之久,形成最终方案。‎ D.京剧是中国独有的表演艺术,它的审美情趣和艺术品位,是中国文化的形象代言之一,是世界艺术之林的奇葩。‎ 答案 A 解析 B项,“以围绕……为重点”句式杂揉。C项,“这场专项整治行动是为了……”“形成最终方案”中途易辙。D项,“它的审美情趣和艺术品位”“是”“奇葩”主宾搭配不当。‎ ‎5.[2016·浙江高考]下列各句中,没有语病的一项是(  )‎ A.面对电商领域投诉激增的现状,政府管理部门和电商平台应及时联手,打击侵权和制售假冒伪劣商品,保护消费者的合法权益。‎ B.自开展禁毒斗争以来,我国每年新发现的吸食海洛因人员增幅从2008年的13.7%降至2013年的6.6%,近五年来戒断毒瘾三年以上人员已逾120万。‎ C.在线教师时薪过万的消息自从引发社会关注后,每一个教育工作者都应当意识到,如何与力量巨大的互联网相处正成为教育不得不直面的问题。‎ D.英国皇家莎士比亚剧团艺术总监对昆曲《牡丹亭》华美的唱腔和演员娴熟的技巧惊叹不已,赞美昆曲精美绝伦的服装与简洁的舞台设计形成了奇妙的平衡。‎ 答案 B 解析 A项,成分残缺,“打击”缺少宾语中心语,可在“伪劣商品”后加“的行为”。C项,语序不当,可把“自从”提到句首。D项,结构混乱,可把“形成了”改为“形成的”,或删去“形成了奇妙的平衡”。‎ ‎6.[2016·山东高考]下列各句中,没有语病、句意明确的一项是(  )‎ A.从意外致残、生活无望到残奥会夺冠,并获得“中国青年五四奖章”,他走出了一条不平凡的人生道路。‎ B.该型飞机在运营成本上是其他同级别机型的1.3至2倍,优势明显;在商载、航程、航速等方面也极具竞争力。‎ C.学校宿舍、教学楼等人群密集区,一旦发生火灾,后果不堪设想,因此学生掌握火灾中自救互救相当重要。‎ D.央视《大国工匠》系列节目反响巨大,工匠们精益求精、无私奉献的精神引发了人们广泛而热烈的讨论和思考。‎ 答案 A 解析 B项,不合逻辑,运营成本高不是优势。C项,成分残缺,在“学生掌握火灾中自救互救”后加“的方法”。D项,搭配不当,“热烈的”与“思考”不搭配。‎ ‎7.[2015·全国卷Ⅰ]下列各句中,没有语病的一句是(  )‎ A.为纪念抗日战争暨世界反法西斯战争胜利70周年,从现在起到年底,国家大剧院宣布将承办31场精心策划的演出。‎ B.这部小说中的“边缘人”是一个玩世不恭、富有破坏性却真实坦白的群体,人们面对这类形象时会引起深深的思索。‎ C.根据国家统计局发布的数据,4月份我国居民消费价格指数出现自去年12月以来的最大涨幅,但仍低于相关机构的预测。‎ D.为进一步保障百姓餐桌安全,国家对施行已超过5年的《食品安全法》作了修订,因加大了惩处力度而被冠以“史上最严”的称号。‎ 答案 C 解析 A项,语序不当,“从现在起到年底”应置于“国家大剧院宣布”之后;B项,句式杂糅,句中用了“人们面对……会……”“这类形象会引起……”两种句式;D项,偷换主语,“被冠以‘史上最严’称号”的主语应是“《食品安全法》”。‎ ‎8.[2015·全国卷Ⅱ]下列各句中,没有语病的一句是(  )‎ A.“地坛书市”曾经是北京市民非常喜爱的一个文化品牌,去年更名为“北京书市”并落户朝阳公园后,依旧热情不减。‎ B.“丝绸之路经济带”横跨亚、非、欧三大洲,其形成与繁荣必将深刻影响世界政治、经济格局,促进全球的和平与发展。‎ C.在那个民族独立和民族解放斗争风起云涌的时代,能激发人们的爱国热情是评判一部文学作品好坏的非常重要的标准。‎ D.父亲住院期间,梅兰每天晚上都陪伴在他身旁,听他讲述一生中经历的种种苦难和幸福,她就算再忙再累,也不例外。‎ 答案 B 解析 A项,暗换主语。前半句主语是“地坛书市”,后半句“依旧热情不减”的主语就暗换为“北京市民”了。C项,一面对两面。应该在“激发”前加“否”。D项,语序不当,改为“就算她再忙再累,也不例外”。‎ ‎9.[2014·全国卷Ⅰ]下列各句中,没有语病的一句是(  )‎ A.作为古希腊哲学家,他在本体论问题的论述中充满着辩证法,因此被誉为“古代世界的黑格尔”。‎ B.由此可见,当时的设计者们不仅希望该过程中艺术活动是富有创造性的,而且技术活动也是富有创造性的。‎ C.本书首次将各民族文学广泛载入中国文学通史,但就其章节设置、阐释深度等方面依然有很大的改进空间。‎ D.古代神话虽然玄幻瑰奇,但仍然来源于生活现实,曲折地反映了先民们征服自然、追求美好生活的愿望。‎ 答案 D 解析 A项,成分残缺、搭配不当,“他在本体论问题的论述中”缺少必要的介词,应该在“本体论”前面添加介词“对于”;“充满”与“辩证法”不搭配,应为“充满着辩证法思想”。B项,语序不当,“不仅”放到“希望该过程中”后,否则,“希望该过程”与“技术活动”成了并列成分,显然不合逻辑;并且,“艺术活动”和“技术活动”也要调换一下位置。C项,结构混乱,通常说“就……而言(来说)”,所以“方面”后面应添加“而言(来说)”等词语。‎ ‎10.[2014·全国卷Ⅱ]下列各句中,没有语病的一句是(  )‎ A.他在新作《世界史》的前言中系统地阐述了世界是个不可分割的整体的观念,并将相关理论在该书的编撰中得到实施。‎ B.作为一名语文老师,他非常喜欢茅盾的小说,对茅盾的《子夜》曾反复阅读,一直被翻得破烂不堪,只好重新装订。‎ C.《舌尖上的中国》这部风靡海内外的纪录片,用镜头展示烹饪技术,用美味包裹乡愁,给观众带来了心灵的震撼。‎ D.如果我们能够看准时机,把握机会,那么今天所投资百万元带来的效益,恐怕是五年后投资千万元也比不上的。‎ 答案 C 解析 A项,搭配不当,“理论”与“实施”不搭配。B项,结构混乱,“一直被翻得破烂不堪”一句暗换主语。D项,语序不当,“所”字应放在“带来的效益”之前。‎ ‎11.[2015·山东高考]下列各句中,没有语病、句意明确的一项是(  )‎ A.除了驾驶员要有熟练的驾驶技术、丰富的驾驶经验之外,汽车本身的状况,也是保证行车安全的重要条件之一。‎ B.帮助家境不好的孩子上大学,是我们应该做的,况且这孩子各方面都很优秀,我们一定要帮助她圆大学梦。‎ C.说到人才培养,人们往往想到要学好各门课程的基础理论,而对与这些理论密切相关的逻辑思维训练却常常被忽视。‎ D.这部影片讲述了一个身患重病的工人的女儿自强不息、与命运抗争的故事,对青少年观众很有教育意义。‎ 答案 B 解析 A项,两面对一面,“汽车本身的状况”有好有坏,与“保证行车安全”不能搭配。C项,句式杂糅,“对……被忽视”杂糅,应该表述为“而常常忽视与这些理论密切相关的逻辑思维训练”,或“而与这些理论密切相关的逻辑思维训练却常常被忽视”。D项,表意不明,“一个身患重病的工人的女儿”有歧义。‎ ‎12.[2015·四川高考]下列各句中,没有语病的一项是(  )‎ A.首届“书香之家”颁奖典礼,是设在杜甫草堂古色古香的仰止堂举行的,当场揭晓了书香家庭、书香校园、书香企业、书香社区等获奖名单。‎ B.专家强调,必须牢固树立保护生态环境就是保护生产力的理念,形成绿水青山也是金山银山的生态意识,构建与生态文明相适应的发展模式。‎ C.市旅游局要求各风景区进一步加强对景区厕所、停车场的建设和管理,整治和引导不文明旅游的各种顽疾和陋习,有效提升景区的服务水平。‎ D.《四川省农村扶贫开发条例》是首次四川针对贫困人群制定的地方性法规,将精准扶贫确定为重要原则,从最贫困村户入手,让老乡过上好日子。‎ 答案 B 解析 A项,句式杂糅,“是设在杜甫草堂古色古香的仰止堂举行的”应为“是在……举行的”或“是设在……(地方)”;C项,搭配不当,可删掉“和引导”;D项,语序不当,“首次”与“四川”位置对调。‎ ‎[2年全国模拟重组]‎ ‎1.[2017·深圳模拟]下列各句中,没有语病的一句是(  )‎ A.随着生活水平的日益提高,中国社会正在由“吃饱”向“吃健康”转变,在这一进程中,能否保证公众的食品安全,让老百姓吃得放心,很大程度上取决于政府的执政水平。‎ B.汽车限购令早已有许多传闻,非但没有为城市治堵,倒是激发了许多购车族的欲望,提前加入有车族,使城市道路更加拥堵不堪。‎ C.新鲜蔬菜口感好,营养丰富,但是它的表面常常黏附着对人体有害的细菌和农药,所以食用新鲜蔬菜应该洗净较为安全。‎ D.针对今年春运期间旅客流量大、交通繁忙的情况,首都机场将加强航站楼前交通管理,加大楼前违章停放车辆的拖移和处罚力度。‎ 答案 A 解析 对于此类考查句子语病的题目,在分析思考时,要注意句子成分在搭配(不当)、句式(杂糅)等方面的错误,要仔细琢磨,推敲。B项,中途易辙,“提前”前加“他们”。C项,“所以食用新鲜蔬菜应该洗净较为安全”句式杂糅,糅合了“食用新鲜蔬菜应该洗净”“洗净较为安全”两种句式。D项,“加大”和“拖移”搭配不当。‎ ‎2.[2017·南京调研]下列各句中,没有语病的一项是(  )‎ A.最新全球超级计算机500强榜单公布,我国使用自主芯片制造的“神威·太湖之光”‎ 不仅登上了榜首,而且“超算”上榜总数也首次名列第一。‎ B.瑞士就给全体国民普发奖励工资的提案进行全民公投,76.9%的国民投了反对票,这一结果引起了各国人民的思考。‎ C.一味使用别人的话语体系解释自己,结果往往自我曲解,能否说好故事,让别人理解自己,关键在于找到适合的话语体系。‎ D.承载着无数人童年记忆的蓝精灵再度成为热点,影片《失落的村庄》将以全新的动画形式,讲述可爱的蓝精灵与邪恶的格格巫斗智斗勇。‎ 答案 B 解析 A项,语序不当,“‘超算’上榜”改为“上榜‘超算’”;C项,两面对一面;D项,成分残缺,“讲述”后面缺宾语中心语。‎ ‎3.[2017·中原名校质量考评]下列句子中,没有语病的一项是(  )‎ A.国内规格最高的首届中华设计大赛的开启,旨在会聚全国的优秀设计人才,确立与时代追求相一致的一种新设计理念。‎ B.为G20峰会准备的文艺晚会呈现出融中国古典艺术与西方现代主义相结合的特色,节目精彩纷呈,各国对此赞不绝口。‎ C.家长只有严于律己,树立正面的形象,为孩子营造健康向上的家庭环境,才能避免孩子今后不会走上违法犯罪的道路。‎ D.国家推行新型城镇化,强调内在质量的全面提升,也就是要推动城镇化由偏重质量规模增加注重质量内涵提升转变。‎ 答案 A 解析 B项,句式杂糅,“融……为一体”与“……相结合”两种句式相杂糅;C项,不合逻辑,删去“不会”;D项,搭配不当,“由……向……”是固定搭配,将“增加”改为“向”。‎ ‎4.[2017·福建四校联考]下列各句中,没有语病的一项是(  )‎ A.汉服是从黄帝即位到公元17世纪中叶,在汉族的主要居住区,以华夏礼仪文化为中心,通过自然演化而形成的独特的汉民族风貌特色。‎ B.近日,“魏则西事件”广泛受到网民关注,国家网信办会同国家工商总局、国家卫生计生委成立联合调查组进驻百度公司展开调查。‎ C.各级纪委要全面履行党章赋予的职责,把维护党章和其他党内法规作为首要任务,加强对遵守党章、党纪情况的监督检查。‎ D.法国巴黎暴恐事件和多名犯罪嫌疑人集体涉案事件的发生,中东难民群体在欧洲社会的形象遭遇重大危机。‎ 答案 C 解析 A项,搭配不当,“汉服是……风貌特色”主宾不搭配,应将“通过自然演化而形成的独特的汉民族风貌特色”改为“通过自然演化而形成的具有独特的汉民族风貌特色的传统服装体系”;B项,语序不当,“广泛受到网民关注”,应为“受到网民广泛关注”;D项,中途易辙,应在“中东难民群体”前加“让”字。‎ ‎5.[2017·甘肃重点中学调研]下列各句中,没有语病的一项是(  )‎ A.据悉,本次G20峰会的主题是“构建创新、活力、联动、包容的世界经济”。在业界看来,该峰会成果值得期待,中国首次举办。‎ B.市场期待的7月份“吃饭”行情没有兑现,股市在7月依然没能顺利突破3100大关,进行大幅度调整。近日沪指虽然重新站上3100点,但大盘或将仍维持震荡格局。‎ C.中国女科幻作家郝景芳凭借其《北京折叠》,获得第74届雨果奖。这是继2015年刘慈欣凭《三体》获奖之后,中国科幻作家再次获得这一殊荣。‎ D.《任意依恋》是韩国KBS电视台于2016年7月6日首播的水木剧,该剧讲述了充满正义感的纪录片编导鲁乙和顶级韩流明星申晙暎再度相遇后展开的爱情。‎ 答案 C 解析 A项,逻辑不当,因是中国首次举办,该峰会成果值得期待;B项,关联词语使用不当,应将“虽然”放在“近日”前;D项,缺宾语,“爱情”后加“故事”。‎ ‎6.[2017·惠州调研]下列各句中,没有语病的一句是(  )‎ A.教科书分为必修和选修两类。必修教科书的内容分“阅读鉴赏”“表达交流”“梳理探究”“名著导读”四个部分组成。‎ B.三星注重通过产品的研发提升智能家居的体验,而国内企业长虹则通过构建服务整体的服务系统来加大智能化水平。在1月初的CES展上,长虹向外界展示了在智能市场上取得的突破。‎ C.夏天要防备虫害的叮咬,需要注意如下几点:尽量穿浅色的衣服;勤洗澡,洗澡可消减人体分泌的汗味;在衣领、袖口等处喷洒花露水。‎ D.在漫长的历史进程中,中国人民和英国人民都创造了灿烂的文明,对人类文明进步产生了深远影响。金融城既是中英深度合作的一个缩影,也是全球开放的一个生动诠释。‎ 答案 D 解析 A项,“分……四个部分组成”句式杂糅;B项,“加大……水平”搭配不当;C项,颠倒因果,不合逻辑,“虫害”是结果,“叮咬”是过程、行为。‎ ‎7.[2017·牡丹江摸底]下列各项中,没有语病的一项是(  )‎ A.据最新数据显示,2016年第二季度,日本经济折算成年率萎缩6.8%,是该国经济自2011年“3·11”大地震受到重挫后遭遇的最大降幅。‎ B.中国国家和发展改革委员会对高通发起了反垄断调查,目前双方正在进行谈判,这项调查正值发改委加大对科技公司的审查力度,特别是授权移动设备和网络专利技术的公司。‎ C.一段时间以来,汉字书写大赛、非遗保护等文化现象引人注目,传统文化的重要性已越来越为国人所认知。‎ D.香港电影和台湾电影,都曾有过各自的“黄金时段”,而后又因为先天的限制而陷入低谷。香港电影人在“北上”的路途上走得更远,也起步得更早。‎ 答案 C 解析 A项,句式杂糅,删除“据”或者“显示”。B项,成分残缺,应在“审查力度”后加“的特殊时刻”。D项,语序不当,“起步得更早”和“走得更远”要调换位置。‎ ‎8.[2017·大冶一中调研]下列各句中,没有语病的一句是(  )‎ A.市环保局通过实施建设工程施工现场扬尘污染综合整治,市区二环以内已基本杜绝了现场搅拌混凝土的违规现象,大气环境质量逐年提高。‎ B.毗卢寺始建于唐天宝年间,由于历史原因,使得曾经名震一方的它现仅存释迦殿、毗卢殿等寥寥几栋建筑,其原貌究竟如何也成了一个谜。‎ C.美联社报道说,2500多名专家历时6年撰写的这份获得了一致通过的气候变化评估报告,对全球升温后果作了迄今为止最全面的科学阐述。‎ D.从最初态度强硬到“真诚道歉”,“不善忏悔”的苹果公司为何又主动向中国“展示顺从”?各媒体都将这罕见的一幕归结为中国有庞大的市场。‎ 答案 C 解析 A项,中途易辙。B项,滥用介词,“由于”使句子缺少主语。D项,成分残缺,应改为“将出现这罕见一幕的原因归结为”。‎ ‎9.[2017·长春质监]下列各句中,没有语病的一句是(  )‎ A.说到人才培养,人们往往想到要学好各门课程的基础理论,而对与这些理论密切相关的逻辑思维训练却常常被忽视。‎ B.一种观念只有被人们普遍接受、理解和掌握并转化为整个社会的群体意识,才能成为人们自觉遵守和奉行的准则。‎ C.音乐剧是19世纪末诞生的,它具有极富时代感的艺术形式和强烈的娱乐性,这使它成为很多国家的观众都喜欢的表演艺术。‎ D.福建土楼具有防匪防盗、防震防潮、冬暖夏凉、生活方便,虽经百年风雨和战争硝烟,至今仍巍然屹立,享有“东方古城堡”之美誉。‎ 答案 C 解析 A项,赘余,应删掉“对”字;B项,语序不当,“接受”和“理解”更换位置;D项,成分残缺,缺少了与“具有”搭配的宾语中心词,“方便”后加“的优点”。‎ ‎10.[2017·宜宾适应性测试]下列各句中,没有语病的一句是(  )‎ A.22日上午8时,运载火箭与搭载载荷组合体的活动发射平台驶出发射场垂直总装测试厂房,平稳行驶约3小时后,安全转运至发射塔架。‎ B.我国自主研制的“神威·太湖之光”登上超级计算机世界500强榜首,令人欣喜。‎ C.第三届丝绸之路国际电影节将于9月19日至23日在西安举办,电影节以“发展中的电影,多样性的文化”为主题而展开。‎ D.这部由歌手谭晶、孙楠,冬奥会首枚奖牌获得者叶乔波,冬奥冠军韩晓鹏、李坚柔出演的音乐电视,以冰雪运动为线索,通过纪实的手法,传递积极向上的奥林匹克精神。‎ 答案 D 解析 A项,成分残缺,在“运载火箭与搭载载荷组合体的活动发射平台”前面加上“承载着”一词。B项,语序不当,改为“登上世界超级计算机500强榜首”。C项,赘余,去掉末尾的“而展开”。‎ ‎11.[2017·四川针对性测试]下列各句中,没有语病的一句是(  )‎ A.民俗学者表示,如果仅仅把过中秋节看成是3天假期,就是吃月饼,那就误解了国家将其设立为法定节假日,中秋节还有很多美好的习俗。‎ B.对长城保护维修,国家文物局将遵守不改变文物原状和最小干预原则,妥善保护长城真实性、完整性和沧桑古朴的历史风貌。‎ C.更多的中国人如果能够多读书,能够让自己的心平静下来,能在这里慢慢找到幸福感并且找到自己的话,中国就会变得更好。‎ D.中国古代诗歌以高度凝练的语言,表达着人们的喜怒哀乐,影响着人们的精神世界。吟诵它,能陶冶情操,提升知识,给人美的享受。‎ 答案 D 解析 A项,成分残缺,缺少宾语,可在“法定节假日”后加上“的初衷”;B项,成分残缺,可在“真实性”前加上“的”;C项,语序不当,把“如果”调到“更多的中国人”前。‎ ‎12.[2017·山西五校联考]下列各句中,没有语病的一句是(  )‎ A.中共中央总书记、国家主席、中央军委主席习近平8月25日下午在北京人民大会堂会见全体第31届奥林匹克运动会中国体育代表团成员,欢迎我国体育健儿凯旋。‎ B.鉴于朝鲜半岛局势的最新变化,中日韩三国就此进行了认真的讨论。王毅外长表明了中方原则立场,强调了全面遵守落实联合国安理会第2270号决议的重要性。‎ C.目前,意大利6级以上破坏性地震造成的遇难者人数已上升至249人,受伤人数超过380人,媒体解读其伤亡惨重的主因是房屋旧、震源浅造成的。‎ D.国家海洋环境预报中心提醒,9月适逢中秋和国庆长假前夕,沿海地区游客集中,请沿海相关单位做好旅游景区海洋灾害防御部署,保障沿海旅游安全有序。‎ 答案 B 解析 A项,语序不当,“全体”应调换到“成员”之前。C项,句式杂糅,改为“媒体解读其伤亡惨重的主因是房屋旧、震源浅”。D项,成分残缺,在“部署”后加“工作”。‎ ‎13.[2017·辽宁葫芦岛联考]下列各句中,没有语病的一句是(  )‎ A.根据全国妇联的调查显示,我国30%的家庭存在着不同程度的家庭暴力,家庭暴力突出表现为婚姻暴力中的“夫对妻”的暴力。‎ B.在俄政府接连出台救市举措的同时,国际油价出现小幅反弹,这在一定程度上推升了卢布,目前美元对卢布的汇率正逐步返回1比50的安全线。‎ C.如果我们的作家动辄“与市场接轨”,轻易认同“卖书比写书更重要”,怎么指望他们沉潜静思、面壁独处,写出具有独特思想和情怀的优秀作品?‎ D.环境工作组研究发现,购物收据含有毒化学物质双酚A,即使人们只是接触收据,双酚A也能经由皮肤进入人体,甚至可能致癌,严重扰乱人体激素分泌。‎ 答案 C 解析 A项,杂糅,删去“根据”或“显示”。B项,成分残缺,应在“推升了卢布”后补出宾语“汇率”。D项,语序不当,把“甚至可能致癌”移到“严重扰乱人体激素分泌”后面。‎ ‎14.[2017·海南七校联盟联考]下列各句中,没有语病的一句是(  )‎ A.军队改革是我军无法回避的一场大考,这场大考的及格线就在于是否成功实现军队组织形态的变革。‎ B.现在许多山村,由于青壮年人外出打工,不仅所剩下的是荒芜的土地,还有许多留守儿童与空巢老人。‎ C.气候变化巴黎大会开幕式上,出现了习近平的身影,响起了中国声音,为协议达成注入了中国动力。‎ D.任何人只要肯开动脑筋,在灵机一动的时候,把握住灵感,并且马上付诸实施,就有机会帮助到我们。‎ 答案 C 解析 A项,一面对两面。应把“这场大考的及格线”改为“这场大考能否及格”。B项,关联词语位置不当,应把“不仅”移到“所剩下的”之后。两个分句为同一主语,关联词语应在主语之后。D项,缺少主语,应在“就有机会帮助到我们”之前加上“灵感”二字。‎ ‎15.[2016·深圳二调]下列各句中,没有语病的一句是(  )‎ A.在实施全面二孩政策之后,新生人口必然会增长,这一方面将会改变现有的家庭结构,另一方面也会减轻适龄劳动人口赡养老年人的负担和职责。‎ B.在物理学上,引力波是一种爱因斯坦在广义相对论里预言的以光速传播的时空波动,如同石头丢进水里产生的波纹一样,引力波被视为宇宙中的“时空涟漪”。‎ C.《中华人民共和国慈善法》的颁布实施标志着中国依法扶危济困全新“善时代”的到来,将推动我国慈善事业健康发展。‎ D.“阿尔法围棋”在李世石走出那极具创造力的一招之后连续下了两步臭棋,那是连业余围棋手都不会下出来的败招,无论你用多么发散的思维思考都不能不认为那两步棋有任何价值。‎ 答案 C 解析 A项,搭配不当。“赡养老年人的负担和职责”与前面的“减轻”不能同时进行恰当搭配,与“负担”可以搭配,但与“职责”搭配则不恰当。B项,语序不当。“一种”调到“预言的”后边。D项,否定不当,“不能不认为”为双重否定,与语意矛盾。‎ ‎16.[2016·山西八校适应性考试]下列各句中,没有语病的一句是(  )‎ A.随着中国制造业成本逐年上涨以及出口疲软的状况,人们希望对劳动密集型服务业需求的增加,有助于吸收工厂剩余工人。‎ B.近年来,一些别有用心的人以所谓“历史研究”为幌子,为了造成民众思想上的混乱,发起否定革命,通过颠覆中国历史、人民英雄以达成目的。‎ C.绿色发展意味着更加环境友好型的生产与消费,但中国正在通过几方面的努力去改善环境问题,包括扩大可替代能源市场规模、减少供给侧污染等方面。‎ D.因沙特处决什叶派宗教人士尼米尔所引发的中东外交冲突仍在持续。这次风波虽以教派冲突的面目示人,但背后却有着沙特国内挑战和海湾安全格局等多重因素。‎ 答案 D 解析 A项,表意不明,“希望”一词应删去。B项,语序不当,去掉“为了”,将“造成民众思想上的混乱”后移,替换掉“达成目的”。C项,连接词“但”运用不当,删除;成分赘余,删除最末两字“方面”。‎ 考点四 扩展语句 压缩语段 考点名片 考点内容 扩展语句:主要考查想象、联想和语言表达能力,要求想象合理、语言生动、善于运用恰当的修辞手法。‎ 压缩语段:语段内容多取自日常生活,涉及国内外重大事件、热门话题,个别试题突出地域特色。‎ 考查形式 扩展语句一般为主观表达(简答)题。考查形式有:‎ ‎①组词成句型;②句子扩展型;③段落扩充型(以段落扩充型为主)。‎ 压缩语段较扩展语句考查力度更大一些。一般为主观表达(简答)题。考查形式主要有:‎ ‎①拟题目;②拟一句话新闻;③下定义;④概括主要内容。‎ 趋势分析 ‎2018年扩展语句考查几率不大,压缩语段可能会与语言表达简明、准确、鲜明等考点综合考查。‎ 题组1 扩展语句 ‎1.请把“生命、季节、年轮”三个词语扩展成一段话。要求:至少使用一种修辞手法;不少于80字。‎ 答:____________________________________________________ ________________________________________________________________________________________________________________________‎ 答案 我们的生长轨迹,如树之年轮。随着季节的一次轮回,我们的生命之树,就会多出一道“年轮”。这些“年轮”或细细的、窄窄的,如软细的发丝;或粗粗的、宽宽的,如盘曲的标尺。而我们,则在这些不一样的“年轮”里,渐渐成熟起来。‎ 解析 本题考查扩展语句的能力。本题的关键是把这三个词联系起来,然后扩展成一段话,表达一个完整的意思。尤其是词与词之间要有一定的关联,并且一定要注意修辞手法的恰当使用。‎ ‎2.请以“山水”和“文章”两个词为中心扩写一段话。要求语意完整,合乎情理,不超过70字。‎ 答:____________________________________________________ ________________________________________________________________________________________________________________________‎ 答案 古人说:山水是地上之文章,文章乃案头之山水。山水可以借文章以灵气,文章亦能增山水之声名。好山水与好文章一旦相遇,便注定双双走向不朽与永恒。‎ 解析 本题考查扩展语句的能力。扩写时,宜展开充分的联想和想象,寻找“山水”和“文章”的相互关系,字数要控制好。‎ ‎3.根据要求,分别写一段话,每段话都要包括下面三个词语(词语顺序可以不分先后),30字左右。要求思想健康,语意连贯。‎ 想象 雾 倾听 ‎(1)表现“欢快”:__________________________________ ____________________________________________________________________________________________________________________________________________________________________________________‎ ‎(2)表现“忧愁”:________________________________ ____________________________________________________________________________________________________________________________________________________________________________________‎ 答案 (1)林中雾气弥漫开来,朦胧一片,但我们依然能倾听溪泉,品嗅百草,想象不远处瀑布的壮美……‎ ‎(2)倾听着泉水幽幽的哀鸣,看着如愁绪般久久不散的雾气,你说,我们该如何想象它曾经的欢乐年华?‎ 解析 题中提供了两种情景,扩展时要根据规定情景合理展开联想和想象,使它们变得生动形象,丰富饱满。需要注意的是,所写情景要符合要求,且要分别融入题目提供的三个词。‎ ‎4.请从诸葛亮、文天祥或谭嗣同中任选一人,续写下面的话。‎ 要求:①紧扣首句观点,符合所选人物境遇;②至少运用一种修辞手法,引用与人物有关的一句诗;③语意连贯,内容充实;④60~100字。‎ 忠诚是一种信念、一种品质、一种追求。________________________________________________________________________________________________________________________‎ 答案 (示例一)诸葛亮为匡扶汉室,剿灭汉贼,鞠躬尽瘁,死而后已。其功绩可追伊尹周公,其执着可比屈子贾生,其人格可与日月争辉。“三顾频烦天下计,两朝开济老臣心”便是其忠诚的见证。‎ ‎(示例二)文天祥面对风雨飘摇的南宋王朝,以扶大厦之将倾的勇气起兵勤王,以“人生自古谁无死,留取丹心照汗青”回应敌人的诱降,其凛然正气如擎天利剑,令敌人胆寒。‎ ‎(示例三)谭嗣同面对清政府的屠刀,高唱着“我自横刀向天笑,去留肝胆两昆仑”的豪言,勇敢地用死亡表达了自己对变法事业的忠诚。有人说他是傻子,然而,如果中国没有这样的许许多多的傻子,又何来国民的觉醒?‎ 解析 本题考查扩展语段的能力。考生解答时要审清题干要求,明确对象,根据给出的起句,选用恰当的表达方式去扩写。‎ ‎5.以“北雁南飞”为开头,按要求各写一段话,每一段话均不少于50个字。‎ ‎(1)运用描述性语言和比喻的修辞手法。‎ 北雁南飞,______________________________________________ ____________________________________________________________‎ ‎(2)运用议论性语言并蕴含一定的哲理。‎ 北雁南飞,______________________________________________ ____________________________________________________________‎ 答案 (1)(示例)(北雁南飞,)它们或呈“一”字形排开,或组成“人”字形,它们振翅翱翔于秋日碧蓝的天空中,仿佛一首隽永美丽的小诗,又像一幅清新淡雅的国画。‎ ‎(2)(示例)(北雁南飞,)是北方的寒冷在驱赶,还是南方的温暖在呼唤?我们不知道。但至少有一点可以确定:它们始终朝着梦想、朝着希望的方向展翅飞翔!‎ 解析 本题考查考生扩展语句的能力。由于两个小题都要求以“北雁南飞”为开头,所以考生可以先在脑海中对“北雁”这个意象进行简单的想象,然后再看其他要求。第(1)小题要求考生写一段描述性文字,而且所写的句子还要运用比喻的修辞手法。第(2)小题要求考生写一段议论性文字,且富有哲理。另外,考生要注意每一段文字都要符合“不少于50字”的要求。‎ ‎6.请以下段文字的首句为论点,按照要求补写一个事例。‎ 要求:①请从达尔文、巴金、苏武中任选一人续写;②紧扣首句观点,符合所选人物事迹;③运用反问的修辞手法,语意连贯,内容充实;④60~100字。‎ 有坚持才会有奇迹。______________________________ ____________________________________________________________‎ 答案 (示例一)如果当初达尔文子承父业,那他后来就是一名普通的医生;如果当初达尔文靠近神学,那他后来就是一名普通的牧师。幸好他坚持了自己的理想,坚持环球旅行进行实地考察和收集,不然又怎会有轰动全球的《物种起源》?‎ ‎(示例二)巴金用自己的文字构建了一部异彩纷呈的人类心灵史,那里面有对美好理想不屈不挠的坚持,对人生真善美的坚持,以及对美好未来的坚持。没有这些坚持,又怎能使他的作品冲破时间和空间的限制植根于广大读者的心底呢?‎ ‎(示例三)苏武手持一节旄节游走在茫茫的草原,与羊群为伴,不恋富贵不怕艰苦,绝不背叛自己的国家,用19年的孤独来坚持自己的承诺,最后终于回到了长安。如果没有这份坚持,他又怎能一步步靠近梦中的故国?‎ 解析 三个人物中,教材中有《苏武传》,也选录了巴金、达尔文的作品。答题时可以选择自己最为熟悉的一人,确保符合人物特点,并紧扣“坚持”进行叙述描写,运用反问的修辞手法,以增强说服力。‎ ‎7.按照以下思路概括说明:“中国的统一大业一定要实现。”‎ ‎①台湾的历史地位。②多数国家的表态。③两岸人民的心愿。④中国的决心、能力。(文字应简明、准确,字数不超过120个)‎ 答:____________________________________________________ ________________________________________________________________________________________________________________________________________________________________,中国的统一大业一定要实现。‎ 答案 台湾自古以来就是中国的领土,世界上绝大部分国家都承认中华人民共和国是中国的唯一合法政府,台湾是中国领土不可分割的一部分,和平统一是海峡两岸人民的共同心愿,中国政府有决心、有能力维护祖国的主权和领土完整,绝不容许台湾从祖国分裂出去,(中国的统一大业一定要实现。)‎ 解析 认真审题可知,题目要求扩展的是“中国的统一大业一定要实现”的原因,且提供了四个思考角度,学生依据这一思路,调动贮存在脑中的时事、地理、历史知识,就不难做出答案了。‎ ‎8.某出版社准备出版一册《陶诗画意》,请你为陶渊明《杂诗十二首(其二)》中的“风来入房户,夜中枕席冷”构思画面内容。‎ 要求:想象合理,语言生动,不超过50个字。‎ 答:____________________________________________________ ________________________________________________________________________________________________________________________‎ 答案 纱帐被风吹起,一片叶子在地上翻转,瘦削的诗人披上衣裳坐了起来,一只小猫依偎在诗人身边不愿离开。‎ 解析 本题考查考生扩展语句的能力。题干中所给的诗句既是扩展的提示句,又是扩展的中心句。考生在答题时要展开丰富的想象,不要拘泥于诗句本身,而要把诗句的意境营造出来。‎ ‎9.将下列语句扩展成借事言理的200字左右的语段。‎ 一串葡萄,有人挑好的先吃,有人则把好的留在后面吃,两种吃法反映了两种不同的人生。‎ 答:____________________________________________________ ________________________________________________________________________________________________________________________‎ 答案 一串葡萄,挑好的先吃,还是把好的留在后面吃,似乎都有道理,但细加推敲,却大有讲究。两种吃法反映了两种不同的人生。‎ 前一种人,先挑好的吃,抱着今朝有酒今朝醉的心态,殊不知自己的境遇却在每况愈下,末了,只能在回忆中叹息过去的人生。而后一种人呢,他们却先挑坏的吃,不断燃起新的希望,他们确信:生活将一天比一天更美好;唯有这种人,才真正懂得人生的真谛。‎ 要正视眼前的小小困难,勇敢地去追求更为美好的未来。这就是吃葡萄给人的启示。‎ 解析 材料提供给我们吃葡萄的两种方式,意味深长,饱含哲理,反映了两种截然不同的人生态度,学生只要从这两种现象中挖掘下去,即可把握其人生哲理,表达时对这两种人生感悟略作议论,就能扩展开来,组织出答案。‎ ‎10.请对下面一组画面加以想象创造,进行解说,使之成为恰当的画外音。注意:解说时,注意主体和陪衬物、背景的关系,注意顺序,要给人很强的可视性。‎ 画面:①FLASH动画——蒲公英老了;②小蒲公英问妈妈,有什么遗产;③妈妈默默地给每一个孩子带上一把远飞的小伞;④孩子高兴地在大地的怀抱里戏耍。‎ 答:____________________________________________________ ________________________________________________________________________________________________________________________‎ 答案 深秋,秋风瑟瑟。田野中一株蒲公英,叶子枯萎,枝上一簇簇的白白的“花”,在风中摇晃。渐渐露出的小小的蒲公英籽,在风中摇晃,仿佛在问妈妈什么。妈妈默默地给他们配上白色的小伞。小蒲公英籽一颗一颗离开妈妈,随风飘去,宛如一群小伞兵,飘呀飘,飘在湛蓝的天幕上。蒲公英在风中摇晃,用它那干枯的手向远飞的孩子们挥手告别。‎ 解析 本题主要考查扩展语句。做题时,要以FLASH动画的画面内容为依托,展开合理的联想和想象,运用描写、抒情等表达方式,并恰当运用拟人、比喻等修辞手法扩写。同时,应特别注意题目的提示与要求。‎ 题组2 压缩语段 ‎11.将下面这段话压缩成一条一句话新闻。(不超过15个字)‎ 前天,中央决定废止劳教制度。这个决定对北京劳教场所、机构带来了哪些变化呢?记者昨天先后到本市多处劳教场所进行探访,发现本市有两处劳教场所已更换成了看守所监狱的牌子。随着牌子的更换,劳教所的职能也转为收押因触犯法律而被判处拘役、有期徒刑等短刑犯的场所。但是,关于劳教机构工作人员的人事安排及北京劳教系统转型问题,目前相关部门仍未予以回应。‎ 答:____________________________________________________ ________________。‎ 答案 北京劳教场所改挂看守所监狱牌(或:北京劳教场所改为看守所监狱)‎ 解析 把所供材料压缩成一句话新闻,应当抓住所供材料的主要信息,即对象和中心事件。就对象而言,应当是“北京劳教场所”;就中心事件而言,应当是“改挂看守所监狱牌”。‎ ‎12.阅读下面的新闻,提炼主要内容,不超过30字。‎ 西班牙药物管理部门已经允许ZMapp作为特例进口,并将对一名感染埃博拉病毒的西班牙公民进行治疗。ZMapp由美国马普生物公司研制,仍处于初期试验阶段,先前仅在猴子身上做过试验。两名先前在利比里亚感染病毒的美国医务人员曾接受ZMapp治疗,随后病情出现好转。但医疗卫生部门说,眼下尚无法断定两人情况好转是否与使用这种药物有关。是否冒险大规模使用这种药物应对当前严重的埃博拉疫情,在医学界引发一场伦理争论。一些专家认为应将药物提供给非洲国家使用;另一些专家则不同意匆忙扩大药物使用范围,理由是尚未确认这种药有效并且安全。‎ 答:____________________________________________________ ________________________________________________________________________________________________________________________‎ 答案 西班牙决定使用抗埃博拉新药,引发医学界伦理争论。‎ 解析 本题考查压缩语段的能力。解答该题一定要注意两个方面的信息,一个是药物的使用,另一个是对药物使用的态度。与此同时,还要注意关键动作的发出者(施动者)与受动者之间的关系等。‎ ‎13.用对联的形式给下面这则新闻拟写一个标题,不超过20字。‎ ‎11月21日,“汉语盘点2015”启动仪式在央视网演播室举行。“年度字词”评选活动,旨在“用一个字、一个词描述2015年的中国和世界”。除了公布年度字词外,主办方还将权威发布十大“年度流行词语”“年度新词语”“年度网络用语”,力图全面真实地展现中国语言生活现状,描画国人语言生活年度风景线。‎ ‎“年度字词”评选活动分为网友推荐、专家评点和网络投票三个阶段。最终获选字、词将通过活动官网向大众公布,并于12月20日举办揭晓颁奖仪式。自2006年以来,“汉语盘点”已举办至第八届。它真实记录了中国视野中的世界万象与社会变迁,勾勒出万千世态和百姓民心,更以特殊的方式书写了汉语新词新语、流行词语的编年史,是国人语言生活的真实写照。‎ 答:____________________________________________________ ________________________________________________________________________________________________________________________‎ 答案 (示例一)启动“汉语盘点”,寻找年度字词 ‎(示例二)征集年度流行字词,描绘国人语言生态(能概括新闻内容;符合对联形式且表达得当即可)‎ 解析 本题综合考查压缩语段和正确运用常见的修辞方法(对偶)的能力。内容上,可从事件本身(“汉语盘点”启动)及其意义的角度概括;形式上,可考虑上下句均采用动宾结构的句式(新闻标题常采用的句式)。注意字数限制。‎ ‎14.请从下列材料中选取必要的信息,用一个单句为“网络水军”下定义。‎ ‎“网络水军”大都是隐藏身份的网民,大都“穿马甲”和雇用者交易,其活动难以掌控。版主把主帖发出去后,获得广大“网民”的注意,进而营造出一个话题事件,所有网络公关公司都必须雇用大批的分散在全国各地的人员来为客户发帖回帖造势。《人民日报》曾发表题为“谨防民意制造者利用网络推手误导舆论”的署名文章,批评“网络水军”制造“民意病毒”,指出“民意病毒”利用和裹挟健康民意,不仅伤害广大网民的感情,也损害网络民意表达的健康肌体。‎ 答:____________________________________________________ ‎ ‎________________________________________________________________________________________________________________________‎ 答案 “网络水军”是指受雇于网络公关公司来发帖回帖造势、活动难以掌控、会制造“民意病毒”的网络人员。‎ 解析 解答该类试题需掌握下定义的格式“……是……”,判断句的形式,且必须是单句,能揭示事物的本质特征。‎ ‎15.为下面一则报道拟一个恰当的标题。(不超过15字)‎ 在瑞士日内瓦国际会议中心,一名国际红十字会的成员展示了联合国即将更换的国际红十字标志。新的标志称为“红水晶”,在本次有192个成员国家参加的为期两天的会议上,“红水晶”将作为国际红十字运动的第三个标志被推广。‎ 答:____________________________________________________ ________________________________________________________________________________________________________________________‎ 答案 国际红十字将使用“红水晶”标志 解析 首先分析语段的主要内容,“联合国即将更换的国际红十字标志”“‘红水晶’将作为国际红十字运动的第三个标志被推广”。然后进行筛选概括,注意语言简洁。‎ ‎16.阅读下面的文字,概括其主要信息。(不超过40字)‎ 近日,大校军官刘声东因思念过世不久的母亲,夜不成眠,写成了祭文《妈妈》。此文一出,很短时间就被网民转载上千万次。他为母子深情能得到如此多的共鸣倍感欣慰。有人认为五六十岁的人可能会重孝心,那些“80后”“90后”“00后”慢慢就不懂孝了。经过这次不经意的“民意测评”,发现跟帖点赞的不少是年轻人,让人们对年轻一代的孩子充满信心。空间的距离、时间的相隔会让子女想起父母的次数少一些,但往往越是远离、久别越是想念。对传统文化的生命力不要怀疑,更不要从怀疑进入到冷漠,从而否定。‎ 答:____________________________________________________ ________________________________________________________________________________________________________________________‎ 答案 祭文《妈妈》引起共鸣;年轻一代不无孝心;传统文化具有生命力。‎ 解析 解答本题,先找出这段文字中的主要信息,“此文一出,很短时间就被网民转载上千万次”概括为:祭文《妈妈》引起共鸣。“经过这次不经意的‘民意测评’,发现跟帖点赞的不少是年轻人,让人们对年轻一代的孩子充满信心”概括为:年轻一代不无孝心。“对传统文化的生命力不要怀疑,更不要从怀疑进入到冷漠,从而否定”概括为:传统文化具有生命力。‎ ‎17.阅读下面一则报道,概括其主要信息。(不超过15个字)‎ 据京华时报报道,国务院总理李克强访问泰国期间,中泰两国签署了《关于深化铁路合作的谅解备忘录》。跨国铁路激发“泛亚铁路”联想,高铁所代表的“中国速度”,将为区域经济一体化注入新动力。铁路专家、同济大学教授孙章介绍,中国高铁在国内大发展的同时,也积极谋划“走出去”。目前中国已经与包括美国、巴西、白俄罗斯在内的多个国家建立高铁合作关系。他认为,中国高铁能在短短几年间驶出国门,主要源于三大优势。首先是性价比。一份研究报告显示,国外建设高铁每公里成本为0.5亿美元,而中国只有0.33亿美元;其次是技术;最后是安全性。‎ 答:____________________________________________________ ________________________________________________________________________________________________________________________‎ 答案 三大优势助中国高铁走出国门。‎ 解析 首先,找到语段的主要事件。语段中迷惑性语言很多。共两个事件,一是李克强总理签署备忘录,另一个是对中国高铁的介绍。如果是介绍李克强签署协议,那么,在后文应该是签署协议的具体情况,后文显然不是。后一事件是中国高铁走向世界,对其概括即可。‎ ‎18.下面是两段介绍环境考古学的材料,请概括其主要内容,根据所给的开头给“环境考古学”下定义,不超过50字。‎ 考古学研究表明,人类社会是一步步由低级到高级逐渐发展起来的。许多地区都有人类社会复杂的兴衰演化历史。地质学、古气候学和古环境学的研究也表明,古代的自然环境面貌也发生过复杂的演变。古代人类居住和活动区域内的自然环境如何?发生了什么样的变化?古代的自然环境同当时当地的人类活动发生了何种关系?这些问题的提出,成为环境考古学产生的思想渊源。‎ 环境考古学的研究内容主要包括两个方面:一是古代环境的复原,二是人地关系的分析研究。‎ 答:环境考古学________________________________________ ____________________________________________________________‎ 答案 (环境考古学)是在复原古代人类生存环境的基础上,探讨古代人类社会及其演化同自然环境之间相互关系的学科。‎ 解析 因为是下定义,所以首先要确定其基本句式,也就是“环境考古学是……的学科”。然后根据“环境考古学的研究内容主要包括两个方面:一是古代环境的复原,二是人地关系的分析研究”,可以确定其基本内容,一是“在……的基础上”,二是“探讨……关系”。‎ ‎19.阅读下面一段文字,提取四个四字短语,与短语“恐龙活动”一起概括恐龙足迹化石形成的过程。‎ 骨骼化石保存了恐龙生前身后一些支离破碎的信息,足迹化石保存的却是恐龙在日常生活中的精彩一瞬。恐龙在太干的地方活动不可能留下脚印,太湿的地方也无法留下脚印,只有在温度、黏度、颗粒度非常适中的地表行走时才会留下足迹。恐龙在地面留下足迹后,如果马上下雨,或者被水淹没,尚未干燥的足迹很快就会在水中消融。因此,足迹在潮湿的地面形成后还需要一定时间的干燥过程,待保存足迹的地表干燥硬化后,再被外来的沉积物所覆盖、掩埋,才能形成化石。‎ ‎①________ ②________ ③________ ④________‎ 答案 留下足迹 干燥硬化 覆盖掩埋 形成化石 解析 此题考查压缩语段的表达能力。解答此题,可先整体感知文段内容,然后根据恐龙足迹化石形成的过程,筛选出四个四字短语,提炼性地概括即可。“留下脚印”可概括为留下足迹;再根据后面的内容筛选出的四字关键词:“太湿的地方也无法留下脚印……干燥过程”可概括为干燥硬化;“沉积物所覆盖、掩埋”可概括为覆盖掩埋;“才能形成化石”概括为形成化石。‎ ‎20.阅读下面这段文字,按要求回答问题。‎ 改革开放以来,人们的幸福指数明显提高。要让人们不断增强幸福感,必须坚持发展,使财富更加丰饶充盈。人在社会中只有得到公正公平的对待,不受歧视,才会感到幸福。如果心理失衡,即使自己的经济收入比过去多,但看到在同样付出的情况下别人却拿得更多,也会感到不快乐。一个基本生活不愁的人,由于对钱财没有贪欲就会知足常乐;而一个腰缠万贯的人,将幸福完全物质化,在钱财上总觉得比上不足,内心就会长戚戚。‎ 请概括出“幸福指数”的三个要素。(每点不超过6个字)‎ 要素一:________________________________________________ ___________________;‎ 要素二:_______________________________________________ ______________________;‎ 要素三:_______________________________________________ ______________________。‎ 答案 经济富裕 社会公平 心态良好 解析 概括“幸福指数”的三个要素,首先把文段分为三个层次:第一层是“要让人们不断增强幸福感,必须坚持发展,使财富更加丰饶充盈”,里面的关键词是“财富”“充盈”;第二层是“人在社会中只有得到公正公平的对待,不受歧视,才会感到幸福”,里面的关键词是“社会”“公正”“公平”;余下为第三层,关键词为“心理失衡”,据此概括可得。‎ ‎ [3年高考真题集训]‎ ‎1.[2016·浙江高考]提取所给材料的主要信息,在横线处写出四个关键词。‎ 引力全称万有引力,指具有质量的物体之间加速靠近的趋势,简单说就是物体之间相互吸引的作用力。在爱因斯坦广义相对论的视野里,引力等价于弯曲的时空。而引力波就是在弯曲的时空这个大背景下,当发生有质量的物体加速运动导致的扰动时,由此产生的波动如波纹一样向外传播的现象。‎ 一个世纪前,爱因斯坦预测了引力波的存在,但近百年来,科学家们并未找到证明它存在的直接证据。华盛顿当地时间2016年2月11日,美国激光干涉引力波观测台(LIGO)实验组召开新闻发布会,宣布首次直接观测到了由两颗恒星级黑洞13亿年前并合产生的引力波。这是科学史上又一次具有划时代意义的发现。‎ 引力波的发现对普通人的生活会产生什么影响?科学家们表示,一个新的重大科学发现,总会给人类社会带来无法预估的发展。18世纪描述电磁波的麦克斯韦理论确认的时候,也没有人知道会给人类带来什么,但是现在不管是电视机还是移动电话,都与电磁现象有关。‎ ‎________ ________ ________ ________‎ 答案 引力波 首次(或“美国”) 发现 影响(或“意义”)‎ 解析 提取新闻的主要信息,应抓住中心事件及其结果。该则材料分3段,第1段说明引力波的概念,第2段介绍引力波的发现,第3段阐述发现引力波的影响。根据段意去提取信息,第1段提取关键词“引力波”,第2段提取关键词“首次”“发现”,第3段提取关键词“影响”。‎ ‎2.[2015·湖南高考]根据首尾语句,发挥想象,有创意地补写中间一段文字。100字左右。‎ 苍天有裂,补天迫在眉睫;苍生有苦,女娲忧心如焚!‎ 面对万民翘盼,__________________________________________ _____________________________________________________________________________________________________。‎ 只见,青冥浩荡,四海升平,万物回春,人们载歌载舞。‎ 答案 (示例)女娲端坐于云间,双手合十,凝神静气,聚天地万物之正气,凝结五彩之石以补苍天,顿时山呼地应,熊咆龙吟,正气升腾,逆气下沉,彩石凝结。正气凝结愈快,苍天裂痕愈大,最后关头,女娲用银河之心堵住裂口,天地澄澈。(所补写文字须联系上下文提供的情境并衔接自然)‎ 解析 补写时要联系上下文提供的情景,上文是女娲补天的原因,下文是女娲补天的结果,要补写的部分应该是女娲补天的过程。表述时,语言要连贯得体,与上下文风格一致,注意字数限制,想象合理。此外,要注意题干中的一个要求,即“有创意地补写”,不能平铺直叙,简单介绍。‎ ‎3.[2015·天津高考]汉字是中华文明的重要载体,许多汉字如山、日、水、火、土、木……都具有丰富的文化意蕴。请参照例句,任选一个字写一段话。要求100字以内。‎ 例:“月”字形如弯弯的月牙,“月”加“日”是“明媚”的“明”。“月”在中华文化中总是与清纯、静谧、乡情相联系,“朗月清风”让人神清气爽,“月是故乡明”则勾起人们对故乡和亲人的思念。‎ 答:____________________________________________________ ________________________________________________________________________________________________________________________‎ 答案 “火”字形如一堆燃烧的篝火,“火”加“人”是“伙伴”的“伙”。“火”在中华文化中总是跟燃烧、兴旺、热烈有着密切的关系。“红红火火”让人感觉到生活的美满幸福,“日出江花红胜火”描摹了春天到来时百花盛开、春光烂漫的景色。‎ 解析 回答本题时要注意把握示例的内容和特点。示例以汉字“月”为例,先从“月”的字形方面说明了其意义,然后介绍“月”与“日”组合后的字义,接着解释“月”在传统文化中的意蕴,并列举了一个词语、一句诗歌来解说。仿写时,注意这些内容,任选一个汉字,注意字数要求。‎ ‎4.[2015·山东高考]阅读下面一段文字,以“志愿服务”为开头,概括志愿服务的作用。要求:①要点全面;②不超过50字。‎ 志愿服务是一项以自愿且不图物质报酬为参与前提的社会事业,重在出力而不是出钱,致力于弥补政府服务和市场服务的不足。这项事业有上百年的历史,在世界各地广泛开展,是推动社会文明发展的一个重要方面。人的内心都有向真、向善、向美的一面,都期望实现个人的美好价值,志愿服务是把这种期望变为现实的一种途径。志愿者在从事服务的过程中,会获得精神上的满足。‎ 答:____________________________________________________ ‎ 答案 “志愿服务”能够弥补政府和市场服务的不足,推动社会文明发展,实现个人美好价值,使人获得精神满足。‎ 解析 此题主要考查压缩语段。解答此题,先要审清题目要求“概括志愿服务的作用”,再从文段中找出相关词句:“弥补政府服务和市场服务的不足”“推动社会文明发展”“实现个人的美好价值”“获得精神上的满足”。然后再考虑用“志愿服务”开头,加以整合即可。‎ ‎5.[2014·四川高考]请紧扣下面第一句话的观点,结合《论语》和《三国演义》的有关内容,运用排比修辞手法,继续写一段意思完整的话,不少于60字。‎ ‎“和”是中华文化精髓之一,不少名著都蕴含有尚和精神。________________________________________________________________________________________________________________________‎ 答案 (示例一)《论语》中既有“礼之用,和为贵”的行事准则,也有“君子和而不同,小人同而不和”的谆谆告诫,也有“和无寡,安无倾”的社会理想,这部儒家经典蕴含了仁爱和谐的尚和精神。‎ ‎(示例二)诸侯割据,群雄逐鹿,三国鼎立。生灵涂炭渴望休养生息,社会动荡渴望和平安宁,战乱频仍祈求国家大治,宏伟的三国长卷蕴含了人们对天下归一的追求。‎ 解析 本续写题不仅注重名著中所蕴含的文化内涵,还要能够正确理解中国人的核心价值观“和”,这就要求考生对名著有很深的认知度,并且具备相应的对名著思想内容的理解与感悟,还要能够通过灵活地运用语言表达自己的思想感悟。‎ ‎6.[2014·湖南高考]阅读下面的文字,完成题目。‎ 一次,妈妈忽见阿必一脸狼狈相,鼻子上抹着一道黑。问她怎么了,她装作若无其事,只说:“我囫囵着跌下来的。”“囫囵着跌下来”,用语是幼稚的创造,意思却很明显,就是整个人从楼上滚下来了。问她跌了多远,滚下多少级楼梯,她也说不清。她那时才两岁多,还不大会说,也许当时惊魂未定,自己也不知道滚了多远。‎ 她是个乖孩子,只两件事不乖:一是不肯洗脸,二是不肯睡觉。‎ 每当佣人端上热腾腾的洗脸水,她便觉不妙,先还慢悠悠地轻声说:“逃——逃——逃——”等妈妈拧了一把热毛巾,她两脚急促地逃跑,一迭连声喊:“逃逃逃逃逃!”总被妈妈一把捉住,她哭着洗了脸。‎ ‎(节选自杨绛《记杨必》)‎ 请为上文续写一段阿必“不肯睡觉”的文字。‎ 答:____________________________________________________ ________________________________________________________________________________________________________________________‎ 答案 (示例)我在家时专管阿必睡午觉。她表示要好,尽力做乖孩子。她乖乖地躺在摇篮里,乖乖地闭上眼,一动都不动,让我唱着催眠歌摇她睡。我把学校里学的催眠歌都唱遍了,以为她已入睡,停止了摇和唱。她睁开眼,笑嘻嘻地“点戏”说:“再唱《喜旦娄》(Sweet And Low,丁尼生诗中流行的《摇篮曲》)。”原来她一直在品评,选中了她最喜爱的歌。我火了,沉下脸说:“快点困!”(无锡话:“快睡!”)阿必觉得我太凶了,乖乖地又闭上了眼。我只好耐心再唱。她往往假装睡着,过好一会儿又睁眼。‎ 解析 本题考查文章续写。解答该题,要扣紧题干中的“续写”二字,围绕“不肯睡觉”的主线,写动作,写语言;要着力表现儿童的性格特点;语言风格要与原作基本一致。‎ ‎7.[2014·浙江高考]根据下面的情境,补写答话。字数不超过30个。‎ 师父训练徒弟爬树,徒弟爬到高处时,师父喊道:“小心,小心!”第二次,徒弟爬到高处时,师父一言不发,等徒弟下到低处时,他才说:“小心,小心!”‎ 徒弟问:“师父,为什么上次在高处时提醒我,这次下到低处才提醒我?”‎ 师父回答:“__________________________________________ ______________________________。”‎ 答案 (示例)没经验时,容易在难处出错;有经验了,往往在易处出错 解析 注意抓住文中的关键句,联系上下文语境。首先要理解,师父的回答是针对徒弟的提问“为什么上次在高处时提醒我,这次下到低处才提醒我”。其次是分析,问题暗含着两方面的内容,即第一次没经验时高处小心和第二次有经验时低处小心。最后是概括,概括时要结合上述两层内容,准确揭示表象背后的意义。考生可以同时结合自己的生活经验得出答案。‎ ‎8.[2014·辽宁高考]概括下面语段的主要信息,不超过45个字(含标点符号)。‎ 长征三号丙改二型火箭将在今年下半年发射,届时将搭载“嫦娥五号试验器”,执行探月工程三期再入返回验证试验的发射任务。为确保“嫦娥五号试验器”准确进入预定轨道,长征三号丙改二型火箭进行了多项技术攻关。整体上火箭“身体长高、翅膀变长”‎ ‎,芯级火箭增高1.5米,两个助推器加长0.8米。长征三号丙改二型火箭同时采用“天基测量”技术和最新的高码率传输技术,可以向地面技术人员传输火箭飞行过程的实时遥测数据,实现了“实时问诊”和“宽带传输”。‎ 答:____________________________________________________ ________________________________________________________________________________________________________________________‎ 答案 长征三号丙改二型火箭将搭载“嫦娥五号试验器”执行发射任务,它进行了多项技术攻关。‎ 解析 本题所给的材料是一则新闻,对新闻内容的压缩,可以先找出其叙述的主要对象,即“长征三号丙改二型火箭”。然后分析新闻材料,筛选出关于这一对象的主要事件及其发展情况,即长征三号丙改二型火箭将搭载“嫦娥五号试验器”执行发射任务,它进行了多项技术攻关。原材料是一则新闻,因此抓导语是关键。该语段的导语是第一句话,是关键信息。为确保压缩后的语段信息涵盖全面,在不超规定字数的前提下,可以在导语之后的文字中抽取部分关键信息。‎ ‎[2年全国模拟重组]‎ ‎1.[2017·石家庄二中检测]请将所给的四个词语扩展为一段文字,不少于40个字。‎ 家风 濡养 与众不同 好像 答:____________________________________________________ ________________________________________________________________________________________________________________________‎ 答案 家风在中国传统文化中源远流长,它濡养了一代又一代的中国人。每一个大大小小的家族的家风都是与众不同的。随着传统大家庭在现代社会的分解,家风好像离我们越来越遥远。我们应该继承历史留给我们的这一宝贵精神遗产,让家风这棵树越长越壮。‎ 解析 要明确题目要求,将这四个词连成不少于40个字的一段话。由“家风”可以联想家风是什么,“家风”“濡养”了什么人等内容。后面的两个词语也可以这样进行联想。‎ ‎2.[2017·常德三校联考]请将下列句子扩展为中心明确、语意连贯的一段话,要有文采,不少于50字。‎ 事业是人生不朽的丰碑。‎ 答:____________________________________________________ ________________________________________________________________________________________________________________________‎ 答案 人类的历史告诉我们,凡是留下英名的人,他们都创造了永存的事业。孔子、屈原、司马迁等,他们为千秋万代所传颂,因为他们为民族文化宝库增添了财富,建起了对人类社会有益的事业大厦,筑起了他们人生中永垂不朽的纪念碑。‎ 解析 扩展时要围绕所给中心句进行阐释,使内容更丰满、生动,形成一个完整的语段。一要扣中心句,二要有文采,最好是运用修辞手法。‎ ‎3.[2016·天津南开中学三模]使用下面的词语,至少运用一种修辞手法,写一段不少于60字且意思完整的话。‎ 节俭 幸福 行动 务实 中国梦 答:____________________________________________________ ________________________________________________________________________________________________________________________‎ 答案 节俭是一种美德,像摇曳的花朵,用无声的语言阐释民族复兴的幸福;务实是一种踏实的作风,似沉静的高山,用实际行动解读中国梦的内涵。‎ 解析 本题考查考生句意丰富式扩展语句的能力。题干中所给的五个词语,其中“务实”“行动”和“中国梦”三者之间既具有一定的逻辑性,又具有一定的引申性,可依据三者的逻辑关系组织语言;而“节俭”是与“浪费”相对的行为,它是中华民族的一种美德,可依据一定的逻辑关系将“节俭”与“幸福”有机联系起来,并和“中国梦”的有关内容构成意思完整的一段话。同时,还要注意题干中的修辞(至少运用一种修辞手法)及字数要求(不少于60字)。‎ ‎4.[2016·江西名校调研]请阅读下面的文字,然后围绕“微笑”进行扩展。要求:运用比喻的修辞手法,语言表达连贯,60字左右。‎ 夏天的一个中午。炎热,喧扰。车子还没有来。百般焦急、无聊之际,忽见一个小男孩朝我微笑,这微笑,__________________________________________________________________________。小男孩的微笑融化了我的烦躁,我的心渐趋平静。‎ 答案 (这微笑,)干净、清澈,就像一阵清风吹过了炎热喧扰的中午,吹过了我烦燥的内心,让周围的空气逐渐降温,让我如同沐浴在夏日早晨的清凉中。‎ 解析 本题考查扩展语句、正确运用常见的修辞手法、语言表达连贯等考点,做到比喻恰当,符合特定情境,上下文连贯即可。‎ ‎5.[2017·嘉峪关摸底]按要求把下面的句子扩写成一段话。‎ 要求:①语言要生动形象、丰富细腻;②使用比喻和排比的修辞手法;③不少于60字。‎ 一首好诗,总能刺激你的想象。‎ 答:____________________________________________________ ________________________________________________________________________________________________________________________‎ 答案 一首好诗,就像一朵浪花,叫你去想象大海的浩瀚;就像一钩弯月,叫你去想象夜空的宁静;就像一颗露珠,叫你去想象黎明的清新。总之,它能够最大限度地刺激你的想象。‎ 解析 扩展语句考查的是语言运用能力,需拓展思维空间,在原句的基础上增添表述内容,丰富语句内涵。本题将语句扩展与修辞手法的运用结合起来考查,在扩展时要按照题目的要求,语言要生动形象、丰富细腻,使用比喻和排比的修辞手法。‎ ‎6.[2017·衡水中学调研]吟诵古典诗歌,脑海中会呈现诗中所描绘的画面,即所谓“诗中有画”。请从下面描写荷花的两句诗中任选一句,发挥想象,用自己的语言把它描绘成一幅美丽的画卷。要求:紧扣诗句;必须原创;语言表达简明、生动;运用比喻或比拟的修辞手法;不超过50字。‎ ‎①嫩碧才平水,圆阴已蔽鱼。(唐·李群玉《新荷》)‎ ‎②十里荷花带月看,花和月色一般般。(宋·杨公远《月下看白莲》)‎ 答:____________________________________________________ ________________________________________________________________________________________________________________________‎ 答案 (示例一)夏日荷塘,娇嫩碧绿的荷叶像一把把绿伞刚好举出水面,阳光照进荷塘,荷阴映水,鱼儿在荷阴下悠闲地游来游去。‎ ‎(示例二)看那月光下的十里荷塘,绿波荡漾,一朵朵月色一般明净的荷花,宛如星星般在碧波中一闪一闪。‎ 解析 古诗句意蕴丰富,可以先把诗句的意境用优美的语言进行表述,然后再根据题干要求的修辞手法、字数限制等进行补充。第①句主要写夏日碧荷游鱼,第②句应紧扣夏夜月色荷花。‎ ‎7.[2016·杭州七校质检]下面是屠格涅夫《猎人笔记》中的一段话,其中部分内容被压缩成画线的句子,请根据语境将其重新扩展开来。(不少于30字)‎ 在早春的日子里,当四周一切都发出闪光而逐渐崩裂的时候,通过融解的雪和浓重的水汽,已经闻得出温暖的土地的气息。云雀歌唱,溪水奔流。‎ 答:____________________________________________________ ________________________________________________________________________________________________________________________‎ 答案 在雪融化的地方,在斜射的太阳光底下,云雀天真烂漫地歌唱着,溪水发出愉快的喧哗声,从一个溪谷奔流到另一个溪谷。‎ 解析 “部分内容被压缩成画线的句子”说明现在只剩下主干了。我们应展开合理的想象,在“云雀”“歌唱”和“奔流”前面加上修饰语。‎ ‎8.[2017·六安月考]根据下面这段文字,写一个能够概括全段意思的总标题和三个层次的板块标题,要求简明生动,每个标题不超过10个字。‎ 为使工作高效、生活方便舒适,人类越来越依赖机械和各种数字技术,却忽略可能为此而付出的代价,如生活在加拿大北部的新一代因纽特人越来越依赖GPS导航工具辨别方向,结果打猎时严重事故日渐增多。一项调查研究显示,让一个人全神贯注地盯着数据显示屏,保持专注的时间很难超过半小时。即使技艺再娴熟,如果长期只负责监测观察,一个操作员也难免技艺生疏,一旦发生故障或意外难以做出正确反应。为了不让自动化把人脑变得迟钝,一种简单的弥补方法就是逼着人多动手多动脑,别事事让电脑程序代劳,即使是机器,也要让管理者随时接手,确保由人脑操作电脑,不要完全放手。‎ 总标题:________‎ 板块标题:①________ ②________ ③________‎ 答案 (总标题)自动化正在使人类退化 ‎(板块标题)为“自动”付出代价(或“自动”就要付出代价) 付出代价的原因 多动手多动脑(别事事让电脑代劳)‎ ‎9.[2016·新余七校联考]阅读下面材料,拟写一条一句话新闻,不超过20个字。‎ 记者从10月25日曲阜市举办的第二届“百姓儒学节”开幕式上获悉,世界文化遗产“三孔”景区将向全国教师免费开放。这是继孔子后裔、曲阜籍市民免费游“三孔”,驻曲高校、曲阜市中小学生特殊时段集体参观免费,游客“背《论语》免费游三孔”活动之后,曲阜再次向游客推出的优惠举措。‎ 第二届“百姓儒学节”开幕式上,曲阜市委书记李长胜说,“兴市须重教,孔子作为‘万世师表’,理应得到全社会教师群体的尊崇。‘三孔’景区面向全国教师免费开放,将为优秀传统文化的传承、弘扬和创新,发挥儒学时代作用搭建新的载体。”‎ 据了解,为做好“三孔”景区面向教师免费开放的前期准备工作,曲阜市文物、旅游等相关部门进行了专题座谈和调研,并就配套方案进行了专题研究和论证,具体的实施方案正在制定之中。据悉,该方案将于2016年1月1日起施行。‎ 答:____________________________________________________ ________________________________________________________________________________________________________________________‎ 答案 2016年起全国教师免费游“三孔”。‎ 解析 压缩语段,要仔细分析语段的层次,分几层,层意是什么。概括层意,抓住要点。本文一共分3段,第1段属于导语段,我们要抓导语中的关键词:时间、地点、人物、事件,重点是人物和事件。根据题干要求结合新闻材料、语境,适当删减替换补充。在导语中,抓住“全国教师”“免费”“游三孔”这几个词,免费的时间体现在第3段“该方案将于2016年1月1日起施行”中。因为有字数限制,所以要高度概括。回答过程中思路要清晰,信息要全面,不能遗漏要点。‎ ‎10.[2017·河北名校联盟联考]用一句话概括下面一段文字反映的主要现象及其原因。(不超过30字)‎ 美国耶鲁大学,在过去的25年里,历史一直是头号热门专业。史景迁的中国史课年年几百人上,已经成了传奇。英语专业在90年代前半期就是第二热门的专业,后来才被经济学超出。耶鲁的经济学,是高度理论化的人文学科,不是实用学科。在哈佛大学,前四大热门专业,第一是社会科学,第二是生物,第三是英语,第四是心理学。普林斯顿传统的五大专业是政治学、经济学、历史、英语和国际关系,46%的学生集中在这五大专业里。这些美国的精英大学培养的是未来的领袖,要把握大方向,其关怀和训练当然必须宏观,不能一天到晚坐在那里学数钱算账。这些精英学校,大部分学生出身于中高产阶层,父母受教育程度很高,对大学有充分的理解:大学主要是一种精神经历,上大学的最基本目标,是培养兴趣、发现自己、认识世界、反省人类最基本的价值;度过了这样的四年人生,再想实际问题也不迟。‎ 答:____________________________________________________ ________________________________________________________________________________________________________________________‎ 答案 发展方向与父母理解(或培养意图与家庭意愿)使美国精英大学人文学科成热门。‎ 解析 先划分层次,再勾画出每句话的中心词,组合成一个因果复句即可。文中前面部分谈到的是美国一些大学的热门专业,后面是产生这种现象的原因。共涉及“美国精英大学”“人文学科”“热门专业”“发展方向”“父母理解”五个要点,意思对即可。‎ ‎11.[2017·西安中学模拟]请筛选并整合下面一段文字的主要信息,为“潜规则”‎ 拟写定义。要求语言简明,条理清楚,不超过50字。‎ 潜规则是看不见的,没有明文规定的,心照不宣、暗箱操作的规则,是潜藏在现实生活中的阴暗面,是见不得阳光的东西。潜规则已经约定俗成,是许多人普遍认同的规则。也就是说,潜规则广泛存在于社会的各个领域、各个层面、各类人群之中,已经通用化了。潜规则没有强制性,但实际上起作用,人们不得不遵循。你如果不按这些规则办事,也就别想办成事。‎ 答:____________________________________________________ ________________________________________________________________________________________________________________________‎ 答案 (示例)潜规则是看不见的、没有明文规定的、约定俗成的、被普遍认同的、实际上起作用的、人们不得不遵循的规则。‎ 解析 本题考查考生下定义型压缩语段的能力。解答本题时,首先要明确概念的定义必须是一个单句,所以要通过分析所给“潜规则”的文字信息,在其中找到一个比“潜规则”大的概念,即“规则”,这样就可以确定这个单句的主干应为“潜规则是……的规则”;然后准确筛选所给文字的关键信息,如“看不见的”“没有明文规定的”“约定俗成”“普遍认同的”“实际上起作用”“人们不得不遵循”,将这些关键信息填充到单句的主干句里即可。‎ ‎12.[2017·焦作联考]请在下面横线上补写这则消息的导语。‎ 抗日战士聚徐州 ‎(本报讯)今天是徐州沦陷纪念日。________。记者了解到,9名老人分别来自上海、北京、镇江、济南、南京、长沙等地,但都是徐州籍,年龄最大的86岁,最小的也已83岁。值抗日战争胜利70周年之际,老人们相约在徐州沦陷纪念日前夕,相聚到徐州,重话昔日的峥嵘岁月,再游当年生活、战斗过的地方。前日上午,他们参观后感叹徐州的变化太大。这些耄耋老人谈起徐州沦陷前后的战斗经历,激昂慷慨,还激动地唱起了《救亡军歌》。‎ 答:____________________________________________________ ________________________________________________________________________________________________________________________‎ 答案 当年曾在徐州生活、战斗过的9名八旬老人从全国各地齐聚徐州,共忆往昔峥嵘岁月 解析 导语是新闻的开头,是新闻结构中重要的组成部分,被称为新闻的“窗口”。它一般是对事件或事件中心的概述,用语要求简明扼要。补写导语:时间+地点+人物+事件。如本文,时间:今天(徐州沦陷纪念日)。地点:徐州。人物:9名耄耋老人。事件:从全国各地相聚徐州,忆往昔峥嵘岁月,重游当年生活、战斗过的地方。‎ ‎13.[2017·华南师大附中模拟]阅读下面的材料,用简洁的语言概括出“代际公平”的三项基本原则。(不超过30字)‎ ‎“代际公平”指当代人和后代人在利用自然资源、满足自身利益、谋求生存与发展上权利均等,当代人必须留给后代人生存和发展的必要环境资源和自然资源。每一代人应该为后代人保存自然和文化资源的多样性,避免限制后代人的权利,使后代人有和前代人相似的可供选择的多样性。每一代人都应该保证地球的质量,在交给下一代时不比自己从前一代人手里接过来时更差,也就是说,地球没有在这一代人手里受到破坏。每代人应该对其成员提供平行接触和使用前代人的遗产的权利,并且为后代人保存这项接触和使用权,使下一代人也能接触到隔代遗留下来的东西。‎ 答:____________________________________________________ ________________________________________________________________________________________________________________________‎ 答案 ①保存选择的多样性原则。②保存质量原则。③保存接触权与使用权原则。‎ 解析 先通读文段,理清层次。语段共四句话,第一句是阐述“代际公平”的概念,后三句是“代际公平”的三项基本原则。每一句话阐述一项,第二句阐述保存选择的多样性原则,第三句阐述保存质量原则,第四句阐述保存接触权与使用权原则。‎ ‎14.[2017·滨州模拟]‎ 共青团中央、全国学联等机构共同发布的《2015年中国大学生消费与生活形态研究报告》表明,当代大学生群体呈现出以下特性。‎ ‎①________。大学生作为无固定收入的群体,无论是必要的学费和生活费,还是休闲娱乐消费,在很大程度上都依赖于家庭的经济支持,但他们又具有明显的独立意识与倾向,他们并不认为花父母的钱天经地义,大多数学生都在自筹收入。‎ ‎②________。大学生追求个性释放,希望自己成为有独特风格的人,也喜欢风格独特的产品与品牌。但是这种独特是群体的独特,作为相互接触的群体,他们具有高度的一致性;同时作为群体中的个体,他们希望并主动与群体保持一致,并以这种一致获得群体的认同。‎ 根据以上表述,在下面的横线处概括这两个特性。(每处不超过10字)‎ 答:①_________________________________________________ ‎ ‎②_____________________________________________________‎ 答案 依附与独立并存(或既依附又独立) 个性与共性同在(或既独特又合群)‎ 解析 抓住关键词“但”,可知两段材料都是转折关系的句子。根据语意,分别提取出语意相对的词语。‎ ‎15.[2017·铜仁一中模拟]用简洁的语言概括下面这段话的两个主要观点。(每一个观点不超过25字)‎ 需要指出的是,气质只是天生的行为方式,其本身并没有好坏之分。每一种气质类型的儿童都有其积极的一面,也有其消极的一面。如容易照看型的儿童,他们的优点在于他们的随和、开朗、适应性强,但他们的不足在于行动有时会过于轻率、感情不够稳定;再如难以照看型的儿童,他们的积极方面在于他们比较敏感、感情丰富,而不足则在于他们的任性、适应慢和易发脾气;又如缓慢发动型的儿童,他们积极的一面是比较冷静、情感深沉、实干,而潜在的不足可能是冷漠、缺乏自信、孤僻。对每一种气质类型的儿童,都应该结合其气质特点,因材施教,这样才能最大限度地发挥他们的优势,避开他们的不足。‎ 答:____________________________________________________ ________________________________________________________________________________________________________________________‎ 答案 ①每一种气质类型的儿童各有其积极和消极的一面。②对不同气质类型的儿童应因材施教,扬长避短。‎ 解析 首先要找到段落中的概括性语句,即第2句和最后一句;其次根据题目要求,概括出两个主要观点。‎ 滚动提升训练(一)‎ ‎  时间:45分钟   满分:89分 ‎1.下列各句中加点成语的使用,全都正确的一项是(3分)(  )‎ ‎①当时的贾平凹每天坚持写作,不断向全国各大报刊投寄自己创作的作品,但往往石沉大海,没有一点儿音讯。‎ ‎②作为中国篮球的希望之星,“大魔王”周琦成为第七位被NBA选中的中国球员,其国家队队友王哲林随后也被孟菲斯灰熊队选中,可谓比翼双飞。‎ ‎③随着网络借贷的快速发展,部分不良网络借贷平台处心积虑地采取虚假宣传的方式诱导学生过度消费,侵犯学生的合法权益。‎ ‎④让人叹为观止的《星球大战》借鉴了神学、神秘主义、神话及经典文学等方面的元素,彰显了人类社会数千年来思想文化的积淀。‎ ‎⑤神舟十一号发射成功,使我国航天事业再一次让世界侧目而视的同时,也为我国空间站的建造和运营奠定了重要基础。‎ ‎⑥县政府一言九鼎,在县财政资金特别紧张的情况下,给我们兑现了100万元的科技创新奖奖金,体现了县政府加快推进新型工业化的决心。‎ A.①③④ B.②④⑥‎ C.①②⑤ D.③⑤⑥‎ 答案 A 解析 ①石沉大海:像石头掉到大海里一样,不见踪影,比喻始终没有消息。使用正确。②比翼双飞:比喻夫妻恩爱情深,形影不离。使用对象不当,可改为“双喜临门”。③‎ 处心积虑:千方百计地盘算(多含贬义)。使用正确。④叹为观止:指赞美看到的事物好到极点。使用正确。⑤侧目而视:形容畏惧或愤恨不满的神情。不合语境,应改为“刮目相看”。⑥一言九鼎:一句话的分量像九鼎那样重,形容所说的话分量很重,作用很大。不合语境,应改为“一诺千金”。‎ ‎2.[2017·全国高中名校名师预测卷]下列各句中加点成语的使用,全都正确的一项是(3分)(  )‎ ‎①中国戏剧家汤显祖、英国戏剧家莎士比亚、西班牙文学家塞万提斯这三位文艺巨匠,置身于不同的文化背景中,生活空间远隔万里,却都创作出了彪炳千古的名作。‎ ‎②当前的网络直播市场给人的直观感受就是鱼目混珠,一方面直播平台中教唆犯罪等违法违规内容并不少见,另一方面直播过程中的侵权问题也屡屡被诟病。‎ ‎③文学作品是影视作品的重要源流,由文学作品改编而成的电影、电视剧不胜枚举,据统计,80%以上的奥斯卡最佳影片改编自文学作品。‎ ‎④20世纪80年代,曾有日本学者称“敦煌在中国,敦煌学在日本”,此虽是三人成虎,当时却极大地激发了中国学者振兴中国学术的雄心壮志。‎ ‎⑤在这个追求速度的“快时代”,对效益的追逐导致一些从业者忘记了初心,这或许本也无可厚非,但文化产品的创造必须要有追求极致的工匠精神。‎ ‎⑥八月份时,他俩同时被邀请到沈阳参加一个经济大论坛,鬼使神差,命运总不让他们两人见面,他们只能通过电话、书信等交流。‎ A.①②④ B.①③⑤‎ C.②⑤⑥ D.③④⑥‎ 答案 B 解析 ①彪炳千古:形容伟大的业绩流传千秋万代。使用正确。②鱼目混珠:比喻拿假的东西冒充真的东西。不合语境,应用“泥沙俱下”(比喻好坏不同的人或事物混杂在一起)。③不胜枚举:指无法一个一个全举出来,形容同一类的人或事物很多。使用正确。④三人成虎:比喻谣言或讹传一再反复,就会使人信以为真。不合语境,句中没有“使人信以为真”的意思。⑤无可厚非:不可过分指责,表示虽有缺点,但是可以理解或原谅。使用正确。⑥鬼使神差:形容意外地发生某种巧合的事或不由自主地做出某种意想不到的事,多用于贬义。应用“阴差阳错”(由于偶然因素而造成差错)。‎ ‎3.[2017·全国高中名校名师预测卷]下列各句中加点成语的使用,全都正确的一项是(3分)(  )‎ ‎①这次在西安召开的学术研讨会,名家云集,不光专业人士表示收获盆满钵满,就连我们这些门外汉也收获颇丰。‎ ‎②许多人朝思暮想能够成为万众瞩目的明星,但很少有人知道,在明星璀璨的光环之下隐藏着许多不为世人所知的心酸。‎ ‎③优势互补、互利共赢是中沙关系长期造福两国人民的强大动力,推诚相见、民心相亲是中沙友谊历久弥新的不竭源泉。‎ ‎④当地政府为发展当地经济,提高农民的收入,殚精竭虑,采取了外派学习取经和聘请专家传宝等多种方式,取得了显著成效。‎ ‎⑤在有限的技术和财力支持下,他们将研究进行到这种程度,确实难能可贵,在整体规划上虽然有些不足,但也无可非议。‎ ‎⑥这件祖传的藏品非常珍贵,王先生一直秘不示人,今天高兴之下公诸同好,让在场的几位颇有见识的大家都赞叹不已。‎ A.①③⑤ B.③④⑥‎ C.①②⑥ D.②④⑤‎ 答案 B 解析 ①盆满钵满:形容赚的钱很多。句子语境是学术研讨会收获颇丰,此处用错对象。②朝思暮想:形容时刻想念。用于此处不合语境。③推诚相见:用真心相待。用于此处正确。④殚精竭虑:用尽精力,费尽心思。用于此处正确。⑤无可非议:没有什么可以指摘的,表示言行合乎情理。句子语境是“有些不足”,用于此处不合语境。⑥公诸同好:把自己喜爱的东西拿出来给有同样爱好的人共同欣赏、享受。用于此处正确。‎ ‎4.[2017·兰州一中阶段测试]下列各句中,加点的词语使用不恰当的两项是(3分)(  )‎ A.由于房地产行业不景气,他已经决定金盆洗手,不再做房地产生意,重操旧业搞养殖。‎ B.前天晚上地铁里遭遇拥挤,昨日“海上世界”也是人满为患,在回来的路上,走望海路堵车,由望海路拐向滨海路也堵车,拥挤不堪的道路让人心烦。‎ C.电视剧《琅琊榜》因故事生动曲折而热播不断,一度达到万人空巷的地步,该剧也因此成了近几年来少有的经典之剧。‎ D.科隆新年夜发生的耸人听闻的群体犯罪事件,再度引发德国民众对外来难民的恐慌情绪,科隆市长发言人说,政府绝不容许“无人区”的出现。‎ E.自2016年6月起,经过三个月的改造,仁和花园焕然一新。如今,小区活动广场内篮球场、健身器材、桌椅等一应俱全。‎ 答案 CD 解析 A项,金盆洗手:指盗贼等改邪归正,不再干坏事。借指退出本行业,罢手不干。这里使用正确。B项,人满为患:指因人多造成困难。这里使用正确,与“遭遇拥挤”对举。C项,万人空巷:指家家户户的人都从巷里出来了。多形容庆祝、欢迎等盛况。句中把“万人空巷”曲解了,以为是街上空空,所有人都窝在家里(看电视剧),用来形容电视剧《琅琊榜》很受欢迎。D项,耸人听闻:指夸大或捏造事实,使人听了感到惊异或震动。此处应为“骇人听闻”。E项,焕然一新:指改变旧面貌,出现崭新的气象。含褒义。焕然:鲜明光亮的样子。这里指小区面貌发生了很大的变化,使用正确。‎ ‎5.[2017·江西六校联考]下列各句中,加点的成语使用不恰当的两项是(3分)(  )‎ A.不少省市片面追求GDP,无节制地开采、出口稀土等资源,这种行为无异于为渊驱鱼,为丛驱雀,长此以往,稀有资源将消耗殆尽。‎ B.他本来想给人耳目一新的感觉,结果过犹不及,一身奇异的装扮反而令人难以接受。‎ C.日本挑起钓鱼岛事端后,在政治上日渐孤立,他们自知理亏,便东奔西跑,逢人说项,试图拉拢不相干的国家,结果碰了一鼻子灰。‎ D.除了每周精选的三场英超赛事直播外,我们还为您准备了精彩的比赛点播,浓缩精华的比赛集锦、令人拍案叫绝的进球、让你捧腹不已的搞笑花絮。‎ E.真正的读书,需要一种不急不躁、不愠不火的从容心境。在淡泊的氛围中,上溯远古、下及未来,含英咀华,沉思遐想,思绪跟随古色古香的书味尽情飞翔,内心就会进入一种清静、清幽的幸福境界。‎ 答案 AC 解析 A项,为渊驱鱼,为丛驱雀:比喻不善团结人,把可以依靠的力量赶到敌对方面去。望文生义。B项,过犹不及:事情办得过火,就跟做得不够一样,都是不好的。C项,不合语境;逢人说项:比喻到处为人说好话。D项,拍案叫绝:拍桌子叫好,形容非常赞赏。E项,含英咀华:比喻细细地玩味和体会文章的精华。‎ ‎6.[2017·黄冈调研]填入下面文段空白处的词语,最恰当的一项是(3分)(  )‎ 高考作文要有读者意识。平时作文,__①__是写作者自赏,__②__是小范围好友传阅,__③__是通过发表让公众了解,文章都是通过阅读获得生命力的,__④__读者对于一篇文章来说很重要。考场作文是一种在特定环境下的戴着镣铐的舞蹈,考生要有读者意识。考场作文的读者,就是那个未曾谋面的阅卷老师。其读者的特殊性__⑤__决定了写作的特殊性,__⑥__决定了树立读者意识的必要性。‎ ‎①‎ ‎②‎ ‎③‎ ‎④‎ ‎⑤‎ ‎⑥‎ A 无论 还 抑或 因此 不仅 而且 B 既 ‎/‎ 也 所以 不但 还 C 不管 也 还 因为 既 又 D ‎/‎ 还 又 因而 不仅 而且 答案 A 解析 “无论”与后面的“都”照应,构成条件关系。“是……还是……抑或是”三句构成选择关系。“文章都是通过阅读获得生命力的”与“读者对于一篇文章来说很重要”之间有因果关系,故用“因此”‎ ‎。最后两个分句之间有递进关系,故选用“不仅……而且”。‎ ‎7.[2017·深圳模拟]填入下面文段空白处的词语,最恰当的一组是(3分)(  )‎ 人的体质需要锻炼,__①__生产劳动就是很好的锻炼,__②__不管是在野外劳动__③__在厂房的工业劳动,对脑力劳动都是一种调剂。__④__多数学生劳动回来,都变得更健康了。__⑤__不锻炼身体,__⑥__课堂上学得很多,将来也未必有大的作为。‎ ‎①‎ ‎②‎ ‎③‎ ‎④‎ ‎⑤‎ ‎⑥‎ A ‎/‎ 可是 或者 不过 所以 即使 B 而 因为 或者 可是 可见 而 C ‎/‎ 因为 还是 所以 如果 即使 D 而 可是 还是 所以 不过 而 答案 C 解析 整个语段是议论性语段,其论点就是第一句话。首句论点与后面的分句之间是因果关系,据此可以排除A、D两项。根据“不管”可以断定第③空应填“还是”,而排除带有“或者”的A、B两项。第④空前后是因果关系,第⑤空之后是假设论证,最后两个分句之间是假设关系,因此可以排除A、B、D。‎ ‎8.[2017·重庆南开中学模拟]填入下面文段空白处的词语,最恰当的一组是(3分)(  )‎ 中国古代的文人墨客,富贵贫贱,通达潦倒,欢喜哀愁,__①__离不开酒。__②__说魏晋南北朝的文人饮酒是隐逸中略带悲剧情调,那么唐朝文人饮酒更多的是放浪和旷达。__③__唐代文人之醉酒,__④__有忘记现实痛苦的因素,更多的是壮志豪情的抒发和对未来的展望。“酒入豪肠,七分酿成了月光,剩下三分啸成了剑气,绣口一吐就成半个盛唐。”盛唐之盛,不仅在于锦绣繁华,满目俊才,__⑤__也少不了诗情勃兴,酒气恣横。__⑥__有人说整个盛唐的诗歌都是由酒香酿成的。‎ ‎①‎ ‎②‎ ‎③‎ ‎④‎ ‎⑤‎ ‎⑥‎ A 总是 既然 而 ‎/‎ 但是 从而 B 都 如果 ‎/‎ 固然 而且 因此 C 可是 倘若 那么 既 ‎/‎ 所以 D 却 与其 所以 虽然 反而 ‎/‎ 答案 B 解析 本题考查正确使用词语的能力。总是:副词,表示一直如此,经常如此/全是,都是。都:副词,表示总括。可是:连词,表转折/副词,真是,实在是。却:连词,表转折。①前后语境没有转折关系,排除C、D两项。既然:连词,表示先提出前提,而后加以推论,常与“就”“也”“还”等搭配。“如果”表假设关系,常与“那么”“就”等搭配。②后有关联词“那么”,应选与之搭配的“如果”。故选B。‎ ‎9.[2017·大同一中质监]填入下面文段空白处的词语,最恰当的一组是(3分)(  )‎ 服从祖国、服务人民、艰苦创业、无私奉献__①__是祖国建设者的高贵品质。哪儿需要他们,他们__②__到哪儿去。__③__自然环境多么恶劣,__④__生活条件多么艰难,他们__⑤__坚强地工作,顽强地战斗。不逃避、不妥协,__⑥__奉献出全部的青春和热血。‎ ‎①‎ ‎②‎ ‎③‎ ‎④‎ ‎⑤‎ ‎⑥‎ A ‎/‎ 才 无论 无论 一定 直至 B 无疑 ‎/‎ 不论 ‎/‎ 仍然 以致 C 一定 总是 尽管 尽管 总是 ‎/‎ D ‎/‎ 就 不管 不管 总是 直至 答案 D 解析 结合整个文段看,①处不填或填“无疑”均可,填“一定”生硬,排除C项。②处填“就”有主动意味,填“才”是被动意味,“会”是对客观情况的陈述。填“总是”也可以,但与前文的“哪里”搭配不是很流畅。③④处与后文是条件关系,填“尽管”与“多么”不搭配。⑤处填“一定”“仍然”都与③④处“不管”衔接不流畅,应填“总是”。排除A、B。⑥处应填表程度逐渐加深的“直至”。故选D项。‎ ‎10.[2017·江淮十校第一次联考]下列各句中,没有语病的一句是(3分)(  )‎ A.对我国民族传统文化的弘扬,不是复古,不是要让人们回到过去,而是要人们能从传统文化中汲取有益的知识,从历史中找寻过去、现在与未来的联系,以史为镜,以史为鉴。‎ B.要是任凭形形色色的“党内圈子”的恶行发展下去,我党用鲜血和生命在群众中所建立的形象就会“断崖式塌陷”,党执政的根基就会动摇。‎ C.我国古代的高考,就是科举考试,科举考试从设立时起就被确立为一种制度,有着严格的规范性操作。‎ D.这一方面需要管理者在加大制度建设完善的同时,将道德约束融入进去,德法合一规范公民行为;另一方面要加强道德教育,引导公民接受道德养成教育。‎ 答案 B 解析 A项,“以史为镜,以史为鉴”,语意重复,表达累赘。C项,“规范性操作”应为“操作规范”。此句也可以说是搭配不当,“严格的操作”搭配不当,正确的搭配是“严格的规范”。D项,“加大制度建设”本来已经说完,又加上一个尾巴“完善”。语病是句式杂糅。可改为“加大制度建设力度”,或者说“完善制度建设”。‎ ‎11.[2017·鹤壁段考]下列各句中,没有语病的一句是(3分)(  )‎ A.本次大赛本着继续扶掖杂文新秀、普及杂文尤其是荒诞杂文为宗旨,这让我们在欣喜之中再一次见证了荒诞杂文确为广大读者所喜闻乐见。‎ B.虽然多家媒体报道了“气功大王王林涉嫌杀害弟子邹勇”的消息,但截至到7月23日上午,警方尚未确认邹勇死亡及王林雇凶杀人的最新消息。‎ C.因为强行推动安保法案在众院表决通过,让安倍晋三内阁的支持率大幅下滑,使得安倍晋三首相连日来登上各大电视台,以期获得国民的理解。‎ D.为了拥有一个健康长寿的生命,我努力学习和掌握生活的百科全书,练就专家般的尖锐目光,坚决防范那些携毒的食品冠冕堂皇地进入本人嘴巴,以维护生命的良好运转。‎ 答案 D 解析 本题先抓典型的语病标志,比如两面词、判断词、并列动词;然后压缩句子,保留主干,看是否残缺、是否搭配。A项,“本着……为宗旨”杂糅,改为“本着……的宗旨”或“以……为宗旨”。B项,语意重复,去掉“到”。C项,缺主语,去掉“因为”。‎ ‎12.[2017·河池摸底]下列各句中,没有语病的一句是(3分)(  )‎ A.近10年来新加坡成为中国留学生选择出国深造的热门之地,毕业之后也选择留在新加坡,成为中新友好交往的民间使者。‎ B.随着政府投入的加大,使中国的健康养老产业发展必将迎来更快的增长期,各市场主体的竞争和合作必将进一步深化。‎ C.如何科学地看待当前中国部分制造企业经营困难的现象,并找到解决这些困难的方法,是有关各方必须认真思考的问题。‎ D.由于售价低廉,药企微利甚至无利,缺乏生产积极性不高,导致不少低价药从市场上消失,消费者再难买到以前常用的廉价好药。‎ 答案 C 解析 A项,偷换主语,可在“毕业之后”前加“这些留学生”。B项,乱用介词,删去“随着”或“使”。D项,结构混乱,删去“缺乏”或“不高”。‎ ‎13.[2017·黄石调研]下列各句中,没有语病的一句是(3分)(  )‎ A.据《菲律宾商报》指出,菲律宾总统杜特尔特22日称,希望中国帮助菲律宾经济发展。‎ B.《咬文嚼字》主编黄安靖认为,流行语要顺应时代发展潮流,传递社会“正能量”,合乎汉语语法规范,既要有流行性,也要有创新性。‎ C.当地政府迅速启动应急预案,紧急安置、转移受灾群众1000余人,并提供衣物、食品、饮用水等生活必需品。‎ D.科学家们认为,由方忠研究员带领的团队首次在实验中发现的外尔费米子,这一发现对拓扑电子学和量子计算机等颠覆性技术的突破具有非常重要的意义。‎ 答案 B 解析 A项,“据……指出”句式杂糅,可去掉“据”;C项,语序不当,应为“转移、安置”;D项,结构混乱,“发现的外尔费米子”应改为“发现了外尔费米子”。‎ ‎14.[2016·西工大适应性训练]以“雪花”为话题,把下面的词语扩展成一段话。(不超60字)(5分)‎ 寒风 大地 生机 答:____________________________________________________ ____________________________________________________________‎ 答案 片片雪花纷纷离开天空的怀抱,在阵阵寒风中飞向大地,偎依在大地母亲的怀抱中,滋润着万物,为春的生机默默奉献着自己的一切。‎ 解析 应结合生活实践想象雪景,恰当使用所给词语,可适当运用拟人等修辞,注意字数限制。‎ ‎15.[2017·定州中学模拟]揣摩下面这首诗的内容和意境,以“山行”为话题,扩写画线句,表达出一种趣味。不少于30个字。(5分)‎ 雨过山村 王建 雨里鸡鸣一两家,竹溪村路板桥斜。‎ 妇姑相唤浴蚕去,闲看中庭栀子花。‎ 答:____________________________________________________ ____________________________________________________________‎ 答案 在霏霏的小雨中沿着斗折蛇行的小路一边走,一边听那萧萧竹韵、潺潺水声,十分惬意。不觉来到一座小桥跟前,这木板搭成的倾斜的小桥与竹溪村路是多么地自然和谐。‎ 解析 解答本题时首先要整体理解诗的内容和意境,然后借助联想和想象,对景物进行描述,突出其情趣。‎ ‎16.[2017·东北育才中学模拟]请根据武元衡的诗句“杨柳阴阴细雨晴,残花落尽见流莺”展开合理的联想和想象,将其扩展成一段话,扩展后不少于60字。(5分)‎ 答:____________________________________________________ ____________________________________________________________‎ 答案 细雨初晴的春日,溪边杨柳的颜色已经由初春时节如烟的鹅黄嫩绿转为一片翠绿,花树枝头的残花在雨中落尽,循着婉转的啼叫,可以看到在树间嬉戏的黄莺。‎ 解析 解答此题,首先要抓取诗句中的意象,然后分析意象呈现的特点,从而把握诗句创设的意境。最后展开合理的联想和想象,描绘出符合要求的画面即可。‎ ‎17.[2017·揭阳、潮州联考]阅读下面一首词,根据词的内容,对画线语句加以扩写。要求:使用排比、夸张两种修辞手法,不超过80字。(5分)‎ 长相思 李煜 一重山,两重山。山远天高烟水寒,相思枫叶丹。‎ 菊花开,菊花残。塞雁高飞人未还,一帘风月闲。‎ 答:____________________________________________________ ____________________________________________________________‎ 答案 菊花那多情的花蕊,那撩人的形态,那四溢的芬芳,还留在昨天的记忆里,而今,那些菊花已经枯萎,残菊随风飘零,三千丈愁思弥漫在风中!‎ 解析 扩展时要抓住“菊花开”与“菊花残”。通过想象描写菊花开时的情态以及菊花残败的景象。要注意景与情的融合,做到情景交融,寓情于景。情感基调可以从诗歌中理解,应是一种愁思。同时还要注意排比和夸张手法的运用。‎ ‎18.[2016·沧州二模]根据下面的文字提示和限定,在语段的横线处扩写相关内容,不少于60字。(5分)‎ 关切人的生命价值,并给人们以精神支柱,是儒、道、佛三家的共同特征:儒家________________________________ ________________________________________________________________________________________________________________;‎ ‎________________________________________________________________________________________________________________________________________________;‎ ‎________________________________________________________________________________________________________________________________________________。‎ 在历史上,三家各以其自身的文化特质发挥着各自的优势,弥补着彼此的不足。‎ 答案 (儒家)以积极的入世精神,并在现实世界中成就最高的道德境界,实现人生的价值,给无数英雄豪杰带来精神鼓舞和巨大满足 道家重视人作为个体的存在价值,主张顺乎自然,从而为不重功名利禄者或默默无闻者提供精神支持 佛家则关心人的生老病死,关心愿望和现实之间永远无法消除的距离所带来的痛苦,给有深切痛苦感的人以心理的温暖和慰藉 解析 扩写内容应根据横线前文字内容的提示和形式的限制,横线前的文字是语段的中心观点句,冒号提示并限制扩写内容应是对中心句的解说,与其形成总分关系。‎ ‎19.[2017·吉林友谊联盟联考]请为下面这则新闻拟定一个标题,不超过20字。(5分)‎ ‎23日晚,首届中国好书评选结果在中央电视台一套综合频道公布。当晚,从40万册当年图书中脱颖而出的25本“2015年中国好书”的作者和编者都现身央视舞台,讲述纸面背后的故事。记者从这份中国式好书单中发现,科技和经济社会类图书超过总数的一半,这与以前明显不同。受访专家认为,社科经济、历史类图书在多元的阅读趣味中占据主流,反映了当代读者关注中国现实的意识和社会责任感。‎ 答:____________________________________________________ ____________________________________________________________‎ 答案 (示例)“中国好书榜”透露阅读新动向 解析 这则消息叙述的对象是“中国好书榜”,事件是“透露阅读新动向”,连缀成句即可。‎ ‎20.[2016·济南二模]阅读下面的语段,概括其主要内容。(不超过25字)(5分)‎ 在天山高处,可以看到一个面积很大的天然湖。湖面明净如镜,水清见底。高空的白云和四周的雪峰清晰地倒映在水中,把湖山天影融为晶莹的一体。在这幽静的湖中,唯一活动的东西就是天鹅。天鹅的洁白增添了湖水的明净,天鹅的叫声增添了湖面的幽静。人家说山色多变,而事实上湖色也是多变的。你站在高处瞭望湖面,眼前是一片赏心悦目的茫茫碧水,如果你再留意一看,接近你的视线的是那闪闪的粼光,像千万条银鱼在游动,而远处平展如镜,没有一点纤尘或者一根游丝的侵扰。湖色越远越深,由近到远,是银白、淡蓝、深青、墨绿,界线非常分明。‎ 答:____________________________________________________ ____________________________________________________________‎ 答案 天山高处的天然湖湖水明净,环境幽静,湖色多变。‎ 解析 首先确定本题的描写对象是天然湖;其次运用“舍偏取正法”将具体描写天然湖的句子找出来,将描写其他对象的句子删除;再次借用语段中“定性”的词语“明净”“幽静”来概括湖水以及周遭环境的特征,用言语转化的方式归纳最后一句,总结出湖色多变的特色;最后按照一定的表达顺序形成答案。‎ ‎21.[2017·重庆南开中学模拟]阅读下面的语段,用一句话概括“丹书铁券”在明代的发展变化。(不超过55字)(5分)‎ ‎“丹书铁券”是古代帝王颁授给功臣、重臣的一种特权信物。之所以称“丹书铁券”,是因为早期的铁券是以朱砂填涂其上的文字。用朱砂填涂文字,可使颜色纯正,又能使色彩经久不褪;以铁为券,取其坚固、久存之意。明初,朱元璋创建了历史上最完备的铁券制度,从法律上规范了“丹书铁券”的有关内容。如其颁授对象,仅限于立有军功,被封为公、侯、伯的勋臣;至于权限范围,仅限于本人和后代犯罪不加刑。洪武三年(1370),李善长、徐达、李文忠等34人获公爵或侯爵封号,并被赐予“丹书铁券”。明中后期,仍不时有功臣及其后裔获赐铁券。明末,崇祯皇帝还曾给大宦官魏忠贤的侄子魏良卿颁赐铁券。清代,“丹书铁券”制度被废除。‎ 答:“丹书铁券”________________________________________ ‎ ‎____________________________________________________________‎ 答案 (“丹书铁券”)在明初创建了最完备的铁券制度,规定了颁授对象和权限范围;到了明朝中后期,颁授对象扩大到有功之臣的后裔。‎ 解析 解答该题,首先要分析语段的意思。第一、二、三句,解说“丹书铁券”以及使用“丹书”“铁券”的具体原因;第四句,写明初朱元璋为“丹书铁券”建立了完备制度;第五句,从颁授对象和权限范围两个角度解说明初铁券制度的内容;第六句,写明初颁授“丹书铁券”的对象;第七、八句,写明中后期及明末“丹书铁券”的颁授对象;最后一句,写“丹书铁券”制度在清朝被废除。由分析可知,要概括“丹书铁券”在明代的发展变迁,就要把握“丹书铁券”在明代的发展变化,即“丹书铁券”在制度、授予对象等方面的发展变化等。‎ ‎22.[2017·成都外国语中学段考]请依据下面的文字叙述,为“幸福指数”下一个定义,字数控制在20字之内。(5分)‎ 幸福和痛苦是我们每个人都会有体会的两种截然相反的感受。幸福指数最早是美国经济学家萨缪尔森提出来的,他认为幸福等于效用与欲望之比。当欲望既定时,效用越大越幸福;当效用既定时,欲望越小越幸福。幸福与效用同方向变化,与欲望反方向变化。如果欲望是无穷大,则幸福为零。幸福指数这个系数衡量的是个人的主观愿望,每个人认为自己幸福与否和自己的欲望及效用有关。‎ 答:幸福指数就是________________________________________ ‎ 答案 (幸福指数就是)衡量人们对生活满意程度的一种主观系数。‎ 解析 应准确筛选出关键句“幸福指数这个系数衡量的是个人的主观愿望”,把握幸福与效用、欲望的关系,进而做出合理的概括。注意字数限制。‎ ‎23.[2017·辽宁葫芦岛联考]请根据下面语段的主要内容,提取三个关键词语。(5分)‎ 高新技术发展特别是互联网的出现,给政府管理带来新的机遇,也带来新的挑战。网络时代公众更加积极地参与到社会管理中来,并且更加勇于承担社会责任。这也要求政府不断适应民主、开放、互动的网络生活,积极探索网络时代推进政治民主、加强和改善社会管理的方法和途径。公众是社会管理活动的共同责任承担者,公众参与到行政管理、社会管理和法制监督过程中,行使参与管理、监督行政的宪法权利和法定权利,形成参与行政、合作行政、共同治理的新格局,这已经成为一种时代潮流。‎ 答:①________ ②________ ③________‎ 答案 网络时代 公众参与 政府管理 解析 提出关键词要掌握语段的主要内容,扣住主要信息。本段的主要内容是网络时代的社会管理。所以应扣住“网络时代”,管理者“公众”及“政府”。‎ 考点五 选用、仿用、变换句式(含修辞)‎ 考点名片 考点内容 选用、变换句式:以长句与短句、整句与散句、否定句与肯定句、陈述句与疑问句的选用和变换为主。‎ 仿用句式:主要考查语言能力、联想能力和创造能力。‎ 修辞:考查重点是比喻、比拟、排比、对偶等正确及有意识的主动运用。‎ 考查形式 本考点多为主观题。①选用句式多与语言连贯等结合在一起考查,一般是从所供选语句中选择合适的语句填入相应的位置;②变换句式以长句与短句、整句与散句的变换较为多见。简答题,4~6分。‎ 仿用:①一般是在一个特定的语段内根据要求进行填空式仿写;②根据所仿语句另选话题仿写。‎ 修辞:多见简答题形式。①单考修辞,或与仿写句式相结合;②试题要求分“明示”(明确提出要求)和“暗含”(未明确提出相关要求,但例句中暗含相应的要求)两种。‎ 趋势分析 今后对本考点的考查会更灵活,会与语言表达简明、得体、准确、鲜明,压缩语段等考点综合考查。‎ 题组1 选用句式 ‎【语句复位】‎ ‎1.下列语句填入语段中画横线处,最恰当的一项是(  )‎ 说着,进入石洞来,只见佳木葱茏,奇花熌灼,一带清流,从花木深处曲折泻于石隙之下。________________,皆隐于山坳树影之间。俯而视之,则清溪泻雪,石磴穿云,白石为栏,环抱为沿,石桥三港,兽面衔吐。桥上有亭。‎ A.渐向北边,再进数步,雕甍绣槛,两边飞楼插空,平坦宽豁 B.再进数步,渐向北边,两边飞楼插空,平坦宽豁,雕甍绣槛 C.渐向北边,雕甍绣槛,再进数步,平坦宽豁,两边飞楼插空 D.再进数步,渐向北边,平坦宽豁,两边飞楼插空,雕甍绣槛 答案 D 解析 做题时要注意:①综观材料,把握材料的中心。②弄清材料的陈述对象,语段的结构特点,感情基调,以及主要句子的句式特点等。③语句的连贯不能只注重语言形式的连贯,主要应看句子的内容是否前后照应,意思是否连贯顺畅。瞻前:先进再向北;顾后:“皆隐于”必须是两者,所以选“飞楼插空,雕甍绣槛”。‎ ‎2.填入下面一段文字横线处的语句,最恰当的一句是(  )‎ 有人曾向希尔伯特请教:明天最重要的技术是什么?这位20世纪科学工作者界的领袖幽默地说:“到月球上去抓苍蝇。”________,希尔伯特在回答为什么时说:“要实现这一目标,意味着我们要解决人类面临的所有物质困难。”今天,人类已经成功地登上了月球。去月球发电、种庄稼,已经像“抓苍蝇”一样实实在在地呈现在我们面前。‎ A.这有点儿像天方夜谭。因为要做到这一点实在需要克服太多太多的困难 B.这不是一句玩笑话。因为要做到这样一件小事,将要碰到一连串的难题 C.这简直不可思议。因为要做到这一点实在需要克服太多太多的困难 D.因为要做到这样一件小事,将要碰到一连串的难题。这是无法想象的 答案 B 解析 本题与排序题有点区别,那就是要用文字将上下文串联起来,故难度要大一些。解答这类题目的关键在于,要反复研读横线前后的句子,辨明前后内容之间的关系,借助这种关系以及前后句的内容和形式来确定横线句子的内容和形式。例如题中空白处就要紧密联系上下文,抓与上下文的衔接,B项,“幽默地说”可以衔接“玩笑话”,“一件小事”呼应“抓苍蝇”。‎ ‎3.填入下面一段文字横线处的语句,下列选项最符合原文的一项是(  )‎ 既有条件,读书万万不能狭窄。凡能找到的书,都要读读。若读书面窄,思路就不广。但是,切切又不要忘了精读,真正的本事掌握,全在于精读。你若喜欢上一本书了,不妨多读:________________。三遍读过,放上几天,再去读读,常常又会有再新再悟的地方。‎ A.第一遍可静心坐下来读,这叫吟味;第二遍就囫囵吞枣读,这叫享受;第三遍便要一句一句想着读,这叫深究 B.第一遍可一句一句想着读,这叫深究;第二遍就静心坐下来读,这叫吟味;第三遍便要囫囵吞枣读,这叫享受 C.第一遍可囫囵吞枣读,这叫享受;第二遍就静心坐下来读,这叫吟味;第三遍便要一句一句想着读,这叫深究 D.第一遍可囫囵吞枣读,这叫享受;第二遍就一句一句想着读,这叫深究;第三遍便要静心坐下来读,这叫吟味 答案 C 解析 解答此题,可根据对文段内容的理解,将选项分别代入原文进行比较分析,找出选项与原文内容的细微差别,读后可知,文段是按照“粗读”“精读”“研究”来讲的,依此思路判断,很容易得出答案为C。‎ ‎4.填入下面横线处的语句,与上下文前后连贯、音节和谐的一组是(  )‎ ‎(1)________,正体现了文人们对粗犷和细腻相结合这一审美理想境界的追求。‎ ‎(2)漫步苏堤,两边都是湖水,远水如烟,________。‎ ‎①远山借青、远水借碧的宏观构思,园中有园、湖里有湖的精巧布置 ‎②园中有园、湖里有湖的精巧布置,远山借青、远水借碧的宏观构思 ‎③近水如练,新荷满布,郁郁青青 ‎④近水如镜,兰舟轻漾,一湖幽静 A.①③ B.②③‎ C.①④ D.②④‎ 答案 C 解析 第(1)句根据后文的“粗犷和细腻”的顺序,应该先“宏观构思”,再“精巧布置”,所以选①;第(2)句从押韵的角度看,“镜”与“静”音节和谐,所以选④。‎ ‎5.依次填入下面一段文字横线处的语句,衔接最恰当的一项是(  )‎ 其实,孔子对于读书有他自己的看法。________。________。________。________。________,________,类似于“不患莫己知,求为可知也”这样的话,《论语》中至少重复了四次。‎ ‎①他说:“知之者不如好之者,好之者不如乐之者。”‎ ‎②他还主张读书是为完善自己,鄙夷那种沽名钓誉的文人 ‎③而无须在乎外在的名声和遭遇 ‎④他主张读书要从兴趣出发,不赞成为求知而求知的态度 ‎⑤他曾说:“古之学者为己,今之学者为人。”‎ ‎⑥他一再强调,一个人重要的是要有真才实学 A.④①②⑤⑥③ B.⑥③⑤②④①‎ C.④①⑥②⑤③ D.⑥③⑤④②①‎ 答案 A 解析 此类题可以用排除法。④①为一组,中心是“兴趣”。②⑤为一组,中心是“完善自己”。⑥③为一组,中心是“真才实学”。所以选A。‎ ‎6.依次填入下面一段文字横线处的语句,最恰当的一组是(  )‎ 循环经济是对生产、流通和消费过程中进行的减量化、再利用、资源化活动的总称。它________,________,________,________,________,________,有助于构建资源节约型、环境友好型社会。‎ ‎①是转变经济增长模式的一个突破口 ‎②有效地利用资源和保护环境 ‎③促进以最小的资源消耗、最少的废物排放和最小的环境代价 ‎④通过建立“资源—产品—再生资源”和“生产—消费—再循环”的模式 ‎⑤换取最大的经济效益 ‎⑥也是贯彻科学发展观的一个重要举措 A.①④⑤③②⑥ B.①⑥④③⑤②‎ C.④⑤⑥③②① D.④②③⑤①⑥‎ 答案 D 解析 建立模式④——利用保护②——是手段措施;结果是最小的资源消耗、最少的废物排放和最小的环境代价③——换取最大的经济效益⑤;最后是评价突破口①——“也是”重要举措⑥,完全符合逻辑顺序。‎ 题组2 变换句式 ‎【长短句变换】‎ ‎7.把下面这个长句改写成几个较短的句子,可以改变语序、增删词语,但不得改变原意。‎ 他的著作用康德、叔本华的美学思想,就境界的主客体及其对待关系、境界的辩证结构及其内在的矛盾运动、境界美的分类与各自特点,对境界这一中国传统的美学范畴进行了详细的阐释。‎ 答:____________________________________________________ ____________________________________________________________‎ 答案 他的著作对境界这一中国传统的美学范畴进行了详细的阐释,阐释的依据是康德、叔本华的美学思想,阐释的内容既有境界的主客体及其对待关系,也有境界的辩证结构及其内在的矛盾运动,还有境界美的分类与各自特点。‎ 解析 解题时,首先要准确把握所给句子的原意,认真分析句子的构成元素,分清主次,提取主要信息点。可以运用“剥离法”,把中心语中的并列修饰成分分开,形成几个并列的分句。然后检查一下所改的句子是否符合原意,句子是否通顺,内容是否符合逻辑。‎ ‎8.将下列句子组合成一个长单句。可以增删个别字词,但不得改变原意。‎ ‎①鲁迅是中国近现代最伟大的知识分子之一。‎ ‎②传统的知识分子,被“四书”“五经”吃空了灵魂。‎ ‎③他创作的《怀旧》《孔乙己》等作品,表达了他对传统知识分子的嘲讽与同情。‎ 答:____________________________________________________ ____________________________________________________________‎ 答案 作为中国近现代最伟大的知识分子之一的鲁迅创作了旨在表达他对被“四书”“五经”吃空了灵魂的传统的知识分子的嘲讽与同情的《怀旧》《孔乙己》等作品。‎ 解析 本题要求把三个短句组合成一个长句。首先抓主干,确定主谓成分,其他句子充当枝叶,以定语的形式插入主干句中。应以第③句为主干;第②句提到“传统的知识分子”的特点,可做定语;第①句无法插入,可放在句前做状语;还要注意连接句子成分的用词,如“旨在”“作为”。‎ ‎9.将下面的四个短句改为一个长单句,可适当增减词语,但不能改变句子的原意。‎ ‎①寺庙、佛塔在地震中往往不容易损毁、倒塌。‎ ‎②这是因为寺庙、佛塔选址和建筑有玄机。‎ ‎③它们比一般建筑更为科学、合理,具有极好的防震抗震性。‎ ‎④科学最终揭开了这一“不倒之谜”。‎ 答:____________________________________________________ ____________________________________________________________‎ 答案 科学最终揭开了选址和建筑有玄机,比一般建筑更为科学、合理,具有极好的防震抗震性的寺庙、佛塔在地震中往往不容易损毁、倒塌的“不倒之谜”。‎ 解析 第一步,分析各短句之间的内在联系。从中找出可以做长句主干的一个句子,作为长句主干。此句的主干是“科学最终揭开了这一‘不倒之谜’”。第二步,根据各句关系,确定修饰成分,并明确它们的位置。第②③句作为第①句中“寺庙、佛塔”的定语补充进去,然后把补充后的句子替代主干句中的指代词“这”。第三步,检查变换后的句子是否是一个单句,是否与原来的短句表达的意思一致。‎ ‎10.把下面的长句改成四个短句,可改变语序、增删词语,但不得改变原意。‎ 微信是腾讯公司于2011年1月率先推出的一种基于跨通信运营商、跨操作系统平台支持的通过网络来为广大用户快速发送语音短信、视频、图片、文字等内容从而为智能终端(如智能手机等)提供即时通讯服务的免费应用程序。‎ 答:____________________________________________________ ____________________________________________________________‎ 答案 微信是一种为智能终端(如智能手机等)提供即时通讯服务的免费应用程序。它由腾讯公司于2011年1月率先推出,它基于跨通信运营商、跨操作系统平台支持服务,它能通过网络为广大用户快速发送语音、视频、图片、文字等内容。‎ 解析 题中可以分为四个并列成分,“腾讯公司于2011年1月率先推出的”“基于跨通信运营商、跨操作系统平台支持的”“通过网络为广大用户快速发送语音短信、视频、图片、文字等内容”“为智能终端(如智能手机等)提供即时通讯服务的免费应用程序”。给他们加上共同的主语“微信”,加以总结形成答案,注意语句通顺。‎ ‎11.请将下面的长句改写成三个短句,不得改变原意,可适当增删词语。‎ 传统的现代派绘画——由毕加索、康定斯基、马蒂斯以不同的方式发展起来的抽象艺术是以高度发达的审视技能以及对其他绘画和艺术史的熟谙程度为先决条件的。‎ 答:____________________________________________________ ____________________________________________________________‎ 答案 ①传统的现代派绘画是抽象艺术。②它是由毕加索、康定斯基、马蒂斯以不同方式发展起来的。③它是以高度发达的审视技能以及对其他绘画和艺术史的熟谙程度为先决条件的。‎ 解析 这是一道句式转换中的长句变短句,此类试题的解题思路是先确立句子的主干,把主干句单独列为一句,然后把其他修饰成分(定语或状语)拆解成若干短句,最后按照一定的关系重新排序即可。‎ ‎12.以所要求的文字为开头,把下面的一段话整合为一个单句。‎ 在中国传统中,用以提高人的精神素养、改变经济对人的支配性影响的“读书”是一种具有特定含义的学习行为,是除直观意义上的阅读书籍或学习技能之外,更在于淡化功利性和超越技能学习的一种学习行为。‎ 答:在中国传统中,“读书”是___________ ____________________________________________________________________________________________________________________________________________________________________________________‎ 答案 (在中国传统中,“读书”是)指除直观意义上的阅读书籍或学习技能之外,更在于淡化功利性和超越技能学习,用以提高人的精神素养、改变经济对人的支配性影响的一种具有特定含义的学习行为。‎ 解析 以“读书”为开头,把一段话组成一个判断单句,可以看作是给“读书”下定义。先选取“学习行为”作为概念的基本属性,构成单句主干“读书是一种学习行为”;再从各句中筛选有效信息,将“直观意义上的阅读书籍或学习技能”“淡化功利性和超越技能学习”“用以提高人的精神素养”“改变经济对人的支配性影响”“具有特定含义”等,放到主干中充当定语,按顺序排列即可。‎ ‎【整散句变换】‎ ‎13.把下面画线的文字改写成整句的形式,要求与前文语意连贯,构成排比。(可酌情增删个别词语)‎ 黄山云海,时隐时现,模模糊糊,给人一种朦胧之美。有时又虚虚实实、似真似幻、缥缥缈缈、远远近近,带给人们的是幽幽玄妙之感,更有着仙境般的虚无。一片烟云,是诗情,是画意,给人留有无限的遐思。‎ 答:____________________________________________________ ____________________________________________________________‎ 答案 黄山云海,虚虚实实,似真似幻,给人一种幽幽玄妙之感。黄山云海,缥缥缈缈,远远近近,给人一种虚无仙境之谜。‎ 解析 分析画线句,首先确定陈述对象是“黄山云海”。其次根据首句的句式特点,将画线句中与之句式相同的部分抽出,如“虚虚实实、似真似幻”“缥缥缈缈、远远近近”;而句式不同的部分则要仿照首句进行变换,如“带给人们的是幽幽玄妙之感”“更有着仙境般的虚无”可仿照“给人一种朦胧之美”进行变换。‎ ‎14.把下面的语段改写成句式整齐的语句。‎ 岁月轻轻掀起了时光的幕布,翩翩少年如梦的双眸便逝去了天真,蓄满了深沉;岁月把年华的刻刀缓缓一划,婀娜少女的额头由光洁、稚嫩变成了成熟。‎ 答:____________________________________________________ ____________________________________________________________‎ 答案 岁月把时光的幕布轻轻一掀,翩翩少年如梦的双眸便逝去了天真,蓄满了深沉;岁月把年华的刻刀缓缓一划,婀娜少女光洁的额头上便抹去了稚嫩,覆上了成熟。‎ 解析 本题考查变换句式的能力。“句式整齐”的句子即语段对应的语句在字数、结构等方面大致一致。改写时,我们可以以分号为分界点,把前后两个部分对应起来,使前后句子结构一致。比如“岁月轻轻掀起了时光的幕布”“岁月把年华的刻刀缓缓一划”两句,如果以第二句为标准,第一句就可以改为“岁月把时光的幕布轻轻一掀”;也可以以第一句为标准,改写第二句。‎ ‎15.把下面的句子改写成散句。‎ 为了充分发挥群众的积极性和创造性,为了提高工作效益,为了更好地支援农业,我们必须坚决贯彻这一指示。‎ 答:____________________________________________________ ____________________________________________________________‎ 答案 为了充分发挥群众的积极性和创造性,提高工作效益,更好地支援农业,我们必须坚决贯彻这一指示。‎ 解析 本题题干是个排比句,找出其中重复使用的词语“为了”,提取“公因式”,再改写句子。‎ ‎16.阅读下面的文字,请将画线的句子改写成整句。可酌情增删个别词语,但不得改变原意。‎ 正如尼尔·波兹曼在《娱乐至死》一书中说:“对于那些强行禁书的人奥威尔感到很害怕,赫胥黎担心的是失去任何禁书的理由,因为再也没有人愿意读书;奥威尔害怕的是那些剥夺我们信息的人,但人们在汪洋如海的信息中日益变得被动和自私却让赫胥黎甚是担心;奥威尔也害怕真理被隐瞒,赫胥黎却对真理被淹没在无聊烦琐的世事中担心。”‎ 答:____________________________________________________ ____________________________________________________________‎ 答案 奥威尔害怕的是那些强行禁书的人,赫胥黎担心的是失去任何禁书的理由,因为再也没有人愿意读书;奥威尔害怕的是那些剥夺我们信息的人,赫胥黎担心的是人们在汪洋如海的信息中日益变得被动和自私;奥威尔害怕的是真理被隐瞒,赫胥黎担心的是真理被淹没在无聊烦琐的世事中。‎ 解析 语料中画线句子的结构有些混乱,从内容看,是在比较说明“奥威尔”的“害怕”和“赫胥黎”的“担心”。明确了这一点后,考生可将“奥威尔”和“赫胥黎”作为陈述主体,将“害怕的是”和“担心的是”分别作为谓语,这样就很容易将画线的散句改写成句式一致的句子。‎ ‎【句子重组】‎ ‎17.在不改变原意的前提下,将下面这段文字重组为三个句子,每句均以“静”字开头。‎ 中国古典园林不论依傍何种建筑流派,都要以“静”为自己的独特韵律。有了“静”,全部构建会组合成一种古筝独奏般的淡雅清丽;而失去了“静”,它内在的整体风致也就不可寻了。‎ 答:____________________________________________________ ____________________________________________________________‎ 答案 “静”是不论依傍何种建筑流派的中国古典园林独特的韵律;“静”能使全部构建组合成一种古筝独奏般的淡雅清丽;“静”一旦失去,园林内在的整体风致也就不可寻了。‎ 解析 第一分句把“‘静’为自己的独特韵律”作为句子主干,以“中国古典园林”替换“自己”,将“不论依傍何种建筑流派”作为“中国古典园林”的定语即可;第二分句“全部构建”受“静”影响“会组合成一种古筝独奏般的淡雅清丽”,重组可选用使动句,让“全部构建”作兼语,让“会组合成一种古筝独奏般的淡雅清丽”作兼语的谓语部分即可;第三分句只要将“静”置于开头,添加“一旦”等这样的关联词语,将代词替换为其所指代对象即可。‎ ‎18.阅读下面一段文字,用“瑜伽”开头,把这段文字压缩成一个单句。(要求保留主要信息,不超过60个字)‎ 古代印度有一套帮助人类充分发挥自身潜能的健身体系。五千年前,古印度高僧们从观察生物中体悟了不少大自然法则,验证到人的身上,竟然有意想不到的效果。从此,人们开始进行自身健康的维护和调理。经过长期的钻研与归纳,一套完整、实用的养生健身体系诞生了,这就是瑜伽。‎ 答:瑜伽是______________________________________________ ____________________________________________________________‎ 答案 (瑜伽是)古印度高僧把从生物中体悟的大自然法则验证到人身上,用以发挥人的潜能、维护和调理自身健康的一套健身体系。‎ 解析 此题类似于“下定义”,首先应确定单句的主干“瑜伽是……健身体系”,然后从语段中筛选出瑜伽的起源、功用等方面的信息处理成附加成分并合理排序。注意字数限制。‎ ‎19.以“对严寒失去了戒备”为开头,重组下面这个句子,不得改变原意,可以适当增删词语。‎ 如果在各种宣传活动中,专家们不是忙着对暖冬做锦上添花式的一次次论证,而是多一点全面、通俗的科学解释、善意的提醒,大家对今年是“暖冬”的观念也许就不会这样深,以至于对严寒失去了戒备。‎ 答:对严寒失去了戒备__________________________ ____________________________________________________________‎ 答案 ‎ ‎(对严寒失去了戒备)是在专家们在各种宣传活动中忙着对暖冬做锦上添花式的一次次论证,而缺少全面、通俗的科学解释、善意的提醒,导致大家对今年是“暖冬”的观念很深的情况下产生的。‎ 解析 这个句子是一个因果关系复句,前因后果,“对严寒失去了戒备”是果,变换后的句子是前果后因,所以在变换时要把握这一点。‎ ‎【语体变换】‎ ‎20.根据语言环境,用口语方式转述下面书面材料的内容。‎ 材料 ①徐凡,男,江苏南京人,东方大学文学院教授。②系我国红学界后起之秀,③尤以考证作者身世见长。④专著《曹雪芹家世考》《大观园人物论》《〈红楼梦〉导读》(获优秀教育图书奖)在海内外享有盛誉。‎ 要求:①内容适当;②表达得体;③符合口语特点;④将转述的话写在相应的横线上。‎ 学校邀请徐凡与学生座谈时,你以文学社成员的身份向同学们介绍说:______________________________________________________ ____________________________________________________________‎ 答案 徐凡老师是江苏南京人,是东方大学文学院教授。他是我国红学界年轻的研究专家,尤其在考证曹雪芹的身世方面有专长。他的专著《曹雪芹家世考》《大观园人物论》和曾获优秀教育图书奖的《〈红楼梦〉导读》都在海内外享有盛誉。‎ 解析 从语体特点看,②中的“系”是文言语汇,③中的“尤以……见长”为典型的文言句式,④中用括号表示解释,是书面语的写法。从语言环境看,由于是“学校邀请徐凡与学生座谈”,因而①中的“男”无须介绍,学生一看就知;由于是“文学社成员”介绍“教授”,因此①中以“徐凡”直呼其名和②中的“后起之秀”均不妥;另外,③中的“作者身世”意思不明。‎ 转换的要点:①“徐凡”后应加称呼,不能介绍性别;②“系”“后起之秀”必须转换,可以转换为“是”“研究专家”等;③“作者身世”应转换为“曹雪芹的身世”或“《红楼梦》作者身世”,“尤以……见长”用“尤其在……方面有专长”一类句子来转换;④括号中的内容必须转换到句中表述。‎ ‎21.以下是一家公司发布的招聘信息,请将这一信息改写成正式的招聘启事(以“本公司”为开头),要求内容准确、层次清晰、表达得体。不超过75个字(含标点符号,电话号码占两格)。‎ 帅哥靓女,你大学本科毕业不?办公室软件使用熟练不?英语交流顺溜不?有没有驾照?会不会粤语?快来看哦,这儿招人啦!这是个中日韩三国合资公司,马上要在“2010亚运会”举办的地方广州开业咯。现需要行政秘书3名。机不可失,时不再来哦!要是有意,可以电话168168,8月31日面试,海心大厦908,不见不散哦。‎ 答:____________________________________________________ ____________________________________________________________‎ 答案 本公司系合资公司,即将开业,现拟聘行政秘书3名,要求会操作办公软件,会讲粤语,英语口语流畅,有驾照,本科,性别不限,待遇面谈,有意者请拨打168168。‎ 解析 本题考查考生对现有信息进行概括归纳提炼表达的能力。它要求把一段话的主要内容用简明的语言表达出来,本题设置为一则招聘启事,那么在上述要求的前提下还需要注意招聘启事的正确写法。招聘启事正文主要包括用人的单位、部门、业务、目的、对象、条件、待遇、询问事宜、联系办法、联系时间等项内容,要有实事求是的态度,特别是介绍本单位情况和应聘待遇等事项,不要言过其实。一般为表示诚意,待遇最好写清楚,以便应聘者自我衡量。所给材料用语不够庄重,口语化过于明显,宜改为较正式的书面语表达。因为材料未提供待遇问题,且有字数限制,所以,做题时仅需将口语改为书面语并适当调整内容顺序即可。‎ ‎22.请以平实的语言表述下面材料中老园艺师一段话的深层含意,不超过25个字。‎ 夜幕下,小松树上彩灯闪烁。路过的老园艺师自言自语地感慨:“人睡觉要关灯,整夜灯光照着,非失眠不可。树也一样,那么多的电线缠着,那么强的灯光照着,能活得好吗?夜色是美了,树可是要生病的。人们啊,要听得懂草木的叹息!”‎ 答:____________________________________________________ ____________________________________________________________‎ 答案 树也需要良好的生长环境,人类应该善待自然。‎ 解析 这是一道语体转换题,考查的是将形象生动的语言转换为平实语言的能力。要关注“能活得好吗?”“树可是要生病的”“草木的叹息”等形象语言的本义。意思对即可。‎ ‎23.阅读下面的文字,分别用说明性语言、描述性语言解说这段话的意思。‎ 心理学家和生态学家认为,窗外的环境十分重要,如果要让孩子健康,就不要住在六楼以上的楼层。身居高层,空气中含有大量的有害物质,而且整天面对的是砖墙,会引起孩子对外界不由自主的敌意。理想楼层是不超过六楼,窗外环境好,有树,有花草,有水。树和水还是极好的过滤器,能使周围空气很洁净。‎ ‎(1)一般性的说明:______________________________________ __________________________________________________________________________________________________________(40字以内)‎ ‎(2)生动性的描述:_______________________________________ _________________________________________________________________________________________________________(45字以内)‎ 答案 (1)六楼以上空气质量不好,四面砖墙易引起孩子的心理障碍;六楼以下,窗外环境好,空气新鲜。‎ ‎(2)身居高层,受砖墙包围,不能游目骋怀,无益身心;居六楼以下,可与绿树秀水为邻,贴近自然。‎ 解析 本题考查语体的选用。语言从语体风格而言,根据不同的标准,有不同的分类。一般而言,可分为两大类:口语语体和书面语体。在书面语体中,又可分为文艺语体和实用语体。本题所要求的“说明性语言”和“描述性语言”则分别属于实用语体和文艺语体的范畴。‎ 题组3 仿用句式 ‎【命题(话题)式仿写】‎ ‎24.有不少名人的名字来源于古代诗文,请仿照示例,从“刘海粟(国画大师)、王朝闻(著名美学家)、周信芳(京剧艺术大师)、李行健(著名语言学家)”四个名人的名字中任选一个,写出相关的名句并对名字加以合理阐释。不要求句式及阐释角度完全一致,不超过80字。‎ 示例:“呦呦鹿鸣,食野之苹。”著名科学家屠呦呦的名字就来源于此。这个名字让我们可以联想到这位科学家为中医药研究而生的诗意人生。‎ 答:____________________________________________________ ____________________________________________________________‎ 答案 “寄蜉蝣于天地,渺沧海之一粟。”国画大师刘海粟的名字就来源于此。这个名字让我们可以感受到这位大师开阔的胸襟和谦虚的品德。‎ ‎“不吾知其亦已兮,苟余情其信芳。”京剧艺术大师周信芳的名字就来源于此。这个名字使人不由想起这位大师不但演艺精湛而且不流世俗、宁折不弯的高大形象。‎ ‎“朝闻道,夕死可矣!”著名美学家王朝闻的名字就来源于此。这个名字让我们可以感受到这位大师不惜以全部生命追求美学之“道”这一至高境界的决心。‎ ‎“天行健,君子以自强不息。”著名语言学家李行健的名字就来源于此。这个名字可让我们感受到他在学术研究上努力上进、永不停息的精神。‎ 解析 此类仿写在结构上要求不严,注意在文句的内涵上要像,此题注意名字中包含的诗句,然后结合诗句阐释名字的内涵。‎ ‎25.请分别以庄子、屈原、史铁生为对象,仿照下面示例中画线的部分,续写两句话。要求:语意连贯,运用比喻的修辞手法,句式基本一致。‎ 鲁迅是寂寞的,在麻木愚昧的民众中独自彷徨呐喊;但他却在寂寞中找到了民族真正的病根,为昏睡的国人开出了一剂疗救精神的良药。‎ 庄子是寂寞的,在喧嚣的时代里默默修身养性;但他却在寂寞中________________________________________________________________________________,______________________________________ _________________________。‎ 屈原是寂寞的,在世人的不解中独自苦苦寻觅;但他却在寂寞中________________________________________________________________________________,_____________________________________ _____________________________。‎ 史铁生是寂寞的,在病痛的折磨下苦苦挣扎思索;但他却在寂寞中________________________________________________________ ________________________,__________________________________ _____________________________________。‎ 答案 (示例)(庄子)顿悟了逍遥的真谛 为无路可走的人们打开了一扇开向清风明月的窗 (屈原)坚守着高洁忠贞的灵魂 为世人皆醉的社会点亮了一盏上下求索的灯 (史铁生)领悟了人生的意义 为失意消沉的人们找到了一扇通向幸福的门 解析 仿写首先是仿结构,仿句式;其次要仿修辞;最后仿主旨,仿句与原句的格调要一致。解答本题时,首先要明确人物形象;示例中的前一个分句写其所处困境及精神状态,后面画线句语意陡转,表现其在困境中取得人生突破、开创辉煌事业、获得高尚品格、具备深远影响的内容要点,需运用论述的表达方式和比喻的修辞手法。‎ ‎26.以“爱心”为陈述对象,分别以“泉水”“歌谣”为喻体,依照下面的句式再造两个句子,使之构成一组排比句。‎ 爱心是一片照射在冬日的阳光,使贫困交迫的人感到人间的温暖;_______________________________________________________ ________________________________________________________________________________________________________________________‎ 答案 (示例一)爱心是一泓出现在沙漠里的泉水,使濒临绝境的人重新看到生活的希望;爱心是一首飘荡在夜空的歌谣,使孤苦无依的人获得心灵的慰藉。‎ ‎(示例二)爱心是一股甜甜的泉水,使不幸的人忘记生活的苦涩;爱心是一首动听的歌谣,使生活枯燥的人感到生活的乐趣。‎ ‎(示例三)爱心是一眼潺潺流动的泉水,使饥渴的行人感到人生旅途的甘甜;爱心是一首优扬动听的歌谣,使踽踽独行的人感到人生的美好。‎ ‎(示例四)爱心是一股流淌在心头的泉水,使沉浸在痛苦中的人们感到人间的甘甜;爱心是一曲回荡在耳边的歌谣,使孤独的人们感到人间的温暖。‎ 解析 这个题目限制较大,它规定了本体和喻体,要同学们在有限的空间内发挥。同时,还要让再造的句子跟所给的句式相同,才能构成排比。‎ ‎27.请仿照画线的句子,结合课本的内容,在横线处填上恰当的文字,使之构成语意连贯的排比句。‎ 人性是有缺点的:祥林嫂因信奉封建迷信而深受其害,最后在心灵的恐惧和煎熬下死在祝福的爆竹声中;别里科夫________________________________________________________________________________________________________________________________________________;‎ 玛蒂尔德______________________________________________ ____________________________________________________________________________________________。‎ 答案 (示例)(别里科夫)因保守落后而胆小多疑,最后在别人的嘲笑中结束了他卑微的生命 (玛蒂尔德)因爱慕虚荣而付出了巨大的代价,最后在十年的艰苦生活中消耗了她的美丽青春 解析 仿句中涉及名著中的经典人物形象。人物“祥林嫂”是鲁迅的小说《祝福》中的主人公,“别里科夫”是契诃夫的小说《装在套子里的人》中的主人公,“玛蒂尔德”是莫泊桑的小说《项链》中的主人公。三个主人公的共同特点:来自社会底层,愚昧无知或贪图虚荣,最终导致悲惨的命运。考生只要熟知人物,注意句式和修辞手法的一致即可。‎ ‎【自由(开放)式仿写】‎ ‎28.仿照下面的示例,另选对象,另写三个句子,要求蕴含一定哲理,句式与示例相近。‎ ‎(1)蜘蛛:能坐享其成,靠的就是那张关系网。‎ ‎(2)虾:大红之日,便是大悲之时。‎ ‎(3)天平:谁多给一点,就偏向谁。‎ 答:(1)_________________________________________________ ‎ ‎(2)_____________________________________________________‎ ‎(3)_____________________________________________________‎ 答案 (示例)(1)瀑布:因居高临下,才口若悬河。锯子:伶牙俐齿,专行离间之举。(2)气球:只要被人一吹,便飘飘然了。(3)钟表:可以回到起点,却已不是昨天。(4)核桃:没有华丽的外表,却有充实的大脑。(5)指南针:思想稳定,东西再好也不被诱惑。(6)花瓶:外表再漂亮,也掩不住内心的空虚。‎ 解析 可结合生活经验进行总结富含哲理的句子,比如可借助寓言或名人名言,进行改编。‎ ‎29.仿照下面的示例,自选话题,另写三个句子。要求:修辞手法、句式与示例相同。‎ 小草伸出稚嫩的纤手,向你描绘原野的新绿;‎ 树叶掬起温润的阳光,向你展示森林的生机;‎ 溪流吟唱欢快的歌曲,向你诉说春天的故事。‎ 答:____________________________________________________ ____________________________________________________________‎ 答案 (示例)大树敞开宽广的臂膀,向你送去夏日的清爽;花朵露出灿烂的笑脸,向你昭示世界的祥和;山岚秀出曼妙的身姿,向你讲述历史的永恒。‎ 解析 仿用句式讲究“神似”和“形似”,“神似”要求其主旨、情感、语意一致,合乎逻辑;“形似”则注意句式、修辞、字数等相同或相似。例句情感积极向上,呈现一派生机勃勃的景象,运用了拟人的修辞手法、排比的句式,仿句也应如此。‎ ‎30.依照下面的句子,另选话题,写一个句式相同的比喻句。‎ 如果说祖国是一只远航的征船,我的笔就是一支用力向前划的浆。‎ 答:____________________________________________________ ____________________________________________________________‎ 答案 如果说人生是一首悠长的曲子,我的脚步便是一个个跳跃的音符。/如果说理想是遥远的彼岸,我的努力便是一只永远前进的小船。/如果说理想是一只飞翔的鸟,我的行动便是一对顽强拍打的双翅。/如果说理想是一座竦立云霄的塔,我的付出便是一级级坚实的阶梯。/如果说未来是一座大厦,我每天的努力便是建造大厦的一砖一瓦。/如果说大自然是一幅优美的图画,我的心灵便是描绘图画的彩笔。/如果说学校是一棵参天大树,我便是一颗即将成熟的果子。‎ 解析 本题要求用假设复句的形式,打两个相互关联的比喻,其中第一个比喻是第二个比喻得以成立的前提,第二个比喻是第一个比喻的延伸,同时,第二个比喻也是句子的重点。‎ ‎【续写式仿写】‎ ‎31.请以“慈善”为话题,再写两句话,与例句构成排比句,要求内容贴切,句式与例句相同。‎ 慈善是一种美德,当别人遇到困难时,它是融化冰雪的阳光,是消除阴霾的春风;________________________________________;________________________。‎ 答案 慈善是一种胸怀,当别人陷入窘境时,它是缓解干渴的雨露,是滋润心田的清泉 慈善是一种境界,当别人遭遇坎坷时,它是躲避风暴的港湾,是带来慰藉的星光 解析 本题为续写式仿写题,仿写时需要注意把形式和内容两个方面审清楚。基本形式是“慈善是一种……,当别人……时,它是……,是……”;同时要注意这里面包含两个比喻句,比喻一定要贴切。还要注意仿写的语句要与例句构成一个整体。‎ ‎32.仿照下面的示例,另写两句话,要求内容贴切,与示例组成排比句式。‎ 故乡的山,巍峨峻峭,回荡着故乡悠扬舒心的山歌;________,________,________________;________,________,______________________________。‎ 答案 故乡的水,清纯甘甜,养育着故乡憨厚纯朴的儿女;故乡的树,饱经沧桑,遮蔽着故乡饱经风霜的土地。‎ 解析 本题为续写式仿写题,仿写时需要注意把形式和内容两个方面审清楚。还要注意仿写的语句要与例句构成一个整体。‎ ‎33.仿照画线的语句,在空格中续写相关的语句。要求:句式、使用的修辞手法与画线句相同;语句连贯、得体。‎ 小小戏台,把万重关山几度春秋,挤进抑扬顿挫的唱腔里。水袖一甩,舒展几多衷情,抡圆几多冀盼;____________________________。奸诈阴险,在丝弦上慢移;____________________________。‎ 答案 高腔一声,震落几多热泪,撼醒几多慨叹 仁义忠勇,在锣鼓中铿锵 解析 注意分析画线句的句式特点和使用的修辞手法等。第一个画线句,描写的内容是演员舞水袖的样子,那接下来要仿写的语句要可以体现演员的动作、神态、唱腔等;第二个画线句,写的是在戏文表演中演员塑造的舞台形象,要仿写的语句可以表现其他形象的特点等。‎ ‎【嵌入式仿写】‎ ‎34.仿照画线句补写两个句子,构成一组排比句,并保持语意连贯。‎ 一年四季,语文相伴。语文是一位画家,__①__;语文是一位音乐家,“稻花香里说丰年,听取蛙声一片”,她用轻快的旋律奏响夏季农家丰年曲;语文是一位诗人,“晴空一鹤排云上,便引诗情到碧霄”,她用豪放的才情吟诵秋的壮阔;语文是一位摄影家,__②__。‎ 答:①_________________________________________________ ‎ ‎②_____________________________________________________‎ 答案 ①“碧玉妆成一树高,万条垂下绿丝绦”,她用柔和的笔触勾勒出生机盎然的春柳图 ‎②“忽如一夜春风来,千树万树梨花开”,她用奇妙的镜头捕捉到诗意寒冬的美好 解析 本题属嵌入式仿写题,解答时既要仿形式更要仿内涵,形式要严格一致。仿写要注意三点,一是注意句式,二是注意修辞,三是注意字数。‎ ‎35.根据上下文的内容,将下面一个语段补充完整。要求语意连贯、流畅,与上下文句式相近。‎ 有一种前行是为了超越。远离闹市的喧嚣,在危机四伏的荒村野岭、冰川雪峰间孤独前行,遇山跋山,遇水涉水。无论是冰封千里的南北极,还是______________________________________;无论是______________________________________,还是连亘万里的喜马拉雅山之巅高不可测的珠穆朗玛;甚至是那遥远的外太空。只要有未开拓的荒野,行者的脚步不止;只要有未征服的土地,行者的脚步不息。‎ 答案 烈焰万丈的大沙漠 烟波浩渺的太平洋深处深不可及的大海沟 解析 解答此题,首先要抓住核心句子,即首句,它揭示了这段话的中心——前行,超越;其次经过分析,我们会发现所表述的内容都是人类所面对的险境;最后进一步分析两个“无论”,可以找到突破点——要呈现出“险境”的多样化。所以,在写的时候,最好的做法是写一个在特征上与之相反的“险境”。‎ ‎36.根据材料内容,仿照画线的句子,再写两组句子,并与画线的句子构成排比句式。‎ 时间的“边角废料”,往往与一些干扰或诱惑紧密相联。在车上,大家都在说笑,到底参加与否;________,______________________,__________________;__________________,__________________,________________——这里不是性格是否合群的问题,而是对一个人的勇气、毅力和精神的考验。‎ 答案 在会场 大家都在议论 到底聆听与否 在家里 家人都在娱乐 到底加入与否 解析 画线的句子均为简短语句,第一句交代地点或场所,第二句是在这一地点或场所里人们的行为,第三句则是你“到底……与否”的行动,中间的一个动词必须是与这个地点或场所里人们的行为相吻合的。‎ ‎【联偶式仿写】‎ ‎37.[2016·衡水二中周测]趵突泉公园扩建后,有人题写了一副楹联。请用下面的词语组成这副对联。‎ 柳色 泉声 鸣环佩 展画屏 入耳 迎眸 答:____________________________________________________ ____________________________________________________________‎ 答案 泉声入耳鸣环佩 柳色迎眸展画屏 解析 对联写作上要做到词性相同,结构一致;对应工整,音韵协调;内容相关,意境完美。本题所给词语,“柳色”与“泉声”相对,“鸣环佩”与“展画屏”相对,“入耳”与“迎眸”相对,考生组合时,只需要做到词性相对,意思通顺,意境优美即可。‎ ‎38.仿照下面的例句,在横线上填上恰当的文字。要求:引用恰当的古诗文,句式一致,内容积极。‎ 何谓真正的“白富美”?身为女子,“出淤泥而不染”,做到洁身自好,是为“白”;“腹有诗书气自华”,做到饱读诗书,是为“富”;“一片冰心在玉壶”,做到心清如水,是为“美”。‎ 何谓真正的“高富帅”?身为男子,______________________________________,是为“高”‎ ‎;________________________________________,是为“富”;____________________________,是为“帅”。‎ 答案 “不坠青云之志”,做到志存高远 “天生我材必有用”,做到自信满满 “谈笑间,樯橹灰飞烟灭”,做到从容淡定 解析 基本句式为“‘……’,做到……”;引用古诗文句要恰当,与“做到”后的内涵一致;内容要积极乐观、健康向上。‎ ‎39.以下是某中学庆祝教师节文艺演出的一段主持词。仿照画线部分的句式,在空缺处补写相应的语句。要求:句式一致,字数相等,语意相关。‎ 学生甲:老师,您坚守一方净土,用粉笔书写忠诚,默默无闻;‎ 学生乙:老师,您勤耕三尺讲台,__①__。‎ 学生甲:加减乘除,算不尽您付出的辛劳;‎ 学生乙:__②__。‎ 答:____________________________________________________ ____________________________________________________________‎ 答案 (示例一)①用汗水浇灌希望,孜孜不倦 ②诗词歌赋,颂不完您带来的感动 ‎(示例二)①用双手托起未来,无怨无悔 ②赤橙黄绿,画不尽您多彩的人生 解析 解答此题的关键是明确例句中的隐性条件。如第一处画线句中的“默默无闻”与“坚守”照应,所以①处仿写的语句中所用的成语也应与“勤耕”照应。第二处画线句中的“加减乘除”是数学学科中的用语,因此,②处的仿句也应是某一学科的内容。‎ ‎40.根据上下文,拟出楹联的下联。‎ 惠州西湖丰渚园内的多对楹联使游人感受到浓厚的文化氛围,在欣赏美景的同时陶冶性情。大门楹联“门倚东江遥接罗浮紫气,________________”,寥寥二十字,把丰渚园的地理位置和惠州的主要名胜古迹概括其中。上联写罗浮山,下联写泗时塔,一“遥”一“近”,一“倚”一“依”,写出了物华天宝聚惠州,紫气彤云映丰渚的特色。‎ 答案 (示例一)园依西湖近连泗时彤云 ‎(示例二)轩依荷淀近连玉塔彤云(“园”还可换成“轩”“庭”“榭”等单音词。“西湖”可换用“平湖”“荷淀”等表示地物、景色的双音词。“泗时塔”又称“玉塔”)‎ 解析 需根据上下文的提示拟下联。下联中要有“近”“依”“彤云”“泗时”等词语。‎ ‎ [3年高考真题集训]‎ ‎1.[2016·浙江高考]填入下面空缺处的语句,最恰当的一项是(  )‎ 贾母因要带着刘姥姥散闷,遂携了刘姥姥至山前树下盘桓了半晌,又说与他这是什么树,这是什么石,这是什么花。刘姥姥一一的领会,又向贾母道:“谁知城里不但人尊贵,连雀儿也是尊贵的。____________。”众人不解……刘姥姥道:“那廊上金架子上站的绿毛红嘴是鹦哥儿,我是认得的。那笼子里黑老鸹子,怎么又长出凤头来,也会说话呢。”众人听了,又都笑将起来。‎ A.这雀儿到了你们这里,变得既俊又会说话了 B.偏这雀儿到了你们这里,他会说话了,也变俊了 C.偏这雀儿到了你们这里,他也变俊了,也会说话了 D.这雀儿到了你们这里,变得既会说话又俊了 答案 C 解析 根据下文中“怎么又长出凤头来,也会说话呢”可知,是先“变俊”后“会说话”,故排除B、D。根据刘姥姥的身份特点,“既……又……”这样较为书面的语言风格与刘姥姥作为下层劳动人民的身份不符;同时,“偏这雀儿到了你们这里”的言外之意是雀儿到其他地方不会这样,这更能衬托、突出贾府家的富贵、繁华,而“这雀儿到了你们这里”则没有这种表达效果,故排除A。‎ ‎2.[2015·江苏高考]下列诗句中,没有使用比拟手法的一项是(  )‎ A.东风便试新刀尺,万叶千花一手裁。‎ B.浮萍破处见山影,小艇归时闻草声。‎ C.有情芍药含春泪,无力蔷薇卧晓枝。‎ D.唯有南风旧相识,偷开门户又翻书。‎ 答案 B 解析 A项,以“试”“裁”赋予“东风”人的动作,是拟人的修辞手法。B项,使用了对偶的修辞手法。C项,“有情”“含春泪”“无力”“卧”,以人的情态、动作写“芍药”“蔷薇”,是拟人的修辞手法。D项,以“开门”“翻书”赋予“南风”人的动作,是拟人的修辞手法。‎ ‎3.[2015·福建高考]阅读下面的文字,请将画线的句子改写成排比句。(要求:不得改变原意)‎ 焦裕禄是闻名全国、感动全国的“县委书记的好榜样”。他在兰考县忘我奋斗一年零五个月,积劳成疾,英年早逝。焦裕禄是为人民拼死拼活谋福祉的领导干部的优秀代表;为信仰无怨无悔做奉献,成为共产党人的光辉典范;在为祖国艰苦奋斗创新功的时代里,他是那一代人的精神符号。人民是不会忘记焦裕禄的。‎ 答:____________________________________________________ ____________________________________________________________‎ 答案 (示例)是为人民拼死拼活谋福祉的领导干部的优秀代表,是为信仰无怨无悔做奉献的共产党人的光辉典范,是为祖国艰苦奋斗创新功的那一代人的精神符号。‎ 解析 改写成排比句,需要我们先选准一个基准句子,再用这种结构调整其他各个句子。这里可以以“是为人民拼死拼活谋福祉的领导干部的优秀代表”为基准句子改写。‎ ‎4.[2015·湖北高考]仿照示例,从下列构字部件中任选两个不同的部件,另组一个汉字,并用该字组成一个双音词,再以该词立意,写两句感想。要求:①立意积极;②语句整齐;③不超过20字;④构字、组词不必单独列出,包含于感想之中即可。‎ 构字部件:禾 亻 口 日 又 月 言 ‎ 构字词示例:+又→友→友善 感想示例:行友善营造温馨社会 讲真诚建设美好家园 答:____________________________________________________ ____________________________________________________________‎ 答案 (示例一)(日+月→明→文明)‎ 创文明社会 建和谐中国 ‎(示例二)(禾+日→香→书香)‎ 博览群书通晓天下事理 沐浴书香养成高尚人格 解析 先分析试题所给示例的构思特点:由构字部件联想到汉字,再由汉字联想到词语,最后以这个词语为核心写出立意积极、句式整齐的两句话。再按照题目要求,依顺序一步一步完成答案。做题时要注意题干中“两个不同的部件”“一个汉字”“一个双音词”等关键信息。‎ ‎5.[2015·安徽高考]某校拟开展以“自然·青春·团队”为主题的郊游活动,全校同学将以班级为单位参与。请撰写本班活动标语。要求:紧扣主题,语言鲜明、生动,至少使用一种修辞手法,不超过16字。‎ 答:‎ 答案 (示例一)郊游放飞青春梦,合作凝聚团队魂 ‎(示例二)让我们在自然中放飞青春的梦想 解析 注意题目要求,紧扣活动主题“自然·青春·团队”,在撰写的标语里要有能体现这三个关键词的因素。一般使用或比喻或拟人或排比的修辞手法,即可达到语言鲜明、生动的目的。注意字数限制。‎ ‎6.[2015·四川高考]‎ 创业、创新是时代的呼唤。请拟写一则宣传语,倡导同学们升入大学后积极参与创业、创新活动,培养创业、创新能力。要求:①紧扣宣传目的;②运用比喻手法;③不超过40字。‎ 答:____________________________________________________ ________________________________________________________________________________________________________________________‎ 答案 (示例)创业是青春的火炬,创新是创业的翅膀,用创业点燃创新梦想,用创新放飞人生希望。‎ 解析 要仔细阅读题目,注意“两暗三明”五点要求,即所写答案必须符合宣传语的要求,要体现导向性、鼓动性、激励性,语言要积极昂扬;所写的宣传语,可能是几句话,但内容必须连贯、集中;紧扣宣传目的,即鼓励学生积极参与创业、创新活动,提高创业、创新能力;运用比喻手法,则需要想好喻体与本体;不超过40字的要求,一般需要写到30字以上,不宜过少。‎ ‎7.[2014·重庆高考]仿照示例,自选话题,另写一个句子。要求:与示例结构基本相同,修辞手法一致;个别词语可与示例重复,字数也可略有增减。‎ 示例:就像穿衣服扣扣子一样,如果第一粒扣子扣错了,剩余的扣子都会扣错;人生的扣子从一开始就要扣好。‎ 仿句:________________,________________,________________;________________。‎ 答案 (示例)就如建房子打地基一样 如果第一层地基打歪了 上面的楼层都会歪斜 人生的根基从一出生就要打牢 ‎[2年全国模拟重组]‎ ‎1.[2017·天津南开中学摸底]填入下面一段文字横线处的语句,与上下文衔接最恰当的一句是(  )‎ 这是我们的时代。我们的时代,是百花齐放的时代,我们不但要盈亩满畦的牡丹和菊花,我们也要树下的紫罗兰,草地边的蒲公英。________。我们的责任是不但让读者能兼收并蓄,而且还可以各取所需。‎ A.世界上没有不爱花卉的人,但是每人的爱好又是多种多样的 B.世界上没有不爱花卉的人,但是每人的爱好不尽相同 C.因为每种花都有生存的权利,人们既爱牡丹、菊花,又爱紫罗兰、蒲公英 D.因为每种花都有生存的权利,那么每位读者也有选择的权利 答案 B 解析 横线前的语句中有“我们不但要……”“我们也要……”,表明是从人们的需要出发的。横线后的语句是对整体内容的概括与提升,其中有“不但让读者能兼收并蓄,而且还可以各取所需”,看来,“各取所需”是此段内容的核心。选项中C、D两项的着重点在“花”(因为每种花都有生存的权利),而非“人”,因此可首先排除。A项中“每人的爱好又是多种多样的”强调的是个人的爱好不止一个,而B项则强调了人与人之间爱好的不同,与语段“各取所需”之意一致。‎ ‎2.[2017·保定十校联考]填入下面一段文字横线处的语句,最恰当的一句是(  )‎ 人民教育家陶行知提出“大学之道,在明民德,在亲民,在止于人民之幸福”。陶先生继承经典文化之精髓而更新两点,将大学对人民的关怀视为现代大学理念的根本。21世纪国际化背景下的大学新使命是什么?在新的时代背景下,世界各国大学的领导者不约而同地提出了大学的新功能:________。‎ A.大学不但要承担人才培养、科学研究、社会服务这些基本职能,而且应实现人类文明的交融和传承,积极发挥文化传承与创新的作用 B.大学除了承担人才培养、科学研究、社会服务这些基本职能之外,还应积极发挥文化传承与创新的作用,实现人类文明的交融和传承 C.大学应在承担人才培养、科学研究、社会服务这些基本职能和积极发挥文化传承与创新的作用、实现人类文明的交融和传承这两方面并重 D.大学除了承担人才培养、科学研究、社会服务这些基本职能之外,还应实现人类文明的交融和传承,积极发挥文化传承与创新的作用 答案 B 解析 首先根据语境,侧重点应该落在“新”字上,不应该“新、旧(基本职能)”并行,所以C项应该排除;然后再看“实现人类文明的交融和传承”“积极发挥文化传承与创新的作用”这两句话的逻辑顺序,“发挥……作用”是手段,“实现……传承”‎ 是结果,而A、D两项都把这种逻辑关系颠倒了,所以排除A、D两项。答案为B项。‎ ‎3.[2017·江西三校联考]填入下面一段文字横线处的语句,最恰当的一句是(  )‎ 拆围过后的洪湖生态环境依旧脆弱,为了使被破坏的植被和鱼类资源尽快恢复,一场生态恢复战渐次打响。洪湖湿地自然保护区管理局新堤保护站副站长曾祥新说,保护区一方面移植水草,进行生态恢复;另一方面,________。此外,每年还设置休渔期、人工投放鱼苗等,促进水生物种休养生息。‎ A.将长江中丰富的鱼类资源带进来,在鱼汛期将长江大闸打开,使江水与湖水相互置换 B.使江水与湖水相互置换,在鱼汛期将长江大闸打开,将长江中丰富的鱼类资源带进来 C.在鱼汛期将长江大闸打开,使江水与湖水相互置换,将长江中丰富的鱼类资源带进来 D.将长江大闸打开,在鱼汛期将长江中丰富的鱼类资源带进来,使江水与湖水相互置换 答案 C 解析 “一方面”与“另一方面”属于并列的关系,应该按照先行动后目的的逻辑关系组织语言,“在鱼汛期将长江大闸打开”是一个行动,目的是通过江水和湖水的相互置换,把长江中的鱼类资源带进来,故C项正确。‎ ‎4.[2017·揭阳一中模拟]填入下面横线上的语句,与上下文衔接最恰当的一项是(  )‎ 文学,你是什么?你是一个忠实的朋友,你只是为热爱你的人们默默奉献,________。只要愿意和你交朋友,你就会毫无保留地把心交给他们。‎ A.让他们懂得人生的真谛,并把他们引入博大精彩的世界,让他们看到世界上最奇丽的风景 B.让他们看到世界上最奇丽的风景,让他们懂得人生的真谛,并把他们引入博大精彩的世界 C.让他们看到世界上最奇丽的风景,并把他们引入博大精彩的世界,让他们懂得人生的真谛 D.把他们引入博大精彩的世界,让他们看到世界上最奇丽的风景,让他们懂得人生的真谛 答案 A 解析 “把他们引入博大精彩的世界”后面一定要接“让他们看到世界上最奇丽的风景”,这两个分句之间是承接关系。由此可以排除B项和C项。D项的排列顺序把“让他们看到世界上最奇丽的风景”与“让他们懂得人生的真谛”看成并列关系了。其实文学的第一功能就是“让他们懂得人生的真谛”,“把他们引入博大精彩的世界,让他们看到世界上最奇丽的风景”是文学的另一功能,两种功能是并列关系,所以A项正确。‎ ‎5.[2017·甘肃重点中学调研]填入下面这段文字横线处的语句,衔接最恰当的一句是(  )‎ 何谓“角色之衡”?________,这是由舞台艺术的整体性和综合性决定的。由于天赋、机缘、人气等各方面的限制,能成头牌的毕竟是极少数。因此,在人才培养上既要勇于“开小灶”,为有潜力成为金字塔顶端的演员创造更多学习、实践和拜师的机会,同时,也要为二三线配角演员的培养制订扎实可行的计划。‎ A.戏曲需要的不仅仅是作为四梁八柱的相对整齐的配角,还需要一两个站在舞台中央的主角 B.戏曲需要的不仅仅是一两个站在舞台中央的主角,还需要作为四梁八柱的相对整齐的配角 C.戏曲需要的不是一两个站在舞台中央的主角,而是需要作为四梁八柱的相对整齐的配角 D.戏曲需要的不是作为四梁八柱的相对整齐的配角,而是需要一两个站在舞台中央的主角 答案 B 解析 根据文意,阐述“主角”的句子应在前,阐述“配角”的句子应在后,而且是递进关系。所以答案选B项。‎ ‎6.[2016·银川三模]依据相关内容将下面的三副对联补充完整,最恰当的一项是(  )‎ 乔迁新居:画栋倚云呈异彩,__①__。‎ 喜结良缘:__②__,丽日池边并蒂莲。‎ 金榜题名:__③__,__④__。‎ A.①花灯映月放光辉 ②春风堂上双飞燕 ③蟾宫折桂立苍穹 ④北斗凌云横碧水 B.①花灯映月放光辉 ②春风堂上双飞燕 ③北斗凌云横碧水 ④蟾宫折桂立苍穹 C.①春风堂上双飞燕 ②花灯映月放光辉 ③北斗凌云横碧水 ④蟾宫折桂立苍穹 D.①春风堂上双飞燕 ②北斗凌云横碧水 ③花灯映月放光辉 ④蟾宫折桂立苍穹 答案 B 解析 依据对联的主题和平仄要求判定。‎ ‎7.[2017·郑州实验高中质检]仿照示例,自选话题,另写两句话,要求使用拟人的修辞手法,句式与示例相同。‎ 雨借着芭蕉或屋瓦,敲打出柔肠寸断或满怀激情。‎ 答:____________________________________________________ ________________________________________________________________________________________________________________________‎ 答案 风借着檐铃或波涛,流转出浅吟低唱或澎湃激昂。云借着明月或山雨,幻化出旖旎缠绵或豪情奔放。‎ 解析 本题考查仿用句式、正确使用常见的修辞手法的能力。仿写时,注意句式、修辞手法、感情色彩的一致性。‎ ‎8.[2017·西安中学质检]仿照示例,自选话题,另写两句话,要求使用拟人的修辞手法,句式与示例相同。‎ 梅花在冰天雪地的季节吐蕾,意在教导我们:学会坚强;‎ 昙花于万籁俱寂的深夜绽放,意在提醒我们:不要张扬。‎ 答:____________________________________________________ ________________________________________________________________________________________________________________________‎ 答案 花瓣在生命旺盛的初夏凋零,意在教导我们:学会放下;‎ 树叶于五彩斑斓的深秋飘落,意在提醒我们:不要逞强。‎ 解析 本题考查仿用句式的能力。题干要求“自选话题,另写两句话”“用拟人的修辞手法,句式与例句相同”,要求比较宽泛。答题时注意修辞和句式,思想积极健康向上即可。‎ ‎9.[2016·衡水联考]仿照下面的示例,自选话题,另写一组句子,要求所写句子与示例使用的修辞手法相同,句式基本一致。‎ 什么是最真挚的友谊?是华美盛宴的觥筹交错,是形影不离的朝夕相守,是依依难舍的缠绵缱绻,还是困境中搀扶你一把的那双温暖的手?‎ 答:____________________________________________________ ________________________________________________________________________________________________________________________‎ 答案 什么是最美丽的风景?是画家笔下的云霞,是缥缈的海市蜃楼,是雨后天晴的彩虹,还是故乡那平淡的山水?‎ 解析 要求如下:句式一致、手法相同、内容相关。示例是一组选择疑问句,所仿写的句式应是“什么是最……?是……,是……,是……,还是……?”所仿写的句子应与示例句式相符,是一组选择问句,语句通顺且合乎逻辑,最后一句升华立意。‎ ‎10.[2016·福建四地六校联考]阅读下面这首小诗,仿照内容和形式再续写一节。‎ 当一颗种子发芽的时候  __________________‎ 别忘了 __________________‎ 你周围的阳光空气和土壤 __________________‎ 在你成长的路上 __________________‎ 学会悄悄将她们珍藏 __________________‎ 答案 (示例)当一棵树木结果的时候/别忘了/春夏秋冬的雨雪风霜/在你蓬勃向上时/学会静静地驻足回望 解析 要点:内容为感恩,形式基本一致,语言贴切优美。‎ ‎11.[2016·西北师大附中诊断]备受关注的2016年《中国汉字听写大会》即将开始,请你为这届活动拟写对联式宣传语。要求根据提供的上联,补写下联。‎ 上联:汉字听写传承华夏文明,彰显民族正能量 下联:__________________________________________________ ‎ 答案 文化传播维系炎黄子孙,践行核心价值观 解析 符合对联格式,内容必须和民族文化相契合,格调积极向上。‎ ‎12.[2016·江西上高二中模拟]仿照下面画线句子的格式,运用对偶和引用的修辞手法,在横线上续写一段话。‎ 动车行至安徽,有这样一段广播稿:‎ 各位旅客:从沃土千里的淮北平原到群山连绵的皖南山区,风光秀美,历史悠久。黄山松奇绝天下,采石矶雄踞大江。曹操父子诗兴盎然,徽州商人叱咤风云。前人曾感叹,“一生痴绝处,无梦到徽州”……现在,你已经随着动车驶出了安徽大地。……‎ 接下来前方到达的是被誉为革命圣地的江西,这里山川形胜,人文荟萃。__________________,__________________。__________________,__________________。__________________。‎ ‎……现在,你已经随着动车进入了赣鄱大地。‎ 答案 (示例一)康庐白雾千古缥缈 井岗翠竹天下奇秀 隐士渊明采菊东篱 革命先辈起义南昌 王勃曾赞叹,“物华天宝,人杰地灵”‎ ‎(示例二)鄱阳湖候鸟壮观天下 婺源山绿茶香飘万里 宋应星奇思构就《天工开物》 汤显祖妙想“临川四梦” 苏轼曾感叹,“不识庐山真面目,只缘身在此山中”‎ 解析 要点:①内容要切合江西的自然风光和人文特点;②形式要符合对偶和引用的要求。‎ ‎13.[2016·云南师大附中适应性考试]把下面这个长句改写成几个较短的句子,可以改变语序,合理增删词语,但不得改变原意。‎ 金砖国家为了自己及其他新兴市场和发展中国家的基础设施、可持续发展等项目筹措资金,遵循维护金融主权、合作促发展、公平互利的原则,于2014年7月在巴西峰会上宣布签署成立金砖国家开发银行协议。‎ 答:____________________________________________________ ________________________________________________________________________________________________________________________‎ 答案 2014年7月在巴西峰会上,金砖国家宣布签署成立金砖国家开发银行协议,目的是为自己及其他新兴市场和发展中国家的基础设施、可持续发展等项目筹措资金,金砖国家开发银行的运行遵循维护金融主权、合作促发展、公平互利的原则。‎ 解析 这句话的中心意思是①“金砖国家宣布签署成立金砖国家开发银行协议”,②目的状语“为了自己及其他新兴市场和发展中国家的基础设施、可持续发展等项目筹措资金”,③遵循原则“遵循维护金融主权、合作促发展、公平互利的原则”,三个句子依次写出,句子要通顺,可以增删词语。‎ ‎14.[2017·烟台自主练习]用排比的修辞手法,改写下面画线部分。要求:可以增删词语,但不得改变原意。‎ ‎“一片丹心向阳开”阎肃艺术成就研讨会15日下午在京召开。来自文艺界、军政界的一百多位代表参加了会议。代表们深情回顾了阎肃在重大文艺活动创排策划中的突出表现和独特建树,高度评价了阎肃在65年艺术生涯中所表现出的时代精神。‎ 大家一致认为,阎肃同志是红心向党、追梦筑梦的忠诚战士;阎肃同志勇立潮头、奋斗不息,堪称时代先锋;阎肃同志是一个文艺标兵,他以服务部队、奉献社会为宗旨;他一向严于律己,是德艺双馨的道德楷模。与会代表一致表示,要以阎肃同志为榜样,高唱正气歌,齐奏时代颂,筑牢民族魂,振奋精气神,为建设社会主义文化强国贡献力量。‎ 答:____________________________________________________ ________________________________________________________________________________________________________________________‎ 答案 阎肃同志是红心向党、追梦筑梦的忠诚战士,是勇立潮头、奋斗不息的时代先锋,是服务部队、奉献社会的文艺标兵,是严于律己、德艺双馨的道德楷模。‎ 解析 本题要求用排比的句式表达句子,如此可以先确定句子主语是“阎肃同志”。整理整句话中的主要内容,使其表达一致,如此可以筛选出“红心向党、追梦筑梦的忠诚战士”“‎ 勇立潮头、奋斗不息的时代先锋”“服务部队、奉献社会的文艺标兵”“严于律己、德艺双馨的道德楷模”。‎ ‎15.[2017·清远段考]把下面的这个长句改写成几个较短的句子,可以改变语序、增删词语,但不得改变原意。‎ 总结是一个组织或个人在工作、学习告一段落,进行回顾、检查、分析和评价,从中找出成功的经验或失败的教训,悟出个中的道理,得出规律性的认识,并用以指导今后的工作而写成的书面材料。‎ 答:____________________________________________________ ________________________________________________________________________________________________________________________‎ 答案 总结是一个组织或个人在工作、学习告一段落后写的书面材料,它常常对前一阶段的情况进行回顾、检查、分析和评价,以利于找出成功的经验或失败的教训,悟出个中的道理,得出规律性的认识,并用以指导今后的工作。‎ 解析 首先找出句子主干,然后剥离原句修饰成分,形成数个短句即可。‎ ‎16.[2017·北大附中河南分校月考]下面句子是对黄河壶口瀑布的描写,适当调整画线部分的语序,把它改写成格式协调一致、匀整对称的排比句。(可以根据题目要求改变个别词语。)‎ 正当平缓似锦缎的黄河得意之时,她突然以数里之阔的水面,跌入百尺之宽的峡谷。发出了震川虎啸般的声响,悬垂的水流如张挂的天幕,黄色的浊流如腾飞的巨龙,激起的水珠如钢花四溅,使每一位风尘仆仆走近她的游客都受到了强烈的震撼。‎ 答:____________________________________________________ ________________________________________________________________________________________________________________________‎ 答案 发出的声响如震川的虎啸,奔涌的浊流如腾飞的巨龙,悬垂的水流如张挂的天幕,激起的水珠如四溅的钢花 解析 对这道题我们可以从两个层面辨析,第一个层面是短语内部,一般表达要求是格式协调一致、句式匀整对称。据此,发现这段话的四个短语,有三个用“如”作比喻词的比喻句,如果把另一个也改造成这样的比喻句,就可以使四个短语排列在一起,整齐匀称。再分析,这几个比喻句的主体都是名词性短语,其定语是动词,喻体也是名词性短语,其定语也是动词,据此,可以把“发出了震川虎啸般的声响”改为“发出的声响如震川的虎啸”,把“黄色的浊流”改为“奔涌的浊流”,把“钢花四溅”改为“如四溅的钢花”,几个句子内部结构的语序就一致了。第二个层面是依据题目中“风尘仆仆走近她”的提示,把四个短语的顺序按照由远及近或者由闻声到见形的逻辑顺序加以调整,即“发出的声响如震川的虎啸,奔涌的浊流如腾飞的巨龙,悬垂的水流如张挂的天幕,激起的水珠如四溅的钢花”。‎ ‎17.[2016·衡水中学期中]阅读下面一段文字,调整画线部分语序,要求句式更协调一致,语句顺序更加合理。‎ 纵观一部宋词史,从北宋到南宋,宋代词人都是一群心怀家国情结的时代精英,也是一群个人情怀拳拳的赤子,无论是怜悯民生的郁郁哀歌,还是深长呼唤的惋惜国运;无论是报效祖国的慷慨歌吟,还是娓娓诉说的个人际遇,感情都是如此投入,声出肺腑,情动于衷,令人动容。‎ 答:____________________________________________________ ________________________________________________________________________________________________________________________‎ 答案 无论是报效祖国的慷慨歌吟,还是惋惜国运的深长呼唤;无论是怜悯民生的郁郁哀歌,还是个人际遇的娓娓诉说 解析 “慷慨歌吟”对“深长呼唤”,“郁郁哀歌”对“娓娓诉说”;注意语段第一句的总说暗示,由北宋到南宋,由民生到个人,答题时必须考虑这两个因素。‎ ‎18.[2017·大同五校联考]米兰·昆德拉说:“生命是一棵长满可能的树。”由于性格、经历和生活环境的不同,每个人对生命都有自己独到的理解。请你从以下作家或文学人物中,任选两位仿照示例各写一句话,表达对生命的理解。要求符合人物特征,句式大致相同。‎ 人物:杜甫 鲁迅 贝多芬 桑地亚哥 示例:苏东坡——生命就是一次风雨兼程的旅行,只有心胸旷达,才能笑到终点。‎ 答:____________________________________________________ ________________________________________________________________________________________________________________________‎ 答案 贝多芬——生命就是一次不惧磨难的历程,只有心中有目标,才能取得成就。‎ 桑地亚哥——生命就是一场单枪匹马的战斗,只有先战胜自己,才能战胜对手。‎ 解析 注意分析例句结构、内容等,尤其注意关联词语的使用,还要注意内容上的特点等。句式上是“生命是……,只有……才……”的形式,这个不难模仿,难的是结合人物的经历、性格或其精神世界的核心对生命做出诠释。好在备选的名人均出自教材,同学们对他们还算熟悉。‎ ‎19.[2017·青岛一中检测]调整下面语段中画线的句子,使整个语段合乎逻辑顺序、表达和谐一致。‎ 人们在同样的时间里奔跑,错过了稻禾沾满金露、树木寄走枯叶的秋;夏天,浪花裂开心花,荷盖展开青霞,错过了;山坡覆盖白雪、水面凝成银冰的冬,错过了;错过了桃花送走雪花的春,错过了春风唤醒田蛙的春。人们应该在这样的季节、这样的景色中踱步,让一个个脚印有翡翠的韵脚、金银的注释。这才是绿色的生活、生动的世界。‎ 答:____________________________________________________ ________________________________________________________________________________________________________________________‎ 答案 错过了桃花送走雪花、春风唤醒田蛙的春,错过了浪花裂开心花、荷盖展开青霞的夏,错过了稻禾沾满金露、树木寄走枯叶的秋,错过了山坡覆盖白雪、水面凝成银冰的冬。‎ 解析 本题画线句子的内部逻辑联系是时间顺序。表达的和谐是指句子结构应一致,加之全段的主语是“人们”,所以应用“错过了……”的句式。‎ 考点六 语言表达简明、连贯、得体、准确、鲜明、生动 考点名片 考点内容 ‎①常单独命题考查“连贯”;②个别单独命题考查“得体”;③“生动”多与其他考点如“仿用句式”“修辞”结合在一起考查;④“简明”“准确”“鲜明”“连贯”往往与其他考点综合考查。‎ 考查形式 ‎①试题材料贴近现实生活,贴近考生实际;②多以单选题形式考查“连贯”或“得体”;③以简答题形式综合考查“简明”“得体”等。‎ 趋势分析 ‎①“连贯”应为全国卷必考点;②“简明”的考查应引起重视;③以简答题形式与应用文、微写作等形式综合考查该考点。‎ 题组1 连贯类 ‎【句子补写题】‎ ‎1.在文字横线处补写恰当的语句,使整段文字语意完整连贯,内容贴切,逻辑严密。每处不超过20个字。‎ 我校曾推行小班化课堂改革实践,要求教师尝试把课堂学习权还给学生,少教多学,少讲多练,先学后教,以学定教,结果引来一片质疑和阻拦。__①__。有一些家长在论坛发帖,并联名到学校来阻拦。我们坦率面对,主动邀请意见强烈的家长前来沟通。有家长质问说:你们这不是把我们的孩子当小白鼠吗?__②__。有更多的家长则质疑:“既然课改那么好,为什么其他学校不搞改革,为什么许多名校不搞改革?”‎ 实际上,我们教育工作者队伍中,转变思想、理念和作风,接受新的变化和革新,__③__。据说,不少学校课改项目遭到家长和社会反对,推行不下去,主要是一部分思想还没有想通的教师从中发挥了负面作用。还有的则是一些教育行政部门领导的顾虑重重,畏缩不前,导致校长和老师们不敢“轻举妄动”。‎ ‎(节选自《中国日报》)‎ 答:①__________________________________________________ ‎ ‎②_____________________________________________________‎ ‎③_____________________________________________________‎ 答案 ①有的来自内部,有的来自家长 ②我们的孩子不允许做你们课改的牺牲品 ③比起家长来,还要艰难和被动得多 解析 ①处,应承接“结果引来一片质疑和阻拦”,根据文意,应从学生和家长两方面来概述;②处,联系“小白鼠”可联想到是做实验的内容;③处,结合上下文,不难得出答案。‎ ‎2.在下面一段文字横线处补写恰当的语句,使整段文字语意连贯,内容贴切,逻辑严密。每处不超过15个字。‎ 每个人在自己的工作、社交、业余生活、饮食起居诸方面,__①__。细想想,这些习惯无一不是一点一滴慢慢形成的。每个人都在不自觉地按着自己的习惯行事,__②__,坏的习惯带来不好的结果。养成良好的习惯,对每个人来说都是一件很重要但也是很困难的事情,不同只在于坏习惯的形成往往是不知不觉的,好习惯的形成则需要通过较长时间的主观努力。坏习惯形成了,改掉颇不易;好习惯形成了,一旦松懈下来,__③__。我们应该明白习惯对人生成功的意义,并学会用好习惯的力量成就自己的一生。‎ 答:①__________________________________________________ ‎ ‎②_____________________________________________________‎ ‎③_____________________________________________________‎ 答案 ①都有自己的习惯 ②好的习惯带来好的结果 ③却比较容易丢掉 解析 第①处结合前文内容以及后文内容“这些习惯”,此处意思应是“都有自己的习惯”;第②处结合后文“坏的习惯带来不好的结果”,此处意思应是“好的习惯带来好的结果”;第③处联系前文内容“坏习惯形成了,改掉颇不易”,此处意思应是“却比较容易丢掉”。‎ ‎3.在下面一段文字横线处补写恰当的语句,使整段文字语意完整连贯,内容贴切,逻辑严密。每处不超过20个字。‎ 中国是一个农业国,自古以来人们依土地而生,自然的山山水水养育了我们,所以__①__,追求人与自然的和谐。山坚毅不拔,沉静、博大、深厚;水至柔至利,屈曲有致,既可润物无声,又能摧枯拉朽。山的阳刚与水的阴柔相生相合,便是宇宙无限的生机。当春风吹来的时候,__②__;当夏天来临的时候,人在自然生命的苍翠中感到生命的蓬勃;秋景的宁静疏落,让人体会收获过后的宁静和天高云淡的旷远;而冬日的萧瑟空寂,让人感受到了生命的艰难和顽强。人在山川的变化中,找到了自己情绪的寄托,获得精神的安顿。因此,__③__,望水则情溢于水。‎ 答:①__________________________________________________ ‎ ‎②_____________________________________________________‎ ‎③_____________________________________________________‎ 答案 ①我们要尊重、热爱自然 ②人在自然生命的萌动中感到美好的希望 ③观山则情满于山 解析 由第①处横线上文讲的是大自然施给人类的恩惠,下文讲的是人类的行为,可知此处也应讲人类的行为。第②处横线,上文讲的是春天,下文讲的是夏天时人的感受。故此处也应写春天时人的感受。第③处横线,上文讲的是人的情绪,下文写的是人在“望水”时的情感,故此处应写人在“看山”时的情感。只要符合语境即可。‎ ‎4.在下面一段文字横线处补写恰当的语句,使整段文字语意完整连贯,内容贴切,逻辑严密。每处不超过15个字。‎ 柠檬是世界上最有药用价值的水果之一,它富含维生素C、糖类、钙、磷、铁、维生素B1、维生素B2、烟酸、奎宁酸、柠檬酸、苹果酸、橙皮苷、柚皮苷、香豆精、高量钾元素和低量钠元素等,__①__。维生素C能维持人体各种组织和细胞间质的生成,并保持它们正常的生理机能。人体内的母质、黏合和成胶质等,__②__。当维生素C缺少了,细胞之间的间质——胶状物也就跟着变少。这样,细胞组织就会变脆,失去抵抗外力的能力,人体就容易出现坏血症;__③__,如预防感冒、刺激造血和抗癌等作用。‎ 答:①__________________________________________________ ‎ ‎②_____________________________________________________‎ ‎③_____________________________________________________‎ 答案 ①对人体十分有益 ②都需要维生素C来保护 ③它还有更多用途 解析 第①处由下文的功能可知,应从对人体的益处角度组织答案;第②处由后面的标点符号及人体的需要可知,此处应从对维生素C的需要角度来组织答案;第③处由后面的预防感冒等可知,它还有更多的用途,以此作为答案即可。‎ ‎5.在下面一段文字横线处补写恰当的语句,使整段文字语意完整连贯,内容贴切,逻辑严密。每处不超过12个字。‎ 关于中国文化符号的调查显示,对于中国传统文化,外国公民有着比较统一的认识,即长城和龙,而对于中国当代文化符号,__①__,比如北京奥运会、上海世博会、令人震惊的新式建筑等。诚然文化符号有助于中国__②__,被外国民众所认识。但是它也限制了外国民众对中国文化的全面认识。应该让外国民众更多地了解中国文化的不同侧面,以便__③__。‎ 答:①__________________________________________________ ‎ ‎②_____________________________________________________‎ ‎③_____________________________________________________‎ 答案 ①他们的认知普遍比较分散 ②在世界传播自己的文化 ③全面了解中国文化 解析 根据陈述对象一致的原则,①处应填“外国公民……”;②根据前后内容一致的原则,应填“在世界传播自己的文化”;③处是总结全文,应填“全面了解中国文化”。‎ ‎6.请在下面一段文字横线处补写恰当的语句,使整段文字语意完整连贯,内容贴切,逻辑严密。每处不超过12个字。‎ 不管是语文教学还是其他功课的教学,都不是像交付一件东西那么便当,__①__,你收到了,东西就在你手里了,这就触及教学上的根本问题:在教学活动中,__②__?叶圣陶先生的看法是,“各种的教学都一样,无非是教师帮着学生学习的一串过程”。换句话说,教学,教学,就是教师教会学生怎么学。主要不是把现成的知识教给学生,__③__,学生就可以受用一辈子。在这个问题上,有一句精辟的话,已经众口传诵,那就是:授人以鱼,不如授人以渔。‎ 答:①__________________________________________________ ‎ ‎②_____________________________________________________‎ ‎③_____________________________________________________‎ 答案 ①我把东西交付出去了 ②教师究竟起什么样的作用(或“教师应该怎么教”或“教师到底扮演什么样的角色”) ③而是把学习的方法教给学生 解析 ①据上文“像交付一件东西”和下文“你收到了……”可知;②据上文“根本问题”和下文“无非是教师……”可知,应填和教师的作用有关的内容;③据上文“不是……”可知,应填“而是……”,内容上强调学习的方法。另外,注意字数限制。‎ ‎7.在下面一段文字横线处补写恰当的语句,使整段文字语意完整连贯,内容贴切,逻辑严密。每处不超过15个字。‎ 萧伯纳说:“人生有两大悲剧,一是没有得到你心爱的东西,二是得到了你心爱的东西。”这话的立足点是占有,所以才会有__①__和已得到满足的无聊这双重悲剧。如果__②__,以审美的眼光看人生,我们是可以反其意而说的:人生有两大快乐,一是没有得到你心爱的东西,于是你可以去寻求和创造;二是得到了你心爱的东西,于是__③__。‎ 答:①__________________________________________________ ‎ ‎②_____________________________________________________‎ ‎③_____________________________________________________‎ 答案 ①占有欲未得到满足的痛苦 ②把立足点移到创造上 ③你可以去品味和体验 解析 ①处,根据前面的“一是没有得到你心爱的东西”及后面“已得到满足的无聊这双重悲剧”可知,该处应该填“占有欲未得到满足的痛苦”等相关内容;②处,根据后面“我们是可以反其意而说的”“你可以去寻求和创造”及前面的“立足点是占有”可知,此处可以从“创造”的角度来说;③处,根据前面“寻求和创造”及“得到了你心爱的东西”可知,此处可填“你可以去品味和体验”等内容。‎ ‎8.在下面一段文字横线处补写恰当的语句,使整段文字语意完整连贯,内容贴切,逻辑严密。每处不超过20字。‎ 雾霾的危害之大,几乎人所共知。因此,“严重雾霾天给学生们放假”的呼吁成了大多数父母们的共同心声。但是,__①__。众所周知,雾霾的形成有多方面的原因,高能耗、高排放、高污染的发展行业,私家车的大量使用,房地产业的蓬勃发展,总之,__②__,其中人为因素对环境的影响是造成雾霾高发的根源。所以,治理雾霾不仅是政府的责任,__③__。‎ 答:①__________________________________________________ ‎ ‎②_____________________________________________________‎ ‎③_____________________________________________________‎ 答案 ①雾霾天给学生放假也解决不了根本性的问题 ②雾霾的发生是方方面面的机构和个人行为所致 ③也是企业和每个人的责任 解析 此题考查语言连贯的能力,补写时要联系上下句,揣摩整段话表达的主题,从而补写出符合要求、适合语境的句子。①处,根据“但是”一词,联系“危害之大”,此句应该是说明“给学生放假解决不了根本问题”。②处,是对上文雾霾发生原因的总结,考虑上下文,必须把各行业和个人囊括进去。③处,考虑责任问题,除政府之外,那就是企业和每个人的责任。‎ ‎9.在下面一段文字横线处补写恰当的语句,使整段文字语意完整连贯,内容贴切,逻辑严密。每处不超过15个字。‎ ‎“一阴一阳之谓道”是中国古代哲学中的一个重要论断。那么__①__?首先,__②__,万事万物都存在这种关系,比如自然界的天地、四季、山水、风火等,都有阴阳之别。其次,阴阳两种属性尽管是对立的,__③__,阳中有阴,阴中有阳。阴阳互相配合,才能化生万物。最后,阴阳是可以互相转化的,比如泰与否、损与益等,都不是一成不变的。由上所述,可见阴阳两种属性是对立统一的关系。‎ 答:①__________________________________________________ ‎ ‎②_____________________________________________________‎ ‎③_____________________________________________________‎ 答案 ①阴阳二者是什么关系呢 ②阴阳是互相对立的 ③但也是互相依存(统一或配合)的 解析 通读语段,可知语段主要讲阴阳之间的关系。从“其次,阴阳两种属性尽管是对立的”来看,②句应该是讲“阴阳是互相对立的”;再从③空后面的内容来看,可知③空应该填“但也是相互配合的”等相关内容;而①句是对全语段的总领,结合后面的问号可知,应该是问阴阳是什么关系。‎ ‎10.在横线上填入恰当语句,使上下文语意协调。‎ 中国有句古话:__①__。这固然不错。不过,远虑是无穷尽的,必须适可而止。有一些远虑,可以预见也可以预作筹划,不妨就预作筹划,以解除近忧;__②__;还有一些远虑,完全不能预见,自然不能预作筹划,就更不必总是怀着一种莫名的近忧。总之,应该尽量少往自己的心里搁忧虑,保持轻松的心情。‎ 答:①__________________________________________________ ‎ ‎②_____________________________________________________‎ ‎③_____________________________________________________‎ 答案 ①人无远虑,必有近忧 ②有一些远虑,可以预见却无法预作筹划,那就暂且搁下吧,何必让它提前成为近忧 解析 第一句提取语段中心词,“远虑”“近忧”。第二句排比句结构,必须以“可以预见,不可以预作筹划”为内容构架表述“远虑”与“近忧”的关系。‎ ‎【语句复位题】‎ ‎11.填入下面句中横线处的语句,与上下文衔接最恰当的一组是(  )‎ 不止一位先贤指出,一个人无论看到怎样的美景奇观,如果没有机会向他人讲述,他绝不会感到快乐。一个人无论遭遇怎样的不幸,如果有向心爱的人倾诉的机会,他的痛苦自然会减轻。________________,________________。没有任何人倾听,绝对的孤独,快乐会成为失望,痛苦会成为绝望!‎ A.没有人分担的痛苦是最可怕的痛苦,没有人分享的快乐绝非真正的快乐 B.没有人分享的快乐绝非真正的快乐,没有人分担的痛苦是最可怕的痛苦 C.痛苦没有人分担是最可怕的痛苦,快乐没有人分享的绝非真正的快乐 D.快乐没有人分享的绝非真正的快乐,痛苦没有人分担是最可怕的痛苦 答案 B 解析 从连贯上讲,“有……没有”是一致的句式表达;从意思和结构上讲,先说“快乐”再说“痛苦”是对前文的自然衔接和对下文的提示。‎ ‎12.填入下面一段文字中横线处的语句,与上下文衔接最恰当的一项是(  )‎ 劳动美学在我国现代化建设中有着广阔的发展前途,__________________。因此,为了充分发掘生产劳动中的审美因素,加强劳动美学的研究已成为我国当前美学研究中的一项重要任务。‎ A.但是,我们一直没给予应有的重视 B.但是,一直没有得到我们应有的重视 C.但是,我们研究得还不够深入 D.但是,一直没有被我们研究过 答案 B 解析 A、C两项与上句主语不一致,排除;D项与下句句意矛盾,排除。‎ ‎13.填入下列横线处的句子,与上下文衔接最恰当生动的一句是(  )‎ 夜色浓了,灯亮了起来。环绕在海湾沿岸山坡上的灯光,从半空倒映在了乌蓝的海面上,随着波浪,晃动着,闪烁着,__________________,和那密布在苍穹里的星斗互相辉映,煞是好看。‎ A.像散落在蓝色天鹅绒上的颗颗钻石 B.像一串流动的珍珠 C.像夜空中眨着眼睛的星星 D.像无数个在水面舞动的小精灵 答案 B 解析 要注意以下两条重要信息:一是“环绕在海湾沿岸”,隐含“串”“线”之意。从这一角度来看,A、C、D三项皆描写“面”“片”(“天鹅绒”“夜空”“水面”)之景象,皆不恰当。二是“随着波浪,晃动着,闪烁着”,显示出一种“动态”的美感,若描写的是一种静态的景象(如A项),那自然不合适。‎ ‎14.填在下面横线上与上下文衔接最好的一项是(  )‎ 人格恰如一种魔力,从人格上发出来的言行,自然使人受到感化。__________________。这就是有人格的背景与否的区别。空城计只能由诸葛亮摆,换了其他人便会失败。‎ A.同是一句话,德高望重者说出来,大家就容易接受 B.同是一句话,因说话人有两种人格,效果就不一样 C.同是一句话,因说话人的人格不同,效力亦往往不同 D.同是一句话,说话人的人格不一样,大伙就觉得劲头不一样 答案 C 解析 从后文“有人格的背景与否”看,A项不够衔接,可排除;由最后一句话可知说同一句话的应是两种不同人格的人,所以B项“说话人有两种人格”的表述不妥;而D项在语言风格上与上下文不够协调。‎ ‎15.在文中的①②处横线上,依次填入的语句,衔接最恰当的一组是(  )‎ 我是到断桥去寻残雪的,可寻到的只是满地的泥泞。然而我敏感的心却突然震颤,仿佛感觉到背后有轻盈的脚步在向我走来。像是初恋的情人,总是悄然而至,在我等待得不耐烦之时,__①__。感觉并没有骗我,在我的背后,__②__。‎ A. B. C. D. 答案 A 解析 解答本题,既要考虑句式,也要注意语意的连贯,第①空强调的是“意外的惊喜”,所以应放在后面表述;第②空承接前句“在……”,应该选“在鲜为……”。‎ ‎【语句排序题】‎ ‎16.依次填入下面一段文字横线处的语句,衔接最恰当的一组是(  )‎ 年轻时不擅长把握自己,做什么事都走极端显得过度。太急切地表现,________________,________________,______________。太过于胆怯,______________,________________,________________。一审势,看准了再做;二适度,得体地表现。古语说:“放者流为猖狂,收者入于孤寂。惟善操身心者,把柄在手,收放自如。”‎ ‎①不太得体 ②就容易太夸张激昂 ③就害怕见人 ④机会来了也显不出你 ⑤常滥情失控 ⑥连一句整话都说不出 A.③①②⑤⑥④ B.②①③⑤④⑥‎ C.②⑤①③⑥④ D.③②①⑤④⑥‎ 答案 C 解析 “太急切地表现”和“太过于胆怯”分别引领着三句话,可以肯定,另外两个“就”领起的句子应该紧跟其后,①④是两种不同的性格缺陷导致的不良后果应放在所属领属句的最后。‎ ‎17.依次填入文段中的语句衔接最恰当的一组是(  )‎ 空气质量优劣与人们健康生活息息相关,________,________。________;________,________,________。‎ ‎①雾霾天气时,空气中的PM2.5等悬浮颗粒及有害气体影响人们的呼吸道健康,给身体带来不良影响 ‎②并且在城市中污染物不易扩散 ‎③严重的雾霾天气造成空气质量下滑 ‎④进一步对人体健康造成危害 ‎⑤给人们的出行、生活造成诸多不便 ‎⑥雾霾天气大大降低空气能见度,影响交通 A.①④⑥②⑤③ B.①⑥②④③⑤‎ C.③⑤②①⑥④ D.③⑤⑥①②④‎ 答案 D 解析 文段整体是总分结构,③⑤是总说应放在前面,也可根据此处的句号来断定③⑤与前面构成一句话。⑥是雾霾对交通的影响,承接第一句分说。①是雾霾对身体的危害,较⑥更进一层。②④再进一步说明雾霾在城市中的严重危害。‎ ‎18.填入下面横线处的句子,与上下文衔接最恰当的一组是(  )‎ 我爱小池,也爱溪流,是因为我爱它们的“清”和“远”。________。________。________。__________,________。像一位开朗活泼的姑娘,唱着欢快的歌儿,踏着轻快的脚步,走出群山迎接朝阳和大海。‎ ‎①几种颜色的小鱼穿行其间,历历可数 ‎②溪流蜿蜒,如一条银蛇 ‎③小池清澈见底,汪汪一碧,宛如一块温润的碧玉 ‎④时隐时现,游向远方 ‎⑤绿油油的水草在水底轻轻晃动 A.③⑤①④② B.②③①⑤④‎ C.②④③⑤① D.③⑤①②④‎ 答案 D 解析 首先抓住语段开头的关键句“我爱小池,也爱溪流,是因为我爱它们的‘清’和‘远’”。所填的5个句子以“清”和“远”为中心可归为两个句群,其中③⑤①为一个句群,其中心为“清”,②④为一个句群,其中心为“远”。根据开头关键句中“清”“远”的先后顺序,可确定③⑤①在前,②④在后。故选D。‎ ‎19.下列句子排列合理的一项是(  )‎ ‎①苏州园林可绝不讲究对称,好像故意避免似的。‎ ‎②这是为什么?‎ ‎③我想,用图画来比方,对称的建筑是图案画,不是美术画,而园林是美术画,美术画要求自然之趣,是不讲究对称的。‎ ‎④东边有了一个亭子或者一道回廊,西边绝不会来一个同样的亭子或者一道同样的回廊。‎ ‎⑤我国的建筑,从古代的宫殿到近代的一般住房,绝大部分是对称的,左边怎么样,右边也怎么样。‎ A.①②④③⑤ B.①④②⑤③‎ C.⑤①④②③ D.⑤②③①④‎ 答案 C 解析 做顺序调整判断题,一般不要忙着找“第一句”,而要先寻找逻辑上结合最紧密的句子。以本题为例:①句讲“苏州园林可绝不讲究对称,好像故意避免似的”,下面就应该举例子来说明,也就是④句,所以①④两句肯定紧挨着,于是就排除了A项;再下面就该交代苏州园林不讲究对称的原因了,那就是②③两句了,②③两句是一组设问句,②问③答,②前③后,于是我们就确定了①④②③的顺序,⑤句是整体的建筑风格,而①④②③是建筑的特例,即以“苏州园林”为例,从而选出正确答案。‎ ‎20.把下列句子组合成语意连贯的语段,排序最恰当的一项是(  )‎ ‎①陈维崧、朱彝尊、纳兰性德为“清初三大家”。‎ ‎②嘉庆年间,张惠言又开创了“常州词派”,主张词应有比兴寄托。‎ ‎③朱彝尊推崇南宋词人姜夔、张炎,标榜醇雅清空,为“浙西派”的宗师。‎ ‎④清末虽然也产生了一些著名词人和优秀作品,但在社会现实和人们生活中的影响越来越小了。‎ ‎⑤陈维崧词效仿苏、辛,才力卓著,开创了“阳羡词派”。‎ ‎⑥纳兰性德崇尚李煜,尤以清新自然的小令著称。‎ ‎⑦清词素有“中兴”之誉,百余年间,各派并出。‎ A.④⑦①⑤③⑥② B.⑦①⑤③⑥②④‎ C.⑦⑤③⑥②④① D.①⑤③⑥②⑦④‎ 答案 B 解析 连贯类题目做题时要注意把握基本内容,初步分层归类,先在小范围内排序,然后再进行层次间的衔接,这其中应先找出关联词、代词以及表时间、地点的词语,然后据此进行句间连缀排列。在上面排列的基础之上,再通读语段,检查确定。由①句,可确定①⑤③⑥的排序,⑦句是总括句,可排为⑦①⑤③⑥,②④既有时间顺序,又有词的发展的另一种状况,由此可确定B项恰当自然。‎ 题组2 简明、得体 ‎21.下面一段文字中,有一句话多余,请指出来。(只填序号)‎ ‎①几天前,收到挪威使馆的一张请柬,是用再生纸做的一张卡片,朴素精致,显示着主人的情谊。②对比我们平时收到的一些请柬,不免心生感叹。③不少单位的请柬,一张比一张豪华精美:加厚铜版纸、烫金大字、色彩艳丽,更甚者还要绒面、磨砂、带上音响。④只是构图稍差,美中不足。⑤请柬的功能很简单,没有必要那么奢华。⑥挪威是个富裕国家,尚且如此节俭,我国作为发展中国家,不是更应该节约吗?‎ 答:____________________________________________________ ‎ 答案 ④‎ 解析 ③④主要说明的是中国贺卡奢华,④“只是构图稍差,美中不足”脱离了中心,而且“美中不足”带有惋惜意味,不符合作者对中国贺卡批评的情感态度。‎ ‎22.阅读下面这则材料,请你把营长的话修改成一条简洁明确的命令。(不超过40个字)‎ 据说在本世纪初,美国有一个营长对值班军官下达了这样一条命令:“明晚8点,哈雷彗星将在这个地区出现。这种情形每隔76年才能看到一次。命令所有士兵,身着野战服到操场集合,我将向他们解释这一罕见的天文现象。如果下雨的话,就到礼堂集合,我为他们放一部有关彗星的电影。”这道命令由传令兵传达给连长,然后连长传达给班长,当班长对士兵下达命令时,命令竟然成了这样:“在明晚8点下雨的时候,著名的76岁的哈雷将军,将在营长的陪同下,身着野战服,乘坐他那辆‘彗星’牌汽车,经过操场前往礼堂观看电影。”‎ 答:____________________________________________________ ____________________________________________________________‎ 答案 明晚8点,全体士兵穿野战服到操场集合;如下雨,到礼堂集合。‎ 解析 营长的命令通过多次转达竟完全“变了样”,是因为传达的内容不够简明,出现了偏差。根据实际需要,本着简明的原则,命令内容只要包括“时间”“地点”“对象”和意外情况即可。‎ ‎23.阅读下面一段话,本着文字要简明的原则,完成文后两题。(把序号填入画线处)‎ 深圳南方公司,①在改革开放形势的推动下,②为了避免对来深圳南方公司应聘的人以是否名牌大学毕业而选择录用的先入为主的弊端,以聚集人才,③今年招聘大学毕业生,不再问毕业学校。他们认为,④任何一个一流企业如果不注重选拔人才注入新鲜血液,⑤如果仅凭是否名牌大学选择人才的话,将很难发展。‎ ‎(1)应删去的两处语句是________。‎ ‎(2)应简略的一处语句是________,可简略为________。‎ 答案 (1)①④ (2)② 为了避免先入为主的弊端 解析 本段文字主要介绍深圳南方公司用人的新理念,①是介绍时代背景,与选拔人才没有必然关系,应删去;④说的是“任何一个一流企业如果不注重选拔人才”,这与本公司用人的新理念没有关系,所以是多余的。②句冗长难以理解,应予以删减压缩。‎ ‎24.下面是盘山公路下坡处一块汽车指示牌上的指示语,用语不大简明。请根据特定场合的用语要求,将原句压缩(包括标点符号至多6个字)。‎ 原文:前面下坡,坡陡弯急。为了你和你的家人,请减速慢行。‎ 修改:__________________________________________________ ‎ 答案 (示例一)坡陡,慢行!‎ ‎(示例二)坡陡,减速!‎ 解析 “前面下坡,坡陡弯急”可压缩为“坡陡”,很简洁,又能引起注意;“为了你和你的家人”表目的的状语,可删去;“请减速慢行”可压缩为“慢行”或“减速”。‎ ‎25.下面是潍坊市某中学读书节活动学生筹委会发给莫言先生的邀请函的正文,其中有四处语言表达不得体或不简明,请找出来并加以改正。‎ 我校第五届读书节将于2017年2月25~28日举行。作为家乡学子,我们特向您发出鼎力邀请,恭请您莅临指导并给广大莘莘学子发表演讲。虽然您现在已是炙手可热的公众人物,事务繁忙,但我们仍十分期盼您的惠顾。‎ 答:____________________________________________________ ____________________________________________________________‎ 答案 ①“鼎力”改为“诚挚”或删去。②“莘莘学子”改为“同学”。③“炙手可热”改为“蜚声中外”。④“惠顾”改为“光临”。(其他改法正确亦可)‎ 解析 解答此类邀请函的题目,要求语言运用上要简明、得体。“鼎力”是敬辞,用于自己不妥。“莘莘学子”表示众多学生,不能与“广大”并列使用。“炙手可热”形容气焰很盛,权势很大。用在这里显然不当。“惠顾”是用于商店对顾客,此处用来欢迎莫言先生也不妥当。‎ ‎26.一句熨帖得体恰到好处的话,不仅可以化解心灵的寒霜,还可以化解一触即发的矛盾。如果你是艾威尔先生,在以下情境中,你将怎样得体而幽默地要回自己的大衣?‎ 某一豪华酒店内,艾威尔发现一个客人错穿了他的大衣,于是他异常谦恭地碰了碰这位客人,说:“对不起,请问您是艾威尔先生吗?”‎ ‎“您弄错了,我不是他。”那人回答。‎ ‎“啊。”艾威尔舒了一口气,“__________________________________________________。”这时两人相视一笑,客人愉快地脱下了大衣。‎ 答案 那我没弄错,我就是他,您错穿了他的大衣 解析 作答本题,要明确题干的要求:得体而幽默。得体方面,即索要大衣时,不应使用过于直白、生硬的话语,如“您拿了我的大衣”“把我的大衣还给我”等;而应含蓄委婉地表达,如“对不起,您好像错穿了我的大衣”‎ ‎。幽默方面,可以从文段中的语境入手,艾威尔已经幽默地用自己的身份设置了一个问题,在那个人回答“您弄错了,我不是他”时,可借机说出类似于“我没弄错,我就是他”的幽默话语。‎ ‎27.以下是某位同学写给某高校的自荐信的正文部分,其在语体风格、用词、语言得体等方面均有不当之处,请找出并改正。‎ ‎①我学习刻苦,考过好几次学年第一。②此外,我爱好广泛,在文学、体育方面均有建树,③曾获市“五四”征文大赛一等奖,④并曾代表班级参加校运动会,取得过400米第九名的骄人成绩。⑤希望贵校能慧眼识金,⑥如有幸能到贵校就读,我将加倍努力,争取成为贵校的优秀学子。‎ 答:(1)将________改为____________________________________ ____________________________________;‎ ‎(2)将________改为___________________________________ _____________________________________;‎ ‎(3)将________改为______________________________________ __________________________________;‎ ‎(4)将________改为______________________________________ __________________________________。‎ 答案 (1)① 我学习刻苦,多次荣登学年榜首 ‎(2)② 在文学、体育方面均有突出表现 ‎(3)④ 并曾代表班级参加校运动会,取得过400米第九名的成绩 ‎(4)⑤ 希望贵校能给我一个机会 解析 ①口语化词语改为书面用语;②“建树”词义太重,用词不当;④“骄人”与第九名的成绩不相称,且不得体;⑤自称“金”,用语不得体。‎ ‎28.2015年6月13日是我国第十个“文化遗产日”,学校开展了保护文化遗产的宣传活动。如果你是该校的志愿者,发现游客在景区文物上刻字留言,你将如何劝阻?请针对以下不同对象,各写一句话。要求:语言得体,有说服力,每句不超过30字。‎ ‎(1)对同龄人:_____________________________________ ___________________________________‎ ‎(2)对年长者:__________________________________________ ______________________________‎ 答案 (1)我们都是祖国的接班人,让我们共同保护好祖国的文化遗产。‎ ‎(2)您是我们的长辈,请您为我们做好榜样,保护好祖国的文化遗产。‎ 解析 本题考查语言表达得体的能力。要针对不同年龄的对象,采用不同的语言,同时要明确指出这种做法的危害。‎ ‎29.在下面对话的横线上补写恰当的语句,使语意完整清晰,语言连贯得体。‎ 张宇:你好。你们的短信说“货品已经发出,预计二日到”,今天已经第三天了,怎么货还没到呢?‎ 网店客服:不好意思,__①__‎ 张宇:噢,那我就在2号等着签收了。还有,你们短信说“已付包装费保险费送货费由收货方付”,我到底该付哪些费用呢?‎ 网店客服:__②__‎ 张宇:我明白了。谢谢!‎ 网店客服:对不起,我们工作不细致,给您带来了不便。‎ 张宇:没关系。你们能改进工作就行了。‎ 答:①__________________________________________________ ‎ ‎②_____________________________________________________‎ 答案 ①我们的意思是“本月2号”到货。②我们已付包装费,保险费和送货费由您付。(或:我们已付包装费、保险费,送货费由您付)‎ 解析 此类试题的解题思路如下,先浏览语段,把握大概内容;然后看横线前后的内容,以此确定横线上的内容和句式。这是一段对话,要特别注意横线前后的内容。第①处,应依据后面“张宇:噢,那我就在2号等着签收了”这句话来确定横线上的内容,应是说“2号到货”;第②处,应依据前面的提问“我到底该付哪些费用呢”来确定横线上的内容。‎ ‎30.下列句子用语得体的一项是(  )‎ A.日前丢失支票,蒙您及时送回,感激不尽。明天我将于百忙中专程前来致谢,请在家等候。‎ B.学生会经过调查研究,写出了《我校食堂服务质量调查报告》,文中提出了改进意见,并且责成学校领导研究落实。‎ C.听说贵公司在经营方面存在困难,你们如需要指点的话,我们将不吝赐教。‎ D.奉上拙著一本,鄙人才疏识浅,书中谬误甚多,特此敬请斧正。‎ 答案 D 解析 A项说自己“将于百忙之中专程前来致谢”不得体;B项“责成”一般用于上级要求下级做某项工作,这里说学生会“责成”学校领导,不当;C项“指点”“不吝赐教”不应该从“自己”嘴里说出。‎ 题组3 准确、鲜明、生动 ‎31.广电总局日前重申“限制方言令”,要求广播电视节目规范使用通用语言文字,在推广普及普通话方面起到带头示范作用。通知要求,播音员主持人除节目特殊需要外,一律使用标准普通话。也就是说,我们看不到小沈阳用东北话埋汰自己的容貌,也看不到周立波用上海话抖包袱,更看不到汪涵客串各地方言来搞笑了。那么,你如何看待“限制方言令”?请说说你的理由。‎ 要求:①观点明确;②语言表达简明、得体;③字数不超过50个字。‎ 答:____________________________________________________ ____________________________________________________________‎ 答案 (1)该禁。①很多方言容易产生歧义,不利于沟通交流。②沉迷于方言的使用,忽略剧情,反而降低了电视剧整体创作水平。‎ ‎(2)不该禁。①方言的“亲切感”是文化的认同感。②在艺术创作中,要讲究美感、艺术性和真实性,电视剧中的方言对白能增加剧情的喜感。‎ 解析 回答此类题先要明确观点,无论同意还是反对,观点都要鲜明;阐述理由一定要结合材料,不能完全脱离材料。如“该禁”可从“方言”的危害谈。‎ ‎32.阅读下面的材料,按要求回答后面的问题。‎ ‎“世界上最遥远的距离,不是生与死,而是我在你身边,你却在低头玩手机。”随着网络信息科技的日益发达,智能移动的电子产品渐渐充斥我们的生活。“低头族”“手机控”数不胜数。正是这些电子产品成为人类情感沟通的屏障,成为扼杀情感的杀手。‎ 请就此现象写一段公益广告词。要求:语言表达鲜明、生动;不超过40字。‎ 答:____________________________________________________ ____________________________________________________________‎ 答案 (示例一)最是那一低头的冷漠,拉开了亲人、朋友间的距离;抬起头来,珍惜身边的人吧。‎ ‎(示例二)让所有的时光,真正成为感情的纪念,而不是被手机消磨掉的指间沙。‎ 解析 “低头族”“手机控”非常普遍,其让人与人之间的各种感情疏远,让人们忽视掉身边的美景。所以公益广告词就要从这个角度入手,意在提醒人们,多关注身边人、身边景,而不是一味沉浸在电子产品中。‎ ‎33.某著名中学为纪念建校一百周年,出版了一本名为“继往开来”的专刊,请你用准确、鲜明、生动的语言为这本专刊写一段简短的刊首语。要求必须从比喻、对偶、排比、反复、借代中选用两种修辞手法,不超过70字。‎ 答:____________________________________________________ ____________________________________________________________‎ 答案 百年名校,虽历经风雨而能与时俱进,桃丰李盛,栋直梁坚,荣誉满载,可喜可贺!忆往昔峥嵘,令人缅怀和欣慰;望前程灿烂,促人励志和扬鞭。‎ 解析 本题考查考生语言表达准确、鲜明、生动的能力。刊首语,又称刊首寄语,主要是反映刊物的创办宗旨、刊物的主要内容、刊物的努力方向等,一般文字较为简练、精到、富有诗意。本题要求为纪念建校一百周年而出版的名为“继往开来”的专刊写刊首语,因此所写的刊首语应该包括以下内容:点明名校建校一百周年,表达“继往开来”之意。同时还要从题干中给出的修辞手法中选用两种,不超过70字。‎ ‎34.下面四句诗的每一句都可以想象成一幅画面,第一句和第四句的画面均已写出,请你补写中间两句的画面,注意表达准确、生动。(不要求句式相同)‎ 诗句:风急天高猿啸哀,渚清沙白鸟飞回。无边落木萧萧下,不尽长江滚滚来。‎ 画面一:秋日天高气爽,此处却猎猎多风。山谷中不断传来“高猿长啸”之声,“空谷传响,哀转久绝”。‎ 画面二:_______________________________________________ ‎ 画面三:________________________________________________ ‎ 画面四:滚滚而来的长江之水奔流不息。‎ 答案 画面二:在水清沙白的江水洲渚上,点缀着迎风飞翔、不住回旋的鸟群。‎ 画面三:无边无际的林木,树叶在秋风中窸窣飘落。‎ 解析 解答此题关键是把握诗歌的意境,补出的画面要与原诗意境保持一致,语言尽量生动、形象。‎ ‎35.为活跃校园文化生活,弘扬学生的爱国情操,某文学社举办了一个“中国梦·我的梦”征文优秀作品展,请你为作品展写一段前言。要求:语言鲜明、生动,语意连贯;至少使用一种修辞手法;不少于120字(含标点符号)。‎ 答:____________________________________________________ ____________________________________________________________‎ 答案 每个人都有自己的梦想,每个人的梦想都必须以国家的强大为坚强后盾,每个人的梦想汇聚在一起就是我们的中国梦。“雄关漫道真如铁,而今迈步从头越”,是不畏艰险,勇于追求梦想的壮怀!在这里,我们能看到你的梦,我的梦,他的梦,十三亿中国人民的梦。让我们携起手来,响应时代的号召,为了光明的未来,为了美好的梦想努力奋斗!‎ 解析 本题考查语言表达准确、鲜明、生动、简明、连贯、得体的能力。作答本题时,注意要开门见山,直接提示作品展的基本信息,要言简意赅,围绕主题介绍作品展的功能、目的、意义等。同时还要注意题干中至少使用一种修辞手法、不少于120字的要求。‎ ‎ [3年高考真题集训]‎ ‎1.[2016·全国卷Ⅰ]在下面一段文字横线处补写恰当的语句,使整段文字语意完整连贯,内容贴切,逻辑严密。每处不超过15个字。‎ 花青素是一种水溶性的植物色素,分布在液泡内的细胞液中,能够决定花的红色、蓝色、紫色等颜色的差别。这是因为花青素__①__:在酸性溶液中呈现红色,在碱性溶液中变为蓝色,处于中性环境时则是紫色。更令人称奇的是__②__,比如有一种牵牛花清晨是粉红色,之后变成紫红色,最后变成蓝色。究其原因,就是花瓣表皮细胞的液泡内pH值发生了变化,__③__,从而形成花的颜色的变化。‎ 答:①__________________________________________________ ‎ ‎②_____________________________________________________‎ ‎③_____________________________________________________‎ 答案 (示例)①在不同环境中会形成不同颜色 ②有些花的颜色可以一日数变 ③花青素也就随之发生变化 解析 本题给出一个语段,其中有三处空缺,要求在空缺处填入恰当的语句。语段的主要内容是介绍花青素和花的颜色之间的关系。表面看起来本题有一定的难度,似乎答题需要有一定的专业知识,实际上只要综观全文,把握所给文字的核心话题,依据上下文的相关提示语和相关信息提示,准确地进行补写并不困难。先看空缺①,①的前面是“这是因为花青素”,可以大致得出,这儿需要填写的是花青素具有某种特点。后面是“:在酸性溶液中呈现红色,在碱性溶液中变为蓝色,处于中性环境时则是紫色”,根据“:”可以知道,后面的句子是对前面句子的解释,根据上下文,可以推导出,补写的语句应为“在不同环境中会形成不同颜色”。再看空缺②,②后面的句子是“比如有一种牵牛花清晨是粉红色,之后变成紫红色,最后变成蓝色”,由“比如”可以知道,这句话是对前面的句子的一个举例,再根据这句话是说一种花的颜色一天之内发生多次变化,可以补写出前面一句“有些花的颜色可以一日数变”。最后看空缺③,这句话所在的句子是解释花为什么可以一日数变,前面说“花瓣表皮细胞的液泡内pH值发生了变化”,后面说“从而形成花的颜色的变化”,再根据全文花青素和pH值的关系,可以正确补出“花青素也就随之发生变化”。‎ ‎2.[2016·全国卷Ⅱ]‎ 在下面一段文字横线处补写恰当的语句,使整段文字语意完整连贯,内容贴切,逻辑严密。每处不超过15个字。‎ 气候是一种复杂的自然现象,不仅决定着土壤、植被类型的形成,改变着地表形态,__①__。人们的生活、生产、建设无不需要考虑气候的影响。气候已成为一种自然资源,供人类充分利用,为人类造福。但是,__②__,有时会带来某些灾害。所以,人们会利用一些方法,在一定区域内改变气候状况,__③__。‎ 答:①__________________________________________________ ‎ ‎②_____________________________________________________‎ ‎③_____________________________________________________‎ 答案 (示例)①而且还影响着人类的活动 ‎②气候对人类也有不利的一面 ‎③使它向着有利于人类的方向发展 解析 解题时,要综观整个语段,把握所给文字的核心话题,依据相关提示进行准确补写,使整段文字语意连贯。第①处,前面说“不仅……”,后面自然是“而且……”,具体内容应结合第①处横线后面的句子填写;第②处,前面的“但是”表转折,前面是说为人类造福,后面自然是从反面谈气候给人类带来的不利的影响;第③处,根据前文内容,应填人们利用一些方法改变气候状况的目的。‎ ‎3.[2016·全国卷Ⅲ]在下面一段文字横线处补写恰当的语句,使整段文字语意完整连贯,内容贴切,逻辑严密。每处不超过10个字。‎ 自第一颗人造地球卫星进入太空以来,除了载人航天飞行器会回收之外,其他上天的人造物体陆续被遗弃在太空中,__①__。太空垃圾已经威胁到人类的航天活动。比如厄瓜多尔的一颗卫星升空后不到一个月,就与太空中的火箭残骸相撞而报废。这种威胁不仅仅发生在太空,甚至地球上的人类生活也会__②__。因此,__③__,应是人类接下来要解决的一个重要课题。‎ 答:①__________________________________________________ ‎ ‎②_____________________________________________________‎ ‎③_____________________________________________________‎ 答案 ①都变成了太空垃圾 ‎②受到太空垃圾的影响 ‎③如何清理太空垃圾 解析 ①处前句提到“被遗弃在太空中”的“其他上天的人造物体”,后句提出了“太空垃圾”的威胁,故①处应填“都变成了太空垃圾”。②处所在句子是递进关系复句,前句强调太空垃圾对太空的威胁,结合后半句的提示,可知②处应接着谈太空垃圾对人类生活的影响,故②处应填“受到太空垃圾的影响”。③处后面的宾语是“重要课题”,而且整个文段谈的均是太空垃圾的问题,因此结论必然与对太空垃圾的处理有关,故此处应填“如何清理太空垃圾”。‎ ‎4.[2016·山东高考]下面是某中学学生会向各班级班长所发通知的正文,请阅读并按要求完成后面的题目。‎ 为了进一步弘扬优秀传统文化,提高同学们的国学素养,校学生会定于10月18日下午4点,在报告厅举办“走近孔子”读书交流会。届时在孔子研究领域享有极高盛誉的孙荣教授将光临指导,并向各班奉送其最新研究著作。请拨冗组织班委推荐两名发言的同学,并告知他们一定务必按时到会。‎ ‎(1)在不改变语意的前提下,为了表达简明,文中必须删掉两个词语,分别是________和________。‎ ‎(2)文中使用不得体的两个词语,分别是________和________。‎ 答案 (1)极高 一定(或“务必”)‎ ‎(2)奉送 拨冗 解析 (1)“极高”和“盛”语意重复,删去“极高”。“一定”和“务必”重复,两者删去其一。‎ ‎(2)奉送:敬辞,赠送。拨冗:客套话,推开繁忙的事务,抽出时间。这两个词均是敬辞,不能用于上级对下级。‎ ‎5.[2016·浙江高考]在空格处分别补写出倡议的理由和具体内容。(两处字数各不超过40个字)‎ 倡议书 各位同学:‎ 乘坐公交是很多市民日常出行的选择。众所周知,。可是,我市不文明乘车现象时有发生,甚至发生老人被人群挤倒而摔成粉碎性骨折的悲剧。‎ 为此,我们向全校同学发出倡议:‎ 。‎ 文明乘车,从我做起!让我们用自己的行动为城市增光添彩!‎ ‎××中学学生会 ‎×年×月×日 答案 (示例)文明乘车让出行更安全,更高效,体现了一个人的基本素养,也有益于社会和谐 自觉排队,有序上车;尊老爱幼,主动让座;举止文明,谈吐有礼 解析 “可是,我市不文明乘车现象时有发生……”是一个转折句,可知第一处应该填写文明乘车的益处。第二处应填写倡议的具体内容,应注意交通方式为文明乘坐公交,答题时可从上下车以及行车途中的言谈举止要文明来谈。‎ ‎6.[2015·全国卷Ⅱ]填入下面一段文字横线处的语句,最恰当的一句是(  )‎ 辣,我们都不陌生,很多人无辣不欢甚至吃辣上瘾。这是因为辣椒素等辣味物质刺激舌头、口腔的神经末梢时,会在大脑中形成类似灼烧的感觉,机体就反射性地出现心跳加速、唾液及汗液分泌增多等现象,________,内啡肽又促进多巴胺的分泌,多巴胺能在短时间内令人高度兴奋,带来“辣椒素快感”,慢慢地我们吃辣就上瘾了。‎ A.大脑在这些兴奋性的刺激下把内啡肽释放出来 B.内啡肽因这些兴奋性的刺激而被大脑释放出来 C.这些兴奋性的刺激使大脑释放出内啡肽 D.这些兴奋性的刺激使大脑把内啡肽释放出来 答案 C 解析 前面陈述的主体是“类似灼烧的感觉”,所以后面从衔接恰当的角度应先陈述“这些兴奋性的刺激”,因此排除A、B两项,C、D相比较,D项使字句与把字句连用累赘。使字句强调的是主体“刺激”,把字句强调的是“大脑”,而这句话没有强调“大脑”的必要,所以累赘。‎ ‎7.[2015·全国卷Ⅰ]填入下面一段文字横线处的语句,最恰当的一句是(  )‎ 随着雾霾频发,油品质量对环境的影响引起了人们越来越多的关注。有测试表明,一些城市空气中PM2.5的20%左右来自机动车尾气,而只要使用符合新标准的汽油和柴油,____________________。有鉴于此,我国将加快推进成品油质量升级国家专项行动。‎ A.即使现有汽车不作任何改造,其尾气中相关污染物的排放也能减少10%‎ B.汽车尾气中相关污染物的排放就可减少10%,现有汽车的改造并不是必须的 C.再加上对现有汽车进行改造,其尾气中相关污染物的排放就将减少10%以上 D.不管是否改造现有汽车,其尾气中的相关污染物排放都将减少10%‎ 答案 A 解析 B项,此项侧重说“现有汽车的改造并不是必须的”,与后面“我国将加快推进成品油质量升级国家专项行动”衔接不紧密;C项,此项有“再加上对现有汽车进行改造”,明显突出了“对汽车改造”的作用,与后面的内容衔接不紧密;D项,“不管……都”句式语气太绝对。‎ ‎8.[2015·全国卷Ⅱ]在下面一段文字横线处补写恰当的语句,使整段文字语意完整连贯,内容贴切,逻辑严密。每处不超过15个字。‎ 读书的目的仅仅是为了记住书中的内容吗?答案是否定的。__①__。记忆型阅读是我们缺乏想象力的根源之一,因为它容易导致盲从书本知识,从而失去质疑精神。批判型阅读是一种创造性阅读,它不追求__②__,而主张激发想象力和灵感,带着自己的思考,让自己变得更有思想。能通过阅读提出有价值的质疑,__③__。通过分析根源找到解决问题的途径和方法,这在泛阅读日益普遍的时代更显得难能可贵。‎ 答:①__________________________________________________ ‎ ‎②_____________________________________________________‎ ‎③_____________________________________________________‎ 答案 ①阅读有记忆型和批判型之分 ‎②简单的、机械的知识记忆 ‎③通过质疑找出问题的根源 解析 第①空应填一个总起句。第②空根据“不追求”可知应填记忆型阅读的特点。第③空要包含上句中的“质疑”和下句中的“根源”。‎ ‎9.[2015·全国卷Ⅰ]在下面一段文字横线处补写恰当的语句,使整段文字语意完整连贯,内容贴切,逻辑严密。每处不超过15个字。‎ 电子商务存在的价值之一,就是通过互联网进行网上购物、网上支付,节省消费者与商家的时间和空间,__①__。对于工作忙碌的上班族而言,__②__,还易于达到货比三家、快乐购物的目的。在信息多元化的21世纪,__③__,完成购物,已经成为许多消费者的习惯。‎ 答:①__________________________________________________ ‎ ‎②_____________________________________________________‎ ‎③_____________________________________________________‎ 答案 ①从而大大提高交易效率 ②除了大量节省宝贵时间 ③上网浏览商品信息 解析 通读语段,了解其大意,观察要填入的各空在语段中所处的位置。第①句应是对前面内容的总结,由“通过互联网进行网上购物、网上支付,节省消费者与商家的时间和空间”,可知第①句应该是说“提高了交易效率”。第②句,前面是“对于工作忙碌的上班族而言”,联系现实可知,对“工作忙碌”的上班族来说缺的是时间,故第②句应与“时间”有关。由后面的“还易于……”可知,第②句中要有“除了”“不仅”等字眼。第③句应是对整个语段的总结,根据“完成购物……”可知,应突出电子商务的特点,即“可以通过网络浏览商品信息”。注意前后的连贯和句子的顺畅。‎ ‎10.[2014·全国卷Ⅰ]依次填入下面一段文字横线处的语句,衔接最恰当的一组是(  )‎ 中国珠算是以算盘为工具进行数字计算的一种方法,借助算盘和口诀,通过人手指拨动算珠,就可以完成高难度计算。________,________,________,________,________,________。2013年12月4日,“中国珠算”被正式列入联合国教科文组织人类非物质文化遗产名录。‎ ‎①即便是不识字的人也能熟练掌握 ‎②珠算算盘结构简单,操作方便 ‎③包含了珠算的所有秘密 ‎④蕴含了坐标几何的原理 ‎⑤用珠算运算,无论速度还是准确率都可以跟电子计算器媲美 ‎⑥珠算口诀则是一套完整的韵味诗歌 A.②③⑥④⑤① B.②④⑥③①⑤‎ C.⑤①②⑥③④ D.⑤②③⑥④①‎ 答案 B 解析 此题考查学生语言连贯能力。六句之间,有着紧密的逻辑联系,排列时,要依照句与句之间的逻辑关系来考虑,不可仅凭语感草率做题。‎ ‎11.[2014·全国卷Ⅰ]在下面一段文字横线处补写恰当的语句,使整段文字语意完整连贯,内容贴切,逻辑严密。每处不超过15个字。‎ 二氧化碳是最主要的大气保温气体之一。大气中的二氧化碳浓度升高会导致全球变暖,造成天气干旱或旱涝不均,甚至可能造成海洋水位上升,淹没大量沿海城市,__①__。然而,也有研究指出,__②__:比如增加的二氧化碳可以给植物“施肥”,有利于植物的生长。但这必须有个前提,植物还活着!如果土壤被污染,__③__,我们就失去了这些向大气中释放氧气的“氧气工厂”和“空气净化器”。‎ 答:①__________________________________________________ ‎ ‎②_____________________________________________________‎ ‎③_____________________________________________________‎ 答案 (示例)①给人类带来巨大的灾难 ②二氧化碳增加会带来好处 ③植物就会生病甚至死亡 解析 第①空应填海水淹没大量沿海城市对人类的影响,即巨大灾难。第②空应填对后面的例子进行概括的句子:二氧化碳增加能带来好处。第③空应填土壤被污染的后果:植物生病甚至死亡。做此类题要注意:①审文意;②看语境;③巧造句。‎ ‎12.[2015·浙江高考]填入下面空缺处的语句,最恰当的一项是(  )‎ 我需要清静……最好去处是到个庙宇前小河旁边大石头上坐坐,________。雨季来时上面长了些绿绒似的苔类。雨季一过,苔已干枯了,在一片未干枯的苔上正开着小小蓝花白花,有细脚蜘蛛在旁边爬。‎ A.阳光和雨露把这石头漂白磨光了 B.这石头被阳光和雨露漂白磨光了 C.阳光和雨露已把这石头漂白磨光了的 D.这石头是被阳光和雨露漂白磨光了的 答案 D 解析 此题考查语句衔接连贯、得体。从上下句子语境来看,后一个句子是“雨季来时上面长了些绿绒似的苔类”,可见前一句子的主语应该是“石头”,由此排除了A和C项。D项用判断动词“是”进一步强调主语为“石头”,因此选D项。‎ ‎13.[2015·江苏高考]在下面一段文字横线处填入语句,衔接最恰当的一项是(  )‎ 自宋元至明清,清明节除了要祭扫家墓,还要在门楣、窗户上插上柳条。________,________。________,________,________,________。‎ ‎①达到人丁兴旺、身体健康的目的 ‎②于是在郊游踏青时 ‎③它便成了人类文化中生命力的象征 ‎④人们企盼将这种生命力转移到自家门庭和家庭成员身上 ‎⑤不会忘记顺便折一些柳条回来 ‎⑥由于柳树最先送来春的消息并且具有旺盛的生殖力 A.⑥③④①②⑤ B.②⑤①④⑥③‎ C.②④⑥③①⑤ D.⑥④②⑤③①‎ 答案 A 解析 解答本题时要先通读全部文句,理清思路,把握文段内容,抓住前后的衔接词、照应词。综合来看,几个句子的中心议题是“清明节人们为什么要在门楣、窗户上插上柳条”。答题时既要抓住前后的衔接词、照应词排序,又要注意横线间的标点符号,注意层次。六句话中间的句号,暗示包含两层意思:(1)谈传统文化中柳的象征意义,句号前两横线暗示此层只能有两句,分析不难看出是③⑥两句;(2)谈人们的企盼及做法,包括①②④⑤四句。因为六句的中心议题是“清明节人们为什么要在门楣、窗户上插上柳条”,⑥句中“由于”衔接上文,领起下文;而③句的“它”代指⑥句中的“生殖力”,与后面的“生命力”相照应,因此第一层排序应为⑥③。④句中的“生命力”应衔接第③句中的“生命力”,因此④句排在前面;而④句中的“企盼”“家庭成员”又分别与①句中的“达到”“人丁”衔接照应,因此①句排在④句后;②句中“于是”暗示本句是对上文的总结,它与⑤句一起放在结尾,因此第二层排序应为④①②⑤。‎ ‎14.[2015·安徽高考]请根据上下文,在材料的空白处补写恰当的内容。要求:语意连贯,表达简明,每处不超过12个字。‎ 文学形象如同绘画形象,一样是心中的形象,一样全凭虚构,一样先要用心来看。写人、写物、写环境,必须看得逼真,直至看到细节,方能落笔。__①__,绘画是片段静止的文学。文学用文字作画,所有文字都是色彩;__②__,所有的线条色彩都是语言。画非画,文非文,画亦文,文亦画。我画,不过再现一句诗、一阕词、一段散文而已;站在画面上千姿万态的树,全是感动过我的不同境遇中的人物,或者全是我自己;淌过纸表的流水,不论舒缓、疾进,还是迷茫虚渺,更是我一时真切的情绪,这与写作的心态又有何异?‎ 答:①__________________________________________________ ‎ ‎②_____________________________________________________‎ 答案 ①文学是绵延不断的画面 ②绘画用线条色彩写作 解析 解答此题,首先,准确理解所给材料的意思,提炼出两个要点:文学和绘画。其次,联系上下文可以发现,①②两处都是文学和绘画并举,由第①句后面将“绘画”比喻为“片段静止的文学”可知,第①句应是将“文学”比作“绘画”,句子采用“文学是……的画面”的形式。根据第②句分号前面的“所有文字都是色彩”和后面的“所有的线条色彩都是语言”相对可知,第②句所填内容应与“文学用文字作画”相对,即“绘画用……”。再次,注意句式的整齐。最后,注意字数的限制。‎ ‎[2年全国模拟重组]‎ ‎1.[2017·沧州一中月考]下面的一段文字中加点的词语,有的必须删去,有的不能删去,有的可删可不删。请把必须删去的和不能删去的找出来,把各自序号填在相应的横线上。‎ 为了感谢广大读者朋友们①长期以来对本刊的②支持,进一步地③提高本刊的④质量,更好地满足大家的⑤阅读要求,现本刊拟⑥决定开展读者调查活动,读者意见已⑦附在本期中,本刊⑧希望广大读者认真地⑨填写读者意见,并⑩及时寄回本刊编辑部。‎ ‎(1)必须删去的是:___________________________________ _____________________________________‎ ‎(2)不能删去的是:_________________________________ _______________________________________‎ 答案 (1)①⑥⑧ (2)②⑤‎ 解析 ①与“广大”重复,⑥与“决定”矛盾,⑧多余了,应承前省略;②⑤删掉后会改变短语结构,而改变文意。‎ ‎2.[2017·阳泉三校联考]本着语言表达简明的原则,修改下面的句子。‎ 这艘新舰艇,机器性能良好,如果按照措施上规定的延长机油使用期的方法,来延长机油使用期,就可避免不必要的人力和机油的浪费。‎ 答:____________________________________________________ ____________________________________________________________‎ 答案 这艘新舰艇,机器性能良好,如果按照措施上规定的延长机油使用期的方法来做,就可避免人力和机油的浪费。‎ 解析 第二个“延长机油使用期”与前面重复,可用“做”代替,同时去掉前面的逗号。“不必要的”多余,浪费都是“不必要的”,没有“必要的浪费”。‎ ‎3.[2016·哈尔滨三中三模]下面语段中有些词语使用不简明,请找出需删去的词语。‎ 市场管理员说:“目前,个体户收入悬殊很大。他们当中的多数是守法的;个别人有违法行为,对这些人已经依法给予了法律制裁。同时还要加强对他们的教育,绝不能让社会上那种坑害顾客的思想任其泛滥。”‎ 答:____________________________________________________ ____________________________________________________________‎ 答案 很大、法律、任其。‎ 解析 “悬殊”就是相差很远的意思,“很大”可删去;“依法”和“法律”可任意去掉其中的一个;“任其”也有“让”的意思,与前面的“让”重复,可删去“任其”。‎ ‎4.[2017·湖南五市十校联考]下面文字中画线部分的词语,有的使用不当,请指出并改正,使这段文字语言简明,衔接自然,语意连贯。‎ 苏泽广真是哭笑不得,觉得儿子合图还不懂事,把家托付给是徒劳的,便失望地起身。然而他刚要离开,突然跳下椅子,吹灭桌前的蜡烛,“扑通”一声跪在地上,抱住的腿,在黑暗中说:“爸爸,你放心吧,要是不回来,管这个家!”‎ 答:____________________________________________________ ____________________________________________________________‎ 答案 ①删除或者改为“他”;③改为“合图”;④删除或改为“他”;⑤改为“苏泽广”或者“父亲”。‎ 解析 ‎ 解答语言运用类题目时,要注意审题。题干往往明确地提出要求,或暗示答题要领。如本题题干中的“语言简明”就是要求不重复,没有歧义。如将①④删除或改为“他”,就避免了语意重复,也达到了简明的效果;将③改为“合图”,使指代明确。“衔接自然,语意连贯”就是要求考生注意上下文在结构上、主语上、语势上等保持前后一致,如将⑤改为“苏泽广”或“父亲”,使指代明确,前后文衔接、连贯。‎ ‎5.[2016·浏阳一中模拟]阅读下面的材料,完成后面的题目。‎ ‎4月15日,清华、北大各自公布了专门针对农村学子的自主招生计划,扩大面向边远、贫困、民族地区农村考生的计划招生人数。入选清华“自强计划”的优秀农村学子,最高可降65分;获北大“筑梦计划”入选资格的考生,最高可获降至当地本科一批控制分数线录取的优惠政策。此消息一出,便在网友中引发热议。有人点赞,也有人指责。‎ 你对高校为农村学子降分的政策有何看法?请简要阐述。(要求:表达简明连贯,言之成理,120字左右)‎ 答:____________________________________________________ ____________________________________________________________‎ 答案 (示例一)我认为高校为农村学子降分的政策能在一定程度上促进教育公平。各级政府在农村教育上投入的物质、人才等力量不如城市教育,农村学子无法与城市学子一样享受到优质的教育资源,无法尽显其才。高校降分政策对农村学子有所倾斜,为他们提供了更多深造成才的机会,可以算作是对农村教育的一种补偿,是力促教育公平的努力作为。‎ ‎(示例二)我认为高校为农村学子降分的政策无法真正体现教育公平。能够被高校降分政策福泽到的农村学子毕竟是极少数,相较于面广量大的农村生源,降分的举措是杯水车薪。而且高考分数1分之差就能卡住成百上千的考生,如此大幅度的降分“优惠”可能会使高考招生陷入另一种不公平,特别是对同样努力奋斗的城市考生。所以为农村学子降分的政策无法真正体现教育公平。‎ 解析 这是一道答案并不唯一的题目。解答此题时,既可发表赞成此项政策的看法,也可发表不赞成此项政策的看法。但不论何种看法,都应持之有据,言之有理,有理有据。‎ ‎6.[2016·安徽示范高中联考]请根据示意图,提取文字材料中的相应信息,并用一句话表述出来。要求:简明、准确,不超过40个字。‎ ——―→ ‎4G是第四代通讯技术的简称,G是generation(一代)的简称。4G系统能以100Mbps的速度下载,比拨号上网快2000倍,比3G网络快10倍,上传的速度也能达到20Mbps。4G能满足几乎所有用户对于无线服务的要求,可实现无线区域环路、数字音讯广播等方面的无线通信增值服务。4G不再局限于电信行业,其技术还可应用于金融、医疗、教育等行业,使局域网、互联网、电信网等融为一体组成一个通播网,无论使用什么终端,都可享受高品质的信息服务,向宽带无线化和无线宽带化演进。‎ 答:____________________________________________________ ____________________________________________________________‎ 答案 4G是通信速度更快、增值服务更多、技术融合更强的第四代通讯技术。(或:4G即第四代通讯技术,通信速度更快,增值服务更多,技术融合更强。)‎ 解析 解答该类题目时,要注意抓住原文的关键词,如“4G”、“第四代通讯技术”、三点优势等,然后根据原意按照题目要求作答即可。‎ ‎7.[2016·广东六校联盟三联]将下列语句重新排序,衔接最恰当的一组是(  )‎ ‎①假如大家都能不折不扣地把这句话当成真理,‎ ‎②因为,如果只是由于我们的判断,坏事才进入我们的世界,‎ ‎③那么,人类的不幸就可得以缓解。‎ ‎④那么,我们完全可以对它们嗤之以鼻,‎ ‎⑤古希腊有条格言说:“人通常被看待事物的方法困扰,而并非事物本身。”‎ ‎⑥或者把它们变为好事。‎ A.⑤②④⑥①③ B.⑤①③②④⑥‎ C.①③②④⑥⑤ D.①④⑥②③⑤‎ 答案 B 解析 解答本题,可结合语段内容及具体语境分析判断,⑤提出格言,①③对格言进行引申并得出关于幸福的结论,①中“这句话”代指上文的格言,关联词“假如……那么”为一组句群,②④⑥依据格言解释“幸福”这一结论并提出做法,④⑥中代词“它们”指代②中的“坏事”,“如果……那么”是又一组关联词句群。故选B。‎ ‎8.[2017·百师联盟联考]填入下面一段文字横线处的语句,最恰当的一句是(  )‎ 从科技性上来说,手机“飞行模式”的推出________。有的“飞行模式”仅是关闭了通话功能,有的是在硬件上停止基带芯片,还有的是关闭包括WiFi、蓝牙等一切无线连接……由此可见,各个手机厂商对“飞行模式”的设置并不相同,“飞行模式”是一个既没有国际标准,也没得到航空行业认证的功能。‎ A.并不是为了在飞机上使用手机,它只是模拟了飞行时“没网”的状态 B.是因为在飞机上不能使用手机,所以模拟了飞行时“没网”的状态 C.是因为模拟了飞行时“没网”的状态,所以在飞机上不能使用手机 D.并不是模拟了飞行时“没网”的状态,它只是为了在飞机上能使用手机 答案 A 解析 全句强调的是手机的“飞行模式”只是手机厂商开发的一个功能,而不是针对飞机上使用而推出的。据此,表因果关系的B、C两项首先排除。根据后几句的意思可知语段强调的是“飞行模式”的设置,所以选A项。‎ ‎9.[2016·苏锡常镇四市二模]在下面一段文字横线处填入语句,衔接最恰当的一项是(  )‎ 在表面上,诗与散文的分别似乎很容易认出,但是如果仔细推敲,________,________。________,________,________,这不是易事,但也不是研究诗学者所能避免的。‎ ‎①说明诗是什么,散文是什么 ‎②从历史经验看 ‎③寻常所认出的分别都不免因有例外而生出问题 ‎④要了解诗与散文的分别,是无异于要给诗和散文下定义 ‎⑤从亚里士多德起,这问题曾引起许多辩论 A.③⑤②④① B.③②⑤①④‎ C.④①③⑤② D.④①②③⑤‎ 答案 A 解析 从逻辑关系看,④后应为①,③应在开头,因为“寻常所认出的”与前文“很容易认出”是对应的逻辑关系,故答案为A。‎ ‎10.[2016·江淮十校模拟]把下列句子组成语意连贯的语段,排序最恰当的一组是(  )‎ ‎①立秋至处暑,秋阳肆虐,温度较高,加之时有阴雨绵绵,湿气较重,天气以湿热并重为特点。‎ ‎②秋季的气候是处于“阳消阴长”的过渡阶段。‎ ‎③故有“秋老虎”之说。‎ ‎④“白露”过后,雨水渐少,天气干燥,昼热夜凉,气候寒热多变,稍有不慎,容易伤风感冒,许多旧病也易复发。‎ ‎⑤被称为“多事之秋”。‎ ‎⑥因此,秋季养生在对精神情志、饮食起居、运动导引等方面进行调摄时,应注重一个“和”字。‎ ‎⑦由于人体的生理活动与自然环境变化相适应,体内阴阳双方也随之发生改变。‎ A.②①③④⑤⑦⑥ B.①③⑤④②⑦⑥‎ C.②③①④⑤⑥⑦ D.②⑤④①③⑦⑥‎ 答案 A 解析 第一步:观察四个选项可知,第一句不是②就是①。揣摩两句的含意,发现第②句属于总起句(总领句),所以排除B项。第二步:根据第①句中“秋阳肆虐,温度较高”及第③句“故有‘秋老虎’之说”,使用“语意排序法”“词语排序法”可知,①③句存在因果关系,第①句后面应该跟第③句,据此可排除C项。第三步:使用“语意排序法”“词语排序法”,由第④句中的“容易伤风感冒,许多旧病也易复发”及第⑤句“被称为‘多事之秋’”可知,④⑤句同样存在因果关系,第④句后面应该跟第⑤‎ 句。最终确定答案为A。‎ ‎11.[2017·荆州中学摸底]在下面一段文字横线处补写恰当的语句,使整段文字语意完整连贯,内容贴切,逻辑严密。每处不超过15个字。‎ 年味在哪里?在吃里?在看里?不!那只是媒介。__①__。人伦情义是中华年的灵魂,中华大家庭各民族皆然。要注入情感,要营造表达情感的环境、气氛和载体。是什么伟力竟能驱动30亿人次投入春运,成就地球上大迁徙的壮观?——__②__。游子们带回的是亲情、是温暖。这就是中国人的感情世界,这就是中国人的精神家园,这就是中国人的过年历程。__③__,就是凝聚力的精神资源,就是向心力的精神元素。‎ 答:①__________________________________________________ ‎ ‎②_____________________________________________________‎ ‎③_____________________________________________________‎ 答案 ①年味在情感里 ②回家过年,天伦之乐 ③国家情怀,家国一体 解析 该题需要把握语段的叙述重点,分清句间关系,把握前后文的信息提示。如第①处根据后文的信息提示,可以得到“情感”这一关键字眼,第②处之前的破折号应该是解释说明的作用,结合自己的生活经验,②处与前文组成一个设问句,即“回家过年,共享团圆”;第三处需要对前文进行有效的升华,前文只有家,后文上升到中国人的高度,从而得出“家国”这一关键信息。‎ ‎12.[2017·牡丹江摸底]在下面一段文字横线处补写恰当的语句,使整段文字语意完整连贯,内容贴切,逻辑严密。每处不超过12个字。‎ 今天为什么还要读中国古代的经典?主要是文化传承的需要。如果你不想完全抛弃自己的民族文化传统,那么阅读代表自己民族文化传统的典范性文本,__①__。其意义又不止于此。就个人的修养而言,阅读经典文本是使阅读者经历一番文化濡染的过程,它可以改变人的气质。受传统文化熏陶比较多的人和受熏陶比较少的人,__②__。很多人都发生气质的变化,一个时代的社会风气就会随之发生变化。所以阅读本民族的文化经典在个人,可以改变气质;对社会而言,__③__。‎ 答:①__________________________________________________ ‎ ‎②_____________________________________________________‎ ‎③_____________________________________________________‎ 答案 ①是继承传统的一种必要方式 ②其气质风度是截然不同的 ③可以转变风气(意思对即可)‎ ‎13.[2016·山西四校联考]下面的文稿在表达和形式上有五处不妥,请指出并改正。‎ 通告 为保障春节期间用电通畅,我公司决定对辖区电网进行维护,现将有关事宜宣布如下:‎ 维护将于2016年1月20日7时至20日19时施工。涉及范围:东到云中路,西到牧马路,南至长征街,北至雁门大道。‎ 对维护工作给贵用户造成的不便,我们深表不安。请予理解和支持。‎ 如有疑问,欢迎垂询。‎ ‎2016年1月15日 忻府区电力公司 答:____________________________________________________ ____________________________________________________________‎ 答案 ①“宣布”改为“通告”“通知”或“公告”;②“维护……施工”改为“维护……展开”“维护……进行”或“我公司……施工”;③“贵用户”改为“您”或“各位用户”或删去“贵”字;④“不安”改为“歉意”;⑤落款单位应在日期之上。‎ 解析 解答此题,首先要清楚“不妥”所包含的内容,如病句、用词甚至标点和格式等。如本题是一则通告,不需要过度放低姿态,客观陈述事实就可以,“贵”是敬辞,“贵用户”改为“您”或“各位用户”或删去“贵”字;“不安”夸大了心意,可以改为“歉意”;落款单位应在日期之上。‎ ‎14.[2017·南阳质检]根据下面的新闻内容,写一段评论性文字,不超过70个字。‎ 张先生和弟弟、妹妹去爷爷家吃晚饭。饭桌上,爷爷想和年轻人聊天,结果三兄妹各拿各的手机玩,受冷落的爷爷说了句“你们就和手机过吧”‎ ‎,便一怒之下摔盘子离席。这样的事情,每个城市都有发生。这平常却戏剧性的一幕,触动了许多人的心弦。现在,网络上流行的一句话是:世界上最遥远的距离莫过于一家人坐在一起,你却在玩手机。‎ 答:____________________________________________________ ________________________________________________________________________________________________________________________‎ 答案 莫让手机伤害亲情。手机在给人们生活带来便利的同时,因使用不当而伤害亲情的事每天都在发生。珍视亲情,要从放下手机,尊重并与亲人真诚交流做起。‎ 解析 本题考查语言表达准确、鲜明的能力。解答本题时,首先要准确抓住新闻所反映的主要社会现象——人们沉迷于手机而忽略了与家人的交流和沟通。其次用准确的语言表达出自己对此种社会现象的看法,还要阐明支撑自己观点的理由。‎ ‎15.[2017·汕头模拟]请根据下面的材料,为高二语文选修课程的宣传海报撰写正文部分。要求:三门课程中任选一门,突出该课程内容的特点,至少运用一种修辞手法,不超过100字。‎ 某校高二年级语文组拟开设“中国传统文化”系列选修课,这一系列包括“中国饮食文化”“中国琴棋文化”“中国建筑文化”三门课程;分别由赵、钱、孙三位老师授课。每门课程拟招收30名学员,报名截止时间为9月28日。‎ 答:____________________________________________________ ________________________________________________________________________________________________________________________‎ 答案 (示例一)中国饮食文化:酸甜与麻辣的比拼,清淡与浓烈的交锋,粤湘川鲁四大菜系风味各具,精彩纷呈。自诩为“吃货”的美食大家赵老师将为同学们奉上这道文化大餐。报名截止时间为9月28日,同学们赶紧行动,本课程只招收30人,不容错过哦!‎ ‎(示例二)中国琴棋文化:高山流水传情韵,楚河汉界斗机谋。琴棋是文人雅士的挚爱,它们陶冶情操,寄托怀抱,滋润心灵。让我们跟随我校风流倜傥的钱老师,一起领略中国琴棋文化的真谛。本课程招收30人,报名时间于9月28日截止。快来报名吧!‎ ‎(示例三)中国建筑文化:建筑是凝固的音乐,是立体的画卷。从建筑中你能体味到劳动人民的智慧,你能收获到愉悦身心的美感。选择“建筑文化”吧,学养深厚的孙老师将带你走进一个美的世界。招收名额:30人。报名截止时间:9月28日。‎ 解析 撰写海报正文时,在内容上,要交代清楚课程内容、任教老师、招收名额、报名截止日期等,要具有号召力,以吸引同学们报名;在形式上,要注意至少运用一种修辞手法,字数上不要超过100字。‎ ‎16.[2017·杭州七校联考]目光敏锐、思想深刻、直言不讳是现代人应有的风范。对于社会现象,要敢于发表自己的看法和认识,要一针见血。认真阅读下面的两个微博,分别对其进行点评。(每个不超过20字)‎ ‎(1)【社会现象】近日,福州一家培训机构的一名外教在斑马线打太极拳拦车的行为引起了网友关注。据报道,这名来自芬兰的外教曾连续五天在一斑马线处观察,发现几乎没有车子在斑马线礼让行人。于是,他决定在过马路时用自己学来的中国太极拳动作提醒司机减速慢行。‎ 答:____________________________________________________ ________________________________________________________________________________________________________________________‎ ‎(2)【社会现象】如果你在校园里看到这样一个小柜子,里面的笔、橡皮、夹子、纸巾等物品无人看管,自取所需,自主投钱,你会付钱吗?河南师范大学校园内就出现了这样一个“诚信小柜”。小柜子不仅没有亏损,半个月来还盈余50多元钱。‎ 答:____________________________________________________ ________________________________________________________________________________________________________________________‎ 答案 (1)“斑马线打太极”打出“车德之失”。‎ ‎(2)“诚信小柜”,“小”亦可贵。(不拘泥于现有答案,只要言之成理即可)‎ 解析 本题考查语言表达准确、鲜明、生动、得体的能力。解答此题,要抓住现象的本质所在,然后谈自己的看法、认识或态度,该褒则褒,该贬则贬。另外,在用语方面,要准确、鲜明、生动、得体,符合“‎ 一针见血”的要求。‎ ‎17.[2017·武邑中学测试]在下面一段文字横线处补写恰当的语句,使整段文字语意完整连贯,内容贴切,逻辑严密。每处不超过12个字。‎ 大约公元前4000年,尼泊尔人发明了自己的“楔形文字”。这些文字__①__,少许几笔即可完成。泥版是尼泊尔人最主要的书写材料,除此之外,__②__。但是,木板文书不易保存,所以现在__③__,即便有些存留下来,也是破损严重;坚固的石料在当地又极为稀少,几乎成为神庙或皇室专用。相比之下,泥版成了最好的选择。‎ 答:①__________________________________________________ ‎ ‎②_____________________________________________________‎ ‎③_____________________________________________________‎ 答案 ①写法简单 ②木板和石料也曾充当过书写材料 ③它们基本都已消踪灭迹 解析 这是一道根据文意填空的题目。题干要求“逻辑严密”,这样就要注意上下文之间的关系,前后文之间的相互的提示和照应,尽量用文中的词语组织句子进行填写,这样能够减小误差。如此题中的“写法简单”“木板和石料”“消踪灭迹”在前后文都有相应的类似或语意相反的提示性的词语,可组织它们作答。‎ ‎18.[2017·四川棠湖中学模拟]在下面一段文字横线处补写恰当的语句,使整段文字语意完整连贯,内容贴切,逻辑严密。每处不超过15个字。‎ 人类为什么要有底线?为了生存。人,是社会的群居物。任何人,都不能一个人活在这世界上。所以,只有让别人生存,__①__;让别人活得好,自己才活得好。希望所有的人都活得好,甚至为了别人的生存放弃自己的利益,这是“境界”。至少不妨碍别人的生存,__②__,不破坏社会的环境,这是“底线”。境界不一定人人都有或要有,__③__。因为底线是人类生存的基础,是最后一道防线。基础不牢,地动山摇;防线失守,全盘崩溃。‎ 答:①__________________________________________________ ‎ ‎②_____________________________________________________‎ ‎③_____________________________________________________‎ 答案 ①自己才能生存 ②不侵犯别人的利益 ③但底线却不能缺失 解析 ①横线处后面是分号,根据上文“只有让别人生存”和下文“让别人活得好,自己才活得好”可知,①处应填入“自己才能生存”。从②处前后句子可以看出,三个句子之间有递进关系,中间句应该是“不侵犯别人的利益”。③处后面强调了底线的重要性,说它是人类生存的基础,是每个人必须有的东西,再根据“境界不一定人人都有或要有”,可推断出③句是对上一句的转折,可填“但底线不能缺失”。‎ ‎19.[2017·湖北枣阳月考]在下面一段文字横线处补写恰当的语句,使整段文字语意完整连贯,内容贴切,逻辑严密。每处不超过15个字。‎ 冬虫夏草也叫虫草,是一种名贵的中药材。其实__①__,而是__②__。秋末冬初,一些昆虫的幼虫在土里潜伏过冬时,有些真菌便沾在幼虫身上,在虫体里萌发菌丝,吸取幼虫的营养。当把幼虫吃光只剩下空壳时,__③__,形成菌核。次年夏天,菌核从虫壳的头上长出,好似一棵小草,“冬虫夏草”也由此而得名。‎ 答:①__________________________________________________ ‎ ‎②_____________________________________________________‎ ‎③_____________________________________________________‎ 答案 ①它既不是虫也不是草 ②一种真菌和昆虫的混合体 ③大量的菌丝便塞满昆虫的躯体(意思正确即可)‎ 解析 这是一道语言运用的综合考核的内容,注意上下文的衔接和前后的照应。根据“其实”一词,可看出此处表示语意的转折,结合后面的实际情况,可知①应填“它既不是虫也不是草”;根据“而是”可知此处应填写冬虫夏草的真实情况,根据后文可知其为“一种真菌和昆虫的混合体”,根据“幼虫吃光只剩下空壳时”,可知③应填“大量的菌丝便塞满昆虫的躯体”。‎ ‎20.[2017·集宁一中月考]古人有名有字。名与字意义上往往有联系,或同义,或相关,如岳飞字鹏举,意思是鲲鹏高飞。给下面的人名取字,并说明字与名的意义关联。(各不超过25字)‎ ‎(1)陈璋,字________。说明:________________________________ ‎ ‎_____________________________________‎ ‎(2)孙冠群,字________。说明:____________________________ ____________________________________________‎ 答案 (示例)(1)玉成 “璋”,玉器名,字“玉成”,表示玉汝于成。‎ ‎(2)谦之 “冠群”意为出类拔萃,字“谦之”,以谦虚戒之。‎ 解析 本题考查语言表达简明、连贯、得体、准确、鲜明、生动的能力。注意字和“名”之间的关系,字要从“名”衍生而来,然后分析名、字之间的关系。‎ 考点七 表达创新与图文转换 考点名片 考点内容 ‎①数据分析类:表格、坐标图(柱状图、曲线图)、饼状图;②图片解读类:徽标(图标)、漫画、照片(画像)、结构图(流程图)、框架图;③贴近生活的应用语言表达题应是语言创新题的考查重点。‎ 考查形式 有数据分析和图片解读两类题,要求考生解读分析后按要求用文字表述。简答题,4~6分。‎ 趋势分析 该考点近年相对固定,应该是选贴近生活的表格、图画作品和常见语言类试题,与修辞、语言表达等综合考查。‎ 题组1 语言创新 ‎1.阅读下面两段文字,回答问题。‎ 子贡问曰:“有一言而可以终身行之者乎?”子曰:“其恕乎!己所不欲,勿施于人。”‎ ‎(《论语·卫灵公》)‎ 从文化的角度来考量,确实有很多事情很难用先进与落后、文明与愚昧来衡量。这种把自认为是先进的、是文明的东西强加给别人的做法,我觉得不适合这个社会。因此“己之所欲,也不要强加于人”。(莫言《己之所欲,也不要强加于人》)‎ ‎(1)第一则材料中画线句子的意思是:_______________________‎ ‎(2)两则材料都体现了处理人际关系的重要原则,请概括其相同点。‎ 答:___________________________________________________‎ ‎___________________________________________________________‎ 答案 (1)自己不想要的,就不要强加给别人。(2)①人应该有宽广的胸怀,待人处事切勿心胸狭窄,而应宽宏大量。②坚持这种原则,是尊重他人、平等待人的体现。‎ ‎2.请阅读下列语段,写出画线句子的言外之意,并就此加以简要点评。‎ 台后一帘深色幕布,台上一架钢琴,柔和的灯光洒在黑白键上,人们屏息等待。女钢琴家悄然出现,衣着简朴。演奏家上台,谁不身着华美的演出服,光彩夺目?人们就此问她,她的回答是:“人,要隐于音乐背后。”‎ 言外之意:_____________________________________________‎ ‎___________________________________________________________‎ 点评:_________________________________________________‎ ‎___________________________________________________________‎ 答案 言外之意:作为一名演奏者,应该展现出音乐的魅力,而不是展示个人形象。(意思对即可)‎ 点评:女钢琴家的回答告诉我们:不要只关注外在的东西,而应该追求内在的修养。(点评符合题意、言之有理即可得分)‎ ‎3.下面是某班竞选班级体育委员公告中的六项内容,请按性质的不同将它们分为两类,并为每类命名(每类不超过4个字)。‎ ‎①具有良好身体素质及团队合作精神。‎ ‎②有较强的沟通、协调和组织能力。‎ ‎③协助体育老师顺利完成课堂任务。‎ ‎④掌握班级同学的体能信息,负责学校各项体育活动中本班人员的安排和调配。‎ ‎⑤热爱班级工作,掌握体育运动的相关知识。‎ ‎⑥协助班主任组织同学安排课间操等日常工作。‎ 答:第一类(填序号):________;命名:________‎ 第二类(填序号):________;命名:________‎ 答案 第一类:③④⑥ 命名:岗位职责(或职责范围、工作内容等,意思正确即可) 第二类:①②⑤ 命名:任职要求(意思正确即可)‎ ‎4.阅读下面两则材料,回答问题。‎ 材料一 大学生就业难的问题日益受到关注,大学生实习与就业息息相关。某网站就大学生实习问题在网上进行调查,情况如下:你最希望通过实习获得什么?工作经验69%,正式工作机会21%,实习鉴定7%,其他3%。‎ 材料二 随着社会的发展,竞争日趋激烈。用人单位在招聘大学生时,除了要求必备的专业知识外,还在社会实践经验、心理素质、沟通交流能力、团队合作能力等方面提出不同程度的要求。‎ 综合两则材料,请概括要点;就大学生培养问题给高校提出合理建议。‎ 答:___________________________________________________‎ ‎___________________________________________________________‎ ‎___________________________________________________________‎ ‎___________________________________________________________‎ 答案 要点:大学生希望通过实习获得工作经验,用人单位对应聘大学生提出综合素质要求。(意思对即可)‎ 建议:高校要适应社会需求,要为大学生搭建参与社会实践活动的平台,要提高大学生的综合素质。(言之成理即可)‎ ‎5.下面是《阿Q正传》的片段,试比较原稿和修改稿的优劣,并说明理由。‎ 要求:①理由充分、合理;②语言表达准确、简明。‎ 原稿:人人愿意知道现钱和新夹袄的阿Q的中兴史,看去是天然的事,所以在酒店里,茶馆里,庙檐下,便渐渐的问出来了。这结果,是阿Q得了新敬畏。‎ 修改稿:人人都愿意知道现钱和新夹袄的阿Q的中兴史,所以在酒店里、茶馆里、庙檐下,便渐渐的探听出来了。这结果,是阿Q得了新敬畏。‎ 答:___________________________________________________‎ ‎___________________________________________________________‎ ‎___________________________________________________________‎ ‎___________________________________________________________‎ 答案 (示例一)修改稿优。作者用“都”强调人人想知道阿Q的“中兴史”,故删去“看去是天然的事”,使行文更简洁。“探听”更形象、深刻,既活画出未庄人对阿Q“中兴史”的浓厚兴趣和询问之细致,又语含讽刺,批判了未庄人爱管闲事的劣根性。‎ ‎(示例二)原稿优。“人人愿意”即含有“全部”的意思,比加“都”更简洁;“看去是天然的事”深刻讽刺了未庄人爱看热闹、管闲事的劣根性;“问”字既有“探听”之意,又符合农民的身份。(理由充分、合理,表达准确、简明即可)‎ 题组2 表文转换 ‎【表格类】‎ ‎6.下面为近几年来我国国民不同阅读方式的调查统计表,阅读后请按要求答题。‎ 阅读 年份  ‎ 网上在线阅读 手机在 线阅读 电子书 阅读 纸质图 书阅读 ‎2012‎ ‎30.1%‎ ‎29.0%‎ ‎3.9%‎ ‎37.0%‎ ‎2013‎ ‎31.4%‎ ‎31.8%‎ ‎4.0%‎ ‎32.8%‎ ‎2014‎ ‎32.9%‎ ‎33.6%‎ ‎4.1%‎ ‎29.4%‎ ‎2015‎ ‎34.6%‎ ‎36.1%‎ ‎4.2%‎ ‎27.1%‎ ‎(1)请概括出我国国民阅读方式的变化趋势。‎ 答:___________________________________________________‎ ‎___________________________________________________________‎ ‎(2)这一变化趋势说明什么?‎ 答:___________________________________________________‎ ‎___________________________________________________________‎ 答案 (1)网上在线阅读、手机在线阅读、电子书阅读均呈上升趋势,纸质图书阅读呈下降趋势。‎ ‎(2)数字化阅读方式越来越受人们青睐,传统阅读方式受到冲击,纸质图书出版业面临着严峻的挑战。‎ ‎7.请阅读下面某校对高三部分班级高三第一次作文书写及错别字情况的统计表,根据要求回答后面的问题。‎ 高三(1)、(20)班第一次作文书写及错别字情况统计表 班级 书写 错别字 备注 整洁 潦草 无 有 高三(1)班 ‎40%‎ ‎60%‎ ‎17%‎ ‎83%‎ 理科班 高三(20)班 ‎85%‎ ‎15%‎ ‎70%‎ ‎30%‎ 文科班 请用简明的语言分别概括两个班的基本情况和主要问题。要求:不得直接引用表中统计数字,每条不超过30字(含标点符号)。‎ 答:___________________________________________________‎ ‎___________________________________________________________‎ ‎___________________________________________________________‎ 答案 ①理科班过半同学书写及错别字两项问题很大,后者问题尤为严重。②文科班过半同学两项统计情况良好,错别字问题比书写问题大些。‎ 解析 基本情况:包括书写、错别字统计内容的班级基本情况。主要问题:班级在统计数据中显现出的主要问题。‎ ‎【柱状图类】‎ ‎8.阅读下面的清华大学有关自主招生的调查统计表,然后回答问题。‎ ‎(1)根据图表内容,得出结论。(不超过26字)‎ 答:___________________________________________________‎ ‎(2)针对自主招生的现状,提出一条具体的建设性意见。‎ 答:___________________________________________________‎ 答案 (1)与城市学生相比,农村的“寒门学子”处于劣势。‎ ‎(2)(示例一)要尽量增加农村学生在自主招生中的名额。‎ ‎(示例二)有必要采取多元的录取方式。‎ ‎(示例三)对农村的“寒门学子”可适当降分录取。‎ 解析 解答图文转换题,要注意三个方面:①把握图表横轴、纵轴代表的意义;②找出纵、横轴中的最大数和最小数;③理解最大数、最小数体现的意义。这个图表,横轴是不同家庭所在地的学生在清华大学自主招生名额中的分布情况;纵轴(柱形)是不同地域在自主招生中通过的比例,其中直辖市通过率最高,农村通过率最低。从这个角度分析,结论是与城市学生相比,农村学生清华大学自主招生通过率低,即处于劣势。对于建设性意见,可以从对农村学生政策倾斜的方面阐释。‎ ‎9.阅读反映浙江省新高考改革方案实施前后考生专业选择面变化情况的两张图表,回答下面问题。‎ ‎(1)比较上面两张图表,可以得出一个结论:‎ ‎___________________________________________________________‎ ‎(2)你认为新高考方案实施后,学生获得的最大实惠是:‎ ‎___________________________________________________________‎ ‎___________________________________________________________‎ ‎___________________________________________________________‎ 答案 (1)新高考方案实施后,考生的专业选择面有较大程度的拓宽。‎ ‎(2)可以根据自己的兴趣,扬长避短,选择自己最擅长的科目作为选考科目,使自身利益最大化。(意思对即可)‎ ‎【曲线图类】‎ ‎10.校园电视台“新闻快递”和“时事酷评”栏目准备各招一名学生记者。请你根据下图的内容,续写一段话,推介张亮、李小明两位同学到合适的栏目。‎ 答:___________________________________________________‎ ‎___________________________________________________________‎ ‎___________________________________________________________‎ ‎___________________________________________________________‎ 答案 (示例)张亮同学议论类文章水平高,笔锋犀利,触觉灵敏,见解独到,做“时事酷评”栏目的记者非常合适;李小明同学记叙类文章写得好,他笔触细腻,能及时捕捉到校园的新鲜事,适合做“新闻快递”栏目的记者。‎ 解析 本题考查表文转换的能力。回答此题,首先要读懂折线图。要读懂折线图,关键在于要分清、搞懂各项“参数”的代表意义,如纵坐标、横坐标各代表什么;还要抓住曲线的变化趋势,在纵或横的对比中,把握对象的特征或变化趋势。综观此图发现,张亮议论类文章的写作能力比李小明好,而李小明记叙类文章的写作能力比张亮强。‎ ‎11.图中的两条曲线均略呈现U型,好似微笑的嘴型,因此被形象地称为“微笑曲线”。仔细观察这幅图,用简洁、准确的语言描述“微笑曲线”反映的经济现象,不超过60个字。‎ 答:___________________________________________________‎ ‎___________________________________________________________‎ ‎___________________________________________________________‎ 答案 产业链上游的研发企业和下游的营销企业利润都要高于制造企业;这种利润差别在六七十年代不明显,现在却非常突出。(“研发”“营销”“制造”后没有“企业”也可以)‎ 解析 这是一道分析统计图的题目,注意曲线的趋势,和曲线的名称以及线条的对比。对统计图表首先注意横纵坐标表明的是利润和产业链,注意三个阶段“研发”“制造”和“营销”,然后可以分析三者之间的关系,最后再对比现在和60~70年代的关系。‎ ‎【饼形图类】‎ ‎12.根据下面的图表提供的信息,将农村转移劳动力务工行业分布的特点写成一段话。要求内容完整,语意连贯,不超过80个字。‎ 答:___________________________________________________‎ ‎___________________________________________________________‎ ‎___________________________________________________________‎ 答案 农村转移劳动力集中分布于低技能劳动密集型行业,主要是制造业和建筑业。此外是就业门槛相对较低、与日常生活有较大联系的行业,如批发和零售业、住宿和餐饮业等。‎ 解析 本题考查语言表达简明、连贯、准确和图文转换的能力。解答图文转换题的关键是准确把握图表信息,然后将图表信息转换成文字表述出来。回答本题时,首先要注意题干的提示和要求。其次要注意对其特点进行归纳,如批发和零售业、住宿和餐饮业是就业门槛相对较低、与日常生活有较大联系的行业,而制造业和建筑业都是低技能劳动密集型行业,归纳时最好不要出现具体数字。‎ ‎13.阅读下面的图表,完成其后试题。‎ ‎(1)图表反映了什么问题?‎ 答:___________________________________________________‎ ‎(2)出现这种问题的原因是什么?请结合现实,谈谈你的看法。‎ 答:___________________________________________________‎ ‎___________________________________________________________‎ ‎___________________________________________________________‎ 答案 (1)大部分市民对政府治霾政策信心不足。‎ ‎(2)近年来雾霾问题严重,影响到了人们的生活,但始终没有得到解决;环境污染严重,治霾工作难度较大;出台的一些治霾政策大多是干预措施,治标不治本。‎ 解析 回答第(1)问,只需对图表中的文字进行分析,即可由图表标题和图中数据得出答案。第(2)问,原因是什么?图表中并没有提示,需要考生“结合现实”来谈,这就考查了考生对现实的关注和思考能力。考生的思维要发散,从市民、霾、政策几个方面来考虑。‎ ‎【结构流程图类】‎ ‎14.“翻转课堂”是一种新的教学模式,在教与学的活动上,它与传统教学有所不同。根据下面图中提供的信息,请用一段文字概括说明“翻转课堂”与传统教学的本质区别。要求:内容完整,语言连贯,不超过100个字。‎ ‎(图注:图中三角形里的内容是教学活动所要实现的六个目标)‎ 答:___________________________________________________‎ ‎___________________________________________________________‎ 答案 “翻转课堂”是将传统教学中课堂与课外的功能和顺序进行了倒置,即把识记、理解这样难度比较小的目标放在课外学习,而把应用、分析、评价和创造等难度比较大的目标放在课堂中进行,老师给予学生比较多的帮助。(如有其他答案,只要符合要求,可酌情给分;字数超出要求,酌情扣分)‎ 解析 作答本题,首先要认真看图,看清图表内容和意思,然后再组织语言。在本图中,“翻转课堂”和传统教学在课堂与课外教学目标的顺序上正好相反,抓住这一点,结合图表上的文字组织答案即可。‎ ‎15.下面是“需求侧三驾马车与供给侧四大要素”图示,请写出与标题有关的主要信息,并概括经济增长的途径。要求内容完整,表述准确,语言连贯,主要信息不超过30字,经济增长的途径不超过60字。‎ 需求侧三架马车与供给侧四大要素 答:___________________________________________________‎ ‎___________________________________________________________‎ ‎___________________________________________________________‎ 答案 信息:需求侧有投资等三驾马车,供给侧有劳动力等四大要素。途径:①通过需求侧直接刺激经济增速;‎ ‎②通过需求侧刺激推动供给侧改革,拉动经济潜在增速,最终实现经济增速。‎ ‎16.下面是海南省居民健康档案建立流程图,请把这个流程写成一段话,要求内容完整,表述准确,语言连贯,不超过75个字。‎ 答:___________________________________________________‎ ‎___________________________________________________________‎ ‎___________________________________________________________‎ 答案 居民档案的建立从登记居民个人基本信息开始,逐步完善为家庭基本信息,再汇总为居民健康档案,最后按照新生儿、儿童、成年(孕产妇)和老年四类人群进行分类。‎ 解析 本题考查语言表达准确、连贯的能力。一是审清试题要求,即根据流程图构思一段话;二是充分利用试题材料,将其体现在语段中;三是注意要求。‎ ‎17.《世界卫生组织烟草控制框架公约》在我国已经生效十年了,但目前仍存在烟草企业不积极履行公约相关内容的行为。北京市消费者协会曾发布了“在知道吸烟会引发多种疾病的前提下,是否还会吸烟”的调查数据。请把下面的图表写成一段话。要求:表述准确,语言连贯,不超过75个字。‎ 答:___________________________________________________‎ ‎___________________________________________________________‎ 答案 吸烟易引发肺癌、肺气肿、心脏病、口腔癌等多种疾病,在得知吸烟的这些危害后,被调查者有将近六成人表示不会吸烟,但仍有四成以上的人选择吸烟。‎ ‎【示意图类】‎ ‎18.下面是用“金字塔”的形式描述的“学习方法及其效果”示意图。请根据图示,就某一点说说你的体会,50字左右。‎ 答:___________________________________________________‎ ‎___________________________________________________________‎ ‎___________________________________________________________‎ 答案 (示例一)学以致用是良方。学习后马上就应用,所学的知识才能巩固,如果不用,很快就会遗忘。所以,边学边用是最好的学习方法。‎ ‎(示例二)要变被动为主动。传统的听讲是被动的学习方法,效果最差。要把被动的“听”变为积极主动的思考和运用,才能学有所得。‎ ‎(示例三)实践、交流是正道。学习效果高于50%的方法,都与实践、交流密切相关。所以要勤于实践,加强交流,这是学习的正道。‎ ‎19.劳务派遣是一种新型的现代人力资源管理方式。下面的示意图反映了劳务派遣的基本流程和当事三方的关系,请根据图中的信息,用文字描述劳务派遣的基本流程(环节)。‎ 根据图示,劳务派遣的流程(环节)依次为:‎ ‎(1)_____________________________________________________‎ ‎___________________________________________________________‎ ‎(2)_____________________________________________________‎ ‎___________________________________________________________‎ ‎(3)_____________________________________________________‎ ‎___________________________________________________________‎ 答案 (1)用人单位与劳务派遣单位签订劳务派遣协议,确立劳务关系;(2)劳务派遣单位和被派遣的员工签订劳动合同,建立劳动关系;(3)用人单位和被派遣的员工签订岗位协议,确立劳务使用关系。‎ ‎20.根据所给材料及地图,补写出下面的问路情景对话中小明应回答的部分。要求:(1)内容清晰,能指引对方顺利到达目的地。(2)表达得体,符合情境。‎ 中午,小明放学后,在校门口遇见一位叔叔向他问路,下面是两人的对话。‎ 叔叔:同学,我要去省四院的北门,请问你知道怎么走吗?‎ 小明:①_______________________________________________‎ ‎___________________________________________________________‎ ‎___________________________________________________________‎ ‎___________________________________________________________‎ 叔叔:谢谢你,同学。‎ 小明:②______________________________________________‎ 答案 ①‎ 您可以从这里向北走到第一个路口,就是正东路,向右转,向东走到右边第二个路口是栗康街,左转向北,走到右边第二个路口是健康路,再向东走一点就看到省四院的北门了。②不客气,叔叔再见。‎ 题组3 图文转换 ‎【图片、漫画类】‎ ‎21.阅读下面这幅公益广告画(主体是一双一次性筷子,背景是一片沙土),根据要求答题。‎ ‎(1)请为画面拟写一条宣传标语。(不超过20字)‎ 答:___________________________________________________‎ ‎(2)请简要评价这幅公益广告画的创意。‎ 答:___________________________________________________‎ ‎___________________________________________________________‎ ‎___________________________________________________________‎ 答案 (1)(示例一)拿起它,你将吃掉整个森林。‎ ‎(示例二)拯救森林,对一次性筷子说不。‎ ‎(示例三)告别一次性筷子,给世界多留点绿色。‎ ‎(2)(示例)一次性筷子上面画着一排树木,意味着一次性筷子是用树木做成的;而使用一次性筷子,则将树木拦腰砍断,无异于毁坏森林,最终造成地球沙漠化。构图简洁而寓意深刻,令人震撼,催人反省,具有强烈的警示效果。‎ 解析 (1)宣传标语要精练,具有鼓动的表达效果,一般会运用对比、夸张、比喻、借代等修辞手法。题中的公益广告画里,主体是一双一次性筷子,背景是一片沙土,其寓意显而易见,即请大家不要用一次性筷子,保护森林,拯救地球。(2)首先要对画面予以描述,在此基础上再揭示其寓意,评价其创意和艺术效果。‎ ‎22.2016年3月2日,在台湾屏东县一所连锁餐馆里,一名餐厅员工细心地拿着汉堡,让双手残疾的顾客进食。请就图片发表感想,要求紧扣图片内容,不超过60个字。‎ 答:___________________________________________________‎ ‎___________________________________________________________‎ ‎___________________________________________________________‎ 答案 ‎ 拿起汉堡很容易,但并不是每一个人都愿意为一个残疾人拿起它。因为拿起它需要的不仅仅是力气,更是一颗善良与助人为乐的心。‎ 解析 解答此题,需要结合图片所展现的内容发表感想。餐厅员工拿着汉堡让双手残疾的顾客进食,这件事情展现出的是人物善良和助人为乐的精神。答题时抓住图片中最感人的地方,围绕其所展现的精神这一点展开。注意字数要求。‎ ‎23.仔细观察下面三幅图片,结合图后的说明和你所了解的知识,写出你思考的结果。‎ 说明:图一是泰国皇宫洗手间的一句中文告示:请便后冲水!图二是珍珠港景区美国人特意在垃圾桶前写下的中文告示:垃圾桶在此!图三是巴黎圣母院的一则中文告示:请保持安静!‎ 据说,这是他们专门写给中国人看的,而其他国家的游客没有这种“礼遇”。‎ 答:___________________________________________________‎ ‎___________________________________________________________‎ ‎___________________________________________________________‎ 答案 在国外的一些旅游景区,一些中国游客有不注意卫生、不讲文明的习惯,以致受到特别提醒。‎ 解析 解答本题应该抓住两个要点:一是结果,二是原因。结果指的是画面上的文字信息。分析时可以采用由果溯因法,思考外国景区为什么专门用中文告示告知游客,那么中国游客的行为怎么样,由此便可以推断出来了。‎ ‎24.下面的漫画《“房娃”厌学》反映了一种较常见的社会现象,请针对此现象写一则点评,要求语言简练,观点鲜明。‎ 答:___________________________________________________‎ ‎___________________________________________________________‎ ‎___________________________________________________________‎ ‎___________________________________________________________‎ 答案 (示例一)在过度功利化的教育背景下,读书是为了谋好职,赚大钱,改变命运。于是,面对“几辈子吃不完”的房租,孩子迷惘了。“房娃”厌学的背后,折射的是功利化教育的尴尬和困境。‎ ‎(示例二)孩子生病,根子则在家长身上。今天的社会又有多少父母对暴发的财富有一个正确的认知?家长的无知直接导致孩子的扭曲、畸形。‎ ‎(示例三)童言无忌,孩子一时迷惘并不可怕,可怕的是我们的社会在“富起来”面前不知所措,陷于价值观的混乱之中。(可以从教育孩子、家长、社会这三个角度的任一角度或其中两个角度点评)‎ ‎25.观察下面这幅关于手机的漫画,请以生动形象的语言为其题诗或配文。‎ 要求:①符合画面情景;②表达画面内涵;③不超过40字。‎ 答:___________________________________________________‎ ‎___________________________________________________________‎ ‎___________________________________________________________‎ ‎___________________________________________________________‎ 答案 (示例)①手机似美女,梦中亦共舞。心醉意迷间,已无自我在。②手机奴役了人,共舞成为痛苦,总被手机牵制而行,人的自我何处寻找?‎ ‎【徽标、图章类】‎ ‎26.下面是“圆梦1+1”爱心助学公益活动标志。请写出该标志的构图要素,并说明图形寓意。要求:语意简明,句子通顺,80字左右。‎ 答:___________________________________________________‎ ‎___________________________________________________________‎ ‎___________________________________________________________‎ ‎___________________________________________________________‎ ‎___________________________________________________________‎ ‎___________________________________________________________‎ 答案 主体图案为一个人扶助另一个人,右下角为活动主题“圆梦1+1”。将两个人组合形成一个心形,表达了爱心公益活动的主旨;两人手挽手以象形“+”‎ 号相连,象征着爱心团体或个人对困难群体的关爱和帮扶;标志的下部线条形成“梦”字的拼音首字母“M”,凸显出“圆梦1+1”公益活动的主题。‎ ‎27.请阅读这幅端午节标志图片,写一段文字说明画面内容,要求语言简明、准确。(不超过80字)‎ 答:___________________________________________________‎ ‎___________________________________________________________‎ ‎___________________________________________________________‎ ‎___________________________________________________________‎ ‎___________________________________________________________‎ 答案 图片中主体外形像个粽子,也是人形,一个人坐在船上,手握着桨在划船。说明端午节主要活动是赛龙舟、吃粽子。主体部分下面是三个汉字“端午节”,再往下是大写的英文“Dragon Boat Festival”,意思是“龙舟节”。‎ 解析 解答本题时,图片说明要完整,说明主体、汉字、英文以及它们之间的关系;主体的解释说明要清晰,粽子、划龙舟的篆体人形;说明的语言要有层次性和条理性。‎ ‎[3年高考真题集训]‎ ‎1.[2016·全国卷Ⅰ]下面是某校“中华文化体验”计划的初步构思框架,请把这个构思写成一段话,要求内容完整,表述准确,语言连贯,不超过85个字。‎ 答:___________________________________________________‎ ‎___________________________________________________________‎ ‎___________________________________________________________‎ ‎___________________________________________________________‎ 答案 (示例)本次“中华文化体验”计划开设旗袍、围棋、国画三个讲座,并开展三项活动:利用体育课体验太极拳,利用手工课体验中国结和剪纸艺术,年终举行太极拳表演和作品展示。‎ 解析 这是一个“中华文化体验”计划的初步构思框架。“中华文化体验”分活动与讲座两部分。活动包括体育课上的太极拳体验,手工课上的中国结与剪纸体验,还有年终的表演、展示。讲座包括旗袍、围棋、国画等内容。把所给信息用连贯的话语连缀起来表达即可。‎ ‎2.[2016·全国卷Ⅱ]下面是某校团委“中国梦演讲赛”工作的初步构思框架,请把这个构思写成一段话,要求内容得当,表述准确,语言连贯,不超过85个字。‎ 答:___________________________________________________‎ ‎___________________________________________________________‎ ‎___________________________________________________________‎ ‎___________________________________________________________‎ 答案 (示例)“中国梦演讲赛”拟于5月4日举行,赛事需要组织和宣传。组织工作需要联系报告厅,选拔20名参赛者,最后评出6个奖项;宣传工作包括出海报、组稿,并在学校网站和校报报道。‎ 解析 首先要读图,读图时要全面理解图中各个组成部分的含义,包括图形的名称、标注等;具体到此图,则要重点分析组织工作与宣传工作之间的交叉关系,然后用准确、简明、连贯的语言表达出来。叙述时要把握框架的结构,按照自上而下的顺序逐层介绍。‎ ‎3.[2016·全国卷Ⅲ]下面是某高中举办迎新生晚会的构思框架,请把这个构思写成一段话,要求内容完整,表述准确,语言连贯,不超过80个字。‎ 答:___________________________________________________‎ ‎___________________________________________________________‎ ‎___________________________________________________________‎ ‎___________________________________________________________‎ 答案 (示例)本次迎新生晚会由高二、高三同学排练演出节目,节目内容以反映中学生学习生活为主,形式不限。要提前通知各年级并动员相关同学积极参加,演出后进行评奖。‎ 解析 这是一个有关迎新生晚会的构思框架图,全图共分三个层级:第一层为主题,即迎新生晚会;第二层分通知动员、演出、评奖三个环节;第三层级有参演人员、演出内容、表演形式(不限)。按照由下到上或由上到下的顺序阐述图中信息均可,但要注意不要遗漏信息。‎ ‎4.[2015·全国卷Ⅰ]下面为中国邮政为保护地球水环境发行的邮票中的主体图形,请写出构图要素,并说明图形寓意,要求语意简明,句子通顺,不超过80个字。‎ 答:___________________________________________________‎ ‎___________________________________________________________‎ ‎___________________________________________________________‎ 答案 (示例)该图由地球、清流、鱼、手和浊流构成。地球上各种鱼在清澈的水流里游动,人类之手正在阻挡排向清流中的污水,整个图形表达了人类保护水环境、拒绝水污染的决心。‎ 解析 本题考查图文转换及语言表达简明、连贯、准确的能力。解答本题要注意题干中的关键词,一是“构图要素”,二是“寓意”。再结合题干中的“保护地球水环境”这一背景,寓意就应该容易理解了。在回答时除了“鱼”“手”“清流”和“污水”外,还要看到“手”挡住了“污水”,这点集中体现了这一图画的寓意,也就是保护水环境、拒绝水污染。‎ ‎5.[2015·全国卷Ⅱ]下面是联合国发行的“联合我们的力量”‎ 邮票中的主体图形,请写出构图要素,并说明图形寓意,要求语意简明,句子通顺,不超过85个字。‎ 答:___________________________________________________‎ ‎___________________________________________________________‎ ‎___________________________________________________________‎ 答案 (示例)图形由橄榄枝和多面旗帜组成,这些旗帜又巧妙地构成一只飞翔的鸽子。旗帜代表不同国家,鸽子代表和平,飞鸽衔着橄榄枝,强化了和平寓意,整个图形表示各国应齐心协力、维护世界和平。‎ 解析 本题考查图文转换的能力。先找出构图要素,再总结寓意。总结寓意时,要结合“联合我们的力量”这句话。‎ ‎6.[2014·全国卷Ⅰ]下面是某中学暑期瑶族村考察的初步构思框架,请把这个构思写成一段话,要求内容完整,表述准确,语言连贯,不超过75个字。‎ 答:___________________________________________________‎ ‎___________________________________________________________‎ ‎___________________________________________________________‎ 答案 (示例)本次瑶族村三日行考察要求参加人员事先查阅资料,了解瑶族概况,备好所需行装;考察期间的主要活动有参观、访谈以及与村民联谊,每人需写日记记录考察情况。‎ 解析 首先认真观察构思框架图,理清活动的先后顺序,然后语言简练、内容全面贴切、上下连贯地进行表述。‎ ‎7.[2014·全国卷Ⅱ]下面是某班级春游活动的构思框架,请把这个构思写成一段话,要求内容完整,表述准确,语言连贯,不超过75个字。‎ 答:___________________________________________________‎ ‎___________________________________________________________‎ ‎___________________________________________________________‎ 答案 (示例)本次春游全班分为5个小组,以组为单位准备所需物品,要求人人参与,组长负责协调;各组拿出美食进行班级评比,并参加游艺活动。‎ 解析 本题考查图文转换的能力。读图时要全面理解图中各个组成部分的含义,包括图形的名称、标注等;具体到此图,还要重点分析小组与班级之间的交叉关系,然后用准确、简明、连贯的语言表达出来。‎ ‎8.[2015·江苏高考]下列对“中国文化遗产”标志理解不恰当的一项是(  )‎ A.标志整体呈圆形,既体现民族团结、和谐包容的文化内涵,也体现文化遗产保护的理念。‎ B.标志中的太阳神鸟图案动感很强,既体现中国文化强大的向心力,也体现自强不息的民族精神。‎ C.标志中的神鸟与太阳光芒的数目,暗合中国文化中四季、四方、十二生肖、十二时辰等元素。‎ D.标志中光芒四射的太阳,既象征着光明、生命和永恒,也象征着我国飞速发展的文化产业。‎ 答案 D 解析 本题属徽标题,解答这类题要注意以下几点:(1)宏观把握徽标的外形特点,注意中英文大小写和变体以及涉及的时间、地点等;(2)要分清画面说明的顺序(时间、空间、逻辑);(3)注意由表及里,分析其内涵和寓意,对图标的创意,要联系具体对象做出合理想象,切忌随意猜想,要扣住图中信息点来联想;(4)要认真观察图画组成部分,避免遗漏说明内容,或疏漏创意之处;(5)一般采用规范的说明语言,做到准确、简明、平实、清晰。D项明显违背第(3)点,对徽标中间图形的内涵和寓意理解有误,它是动感很强的“太阳神鸟”,D项却理解为光芒四射的“太阳”,想象也不够合理。‎ ‎9.[2015·广东高考]下图是北京市控烟协会遴选的两个劝阻吸烟的手势,分别是“我介意”和“不可以”。请写一段话分析这两个手势的劝阻效果。要求语意完整,前后连贯,不少于50字(含标点符号)。‎ 答:___________________________________________________‎ ‎___________________________________________________________‎ ‎___________________________________________________________‎ ‎___________________________________________________________‎ 答案 “我介意”的手势表达委婉,劝阻效果相对较弱。“不可以”的手势直接具体,劝阻效果较强。‎ 解析 这是一道图文转换的题目,首先明确画面内容的内涵,然后分析效果。‎ ‎10.[2015·安徽高考]请根据示意图,提取文字材料中的相应信息,并用一句话表述出来。要求:简明、准确,不超过70字。‎ ‎2015年4月15日,亚洲基础设施投资银行的57个意向创始成员国已全部确定。在完成各国审批程序后,亚投行有望在2015年底之前正式成立投入运行。‎ 据测算,2010~2020年,亚洲每年大约需要8000亿美元的基础设施投资,而现有的世界银行、亚洲开发银行等国际多边机构都无法满足需求,亚投行将有效弥补其中的资金缺口,具体方式有贷款、股权投资以及提供担保等。‎ 可以预期,在亚投行的支持下,亚洲各国将掀起新一轮基础设施建设高潮,建设项目集中在公路、铁路、港口、通信、电力电网、油气运输等方面,这必将带动亚洲经济未来的强劲增长。‎ 答:___________________________________________________‎ ‎___________________________________________________________‎ ‎___________________________________________________________‎ ‎___________________________________________________________‎ 答案 (示例一)亚投行将采用贷款、股权投资等方式,在交通、通信、能源等基础设施建设领域向亚州各国提供资金支持,这必将带动亚洲经济未来的强劲增长。‎ ‎(示例二)亚投行在交通、通信、能源等基础设施建设领域以贷款、股权投资等方式向亚州各国提供的资金支持必将带动亚洲经济未来的强劲增长。‎ 解析 本题考查提取信息和语言概括能力。看图一定要注意文字关系,寻找图表和文字材料的共同点。提取概括的方法:寻找主语“亚洲基础设施投资银行”;提出问题“怎么了”——将掀起亚洲新一轮基础设施建设的高潮。就基本概括完毕了。‎ ‎11.[2015·湖北高考]南水北调中线干线工程输水路线,源起湖北丹江口水库,终至北京、天津。请依据下图,用一段文字描述该干线工程的输水路线。要求:①包含图示总干渠经过地;②不少于5个动词;③语言表达准确、简明、连贯;④不超过80字。‎ 答:___________________________________________________‎ ‎___________________________________________________________‎ ‎___________________________________________________________‎ ‎___________________________________________________________‎ 答案 (示例)一渠清泉,源自丹江口水库,从南阳上郑州,穿越黄河,过焦作,出安阳,入河北邯郸,经邢台,走石家庄,分流东至天津,北达北京。源源南水北流去,绵绵千里饮甘泉。‎ 解析 本题考查图文转换的能力。要求描述输水路线,该路线从图中能轻易找到,重要的是要选择合适的动词,表达要顺畅。‎ ‎12.[2015·重庆高考]下图是“儿童在不同场地的时间分配”,根据该图提供的信息,将儿童在不同场地休息和运动时间分配的情况写成一段话。要求:内容完整,概括准确,不超过80字。‎ 答:___________________________________________________‎ ‎___________________________________________________________‎ ‎___________________________________________________________‎ ‎___________________________________________________________‎ ‎___________________________________________________________‎ 答案 (示例)女孩在五种场地原地休息的时间均多于进行运动的时间,而男孩只有在硬场地和天然场地原地休息的时间多于进行运动的时间。女孩在五种场地原地休息的时间均多于男孩。‎ 解析 ‎ 图文转换题是语言表达题的一类常见题型。要正确作答这类试题,必须全面理解图文信息。就本题而言,既要分别指出男孩女孩在不同场地原地休息和进行运动的时间的差别;也要指出在时间分配上男孩女孩之间的差异。‎ ‎[2年全国模拟重组]‎ ‎1.[2017·海南七校联盟联考]下面是家长对网络的态度和孩子上网相关行为的情况调查表,请结合具体项目和数据完成后面的问题。‎ 项目 赞成(%)‎ 反对(%)‎ 不置可否(%)‎ 对子女上网 ‎27‎ ‎15.9‎ ‎57.1‎ 希望子女以后工作与网络有关 ‎42.6‎ ‎32‎ ‎25.4‎ 能使子女增长见识,掌握电脑技巧 ‎90.3‎ ‎8.6‎ ‎1.1‎ 网络能促进子女的学校学习的兴趣 ‎29.5‎ ‎11‎ ‎59.5‎ 孩子上网会浪费时间,耽误学业 ‎58.5‎ ‎10.8‎ ‎30.7‎ ‎(1)通过图表可以看出,多数家长对于子女上网的内心矛盾在于:___________________________________________________________‎ ‎___________________________________________________________‎ ‎(2)请从学生、家长和学校三个角度提出合理化建议。(每一角度不超过12个字)‎ 答:___________________________________________________‎ ‎___________________________________________________________‎ 答案 (1)既希望自己的子女能享受网络的好处,又怕他们因迷恋网络而荒废学业。‎ ‎(2)学生:趋利避害,为我所用。家长:积极引导,平等交流。学校:加强管理,严格规定。(言之成理即可)‎ 解析 第(1)问,可抓住数字对比悬殊的项目进行分析,将图表上的文字信息转化为自己的语言。‎ 第(2)问,可根据不同对象提出不同建议,学生侧重趋利避害,家长侧重引导,学校侧重管理。‎ ‎2.[2017·宜昌统考]分析下图,得出结论,并合理推断其原因。要求:①语言表达准确、简明;②结论和原因均不超过25字。‎ 宜万铁路开通前后恩施州公路、铁路和民航客运量对比 答:___________________________________________________‎ ‎___________________________________________________________‎ ‎___________________________________________________________‎ 答案 (1)结论:铁路开通后,客运量超过公路、民航,成为出行首选。(或公路客运量骤减,铁路客运量大幅上升)‎ ‎(2)原因:铁路出行比公路速度快,比民航费用低。(或:铁路速度提高,且经济、安全、省时)‎ 解析 本题考查图表内容转换成文字的能力。这个图表,主要内容为公路、铁路、民航客运量的对比,主图为柱形图,解题时要注意横坐标、纵坐标所标注的数据,全面、准确地分析表格中的数字以及呈现出的规律,通过比对,按设题及答题要求,概括出结论和原因。‎ ‎3.[2016·衡水二中模拟]阅读下面的图表,回答问题。‎ ‎(1)根据图表,写出一条结论。‎ 答:___________________________________________________‎ ‎(2)根据图表揭示的现象,用简明的语言谈谈你的看法。‎ 答:___________________________________________________‎ ‎___________________________________________________________‎ ‎___________________________________________________________‎ 答案 (1)结论:随着学历和家庭收入水平的提高,消费者购买的奢侈品数量也在增加。‎ ‎(2)看法:(示例)奢侈品的消费主要还是要量力而行,不能因为别人在消费,就盲目攀比,在自身条件不允许的情况下进行消费,将会带来更多的负面影响。‎ 解析 本题考查表文转换的能力。解答此题,要读懂折线图的基本内容。折线图的基本内容包括相关数据、相关项目,要通过两个图形内容的对比,发现规律性的东西。也就是学历、家庭收入水平的高低与购买奢侈品数量的多少的关系如何,还要针对这种现象提出自己的看法。‎ ‎4.[2017·宜昌一中测试]下面是关于理想的一个框架图,请你把它写成一段话,要求内容完整,表述准确,语言连贯,不超过70个字。‎ 答:___________________________________________________‎ ‎___________________________________________________________‎ ‎___________________________________________________________‎ 答案 我们树立理想的前提是要保证它科学合理,并且要具体可实施。在实现理想的过程中,我们需要踏实,坚韧不拔,同时还要具备良好的心态。‎ ‎5.[2017·焦作联考]下图是某医院的“就诊流程图”,请把这个就诊流程写成一段话,要求内容完整,表述准确,语言连贯,不超过85个字。‎ 答:___________________________________________________‎ ‎___________________________________________________________‎ 答案 病人在门诊挂号,经医生检查、诊断后,需要住院的,拿医生开的住院单去挂号收费处缴费住院;不需要住院的,拿医生开的处方去缴费取药后离院,如需输液,就到急诊科输液后离院。‎ 解析 本题考查将流程图转换为文字的能力。首先要明确叙述对象应是“病人”,表述时“病人”应是整个句子的主语;其次要运用“后”“接着”“再”之类的词清晰地表述出流程的变化,条理要清晰。考生要特别注意“需要住院的”和“不需要住院的”两条流程,不能混淆在一起表述,还要注意字数的限制。‎ ‎6.[2017·合肥二中模拟]请为右面的漫画拟写题目(不超过8个字),并写出漫画寓意(不超过30个字)。‎ 题目:_________________________________________________‎ 寓意:_________________________________________________‎ ‎___________________________________________________________‎ ‎___________________________________________________________‎ 答案 题目:盘根错节,人机难离。寓意:手机已经生根于身体,人已经异化为手机的奴隶。(意思对即可。字数超出要求,酌情扣分)‎ ‎7.[2016·南昌一模]下面是中国邮政邀请黄永玉先生设计的2016年一套两枚猴票《丙申年》的主体图形。请选择其中的一幅,简要介绍画面内容,并说明图形寓意。要求语意简明,句子通顺,不超过80个字。‎ 答:___________________________________________________‎ ‎___________________________________________________________‎ ‎___________________________________________________________‎ ‎___________________________________________________________‎ 答案 图(一)内容:猴子一只手攀着树枝,尾巴缠在桃树上,另一只手捧着桃子。寓意:“捧桃献瑞。”‎ 图(二)内容:一只大猴双腿盘坐着,双手分别抱着两只可爱的小猴,两只小猴亲吻大猴。寓意:“合家欢乐。”‎ 解析 抓漫画中的要素,第一幅漫画:猴子、桃子,画面是猴子一手抓着树枝,一手捧着桃子,寓意祝福吉祥。第二幅漫画:一只大猴盘坐并抱着两只小猴,小猴亲吻大猴,寓意合家欢乐。根据图示,只要寓意出喜庆的氛围即可。‎ ‎8.[2017·大连双测]下面是有关地下水污染的图片,请用一段话表述图中的内容,要求内容完整,表述准确,语言连贯,不允许出现数字,不得超过95个字。‎ 答:___________________________________________________‎ ‎___________________________________________________________‎ ‎___________________________________________________________‎ ‎___________________________________________________________‎ 答案 地下水污染来自工业污染、农业污染和生活污染。工业污染来自矿渣废液渗漏、工业废水排放和石油煤炭开采污染;农业污染来自过量使用农药和化肥渗透;生活污染来自生活污水排放和垃圾填埋渗漏液。‎ 解析 从图示来看,主要讲地下水污染的来源,表述的时候要注意大类和小类之间的关系,还要注意表述全面且重点突出。如先谈三大类,再谈每类中的具体污染来源。‎ ‎9.[2017·太原五中阶段检测]右面是中国人民银行的标志,请写出该标志的构图要素及其寓意,要求语意简明,句子通顺,不超过90字。‎ 答:___________________________________________________‎ ‎___________________________________________________________‎ ‎___________________________________________________________‎ 答案 中国人民银行标志由3个古代春秋战国时期流行的布币组成,整个外形像一个“人”字,中心构成一个“人”字,古代布币的形象近似“人”字。3个“人”字有机组合,寓意“众”多。三角形的构成也创造了一种稳定的感觉,也寓意了稳定的金融体系。‎ 解析 本题考查图文转换的能力。解答该题,应当认真审读所给图形,确定其中包含的构图要素,如三个古代布币,整体的人形等;然后再根据“中国人民银行”这一标志的主体来判断,从而确定其中的寓意,分析合理,意思对即可。‎ ‎10.[2017·河池模拟]下面是我国的“节约粮食宣传画”,请写出该图画的构图要素及其寓意,要求语意简明,句子通顺,不超过90字。‎ 答:___________________________________________________‎ ‎___________________________________________________________‎ ‎___________________________________________________________‎ 答案 图画由双手捧大米、蓝天、白云、阳光、白鸽、清新背景、农民插秧等元素构成。手捧大米象征着珍惜粮食,农民插秧象征着粮食来之不易,蓝天、白云、阳光、白鸽、清新背景象征着美好的生活。‎ 解析 解答此题,首先注意题干的提示和要求,其次要认真观察图画,再根据要求作答。‎ ‎11.[2016·邯郸调研]下面是某校新生入学报到流程图,请用文字转述新生报到流程,要求简洁、明了。‎ 答:___________________________________________________‎ ‎___________________________________________________________‎ ‎___________________________________________________________‎ ‎___________________________________________________________‎ 答案 已缴费的学生可直接到所在学院报到,然后入住公寓。未缴费的学生先到银行收费点缴纳学费,然后再到所在学院办理报到、入住手续;其中需要办理绿色通道的同学,则须到学院审批,经计财处核准后,方可办理报到、入住手续。‎ 解析 作答本题,首先要找出说明顺序,可以先说明“已缴费学生”,再说明“未缴费学生”,对“未缴费学生”还要按“缴纳学费”和“绿色通道”两类予以说明。‎ ‎12.[2016·潍坊统考]下面是某商业网站对本网站用户行为分析的构思框架,请把这个构思写成一段话,要求内容完整,表述准确,语言连贯,不超过100个字。‎ 答:___________________________________________________‎ ‎___________________________________________________________‎ ‎___________________________________________________________‎ 答案 网站可从黏性、活跃、产出三个角度进行分析。其中黏性关注用户对网站的访问频率和访问间隔时间;活跃关注用户访问网站的平均停留时长和平均访问页面数;产出关注用户访问网站所产生的订单数和客单价。‎ ‎13.[2017·汕头金山中学摸底]下面是5个城市角逐2020年奥运会承办权的流程框架图,请把这个流程写成一段话,要求内容完整,表述准确,语言连贯,不超过100个字。‎ 答:___________________________________________________‎ ‎___________________________________________________________‎ ‎___________________________________________________________‎ ‎___________________________________________________________‎ ‎___________________________________________________________‎ 答案 此次角逐首先进行第一轮投票,如果有一个城市得票过半,那么这个城市就获得主办权;如果所有申办城市得票都不过半,则淘汰得票最少的城市,然后重复上述投票过程,直到选出一个主办城市为止。‎ ‎14.[2017·江西上高二中模拟]下面是一幅公益广告图片“祖辈的井、父辈的井、子孙的井”,请简要介绍图片的主要内容并揭示其寓意,要求语意简明,句子通顺,不超过90个字(标点不要单独占格)。‎ 答:___________________________________________________‎ ‎___________________________________________________________‎ ‎___________________________________________________________‎ ‎___________________________________________________________‎ 答案 图示以三口井为主体,每口井都配有一个木桶,而打水的绳子依“祖辈的井、父辈的井、子孙的井”的顺序大幅增长。寓意:人们对地下水资源的过度利用,导致地下水资源日渐匮乏。呼吁人们要合理开发、利用和珍惜水资源。‎ 解析 本题考查图文转换的能力。首先要认真观察图示,找出三口井的相同和不同之处,再结合配图文字,描述出直观内容,注意语言简洁。揭示寓意要能透过现象看本质,追溯井绳越来越长的原因,由此联想到水资源匮乏,联系社会现实,进而揭示深刻内涵。‎ ‎15.[2016·山西康杰中学模拟]下面的两幅饼状图是2015年3月福布斯中文网对我国大众富裕阶层(俗称“白领”)的年龄及学历分布情况调查结果,请根据图表所反映的情况补充下面文段中空缺的内容,使上下文连贯。要求:补充内容不出现图表中的具体数据。‎ 中国大众富裕阶层年龄及学历分布情况 调查显示,中国的大众富裕阶层在年龄结构方面,__①__,而“80后”在其中所占的比重也越来越大。大众富裕阶层的学历普遍较高,本科及以上学历的总占比超过三分之二。综合以上两图,大众富裕阶层的总体形象可以大致概括为是一群__②__。‎ 答:①_________________________________________________‎ ‎②_____________________________________________________‎ 答案 ①“60后”与“70后”占较大比重 ②有文化的中青年人 解析 根据所给内容及所缺句子的前一句可以从“年龄”和“文化程度”方面考虑。‎ ‎16.[2017·成都龙泉中学模拟]5月31日是世界无烟日,某校“健康与生命”研究小组做了“居民禁烟支持度”调查。阅读下面的调查数据统计图,回答问题。‎ ‎(1)概括图表反映出的信息。(不超过30字)‎ ‎30‎ ‎(2)请你任选图表中的一个场所拟写一条禁烟提示语。(不超过20字)‎ ‎20‎ 答案 (1)除大部分吸烟者不支持酒店禁烟外,居民普遍支持公共场所禁烟。‎ ‎(2)(示例)学校:儿童易模仿,请您勿吸烟!‎ 医院:您的指尖夹着他人的生命,请勿吸烟!‎ 酒店茶楼:提神不妨清茶,消愁莫若朋友,何必吸烟!‎ 解析 “学校”“医院”“酒店茶楼”“公交工具”“办公场所”等地均属公共场所,统计表中显示,除了酒店这一场所有较大争议外,无论是吸烟者还是不吸烟者几乎都支持其他公共场所应禁烟。写禁烟提示语,重在观点明确,有警示提醒的作用,同时应分别根据场所的不同,选择恰当的切入角度和表达用意角度。‎ ‎17.[2016·江淮十校三联]下面是“乡村学校从教30年教师荣誉徽标评选入围作品”的主体图形,请写出除文字外的构图要素,并说明图形寓意,要求语意简明,句子通顺,不超过80个字。‎ 答:___________________________________________________‎ ‎___________________________________________________________‎ ‎___________________________________________________________‎ ‎___________________________________________________________‎ ‎___________________________________________________________‎ 答案 ‎ 徽标由蜡烛、书、田野组成,蜡烛象征老师的奉献精神,书象征教育,田野象征乡村学校和希望;徽标寓意是乐于奉献的乡村教师传授知识、给乡村带来希望。‎ 解析 回答问题注重把握题干中给出的徽标标题体现的意义和与之相关的徽标的构图要素。从“乡村学校从教30年教师”的角度分析,构图要素有书、蜡烛、田野等;从“乡村教师从教”的角度分析,书象征教育、教学,蜡烛象征老师以及老师的奉献精神,田野象征乡村;从整体寓意的角度分析,徽标体现了乡村教师的奉献精神、教育给农村带来的希望等。‎ ‎18.[2016·重庆一中模拟]下面是文章写作中有关写作材料收集后的处理方式,请把这个处理方式写成一段话,要求内容完整,表述准确,语言连贯,不超过75个字。‎ 答:___________________________________________________‎ ‎___________________________________________________________‎ ‎___________________________________________________________‎ ‎___________________________________________________________‎ 答案 材料收集好后,需按照取舍标准选择材料,材料选择不仅要适切题目,而且还要注重特色,材料选择好后,就整合同类材料,并将材料中冗繁的部分删去。‎ 解析 分析出有关材料的内在联系,再按题目要求(句式、字数)归纳概括,就符合简答要求了。材料收集好后,处理分两步,介绍时注意先后顺序。‎ ‎19.[2017·惠州调研]请你提出修改意见,使下图文字内容变得准确、简明、得体。‎ 答:___________________________________________________‎ ‎___________________________________________________________‎ 答案 将“做饭阿姨”改为“厨师(炊事员、厨工、掌厨、掌勺)。‎ 解析 “阿姨”与“男女不限”矛盾。‎ ‎滚动提升训练(二)‎ ‎  时间:60分钟   满分:87分 ‎1.[2017·海南七校联盟联考]下列各句中,加点的成语使用不恰当的两项是(3分)(  )‎ A.从历史上看,人类社会每一个新时代的开启,似乎都与技术变革息息相关。如今,正是新的技术革命爆发的前夜。‎ B.埃塞姑娘鼻子挺直,柳眉大眼,皮肤细嫩光滑,带有与其他非洲国家女性明显不同的面貌特征,风情万种、落落大方而又长袖善舞。‎ C.法规起草工作中存在一些粗枝大叶的现象:一些法规草案体例结构不合理,条款之间逻辑关系不严谨,条文表述不规范。‎ D.美国总统奥巴马在五角大楼就打击“伊斯兰国”组织的战略及进展发表电视讲话,强调美国主导的战略是卓有成效的。‎ E.虽然美式橄榄球,不可能像足球那样让伦敦万人空巷,但体育场座无虚席、蓊蓊郁郁的盛况,给我留下了深刻的印象。‎ 答案 BE 解析 A项,息息相关:形容关系密切。B项,长袖善舞:多用来形容有财势、有手腕的人善于钻营取巧。不合语境,应该用“能歌善舞”。C项,粗枝大叶:形容不细致,做事粗心大意。D项,卓有成效:有卓越的成绩、效果。E项,蓊蓊郁郁:树木茂盛的样子。不能用来形容人很多。‎ ‎2.[2017·河南摸底]填入下面文段空白处的词语,最恰当的一组是(3分)(  )‎ 让孩子选择未来,__①__不是放手不管不问,__②__在既有的经验和孩子的意愿之间,寻找最恰当的平衡点,并__③__给孩子更多的尊重。自主选择,__④__一种意愿,__⑤__一种能力,__⑥__这种能力呼应了时代的要求。‎ ‎①‎ ‎②‎ ‎③‎ ‎④‎ ‎⑤‎ ‎⑥‎ A ‎/‎ 要 尽力 不仅是 也是 因为 B 并 而是 尽可能 是 更是 ‎/‎ C 也 ‎/‎ 一定要 不但是 同样是 所以 D 虽然 而是 尽可能 是 ‎/‎ 当然 答案 B 解析 ②处“不是……而是……”须成对使用,③处“尽力给孩子更多的尊重”不妥,④⑤⑥之间不能构成因果关系,故可排除A、C两项。全段文字没有转折的意思,①处用“虽然”不当;“当然”表示肯定,若用于⑥处,与④⑤不能衔接,可排除D项。‎ ‎3.[2017·四川棠湖中学模拟]下列各句中,没有语病的一句是(3分)(  )‎ A.“唯有除贪,才能安民心”——越南国家主席张晋创接受越南通讯社采访时表示,越南反腐工作已经取得一定成果,但这些成绩仍不足以完全根除贪污腐败与浪费现象的发生。‎ B.2016猴年央视春晚着力演绎“幸福中国”的主题。与往届春晚相比,节目更加精准,主题更加集中,铺陈更加合理,现场观众的反响更加热情。‎ C.从文化哲学意义上来理解,乡愁是归属感的一种表现,同时是一种根源意识的体现,也是对这个归属和根源的一种亲和感,这种归属感是人的民族文化认同的基础。‎ D.“春运铁路格局的改变,反映的不仅仅是区域性乃至全国性经济结构、生产力布局的调整和产业结构的优化升级,更是交通基础设施的重大变化。”北京交通大学教授石培华说。‎ 答案 C 解析 A项,句式杂糅,将“的发生”删除,或将“根除”改为“杜绝”;B项,搭配不当,“反响”与“热情”主谓不搭配,将“热情”改为“热烈”;D项,不合逻辑或语序不当,将“不仅仅是”和“更是”后的内容对调。‎ ‎4.[2016·衡水中学模拟]依次填入下面一段文字横线处的语句,衔接最恰当的一组是(3分)(  )‎ 在学校的日子里,我没有什么特别的感觉,________,________,________,________,________,________。我默默地注视着学校红色的大门,由衷地感谢她带给我的一切。‎ ‎①很多时候你可能觉得今天跟昨天没什么不同 ‎②这时你可能非常留恋过去的日子 ‎③突然发现它写得真好 ‎④你回过头来,其实一切都在改变 ‎⑤不禁哼出一句“月亮的脸偷偷地在改变”‎ ‎⑥现在要离开这个工作了七年的学校 A.①②④⑤⑥③ B.①⑥②⑤③④‎ C.⑥②⑤①④③ D.⑥⑤③①④②‎ 答案 D 解析 首先确定起始句,横线之前说“在学校的日子里,我没有什么特别的感觉”,而⑥中“现在”“离开”能与之呼应,确定为起始句。剩余五句“按图索骥”,寻找句与句之间的衔接点、呼应点即可。这里注意“我”“你”人称的转变,⑥⑤③的称谓都是“我”,且因要“离开”所以“哼出”歌曲,并“发现它”好,这构成因果顺承关系,而下文围绕“变”抒发感慨。①中“没什么不同”又与④中“改变”形成转折关系。最后是②,“这时”指代④中“回过头来”。‎ ‎5.[2017·潍坊一中测试]阅读下面文段,在文中两处横线上依次填入语句,衔接最恰当的一项是(3分)(  )‎ 走出一片松林,眼前豁然一亮。箕形山坡下,累累卧石和簇簇青葱之间,好鸟相鸣,荷叶田田,萼红灼灼,一池晶莹,笑容可掬。池塘约三四亩,澄澈宁静,映射着青山绿树,蓝天白云,日月沉浮,好一个安详的世界!那是大地的脉络,那是历史的记忆。________,重檐六角,斗拱高耸,恰似魏晋高士的峨冠。池水周围是一丛丛毛竹,________,又如美女明眸的睫羽。‎ A.池中有一座湖心亭 滴青流翠,楚楚动人 B.湖心亭位于池中 楚楚动人,滴青流翠 C.池中有一座湖心亭 楚楚动人,滴青流翠 D.湖心亭位于池中 滴青流翠,楚楚动人 答案 A 解析 “池中”承接上文,“湖心亭”关照下文。“滴青流翠”承接上文,“楚楚动人”关照下文。‎ ‎6.[2017·北大附中河南分校考试]填入下面一段文字横线处的语句,衔接最恰当的一组是(3分)(  )‎ 雁门关在晋北崇山峻岭中,它虽说早已只剩下颓墙断垣,但雄风不减当年。________________,如血的残阳给它涂上了一抹无可言状的朦胧。‎ A.晋王李克用墓在关左的朔风白草中沉寂不语;龟裂的古道在关前向西拐去 B.关左,晋王李克用墓在朔风白草中沉默不语;关前,龟裂的古道向西拐去 C.晋王李克用墓在关左的朔风白草中沉寂不语;关前,龟裂的古道向西拐去 D.关左,晋王李克用墓在朔风白草中沉默不语;龟裂的古道在关前向西拐去 答案 B 解析 语句连贯的法则之一是话题要统一,本题陈述的中心是雁门关,只有B项围绕这一中心,故答案为B。‎ ‎7.[2016·唐徕回民中学模拟]填入下面横线处的一段文字,与上文衔接最恰当的一项是(3分)(  )‎ 大雁是有灵性的,它只能选择适宜于它生存的环境;同时,大雁也无法改变它的天性——飞翔,它们原是自由的鸟儿,为了生命的自由和自由的生命,远远地避开人群,________。‎ A.远远地飞走,甚至不愿回来,飞走 B.飞走,远远地飞走,甚至不愿回来 C.甚至不愿回来,飞走,远远地飞走 D.远远地飞走,飞走,甚至不愿回来 答案 B 解析 解答此题首先要把握语段的内容要点,要明确所选的语句是对上文的延伸;其次根据由近及远的层次步步深入;再次要做到所选句式能使语句表达连贯通畅;最后要注意句式之间的疏密关系。要善于比较各选项的异同,密切结合语境要求和表达需要来寻找最佳答案。B项与前面内容衔接紧密,且有时间的先后。‎ ‎8.[2017·山东枣庄市统考]把下面四个短句改成一个语意连贯的长单句,可以改变语序、增删词语,但不得改变原意。(4分)‎ ‎①1932年出版了一套《开明国语课本》。‎ ‎②该课本由叶圣陶编写内容,丰子恺书写并画插图。‎ ‎③这套课本当时就再版了40余次。‎ ‎④2010年应读者要求,再次出版。‎ 答:___________________________________________________‎ ‎___________________________________________________________‎ 答案 (示例)由叶圣陶编写内容,丰子恺书写并画插图,1932年出版当时就再版了40余次的《开明国语课本》应读者要求在2010年再次出版。‎ 解析 本题考查变换句式的能力。句子主干:《开明国语课本》再次出版。其他短句可按照编写人、出版时间及出版次数的顺序作为定语进行组织答案。‎ ‎9.[2017·定州中学模拟]把下面这个长句改写成四个较短的句子,使其明白易懂,但不得改变原意。(4分)‎ 位于宁夏回族自治区银川市西部贺兰山东麓的承接着鲜卑拓跋氏从北魏平城到党项西夏的拓跋氏历史的将汉族文化、佛教文化、党项族文化有机结合构成我国陵园建筑中别具一格形式的西夏王陵在中国119处国家重点风景名胜区中是唯一的以单一的帝王陵墓构成的景区。‎ 答:(1)_________________________________________________‎ ‎(2)_____________________________________________________‎ ‎(3)_____________________________________________________‎ ‎(4)_____________________________________________________‎ 答案 (1)西夏王陵位于宁夏回族自治区银川市西部贺兰山东麓。(2)它承接着鲜卑拓跋氏从北魏平城到党项西夏的拓跋氏历史。(3)它将汉族文化、佛教文化、党项族文化有机结合构成我国陵园建筑中别具一格的形式。(4)它在中国119处国家重点风景名胜区中是唯一的以单一的帝王陵墓构成的景区。‎ 解析 此题的句子主干是:西夏王陵在中国119处国家重点风景名胜区中是唯一的以单一的帝王陵墓构成的景区。修饰成分主要是定语,抓住“位于”“承接”“将……有机结合”可以概括出单独的句子。‎ ‎10.[2017·集宁一中月考]用排比的修辞方式,改写下面画线部分。要求:①句式一致;②字数相等;③与上文语意连贯;④不改变原意。(4分)‎ 长途跋涉后,我终于在林中寻找到这幽深澄碧的水潭。这潭水,可以将我的容颜映照在它明镜一般的水面上;我把这潭水当作激发我诗性的佳酿;这潭水还可以成为我的墨池,供我笔走龙蛇。‎ 答:___________________________________________________‎ ‎___________________________________________________________‎ 答案 (示例一)如明镜,让我映照容颜;似佳酿,助我激发诗性;若墨池,供我笔走龙蛇。‎ ‎(示例二)可以成为映照我的容颜的明镜;可以成为激发我诗性的佳酿;还可以成为供我笔走龙蛇的墨池。‎ 解析 先明确改写要求,然后分析画线部分的特点,可按“这潭水,可以成为……”的格式改写,也可按“如……,……我……”的句式来写。‎ ‎11.[2017·河北师大附中段考]仿照下面的示例,另写三句话,要求句式与示例相同。(4分)‎ 春蚕死去了,但留下了华贵丝绸;花朵凋谢了,但留下了缕缕幽香;雷雨过去了,但留下了七彩霓虹。‎ 答:___________________________________________________‎ ‎___________________________________________________________‎ 答案 (示例)蝴蝶死去了,但留下了漂亮的衣裳;画眉飞去了,但留下了美妙的歌声;蜡烛燃尽了,但留下一片光明。(答案不唯一)‎ 解析 ‎ 本题示例本身就是一组排比句,均为表转折关系的简单复句;从选材和仿句的角度说,本题难度不是很大,只要能表达出合理的转折意思即可。‎ ‎12.[2017·甘肃重点中学调研]阅读下面一则新闻,回答问题。(4分)‎ 腾讯体育2016年8月21日讯 在中国女排以3-1逆转击败塞尔维亚夺得里约奥运会女排金牌后,中国国内固然是一片欢天喜地,外国网友也纷纷盛赞中国女排表现出色。‎ 一个名叫JL Foundation的网友写道:“这场比赛不适合胆小鬼!中国女排平均身高6英尺3英寸(约1.9米),平均年龄只有21岁!简直是一群女巨人!”Rocio Patino则写道:“中国女排打出了一届出色的奥运会比赛!祝贺她们夺得金牌!”Leta Johnson则说:“中国女排输掉了本届奥运会的第一场比赛,然后在淘汰赛阶段一路取胜,并且夺得了最后的冠军!真是非常惊人!”‎ ‎(1)请你为该新闻拟一条新闻标题。要求:切合主旨,言简意赅,15字以内。‎ 答:___________________________________________________‎ ‎(2)请你写句话来表达对中国女排夺冠的自豪和喜悦。‎ 答:___________________________________________________‎ 答案 (1)女排夺冠引国外网友热议 ‎(2)女排精神铸就里约金牌;如果奇迹有颜色,那么一定是中国红。‎ ‎13.[2017·山西五校联考]请在下面横线上补上恰当的三个句子,与前面的语句构成排比,且保持语意连贯。(6分)‎ 被人误解的时候能微微地一笑,这是一种素养;受委屈的时候能坦然地一笑,这是一种大度;________________,________________;________________,________________;________________,________________。不管有什么事情,为了什么原因,我们每天都要开心一笑。‎ 答案 吃亏的时候能开心地一笑,这是一种豁达;无奈的时候能达观地一笑,这是一种境界;危难的时候能泰然地一笑,这是一种大气;失恋的时候能轻轻地一笑,这是一种洒脱;处窘境的时候能自嘲地一笑,这是一种智慧;被轻视的时候能平静地一笑,这是一种自信。‎ ‎14.[2017·九江一中测试]在下面一段文字横线处补写恰当的语句,使整段文字语意完整连贯,内容贴切,逻辑严密。每处不超过10个字。(5分)‎ 数学美是自然美的客观反映,是科学美的核心,__①__。例如,普洛克拉斯早就断言:“哪里有数学,哪里就有美。”亚里士多德也曾讲过:“美的主要形式是秩序、匀称和确定性,这些正是数学研究的原则。”数学家徐利治说:“作为科学语言的数学,具有一般语言文字与艺术所共有的美的特点,即数学在其内容结构上和方法上也都具有自身的某种美,即所谓数学美。”数学美有别于其他的美,没有美妙的声音,__②__,没有动人的节奏,没有深邃的语言,它却是一种独特的美。德国数学家克莱因曾对数学美作过这样的描述:“音乐能激发或抚慰情怀,绘画使人赏心悦目,诗歌能动人心弦,哲学使人获得智慧,但数学却能提供以上一切。”可见,__③__。‎ 答:①_________________________________________________‎ ‎②_____________________________________________________‎ ‎③_____________________________________________________‎ 答案 ①许多学者都这样认为 ②没有鲜艳的色彩(没有动感的画面) ③数学美具有丰富的内涵 ‎15.[2017·西安长安一中质检]在下面一段文字横线处补写恰当的语句,使整段文字语意完整连贯,内容贴切,逻辑严密。每处不超过15个字。(6分)‎ 文艺复兴是一个文化奇迹。在那个年代,__①__,还有我们伟大的伽利略等物理学家,还有弗兰西斯·培根等哲学家。伽利略开创了实验验证真理的先河,而培根子则是宣扬这一科学研究方法的第一人。他提出了唯物主义经验论的原则,认为知识和观念起源于感性世界,感觉经验是一切知识的源泉,主张以实验和观察材料为基础,经过分析、比较、选择、排除,最后得出正确的结论。__②__,是因为他就像中国春秋战国时代的诸子百家一样,创立一派学说并广收信徒。培根曾经说过:“知识就是力量。”于是,__③__,人们就不断用实验物理的方法去探究新的知识。‎ 答:①___________________________________________________‎ ‎②______________________________________________________‎ ‎③______________________________________________________‎ 答案 ①诞生的不只是莎士比亚等文艺大家 ②之所以称呼培根为“子” ③为了寻求力量并汲取力量 ‎16.[2017·天津河东区模拟]阅读下面的材料,按要求作答。(6分)‎ 材料一 出门忘带手机便“惴惴不安”,好像丢了魂一样;当一段时间手机铃声没响,就感到异常失落;刷微博、玩游戏是睡觉前的必修课,否则怎么也睡不着;公车上、饭局上、朋友聚会上,总是低头玩手机。中国城市通信行为研究报告也显示,62%的智能手机用户在睡觉前玩手机;人们最常在上下班途中玩手机,这个比例高达75%。在中国近1.6亿的手机拥有者中,患上“手机依赖症”的人数正在与日俱增。首都师范大学心理咨询中心的一项调查显示:77%的人每天开机12小时以上,33.55%的人24小时开机,65%的人表示“如果手机不在身边会有些焦虑”,超过九成人离不开手机。‎ 材料二 好莱坞动画大片《机器人总动员》中,描述了公元2700年的“低头族”:那时的人类文明高度发达,但由于过度依赖智能设备,人们都变成四体不勤的大胖子,每时每刻面对的只有一个支在他们眼前的电脑屏幕。除了和屏幕对话,他们不懂得如何与其他人交流,甚至离开屏幕就几乎无法生存……‎ 未来的人类是否真的会“退化”成这个样子,我们不得而知,但是智能手机带来的负面作用,现在就已经开始显现了。‎ 针对越来越多的“手机控”,请你根据手机依赖症的特点为“手机控”下一个定义,并且在公共场合、客流量较大的地方策划一个主题活动,提醒人们正确对待手机。请拟出活动主题和宣传语。‎ 答:(1)手机控:________________________________________‎ ‎(2)活动主题:__________________________________________‎ ‎(3)宣传语:____________________________________________‎ 答案 (1)指极度喜欢且在生活中过度依赖手机不能自拔的人 ‎(2)多看身边人,少当“低头族”/不做看手机的“盲人”/不要被手机所控 ‎(3)心与心的距离,要在“线下”构建/移动终端中的精彩,无法代替现实世界的美好/放下手机,拥抱世界 解析 答题时要注意下定义的格式,注意活动主题概括与宣传语的常用表达方式;如有可能,使用一定的修辞手法,使宣传语能深入人心。‎ ‎17.[2017·会宁一中月考]下面是微信公众账号(公众平台)的工作原理图示,请用文字表述这个图示,要求内容完整,表述准确,语言连贯,不超过80个字。(6分)‎ 答:___________________________________________________‎ ‎___________________________________________________________‎ 答案 微信用户向微信后台服务器发送信息,公众账号(公众平台)通过微信后台服务器接收信息,再将信息响应给微信后台服务器,最后由微信用户接收公众账号回复的信息。‎ ‎18.[2017·黄冈调研]下面是我市一所高中创设的一种课堂教学流程图,请把这个流程图写成一段话,要求内容完整,表述准确,语言连贯,不超过130个字。(6分)‎ 答:___________________________________________________‎ ‎___________________________________________________________‎ ‎___________________________________________________________‎ ‎___________________________________________________________‎ 答案 该教学流程分为问题、探究、迁移、拓展和小结五步,具体的过程是:第一步提出问题,包括自主学习、课堂调查、问题界定三个环节;第二步探究学习,按照合作探究、小组汇报、归纳建构的步骤进行;第三步知识迁移,包括巩固训练、课堂观察、适时补差三个环节;第四步知识与能力拓展,第五步课堂小结。‎ ‎19.[2017·贵州湄江中学模拟]认真分析下面数据,根据要求答题。(5分)‎ 类别 身高(平均)‎ 体重(平均)‎ 身体机能综合 素质(基数为100)‎ ‎70年代 ‎155.5厘米 ‎41.3公斤 ‎99.04‎ ‎80年代 ‎158.6厘米 ‎42.1公斤 ‎97.37‎ ‎90年代 ‎160.6厘米 ‎46.5公斤 ‎90.86‎ ‎(1)通过上表,可以得出一个结论:_________________________。‎ ‎(2)根据你的认知,简要说说出现这一现象的原因。(20字左右)‎ 答:___________________________________________________‎ ‎___________________________________________________________‎ 答案 (1)80后、90后中学生,平均身高、体重都较70后增加了,但身体机能综合素质却下降了 ‎(2)生活条件好了(营养好了),但身体锻炼少了。‎ ‎20.[2016·辽宁考前质监]请你根据下面图表提供的相关信息,写一段结论性的文字。要求内容完整,表述准确,语言连贯,不超过80个字。(6分)‎ 答:___________________________________________________‎ ‎___________________________________________________________‎ ‎___________________________________________________________‎ ‎___________________________________________________________‎ 答案 无论是整体还是各年龄段,该省0~17周岁未成年人的阅读率均高于全国平均水平;从各年龄段来看,无论该省还是全国,9~13周岁未成年人的阅读率均高于其他两个年龄段。(意思对即可)‎ 解析 解答此题,要注意观察图表,从中提取有效信息。如整体和三个年龄段,某省未成年人图书阅读率均高于全国平均水平;三个年龄段中9~13周岁未成年人的阅读率高于其他年龄段。‎ 第二部分 古代诗文阅读 考点八 文言文断句 考点名片 考点内容 文言文断句,传统上称为“句读”,是指根据试题要求,用分隔号(/)将文言文语段中需要停顿(包括句中停顿和句末停顿)的地方断开,类似于给文言文加标点。‎ 考查形式 ‎①从传记文段中划出几个句子来,中间删去标点符号,要求用斜线划出停顿。②单独给出一个古文段落,要求用斜线划出停顿。③选择题,3分;或主观题,4~5分。‎ 趋势分析 ‎2018年将会继续保留此考查点,也可能以主观题的形式出现,且要求停顿之处的数目。‎ 题组1 独立材料型断句 ‎1.用斜线“/”给下面文言文断句。‎ 天 不 为 人 之 恶 寒 也 辍 冬 地 不 为 人 之 恶 辽 远 也 辍 广 君 子 不 为 小 人 之 匈 匈 也 辍 行 天 有 常 道 矣 地 有 常 数 矣 君 子 有 常 体 矣 君 子 道 其 常 而 小 人 计 其 功 诗 曰 礼 义 之 不 愆 何 恤 人 之 言兮 此 之 谓 也。‎ 答案 天不为人之恶寒也辍冬/地不为人之恶辽远也辍广/君子不为小人之匈匈也辍行/天有常道矣/地有常数矣/君子有常体矣/君子道其常//而小人计其功/诗曰/礼义之不愆/何恤人之言兮/此之谓也(“//”处可断可不断)‎ 参考译文:‎ 天不因为人憎恶寒冷就取消冬天,地不因为人憎恶辽远就缩小面积,君子不因为小人吵吵嚷嚷就停止他正义的行为。天有一定的规律,地有一定的法则,君子有一定的规范。君子遵循事物的常规,小人则是盘算眼前的功利。《诗经》里说:“自己在礼仪法度上不出差错,何必要考虑别人的议论?”说的就是这个道理。‎ ‎2.用斜线“/”给下面的文言文断句。‎ 今 夫 水 非 火 则 无 以 济 非 木 则 无 以 屯 非 金 则 无 以 节 非 土 则 无 以 比 是 故 夫 智 不 丽 乎 仁 则 察 而 刻 不 丽 乎 礼 则 慧 而 轻 不 丽 乎 义 则 巧 而 术 不 丽 乎 信 则 变 而 谲 俱 无 所 丽 则 浮 荡 而 炫 其 孤 明 ‎(取材于《周易外传·卷一》)‎ 答案 今夫水 / 非火 // 则无以济 / 非木 // 则无以屯 / 非金 // 则无以节 / 非土 // 则无以比 / 是故夫智/ 不丽乎仁 // 则察而刻 / 不丽乎礼 // 则慧而轻 / 不丽乎义 // 则巧而术 / 不丽乎信 // 则变而谲 / 俱无所丽 // 则浮荡而炫其孤明(“/”处为必断,“//”处可断可不断)‎ 参考译文:‎ ‎(五行中)水不依靠火(的加热)就无法升华,不依靠木(的生长)就无法聚集,不依靠金属器具(的约束)就无法节制,不依靠土(的沾润)就无法和顺。因此那(和水相应的)智,不附着于仁就会表现得过于精明而苛刻,不附着于礼就会表现得狡黠而轻佻,不附着于义就会表现得虚伪而俗套,不附着于信就会表现得不正常而又诡诈,不附着于仁礼义信中的任何方面就会让人轻浮放纵而只会矜夸自己孤零零的聪明。‎ ‎3.用斜线“/”给下面短文断句。(限画9个)‎ ‎ 周 室 既 衰 诸 侯 恣 行 仲 尼 悼 礼 废 乐 崩 追 修 经 术 以 达 王 道 匡 乱 世 反 之 于 正 见 其 文 辞 为 天 下 制 仪 法 垂 六 艺 之 统 纪 于 后 世 作 孔 子 世 家 第 十 七 ‎ ‎(选自《史记·太史公自序》)‎ 答案 周室既衰/诸侯恣行/仲尼悼礼废乐崩/追修经术/以达王道/匡乱世反之于正/见其文辞/为天下制仪法/垂六艺之统纪于后世/作孔子世家第十七 参考译文:‎ 周王室已经衰微,天下诸侯任意横行霸道。孔仲尼为礼崩乐坏而哀伤,于是追述先贤整理前圣经典学说,以求重建王道之术,匡救混乱之世以返回正道。(我)阅读过他的著作,(他)要替天下订立礼法制度,留传“‎ 六艺”的统绪纲纪重范后世。(我因此)作了《孔子世家》第十七。‎ ‎4.用斜线“/”给下面短文画横线的部分断句。‎ 齐宣王见孟子于雪宫。王曰:“贤者亦有此乐乎?”对曰:“有。人_不_得_则_非_其_上_也_不_得_而_非_其_上_者_非_也_为_民_上_而_不_与_民_同_乐_者_亦_非_也_乐_民_之_乐_者_民_亦_乐_其_乐_忧_民_之_忧_者_民_亦_忧_其_忧_乐_以_天_下_忧_以_天_下_然_而_不_王_者_未_之_有_也。”‎ 答案 对曰:“有。人不得/则非其上也/不得而非其上者/非也/为民上而不与民同乐者/亦非也/乐民之乐者/民亦乐其乐/忧民之忧者/民亦忧其忧/乐以天下/忧以天下/然而不王者/未之有也。”‎ 参考译文:‎ 齐宣王在雪宫里接见孟子。齐宣王说:“有道德的人也有这样的快乐吗?”孟子回答说:“有。如果人们得不到这样的快乐,就会抱怨他的君主。因得不到而抱怨他们的君主,是不对的;作为百姓的君主不与百姓一起分享这种快乐,也是不对的。(君主)以百姓的快乐为自己的快乐,百姓就会以君主的快乐为自己的快乐;(君主)以百姓的忧愁为自己的忧愁,百姓就会以君主的忧愁为自己的忧愁。以天下人的快乐为快乐,以天下人的忧愁为忧愁,做到这样,还不能统治天下的,是不可能有的。”‎ ‎5.用斜线“/”给下面画线的文言文断句。‎ 愚所谓圣人之道者如之何?曰“博学于文”,曰“行己有耻”。耻之于人大矣。不耻恶衣恶食,而耻匹夫匹妇之不被其泽,故曰“万物皆备于我矣,反身而诚”,呜呼。士_而_不_先_言_耻_则_为_无_本_之_人_非_好_古_而_多_闻_则_为_空_虚_之_学_以_无_本_之_人_而_讲_空_虚_之_学_吾_见_其_日_从_事_于_圣_人_而_去_之_弥_远_也。‎ ‎(选自顾炎武《与友人论学书》,有删改)‎ 答案 士而不先言耻/则为无本之人/非好古而多闻/则为空虚之学/以无本之人/而讲空虚之学/吾见其日从事于圣人/而去之弥远也 参考译文:‎ 我所说的圣人之道是什么呢?就是“从丰富的文献典籍中学习”,就是“自己认为可耻的事情就不去做”。羞耻对于人来说,关系重大啊!(我)不认为穿破旧的衣服、吃粗劣的食物是耻,而会以人们没有受到仁德的恩泽为耻,所以说“万事万物的道理都在我心中具备了,反躬自问能保持真诚”,呜呼!士人不先知羞耻(而约束自己),那么就是失去了做人的根本。并非爱好古典文献就会多见识,那是空洞和虚假的学问。没有做人根本的人,他们讲些空洞和虚假的学问,我看他们虽每天讲着圣人的道理,但做的事情离圣人很远。‎ ‎6.用斜线“/”给下面文言文中的画线部分断句。‎ 曾子居卫,缊袍无表,颜色肿哙,手足胼胝。三日不举火,十年不制衣。正 冠 而 缨 绝 捉 衿 而 肘 见 纳 屦 而 踵 决 曳 而 歌 《商 颂》 声 满 天 地 若 出 金 石 天 子 不 得 臣 诸 侯 不 得 友。故养志者忘形,养形者忘利,致道者忘心矣。‎ ‎(选自庄周《庄子·让王》,有删改)‎ 答案 正冠而缨绝/捉衿而肘见/纳屦而踵决/曳而歌《商颂》/声满天地/若出金石/天子不得臣/诸侯不得友 参考译文:‎ 曾子住在卫的时候,衣服破烂没有罩衫,面有病色,手和脚都磨出了很厚的老茧。曾经三天没有生火做饭,连续十年不曾添置新衣。整理帽子便拉断了帽带,拉过衣襟就露出了手臂,穿起鞋子就露出了脚跟。但他拖着烂鞋,口中却高歌《商颂》,声音十分响亮,好像金石乐器奏出来的一样清脆。天子不能使他做臣子,诸侯不能和他交朋友。所以说养志的人忘了形骸,养形的人忘了利禄,求道的人忘了心机。‎ ‎7.阅读下面的文言文,完成后面的问题。‎ 景公饮酒酣,曰:“今日愿与诸大人为乐饮,请无为礼。”晏子蹴然改容曰:“君子言过矣!群臣固欲君之无礼也力多足以胜其长勇多足以弑君而礼不使也。禽兽以力为政,强者犯弱,故日易主,今君去礼,则是禽兽也。群臣以力为政,强者犯弱,则日易主,君将安立矣!凡人之所以贵于禽兽者,以有礼也,故诗曰:‘人而无礼,胡不遄死!’礼不可无也。”公湎而不听,少间,公出,晏子不起,公入,不起;交举则先饮。公怒,色变,抑手疾视曰:“向者夫子之教寡人无礼之不可也,寡人出入不起,交举则先饮,礼也?”晏子避席再拜稽首而请曰:“婴敢与君言而忘之乎?臣以致无礼之实也。君若欲无礼,此是已!”公曰:“‎ 若是,孤之罪也。夫子就席,寡人闻命矣。”觞三行,遂罢酒。盖是后也,饬法修礼以治国政,而百姓肃也。‎ 下列用“/”给文中画波浪线部分的断句,正确的一项是(  )‎ A.群臣固欲君之/无礼也力/多足以胜其长勇/多足以弑君而礼不使也 B.群臣固欲君之无礼也/力多足以胜其长/勇多足以弑君/而礼不使也 C.群臣固欲君之无礼也/力多足以胜其长/勇多足以弑君而礼/不使也 D.群臣固欲君之无/礼也力多足以胜/其长勇多足以弑君/而礼不使也 答案 B 解析 “力多足以胜其长”与“勇多足以弑君”为对称结构,据此可得出答案。‎ 参考译文:‎ 齐景公酒喝得很畅快,(对陪酒的众臣)说:“今天我想要和各位大夫快活地饮酒,请(大家)不要拘泥于礼。”晏子(听了)惊讶不安,脸色都变了,说:“君王的话错了!群臣本就希望君王不讲礼。力大的人能够凭力气欺凌长辈,勇猛的人能够凭勇猛刺杀君王,而礼就不便施行了。禽兽凭借力施行统治,强者欺凌弱者,因而天天都在改换首领,现在君王抛开礼,那么这就和禽兽一样了。群臣凭借勇力治理政事,强者欺凌弱者,因而会天天更换君主,君王将如何稳固君位!大凡人比禽兽高贵的原因,是(人)讲究礼,所以《诗经》说:‘人如果不讲礼,何不快点去死!’礼不可以没有啊。”齐景公沉湎于饮酒而不听,过了一会儿,景公出去,晏子不站起来,景公进来,(晏子还是)不站起来;相互举杯晏子比景公先饮。景公很生气,脸色都变了,手按桌子怒目而视说:“刚才先生告诉我不可以没有礼,我出去进来你都不起立,相互举杯你却先饮,这合乎礼吗?”晏子起身离开坐席两次叩首跪拜,恭敬地说:“我岂敢把和君王说的话忘记呢?我只是用这种做法来表达没有礼的结果。您如果想不要礼,这就是了。”景公说:“如果这样,这是我的过失。先生请入座,我听从您的劝告了。”(君臣)举杯三次后,就(按照规矩)结束了酒宴。这以后,(景公)整治法度、修明礼仪来治理国家政事,百姓也都恭敬有礼。‎ ‎8.阅读下面的文言文,完成后面的问题。‎ 退让 贾谊 翟王使使至楚。楚王欲夸之,故飨客于章华之台上。上者三休,而乃至其上。楚王曰:“翟国亦有此台乎?”使者曰否翟窭①国也恶见此台也翟王之自为室也堂高三尺壤陛三累茅茨弗翦②采椽弗刮。且翟王犹以作之者大苦,居之者大佚。翟国恶见此台也!”楚王愧。‎ ‎[注] ①窭(jù):贫而简陋。②翦:同“剪”,修剪。‎ 下列对文中画波浪线部分的断句,正确的一项是(  )‎ A.使者曰/否翟窭国也/恶见此台也/翟王之/自为室也/堂高三尺壤/陛三累茅茨/弗翦采/椽弗刮 B.使者曰/否/翟/窭国也/恶见此台也/翟王之自为室也/堂高三尺/壤陛三累/茅茨弗翦/采椽弗刮 C.使者曰/否翟/窭国也/恶见此台也/翟王之自为室也/堂高三尺壤/陛三累茅茨/弗翦采/椽弗刮 D.使者曰/否翟窭国也/恶见/此台也/翟王之自为室也/堂高三尺/壤陛三累/茅茨弗翦/采椽弗刮 答案 B 解析 “否”,否定词,不能直接修饰名词,应单独停顿,即可排除A、C、D三项,再结合句意及“茅茨弗翦”“采椽弗刮”等并列结构,可知答案为B。‎ 参考译文:‎ 翟王派遣使臣到楚国。楚王打算向使者夸耀楚国的豪富,所以在章华台上宴请宾客。登台的人一路休息多次,才到了顶上。楚王说:“翟国也有这样的高台吗?”使者说:“没有。翟国是个贫穷的国家,怎么能看到这样的高台呢?翟王自己盖的宫室,堂高三尺,土台阶三层,茅草屋顶不剪齐,柞木椽子不削皮。翟王尚且认为盖房者太劳苦,居住者太安乐。翟国怎么能看到这样的高台呢?”楚王感到惭愧。‎ ‎9.阅读下面的文言文,完成后面的问题。‎ 贻诸弟砥石命(并铭)‎ ‎[唐]舒元舆 吾因叹以为金刚首五材及为工人铸为器复得首出利物以刚质铓利苟暂不砥砺尚与铁无以异况质柔铓钝而又不能砥砺当化为粪土耳又安得与死铁伦齿耶!‎ 以此益知人之生于代,苟不病盲聋喑哑,则五常之性全;性全则豺狼燕雀亦云异矣。而或公然忘弃砺名砥行之道,反用狂言放情为事,蒙蒙外埃,积成垢恶,日不觉寤,以至于戕正性,贼天理,生前为造化剩物,殁复与灰土俱委,此岂不为辜负日月之光景耶!‎ 下列对文中画波浪线部分的断句,正确的一项是(  )‎ A.吾因叹以为金刚/首五材/及为工人铸为器/复得首出/利物以刚质铓利/苟暂不砥砺/尚与铁无以异/况质柔铓钝/而又不能砥砺/当化为粪土耳/又安得与死铁伦齿耶!‎ B.吾因叹以为金刚/首五材及为工人/铸为器复得首/出利物以刚质铓利/苟暂不砥砺/尚与铁无以异况质柔铓钝/而又不能砥砺/当化为粪土耳/又安得与死铁伦齿耶!‎ C.吾因叹/以为金刚首五材/及为工人铸为器/复得首出利物/以刚质铓利/苟暂不砥砺/尚与铁无以异/况质柔铓钝/而又不能砥砺/当化为粪土耳/又安得与死铁伦齿耶!‎ D.吾因叹/以为金刚首五材/及为工人铸为器/复得首出利物/以刚质铓利苟/暂不砥砺尚与铁无以异/况质柔铓钝而又不能砥砺/当化为粪土耳/又安得与死铁伦齿耶!‎ 答案 C 解析 首句“叹”字应做停顿,而不能同“以为金刚”衔接;再者“工人铸为器”不可分割,故答案为C。‎ 参考译文:‎ 我因而感叹,认为金属的坚硬,在金木水火土五材中居首位,等到工人铸造为剑器,又成为器物中最锋利的。这把剑依仗着质地的坚硬和锋口的锐利,如果一时不磨砺,尚且跟一般的铁没有区别,何况质地柔软锋刃钝涩的其他东西,并且又得不到磨砺,就会化为粪土,又怎能和烂铁相比较呢?由此更加明白人生于世间,只要没有眼瞎、耳聋、声音哑失等疾病,那么仁义礼智信等五常的本性就都具备;本性完备,那么和豺狼燕雀等走兽飞禽也就不同了。如果有人公然忘掉和舍弃像在磨刀石上磨刀那样不断提高名节品德的正道,反而用狂妄的语言和放纵情欲的方式做事情,模糊不清的外来的脏物,累积成污垢和丑恶,每天都还不觉悟,以至于残害天性和天理,那么活着的时候是天地间的一个废物,死了又与尘土一起被丢弃,这难道不是白白辜负了日月时光吗?‎ ‎10.阅读下面的文言文,完成后面的问题。‎ 猛狗与社鼠 韩非子 尧欲传天下于舜。鲧谏曰:“不祥哉!孰以天下而传之于匹夫乎?”尧不听,举兵而诛杀鲧于羽山之郊。共工又谏曰:“孰以天下而传之于匹夫乎?”尧不听,又举兵而诛共工于幽州之都。于是天下莫敢言无传天下于舜。仲尼闻之曰尧之知舜之贤非其难者也夫至乎诛谏者必传之舜乃其难也。”一曰:“不以其所疑败其所察则难也。”‎ ‎(节选自《韩非子集释》)‎ 下列用“/”给文中画波浪线部分的断句,正确的一项是(  )‎ A.仲尼闻之曰/尧之知舜之贤/非其难者也/夫至乎诛谏者/必传之舜/乃其难也 B.仲尼闻之/曰尧之知舜之贤/非其难者也夫/至乎诛谏者/必传之舜乃其难也 C.仲尼闻之曰/尧之知/舜之贤非其难者也/夫至乎诛谏者必传之/舜乃其难也 D.仲尼闻之曰/尧之知舜之贤非其难者/也夫至乎/诛谏者必传之舜/乃其难也 答案 A 解析 “曰”字后须停顿,排除B项,“者也”句尾语气词,须停顿,排除B、D。“尧之知舜之贤”为动宾结构,“夫”发语词置于句首,故答案为A。‎ 参考译文:‎ 唐尧想要将天下传给虞舜。大臣鲧进谏说:“不吉祥啊!谁拿天下去传给一个匹夫呢?”唐尧不听取(这种蒙蔽君主的意见),发兵将鲧诛杀在羽山的郊野。共工再进谏说:“谁拿天下去传给一个匹夫呢?”唐尧不听取(这种蒙蔽君主的意见),又发兵将共工诛杀在幽州的城邑。因此天下没有谁再敢说不要传天下给虞舜。孔子听说这件事说:“唐尧了解虞舜贤明,不见得是很难的事。到了诛杀进谏(错误意见)的人,也必定要传天下给虞舜的地步,才是真正难的事啊。”又说:“尧不因进谏的人提出疑问而败坏自己明察的事情才是困难的啊。”‎ 题组2 综合材料型断句 ‎11.阅读下面的文言文,完成后面的问题。‎ 曼卿,讳延年,姓石氏,其上世为幽州人。幽州入于契丹,其祖自成始以其族间走南归。天子嘉其来,将禄之,不可,乃家于宋州之宋城。父讳补之,官至太常博士。‎ 幽燕俗劲武,而曼卿少亦以气自豪,读书不治章句,独慕古人奇节伟行非常之功,视世俗屑屑,无足动其意者。自顾不合于时,乃一混以酒,然好剧饮,大醉,颓然自放,由是益与时不合。而人之从其游者,皆知爱曼卿落落可奇,而不知其才之有以用也。年四十八,康定二年二月四日,以太子中允、秘阁校理卒于京师。‎ 曼卿少举进士,不中。真宗推恩,三举进士,皆补奉职。曼卿初不肯就,张文节公素奇之,谓曰:“母老,乃择禄耶?”曼卿矍然起就之,迁殿直,久之,改太常寺太祝、知济州金乡县,叹曰:“此亦可以为政也。”县有治声。通判乾宁军,丁母永安县君①李氏忧,服除,通判永静军,皆有能名。充馆阁校勘,累迁大理寺丞,通判海州,还为校理。‎ 庄献明肃太后临朝,曼卿上书,请还政天子。其后太后崩,范讽以言见幸引尝言太后事者遽得显官欲引曼卿曼卿固止之乃已。‎ 自契丹通中国,德明②尽有河南而臣属,遂务休兵养息天下,然内外弛武三十余年。曼卿上书言十事,不报。已而元昊反,西方用兵,始思其言,召见。稍用其说,籍河北、河东、陕西之民,得乡兵数十万。曼卿奉使籍兵河东,还,称旨,赐绯衣银鱼。天子方思尽其才,而且病矣。既而闻边将有欲以乡兵扞贼者,笑曰:“此得吾粗也。夫不教之兵,勇怯相杂,若怯者见敌而动,则勇者亦牵而溃矣。今或不暇教,不若募其敢行者,则人人皆胜兵也。”其视世事,蔑若不足为,及听其施设之方,虽精思深虑,不能过也。状貌伟然,喜酒自豪,若不可绳以法度,退而质其平生,趣舍大节无一悖于理者。遇人无贤愚,皆尽忻欢。及间而可否天下是非善恶,当其意者无几人。其为文章,劲健称其意气。‎ ‎(节选自欧阳修《石曼卿墓表》,有删改)‎ ‎[注] ①县君:中国古代宗女、命妇的位号。②德明:北宋时期,西夏李元昊的父亲,是宋朝的定难军节度使,被宋朝封为西平王。‎ 下列对文中画波浪线部分的断句,正确的一项是(  )‎ A.范讽以言见幸/引尝言/太后事者遽得显官/欲引曼卿/曼卿固止之/乃已 B.范讽以言见幸/引尝言太后事者/遽得显官/欲引曼卿/曼卿固止之/乃已 C.范讽以言见幸/引尝言太后/事者遽得显官/欲引曼卿/曼卿固止之/乃已 D.范讽以言/见幸引尝言/太后事者遽得/显官欲引曼卿/曼卿固止之/乃已 答案 B 解析 解答本题,应注意三点:一是注意分析名词,如本语段中第一个“曼卿”是“引”的宾语,第二个是“固止之”的主语,两者中间应断开;二是注意分析副词,如“遽”和“乃”,其前需断开;三是注意句意。三管齐下,答案可出。‎ 参考译文:‎ 曼卿,名延年,姓石,他的祖先是幽州人。幽州被北方的契丹占领后,他的祖先自成才率领他的一族人偷偷地向南逃逸,投归北宋。北宋皇帝为嘉奖他们来投奔,准备让他们享受国家的俸禄,没有实行,于是在宋州的宋城定居下来。曼卿的父亲,名叫补之,官至太常博士。‎ 幽燕一带一般人都强健勇武,曼卿年少时也豪气逼人,读书不研究章节句读,只仰慕古代那些有奇伟的节操品行并建立过非同寻常功劳的人,看轻世俗的小功名,没有什么能够动摇他的志向。自认为跟时代不相合,于是整日借酒混世,且喜欢痛饮,大醉,颓废地放纵自己,因此更加与时代不相合。那些跟他交游的人,都只懂得爱慕曼卿卓异超群的气度,却不懂得他的才华可以为世所用。时年四十八岁,康定二年二月四日,担任太子中允、秘阁校理时在京城去世。‎ 曼卿年少时便参加进士考试,但没考中。真宗皇帝广施恩惠,凡三次参加进士考试不中的人,都给予职务。曼卿开始不肯就职,张文节一向认为他是奇才,对他说:“你母亲去世了,你才选择官禄吗?”曼卿惊惧,同意就职了,后调任殿直,许久,改任太常寺太祝、济州金乡县知县,感叹说:“‎ 这也可以施展治理国家的才华啊!”治县得到好评。任乾宁军通判,(他)遭逢母亲永安县君李氏丧事,服丧完后,任永静军通判,都得到好评。充任馆阁校勘,多次升迁任大理寺丞,(先任)海州通判,后任校理。‎ 庄献明肃太后上朝处理国政,曼卿上书,请求太后还政给天子。后来太后去世,范讽因进言被皇上宠爱,推荐那些曾经为太后还政天子一事进过言的人,那些人很快得了高官,(范讽)要推荐曼卿,曼卿坚决推辞不就,(范讽)才作罢。‎ 自从契丹与中原结好,李德明全部占有黄河以南并臣服(北宋),(北宋)就在天下致力于(推行)休兵养息政策,竟从中央到地方放松军事三十多年。曼卿上书谈了十件事,皇上没有答复。不久元昊反叛,(北宋)向西边用兵,(皇上)才想起曼卿的言论,召见了(曼卿)。逐渐采用了他的主张,在河北、河东、陕西百姓中征兵,得到乡兵几十万。曼卿也奉命在河东征兵,回来后,符合(皇帝)旨意,(皇帝)赐给他绯衣银鱼。皇上正想充分发挥他的才智,重用他,他却患病。不久听说有一位边将想率领乡兵与叛贼决斗,(曼卿)笑着说:“这只领悟了我的表面意思。没有经过调教的士兵,勇敢的和胆小的混杂在一起,假如胆小的见到敌人就动摇溃逃,那么勇敢的人也会跟着溃逃。现在或许没有空闲来调教,不如招募那些敢于行动的人,这样招来的人个个都是强兵。”他看待世事,轻得像不值得做似的,到听了他设计的方案,即使是深思熟虑,也不能超过他。他仪表英俊,喜欢饮酒,性格豪爽,表面上看去好像不可用法度来约束他,退一步想想他的一生,做事追求大节,从不违背事理。遇到的人不管是贤者还是不贤者,他都十分喜欢结交。等到闲暇议论天下是非善恶,却没有几个人能符合他的心意。他写文章遒劲刚健,正像他的为人。‎ ‎12.阅读下面的文言文,完成后面的问题。‎ 金濂,字宗瀚,山阳人。永乐十六年进士,授御史。尝言郡县吏贪浊,宜敕按察司、巡按御史察廉能者,如洪武间故事,遣使劳赉,则清浊分,循良劝。帝嘉纳之。用荐迁陕西副使。‎ 正统元年上书请补卫所缺官,益宁夏守兵,设汉中镇守都指挥使,多议行。三年擢佥都御史,参赞宁夏军务。濂有心计,善筹画,西陲晏然。时诏富民输米助边千石以上褒以玺书濂言边地粟贵请并旌不及额者储由此充。‎ 福建贼邓茂七等为乱,都督刘聚、都御史张楷征之,不克。十三年十一月大发兵,命宁阳侯陈懋等为将军往讨,以濂参军务。比至,御史丁瑄已大破贼。茂七死,余贼拥其兄子伯孙据九龙山,拒官军。濂与众谋,羸师诱之出,伏精兵,入其垒,遂擒伯孙。帝乃移楷讨浙寇,而留濂击平余贼未下者。会英宗北狩,兵事棘,召还。言者交劾濂无功,景帝不问,加濂太子宾客,给二俸。寻改户部尚书,进太子太保。‎ 时四方用兵,需饷急,濂综核无遗,议上撙节便宜十六事,国用得无乏。未几,上皇还。也先请遣使往来如初,帝坚意绝之。濂再疏谏,不听。初,帝即位,诏免景泰二年天下租十之三。濂檄有司,但减米麦,其折收银布丝帛者征如故。三年二月,学士江渊以为言,命部查理。濂内惭,抵无有。给事中李侃等请诘天下有司违诏故。濂恐事败,乃言:“银布丝帛,诏书未载,若概减免,国用何资?”于是给事中御史劾濂失信于民,为国敛怨,且讦其阴事。帝欲宥之,而侃与御史王允力争,遂下都察院狱。越三日释之,削宫保,改工部。吏部尚书何文渊言理财非濂不可,乃复还户部。濂上疏自理,遂乞骸骨,帝慰留之。东宫建,复宫保。寻复条上节军匠及僧道冗食共十事。五年卒官,以军功追封沭阳伯,谥荣襄。‎ 濂刚果有才,所至以严办称,然接下多暴怒。在刑部持法稍深。及为户部,值兵兴财诎,颇厚敛以足用云。‎ ‎(选自《明史·列传四十八》,有删节)‎ ‎[注] 明正统十四年,蒙古瓦剌部落在太师也先的率领下进攻明朝。英宗御驾亲征,战败被俘。英宗之弟朱祁钰即位(史称明景帝),改元景泰,并尊英宗为太上皇。景泰元年,英宗被明廷迎回。‎ 下列对文中画波浪线部分的断句,正确的一项是(  )‎ A.时诏富民输米助边/千石以上褒/以玺书濂言边地粟贵/请并旌不及额者/储由此充 B.时诏富民输米助边千石以上/褒以玺书/濂言边地粟贵/请并旌不及额者/储由此充 C.时诏富民输米助边/千石以上褒以玺书/濂言边地粟贵/请并旌不及额者/储由此充 D.时诏富民输米助边千石以上/褒以玺书/濂言边地粟贵请并旌/不及额者储由此充 答案 C 解析 “褒以玺书”不可拆分;“输米助边”“边地粟贵”固定结构,故答案为C。‎ 参考译文:‎ 金濂,字宗瀚,山阳人。永乐十六年(1418),中进士,被授予御史。他曾说郡县官员贪污,应敕令按察司、巡按御史考察廉洁能干的人,就如洪武年间的做法那样,再派使者去慰劳和奖赏,这样则官员清浊自分,守法而有治绩的官员也受到鼓励。皇上嘉许采纳了他的建议。因受推荐他升为陕西副使。‎ 正统元年(1436),他上书请将各卫的缺官补上,增加宁夏守兵,设汉中镇守都指挥使,这些建议多得讨论实行。三年,他升为佥都御史,参赞宁夏军务。金濂有心计,善于筹划,西陲安然。当时(皇上)下诏令富民输运粮食助边,运送一千石以上的可得玺书褒奖。金濂说边防地区粮食很贵,请对达不到这个数目的人也进行表彰,边粮储备因此得到充实。‎ 福建贼邓茂七等人作乱,都督刘聚、都御史张楷去征讨,没有胜利。十三年(1448)十一月,(朝廷)大发兵,命宁阳侯陈懋等人为将军前往征讨,用金濂参赞军务。他们到达时,御史丁瑄已大破贼人。邓茂七死,其余的贼人拥戴他哥哥的儿子邓伯孙占据九龙山,抵抗官军。金濂与众人商议,便用弱兵引诱他们出来,而埋伏的精兵便攻入他们的堡垒,于是擒住了邓伯孙。皇上于是调张楷去讨伐浙江之寇贼,而留下金濂扫平还没攻下的其余贼人。正值英宗被俘,军事紧急,(金濂)被召回。议论者纷纷弹劾金濂无功,景帝不问,加封他为太子宾客,支给二职俸禄。不久他改任户部尚书,进封为太子太保。‎ 当时四方用兵,粮饷需要紧迫,金濂全面安排计划,提出十六项节减开支的建议上奏实行,国家用度得以不缺乏。不久,上皇回来了。也先请如先前派遣使者往来,皇上坚决拒绝。金濂一再上疏劝谏,(皇上)不听。当初,皇上即位时,下诏免除景泰二年(1451)全国租税的十分之三。金濂传檄有关官员,只减征米麦,其他折收银两和布匹丝帛的照旧征收。三年二月,学士江渊对皇上谈到此事,(皇上)命户部调查处理。金濂心中惭愧,抵赖说没有。给事中李侃等人请询问天下有关官员违反诏令的原因。金濂担心事情败露,便说:“银两布匹和丝帛,诏书上没写明,如果一概减免,国家用度怎够开支?”于是给事中和御史都弹劾金濂失信于民,使人民怨恨国家,并且还揭发了他的私事。皇上想宽宥他,而李侃和御史王允极力反对,遂把他关进都察院监狱。过了三天把他放了出来,削去太子太保之衔,改到工部。吏部尚书何文渊说理财非金濂不可,于是又调他回户部。金濂上疏自述,顺便请求退休,皇上安慰挽留他。东宫太子建立后,恢复金濂太子太保之衔。不久他又提出节减军匠和僧道人员过滥的食粮等十项建议上奏。五年后,他在任上去世,以军功追封他为沭阳伯,谥荣襄。‎ 金濂刚毅果断,有才干,所到之处以严办案著称,然而对待属下也因严办案件多暴怒。在刑部时,他执法过严。到他任户部尚书时,正值用兵,国家财物短缺,多采取厚敛的办法来保证供给。‎ ‎13.阅读下面的文言文,完成下题。‎ 范仲淹,字希文,少有志操。之应天府,依戚同文学。昼夜不息,冬月惫甚,以水沃面;食不给,至以糜粥继之。人不能堪,仲淹不苦也。晏殊知应天府,闻仲淹名,召置府学。尝推其奉以食四方游士,诸子至易衣而出,仲淹晏如也。‎ 天圣七年,章献太后将以冬至受朝,天子率百官上寿。仲淹极言之,且曰:“奉亲于内,自有家人礼,顾与百官同列,南面而朝之,不可为后世法。”且上疏请太后还政,不报。‎ 时吕夷简执政,进用者多出其门。仲淹上《百官图》,指其次第曰:“如此为序迁,如此为不次,如此则公,如此则私。况进退近臣,凡超格者,不宜全委之宰相。”夷简不悦。‎ 葛怀敏败于定川贼大掠至潘原关中震恐民多窜山谷间仲淹率众六千由邠泾援之闻贼已出塞乃还。始,定川事闻,帝按图谓左右曰:“若仲淹出援,吾无忧矣。”奏至,帝大喜曰:“吾固知仲淹可用也。”进枢密直学士。仲淹以军出无功,辞不敢受命,诏不听。‎ 帝方锐意太平,数问当世事,仲淹皇恐,退而上十事。天子方信向仲淹,悉采用之,宜著令者,皆以诏书画一颁下。独府兵法,众以为不可而止。‎ 仲淹内刚外和,性至孝,以母在时方贫,其后虽贵,非宾客不重肉。妻子衣食,仅能自充。而好施予,置义庄里中,以赡族人。泛爱乐善,士多出其门下,虽里巷之人,皆能道其名字。死之日,四方闻者,皆为叹息。为政尚忠厚,所至有恩,邠、庆二州之民与属羌,皆画像立生祠事之。及其卒也,羌酋以数百人,哭之如父,斋三日而去。‎ 初,仲淹病,帝常遣使赐药存问,既卒,嗟悼久之。又遣使就问其家,既葬,帝亲书其碑曰“褒贤之碑”。‎ ‎(节选自《宋史·范仲淹传》)‎ 下列对文中画波浪线部分的断句,正确的一项是(  )‎ A.葛怀敏败于定川贼/大掠至潘原/关中震恐/民多窜山谷间/仲淹率众六千/由邠泾援之/闻贼已/出塞乃还。‎ B.葛怀敏败于定川贼/大掠至潘原/关中震恐/民多窜山谷间/仲淹率众六千/由邠泾援之/闻贼已出塞/乃还。‎ C.葛怀敏败于定川/贼大掠至潘原/关中震恐/民多窜山谷间/仲淹率众六千/由邠泾援之/闻贼已/出塞乃还。‎ D.葛怀敏败于定川/贼大掠至潘原/关中震恐/民多窜山谷间/仲淹率众六千/由邠泾援之/闻贼已出塞/乃还。‎ 答案 D 解析 由句意可知,“大掠”的主语是“贼”而不是“葛怀敏”,排除A、B。“贼已出塞”不可拆分,排除A、C,故答案为D。‎ 参考译文:‎ 范仲淹,字希文,他少时就有志气,奉行操守。前往应天府,依从戚同文学习。他昼夜不停地刻苦学习,冬天读书十分疲乏时,就用冷水浇脸;有时无东西吃,甚至不得不靠喝稀粥度日。一般人不能忍受的困苦生活,范仲淹却从不叫苦。晏殊掌管应天府时,听说范仲淹很有名,就召请他到府学任职。(范仲淹)曾经拿出自己的俸禄来供养四方游学之士,几个儿子甚至要轮换穿一件好衣服才能出门,范仲淹却始终泰然处之。‎ 天圣七年(1029),章献太后将在冬至日接受朝拜,仁宗皇帝也准备率领文武百官为太后祝寿。范仲淹上疏详细地论述了这件事,并且说:“在内宫侍奉亲长,自当有家人礼仪,只是在朝堂上皇帝与百官站在一起,(太后)面朝南接受(皇帝和百官)的朝拜,不能够成为后世的规范。”而且上疏请求章献太后将朝政大权交还仁宗皇帝,但没有得到任何答复。‎ 当时吕夷简执掌朝政,被任用和得到提拔的人大都出自他的门下。范仲淹向仁宗呈上一份《百官图》,指着图上百官升迁的次序说:“像这样的是循序升迁,像这样的是不合顺序的升迁,循序升迁是(符合)公理的,越序升迁是(遵循)个人意愿。况且提拔和黜降(天子的)近臣,凡是超过标准的提拔,不应该全部委托宰相处理。”吕夷简对此很不高兴。‎ 葛怀敏在定川被敌军打败,敌军大肆抢掠至潘原,关中地区震动惊恐,老百姓大多躲藏到山谷中。范仲淹率领六千军队,从邠州、泾州出发进行援救,听说敌军已经撤出边塞,才率领军队返回。开始的时候,定川战败的消息传到朝廷,仁宗皇帝手按着地图对左右大臣说:“如果范仲淹出兵救援,我就没有什么可以忧虑的了。”范仲淹出援兵的奏报一到,仁宗皇帝非常高兴地说:“我一直就认为范仲淹是可以重用的。”于是就任命范仲淹为枢密直学士。范仲淹因为这次军队出征没有立功,辞谢了皇帝的任命,仁宗皇帝没有接受他的辞呈。‎ 仁宗皇帝当时正一意要实现天下太平,多次询问当前急需办理的大事,范仲淹惶恐不安,退朝后给皇帝上奏十件事。仁宗皇帝正专意信任范仲淹,因而全部采纳了他的意见,凡适宜立为法令的,都用诏书统一颁布下发。只有府兵法,大家认为不能施行而最后作罢。‎ 范仲淹性情刚烈,但外表温和,本性十分孝顺,因为他母亲在时,家境正是贫困的时候,后来,(范仲淹)虽然做了大官,但不是家中来客人,吃饭时也不会有两个荤菜。妻儿的衣服和饭食,仅仅只能自己御寒和充饥。(范仲淹)对人好施予,在乡族中设置义庄,用以赡养族人。博爱善施,士大夫大多出自其门下,即使是小巷之人,都能说出他的名字。死的那天,四面八方凡是听到这一消息的人,都替他叹惜。(范仲淹)治理国家崇尚忠厚,他到过的地方都留下了他的恩德,邠州、庆州的老百姓和众多的羌族部族,都画上他的像在他生前就建立祠堂祭祀他。等到他死了,羌族首领率领数百人,像(失去自己的)父亲一样为他痛哭,并斋戒三日才离开。‎ 当初,范仲淹生病时,仁宗皇帝经常派人送药、慰问,病逝后,仁宗皇帝嗟叹哀悼很长时间。又派人去慰问其家人,下葬后,仁宗皇帝亲自题写了墓碑,叫“褒贤之碑”。‎ 考点九 古代文化常识 考点名片 考点内容 文学常识和文化常识。‎ 包括作家、作品、国别、朝代、风格流派、作品内容、主要情节、人物形象;古代典籍知识、文物典故、朝代沿革、政治制度等。‎ 考查形式 ‎①单项客观选择题,夹杂在古文阅读题中出现。②主观表述题。结合名句名篇默写考查。‎ 趋势分析 ‎2018年文学常识、文化常识有可能单独设题或在文言文试题中设题考查。‎ 一、基础题型 ‎1.下列关于文化常识的解说,不正确的一项是(  )‎ A.古代经考试录取而进入中央、府、州、县各级学校,包括太学学习的生员,统称“诸生”。‎ B.“季秋”是指秋天的最后一个月,即农历九月,古代一个季度的三个月分别用“孟、仲、季”来表示。‎ C.因古代设官分职各有专司,故称“有司”;但“有司”并不是某个具体的部门,而是代指官吏。‎ D.“再拜”是指再次拜谢,这是古代一种隆重的礼节,用于表达敬意。‎ 答案 D 解析 “再次拜谢”错,应该是“拜两次”。‎ ‎2.下列关于文化常识的解说,不正确的一项是(  )‎ A.《左氏春秋》,简称《左传》,相传为鲁国史官左丘明所著,是我国现存最早的叙事详细的编年体史书。‎ B.“甫冠”指刚刚20岁。古代男子20岁行成年礼,束发戴冠,表示已成年。古代表示年龄的词语还有“总角”“豆蔻”“耄耋”等。‎ C.“昆弟”指兄弟,常比喻亲密友爱。古代表达此类意义的词语有很多,如“琴瑟和鸣”就特指兄弟情深。‎ D.“庙食”指的是古代有功之人死后,政府为他立庙,让他接受奉祀,享受祭飨。‎ 答案 C 解析 “琴瑟和鸣”特指夫妻感情。‎ ‎3.下列关于文化常识的解说,不正确的一项是(  )‎ A.“黜陟”中的“黜”指晋升、进用,“陟”指降职或罢免,词语指人才的进退、官吏的升降。‎ B.“射礼”,古代重武习射,常举行射礼。射礼有大射、宾射、燕射、乡射四种。‎ C.“旧典”,“典”有可以作为标准的书籍、标准或法则等意思,“旧典”指旧时的制度、法则,古代的典籍或老典故等。‎ D.“留守”,古时皇帝出巡或亲征,命大臣督守京城,便宜行事,担当这种职责的就称为“留守”。‎ 答案 A 解析 “黜”指降职或罢免,“陟”指晋升、进用。‎ ‎4.下列关于文化常识的解说,不正确的一项是(  )‎ A.《诗经》,中国古代第一部诗歌总集,分“风”“雅”“颂”三大类。‎ B.“致仕”指官员交还官职,类似于今天的退休制度。源于周代,汉以后形成制度。‎ C.古代官员为去世的父母在官府内闭门守孝叫“丁忧”,遭逢母亲丧事叫“丁母忧”。‎ D.“侍读”是官职名,其主要职责是陪帝王或皇子读书,为其讲学,供其顾问。‎ 答案 C 解析 “在官府内闭门守孝”的说法错误。“丁忧”‎ 指遭父母之丧,古代朝廷官员的父母亲若去世,无论此人任何官何职,从得知丧事的那一天起,必须离职回到原籍守孝。‎ ‎5.下列关于文化常识的解说,不正确的一项是(  )‎ A.“内艰”,古代指遭逢父母丧事。按照规定,父母死后子女要居家守丧,服丧期满后才能复出做官。‎ B.“夺情”指丧期未满,官员应诏除去丧服,出任官职。‎ C.“乞留”指百姓请求挽留官员。明朝重视当地百姓对地方官任期届满后去留的意见,很多官员都因百姓“乞留”而提升官级留任原职。‎ D.“大计”是明、清两代考核外官的制度,根据官员德业表现的等第来定赏罚。‎ 答案 A 解析 “内艰”指遭逢母亲丧事。‎ ‎6.下列关于文化常识的解说,不正确的一项是(  )‎ A.“微服”,指帝王或高官为隐蔽身份而改穿平民便服。‎ B.“赙助”,指赠助丧家的财物。‎ C.“践祚”,特指皇帝登临皇位。‎ D.“门生”,汉人称亲受业者为门生,相传受业者为弟子。‎ 答案 D 解析 汉人称亲受业者为弟子,相传受业者为门生。‎ ‎7.下列关于文化常识的解说,不正确的一项是(  )‎ A.古代帝王及其嫡长子养一些德高望重的老人,以便向他们求教,叫“乞言”。‎ B.“东宫”既指太子所居之宫,亦指太子,古代设三少、三公等侍奉太子。‎ C.《聊斋志异》简称《聊斋》,俗名《鬼狐传》,是中国清代著名小说家蒲松龄创作的文言短篇小说集;《女聊斋志异》是他专写女性主人公的作品。‎ D.“江左”是一个地理名词,一般指江东,即长江下游江南地区。古人习惯以东为左,以西为右。‎ 答案 C 解析 《女聊斋志异》不是蒲松龄的作品。‎ ‎8.下列关于文化常识的解说,不正确的一项是(  )‎ A.“吊伐”指把要惩处的人或有罪的人悬挂起来,人们一齐讨伐他(们)。这是古代在国家军队收复失地时常见的做法。‎ B.“生祠”是指为活人修建的祠堂。立生祠是古代信仰民俗,对于有德政的官员,人民为之立生祠,以颂其功德,并加以奉祀。‎ C.《易》也称《周易》,是中国传统思想文化中自然哲学与伦理实践的根源,是中国最古老的占卜术原著。‎ D.“部伍”指部曲行伍,是军队的编制单位,泛指部队。‎ 答案 A 解析 “吊伐”是“吊民伐罪”的省略,指抚慰百姓,讨伐有罪的人。‎ ‎9.下列关于文化常识的解说,不正确的一项是(  )‎ A.“黄老”是黄帝与老子的并称,道家以黄、老为始祖,因此也称道家为黄老。‎ B.“总角”指的是古时男子未成年时的发型,常用来指儿童时代。‎ C.《礼记》,中国儒家经典之一,是战国至汉初儒家礼仪论著的总集。其内容包括礼制和儒家哲学两部分。‎ D.古代以亲属关系的远近定丧服的轻重。期,穿一周年孝服的人。功,穿大功服(九个月)、小功服(五个月)的亲族。这都指关系比较近的亲属。‎ 答案 B 解析 “总角”指的是古人未成年时梳的发髻,不分男女。‎ ‎10.下列关于文化常识的解说,不正确的一项是(  )‎ A.古代称呼人时类型比较多,可以直称姓名,也可以称字、号、谥号,还可以称籍贯、官职。如称杜甫为杜工部,这是称字。‎ B.“爵”,君主国家贵族封号(中国古代分为“公”“侯”“伯”“子”“男”五等),如王安石被称为荆国公。‎ C.“左徒”,战国时楚国的官名,后人因屈原曾为楚怀王左徒,即用以指屈原。‎ D.“乞骸骨”是古代官吏因年老请求退职的一种说法,意谓使骸骨得归葬故乡,如《张衡传》:“视事三年,上书乞骸骨,征拜尚书。”‎ 答案 A 解析 称杜甫为杜工部,这是称官职。‎ ‎11.下列关于文化常识的解说,不正确的一项是(  )‎ A.太学,我国古代设于京城的最高学府。魏晋至明清或设太学,或设国子学(监),或两者同时设立,名称不一,制度也有变化,但均为传播儒家经典的最高学府。‎ B.教授,原指传授知识、讲课授业,后成为学官名。汉唐以后各级学校均设教授,主管学校课试具体事务。‎ C.殿试,国家级考试,皇帝主考,贡士参加,考上为“进士”。其中,第一名叫“状元”,第二名叫“探花”,第三名叫“榜眼”。‎ D.谥,谥号,是古人死后依其生前行迹而为之所立的称号,帝王的谥号一般由礼官议定;臣下的谥号由朝廷赐予。‎ 答案 C 解析 第二名叫“榜眼”,第三名叫“探花”。‎ ‎12.下列关于文化常识的解说,不正确的一项是(  )‎ A.诏,特指皇帝颁发的命令文告;诏令,文体名,古代帝王、皇太后或皇后所发命令、文告的总称。包括册文、制、敕、诏、策、令、玺书、教、谕等。‎ B.迟明,古代用于表示时辰的词语,意思是天亮得比较晚。古人最初是根据天色的变化将一昼夜划分为十二个时辰,各时辰各有其名,诸如平明、质明、平旦、旦明、食时、晡时、黄昏、人定等。‎ C.移,指调动官职,是平级调动。迁,也指调动官职,包括升级、降级、平级转调三种情况。为易于区分,人们常在“迁”字的前面或后面加一个字,升级叫迁升、迁授、迁叙、右迁,降级叫迁削、迁谪、左迁,平级转调叫转迁、迁官、迁调,离职后调复原职叫迁复。‎ D.左右,是古今异义词,一般指侍从,如“传以示美人及左右”一句中的“左右”就是这个意思;现代汉语中,一般作方位词,左和右两方面;也可作实词讲,意思是支配,操纵。‎ 答案 B 解析 迟明,表示时辰的词语,天将亮的时候,不是“天亮得比较晚”;也不是十二时辰之一。‎ ‎13.下列关于文化常识的解说,不正确的一项是(  )‎ A.进士甲科,科举制度中通过朝廷考试者称为进士,唐宋进士分甲乙科。‎ B.笏,古代大臣上朝拿着的手板,用玉、象牙或竹片制成,上面可以记事。‎ C.铁券,又称“铁契”,由古代皇帝颁赐功臣,世代可据此享受某种特权。‎ D.兵部尚书,主要负责兵部的文秘工作,兼有参谋职能,但是没有实权。‎ 答案 D 解析 兵部尚书,别称为大司马,统管全国军事的行政长官。‎ ‎14.下列关于文化常识的解说,不正确的一项是(  )‎ A.古代称呼一个人,可以称其名、字、号、谥号、籍贯、官职等,还可以兼称,“太宰文简公”就是兼称官职“太宰”与谥号“文简公”。‎ B.“起家”意思较多,在“白手起家”中指“创立事业”,在“布衣起家”中指“出身”,还有一个意思是指“从家中征召出来,授以官职”。‎ C.古代称父亲的姊妹(姑母)的儿子为内兄弟,称母亲的兄弟(舅父)的儿子为外兄弟。‎ D.“太守”为州郡最高行政长官。秦时置“郡守”,汉景帝时改称“太守”,隋初称“刺史”。《醉翁亭记》中就塑造了一个“醉能同其乐,醒能述以文”的太守形象。‎ 答案 C 解析 《仪礼·丧服》“舅之子”汉郑玄注:“内兄弟也。”《仪礼·士丧礼》:“外兄弟在其南南上。”郑玄注:“外兄弟,异姓有服者也。”孔颖达疏:“谓若舅之子、姑姊妹、从母之子等,皆是有服者也。”‎ ‎15.下列关于文化常识的解说,不正确的一项是(  )‎ A.礼部,六部之一,掌管礼仪、祭祀、学校、接待外宾等事,长官称为吏部尚书。‎ B.檄,古代最初写在木简上的官方文书,用于晓谕、征召、声讨,也可特指声讨的文告,著名的如骆宾王的《讨武曌檄》。‎ C.践阼,特指皇帝登临皇位。君主即位也可叫“践祚”“登极”“登庸”“御极”。‎ D.卒,“死”的一种说法,古代天子、太后之死称崩、百岁、千秋、山陵崩等;对公、侯、伯的世爵之死,称为“薨”;有官职,有名望的人死曰“卒”;只有平民之死,才直言不讳地通称“死”。‎ 答案 A 解析 礼部的长官称为礼部尚书。‎ ‎16.下列关于文化常识的解说,不正确的一项是(  )‎ A.“太平兴国”是皇帝的年号,年号是历代封建王朝用来纪年的一种名号,一个皇帝所用年号少则一个,多则十几个。‎ B.“秩”的本意是根据官员的功过确定的官员俸禄,引申为俸禄。“秩满”是指官员在年终评定时获得了最高等级。‎ C.“金紫”,指“金印紫绶”,即黄金印章和系印的紫色绶带,借指高官显爵。唐宋后指金鱼袋及紫衣,是唐宋的佩饰和官服。‎ D.“转运使”,是封建王朝主管运输事务的官职。宋初为集中财权,置诸路转运使掌一路财赋,“都转运使”一般由官高位重者担任。‎ 答案 B 解析 “秩满”是任职期满之意。‎ 二、阅读题型 ‎[2017·中原名校质量考评]阅读下面的文言文,完成17~18题。‎ 吴元扆,字君华。太平兴国八年,选尚太宗第四女蔡国公主。‎ 雍熙三年,有事北边,元扆表求试剧郡,命知郓州。逾年召入,寻知河阳。还朝,改鄯州观察使。特诏朝会序班次节度使,奉禄赐予悉增之。再知河阳。淳化五年,秋霖河溢,奔注沟洫,城垒将坏,元扆躬涉泥滓,督工补塞。民多构木树杪以避水,元扆命济以舟楫,设饼饵以食。时澶、陕悉罹水灾,元扆所部赖以获安。‎ 真宗即位,换安州观察使,俄知澶州。咸平三年,转运使刘锡上其治状,诏书嘉奖,迁宁国军留后、知定州。时王超、王继忠领兵逾唐河,与辽人战,元扆度其必败,乃急发州兵护河桥。既而超辈果败,辽人乘之,至桥,见阵兵甚盛,遂引去。考满,吏民诣阙贡马,疏其善政十事,愿借留树碑,表其德政。诏褒之。属岁旱,吏白召巫以土龙请雨。元扆曰:“巫本妖民,龙止兽也,安能格天?惟精诚可以动天。”乃集道人设坛,洁斋三日,百拜祈祷,澍雨沾洽。‎ 景德三年代归,拜武胜军节度。出知潞州。初,并、代、泽、潞皆分辖戍卒,后并于太原。至是以元扆临镇遂分领七州军戎事委元扆专总之东封表求扈从命祀青帝礼毕加检校太傅知徐州大中祥符四年,元扆被疾卒,年五十。赠中书令,谥忠惠。子弟进秩者五人。五年,葬元扆,时上元欲观灯,帝为移次夕。‎ 元扆性谨让,在藩镇有忧民心,待宾佐以礼。喜读《春秋左氏》,声色狗马,一不介意。所得禄赐,即给亲族孤贫者。将赴徐州,请对言:“臣族属至多,其堪禄仕者皆为表荐,余皆均奉赡之。”公主有乳媪,得入参宫禁,元扆虑其去后妄有请托,白上拒之。真宗深所嘉叹,于帝婿中独称其贤。及殁,甚悼惜之。‎ ‎(节选自《宋史·吴元扆传》)‎ ‎17.对文中画波浪线部分的断句,正确的一项是(  )‎ A.至/是以元扆临镇/遂分领七州军戎事/委元扆专总之/东封/表求扈从命/祀青帝/礼毕/加检校太傅/知徐州 B.至是/以元扆临镇/遂分领七州军戎事/委元扆专总之/东封/表求扈从/命祀青帝/礼毕/加检校太傅/知徐州 C.至/是以元扆临镇/遂分领七州/军戎事委元扆/专总之东封/表求扈从/命祀青帝/礼毕/加检校太傅/知徐州 D.至是/以元扆临镇/遂分领七州/军戎事委元扆/专总之东封/表求扈从命/祀青帝/礼毕/加检校太傅/知徐州 答案 B 解析 “至是”,到这时,不能拆开,排除A、C两项;“遂分领七州军戎事”的主语为“元扆”,“委元扆专总之”的主语是朝廷,排除D项。‎ ‎18.对文中加点词语相关内容的解说,不正确的一项是(  )‎ A.公主是中国古代对皇女、王女的封号,公主封号命名常用封国名、郡县名和褒义词。‎ B.河阳指黄河北岸某地区,古代的“河”多专指黄河,古人认为“山南水北谓之阳”。‎ C.上元本指新的一年第一次月圆之夜,上元节也就是元宵节,又称“元夕”“灯节”。‎ D.《春秋左氏》即《左传》,是我国第一部编年体史书,相传为鲁国史官左丘明所作。‎ 答案 A 解析 公主,是中国古代对皇女、王女、宗女的封号。通常是皇女位号,只有在特殊情况下,宗女才能破格晋封为公主。‎ 参考译文:‎ 吴元扆字君华。太平兴国八年,被选中与宋太宗的第四个女儿蔡国公主成亲。‎ 雍熙三年,北边有战事,元扆上表请求任用为剧郡郡守,令他掌管了郓州。第二年命他入朝,不久又掌管河阳。回到朝廷,改任鄯州观察使。又特别诏令他为朝会上排列官员位次的节度使,俸禄赐予全部增加。再次任河阳知州。淳化五年,秋雨不止,河流涨水,奔流注入沟渠,城池营垒将要被毁坏,元扆亲自踏入(进入)泥渣,督促工匠填补堵塞。百姓多在树梢上架木造屋来避水,元扆命令用船载他们渡水,安排饼子给他们吃。当时澶、陕二地都遭受水灾。元扆所管辖的地方依赖他都获得了安宁。‎ 宋真宗即位,(元扆)改任安州观察使,不久掌管澶州。咸平三年,转运使刘锡把元扆施政的成绩上奏给朝廷,皇帝下诏书嘉奖元扆,他升为宁国军留后,掌管定州。当时王超、王继忠带兵越过唐河,和辽人作战,元扆估计他们必定失败,于是紧急调发定州军队保护河桥。不久,王超那些人果然失败,辽人乘胜到了桥边,见到当地的宋军布置了很多士兵,就率军退回了。任朝(考核期)已满,官民们到朝廷进贡马,上疏他良好政绩的十件事,希望继续留用他,为他树碑,表彰他的德政。皇帝下诏褒奖了他。恰好这一年大旱,官吏告诉他要叫巫师用土龙求雨。元扆说:“巫师本是妖民,龙只是兽,哪里能够推究天意?只有用至诚可以来打动天。”于是集结道士设祭坛,清洁斋戒三天,经百拜祈祷,下了及时雨,且雨水充沛。‎ 景德三年别人代任回朝,被授予武胜军节度。又外调掌管潞州。起初,并、代、泽、潞等州都分管辖戍边士卒,后来全归并太原。到这时,因元扆镇守此镇,就分管七州军戎事,委派元扆专权总揽军事。皇帝到东边泰山封禅,他上表请求随从,命令他祭祀青帝。礼完,他被加官检校太傅、掌管徐州。大中祥符四年,元扆遭遇疾病死去,年五十。赠官中书令,谥号忠惠。子弟中晋升官职,增加俸禄的人五个。(景德)五年,安葬元扆,当时元宵节要观灯,皇帝为此移到了第二天晚上。‎ 元扆性情谨慎谦让,在藩镇时有忧民之心,以礼对待手下的谋士属官。他喜读《春秋左氏》,声色狗马,全不在意。所得的俸禄赏赐,都分给亲戚家族中孤苦贫寒的人。将要到徐州时,请求说:“我的同族亲属很多,那些能够拿俸禄当官的都被上表推荐了,其余的都由我侍奉赡养他们。”公主有个乳母,能够进入皇宫参见皇帝妃子等,元扆怕她离开后会随意请求,告诉皇上拒绝她。真宗对他很赞赏,在帝婿中唯独称他贤能。等到他死了,很是痛惜深切悼念他。‎ ‎[2017·黄石调研]阅读下面的文言文,完成19~20题。‎ 罗亨信,字用实,东莞人。永乐二年进士,授工科给事中。进吏科右给事中,坐累谪交阯为吏。仁宗嗣位,始召入为御史。英宗即位之三月,擢右佥都御史,练兵平凉、西宁。正统二年,蒋贵讨阿台、朵儿只伯,亨信参其军务。至鱼儿海,贵等以刍饷不继,留十日引还。亨信让之曰:“公等受国厚恩,敢临敌退缩耶?死法孰与死敌?”贵不从。亨信上章言贵逗遛状。帝以其章示监督尚书王骥等。明年进兵,大破之。亨信以参赞功,进秩一等。父丧归葬。还朝,改命巡抚宣府、大同。参将石亨请简大同民三之一为军,亨信奏止之。十年进右副都御史,巡抚如故。时遣官度二镇军田,一军八十亩外,悉征税五升。亨信言:“‎ 文皇帝时,诏边军尽力垦田,毋征税,陛下复申命之。今奈何忽为此举?塞上诸军,防边劳苦,无他生业,惟事田作。每岁自冬徂春迎送瓦剌使臣三月始得就田七月又复刈草八月以后修治关塞计一岁中曾无休暇。况边地硗瘠,霜早收薄,若更征税,则民不复畊,必致窜逸。计臣但务积粟,不知人心不固,虽有粟,将谁与守?”帝纳其言而止。初,亨信尝奏言:“也先专候衅端,以图入寇。宜预于直北要害,增置城卫为备。不然,恐贻大患。”兵部议,寝不行。及土木之变,人情汹惧。有议弃宣府城者,官吏军民纷然争出。亨信仗剑坐城下,令曰:“出城者斩!”又誓诸将为朝廷死守,人心始定。也先挟上皇至城南,传命启门。亨信登城语曰:“奉命守城,不敢擅启。”也先逡巡引去。赤城、雕鹗、怀来、永宁、保安诸守将弃城遁,并按其罪。当是时,车驾既北,寇骑日薄城下,关门左右皆战场。亨信与总兵杨洪以孤城当其冲,外御强寇,内屏京师。着兜鍪处,颠发尽秃。景帝即位,进左副都御史。明年,年七十有四矣,乞致仕,许之。归八年,卒于家。‎ ‎(节选自《明史·罗亨信传》,有删改)‎ ‎19.给文中画波浪线的句子断句,正确的一项是(  )‎ A.每岁自冬徂春/迎送瓦剌/使臣三月始得就田/七月又复刈草/八月以后/修治关塞计一岁/中曾无休暇。‎ B.每岁自冬徂春/迎送瓦剌/使臣三月始得就田/七月又复刈草/八月以后修治关塞/计一岁中曾无休暇。‎ C.每岁自冬徂春/迎送瓦剌使臣/三月始得/就田七月/又复刈草八月/以后修治关塞/计一岁中曾无休暇。‎ D.每岁自冬徂春/迎送瓦剌使臣/三月始得就田/七月又复刈草/八月以后/修治关塞/计一岁中曾无休暇。‎ 答案 D 解析 由句意可知,“三月始得就田”的主语不是“使臣”,排除A、B。“三月就田”“七月刈草”对应,不可拆分,排除C项,故答案为D。‎ ‎20.下列对文中加点词语的相关内容的解说,不正确的一项是(  )‎ A.自秦朝开始,御史专门作为监察性质的官职,负责监察朝廷、诸侯官吏,一直延续到清朝。‎ B.正统为明朝第六个皇帝明英宗朱祁镇登基后的年号。汉武帝起开始有年号,从此以后,新皇帝即位,都要确定自己的年号,一个皇帝一般有多个年号。‎ C.车驾本是帝王所乘的车,这里亦用为帝王的代称。‎ D.古代官员正常退休叫作”致仕”,古人还常用致事、致政、休致等名称,盖指官员辞职归家。‎ 答案 B 解析 元朝以后一般只有一个年号。‎ 参考译文:‎ 罗亨信,字用实,东莞人。永乐二年考中进士,授予工科给事中。晋升为吏科右给事中,因受牵连获罪被贬谪到交阯为小吏。仁宗即位,才召入朝中为御史。英宗即位后的第三个月,提升为右佥都御史,在平凉、西宁操练兵马。正统二年,蒋贵征讨阿台和朵儿只伯,罗亨信参与军务。到鱼儿海,蒋贵等人因粮饷不继,居留十日率军返回。罗亨信责备他们说:“你们受到国家优厚的恩待,怎么敢面对敌人而退缩呢?因触犯国法而死与和敌人作战而死哪一个更值得?”蒋贵不听从。罗亨信上奏章说蒋贵逗留的状况。皇帝把他的奏章给监督尚书王骥等人看。第二年进兵,大破敌军。罗亨信凭借参赞功劳,晋级一等。父亲去世后回家送葬。返回朝廷时,改任为宣府、大同巡抚。参将石亨请求选拔三分之一的大同居民为兵员,亨信上奏制止这件事。正统十年,晋升为右副都御史,仍任宣府大同巡抚。当时皇上派遣官员测量两镇军田,一军八十亩以外,都征税五升。罗亨信说:“文皇帝时,诏令边防军队尽力垦田,不征税,陛下又重申这条命令。如今为何忽然采取这一举措呢?塞上各支军队,防卫边疆劳苦,没有其他生计,只有从事田耕。每年自冬到第二年春,迎送瓦剌使臣,三月才开始耕种,七月又开次割草,八月以后,修治关塞,一年中几乎没有闲暇的时间。况且边塞土地贫瘠,霜露早降,收入微薄,如果再征税,那么人民不会再耕种,必然会导致逃逸。谋臣只专注于收粮食,不知道民心不稳定,即使有粮食,边关谁来守?”皇帝采纳他的话,停止了这种做法。当初,罗亨信曾上奏说:“也先专等时机,以图入侵。应事先在正北要塞,增置城卫来防备。不这样做,恐怕贻留大患。”兵部议论,废置不用。等到土木之变,人心惶恐不安。有建议放弃宣府城的,官吏军民纷争而出。罗亨信持剑坐在城下,下令说:“出城的人杀头。”又与诸将发誓为朝廷死守,人心才安定下来。也先挟持上皇到城南,传令开门。罗亨信登城告诉他们说:“奉命守城,不敢擅自开启。”‎ 也先徘徊一会离开了。赤城、雕鹗、怀来、永宁、保安各城守将弃城逃跑,一并追究他们的罪责。正在那时候,皇帝被俘,敌骑兵日日逼近城下,关门左右都是战场。亨信与总兵杨洪因为孤城处在交通要道,对外抵御强敌,对内保卫京城。戴头盔之处,头发全秃光。景帝即位,晋升为左副都御史。第二年,已经七十四岁了,他请求辞官,(皇帝)答应了他。回去八年后,死在家中。‎ 考点十 归纳内容要点、概括中心意思、分析观点态度 考点名片 考点内容 古代人物传记,考查点包括:时间、地点、人物、事件的前因后果;记了哪几件事,表现了人物的哪些性格特点和精神风貌。‎ 考查形式 选择对文章内容的概括和分析正确或者不正确的一项,单项选择,3分。‎ 趋势分析 题型保持相对稳定,一般设一至两道客观题。‎ 一、阅读下面的文言文,完成1~2题。‎ 贾谊 班固赞曰:“刘向称‘贾谊言三代与秦治乱之意,其论甚美,通达国体,虽古之伊、管未能远过也。使时见用,功化必盛,为庸臣所害,甚可悼痛!’追观孝文玄默躬行,以移风俗,谊之所陈略施行矣。及欲改定制度,以汉为土德,色上黄,数用五,及欲试属国,施五饵三表以系单于,其术固以疏矣。谊亦天年早终,虽不至公卿,未为不遇也。凡所著述五十八篇,掇其切要于事者著于《传》云。”‎ 李卓吾曰:班氏文儒耳,只宜依司马氏例以成一代之史,不宜自立论也。立论则不免搀杂别项经史闻见,反成秽物矣。班氏文才甚美,其于孝武以前人物,尽依司马氏之旧,又甚有见,但不宜更添论赞于后也。何也?论赞须具旷古双眼,非区区有文才者所能措也。刘向亦文儒也,然筋骨胜,肝肠胜,人品不同,故见识亦不同,是儒而自文者也。虽不能超于文之外,然与固远矣。‎ 汉之儒者咸以董仲舒为称首,今观仲舒不计功谋利之云,似矣。而以明灾异下狱论死,何也?夫欲明灾异,是欲计利而避害也。今既不肯计功谋利矣,而欲明灾异者何也?既欲明灾异以求免于害,而又谓仁人不计利,谓越无一仁又何也?所言自相矛盾矣。且夫天下曷尝有不计功谋利之人哉!若不是真实知其有利益于我,可以成吾之大功,则乌用正义明道为耶?其视贾谊之通达国体,真实切用何如耶?‎ 班氏何知,知有旧时所闻耳,而欲以贬谊,岂不可笑!董氏章句之儒也,其腐固宜。虽然,董氏特腐耳,非诈也,直至今日,则为穿窬之盗矣。其未得富贵也,养吾之声名以要朝廷之富贵,凡可以欺世盗名者,无所不至。其既得富贵也,复以朝廷之富贵养吾之声名,凡所以临难苟免者,无所不为。岂非真穿窬之人哉!是又仲舒之罪人,班固之罪人,而亦敢于随声雷同以议贾生。故余因读贾、晁二子经世论策,痛班氏之溺于闻见,敢于论议,遂为歌曰:驷不及舌,慎莫作孽!通达国体,刘向自别。三表五饵,非疏匪拙。彼何人斯?千里之绝。汉廷诸子,谊实度越。利不可谋,何其迂阔!何以用之?皤须鹤发。从容庙廊,冠冕佩玦。世儒拱手,不知何说。‎ ‎1.以下各句子中,全都表现欺世盗名的“穿窬之人”的行径的一组是(  )‎ ‎①养吾之声名以要朝廷之富贵 ‎②欲明灾异,是欲计利而避害 ‎③复以朝廷之富贵养吾之声名 ‎④董氏特腐耳,非诈也 ‎⑤而亦敢于随声雷同以议贾生 ‎⑥凡所以临难苟免者,无所不为 A.①②④ B.②③⑤‎ C.③④⑥ D.①⑤⑥‎ 答案 D 解析 ②表明弄清灾祸与异常之事,就是想要趋利避害的道理。④说明董仲舒与那些欺世盗名的人还不同,不符合题干要求。此题的人物是“欺世盗名”“穿窬之人”,要点是“行径”,②④不符合题干要求。‎ ‎2.下列对本文的理解与分析,不正确的一项是(  )‎ A.刘向认为贾谊通晓国家治理之道,如果能得到重用,一定能让国家兴盛,却被庸臣残害,很让人痛惜。‎ B.李卓吾认为,班固的文章很有文采,他对孝武以前的人物,都按照司马氏的旧论,很有见解。‎ C.作者认为董仲舒的言行自相矛盾,他是寻章摘句的儒生。‎ D.文章论点明确,赞美了贾谊,也兼评了董仲舒,对班固褒扬,而对刘向贬抑,认为他不及班固。‎ 答案 D 解析 归纳内容要点,概括中心意思的题目是文言文必考的题目,错误选项的设置多以人物、事件、时间、地点的错位为重点,答题时注意分析选项涉及的上下文,注意核实这些情节,如此题选项D概括不准确,作者对班固和刘向均有褒有贬。‎ 参考译文:‎ 班固评论(《汉书》《后汉书》纪传结尾部分有“赞”,相当于对人物或史实的总评)说:“刘向认为:‘贾谊解说夏商周三代和秦朝安定与动乱的观点,其论断之佳,通晓洞达国家的典章制度,即使古时候的伊尹、管仲,也不能超过他。如果他在当时能得到重用,一定能让国家兴盛,却被庸臣残害,很让人哀悼痛惜。’我们回顾历史,看孝文帝沉静地身体力行,来改变风俗,贾谊所陈述的政治主张基本上得到了实施。至于说到他想改革制度,认为汉是土德,(古代君主认为自己的统治与金木水火土中某一物质的属性相符,该物质能给他的统治带来祥瑞征兆,则称有某德,或以某德王,如黄帝以土德之瑞,炎帝以火德王等),颜色崇尚黄色(上,通‘尚’),数字以五为吉利数字,等到他把这种政治主张试用于属国,用五饵三表的政策来对付单于,他的办法本来就很浅薄了。贾谊英年早逝,虽然没做到公卿,也不算是不得志。他所著述的五十八篇著作,选择那些言论时事切中肯綮的收录在《传》里。”‎ 李卓吾说:班氏只是一个书生,只应该按照司马迁的体例来修订历史,不应该自己发表议论。自己立论就不免受到别的经史书籍上的观点的影响,反而成了秽物(比喻平庸的见识)。班氏很有文采,他对孝武以前的人物,都按照司马氏的旧论,很有见解,但不应该再在后面添加论赞。为什么呢?评论必须具有洞察历史的真知灼见,不是光有点文采的人就能做到的。刘向也是文儒,但是他的筋骨、肝肠胜过班固,人品不一样,所以见识也就不同,是有学问而自然有文采的人。虽然没超出“文人”的境界,但是跟班固相比就远胜于他了。‎ 汉代的儒者最推崇董仲舒,现在看仲舒不计功谋利的观点,这评价大致是恰当的了。但董氏却因为明察灾祸与异常之事下狱被判处死罪,为什么呢?想弄清楚灾祸与异常之事,这是想要趋利避害呀!现在既然不肯算计、谋求功利,却想要洞察灾异之事的原因是什么呢?既然想明察灾民之事来求得避免祸患,却又说仁德的人不算计利益,认为越国人没有一个有仁德的人又是为什么呢?他所说的话自相矛盾了。况且天下何尝有不计较功利的人呢?如果不是确实知道它对我有利,可以成就我的事业,那么还用得着推崇道义,明白道理吗?他与贾谊的通晓国家大事,(并且他的主张)切合实用比起来怎么样呢?(二人孰高孰下呢)‎ 班氏懂得什么,知道一些历史事件罢了,却想凭借这个来批评贾谊,难道不可笑吗?董氏是寻章摘句的儒生(指只会死读书,解释书中的字词而不切实际),他的迂腐本来就是理所当然的。虽然如此,董氏只是迂腐,并不是虚伪。到了今日的儒生,就成了爬墙的窃贼了。他们未得到富贵时,想办法获得美好的名声来得到朝廷的富贵,凡是能够欺骗世人盗取名声的事,没有干不出来的,他们得到富贵之后,又用朝廷给他的富贵来替自己赢得美好的声名。凡是遇到危难时可以苟且偷生的事,没有干不出来的。难道不真是爬墙的窃贼吗?这些人又是仲舒的罪人,班固的罪人(意为他们的品行道德还远不如董、班,在他们面前会觉得惭愧,感到有罪),却也敢于随声附和来议论贾生。所以我因为读了贾谊、晁错二人治理天下的论策(论、策均为一种文体),痛恨班氏受到自己见闻的影响,(没有见识却)敢于来发议论,于是作歌道:“马还没有舌头跑得快,小心不要说错话造孽呀!(贾谊是否)通晓洞达国家的典章制度,刘向自然能判断(别,区别、辨别,引申为判断,意为刘向能作出恰当的评价)。三表五饵,不是浅薄愚笨的见解。贾谊是什么人呢?是千里挑一的人才呀!汉朝诸人,贾谊确实是出类拔萃的(度越,指远远超过他人)。(董氏认为)不能谋求利益,见解是多么迂腐呀!谁可以采用他的主张呢?恐怕只有白胡子白头发的仙人才能这样吧(不谋求利益)!(贾谊)从容站立在庙堂之上,戴着帽子,佩着玉玦。世上的儒生拱手表示钦服,不知道有什么话说(指佩服之极)。‎ 二、阅读下面的文言文,完成后面3~4题。‎ 顾成,字景韶,其先湘潭人。祖父业操舟,往来江、淮间,遂家江都。成少魁岸,膂力绝人,善马槊,文其身以自异。太祖渡江,来归,以勇选为帐前亲兵,擎盖出入。尝从上出,舟胶于沙,成负舟而行。从攻镇江,与勇士十人转斗入城,被执,十人皆死。成跃起断缚,仆持刀者,脱归。导众攻城,克之,授百户。大小数十战,皆有功,改成都后卫。洪武六年,擒重庆妖贼王元保。‎ 八年调守贵州。时群蛮叛服不常,成连岁出兵,悉平之。已,从颍川侯傅友德征云南,为前锋,首克普定,留成列栅以守。蛮数万来攻,成出栅,手杀数十百人,贼退走。余贼犹在南城,成斩所俘而纵其一,曰:“吾夜二鼓来杀汝。”夜二鼓,吹角鸣砲,贼闻悉走,获器甲无算。进指挥使。诸蛮隶普定者悉平。成在贵州凡十余年,讨平诸苗洞寨以百数,皆诛其渠魁,抚绥余众。恩信大布,蛮人帖服。是年二月,召还京。‎ 建文元年,为左军都督。从耿炳文御燕师,战真定,被执。燕王解其缚曰:“此天以尔授我也!”送北平,辅世子居守。南军围城,防御、调度一听于成。燕王即位,论功,封镇远侯,食禄千五百石,予世券。命仍镇贵州。‎ 成性忠谨,涉猎书史。始居北平,多效谋画,然终不肯将兵,赐兵器亦不受。再镇贵州,屡平播州、都匀诸叛蛮,威镇南中,土人立生祠祀焉。其被召至京也,命辅太子监国。成顿首言:“太子仁明,廷臣皆贤,辅导之事非愚臣所及,请归备蛮。”时群小谋夺嫡,太子不自安。成入辞文华殿,因曰:“殿下但当竭诚孝敬,孳孳恤民。万事在天,小人不足措意。”十二年五月卒,年八十有五。赠夏国公,谥武毅。‎ ‎(选自《明史·列传第三十二》,有改动)‎ ‎3.以下各组句子中,全都直接表明顾成多谋善战的一组是(  )‎ ‎①洪武六年,擒重庆妖贼王元保 ‎②手杀数十百人,贼退走 ‎③成斩所俘而纵其一 ‎④燕王即位,论功,封镇远侯 ‎⑤土人立生祠祀焉 ‎⑥万事在天,小人不足措意 A.①③⑤ B.①②③‎ C.②④⑥ D.③⑤⑥‎ 答案 B 解析 文言文信息筛选的题目,大多是人物性格分析,答题时注意题干的要求是“什么人”的“什么”品质,还要注意有没有“直接表现”“间接表现”等字样,该题中是“全都直接表明”,注意运用排除法作答。此题是人物是“顾成”,品质是“多谋善战”,然后分析列出的各项,④不是直接表现其多谋善哉,⑤⑥都不是表战争的内容;排除这三项,自然得到答案。‎ ‎4.下列对原文有关内容的概括和分析,不正确的一项是(  )‎ A.顾成,祖上以摆渡为业。他身材魁梧,身刺文身,力大无穷,作战勇敢,因此得以在太祖身边擎举华盖,深得太祖的信任。‎ B.明太祖时,顾成凭借战功,从帐前亲兵升为百户、成都后卫、指挥使,后因随傅友德征云南立功,而被封为左军都督。‎ C.虽然是败军之将,但是顾成深受燕王的倚重,得到他后,就把他送到北平,让他去辅佐世子。南军围城的时候,又让他统管防御、调度。‎ D.顾成侍奉三朝,他除了忠诚还很谨慎,比如他在北平,虽然常出谋划策,但是不肯统兵;也不肯辅佐太子监国。‎ 答案 B 解析 到文中找到相关内容,然后再与选项进行比较,即可发现,“明太祖时被封为左军都督”错,顾成被封为左军都督是在建文元年。‎ 参考译文:‎ 顾成,字景韶,祖先是湘潭人。他的祖父以操舟为业,架船在长江、淮河之间往来谋生,于是就在江都安了家。顾成年少时身材魁伟高大,臂力超人,善于骑马使槊,文身来显示自己与众不同。太祖渡过长江,前来归顺,因为英勇被选为帐前亲兵,为太祖出入撑举华盖。曾经跟随皇上出征,船陷在沙中,顾成背负船只前行。跟从攻打镇江,和十名勇士转战入城,被捉拿,那十人都死了。顾成跃起挣断绑绳,打倒持刀要杀他的人,逃脱回来。引导众人攻城,攻下了城池,太祖授他百户之职。他参加大小战役数十次,都有功劳,改任成都后卫。洪武六年,擒获重庆的妖贼王元保。‎ 洪武八年调任驻守贵州。当时群蛮叛乱归服反复无常,顾成连年出兵,全都平定。不久,跟随颍川侯傅友德出征云南,作为前锋,首先攻下普定,留下顾成排列栅栏来防守。蛮族数万人来攻打,顾成出栅迎战,亲手杀死数十百人,贼人逃走。余下的贼人还在南城,顾成斩杀俘获的贼寇而释放了一人,说:“我夜晚二更来杀你们。”夜二更,吹起号角放起火砲,贼人听到后全都逃走,缴获器物甲衣无数。进升指挥使。隶属普定的众蛮全都被平定。顾成在贵州总共十余年,讨伐平定诸苗洞寨以百计,全都诛杀了他们的头领,安抚其余众人。广泛布施恩德信义,蛮人顺服。这年二月,被召回京。‎ 建文元年,任左军都督。跟随耿炳文抵御燕军,在真定作战,被俘。燕王解开他身上的绑绳说:“这是上天把你送给我啊!”送往北平,辅佐世子守卫。南军围困城池,(燕军)防御、调度全都听从于顾成。燕王即位,论功行赏,封(顾成)为镇远侯,俸禄一千五百石,给予世袭的文书。任命他仍旧镇守贵州。‎ 顾成性情忠诚谨慎,广泛阅读史书。当初在北平居住时,多出谋划策,然而始终不肯统率军队,赐给他兵器也不接受。再次镇守贵州,多次平定播州、都匀的叛乱蛮众,威镇南中,当地人建立生祠祭祀他。他被召回到京师,被命令辅佐太子监国。顾成叩首说:“太子仁德圣明,朝中大臣都是贤才,辅导的事情不是我所做到的,请准予我回去防备蛮人。”当时有些奸臣阴谋改立太子,太子因而忧心忡忡。顾成到文华殿向太子辞行,趁机劝太子说:“殿下只该竭诚孝敬皇帝,兢兢业业为老百姓办事,万事在天,对那帮生事的小人不足费心。”(永乐)十二年五月去世,时年八十五岁。追赠为夏国公,谥号为“武毅”。‎ 三、阅读下面的文言文,完成后面5~6题。‎ 杜暹,濮阳人也。自暹高祖至暹,五代同居,暹尤恭谨,事继母以孝闻。初举明经,补婺州参军,秩满将归,州吏以纸万余张以赠之,暹惟受一百,余悉还之。时州僚别者,见而叹曰:“昔清吏受一大钱,复何异也!”俄授郑尉,复以清节见知,华州司马杨孚深赏重之。寻而孚迁大理正,暹坐公事下法司结罪,孚谓人曰:“若此尉得罪,则公清之士何以劝矣?”‎ 开元四年,迁监察御史,仍往碛西覆屯。会安西副都护郭虔瓘与西突厥可汗史献等不协,更相执奏,诏暹按其事实。时暹已回至凉州,承诏复往碛西,因入突骑施,以究虔瓘等犯状。蕃人赍金以遗,暹固辞不受。左右曰:“公远使绝域,不可失蕃人情。”暹不得已受之,埋幕下,既去出境,乃移牒令收取之。蕃人大惊,度碛追之,不及而止。暹累迁给事中,丁继母忧去职。十二年,安西都护张孝嵩迁为太原尹,或荐暹往代之,蕃人伏其清慎,深思慕之,乃夺情擢拜安西副大都护。明年,于阗王尉迟眺阴结突厥及诸蕃国图为叛乱,暹密知其谋,发兵捕而斩之,并诛其党羽五十余人,更立君长,于阗遂安。暹在安西四年,绥抚将士,不惮勤苦,甚得夷夏之心。‎ 二十年,为户部尚书。上行幸东都,诏暹为京留守。暹因抽当番卫士,缮修三宫,增峻城隍,躬自巡检,未尝休懈。上闻而嘉之曰:“卿素以清直,兼之勤干。自委居守,每事多能,政肃官僚,惠及黎庶。慰朕怀也。”二十八年,病卒,谥曰“贞孝”。‎ 暹在家孝友,爱抚异母弟昱甚厚。常以公清勤俭为己任,弱冠便自誓不受亲友赠遗,以终其身。及卒,上甚悼惜之,遣中使就家视其丧事,内出绢三百匹以赐之。尚书省及故吏赙赠者,其子孝友遵其素约,皆拒而不受。‎ ‎(选自《旧唐书·列传第四十八》,有删改)‎ ‎5.以下各组句子中,全都直接表明杜暹“清直”的一组是(  )‎ ‎①州吏以纸万余张以赠之,暹惟受一百 ‎②蕃人赍金以遗,暹固辞不受 ‎③蕃人伏其清慎,深思慕之 ‎④躬自巡检,未尝休懈 ‎⑤弱冠便自誓不受亲友赠遗,以终其身 ‎⑥孝友遵其素约,皆拒而不受 A.①②④ B.③⑤⑥‎ C.①②⑤ D.③④⑥‎ 答案 C 解析 ③⑥均是间接表明杜暹的“清直”;④是表明杜暹的“勤干”。‎ ‎6.下列对原文有关内容的概括和分析,不正确的一项是(  )‎ A.杜暹孝顺友爱。自高祖到杜暹,五代同居,他在家里不仅特别恭敬谨慎地侍奉继母,而且十分关心爱护异母弟弟杜昱;在任给事中时,还因继母去世而辞去官职,回家守丧。‎ B.杜暹为官清廉。在他进入突骑施营帐查究蕃人与郭虔瓘等人的冲突时,不得已接受了蕃人赠送的黄金,但他把黄金埋在幕帐之下,走出蕃境以后,才传书让蕃人去收取。‎ C.杜暹行事果敢有方。在他出使碛西期间,于阗王尉迟眺秘密勾结突厥和一些蕃国图谋叛乱,杜暹暗中探知他的阴谋,便发兵逮捕斩杀了他,并诛杀其党羽,重新选任君长,使于阗得以安定。‎ D.杜暹勤干多能。他在安西四年,安抚将士,不怕劳苦,深得蕃汉人民拥戴;皇上巡幸东都,作为京城留守的杜暹抽调轮流值勤的卫士,修缮三宫,加高城墙,受到了皇上的称赞。‎ 答案 C 解析 此题要在准确把握文意的基础上,带着选项回到原文,从人物、时间、地点、事件的混淆和关键词语的误译等角度进行比较分析,做出判定。C项,见原文第二段应为“杜暹担任安西副大都护期间”。‎ 参考译文:‎ 杜暹,濮阳人。自杜暹的高祖到杜暹,一家五代同居,杜暹特别恭敬谨慎,侍奉继母以孝顺闻名。起初通过明经科考试,补任婺州参军,任满将还乡,州吏赠送他一万多张纸,杜暹只接受了一百张,其余全部还给州吏。当时来告别的州府官员,看到这种情景感叹道:“从前清廉的官吏接受一枚大钱,杜暹之举又有什么不同!”不久(杜暹)被任命为郑县县尉,又因清廉节俭而被人所知,华州司马杨孚非常赏识器重他。不久杨孚升任大理正,杜暹因公事获罪交由法司断罪,杨孚对人说:“如果这个县尉获罪,那么公正清廉之士靠什么来勉励呢?”‎ 开元四年,(杜暹)升任监察御史,于是前往漠西检查屯田驻军。适逢安西副都护郭虔瓘与西突厥可汗史献等不和,互相上奏指责,诏令杜暹查明事实。当时杜暹已返回凉州,接到诏令又前往漠西,于是进入突骑施营帐,以查究他们与郭虔瓘等人冲突的情况。蕃人以黄金相赠,杜暹坚决推辞不受。左右的人说:“您远道出使这边远的地方,不可辜负了蕃人的情意。”杜暹在不得已的情况下接受了黄金,埋在幕帐之下,走出蕃境以后,他才传书让蕃人去收取。蕃人非常吃惊,越过沙漠追赶他,追赶不上,方才作罢。杜暹多次升迁,任给事中,因继母去世辞去官职回家守丧。(开元)十二年,安西都护张孝嵩调任太原尹,有人推荐杜暹前往代替他,蕃人佩服他的清廉谨慎,十分思念仰慕他,于是(杜暹)丧期未满起任安西副大都护。第二年,于阗王尉迟眺秘密勾结突厥和一些蕃国图谋叛乱,杜暹暗中探知了他的阴谋,便发兵逮捕斩杀了他,并诛杀了他的党羽五十多人,重新选任了君长,于阗才安定下来。杜暹在安西四年,安抚将士,不怕劳苦,很得少数民族和汉人的拥戴。‎ ‎(开元)二十年,(杜暹)任户部尚书。皇上将要巡幸东都,诏令杜暹为京城留守。杜暹因此抽调轮流值勤的卫士,修缮三宫,加高城墙,他亲自巡查,从不懈怠。皇上听说后称赞他说:“卿一向清廉正直,而且又勤劳能干。自从担任留守,事事都能尽职,治政使官吏整肃,恩惠能施及百姓。朕心中十分欣慰。”(开元)二十八年,病逝,谥号为“贞孝”。‎ 杜暹在家孝顺友爱,十分关心爱护异母弟弟杜昱。(杜暹)常以公正清廉勤劳节俭为己任,年轻时立誓不接受亲友馈赠,一生如此。(他)去世后,皇上感到十分惋惜,派宦官到他家中探望丧事的办理,宫内拿出三百匹绢赐与他家。尚书省及过去与他共事的官吏赠给的丧葬财物,他的儿子杜孝友遵守父亲一向的约束,一概拒绝不曾接受。‎ 四、阅读下面的文言文,完成第7小题。‎ 司马光,字君实,陕州夏县人也。父池,天章阁待制。光生七岁,凛然如成人,闻讲《左氏春秋》,爱之,退为家人讲,即了其大指。自是手不释书。仁宗宝元初,中进士甲科,年甫冠。性不喜华靡,闻喜宴独不戴花,同列语之曰:“君赐不可违。”乃簪一枝。从庞籍辟,通判并州。麟州屈野河西多良田,夏人蚕食其地,为河东患。籍命光按视,光建:“筑二堡以制夏人,募民耕之,耕者众则籴贱,亦可渐纾河东贵籴远输之忧。”籍从其策。而麟将郭恩勇且狂,引兵夜渡河,不设备,没于敌,籍得罪去。光三上书自引咎,不报。籍没,光升堂拜其妻如母,抚其子如昆弟,时人贤之。改直秘阁、开封府推官。交趾贡异兽,谓之麟,光言:“真伪不可知,使其真,非自至不足为瑞,愿还其献。”又奏赋以风。修起居注,判礼部。有司奏日当食,故事食不满分,或京师不见,皆表贺。光言:“四方见京师不见,此人君为阴邪所蔽;天下皆知而朝廷独不知,其为灾当益甚,不当贺,从之。仁宗始不豫,国嗣未立,天下寒心而莫敢言。谏官范镇首发其议,光在并州闻而继之,且贻书劝镇以死争。至是,复面言:“臣昔通判并州,所上三章,愿陛下果断力行。”‎ 帝沉思久之,曰:“得非欲选宗室为继嗣者乎?此忠臣之言,但人不敢及耳。”光曰:“臣言此,自谓必死,不意陛下开纳。”帝曰:“此何害,古今皆有之。”光退未闻命,复上疏曰:“臣向者进说,意谓即行,今寂无所闻,此必有小人言陛下春秋鼎盛,何遽为不祥之事?小人无远虑,特欲仓卒之际,援立其所厚善者耳。‘定策国老’‘门生天子’之祸,可胜言哉?”帝大感动曰:“送中书。”光见韩琦等曰:“诸公不及今定议,异日禁中夜半出寸纸,以某人为嗣,则天下莫敢违。”琦等拱手曰:“敢不尽力。”未几,诏英宗判宗正,辞不就,遂立为皇子,又称疾不入。光言:“皇子辞不赀之富,至于旬月,其贤于人远矣。然父召无诺,君命召不俟驾,愿以臣子大义责皇子,宜必入。”英宗遂受命。‎ ‎(节选自《宋史·司马光传》)‎ ‎7.下列对原文有关内容的概括和分析,不正确的一项是(  )‎ A.司马光性情不喜华靡,禀赋迥异。七岁时,就能说出《左氏春秋》的大意;“闻喜宴”独不戴花,体现了他与众不同的胆识和品性。‎ B.司马光做事坚持原则,见解独到。在异兽和日食事件中,他设譬引喻,据理劝谏,朝廷最终听从了他的意见。‎ C.司马光为人忠厚笃诚,重情重义。在庞籍因事受牵连后,他能连续上书承担罪责,庞籍死后又能善待其家人。‎ D.司马光为臣鞠躬尽瘁,尽忠尽职。在关乎国嗣的问题上,他与范镇等人一起劝说天子为国家长远考虑,早立储君。‎ 答案 B 解析 “设譬引喻”错;异兽事件并没有提到“听从他的意见”。‎ 参考译文:‎ 司马光,字君实,是陕州夏县人。他的父亲司马池,曾任天章阁待制。司马光七岁的时候,严肃如同成年人,听别人讲《左氏春秋》,他特别喜爱,回家后将所听到的讲给家人,竟能说出其中的大意。此后,他整天手不释卷。宋仁宗宝元初年,(司马光)考中进士甲科,这年他刚二十岁。他不喜欢华丽,“闻喜宴”唯独司马光不戴花,一位同中进士的人告诉他说:“君王赏赐的花,不可违背。”于是(司马光)才戴上一枝。司马光跟随庞籍,担任并州判官。麟州屈野河西良田颇多,西夏人蚕食这一带土地,已成为河东的祸患。庞籍命令司马光去巡视,司马光建议说:“修筑两个城堡来抵御西夏人,招募老百姓去耕种土地,耕种的人多,买入粮食的价格就低,也就可以慢慢缓和河东粮食价格昂贵而依赖远处供给的忧虑。”庞籍听从了他的计策。麟州将领郭恩勇敢却有点狂妄,率兵在夜间渡河,不设防备,被敌方消灭,庞籍因此获罪,离开了麟州。司马光三次上书陈述郭恩覆灭是自己的过失,但没有得到答复。庞籍死后,司马光上堂拜见他的妻子,待她像母亲一样,抚养他的儿子就像自己的兄弟一样,当时的人都很称赞他。后来,(司马光)改任直秘阁、开封府推官。这时,交趾贡纳一种奇怪的野兽,说是麟,司马光说:“真假不得而知,即使是真的,不是自己来的也不能说明是祥瑞之兆,希望送还他们的所献之物。”他又写了一篇赋进行讽谏。他又修撰起居注,供职礼部。有关部门说有日食,按惯例,日食不满度数,或京师看不见,都应当上表祝贺。司马光说:“四方都看得见,京师看不见,这说明君王被阴险邪恶的小人所蒙蔽;天下的人都知道,唯独朝廷不知道,它所带来的灾害会更厉害,不应当庆贺。”朝廷听从了他的意见。仁宗身体不好,可国家的继承人还没有确定,天下的人都很忧虑却不敢说话。谏官范镇首先发表自己的看法,司马光在并州听说后,也发表意见,并且赠送书信给范镇,劝他以死抗争。到这时,司马光又对皇上当面说:“我过去在并州作通判,所上的文书三章,希望陛下果断笃行。”皇帝深思了好久,然后说:“莫非想要挑选宗室后代作为继承人吗?这的确是忠臣说的话,只是别人不敢说罢了。”司马光说:“我说这个,自己认为一定会招来死罪,没想到皇上如此开明,竟然接受了。”皇上说:“这有什么不好,古今都有这类事。”司马光退朝后没有听说有诏令,就又上书说:“我前面所说,心想立即就会施行,现在却没听到什么,这中间一定有小人说陛下正当壮年,怎么立即(就)做不吉祥的事?那些小人没有长远的考虑,只是想在仓猝之间,拉拢他们所着重的和友善的。‘定策国老’‘门生天子’之祸,能说得完吗?”皇帝大为感动,说:“将司马光的上书送到中书省。”司马光见到韩琦等人说:“诸公趁早商定,他日宫中半夜传出寸纸,把某人作为继承人,全国的人不敢违背。”韩琦等双手作揖说:“不敢不尽力。”不久,下诏让英宗负责宗正事务,(英宗)一再推辞,不肯答应,于是被立为皇子,(英宗)又说自己有病,不肯入宫。司马光说:“‎ 皇子推辞富贵,已经快一个月了,可见他比一般人贤惠得多。然而,对父亲的召唤不答应,君王征召,却不肯应征,希望以臣下的道义来责备皇子,想必他一定会入宫。”英宗果然接受了征召。‎ 五、阅读下面的文言文,完成第8小题。‎ 安守忠,字信臣,并州晋阳人。晋天福八年,以守忠为牙内指挥使,领绣州刺史。周显德四年春,改鞍辔库使。会淮南初下命守忠驰往宣谕时藩臣骄蹇遇朝使多简傲守忠抗以正礼无所辱命。未几,改卫州刺史。‎ 宋初,入为左卫将军。蜀平,太祖知远俗苦苛虐,南郑为走集之地,故特命守忠知兴元府以抚绥之。四年,改汉州刺史。时寇难甫平,使车旁午,公帑不足,守忠出私钱以给用。每遣使,太祖必戒之曰:“安守忠在蜀,能律己以正,汝行见之,当效其为人也。”开宝五年,知辽州。民有阴召并[注]寇谋内应者,事泄,守忠悉斩以徇。九年,命将征太原,守忠受诏与孙晏宣由辽州入,既而与路罗寨监押马继恩遇,乃相与会兵入贼境,燔寨四十余,获牛羊数千。‎ 雍熙二年,改知易州,徙夏州。每西戎犯边,战无不捷。淳化二年,徙知雄州。方与僚佐宴饮,有军校谋变,擐甲及阍,阍者仓卒入白。守忠言笑自若,徐顾坐客曰:“此辈酒狂尔,擒之可也。”人服其量焉。明年,加耀州观察使,兼判雄州。未几,召还,条陈边事,敷奏称旨,赐钱五百万。五年,又知沧州。至道初,移雄州。三年,复知沧州,拜感德军节度观察留后。徙宋州,兼制置营田使。威德兼著,吏民不忍其去。咸平三年,入觐,遣还未行,暴卒,年六十九,赠太尉。‎ 守忠谨悫淡薄,为治简静。太祖居藩日,素相厚善,及受禅后,每优任之,守忠处之益谦。从征太原,多与谋略,人罕知之者。所至藩郡,乐施予,丰宴犒,且喜与士大夫游从,故时论多与之。‎ ‎(选自《宋史·安守忠传》,有删改)‎ ‎[注] 并:指并州,现在的山西太原一带。‎ ‎8.下列对原文有关内容的概括和分析,不正确的一项是(  )‎ A.安守忠在蜀地任职时,为官清廉。由于战争刚刚平息,满目疮痍,民生凋敝,库藏的钱财不足,他就用自己的钱来贴补。‎ B.安守忠军事才能卓越,战功显赫。征伐太原,“燔寨四十余,获牛羊数千”,但因不是主帅,所以很少有人知道他参与了谋略。‎ C.安守忠改任雄州知州,处惊不变。面对军校的阴谋叛变,不仅有胆,而且有识,从容面对,在谈笑之间消除了祸端。‎ D.安守忠在太祖登基后,深受重用。他跟随太祖南征北战,因冷静诚实地处理政务,太祖不仅厚待,而且优先任用。‎ 答案 D 解析 这道题目是概括归纳文章内容。对似是而非处,要有借题解文的意识。题中D项由原文文末“太祖居藩日,素相厚善,及受禅后,每优任之,守忠处之益谦”可以推测,原因是“素相厚善”,故“因冷静诚实地处理政务”原因分析不当。‎ 参考译文:‎ 安守忠,字信臣,并州晋阳人。后晋天福八年,任命安守忠为牙内指挥使,兼任绣州刺史。后周显德四年春,改任为鞍辔库使。正值淮南刚攻下,命令安守忠急速前去宣布告谕。当时藩镇大臣傲慢不顺从,接待朝廷使者大多很傲慢,安守忠以正礼相待,不辱使命。不久,改为卫州刺史。‎ 宋朝初年,(安守忠)入朝为左卫将军。蜀地平定后,太祖知道边远地区苦于严苛残暴的统治,而南郑是交通要冲,所以特别任命安守忠为兴元府知府加以安抚。四年,改为汉州刺史。当时战争刚刚平息,朝廷使臣来往纷繁,官府库藏的金帛不足,安守忠拿出自己的钱来提供使用。每次派遣使臣,太祖必定告诫他们说:“安守忠在蜀地,能严以律己,你们此行见到他,应当效法他的为人。”开宝五年,任辽州知州。百姓中有人暗中召来并州的叛军谋划作内应,事情败露,安守忠把他们全部斩首示众。九年,朝廷任命将领征伐太原,安守忠接受诏令与孙晏宣由辽州进入,不久与路罗寨监押马继恩相遇,于是会合兵力深入敌境,烧毁四十多寨,缴获牛羊数千。‎ 雍熙二年,改任易州知州,调任夏州知州。每次西戎侵犯边境,(安守忠)战无不胜。淳化二年,转为雄州知州。正与僚属宴饮时,有军校阴谋叛变,披甲到了门口,守门人仓促进去禀报。安守忠谈笑自如,慢慢回头看着座上宾客说:“这人饮酒发疯罢了,捉住他就可以了。”‎ 人们佩服他的气量。第二年,加官耀州观察使,兼任知州雄州。不久,召回,分条陈述边境之事,符合皇帝旨意,赐给钱五百万。五年,又为沧州知州。至道初年,转为雄州知州。三年,再为沧州知州,授职为感德军节度观察留后。转任宋州,兼任制置营田使。安守忠的声威德行都很显著,官吏百姓不忍他离去。咸平三年,入朝晋见皇帝,派遣回去还未出行,突然死去,享年六十九岁,赠官太尉。‎ 安守忠谨慎忠厚淡泊名利,处理政务简要平静。太祖为节度使时,对他很好,太祖登上皇位后,常优先任用他,安守忠由此更加谦逊。跟随征伐太原时,常常参与谋略,但人们很少知道这些事。每到一个藩镇,(安守忠)都乐于施予,会安排丰盛的宴席犒劳大家,并且喜欢与士大夫交往,所以当时的舆论大多赞许他。‎ 六、阅读下面的文言文,完成9~10题。‎ 屈突通,其先盖昌黎徒何人,后家长安。仕隋为虎贲郎将。文帝命覆陇西牧簿,得隐马二万匹,帝怒,收太仆卿慕容悉达、监牧官吏千五百人,将悉殊死。通曰:“人命至重,死不复生。陛下以至仁育四海,岂容以畜产一日而戮千五百士?”帝叱之,通进顿首曰:“臣愿身就戮,以延众死。”帝寤,曰:“朕不明,乃至是。今当免悉达等,旌尔善言。”遂皆以减论。擢左武卫将军。莅官劲正,有犯法者,虽亲无所回纵。‎ 炀帝即位,遣持诏召汉王杨谅。先是,文帝与谅约,若玺书召,验视敕字加点,又与玉麟符合,则就道。及是,书无验,谅觉变,诘通,通占对无屈,竟得归长安。大业中,与宇文述共破杨玄感,以功迁左骁卫大将军。秦、陇盗起,授关内讨捕大使。安定人刘迦论反,众十余万据雕阴。通发关中兵击之,次安定,初不与战,军中意其怯。通阳言旋师,而潜入上郡。贼未之觉,引而南,去通七十里舍,分兵徇地。通候其无备,夜简精甲袭破之,斩迦论并首级万余,筑京观于上郡南山,虏老弱数万口。后隋政益乱,盗贼多,士无斗志,诸将多覆。通每向必持重,虽不大克,亦不败负。‎ 高祖起,代王遣通守河东,战久不下,高祖留兵围之。通势蹙,或说之降,曰:“吾蒙国厚恩,事二主,安可逃难?独有死报尔!”每自摩其颈曰:“要当为国家受人一刀!”其训勉士卒必流涕,故力虽穷,而人尚为之感奋。帝遣其家僮往召,通趋斩之。俄闻京师平,家尽没,乃留桑显和保潼关,率兵将如洛。刘文静遣窦琮、段志玄精骑追及于稠桑,通结阵拒之。琮纵其子寿往谕使降,通大呼曰:“昔与汝父子,今则仇也!”命左右射之,显和呼其众曰:“京师陷,诸君皆家关西,何为复东?”众皆舍兵。通知不免,遂下马东南向,再拜号哭曰:“臣力屈兵败,不负陛下。”遂被禽,送长安。帝劳曰:“何相见晚邪?”泣曰:“通不能尽人臣之节,故至此,为本朝羞。”帝曰:“忠臣也!”释之,授兵部尚书、蒋国公,为秦王行军元帅长史。‎ ‎9.下列各组句子中,分别表明屈突通“善战”和“忠诚”的一组是(  )‎ A.①与宇文述共破杨率 ②帝遣其家僮往召,通趋斩之 B.①通发关中兵击之 ②要当为国家受人一刀 C.①通侯其无备,夜简精甲袭破之 ②有犯法者,虽亲无所回纵 D.①通每向必持重,虽不大克,亦不败负 ②释之,授兵部尚书、蒋国公 答案 A 解析 此题人物是“屈突通”,品质是“善战”和“忠诚”,然后分析列出的各项,B项①是奉命发兵,并非显其“善战”;C项②言其为官刚正,非见其“忠心”;D项②言其被唐高祖授予官职,并不能表明其“忠心”,排除这三项,自然得到答案。‎ ‎10.下列对原文有关内容的分析和概括,不正确的一项是(  )‎ A.屈突通祖籍昌黎徒何,后迁居长安,曾出仕隋朝为官。屈突通十分爱惜士人,曾冒死劝谏隋文帝,反对因牲畜财产而滥杀士人,深得文帝赏识。‎ B.屈突通遇事从容,机智灵活。隋炀帝时,他奉诏前去召见汉王杨谅,在杨谅觉得事情有变,心生狐疑时,屈突通沉着应对,最终圆满完成任务。‎ C.屈突通骁勇善战,深谙兵法,曾与他人一起击败杨玄感,用奇计平定刘迦论的叛军,即使在军无斗志的情况下,仍然能大获全胜,一生从无败绩。‎ D.屈突通崇尚气节,忠心耿耿,先后侍奉过隋代二帝,即使在隋朝政事混乱时也从无叛心,后兵败被俘,虽未殉节而死,但其忠心依然为唐高祖所称道。‎ 答案 C 解析 ‎ 本题解答要求在读懂原文的基础上,能够对文章的内容进行分析、归纳,准确把握文中的主要信息。如文中为“虽不大克,亦不败负”,题干中为“仍然能大获全胜,一生从无败绩”,从而可以分析判断,C项,“仍然能大获全胜,一生从无败绩”分析有误。‎ 参考译文:‎ 屈突通,他的祖先大概是昌黎徒何人,后迁居长安。出仕隋朝虎贲郎将。隋文帝命人复核陇西牧的文簿,查出隐匿的两万匹马,文帝大怒,收捕太仆卿慕容悉达、监牧官吏共一千五百人,准备全部斩首。屈突通对(文帝)说:“人命至关重大,人死不能复生。陛下用最仁慈的政令抚育天下,难道容许因牲畜财产而在一天之内杀戮一千五百人?”文帝叱责他,屈突通上前叩头说:“我情愿自身受杀戮,来免除众人的死罪。”文帝醒悟了,说:“我不明此理,竟至于此。现在应当免除慕荣悉达等人的死罪,表彰你的好建议。”于是全部按照减刑论处。后擢升为左武卫将军。上任后为官刚正,有人犯法时,即使是亲朋也不放纵。‎ 隋炀帝即位后,派他带着诏书去召见汉王杨谅。在这之前,文帝与杨谅约定,如果有玺书召,验看敕字加点,还要与玉麟符相合,那么就能上路。待到这时,验看诏书没有点,杨谅觉得事情有变,责问屈突通,屈突通随口对答,毫无破绽,最终得以返回长安。大业年间,与宇文述一起击败杨玄感,因功升迁为左骁大将军。秦、陇等地群盗兴起,(屈突通)被授任为关内讨捕大使。安定人刘迦论造反,拥兵十余万盘踞在雕阴。屈突通调发关中军队进击敌军,驻扎安定,起初不与敌军交战,军中人都猜想他胆怯。屈突通假意说要撤军,(其实)却悄悄进入上郡。贼军未能发觉,率军南下,在距离屈突通七十里的地方驻军,分兵侵占土地。屈突通乘敌军毫无防备,夜晚选择精锐士兵偷袭打败敌军,斩杀刘迦论连同其他贼人共万余首级,埋在上郡南山封土做成高冢,俘虏老弱数万人。后来隋朝政治更加混乱,盗贼越来越多,(朝廷)军士毫无斗志,诸将大多覆没。屈突通所到之处非常谨慎,虽无大胜,也未打过败仗。‎ 高祖起兵,代王派屈突通守河东,交战后久攻不下,高祖于是留下兵分力包围河东。屈突通形势危急,有人游说他投降,他说:“我蒙受国家厚恩,侍奉二主,怎能逃避国难?只能以死报国!”时常自己抚摸着脖颈说:“应当为国家受人一刀!”他训导勉励士兵时必定流下热泪,所以士兵们力气虽然用尽,但人们还是被他的精神激励。皇帝派他的家僮前往招降,屈突通催令斩杀家僮。不久听说京城平定,家人全都沦陷,于是留下桑显和保卫潼关,自己率兵将前往洛阳。刘文静派遣窦琮、段志玄率精锐骑兵追至稠桑,屈突通列阵抵抗他们。窦琮让他的儿子屈突寿前往劝降,屈突通大喊到:“往昔与你是父子,现在就是仇敌了!”命令身边人射杀屈突寿。桑显和对他的军兵大声喊道:“京师陷落,各位都家住关西,为何还要向东去?”众人全都丢掉兵器。屈突通知道无法解脱,于是下马面向东南,连连叩拜哭喊道:“我力尽兵败,没有辜负陛下。”于是被俘,押送到长安。皇帝慰劳他说:“为何相见得这么晚呢?”屈突通哭着说:“我未能尽人臣的忠节,所以到了这个地步,让本朝蒙羞。”皇帝说:“这是忠臣哪!”释放了他,授职为兵部尚书、蒋国公,担任秦王行军元帅长史。‎ 考点十一 理解文言句式并翻译文中句子 考点名片 考点内容 所谓“理解”,是说能正确领会文章的内容,正确理解文意、文句;所谓“翻译”,是说能用现代汉语的词汇和语法来准确翻译所提供的文言语句,用现代汉语把古人文章的内容准确表达出来。‎ 考查形式 ‎①从文言文段中划出重要句子,以理解实词和虚词为基础,对词类活用、一词多义、古今异议、通假现象,直至对文言固定短语、文言修辞格、文言特殊句式、文言文一些特殊表达现象,都有准确把握,特别要注意译出重点实、虚词和有活用的词语,灵活地翻译指定的句子。②主观题,两个长句子。③每句5分,共10分。‎ 趋势分析 ‎①可能选取传记文段之外的段落单独考查翻译。②分值可能有增加。‎ 题组1 文言句式 ‎1.下列各组句子中,句式不相同的一组是(  )‎ A.夺项王天下者,必沛公也 如今人方为刀俎,我为鱼肉 B.故今之墓中全乎为五人也 刘备天下枭雄 C.《诗》三百篇,大底圣贤发愤之所为作也 妪,先大母婢也 D.屈平疾王听之不聪也 城北徐公,齐国之美丽者也 答案 D 解析 “屈平”句为陈述句,其余为判断句。‎ ‎2.下列句子中与另外三句的句式特点不相同的一句是(  )‎ A.予本非文人画士 B.此言士节不可不勉励也 C.六国破灭,非兵不利,战不善 D.夫病者所见非鬼也 答案 B 解析 其余为否定性判断句。‎ ‎3.下列句中没有宾语前置现象的一句是(  )‎ A.甚矣,汝之不惠 B.古之人不余欺也 C.天大寒,砚冰坚,手指不可屈伸,弗之怠 D.臣实不才,又谁敢怨 答案 A 解析 A项,主谓倒装。‎ ‎4.下列各组句子中,句式不相同的一组是(  )‎ A.(亮)每自比于管仲、乐毅,时人莫之许也//七十者衣帛食肉,黎民不饥不寒,然而不王者,未之有也 B.忌不自信,而复问其妾//与言皇上无权,君未之信也 C.沛公安在//大王来何操 D.微斯人,吾谁与归//大阉之乱,缙绅而能不易其志者,四海之大,有几人欤 答案 D 解析 D项,“大阉之乱”和“四海之大”两句为定语后置,其余均为宾语前置句。‎ ‎5.下列各组句子中句式相同的一组是(  )‎ A.句读之不知,惑之不解//岂不榖是为?先君之好是继 B.夫晋,何厌之有//子何恃而往 C.安在公子能急人之困也//王尝语庄子以好乐,有诸 D.子非鱼,安知我不知鱼之乐//无情郎安在 答案 A 解析 A项,都是宾语前置,且以“之”“是”为标志;B项,“夫”句以“之”为标志的宾语前置句,“子”句疑问代词作宾语前置;C项,“安在”,即“在安”,疑问代词作宾语前置,“王”句无前置宾语;D项,“子”句无前置宾语,“无”句疑问代词作宾语前置。‎ ‎6.下列句子中句式不同于其他三句的一句是(  )‎ A.且相如素贱人,吾羞,不忍为之下 B.秦城恐不可得,徒见欺 C.信而见疑,忠而被谤 D.兵挫地削,亡其六郡,身客死于秦 答案 A 解析 A项,判断句,其余为被动句。‎ ‎7.下列句子分类正确的一项是(  )‎ ‎①卿欲何言 ②夺项王天下者,必沛公也 ③吾属今为之虏矣 ④不然,籍何以至此 ⑤或脱身以逃,不能容于远近 ⑥此则岳阳楼之大观也 ⑦子何恃而往 ⑧傅说举于版筑之间 A.①④/②③⑥/⑤⑦⑧ B.①④⑦/②③/⑤⑥⑧‎ C.①④②③⑥/⑤⑦⑧ D.①④⑦/②⑥/③⑤⑧‎ 答案 D 解析 D项,①④⑦疑问句中疑问代词作宾语前置;②⑥判断句;③⑤⑧被动句,“举”,被推举。‎ ‎8.下列句子中没有成分省略现象的一项是(  )‎ A.老臣今者殊不欲食,乃自强步,日三四里 B.永州之野产异蛇,黑质而白章,触草木,尽死 C.燕赵之君,始有远略,能守其土,义不赂秦 D.今以钟磬置水中,虽大风浪不能鸣也 答案 C 解析 A项,“日三四里”省略谓语动词“行”;B项,三处省略主语;D项,省略“鸣”的宾语“之”,代指钟磬。‎ ‎9.下列句中“为”字用法不表判断的一项是(  )‎ A.是为何谷 B.何为以公名 C.以臣为愚 D.故今之墓中全乎为五人也 答案 B 解析 B项,介词,“为何”,为什么;其余表判断。‎ ‎10.下列句中省略宾语的一句是(  )‎ A.见一老公而问之曰 B.遂持驹去 C.齐桓公出猎 D.明日朝,以告管仲 答案 D 解析 省略介词宾语:以告,以之告。‎ ‎11.下列各组句子全是否定句中宾语前置的一组是(  )‎ A.乃入见,曰:“何以战?”//将子无怒,秋以为期 B.故凡为愚者,莫我若也//我无尔诈,尔无我虞 C.知我者谓我心忧,不知我者谓我何求//权知其意,执肃手曰:“卿欲何言?”‎ D.楚战士无不一以当十//当臣之临河持竿,心无杂虑,唯鱼之念 答案 B 解析 B项,应为“莫若我也”和“我无诈尔,尔无虞我”。A项,均为宾语前置,但都不是否定句;C项,均为宾语前置,但都不是否定句;D项,第二句是宾语前置,但不是否定句。‎ ‎12.下列句子中“见”不表被动的一句是(  )‎ A.欲与秦,秦城恐不可得,徒见欺 B.食不饱,力不足,才美不外见 C.众人皆醉而我独醒,是以见放 D.人皆以见侮为辱,故斗也 答案 B 解析 B项,“外见”之“见”,通“现”,显露之意;A、C、D三项的“见”都表被动。‎ ‎13.下列省略句补充的成分正确的一项是(  )‎ A.良曰:“(我)闻大王有意督过之,脱身独去,已至军矣。”‎ B.今少卿乃教(我)以推贤进士,无乃与仆私心剌谬乎 C.军中无以为乐,请以剑舞(之)‎ D.死马且买之(于)五百金,况生马乎 答案 B 解析 A项应为:沛公;C项应是“以剑舞为乐”;D项应是“以”。‎ ‎14.下面句子没有成分省略现象的一句是(  )‎ A.即墨大夫出与战,败死 B.燕王以为然,使骑劫代乐毅 C.老臣窃以为媪之爱燕后,贤于长安君 D.城中相与推田单,立以为将军 答案 C 解析 A项,“出与(之)战”;B项,“以(之)为然”;D项,“城中(齐人)相与”“立(之)以(之)为”。‎ ‎15.下列句子中都省略了介词“于”的一组是(  )‎ A.臣尝从大王与燕王会境上//今臣至,大王见臣列观 B.沛公欲王关中//填然鼓之,兵刃既接,弃甲曳兵而走 C.每念斯耻,汗未尝不发背沾衣也//上称帝喾,下道齐桓,中述汤武 D.遂与秦王会渑池//寡人窃闻赵王好音,请奏瑟 答案 A 解析 A项,“会(于)境上”∥“见臣(于)列观”;B项,“欲王(于)关中”;C项,“发(于)背沾(于)衣”;D项,“会(于)渑池”。‎ 题组2 综合翻译 ‎16.阅读下面的文段,将画横线的句子翻译成现代汉语。‎ 蔺相如之完璧,人皆称之,予未敢以为信也。‎ 夫秦以十五城之空名,诈赵而胁其璧,是时言取璧者,情也,非欲以窥赵也。赵得其情则弗予,不得其情则予;得其情而畏之则予,得其情而弗畏之则弗予。此两言决耳,奈之何既畏而复挑其怒也?……‎ 吾故曰:蔺相如之获全于璧,天也。若其劲渑池,柔廉颇,则愈出而愈妙于用,所以能完赵者,天固曲全之哉!‎ ‎(选自王世贞《蔺相如完璧归赵论》,有删改)‎ ‎(1)蔺相如之完璧,人皆称之,予未敢以为信也。‎ 译文:_________________________________________________‎ ‎(2)此两言决耳,奈之何既畏而复挑其怒也?‎ 译文:_________________________________________________‎ 答案 (1)蔺相如完璧归赵,人人都称赞他,但我不敢相信这件事。‎ ‎(2)这只要两句话就能解决了,为何既惧怕它而又去激怒它呢?‎ 参考译文:‎ 蔺相如完璧归赵,人人都称赞他,但我不敢相信这件事。‎ 秦国用十五座城池的空名,欺骗赵国而勒索和氏璧,这时说要得到和氏璧,是实情,不是借此来窥视赵国。赵国知此实情就不给,不知此实情就给;知此实情而惧怕秦国就给,知此实情而不惧怕秦国就不给。这只要两句话就能解决了,为何既惧怕它而又去激怒它呢?……‎ 因此我说,蔺相如之所以能保全和氏璧,那是天意。至于他在渑池与秦国的强硬较量,以友好使廉颇悔悟,就越来越显得高妙了,而他之所以能完璧归赵,的确是上天偏袒他啊!‎ ‎17.阅读下面的文段,将画横线的句子翻译成现代汉语。‎ 王东亭①为桓宣武主簿,既承藉,有美誉,公②甚欲其人地为一府之望。初,见谢失仪,而神色自若。坐上宾客即相贬笑,公曰:“不然。观其情貌,必自不凡,吾当试之。”后因月朝阁下伏,公于内走马,直出突之。左右皆宕仆,而王不动。名价于是大重,咸云“是公辅器也”。‎ ‎(选自刘义庆《世说新语》,有删改)‎ ‎[注] ①王东亭:即王珣,是东晋初年大臣王导的孙子,曾因功被封为东亭侯。②公:即桓宣武,桓温。‎ ‎(1)观其情貌,必自不凡,吾当试之。‎ 译文:_________________________________________________‎ ‎(2)名价于是大重,咸云“是公辅器也”。‎ 译文:_________________________________________________‎ 答案 (1)看他的神情态度,一定不平常,我要试试他。‎ ‎(2)于是王珣的名声大为提高,大家都说“这是做辅弼大臣的人才呀”。‎ 参考译文:‎ 东亭侯王珣担任桓温的主簿,受到祖辈的福荫,名声又很好,桓温很希望他在品学门第上能成为全府所敬仰的榜样。当初,(王珣)回答桓温问话时有失礼之处,可是他神色自若。在座的宾客立刻贬低并且嘲笑他,桓温说:“不是这样的。看他的神情态度,一定不平常,我要试试他。”后来在初一僚属进见的时候,桓温从后院骑着马直冲出来。手下的人都被吓得躲闪跌倒,王珣却稳坐不动。于是王珣的名声大为提高,大家都说“这是做辅弼大臣的人才呀”。‎ ‎18.阅读下面的文段,将画横线的句子翻译成现代汉语。‎ 何景明与梦阳并有国士风。两人为诗文,初相得甚欢,名成之后,互相诋。说者谓景明之才本逊梦阳,而其诗秀逸稳称,视梦阳反为过之。其持论,谓:“诗溺于陶,谢力振之,古诗之法亡于谢。文靡于隋,韩力振之,古文之法亡于韩。”其意盖诋梦阳,非笃论也。‎ ‎(选自《明史》,有删改)‎ ‎(1)两人为诗文,初相得甚欢,名成之后,互相诋。‎ 译文:_________________________________________________‎ ‎(2)其意盖诋梦阳,非笃论也。‎ 译文:_________________________________________________‎ 答案 (1)两人写诗文,刚开始相处得很好,成名之后,相互诋毁。‎ ‎(2)这个意思大概是在毁谤李梦阳,不是确切的观点。‎ 参考译文:‎ 何景明与李梦阳同有国士风度。两人写诗文,刚开始相处得很好,成名之后,相互诋毁。议论的人说何景明的才气本来不如李梦阳,但他的诗秀逸妥帖,反而超过李梦阳的诗。何景明把自己的观点发表出来,说:“古诗在陶渊明时陷入困境,谢灵运奋力振兴它,古诗的写作方法在谢灵运时就消亡了。文章在隋朝时萎靡不振,韩愈奋力振兴它,古文的写作方法在韩愈时就消亡了。”这个意思大概是在毁谤李梦阳,不是确切的观点。‎ ‎19.阅读下面的文段,将画横线的句子翻译成现代汉语。‎ 闾丘卬曰:“不然。夫尺有所短,寸有所长。骅骝马录骥,天下之俊马也。使之与狸鼬试于釜灶之间,其疾未必能过狸鼬也。‎ 黄鹄白鹤,一举千里,使之与燕蝠翼试之堂庑之下,庐室之间,其便未必能过燕蝠翼也。辟闾、巨阙,天下之利剑也,击石不缺,刺石不锉,使之与管稿决目出眯,其便未必能过管稿也。由此观之,华发堕颠,与卬何以异哉?”‎ ‎(选自刘向《新序》,有删改)‎ ‎(1)使之与狸鼬试于釜灶之间,其疾未必能过狸鼬也。‎ 译文:_________________________________________________‎ ‎(2)由此观之,华发堕颠,与卬何以异哉?‎ 译文:_________________________________________________‎ 答案 (1)(但是)让它们与狸鼬一起在锅灶之间赛跑,它们的速度未必能超过狸鼬。‎ ‎(2)由此看来,花白头发或秃顶的老人,和我相比拿什么来区别呢?‎ 参考译文:‎ 闾丘卬说:“不是这样的。尺有所短,寸有所长。骅骝马录骥,都是天下著名的骏马。(但是)让它们与狸鼬一起在锅灶之间赛跑,它们的速度未必能超过狸鼬。黄鹄白鹤,一飞千里,(但是)让它们与燕子、蝙蝠一起在堂屋走廊下、住宅房间里比赛飞翔,它们的轻便未必能超过燕子和蝙蝠。辟闾、巨阙,是天下最锐利的宝剑,用它们敲击石头,剑刃不会缺损,用它们来刺石头,剑身也不会折断,(但是)让它们与萱草、稻秆一起去取钻进眼里的灰沙,它们的便利未必能超过萱草和稻秆。由此看来,花白头发或秃顶的老人,和我相比拿什么来区别呢?”‎ ‎20.阅读下面的文段,将画横线的句子翻译成现代汉语。‎ 祥性至孝。早丧亲,继母朱氏不慈,数谮之,由是失爱于父,每使扫除牛下。祥愈恭谨。父母有疾,衣不解带,汤药必亲尝。母常欲生鱼,时天寒冰冻,祥解衣将剖冰求之,冰忽自解,双鲤跃出,持之而归。母又思黄雀炙,复有黄雀数十飞入其幕,复以供母。乡里惊叹,以为孝感所致焉。有丹柰结实,母命守之,每风雨,祥辄抱树而泣。其笃孝纯至如此。‎ ‎(选自《晋书》,有删改)‎ ‎(1)父母有疾,衣不解带,汤药必亲尝。‎ 译文:_________________________________________________‎ ‎(2)乡里惊叹,以为孝感所致焉。‎ 译文:_________________________________________________‎ 答案 (1)父母亲有病(的时候),他睡觉都不脱衣服,必定亲自尝试汤药。‎ ‎(2)乡里人都惊叹,认为这是他的孝心感动天地所致。‎ 参考译文:‎ 王祥生性非常孝顺。他早年丧母,继母朱氏对他不好,多次说他坏话,因此父亲也不喜欢他,常常派他清扫牛粪。王祥更加恭敬谨慎地做这些事情。父母亲有病(的时候),他睡觉都不脱衣服,必定亲自尝试汤药。有一次继母一心想吃鲜鱼,当时天寒地冻,王祥解下衣裳正要剖开冰面来抓鱼,冰面忽然自己解冻,一对鲤鱼跃出,他拿着鲤鱼回了家。继母又想吃烤黄雀,于是又有数十只黄雀飞入他的帐幕,他又拿来进献继母。乡里人都惊叹,认为这是他的孝心感动天地所致。家里的丹柰结了果实,继母命他看守,每当风雨来临,王祥就抱着树哭泣。他的孝心竟到了如此地步。‎ ‎21.阅读下面的语段,把文中画横线的句子翻译成现代汉语。‎ 罗洪先,字达夫,吉水人。父循,进士。历兵部武选郎中。会考选武职,有指挥二十余人素出刘瑾门,循罢其管事。瑾怒骂尚书王敞,敞惧,归部趣易奏。循故迟之,数日瑾败,敞乃谢循。循历知镇江、淮安二府,徐州兵备副使,咸有声。‎ 洪先归,益寻求守仁学。甘淡泊,炼寒暑,跃马挽强,考图观史,自天文、地志、礼乐、典章、河渠、边塞、战阵攻守,下逮阴阳、算数,靡不精究。至人才、吏事、国计、民情,悉加意谘访。曰:“苟当其任,皆吾事也。”邑田赋多宿弊,请所司均之,所司即以属。洪先精心体察,弊顿除。岁饥,移书郡邑,得粟数十石,率友人躬振给。流寇入吉安,主者失措。为画策战守,寇引去。素与顺之友善。顺之应召,欲挽之出,严嵩以同乡故,擢假边才起用,皆力辞。‎ 洪先虽宗良知学,然未尝及守仁门,恒举《易大传》“寂然不动”、周子“无欲故静”之旨以告学人。又曰:“儒者学在经世,而以无欲为本。惟无欲,然后出而经世,识精而力钜。”‎ 时王畿谓良知自然,不假纤毫力。洪先非之曰:“世岂有现成良知者耶?”虽与畿交好,而持论始终不合。山中有石洞,旧为虎穴,葺茅居之,命曰石莲。谢客,默坐一榻,三年不出户。‎ ‎(节选自《明史·罗洪先传》)‎ ‎(1)敞惧,归部趣易奏。循故迟之,数日瑾败,敞乃谢循。‎ 译文:_________________________________________________‎ ‎(2)顺之应召,欲挽之出,严嵩以同乡故,擢假边才起用,皆力辞。‎ 译文:_________________________________________________‎ 答案 (1)王敞害怕,回到兵部催促改变奏折。罗循故意拖延,几天后刘瑾倒台,王敞于是向罗循道歉。‎ ‎(2)唐顺之应召入朝,想引他出山,严嵩因是同乡的缘故,打算假托治理边疆的人才起用他,他都极力推辞。‎ 参考译文:‎ 罗洪先,字达夫,吉水人。父亲罗循,是进士。历任兵部武选郎中。恰逢考核选拔武官,有二十多个指挥平素出自刘瑾门下,罗循罢除其管事。刘瑾愤怒地骂尚书王敞,王敞害怕,回到兵部催促改变奏折。罗循故意拖延,几天后刘瑾倒台,王敞于是向罗循道歉。罗循历任镇江、淮安两府的知府和徐州兵备副使,都有治政的名声。‎ 罗洪先回到家中,更加寻求王守仁的学问。甘于淡泊,锻炼于寒冬酷暑,策马腾跃,拉引强弓,查考地图翻阅历史,从天文、地理、礼乐、典章、河渠、边塞、打仗的阵法和攻守方式,下至阴阳、算数,无不精心研究。至于人才、吏治之事、国家大计、百姓的情况,都用心咨询访问。(他)说:“如果身在其位,都是我的事。”县里的田赋有很多积久的弊病,请求有关部门均衡处理,有关部门就交托他来处理。罗洪先细心考察,积弊马上除去。收成不好,他转交书信到郡县,得到粮食几十石,都是友人亲自拿出救济的。流窜的盗贼进入吉安,主事官惊慌失措。(罗洪先)为他出谋划策迎战守卫,盗贼退兵离去。他向来与唐顺之友好。唐顺之应召入朝,想引他出山,严嵩因是同乡的缘故,打算假托治理边疆的人才起用他,他都极力推辞。‎ 罗洪先虽然宗奉良知学说,然而不曾拜师在王守仁门下,经常举《易大传》“寂静不动”、周子“没有欲望,所以静”的意旨来告诫学习的人。又说:“儒生学习在于治理国家,而以没有欲望为根本。只有无欲,然后出来治理国家,见解精辟而力量巨大。”当时王畿认为良知讲求自然,不借助丝毫外力。罗洪先批评说:“世上哪有现成的良知呢?”虽然与王畿交情好,而所持见解始终不合。山中有石洞,以前是老虎的巢穴,他盖上茅草居住在里面,命名为石莲。谢绝客人,默坐在一木榻上,三年不出门。‎ ‎22.阅读下面的文言文,把文中画横线的句子翻译成现代汉语。‎ 刘隆,字元伯,南阳安众侯宗室也。‎ 十一年,守南郡太守,岁余,上将军印绶。十三年,增邑,更封竟陵侯。是时,天下垦田多不以实,又户口年纪互有增减。十五年,诏下州郡检核其事,而刺史太守多不平均,或优饶豪右,侵刻羸弱,百姓嗟怨,遮道号呼。时诸郡各遣使奏事,帝见陈留吏牍上有书,视之,云“颍川、弘农可问,河南、南阳不可问”。帝诘吏由趣,吏不肯服,抵言于长寿街上得之。帝怒。时显宗为东海公,年十二,在幄后言曰:“吏受郡敕,当欲以垦田相方耳。”帝曰:“即如此,何故言河南、南阳不可问?”对曰:“河南帝城,多近臣,南阳帝乡,多近亲,田宅逾制,不可为准。”帝令虎贲将诘问吏,吏乃实首服,如显宗对。于是遣谒者考实,具知奸状。明年,隆坐征下狱,其畴辈十余人皆死。帝以隆功臣,特免为庶人。‎ 隆奉法自守,视事八岁,上将军印绶,罢,赐养牛,以列侯奉朝请。三十年,定封慎侯。中元二年,卒,谥曰靖侯。‎ ‎(选自《后汉书·列传第十二》,有删改)‎ ‎(1)明年,隆坐征下狱,其畴辈十余人皆死。帝以隆功臣,特免为庶人。‎ 译文:_________________________________________________‎ ‎___________________________________________________________‎ ‎(2)隆奉法自守,视事八岁,上将军印绶,罢,赐养牛,以列侯奉朝请。‎ 译文:_________________________________________________‎ ‎___________________________________________________________‎ 答案 ‎ ‎(1)第二年,刘隆因犯罪被召回关进监狱,他的同辈十多人都被处死。皇上因为刘隆是功臣,只特地把他免官,成为平民。‎ ‎(2)刘隆奉公守法,任职八年,交上将军官印,罢官,皇上赐给他自己饲养的牛,让他以列侯的身份入朝请见。‎ 参考译文:‎ 刘隆,字元伯,是南阳安众侯的宗室后裔。‎ ‎(建武)十一年,代理南郡太守,一年多,交上将军印。十三年,增加封邑,改封竟陵侯。这时,天下开垦的土地多不以实数上报,另外上报户口年纪各有增减。十五年,有诏令各州郡考查核实这件事,但刺史太守多不一样,有些厚待豪门大族,欺凌贫弱,百姓哀怨,拦路呼号。当时,各郡各自派使者上朝禀报政事,皇上看到陈留官吏文书上的字,看那些字,写的是“颍川、弘农可以查问,河南、南阳不可查问”。皇上追问小官这字的来历,小官不肯承认,抵赖说在长寿街上得到的。皇上愤怒。当时,显宗做东海公,十二岁,在帷幕后面说:“小官受郡守命令,应该是想拿开垦的土地来举个例子。”皇上说:“假如是这样,为什么说河南、南阳不可以追究呢?”(东海公)回答说:“河南是皇城,有很多皇上身边的臣子,南阳是皇帝的故乡,有很多皇家亲戚,田地房宅超过规定,不能按标准。”皇上命令虎贲将盘问小官,小官这才从实招认,正如显宗(东海公)所回答的。于是派谒者考查核实,全部获知欺骗作假的情况。第二年,刘隆因犯罪被召回关进监狱,他的同辈十多人都被处死。皇上因为刘隆是功臣,只特地把他免官,成为平民。‎ 刘隆奉公守法,任职八年,交上将军官印,罢官,皇上赐给他自己饲养的牛,让他以列侯的身份入朝请见。(建武)三十年,确定封为慎侯。中元二年,去世,谥号靖侯。‎ ‎23.阅读下面的文言文,把文中画横线的句子翻译成现代汉语。‎ 崔祖思,字敬元,清河东武城人。祖思少有志气,好读书。年十八,为都昌令,随青州刺史垣护之入尧庙,庙有苏侯神偶坐。护之曰:“唐尧圣人而与苏侯神共坐,今欲正之何如?”祖思曰:“使君若清荡此坐,则是尧庙重去四凶。”由是诸杂神并除。‎ 齐高帝在淮阴,祖思闻风自结,为上辅国主簿,甚见亲待,参豫谋议。宋朝初议封高帝为梁公,祖思启高帝曰:“谶云‘金刀利刃齐刈之[注]’。今宜称齐,实应天命。”从之。自相国从事中郎迁齐国内史。‎ ‎…………‎ 后为青、冀二州刺史,在政清勤,而谦卑下士,言议未尝及时事,上更以敬重之。未几卒,上深加叹惜。‎ ‎(选自《南史·列传第三十七》,有删改)‎ ‎[注] 金刀利刃齐刈之:意思是齐能够替代宋。“金刀利刃”指宋国君王的姓“刘”。‎ ‎(1)宋朝初议封高帝为梁公,祖思启高帝曰:“谶云‘金刀利刃齐刈之’。今宜称齐,实应天命。”‎ 译文:_________________________________________________‎ ‎___________________________________________________________‎ ‎(2)后为青、冀二州刺史,在政清勤,而谦卑下士,言议未尝及时事,上更以敬重之。‎ 译文:_________________________________________________‎ ‎___________________________________________________________‎ 答案 (1)宋朝当初讨论封齐高帝为梁公,崔祖思启奏齐高帝说:“谶书上说‘金刀利刃齐刈之’。现在适合称齐,实际应验天命。”‎ ‎(2)后来,(崔祖思)任青州、冀州二州刺史,为政清廉勤勉,而且礼贤下士,言谈议论从未涉及时事,皇上因此更加敬重他。‎ 参考译文:‎ 崔祖思,字敬元,清河东武城人。崔祖思年少时就有志气,喜欢读书。十八岁那年,担任都昌令,跟随青州刺史垣护之一起到尧庙祭神,庙里有坐姿的苏侯的雕像。垣护之说:“唐尧是圣人,却与苏侯神坐在一起,现在改正如何?”崔祖思说:“使君如果是清除这座像,就是为尧庙重新除去四凶。”因此,诸座杂神一起除去。‎ 齐高帝在淮阴,崔祖思闻风而动,主动投靠,担任上辅国主簿,很被亲信、优待,参与谋议。宋朝当初讨论封齐高帝为梁公,崔祖思启奏齐高帝说:“谶书上说‘金刀利刃齐刈之’。现在适合称齐,实际应验天命。”(齐高帝)听从了他的建议。他从相国从事中郎升为齐国内史。‎ ‎…………‎ 后来,崔祖思任青州、冀州二州刺史,为政清廉勤勉,而且礼贤下士,言谈议论从未涉及时事,皇上因此更加敬重他。不久后去世,皇上深深叹息。‎ ‎24.阅读下面的文言文,把文中画横线的句子翻译成现代汉语。‎ 胡松年,字茂老,海州怀仁人。幼孤贫,母粥机织,资给使学,读书过目不忘,尤邃于《易》。政和二年,上舍释褐,补潍州教授。八年,赐对便殿,徽宗伟其状貌,改校书郎兼资善堂赞读。为殿试参详官,以沈晦第一,徽宗大悦曰:“朕久闻晦名,今乃得之。”‎ 岳飞收复襄、汉,令松年筹度守御事。松年奏:“乞飞班师,徐窥刘豫意向,若豫置不问,其情叵测,当饬将士谨疆场可也。”‎ 松年平生不喜蓄财,每除官例赐金帛,以军兴费广,一无所陈请,或劝其白于朝,曰:“弗请则已,白之是沽名也。”喜宾客,奉入不足以供费,或请节用为子孙计。松年曰:“贤而多财,则损其志,况俸廪,主上所以养老臣也。”自持囊至执政,所举自代,皆一时闻人,所荐一以至公,权势莫能夺。‎ 方秦桧秉政,天下识与不识,率以疑忌置之死地,故士大夫无不曲意阿附为自安计。松年独鄙之,至死不通一书,世以此高之。‎ ‎(1)松年奏:“乞飞班师,徐窥刘豫意向,若豫置不问,其情叵测,当饬将士谨疆场可也。”‎ 译文:_________________________________________________‎ ‎___________________________________________________________‎ ‎(2)方秦桧秉政,天下识与不识,率以疑忌置之死地,故士大夫无不曲意阿附为自安计。‎ 译文:_________________________________________________‎ ‎___________________________________________________________‎ 答案 (1)胡松年上奏说:“请岳飞带军队撤离,慢慢观察刘豫的意向,如果刘豫置而不问,那么情况难以预测,应当告诫将士严守边疆就可以了。”‎ ‎(2)当时,秦桧当权,天下无论是否有见识的人,一律因为遭到怀疑、猜忌而被置于死地,因此士大夫无不曲意阿附以求自保。‎ 参考译文:‎ 胡松年,字茂老,海州怀仁人。幼年时丧父家贫,母亲织布卖钱,供养他上学,他读书过目不忘,尤其精通《易经》。政和二年,以上舍生出仕,担任潍州教授。(政和)八年,在便殿赏赐应对,徽宗见他相貌雄伟,改任他为校书郎兼资善堂赞读。任殿试参详官,以沈晦为第一名,徽宗大喜说:“我很早就听说了沈晦的名字,今天才得到他。”‎ 岳飞收复襄、汉,命令胡松年筹划守御事宜。胡松年上奏说:“请岳飞带军队撤离,慢慢观察刘豫的意向,如果刘豫置而不问,那么情况难以预测,应当告诫将士严守边疆就可以了。”‎ 胡松年平生不喜欢积敛财产,每次任官按例赐予金帛,因为军队耗费巨大,但一点也不向朝廷申请,有人劝他告诉朝廷,他说:“不申请就算了,告诉朝廷就成了沽名钓誉。”(他)喜好接待宾客,俸禄不够用来花费,有人请他为子孙着想节省用度。胡松年说:“贤能但多财,就会有损志节,况且俸禄,是皇上用来赡养老臣的。”自持囊到执政,所推荐代替自己的,都是当时的名人,所推荐的全部出于至公,即使是权贵也不能让他改变。‎ 当时,秦桧当权,天下无论是否有见识的人,一律因为遭到怀疑、猜忌而被置于死地,因此士大夫无不曲意阿附以求自保。唯独胡松年鄙视他,到死也没有给秦桧写一封书信,世人因此赞扬他。‎ ‎25.阅读下面的文言文,将文中画横线的句子翻译成现代汉语。‎ 黄尊素,字真长,余姚人。万历四十四年进士。除宁国推官,精敏强执。天启二年,擢御史,谒假归。明年冬还朝,数陈边事,力诋大将马世龙,忤枢辅孙承宗意。时帝在位数年,未尝一召见大臣。尊素请复便殿召对故事,面决大政,否则讲筵之暇,令大臣面商可否,帝不能用。‎ 四年二月,大风扬沙,昼晦,天鼓鸣,如是者十日。尊素力陈时政十失,末言:“不于此进贤退不肖,而疾刚方正直之士如仇雠,陛下独不为社稷计乎?”疏入,魏忠贤大怒,谋廷杖之。韩力救,乃夺俸一年。‎ 既而杨涟劾忠贤,被旨谯让。尊素愤,抗疏继之,忠贤得疏愈恨。‎ 八月,河南进玉玺。忠贤欲侈其事,命由大明门进,行受玺礼,百僚表贺。尊素上言,事获中止。五年春,遣视陕西茶马。甫出都,逆党曹钦程劾其专击善类,助高攀龙、魏大中虐焰,遂削籍。‎ ‎(1)不于此进贤退不肖,而疾刚方正直之士如仇雠,陛下独不为社稷计乎?‎ 译文:_________________________________________________‎ ‎___________________________________________________________‎ ‎___________________________________________________________‎ ‎(2)忠贤欲侈其事,命由大明门进,行受玺礼,百僚表贺。尊素上言,事获中止。‎ 译文:_________________________________________________‎ ‎___________________________________________________________‎ ‎___________________________________________________________‎ 答案 (1)不在此时举荐贤才斥退不肖之人,反而厌恶刚毅正直的人,把他们看作仇敌,陛下难道就不为国家考虑吗?‎ ‎(2)魏忠贤想大肆宣扬这件事,命令由大明门进入,行受玉玺礼,百官上表祝贺。黄尊素上表,这件事情得以中途停止。‎ 参考译文:‎ 黄尊素,字真长,余姚人。万历四十四年进士。被任命为宁国推官,精明敏悟坚强执着。天启二年,(黄尊素被)提拔为御史,请假回家。第二年冬天回到朝廷,多次陈述边防事务,极力批评大将马世龙,违背了枢辅孙承宗的心意。这时皇帝即位已有好几个年头了,从没有召见过大臣。黄尊素请求恢复在便殿召见大臣令其回答有关政事的先例,当面决定国家大事,否则也可利用讲解经史的机会,让大臣们当面商讨事情可不可以执行,皇帝没有采纳。‎ ‎(天启)四年二月,大风吹起黄沙,白昼昏暗,天像鼓一样鸣响,一连十天都是如此。黄尊素极力陈述时事政策的十大过失,最后说:“不在此时举荐贤才斥退不肖之人,反而厌恶刚毅正直的人,把他们看作仇敌,陛下难道就不为国家考虑吗?”奏章递入,魏忠贤大怒,打算在朝廷上杖打他。韩大力营救,于是只剥夺一年俸禄。‎ 不久杨涟弹劾魏忠贤,被下旨责备。黄尊素很愤怒,直言上书接着弹劾,魏忠贤得到奏章更加怀恨。‎ 八月,河南进献玉玺。魏忠贤想大肆宣扬这件事,命令由大明门进入,行受玉玺礼,百官上表祝贺。黄尊素上表,这件事情得以中途停止。五年春,(皇帝)派黄尊素视察陕西茶马互市的情况。刚出京城,逆党曹钦程弹劾黄尊素专门攻击贤良的人,助长高攀龙、魏大中暴虐的气焰,于是被削去官籍。‎ ‎[3年高考真题集训]‎ 一、[2016·全国卷Ⅰ]阅读下面的文言文,回答1~4题。‎ 曾公亮,字明仲,泉州晋江人。举进士甲科,知会稽县。民田镜湖旁,每患湖溢。公亮立斗门,泄水入曹娥江,民受其利。以端明殿学士知郑州,为政有能声盗悉窜他境至夜户不闭尝有使客亡橐中物移书诘盗公亮报吾境不藏盗殆从者之廋耳索之果然公亮明练文法,更践久,习知朝廷台阁典宪,首相韩琦每咨访焉。仁宗末年,琦请建储,与公亮等共定大议。密州民田产银,或盗取之,大理当以强。公亮曰:“此禁物也,取之虽强,与盗物民家有间矣。”固争之,遂下有司议,比劫禁物法,盗得不死。契丹纵人渔界河,又数通盐舟,吏不敢禁,皆谓:与之校,且生事。公亮言:“萌芽不禁,后将奈何?雄州赵滋勇而有谋,可任也。”使谕以指意,边害讫息。英宗即位,加中书侍郎兼礼部尚书,寻加户部尚书。帝不豫,辽使至不能见,命公亮宴于馆,使者不肯赴。公亮质之曰:“锡宴不赴,是不虔君命也。人主有疾,而必使亲临,处之安乎?”使者即就席。熙宁三年,拜司空兼侍中、河阳三城节度使。明年,起判永兴军。居一岁,还京师。旋以太傅致仕。元丰元年卒,年八十。帝临哭,辍朝三日。公亮方厚庄重,沉深周密,平居谨绳墨,蹈规矩;然性吝啬,殖货至巨万。初荐王安石,及同辅政,知上方向之,阴为子孙计,凡更张庶事,一切听顺,而外若不与之者。尝遣子孝宽参其谋,至上前略无所异,于是帝益信任安石。安石德其助己,故引擢孝宽至枢密以报之。苏轼尝从容责公亮不能救正,世讥其持禄固宠云。‎ ‎(节选自《宋史·曾公亮传》)‎ ‎1.下列对文中画波浪线部分的断句,正确的一项是(  )‎ A.为政有能声/盗悉窜他境/至夜户不闭/尝有使客亡橐中物移书/诘盗/公亮报/吾境不藏盗/殆从者之廋耳/索之/果然/‎ B.为政有能声/盗悉窜他境/至夜户不闭/尝有使客亡橐中物/移书诘盗/公亮报/吾境不藏盗/殆从者之廋耳/索之/果然/‎ C.为政有能声/盗悉窜/他境至夜户不闭/尝有使客亡橐中物移书/诘盗/公亮报/吾境不藏盗/殆从者之廋耳/索之/果然/‎ D.为政有能声/盗悉窜/他境至夜户不闭/尝有使客亡橐中物/移书诘盗/公亮报/吾境不藏盗/殆从者之廋耳/索之/果然/‎ 答案 B 解析 “盗悉窜他境”指盗贼全逃窜到别的州境,中间不能断开,排除C、D两项。“移书诘盗”意为下发公文追问盗贼,语意完整,排除A项。‎ ‎2.下列对文中加点词语的相关内容的解说,不正确的一项是(  )‎ A.首相指宰相中居于首位的人,与当今某些国家内阁或政府首脑的含义并不相同。‎ B.建储义为确定储君,也即确定皇位的继承人,我国古代通常采用嫡长子继承制。‎ C.古代朝廷中分职设官,各有专司,所以可用“有司”来指称朝廷中的各级官员。‎ D.契丹是古国名,后来改国号为辽,先后与五代和北宋并立,与中原常发生争端。‎ 答案 C 解析 “可用‘有司’来指称朝廷中的各级官员”错,“有司”应该指朝廷中的有关部门。‎ ‎3.下列对原文有关内容的概括和分析,不正确的一项是(  )‎ A.曾公亮初入仕途,为民兴利除弊。他进士及第后任职会稽县,当时湖水常常外溢,民田受害,他兴修水利工程,将水引入曹娥江,民众因此得益。‎ B.曾公亮久经历练,通晓典章制度。他熟知朝廷政务,首相韩琦每每向他咨询;密州有人偷盗民田产银,他认为判处死刑过重,据理力争,最终改判。‎ C.曾公亮防患未然,止息边地事端。契丹违约在界河捕鱼运盐,他认为萌芽不禁终将酿成大祸,派使者偕同雄州赵滋前往调解,边地双方得以相安无事。‎ D.曾公亮老谋深算,暗中为子孙计。他为人深沉,思虑周密,曾举荐王安石,安石受到宠信,他考虑子孙前程,不露痕迹地处处随顺安石,终于得到回报。‎ 答案 C 解析 “派使者偕同雄州赵滋前往调解”曲解原文,原文“使(之)谕以指意”的意思是:派他前去告谕旨意。‎ ‎4.把文中画横线的句子翻译成现代汉语。‎ ‎(1)锡宴不赴,是不虔君命也。人主有疾,而必使亲临,处之安乎?‎ 译文:_________________________________________________‎ ‎___________________________________________________________‎ ‎(2)苏轼尝从容责公亮不能救正,世讥其持禄固宠云。‎ 译文:_________________________________________________‎ ‎___________________________________________________________‎ 答案 (1)赐宴不到场,这是对君主命令的不敬。君主有病,却一定要他亲临宴会,做这样的事能心安吗?‎ ‎(2)苏轼曾严肃地责备公亮不能纠正弊病,世人讥讽他保持禄位加固宠幸。‎ 解析 (1)锡:通“赐”,赐予。是:代词,这。虔:恭敬。‎ ‎(2)救正:纠正弊病。持禄:保持禄位。固宠:加固宠幸。‎ 参考译文:‎ 曾公亮,字明仲,泉州晋江人。考中进士甲科,任会稽县知县。百姓在镜湖旁边耕种农田,常常担心湖水泛滥。曾公亮建立闸门,将水排到曹娥江,百姓在这件事情上得到益处。(曾公亮)以端明殿学士的身份任郑州知州,听到为政有能干的名声,盗贼都流窜到外地,以至于夜不闭户。曾经有使者丢失了口袋中的东西,下公文追问盗贼,曾公亮上报:“我们境内没有窝藏盗贼,大概随从之人偷藏而已。”‎ 搜查使者的随从,果然如此。曾公亮明达详熟公文法令,任职时间长,熟悉懂得朝廷台阁的规章,首相韩琦经常咨询访问他。仁宗末年,韩琦请求立储,与曾公亮等共同商定大计。密州民田盛产银子,有人盗取民田产银,大理寺把他们当作强盗论处。曾公亮说:“这是禁物,盗取民田产银虽然是强盗行为,却与从百姓家中盗取财物有区别。”他坚持争论,于是朝廷就下达给有关部门讨论,比照抢劫盗窃禁物的法律,盗取民田产银不判死刑。契丹纵容百姓在界河捕鱼,又多次通行盐船,官吏不敢禁止,都说:和他们计较,将要生出事端。曾公亮说:“刚开始时不禁止,以后将怎么办呢?雄州赵滋勇敢有计谋,能够胜任。”派他前去告谕旨意,边境的祸害终于平息了。英宗即位,加封(曾公亮)为中书侍郎兼礼部尚书,不久加户部尚书。皇帝身体不适,辽国使者到来不能接见,让曾公亮在馆中设宴,使者不赴宴。曾公亮质问使者道:“赐宴不到场,这是对君主命令的不敬。君主有病,却一定要他亲临宴会,做这样的事能心安吗?”使者于是赴宴。熙宁三年,被任命为司空兼侍中、河阳三城节度使。第二年,被启用兼管永兴军。过了一年,返回京城。不久以太傅的身份退休。元丰元年去世,享年八十岁。皇帝亲临哭悼,停止上朝三天。曾公亮端方庄重,深沉周密,平时谨守法律,遵循蹈矩;但天性吝啬,家产增加到巨万。当初推荐王安石,等到和他一起辅政,知道皇上正偏向他,暗中为子孙考虑,凡是改革各事,一概听从,而表面上装作不同意。曾经派儿子曾孝宽参与他的谋划,在皇上面前几乎没有异议,于是皇帝更信任王安石。王安石感谢他协助自己,因而引见提拔曾孝宽到枢密院来报答他。苏轼曾严肃地责备曾公亮不能纠正弊病,世人讥讽他保持禄位加固宠幸。‎ 二、[2016·全国卷Ⅱ]阅读下面的文言文,完成1~4题。‎ 陈登云,字从龙,唐山人。万历五年进士。除鄢陵知县,征授御史。出按辽东,疏陈安攘十策,又请速首功之赏。改巡山西。还朝,会廷臣方争建储。登云谓议不早决,由贵妃家阴沮之。十六年六月遂因灾异抗疏,劾妃父郑承宪,言:“承宪怀祸藏奸窥觊储贰且广结术士之流曩陛下重惩科场冒籍承宪妻每扬言事由己发用以恐喝勋贵簧鼓朝绅不但惠安遭其虐焰,即中宫与太后家亦谨避其锋矣。陛下享国久长,自由敬德所致,而承宪每对人言,以为不立东宫之效。干挠盛典,蓄隐邪谋,他日何所不至?”疏入,贵妃、承宪皆怒,同列亦为登云危,帝竟留中不下。久之,疏论吏部尚书陆光祖,又论贬四川提学副使冯时可,论罢应天巡抚李涞、顺天巡抚王致祥,又论礼部侍郎韩世能、尚书罗万化、南京太仆卿徐用检。朝右皆惮之。时方考选科道,登云因疏言:“近岁言官,壬午以前怵于威,则摧刚为柔;壬午以后昵于情,则化直为佞。其间岂无刚直之人,而弗胜龃龉,多不能安其身。二十年来,以刚直擢京卿者百止一二耳。背公植党,逐嗜乞怜,如所谓‘七豺’‘八狗’者,言路顾居其半。夫台谏为天下持是非,而使人贱辱至此,安望其抗颜直绳,为国家锄大奸、歼巨蠹哉!与其误用而斥之,不若慎于始进。”因条数事以献。出按河南。岁大饥,人相食。副使崔应麟见民啖泽中雁矢,囊示登云,登云即进之于朝。帝立遣寺丞锺化民赍帑金振之。登云巡方者三,风裁峻厉。以久次当擢京卿,累寝不下,遂移疾归。寻卒。‎ ‎(节选自《明史·陈登云传》)‎ ‎1.下列对文中画波浪线部分的断句,正确的一项是(  )‎ A.承宪怀祸藏奸/窥觊储贰且广结术士之流/曩陛下重惩科场/冒籍承宪妻每扬言事由己发/用以恐喝勋贵/簧鼓朝绅/‎ B.承宪怀祸藏奸/窥觊储贰/且广结术士之流/曩陛下重惩科场冒籍/承宪妻每扬言/事由己发用以恐喝勋贵/簧鼓朝绅/‎ C.承宪怀祸藏奸/窥觊储贰/且广结术士之流/曩陛下重惩科场冒籍/承宪妻每扬言事由己发/用以恐喝勋贵/簧鼓朝绅/‎ D.承宪怀祸藏奸/窥觊储贰且广结术士之流/曩陛下重惩科场/冒籍承宪妻每扬言/事由己发用以恐喝勋贵/簧鼓朝绅/‎ 答案 C 解析 本题在A、B两项中共有三处错误,分别为“窥觊储贰”后未断、“冒籍”属下以及“事由己发”属下,D项则兼有这三处错误。A项“窥觊储贰”与后面“广结术士之流”均为陈登云所列举郑承宪罪状,属并列关系,二者之间有“且”字表示顺接,应予断开,“科场冒籍”作为“重惩”的宾语不能分开;B项“事由己发”是承宪妻“扬言”的内容,应属上,“用以恐喝勋贵”则是“扬言”的目的。C项没有错误,是正确选项。‎ ‎2.下列对文中加点词语的相关内容的解说,不正确的一项是(  )‎ A.中宫是皇后所居之宫,后来又可以借指皇后,这与东宫又可借指太子是同样道理。‎ B.陛下指宫殿中立有护卫的台阶下,因群臣不可直呼帝王,于是借用为对帝王的尊称。‎ C.吏部是古代六部之一,掌管文官任免、考核、升降、调动等,长官为吏部尚书。‎ D.移疾指官员上书称病,实际是官员受到权臣诋毁,不得不请求退职的委婉说法。‎ 答案 D 解析 A项是关于古代的宫廷制度。中宫,是皇后居所的代称,也可用以代称皇后。中宫一词的由来是,因古代建筑宫城时,皇后的宫室都位于子午线上,而一般而言,皇后寝宫前是君王的居室,左右两旁夹挟着嫔妃的居室,后方多为太后及宫中年老女性养老之处,整体而言,是以周围建筑来衬托出皇后统辖后宫的领导地位,因而皇后又有六宫之长一称。B项的“陛下”是对古代帝王的尊称,陛下的“陛”指帝王宫殿的台阶。“陛下”原来指的是站在台阶下的侍者。臣子向天子进言时,不能直呼天子,必须先呼台下的侍者而告之。后来“陛下”就成为与帝王面对面应对的敬称。C项“吏部”考查的是古代职官知识,吏部是中国古代官署之一,东汉始置吏曹,改自尚书常侍曹,魏晋以后称吏部。隋、唐五代,列为尚书省六部之首,长官称为吏部尚书,下设吏部司、司封司、司勋司、考功司,掌管天下文官的任免、考课、升降、勋封、调动等事务。以上三项表述并无不当之处,都是正确的。只有D项说到的“移疾”指官员上书称病请求退职,这话是对的;但称病的原因有很多种,只说是因受到权臣诋毁不准确。所以,D项的表述是错误的,是正确选项。‎ ‎3.下列对原文有关内容的概括和分析,不正确的一项是(  )‎ A.陈登云不畏权贵,弹劾贵妃之父。他出于对朝廷的忠心,即便对郑承宪这样的国戚,也大胆揭发对方为非作歹,包藏祸心,幸而皇上并未因此发怒。‎ B.陈登云敢于直言,检举多名重臣。他在朝既久,发现诸多问题,于是奏告一干大臣,其中有些人因此遭到贬职或罢免,以至朝廷大官们都很畏惧他。‎ C.陈登云上疏指出,选才慎于始进。他认为二十年来,刚直者很少被提拔进京,在朝者却背公结党,谄媚权贵,与其误用后罢免,不如进用时慎重。‎ D.陈登云关心百姓,奏请救助灾区。在他巡视河南期间,当地年成歉收,百姓相食,他向朝廷呈告灾情,皇上当即派遣寺丞锺化民筹措钱款赈济灾民。‎ 答案 D 解析 A项的分析概括是陈登云不畏权贵,弹劾贵妃之父。他出于对朝廷的忠心,即便对郑承宪这样的国戚,也大胆揭发对方为非作歹,包藏祸心,这样的举动引起了被弹劾者的痛恨与同僚们的担忧,但皇帝并未做出反应,也就是没有发怒。B项的分析概括是陈登云敢于直言,检举多名重臣。他在朝既久,发现诸多问题,于是奏告一干大臣,“论贬”“论罢”,即其中有些人因此遭到贬职或罢免,以至朝廷大官们都很畏惧他。C项的分析概括是陈登云上疏指出,选才慎于始进。他认为二十年来,刚直者很少被提拔进京,在朝者却背公结党,谄媚权贵,与其误用后罢免,不如进用时慎重。这三个选项的概括和分析均符合原文的意思,没有不当之处。D项的分析概括说陈登云关心百姓,奏请救助灾区。在他巡视河南期间,当地年成歉收,百姓相食,他向朝廷呈告灾情。行文至此,也都是符合原文意思的。但下文又说,皇上当即派遣寺丞锺化民筹措钱款赈济灾民,这不符合原文意思。因为原文的表述是皇上当即派遣寺丞锺化民“赍帑金”赈济灾民,所谓“帑金”是指国库所藏的钱币,所以这句话的意思是皇帝专门从国库拨付了救灾钱款,因此不需要锺化民去筹措款项。D项所述有误,是正确选项。‎ ‎4.把文中画横线的句子翻译成现代汉语。‎ ‎(1)其间岂无刚直之人,而弗胜龃龉,多不能安其身。‎ 译文:_________________________________________________‎ ‎(2)副使崔应麟见民啖泽中雁矢,囊示登云,登云即进之于朝。‎ 译文:_________________________________________________‎ 答案 (1)其中难道没有刚正的人,但禁不住抵触排挤,大多无法安身。‎ ‎(2)副使崔应麟见到百姓吃湖泽中的雁粪,便装入袋中给陈登云看,陈登云随即送至朝廷。‎ 解析 第(1)小题的关键词语有“弗胜”“龃龉”两处。“胜”旧读平声,义为能承受,能承担。如《管子·入国》:“子有幼弱不胜养为累者。”杜甫《春望》:“白头搔更短,浑欲不胜簪。”“弗胜”即不能承受、禁不住。“龃龉”‎ 原指牙齿上下对不上,用来比喻意见不合、不相投合,抵触等意。韩愈《答窦秀才书》:“又不通时事,而与世多龃龉。”这个词在现代汉语中也常有使用,如丁玲《一九三零年春上海(之一)》二:“虽然两人的性格实在并不相同,但也从不龃龉的过下来了。”考生对此不应感到陌生。第(2)小题中的关键词语有“啖”“矢”“囊”三处。“啖”的含义是“吃”,如苏轼诗“日啖荔枝三百颗,不辞长作岭南人”,比较简单。“矢”在这里同“屎”,即粪便。如《左传·文公十八年》:“杀而埋之马矢之中。”《史记·廉颇蔺相如列传》:“顷之,三遗矢矣。”“囊”指口袋,这里用作动词,用口袋装。类似的例子如贾谊《过秦论》:“有席卷天下、包举宇内、囊括四海之意,并吞八荒之心。”‎ 参考译文:‎ 陈登云,字从龙,唐山人。万历五年(1577)进士。授职鄢陵知县,征召升为御史。出京巡视辽东,上疏陈明安定边境的十条对策,又请求加速建立首功的赏赐制度。后改为山西巡抚。回到京城,恰好朝廷大臣正在争论立储君的事。陈登云认为朝议迟迟不能决定,是贵妃家人暗中捣鬼。(万历)十六年(1588)六月因发生灾害上疏弹劾贵妃父亲郑承宪,说:“郑承宪包藏祸心,觊觎储君。天天与宦官往来,商量对策,而且广泛交结山人、术士、道士、和尚之流。当初陛下重惩科场冒名顶替之人,郑承宪的妻子每每扬言事情是自己揭发的,用来恐吓勋贵,以巧言惑动朝廷人士。不但惠安遭到他们算计,即使中宫与太后家也谨慎避开他们的锋芒。陛下统治国家已很久了,这是惠政的结果,而郑承宪每次对人说,认为是不立太子的结果。干扰盛典,蓄谋已久,以后有什么事他干不出来呢?奏折呈上,贵妃、郑承宪都发了脾气,同人也以为陈登云危险,但皇上把奏疏留下了。很久以后,他又上疏弹劾吏部尚书陆光祖,谈论贬去四川提学副使冯时可,议论免去应天巡抚李涞、顺天巡抚王致祥,又谈论礼部侍郎韩世能、尚书罗万化、南京太仆卿徐用检。朝廷的大官都很怕他。当时,正好考选科道,陈登云因而上疏说:“近来御史官,壬午以前害怕淫威,刚正的变温柔了;壬午以后拘于情面,正直的变为谄媚。其间难道没有刚正的人吗,但不胜被攻击倾轧,没有安身的地方。二十年来,以正直提升为京官的一百名中只有一二个人。背着皇上,培植党羽,摇尾乞怜,像所谓‘七豺’‘八狗’,谏官占了一半。台谏是为天下主持是非的,而使人践踏到这种地步,怎能希望他不顾情面,正直地处理事情,为国除掉奸人、消灭败类呢!与其因误用而贬退,不如谨慎地考察人选。”因而条陈数件事献给皇上。他出任河南巡按。那年发生大饥荒,人相互吞食。副使崔应麟看见百姓吃湖中的雁屎,便包来给陈登云看,陈登云便送到朝廷。皇上立即派遣寺丞钟化民分发库银赈恤百姓。陈登云三次巡视地方,执政严厉。按规定应当提升为京官,屡次被宫中扣住不下发,于是他称病归家。不久之后就死了。‎ 三、[2016·全国卷Ⅲ]阅读下面的文言文,完成1~4题。‎ 傅珪,字邦瑞,清苑人。成化二十三年进士。改庶吉士。弘治中,授编修,寻兼司经局校书。与修《大明会典》成,迁左中允。武宗立,以东宫恩,进左谕德,充讲官,纂修《孝宗实录》。时词臣不附刘瑾,瑾恶之。谓《会典》成于刘健等多所糜费镌与修者官降珪修撰俄以《实录》成进左中允再迁翰林学士历吏部左右侍郎正德六年代费宏为礼部尚书。礼部事视他部为简,自珪数有执争,章奏遂多。帝好佛,自称大庆法王。番僧乞田百顷为法王下院,中旨下部,称大庆法王与圣旨并。珪佯不知,执奏:“孰为大庆法王,敢与至尊并书,大不敬。”诏勿问,田亦竟止。珪居闲类木讷者。及当大事,毅然执持,人不能夺,卒以此忤权幸去。教坊司臧贤请易牙牌,制如朝士,又请改铸方印,珪格不行。贤日夜腾谤于诸阉间,冀去珪。御史张羽奏云南灾,珪因极言四方灾变可畏。八年五月,复奏四月灾,因言:“春秋二百四十二年,灾变六十九事。今自去秋来,地震天鸣,雹降星殒,龙虎出见,地裂山崩,凡四十有二,而水旱不与焉,灾未有若是甚者。”极陈时弊十事,语多斥权幸,权幸益深嫉之。会户部尚书孙交亦以守正见忤,遂矫旨令二人致仕。两京言官交章请留,不听。珪归三年,御史卢雍称珪在位有古大臣风,家无储蓄,日给为累,乞颁月廪、岁隶,以示优礼。又谓珪刚直忠谠,当起用。吏部请如雍言,不报。而珪适卒,年五十七。嘉靖元年录先朝守正大臣,追赠太子少保,谥文毅。‎ ‎(节选自《明史·傅珪传》)‎ ‎1.下列对文中画波浪线部分的断句,正确的一项是(  )‎ A.谓《会典》成于刘健等/多所糜费/镌与修者/官降珪修撰/俄以《实录》成/进左中允/再迁翰林学士/历吏部左/右侍郎/‎ B.谓《会典》成于刘健等/多所糜费/镌与修者官/降珪修撰/俄以《实录》成/进左中允/再迁翰林学士/历吏部左/右侍郎/‎ C.谓《会典》成于刘健等/多所糜费/镌与修者官/降珪修撰/俄以《实录》成进/左中允再迁翰林学士/历吏部左/右侍郎/‎ D.谓《会典》成于刘健等/多所糜费/镌与修者/官降珪修撰/俄以《实录》成进/左中允再迁翰林学士/历吏部左/右侍郎/‎ 答案 B 解析 作为人物传记中的叙述语句,最好先抓人物事件,这句话主要讲傅珪参与修撰,因此而被降职和升官的历程,意思是:“说《会典》成于刘健等人,糜费很多,降参与修撰者的官,傅珪降为修撰,不久因《孝宗实录》修成,晋升左中允,再次晋升至翰林学士,历任吏部左、右侍郎。”然后做语法分析,即可得到答案。同时选择题,可以比对排除。所以选B。‎ ‎2.下列对文中加点词语的相关内容的解说,不正确的一项是(  )‎ A.礼部为六部之一,掌管礼仪、祭祀、土地、户籍等职事,部长官称为礼部尚书。‎ B.教坊司是管理宫廷音乐的官署,专管雅乐以外的音乐、歌舞的教习等演出事务。‎ C.致仕本义是将享受的禄位交还给君王,表示官员辞去官职或到规定年龄而离职。‎ D.历史上的“两京”有多种所指,文中则指明代永乐年间迁都以后的南北两处京城。‎ 答案 A 解析 A项中“土地、户籍”等职事归户部掌管。‎ ‎3.下列对原文有关内容的概括和分析,不正确的一项是(  )‎ A.傅珪进入仕途,参与纂修文献。弘治年间,他兼任司经局校书,参与编修《大明会典》得以升职;武宗继位,他进位左谕德,充讲官,修撰《孝宗实录》。‎ B.傅珪任职礼部,劝谏讲究策略。他担任礼部尚书时,由于屡有争端,上奏增多;番僧因帝好佛求地百顷,他佯作不知皇上自称大庆法王,不理会给地的事。‎ C.傅珪守正不阿,反遭诬蔑报复。每遇大事,他都能坚持己见,不肯随意改易。因而触怒许多人;后因得罪权贵被迫退休,虽有言官请留,他仍坚持离职。‎ D.傅珪为官清廉,死后受到好评。御史卢雍称赞他在位时有古代大臣风范,归乡后家无积蓄,艰难度日,嘉靖元年,他被列为先朝守正大臣,追谥为文毅。‎ 答案 C 解析 C项“后因得罪权贵被迫退休”错误,原文是“遂矫旨令二人致仕”即“(权贵)假托皇上诏命,让两个人辞去官职”需要对照原文,并对原文相应语句做翻译,错误往往在细微处。‎ ‎4.把文中画横线的句子翻译成现代汉语。‎ ‎(1)极陈时弊十事,语多斥权幸,权幸益深嫉之。‎ 译文:_________________________________________________‎ ‎(2)又谓珪刚直忠谠,当起用。吏部请如雍言,不报。‎ 译文:_________________________________________________‎ 答案 (1)(傅珪)极力奏陈其时社会弊病十件事,话语多指斥受宠的权贵,权贵愈加痛恨他。‎ ‎(2)又认为傅珪刚正忠实,敢于直言,应当起用,吏部按照卢雍的话上奏,没有回复。‎ 解析 第(1)句比较简单,翻译中注意通顺即可。权幸:权指权贵,幸指宠幸。这里是被宠幸。这是课本中的常用实词,如《阿房宫赋》中的“缦立远视,而望幸焉”。第(2)句状语后置“如雍言”需要注意。‎ 参考译文:‎ 傅珪,字邦瑞,清苑人。成化二十三年进士。改任庶吉士。弘治年中,授予编修一职,不久兼任司经局校书。参与编修《大明会典》完成,迁任左中允。武宗即位,因为东宫恩赐,升任左谕德,充当为皇帝经筵进讲的官员,编修《孝宗实录》。当时文学侍从不攀附刘瑾,刘瑾很憎恨他们。说《大明会典》在刘健等人手中完成,耗费很多,削减参与编修者的官职,降傅珪为修撰,不久因为《孝宗实录》编成,升任左中允,再升翰林学士,历任吏部左右侍郎,正德六年代替费宏担任礼部尚书。礼部的事务与其他各部比较相对简单,但从傅珪开始多次直言规劝,章奏就多了起来。皇帝喜好佛,自称“大庆法王”。西域僧人求百顷田地作法王下院,皇帝的亲笔命令下到部里,说大庆法王的命令与圣旨等同。傅珪假装不知道,执意上奏说:“谁是大庆法王,敢与皇帝并列,是对皇帝大不敬。”皇帝下诏不过问,求田之事也最终停止。傅珪 闲居时类似木讷不会说话的人。到担当大事时,毅然固执地坚持自己的意见,他人不能改变,最终因为这忤逆权贵佞臣而离职。教坊司乐官臧贤请求换牙牌,式样要做得像朝廷官员的一样,又请求重新铸造他自己的方印,傅珪阻止没有做成。臧贤经常在阉党面前指责傅珪,盼望着除去傅珪。御史张羽奏报云南灾情,傅珪趁此竭力陈说四方灾害可怕。(正德)八年五月,再次奏报四月灾情,(傅珪)于是进言说:“春秋二百四十二年,灾害六十九次。现今从去年秋天以来,地震雷鸣,天降冰雹,辰星坠落,龙虎出现,地裂山崩,总共四十二次,加之水涝旱灾不断,灾害没有像这样严重的。”他极力奏陈其时社会弊病十件事,话语多指斥受宠的权贵,权贵愈加痛恨他。恰逢户部尚书孙交也因为恪守正道被欺凌,阉党就假托帝王诏命令二人退休。长安和洛阳监官和谏官交互向皇帝上书请求二人留任,(皇帝)不听。傅珪归家三年,御史卢雍称赞傅珪在位时有古大臣风范,家里没有积蓄,每天为吃饭发愁,乞求发放每月的禄米和仆人每年的食米,以表示朝廷对官员的优待。又认为傅珪刚正忠实,敢于直言,应当起用。吏部按照卢雍的话上奏,没有回复。傅珪去世时,终年五十七岁。嘉靖元年录入先朝守正大臣,追赠太子少保,谥号文毅。‎ 四、[2015·全国卷Ⅰ]阅读下面的文言文,完成1~4题。‎ 孙傅,字伯野,海州人。登进士第,为礼部员外郎。时蔡翛为尚书,傅为言天下事,劝其亟有所更,不然必败。翛不能用。迁至中书舍人。宣和末高丽入贡使者所过调夫治舟骚然烦费傅言索民力以妨农功而于中国无丝毫之益宰相谓其所论同苏轼奏贬蕲州安置给事中许翰以为傅论议虽偶与轼合,意亦亡他,以职论事而责之过矣,翰亦罢去。靖康元年,召为给事中,进兵部尚书。上章乞复祖宗法度,钦宗问之,傅曰:“祖宗法惠民,熙、丰法惠国,崇、观法惠奸。”时谓名言。十一月,拜尚书右丞,俄改同知枢密院。金人围都城,傅日夜亲当矢石。金兵分四翼噪而前,兵败退,堕于护龙河,填尸皆满,城门急闭。是日,金人遂登城。二年正月,钦宗诣金帅营,以傅辅太子留守,仍兼少傅。帝兼旬不返,傅屡贻书请之。及废立檄至,傅大恸曰:“吾唯知吾君可帝中国尔,苟立异姓,吾当死之。”金人来索太上、帝后、诸王、妃主,傅留太子不遣。密谋匿之民间,别求状类宦者二人杀之,并斩十数死囚,持首送之,绐金人曰:“宦者欲窃太子出,都人争斗杀之,误伤太子。因帅兵讨定,斩其为乱者以献。苟不已,则以死继之。”越五日,无肯承其事者。傅曰:“吾为太子傅,当同生死。金人虽不吾索,吾当与之俱行,求见二酋面责之,庶或万一可济。”遂从太子出。金守门者曰:“所欲得太子,留守何预?”傅曰:“我宋之大臣,且太子傅也,当死从。”是夕,宿门下;明日,金人召之去。明年二月,死于朔廷。绍兴中,赠开府仪同三司,谥曰忠定。‎ ‎(节选自《宋史·孙傅传》)‎ ‎1.下列对文中画波浪线部分的断句,正确的一项是(  )‎ A.宣和末/高丽入贡/使者所过/调夫治舟/骚然烦费/傅言/索民力以妨农功/而于中国无丝毫之益/宰相谓其所论同苏轼/奏贬蕲州安置/‎ B.宣和末/高丽入贡/使者所过/调夫治舟/骚然烦费/傅言/索民力以妨农功/而于中国无丝毫之益/宰相谓其所论/同苏轼奏/贬蕲州安置/‎ C.宣和末/高丽入贡使者/所过调夫治舟/骚然烦费/傅言/索民力以妨农功/而于中国无丝毫之益/宰相谓其所论/同苏轼奏/贬蕲州安置/‎ D.宣和末/高丽入贡使者/所过调夫治舟/骚然烦费/傅言/索民力以妨农功/而于中国无丝毫之益/宰相谓其所论同苏轼/奏贬蕲州安置/‎ 答案 A 解析 回答此题,要对所给语段进行语法分析。所给语段中,“入贡”“过”“调”“治”“言”“索”“谓”“同”等可为谓语,“高丽”“使者”“宰相”等为专有名词,同时要注意文句意思的完整性和连贯性,根据这些内容,便可以准确判断各句间的层次。‎ ‎2.下列对文中加点词语的相关内容的解说,不正确的一项是(  )‎ A.登进士第,又可称为进士及第,指科举时代经考试合格后录取成为进士。‎ B.兵部是古代“六部”之一,掌管全国武官选用和兵籍、军械、军令等事宜。‎ C.庙号是皇帝死后,在太庙立室奉祀时特起的名号,如高祖、太宗、钦宗。‎ D.太子指封建时代君主儿子中被确定继承君位的人,有时也可指其他儿子。‎ 答案 D 解析 封建时代只有“被确定继承君位的人”才能称为“太子”,而君主的“其他儿子”则不能称为“太子”。‎ ‎3.下列对原文有关内容的概括和分析,不正确的一项是(  )‎ A.孙傅入仕以后,积极向上建言。他担任礼部员外郎,对尚书蔡翛纵论天下大事,劝蔡迅速有所变更,否则必将失败,可惜他的建议没有被采纳。‎ B.孙傅上奏,请求恢复祖宗法度。他任兵部尚书后,从效用角度高度评价祖宗法度和熙、丰年间的法度,批评崇、观年间的法度,受到时人赞许。‎ C.孙傅不畏金人,努力保全太子。金人掳走钦宗后又索求太子,他密谋藏匿太子,杀二宦者将首级送至金营,欺骗金人说这就是误伤太子之人。‎ D.孙傅舍身取义,死后谥为忠定。太子被迫至金营,孙傅随往,却受到守门者劝阻,他表示身为太子傅,应誓死跟从太子;后被金人召去,死于北廷。‎ 答案 C 解析 “杀二宦者……误伤太子之人”错误,文中是说杀了两个像宦官的人和十几个死囚,献上首级,说这十几个人是误伤太子之人。‎ ‎4.把文中画横线的句子翻译成现代汉语。‎ ‎(1)吾唯知吾君可帝中国尔,苟立异姓,吾当死之。‎ 译文:_________________________________________________‎ ‎(2)金人虽不吾索,吾当与之俱行,求见二酋面责之,庶或万一可济。‎ 译文:_________________________________________________‎ 答案 (1)我只知道我的君王可以在中国称帝而已,如果另立异姓,我将为此而死。‎ ‎(2)金人虽然没有点名要我,我却应该与太子同去,求见两名首领当面指责他们,或许有成功的可能。‎ 解析 (1)句,“唯”“帝”“苟”“死”为关键词,其中“帝”是名词作动词,“称帝”;“死”是为动用法,“为……而死”。(2)句中,“不吾索”“酋”“庶或”“济”为关键词,其中“不吾索”为宾语前置句,翻译成“没有点名要我”;“庶”是副词,“可能”“希望”,或译为“或许”。‎ 参考译文:‎ 孙傅,字伯野,海州人。考中进士,任礼部员外郎。当时蔡翛担任尚书,孙傅向他陈述天下政事,劝他赶快做些更改,否则一定会失败。蔡翛没有听从。孙傅升至中书舍人。宣和末年,高丽前来进贡,使者所经过之处,调发民夫备办舟船,引起骚动,费用又颇多。孙傅说:“滥用民力妨碍农事,对于国家却没有丝毫好处。”宰相认为他的言论与苏轼的相同,上奏贬他到蕲州安置。给事中许翰认为孙傅的议论只是偶然与苏轼的相同,也没有其他意思,依据职责论事而受到指责实在过分了,许翰也被免职。靖康元年,(孙傅)受召入京任给事中,升任兵部尚书。他上奏章请求恢复祖宗法度,钦宗问孙傅,孙傅说:“祖宗法度有利于百姓,熙宁、元丰时的法度有利于国家,崇宁、大观时的法度有利于奸臣。”当时的人认为是名言。十一月,孙傅担任尚书右丞,不久改任同知枢密院。金人围攻都城,孙傅日夜亲自冒着箭石督战。金兵分别从四面鼓噪而攻,孙傅的军队败退,掉进护龙河,护龙河被尸体填满,城门急忙关闭了。当天,金兵就攻进城里。(靖康)二年正月,钦宗到金兵元帅营去,安排孙傅辅助太子留守京城,(孙傅)仍然兼任少傅。钦宗二十天还没回来,孙傅多次寄信给金营请求放回钦宗。等到废立皇帝的檄书传来,孙傅大哭道:“我只知道我的君王可以在中国称帝而已,如果另立异姓,我将为此而死。”金人来索要太上皇、皇后、诸王、妃主,孙傅留住太子不放行。秘密谋划把太子藏在民间,另外找两个像宦官的人杀死,并杀死十几个死囚,拿着他们的头送给金人,欺骗金人说:“宦官打算把太子秘密送出,京城之人争相斗杀宦官,误伤了太子。于是率兵讨伐平定,杀死作乱的人来献上其首级。如果不停止索求,太子就会自杀。”过了五天,没有人肯承担这件事。孙傅说:“我是太子的老师,应当与太子同生死。金人虽然没有点名要我,我却应该与太子同去,求见两名首领当面指责他们,或许有成功的可能。”于是跟从太子出城。守城门的金兵说:“金人想要的是太子,留守何必参与?”孙傅说:“我是宋朝大臣,而且是太子的老师,应当随他去死。”当晚,住在城门下;第二天,金人召他前去。第二年二月,死在北廷。绍兴年间,被追赠为开府仪同三司,谥号为忠定。‎ 五、[2015·全国卷Ⅱ]阅读下面的文言文,完成1~4题。‎ 来护儿,字崇善,未识而孤,养于世母吴氏。吴氏提携鞠养,甚有慈训。幼而卓荦,初读《诗》,舍书叹曰:“大丈夫在世,会为国灭贼以取功名!”群辈惊其言而壮其志。及长,雄略秀出,志气英远。‎ 会周师定淮南所住白土村地居疆埸数见军旅护儿常慨然有立功名之志及开皇初宇文忻等镇广陵平陈之役护儿有功焉进位上开府,赏物一千段。仁寿初,迁瀛州刺史,以善政闻,频见劳勉。炀帝嗣位,被追入朝,百姓攀恋,累日不能出境,诣阙上书致请者,前后数百人。帝谓曰:“昔国步未康,卿为名将,今天下无事,又为良二千石,可谓兼美矣。”大业六年,车驾幸江都,谓护儿曰:“衣锦昼游,古人所重,卿今是也。”乃赐物二千段,并牛酒,令谒先人墓,宴乡里父老。仍令三品已上并集其宅,酣饮尽日,朝野荣之。十二年,驾幸江都,护儿谏曰:“陛下兴军旅,百姓易咨怨。车驾游幸,深恐非宜。伏愿驻驾洛阳,与时休息。陛下今幸江都,是臣衣锦之地,臣荷恩深重,不敢专为身谋。”帝闻之,厉色而起,数日不得见。后怒解,方被引入,谓曰:“公意乃尔,朕复何望!”护儿因不敢言。及宇文化及构逆,深忌之。是日旦将朝,见执。护儿曰:“陛下今何在?”左右曰:“今被执矣。”护儿叹曰:“吾备位大臣,荷国重任,不能肃清凶逆,遂令王室至此,抱恨泉壤,知复何言!”乃遇害。护儿重然诺,敦交契,廉于财利,不事产业。至于行军用兵,特多谋算,每览兵法,曰:“此亦岂异人意也!”善抚士卒,部分严明,故咸得其死力。‎ ‎(节选自《北史·来护儿传》)‎ ‎1.下列对文中画波浪线部分的断句,正确的一项是(  )‎ A.会周师定淮南所/住白土村/地居疆埸/数见军旅护儿/常慨然有立功名之志/及开皇初/宇文忻等镇广陵/平陈之役/护儿有功焉/‎ B.会周师定淮南所/住白土村/地居疆埸/数见军旅/护儿常慨然有立功名之志/及开皇初/宇文忻等镇广陵/平陈之役/护儿有功焉/‎ C.会周师定淮南/所住白土村/地居疆埸/数见军旅护儿/常慨然有立功名之志/及开皇初/宇文忻等镇广陵/平陈之役/护儿有功焉/‎ D.会周师定淮南/所住白土村/地居疆埸/数见军旅/护儿常慨然有立功名之志/及开皇初/宇文忻等镇广陵/平陈之役/护儿有功焉/‎ 答案 D 解析 作答此类试题,要注意常用于句首、句尾的词语及句子成分,如,解答本题时要特别注意名词或名词性短语作主语的情况。句中的所字结构“所住”为名词性词组,根据句意应放在句首,据此可排除A、B两项;护儿、宇文忻为人名,根据句意应放在句首,据此可排除C项。‎ ‎2.下列对文中加点词语的相关内容的解说,不正确的一项是(  )‎ A.古代男子有名有字,名是出生后不久父亲起的,字是二十岁举行冠礼后才起的。‎ B.谥号是古代帝王、大臣等死后,据其生平事迹评定的称号,如武帝、哀帝、炀帝。‎ C.嗣位指继承君位,我国封建王朝通常实行长子继承制,君位由最年长的儿子继承。‎ D.阙是宫门两侧的高台,又可借指宫廷;“诣阙”既可指赴朝廷,又可指赴京都。‎ 答案 C 解析 我国封建王朝通常实行嫡长子继承制,君位由最年长的嫡子继承。‎ ‎3.下列对原文有关内容的概括和分析,不正确的一项是(  )‎ A.来护儿少有大志,成年后秀拔于群。他自幼而孤,得到吴氏教诲,立下为国杀敌、求取功名的志向;长大以后,更是雄略超群,志气英发。‎ B.来护儿推行善政,深受百姓拥戴。在瀛州刺史任上,他声名远闻,屡受嘉奖;炀帝时,百姓舍不得他回朝廷任职,上书请愿者达数百人。‎ C.来护儿直言劝谏,后被奸人杀害。他谏请炀帝停驾洛阳,不再远游江都,引发炀帝大怒,以致宇文化及杀害他时,炀帝也没有设法保护。‎ D.来护儿廉于财利,用兵极有谋略。他信守承诺,注重友情,轻视钱财,不置产业;善待士卒,处事严明,谋略多合兵法,部属争相尽力。‎ 答案 C 解析 从原文“今被执矣”可知,皇帝已先于来护儿被抓,是无力保护他,所以,“引发炀帝大怒……没有设法保护”错。‎ ‎4.把文中画横线的句子翻译成现代汉语。‎ ‎(1)陛下兴军旅,百姓易咨怨。车驾游幸,深恐非宜。‎ 译文:_________________________________________________‎ ‎___________________________________________________________‎ ‎(2)不能肃清凶逆,遂令王室至此,抱恨泉壤,知复何言!‎ 译文:_________________________________________________‎ ‎___________________________________________________________‎ 答案 (1)陛下兴起战事,易于引起百姓叹息怨恨。如今又要外出巡游,恐怕很不合适。‎ ‎(2)不能清除凶恶悖逆之人,终致朝廷落到如此地步,我只能抱憾于黄泉之下,还能再说什么呢!‎ 参考译文:‎ 来护儿,字崇善,还不记事便成了孤儿,后被伯母吴氏抚养。吴氏关怀养育他,给予他许多慈母般的教诲。(来护儿)年纪虽小,但超绝出众,初次读《诗经》,便放下书感叹道:“大丈夫在世,一定要为国家消灭贼寇以求取功名!”周围的人对他的话感到惊奇,并感叹他志向的远大。长大以后,(来护儿)具有非凡的谋略,十分出众,志向气量非常人所及。恰逢周国军队平定淮南,(来护儿)所住的白土村,地处边界,经常见到军队,来护儿常常情绪激昂,有建立功名的志向。等到开皇初年,宇文忻等人镇守广陵,(来护儿)因在平定陈的战役中立下战功,进升为上开府,被赏赐缣帛一千段。仁寿初年,调任瀛州刺史,以擅长理政闻名,多次受到慰问勉励。隋炀帝即位,被征入朝,当地百姓拽住他恋恋不舍,以至于几天后他还没有离开这里,上书朝廷挽留他的前后有几百人。炀帝对他说:“过去国家没有安定时,你是名将,如今天下无事,你又是好郡守,可以说是样样擅长啊!”大业六年,隋炀帝到江都,对来护儿说:“衣锦还乡,这是古人所看重的,你今天就是这样。”于是赏赐他缣帛两千段以及牛和酒,叫他到先人的墓前祭拜,宴请同乡父老。又叫三品以上的官员一齐到他的住宅,畅饮一整天,朝野上下非常羡慕他。(大业)十二年,炀帝出游江都,来护儿进谏说:“陛下兴起战事,易于引起百姓叹息怨恨。如今又要外出巡游,恐怕很不合适。希望陛下暂留在洛阳,趁机休息一段时间。陛下如今去江都,那是臣的家乡,臣受恩深重,不敢只为自己谋划。”隋炀帝听后,发怒而起,几天都没让来护儿晋见。后来隋炀帝怒气消散,才叫人领他进来,对来护儿说:“你既然想要这样,朕还有什么指望呢?”来护儿于是不敢说话。等到宇文化及造反时,他对来护儿十分忌恨。那天早晨将去上朝,(来护儿)被叛贼捉住了。来护儿说:“陛下现在在哪里?”旁边的人说:“如今被捉住了。”来护儿叹息说:“我身为大臣,担负国家重任,不能清除凶恶悖逆之人,终致朝廷落到如此地步,我只能抱憾于黄泉之下,还能再说什么呢!”于是被杀害。来护儿重信用,讲交情,淡泊财利,不经营产业。至于出征用兵,计谋特多,每次阅览兵书,说:“这难道也是异于人的想法吗?”他善于安抚士兵,裁决严明,所以士兵都愿为他效力。‎ 六、[2014·全国卷Ⅰ]阅读下面的文言文,完成1~4题。‎ 于休烈,河南人也。至性贞悫,机鉴敏悟。自幼好学,善属文。举进士,授秘书省正字。转比部员外郎,郎中。杨国忠辅政,排不附己者,出为中部郡太守。值禄山构难,肃宗践祚,休烈迁太常少卿,知礼仪事,兼修国史。肃宗自凤翔还京,励精听受,尝谓休烈曰:“君举必书,良史也。朕有过失,卿书之否?”对曰:“禹、汤罪己,其兴也勃焉。有德之君,不忘规过,臣不胜大庆。”时中原荡覆,典章殆尽,无史籍检寻。休烈奏曰:“《国史》《实录》,圣朝大典,修撰多时,今并无本。伏望下御史台推勘史馆所由,令府县招访。有人别收得《国史》《实录》,如送官司,重加购赏。”前修史官工部侍郎韦述陷贼,入东京,至是以其家藏《国史》一百一十三卷送于官。休烈寻转工部侍郎、修国史,献《五代帝王论》,帝甚嘉之。宰相李揆矜能忌贤以休烈修国史与己齐列嫉之奏为国子祭酒权留史馆修撰以下之休烈恬然自持殊不介意代宗即位,甄别名品,宰臣元载称之,乃拜右散骑常侍,依前兼修国史,累封东海郡公,加金紫光禄大夫。在朝凡三十余年,历掌清要,家无儋石之蓄。恭俭温仁,未尝以喜愠形于颜色。而亲贤下士,推毂后进,虽位崇年高,曾无倦色。笃好坟籍,手不释卷,以至于终。大历七年卒,年八十一。是岁春,休烈妻韦氏卒。上特诏赠韦氏国夫人,葬日给卤簿鼓吹。及闻休烈卒,追悼久之,褒赠尚书左仆射,赙绢百匹、布五十端,遣谒者内常侍吴承倩就私第宣慰。儒者之荣,少有其比。‎ ‎(节选自《旧唐书·于休烈传》)‎ ‎1.对下列句子中加点的词语的解释,不正确的一项是(  )‎ A.自幼好学,善属文 属文:撰写文章 B.值禄山构难,肃宗践祚 践祚:帝王即位 C.肃宗自凤翔还京,励精听受 励精:专心致志 D.时中原荡覆,典章殆尽 荡覆:动荡倾覆 答案 C 解析 励精:振作精神。‎ ‎2.对文中画波浪线部分的断句,正确的一项是(  )‎ A.宰相李揆矜能忌贤/以休烈修国史与己齐列/嫉之/奏为国子祭酒/权留史馆/修撰以下之/休烈恬然自持/殊不介意/‎ B.宰相李揆矜能忌贤/以休烈修国史与己齐列/嫉之/奏为国子祭酒/权留史馆修撰以下之/休烈恬然/自持殊不介意/‎ C.宰相李揆矜能忌贤/以休烈修国史与己齐列/嫉之/奏为国子祭酒/权留史馆/修撰以下之/休烈恬然/自持殊不介意/‎ D.宰相李揆矜能忌贤/以休烈修国史与己齐列/嫉之/奏为国子祭酒/权留史馆修撰以下之/休烈恬然自持/殊不介意/‎ 答案 D 解析 断句首先要反复默读文句,做到意通句顺,合乎文言文语言习惯,认真体会词语含义和词语间的关系,由易到难、循序渐进地断开文段。利用句中的人名、地名、官职名、文言虚词、句子结构等断句。如,句中的“国子祭酒”“休烈”等专有名词,“之”“以”等虚词,都是断句的重要参考因素。‎ ‎3.下列对原文有关内容的概括和分析,不正确的一项是(  )‎ A.休烈忠诚机敏,谨遵职业操守。他自幼好学,入仕后受到杨国忠排挤,离京到地方任职;安禄山叛乱后,他直言不讳地回答了肃宗关于史官职责的问题。‎ B.休烈审察形势,做好本职事务。当时历经战乱,典章史籍散佚,他提出购求当朝大典以备查检使用,最终得到前修史官韦述家藏《国史》一百余卷。‎ C.休烈淡泊名利,终生好学不倦。他虽遭贬职,却恬然处之,毫不在意,在朝三十余年,历任要职,并无多少积蓄;喜好典籍,终日捧读,直至去世。‎ D.休烈夫妇去世,尽享身后哀荣。他夫人去世,皇上特诏追赠她国夫人;他本人去世,皇上追念许久,追赠他尚书左仆射,并派专人到他家表示慰问。‎ 答案 A 解析 休烈并没有直言不讳地回答肃宗关于史官职责的问题,而是举禹、汤之例,委婉含蓄地表明了自己的观点。‎ ‎4.把文中画横线的句子翻译成现代汉语。‎ ‎(1)禹、汤罪己,其兴也勃焉。有德之君,不忘规过,臣不胜大庆。‎ 译文:_________________________________________________‎ ‎___________________________________________________________‎ ‎(2)而亲贤下士,推毂后进,虽位崇年高,曾无倦色。‎ 译文:_________________________________________________‎ ‎___________________________________________________________‎ 答案 (1)夏禹、商汤归罪自己,他们能够蓬勃兴起。有道德的君王,不忘改正过错,我深表庆贺。‎ ‎(2)而亲近贤才,屈身交接士人,荐举后辈,虽位尊年高,但一点倦怠的神色都没有。‎ 解析 (1)罪:归罪。兴:兴起。勃:蓬勃。规:改正。‎ ‎(2)亲:动词,亲近。推毂:比喻推荐人才。崇:尊贵。曾:竟然,却。‎ 参考译文:‎ 于休烈,河南人。(于休烈)性情正直诚实,机敏聪明。从小好学,善于写文章。考中进士科,授职秘书省正字。改任比部员外郎,郎中。杨国忠辅佐朝政,排挤不依附自己的人,(于休烈)出京任中部郡太守。正值安禄山叛乱,肃宗即位,于休烈改任太常少卿,掌管礼仪事务,兼修国史。肃宗从凤翔返回京城,振作精神听取采纳臣下的建议,曾经对于休烈说:“国君的任何举动都要记录下来,这才是良史。朕有过失,卿是否要记呢?”他回答说:“夏禹、商汤归罪自己,他们能够蓬勃兴起。有德之君,不忘纠正过错,我深表庆贺。”当时中原动荡倾覆,典章几乎散失尽,没有史籍可供查寻。于休烈上奏说:“‎ ‎《国史》《实录》,是圣朝大典,修撰多年,如今一部也没有保存下来。希望交付御史台审问勘查史馆的有关官员,令府县招致访求。有人另外收存《国史》《实录》的,若送到官府,重金购回并从优奖赏。”前任修史官工部侍郎韦述身陷贼城,此时进入东京,至此将他家收藏的《国史》一百一十三卷送到官府。于休烈不久改任工部侍郎,修国史,献上《五代帝王论》,皇上非常赞赏。宰相李揆自负才能嫉妒贤人,因于休烈修国史与自己同列,嫉妒他,奏请任他为国子祭酒,暂留史馆修撰,以此来压制他。于休烈安然自处,毫不介意。代宗即位,鉴别官员的名望品德,(于休烈)受到宰相元载称赞,于是被授为右散骑常侍,依旧兼修国史,被累封至东海郡公,加授金紫光禄大夫。在朝中共三十多年,历任清贵显职,家中没有一两石的积蓄。他恭谨俭朴温和仁义,从不将喜怒之情表现在脸上。而亲近贤才,屈身交接士人,荐举后辈,虽位尊年高,但一点倦怠的神色都没有。(于休烈)酷好古籍,手不释卷,直到临终。他于大历七年去世,终年八十一岁。这年春天,于休烈的妻子韦氏去世。皇上特意下诏追赠韦氏为“国夫人”,安葬之日赐给仪仗鼓乐。等到听说于休烈去世,追怀悼念了很长时间,褒奖追赠尚书左仆射,赐给绢一百匹、布五十缎,派谒者内常侍吴承倩到他家中宣旨慰问。儒士的荣耀,很少有人能与他相比的。‎ 七、[2014·全国卷Ⅱ]阅读下面的文言文,完成1~4题。‎ 韩文,字贯道,成化二年举进士,除工科给事中,出为湖广右参议。中贵督太和山,干没公费。文力遏之,以其羡易粟万石,备振贷。九溪土酋与邻境争地相攻,文往谕,皆服。弘治十六年拜南京兵部尚书。岁侵,米价翔踊。文请预发军饷三月,户部难之。文曰:“救荒如救焚,有罪,吾自当之。”乃发廪十六万石,米价为平。明年召拜户部尚书。文凝厚雍粹,居常抑抑。至临大事,刚断无所挠。武宗即位,赏赉及山陵、大婚诸费,需银百八十万两有奇,部帑不给。文请先发承运库,诏不许。文言:“帑藏虚,赏赉自京边军士外,请分别给银钞,稍益以内库及内府钱,并暂借勋戚赐庄田税,而敕承运库内官核所积金银,著之籍。且尽罢诸不急费。”旧制,监局、仓库内官不过二三人,后渐添注,或一仓十余人,文力请裁汰。淳安公主赐田三百顷,复欲夺任丘民业,文力争乃止。文司国计二年,力遏权幸,权幸深疾之。而是时青宫旧奄刘瑾等八人号“八虎”日导帝狗马鹰兔歌舞角抵不亲万几文每退朝对僚属语及辄泣下郎中李梦阳进曰:“公诚及此时率大臣固争,去‘八虎’易易耳。”文捋须昂肩,毅然改容曰:“善。纵事勿济,吾年足死矣,不死不足报国。”即偕诸大臣伏阙上疏,疏入,帝惊泣不食,瑾等大惧。瑾恨文甚,日令人伺文过。逾月,有以伪银输内库者,遂以为文罪。诏降一级致仕。瑾恨未已,坐以遗失部籍,逮文下诏狱。数月始释,罚米千石输大同。寻复罚米者再,家业荡然。瑾诛,复官,致仕。嘉靖五年卒,年八十有六。‎ ‎(节选自《明史·韩文传》)‎ ‎1.对下列句子中加点的词的解释,不正确的一项是(  )‎ A.以其羡易粟万石,备振贷  振:救济 B.救荒如救焚,有罪,吾自当之 当:承担 C.至临大事,刚断无所挠 临:面对 D.核所积金银,著之籍 著:彰显 答案 D 解析 著:登记。‎ ‎2.对文中画波浪线部分的断句,正确的一项是(  )‎ A.而是时青宫旧奄刘瑾等八人/号“八虎”日导帝/狗马/鹰兔/歌舞/角抵/不亲万几/文每退朝/对僚属语及/辄泣下/‎ B.而是时青宫旧奄刘瑾等八人/号“八虎”日导帝/狗马/鹰兔/歌舞/角抵/不亲万几/文每退朝对僚属/语及辄泣下/‎ C.而是时青宫旧奄刘瑾等八人号“八虎”/日导帝狗马/鹰兔/歌舞/角抵/不亲万几/文每退朝/对僚属语及/辄泣下/‎ D.而是时青宫旧奄刘瑾等八人号“八虎”/日导帝狗马/鹰兔/歌舞/角抵/不亲万几/文每退朝对僚属/语及辄泣下/‎ 答案 C 解析 根据文意应在“刘瑾等八人号‘八虎’”后停顿,排除A、B两项;“文每退朝”与“对僚属语及”间应停顿,故排除D项。‎ ‎3.下列对原文有关内容的概括和分析,不正确的一项是(  )‎ A.韩文为官清正,关注民众生活。他在湖广,妥善处理九溪土酋与邻境争地一事;担任南京兵部尚书时,年成歉收,他开仓取粮十六万石,平抑米价。‎ B.韩文刚正不屈,敢于奏议国事。武宗继位,诸项费用供给不足,他不顾非议,一再提出自己的看法;有关机构冗员渐增,他援引成例,着手压缩编制。‎ C.韩文疾恶如仇,尽力遏制权幸。宦官刘瑾等每日引诱皇上沉溺于声色狗马,不理政事,他采用李梦阳的建议,冒死谏诤,打击了刘瑾等的嚣张气焰。‎ D.韩文刚者易折,饱受政敌陷害。刘瑾以遗失部籍作为罪名,逮捕韩文,释放后又两次罚米,使他倾家荡产;直到刘瑾被诛后,韩文才复官而后退休。‎ 答案 B 解析 “他不顾非议”文中没有提及,只是说“诏不许”,即(皇帝)下诏不允许这样做。‎ ‎4.把文中画横线的句子翻译成现代汉语。‎ ‎(1)淳安公主赐田三百顷,复欲夺任丘民业,文力争乃止。‎ 译文:_________________________________________________‎ ‎___________________________________________________________‎ ‎(2)即偕诸大臣伏阙上疏,疏入,帝惊泣不食,瑾等大惧。‎ 译文:_________________________________________________‎ ‎___________________________________________________________‎ 答案 (1)淳安公主受赐田地有三百顷,又想强夺任丘民众的产业,因韩文尽力相争才停止。‎ ‎(2)当即与各位大臣一道拜伏宫阙上奏,奏章呈进,皇上惊哭不食,刘瑾等人大为恐惧。‎ 解析 (1)“赐田”含被动义;复,又;民业,民众的产业;力,尽力;乃,才。‎ ‎(2)偕,一道;伏阙,拜伏宫阙;疏入,奏章呈进。‎ 参考译文:‎ 韩文,字贯道,成化二年考取进士,被授予工科给事中一职,出任湖广右参议。有一个朝中权贵督察太和山,贪污公费。韩文竭力阻止他,用其财政盈余换取一万石米,准备用来救济借贷。九溪当地的酋长和邻境因争土地相互攻打,韩文前往晓谕,双方都称服。弘治十六年被授予南京兵部尚书。当年粮食歉收,米价飞涨。韩文请求先发三个月军饷,户部感到为难。韩文说:“救灾荒如同救火,若有罪,我自愿承担它。”他打开粮仓发放粮食十六万石,米价最终平定下来。第二年被授予户部尚书。韩文稳重平和,平时谦卑。但面对大事,刚毅果断,从不屈服。武宗登上皇位后,赏赐和修墓、大婚等费用,需要白银一百八十多万两,国库供应不起。韩文请求先打开承运库,(皇帝)下诏不允许这样做。韩文说:“国库空虚,赏赐除京边军士外,其余分别发放银钞,这样稍微增加内库及内府的钱,同时借功臣权贵的庄田税,又下令把承运库官员核实积存的金银,登记在籍,并且去掉不急的费用。”按旧制,监局、仓库官员不超过二三人,后来渐渐增加,有的仓库达十余人,韩文竭力上疏请求裁员淘汰。淳安公主受赐田地有三百顷,又想强夺任丘民众的产业,因韩文尽力相争才停止。韩文掌管国家财政两年,竭力遏制权贵,权贵们非常痛恨他。在当时青宫旧阉刘瑾等八人,号称“八虎”,他们每天引诱皇帝走狗跑马、放鹰猎兔、唱歌跳舞、摔跤游戏,致使皇帝不理国事。韩文每次退朝后,就对下属同僚说起这些,总是止不住落泪。郎中李梦阳进言:“韩公您如果能在此时率诸位大臣坚持抗争,除去‘八虎’(就)容易了。”韩文捋着胡须,耸耸肩膀,神情毅然而坚定地说:“好。这样做即使无济于事,我这把年纪死也值了,不死,也不足以报国。”当即与各位大臣一道拜伏宫阙上奏,奏章呈进,皇帝惊哭不食,刘瑾等人大为恐惧。刘瑾非常痛恨韩文,每天派人监视,尽力搜索韩文过失。过了一个月,有人给国库进纳假银,于是,刘瑾抓住这事治了韩文的罪。(皇上)下诏降韩文一级并免官。刘瑾愤恨难平,又以遗失部籍的罪名,逮捕韩文下狱。数月后获释,罚一千石米运送到大同。不久又一次罚米,致使(韩文)家业荡尽。等刘瑾被诛杀,韩文才恢复官职,后来退休。嘉靖五年韩文去世,享年八十六岁。‎ ‎[2年全国模拟重组]‎ 一、[2017·黄冈调研]阅读下面的文言文,完成1~4题。‎ 苏易简,字太简,梓州铜山人。易简少聪悟好学,风度奇秀,才思敏赡。太平兴国五年,年逾弱冠 ‎,举进士。太宗方留心儒术,贡士皆临轩覆试。易简所试三千余言立就,奏上,览之称赏,擢冠甲科。解褐将作监丞。雍熙二年,与贾黄中同知贡举。易简幼时随父河南,贾黄中来使,尝教之属辞;及是,悉为同列。‎ 淳化元年,丁外艰。易简续唐李肇《翰林志》二卷以献,帝赐诗以嘉之。他日,易简直禁中,以水试欹器。上密闻之,因晚朝,命取试之。易简奏曰:“臣闻日中则昃,月满则亏,器盈则覆,物盛则衰。愿陛下持盈守成,慎终如始,以固丕基,则天下幸甚。”会郊祀,充礼仪使。先是,扈蒙建议以宣祖升配。易简引唐故事,请以宣祖、太祖同配。从之。知审官院,言初任京朝官,未尝历州县,不得拟知州、通判。诏可。改知审刑院,俄掌吏部选,迁给事中、参知政事。至道二年,卒,年三十九,赠礼部尚书。‎ 易简外虽坦率,中有城府。由知制诰入为学士,年未满三十。属文初不达体要,及掌诰命,颇自刻励。在翰林八年,太宗遵旧制,且欲稔其名望而后正台辅,易简以亲老急于进用,因亟言时政阙失,遂参大政。‎ 蜀人何光逢,易简之执友也,尝任县令,坐赂削籍,流寓京师。会易简典贡部,光逢代人充试以取赀,易简于稠人中屏出之。光逢遂造谤书,斥言朝廷事,且讥易简。易简得其书以闻,逮捕光逢,狱具,坐弃市。易简以杀光逢非其意,居常怏怏。母薛氏以杀父执切责之,易简泣曰:“不谓及此,易简罪也。”及易简参知政事,召薛氏入禁中,问曰:“何以教子成此令器?”对曰:“幼则束以礼让,长则教以诗书。”上顾左右曰:“真孟母也。”‎ ‎(节选自《宋史·苏易简传》)‎ ‎1.对下列句子中加点的词的解释,不正确的一项是(  )‎ A.雍熙二年,与贾黄中同知贡举  知:主持 B.贾黄中来使,尝教之属辞  属:写作 C.且欲稔其名望而后正台辅  稔:成熟 D.逮捕光逢,狱具,坐弃市  狱:监牢 答案 D 解析 狱:官司。‎ ‎2.下列对文中加点词语的相关内容的解说,不正确的一项是(  )‎ A.源于周朝的“冠礼”是中国古代的成年礼。古代无论男女到了二十岁都要行“加冠之礼”,以示成人,但因尚未至壮年,故称“弱冠”。‎ B.丁艰,即丁忧,指遭逢父母的丧事,子女礼须持丧三年,官员须停职守制。丁外艰,旧指父丧或承重祖父之丧;丁内艰,则是指母丧。‎ C.削籍,指削除官籍中的名氏,籍,指做官名册。革职,只是革去现有职务,有重新起用的可能;削籍,则意味着除名,意味着永不录用了。‎ D.弃市是一种在闹市执行死刑并将犯人暴尸街头的刑法。古代死刑种类众多,如车裂、腰斩、枭首等,弃市为死刑的一种,商周时期就有。‎ 答案 A 解析 古代无论男女到了二十岁都要行“加冠之礼”有误,只有男子到了二十岁才行“加冠之礼”。‎ ‎3.下列对原文有关内容的概括和分析,不正确的一项是(  )‎ A.苏易简聪明好学,二十岁就考取进士,才思敏捷,临殿复试时,很快完成了三千字的答卷,后来,他和曾经教他作文的贾黄中一同主持贡举考试。‎ B.苏易简的才能得到皇帝的赏识,他续写《翰林志》,皇上赐诗嘉奖;皇上认可他关于郊祀和京中朝官任用的建议,让他掌管吏部选官,升为参知政事。‎ C.皇上听说苏易简好奇心强,喜欢“欹器”这件事后,劝诫他要“慎终如始”,不要玩物丧志。苏易简在翰林任职多年,太宗准备等他名望大了再委以重任。‎ D.挚友何光逢因贿赂丢官,客居京城时替人代考,被苏易简清除出来,于是怀恨在心,捏造谤书,因苏易简的举报被定罪诛杀,苏易简因此受到母亲的指责。‎ 答案 C 解析 理解有误。原文是说苏易简借实验欹器这件事情劝诫皇上,告诫皇上要“持盈守成,慎终如始,以固丕基”,而非皇上劝诫苏易简。‎ ‎4.把文中画横线的句子翻译成现代汉语。‎ ‎(1)会郊祀,充礼仪使。先是,扈蒙建议以宣祖升配。易简引唐故事,请以宣祖、太祖同配。‎ 译文:_________________________________________________‎ ‎___________________________________________________________‎ ‎(2)召薛氏入禁中,问曰:“何以教子成此令器?”对曰:“幼则束以礼让,长则教以诗书。”上顾左右曰:“真孟母也。”‎ 译文:_________________________________________________‎ ‎___________________________________________________________‎ 答案 (1)遇上郊祀典礼,苏易简担任礼仪使。这之前,扈蒙建议把宣祖升到配祀。苏易简援引唐朝旧例,请以宣祖、太祖一同享祀。‎ ‎(2)(皇上)召薛氏进入宫中,问(薛氏)说:“(你)用什么方法教育孩子成为这样有用的人才?”(薛氏)回答说:“年幼时则用守礼谦让约束他,年长后则用《诗经》、《尚书》教育他。”皇上看着身边大臣说:“真如孟母啊!”‎ 参考译文:‎ 苏易简,字太简,梓州铜山人。易简年幼时聪明好学,风度奇秀,才思敏捷。太平兴国五年,年满二十,考中进士。太宗正留心儒术,贡士都临殿复试。易简所作答卷三千多字很快写成,进奏皇上,(太宗)看后称善,选拔为甲科之首。授职将作监丞。雍熙二年,与贾黄中一同主持贡举。易简年幼时随父亲到河南,贾黄中任使者来河南,曾经教他作文;到此时,成为同列官吏。‎ 淳化元年,遭父丧。苏易简续撰唐朝李肇的《翰林志》二卷献给皇上,皇上赐诗嘉奖他。有一天,苏易简在宫中值班,用水来试验欹器(古代一种计时工具)。皇上暗地听说这件事,趁晚朝的时候,命令(苏易简)取来试玩。苏易简上奏说:“我听说太阳到了正午就要偏西,月亮盈满就要亏缺,器物装满了就会倾覆,事物繁盛后就将衰败。希望陛下保持已成就的盛业,谨慎收尾,如同开始时一样,来加固宏大的基业,那么天下就非常幸运了。”遇上(碰上、恰逢)郊祀典礼,(易简)充任(担任)礼仪使。这之前(先前),扈蒙建议把宣祖升到配祀。易简援引唐朝旧例(先例),请以宣祖、太祖一同享祀。(皇上)听从了他的意见。主持审官院,建议初任京中朝官,从未历任州县职的,不得任为知州、通判。下达诏书同意。改为主持审刑院,不久掌管吏部选官,升为给事中、参知政事。至道二年去世,终年三十九岁,追赠礼部尚书。‎ 苏易简表面虽然坦诚直率,心中却有城府。由知制诰入翰林为学士,年纪不满三十岁。写文章开始不达要点,到掌管诰命,非常刻苦磨炼。在翰林任职八年,太宗遵照旧有制度,想等他名望成熟后任宰相,苏易简因为双亲年老急于被选拔任用,因为及时谈论时政得失,于是参与大政。‎ 蜀人何光逢,是苏易简的挚友,曾经担任过县令,因贿赂被削除官籍,客居京城。正遇上苏易简主管贡部,何光逢替人代试以收取费用,苏易简在考试的人群中把他清除出来。何光逢于是捏造谤书,指斥朝廷政事,并且讥刺苏易简。苏易简获得他的谤书告诉皇上,逮捕何光逢,官司审结,定罪诛杀。苏易简因杀何光逢不是本意,平时常常怏怏不乐。其母薛氏以杀死父亲挚友严厉责备他,苏易简哭着说:“本认为不会致死,是我的罪过啊!”等到苏易简任参知政事后,(皇上)召薛氏进入宫中,问(薛氏)说:“(你)用什么方法教育孩子成为这样有用的人才?”(薛氏)回答说:“年幼时则用礼仪谦让约束他,年长后则用《诗经》《尚书》教育他。”皇上看着身边大臣说:“真如孟母啊!”‎ 二、[2017·广东四校联考]阅读下面的文言文,完成1~4题。‎ 罗洪先,字迭夫,吉水人。父循,进士。历兵部武选郎中。会考选武职,有指挥二十余人素出刘瑾门,循罢其管事。瑾怒骂尚书王敞,敞惧,归邵趣易奏。循故迟之,数日瑾败,敞乃谢循。循历知镇江、淮安二府,徐州兵备副使,成有声。‎ 洪先幼慕罗伦为人。年十五,读王守仁《传习录》,好之,欲往受业,循不可而止。乃师事同邑李中,传其学。嘉靖八年,举进士第一,授修撰,即请告归。外舅太仆卿曾直喜曰:“幸吾婿成大名。”洪先曰:“儒者事业有大于此者。此三年一人,安足喜也?”洪先事亲孝。父每肃客,洪先冠带行酒、拂席、授几甚恭。居二年,诏劾请告逾期者,乃赴官。寻遭父丧,苫堆蔬食,不入室者三年。继遭母忧,亦如之。‎ 十八年,简宫僚,召拜春坊左赞善。明年冬,与司谏唐顺之、校书赵时春疏请来岁朝正后,皇太子出御文华殿,受群臣朝贺。时帝数称疾不视朝,讳言储贰临朝事,见洪先等疏,大怒曰:“是料朕必不起也。”降手诏百余言切责之,遂除三人名。‎ 洪先归,益寻求守仁学。甘淡泊,炼寒暑,跃马挽强,考图观史,自天文、地志、礼乐、典章、河渠、边塞、战阵攻守,下逮阴阳、算数,扉不精究。至人才、吏事、国计、民情,悉如意谘访。曰:“苟当其任,皆吾事也。”邑田赋多宿弊,请所司均之,所司即以属。洪先精一心体察,弊顿除。岁饥,移书郡邑,得粟数十石,率友人躬振给。流寇入吉安,主者失措,为画策战守,寇引去。素与顺之友善,顺之应召,被挽之出,严嵩以同乡故,拟假边才起用,皆力辞。‎ 初,告归,过仪真,同年生主事项乔为分司。有富人生死,行万金求为地,洪先拒不听。乔微讽之,厉声曰:“君不闻志士不忘在沟壑耶,”江涨,坏其室,巡抚马森欲为营之,固辞不可。隆庆初卒,赠光禄少卿,谥文庄。‎ ‎(节选自《明史·罗洪先传》)‎ ‎1.对下列句子中加点的词的解释,不正确的一项是(  )‎ A.乃师事同邑李中,传其学  传:学习 B.父每肃客  肃:庄重 C.十八年,简宫僚  简:选拔 D.率友人躬振给  振:通“赈”,救济 答案 B 解析 肃:迎接。‎ ‎2.下列对文中相关内容的解说,不正确的一项是(  )‎ A.文中多有表示官员授职升迁的词,如“授修撰”中的“授”,“召拜春坊左赞善”中的“拜”,“除三人名”中的“除”等,都是表示授予官职的。‎ B.“进士”指中国古代科举制度中通过殿试的人。明清科举考试分三级进行,其中殿试一般每三年进行一次,文中“三年一人”即可印证。‎ C.王守仁是明代著名思想家,精通儒、释、道三教,并能够统军征战,是中国历史上罕见的全能大儒。罗洪先涉猎广泛且精心研究正是对王守仁为学的继承和学习。‎ D.守丧在中国古代文化中是一种孝顺的行为,明清时期官员的父母死去,官员必须停职守制,守丧期一般为三年,“苫块蔬食,不入室者三年”即守丧之礼。‎ 答案 A 解析 “除”在这里为削除官职。‎ ‎3.下列对原文有关内容的概括和分析,不正确的一项是(  )‎ A.罗洪先志向远大。他幼年仰慕罗伦的为人,后又想拜王守仁为师,因父亲没有同意才罢休;他认为考中进士并不值得炫耀。‎ B.罗洪先心忧国民。他留心察访人才、吏事、国计、民情,发现田赋弊端,请求均衡处理并使积弊消除;流寇侵入吉安,他献策战守并使盗贼退兵离去。‎ C.罗洪先与唐顺之交情很好,唐顺之被召用了。严嵩因为与罗洪先是同乡,打算借用边疆的人才来起用。‎ D.罗洪先正直淡泊。他拒不同意犯罪富人拿钱请求免死的行为;因江水上涨,屋子被毁,他坚决推辞巡抚马森为其营造房屋的好意。‎ 答案 C 解析 文中说“严嵩以同乡故,拟假边才起用”,是说打算假托治理边疆的人才起用。‎ ‎4.把文中画横线的句子翻译成现代汉语。‎ ‎(1)“儒者事业有大于此者。此三年一人,安足喜也?”洪先事亲孝。‎ 译文:_________________________________________________‎ ‎(2)时帝数称疾不视朝,讳言储贰临朝事,见洪先等疏,大怒曰:“是料朕必不起也。”‎ 译文:_________________________________________________‎ 答案 (1)“读书人的事业有比这个更大的。这种人三年就一个,有什么(哪里或怎么)值得高兴的?”罗洪先侍奉双亲很孝顺。‎ ‎(2)当时皇帝多次声称有病不上朝,忌讳臣子说让太子临朝的事,见到罗洪先等人的奏疏,非常愤怒地说:“这是料定朕卧病必定不能好了。”‎ 参考译文:‎ 罗洪先,字迭夫,吉水人。父亲罗循,是进士。历任兵部武选郎中。恰逢考核选拔武官,有二十多个指挥平素出自刘瑾门下,罗循罢除其管事。刘瑾愤怒地骂尚书王敞,王敞害怕,回到兵部催促改变奏折。罗循故意拖延,几天后刘瑾倒台,王敞于是向罗循道歉。罗循历任镇江、淮安两府的知府和徐州兵备副使,都有治政的名声。‎ 罗洪先幼年仰慕罗伦的为人。十五岁时,读王守仁的《传习录》,喜欢上它,想前往拜师求学,罗循没有同意而罢休。于是拜同乡的李中为师,学习李中的学说。嘉靖八年,考中进士第一名,授官修撰,马上请求辞官回家。岳父太仆卿曾直高兴地说:“有幸我的女婿成就了大名声。”罗洪先说:“读书人的事业有比这个更大的。这种人三年就一个,有什么值得高兴的?”罗洪先侍奉双亲很孝顺。父亲每次迎接客人,罗洪先都穿上礼服依次给客人斟酒、拂扫座席、端授茶几,非常恭敬。过了两年,下诏弹劾请假超过期限的官员,他才去任职。不久遭遇父亲的丧事,寝苫枕块,吃素食,不回家中,这样过了三年。接着又遇上母亲的丧事,也如此守丧。‎ 十八年,选拔东宫的官员,召任春坊左赞善。第二年冬天,与司谏唐顺之、校书赵时春上疏请求来年朝贺正旦日后,皇太子出宫亲临文华殿,接受群臣朝贺。当时皇帝多次声称有病不上朝,忌讳臣子说让太子临朝的事,见到罗洪先等人的奏疏,非常愤怒地说:“这是料定朕卧病必定不能好了。”降下亲手写的一百多字的诏令责备他们,于是削除三人的名籍。‎ 罗洪先回到家中,更加寻求王守仁的学问。甘于淡泊,锻炼于寒冬酷暑,策马腾跃,拉引强弓,查考地图翻阅历史,上自天文、地理、礼乐、典章、河渠、边塞、打仗的阵法和攻守方式,下至阴阳、算数,无不精心研究。至于人才、吏治之事、国家大计、百姓的情况,都用心咨询访问。还说:“如果身在其位,都是我的事。”县里的田赋有很多积久的弊病,请求有关部门均衡处理,有关部门就交托他来处理。罗洪先细心考察,积弊马上除去。收成不好,他转交书信到郡县,得到粮食几十石,都是友人亲自拿出救济的。流窜的盗贼进入吉安,主事官惊慌失措,(罗洪先)为他出谋划策迎战守卫,盗贼退兵离去。他向来与唐顺之友好,唐顺之应召入朝,想引他出山,严嵩因是同乡的缘故,打算假托治理边疆的人才起用他,他都极力推辞。‎ 当初,请假回家,经过仪真,同年考中进士现任主事的项乔在盐运司任职分管。有富人犯罪定为死罪,用一万两银子请求为自己开脱罪名,罗洪先拒不同意。项乔暗示劝告,他严厉地说:“您没听说有志之士不忘记身处沟壑吗?”江水上涨,毁坏了他的屋子,巡抚马森想为他营造,他坚决推辞不同意。隆庆初年死去,赠官光禄少卿,缢号文庄。‎ 三、[2017·湖南五市十校联考]阅读下面的文言文,完成1~4题。‎ 吴少诚,幽州潞县人。父为魏博节度都虞候。少诚以父勋授一子官,释褐王府户曹。后至荆南,节度使庾准奇人,留为衙门将。准入觐从至襄汉见梁崇义不遵宪度知有异志少诚密计有成擒之略将自陈于阙下。属李希烈初授节制,锐意立功,见少诚计虑,乃以少诚所见录奏,有诏慰饬,不次封通义郡王。未几,崇义违命,希烈受制专征,以少诚为前锋。崇义平,赐实封五千户。后希烈叛,少诚颇为其用。希烈死,少诚等初推陈仙奇统戎事,朝廷已命仙奇,寻为少诚所杀,众推少诚知留务。朝廷遂授以申光蔡等州节度观察兵马留后[注],寻正授节度。‎ 少诚善为治,勤俭无私,日事完聚,不奉朝廷。贞元三年,判官郑常及大将杨冀谋逐少诚以听命于朝,试校书郎刘涉假为手诏数十,潜致于大将,欲因少诚之出,闭城门以拒之。属少诚将出饯中使,常、冀等遂谋举事,临发,为人所告,常、冀先遇害。其将李嘉节等各持假诏请罪,少诚悉宥之。其大将宋炅、曹齐奔归京师。‎ 十五年,陈许节度曲环卒,少诚擅出兵攻掠临颍县,节度留后上官况遣兵赴救,临颖镇使韦清与少诚通,救兵三千余人,悉擒缚而去。九月,遂围许州。寻下诏削夺少诚官爵,分遣十六道兵马进讨。十二月,官军败衄于小溵河。明年七月,韩全义顿军于五楼行营,为贼所乘,大溃。全义与都监军使贾秀英、贾国良等夜遁,遂城守溵水。汴宋、徐泗、淄青兵马直趣陈州,列营四面。少诚兵逼溵水五六里下营。韩全义诸军又退保陈州。少诚寻引兵退归蔡州。顺宗即位,加同中书门下平章事。元和初,迁检校司空,依前平章事。元和四年十一月卒,年六十,废朝三日,赠司徒。‎ ‎(选自《旧唐书·列传第九十五》,有删节)‎ ‎[注] 留后:唐代节度使、观察使缺位时设置的代理职称。‎ ‎1.对下列句子中加点的词的解释,不正确的一项是(  )‎ A.释褐王府户曹 释褐:出任 B.众推少诚知留务 知:主管 C.潜致于大将 潜:潜藏 D.少诚寻引兵退归蔡州 引:率领 答案 C 解析 此类题目一定要在答题时把实词放回到原文中结合具体语境和上下文来理解。潜:暗中。‎ ‎2.对文中画波浪线部分的断句,正确的一项是(  )‎ A.准入觐/从至襄汉/见梁崇义不遵宪度/知有异志/少诚密计有/成擒之略/将自陈于阙下 B.准入觐/从至襄汉/见梁崇义不遵宪度知/有异志/少诚密计有成擒之略/将自陈于阙下 C.准入觐从/至襄汉/见梁崇义不遵宪度/知有异志/少诚密计有成擒之略/将自陈于阙下 D.准入觐/从至襄汉/见梁崇义不遵宪度/知有异志/少诚密计有成擒之略/将自陈于阙下 答案 D 解析 “入觐”为进京朝见之意,故应停顿,排除C项。“不遵宪度”后停顿属常识性知识,可知B项错。“密计”与“有”为并列谓语,不能断开,A项错。‎ ‎3.下面对文章内容的理解和分析不正确的一项是(  )‎ A.吴少诚,幽州潞县人。他因为父亲曾担任魏博节度都虞候期间立有战功,出任了王府户曹。‎ B.李希烈死后,吴少诚等人最初推举陈仙奇统管军务。而陈仙奇被朝廷任命以后,不久就被吴少诚杀害。‎ C.郑常、杨冀等人谋划着趁吴少诚出城为宦官使者饯行。时实施赶走吴少诚的行动,但因为走漏风声,最终失败。‎ D.临颍镇节度使韦清与吴少诚相互勾结,从而让上官况派去的三千多救兵全部被生擒捆绑。‎ 答案 A 解析 A项,文中只说他是因为父亲立下战功而被授予的官职,并没有明确他父亲是在担任魏博节度都虞候期间立下的战功。解答此类题的思路是:先抓住题干,读全读准。在阅读题目时,需读全、读准题干,切忌走马观花。所谓读全,就是对题干中的所有要求要一个不漏、原原本本地分析;所谓读准,就是要准确地把握题干所提的要求,看清是选对的还是选错的,是概括内容还是分析观点。只有对题干作全面、准确的分析理解,才能准确地答题。再把选项放回原文,查对正误。特别是在时间、地点、官职,人物的行为、实效方面,应仔细查对原文的词句,全面理解,综合分析,两者间的差别正是把握全文的关键所在。对似是而非处,要有借题解文的意识。‎ ‎4.把文中画横线的句子翻译成现代汉语。‎ ‎(1)少诚善为治,勤俭无私,日事完聚,不奉朝廷。‎ 译文:_________________________________________________‎ ‎(2)明年七月,韩全义顿军于五楼行营,为贼所乘,大溃。‎ 译文:_________________________________________________‎ 答案 (1)吴少诚擅长治理军政事务,勤俭无私,终日只是修葺城郭积聚粮食,不听从朝廷命令。‎ ‎(2)第二年七月,韩全义在五楼行营驻军,被叛军偷袭,大败。‎ 解析 此类题目在翻译时首先要找出关键字或句式进行翻译,一般为直译,除一些带有比喻性的词语然后再整体翻译,并按现代汉语的规范,达到词达句顺。关键词有:善、日、事、为……所、溃。‎ 参考译文:‎ 吴少诚,幽州潞县人。他的父亲担任魏博节度都虞候。吴少诚因父亲立有战功可以授任儿子一个官职,出任王府户曹。后来来到荆南,节度使庾准很赏识他,留下任衙门将。庾准进京朝见,他跟随到襄汉,见梁崇义不遵守纲纪,知道他有异心,吴少诚秘密谋划捉拿他的计策,准备亲自上奏朝廷。恰逢李希烈刚被授任节度使,一心想着立功,见到吴少诚的计策,就将吴少诚的见识记述下来上奏了皇上,皇上下诏书予以慰劳,破格封他为通义郡王。没多久,梁崇义违抗命令,李希烈受命专门征讨梁崇义,让吴少诚为先锋。梁崇义被平定以后,(朝廷)给吴少诚赐实封五千户。后来李希烈反叛,吴少诚特别受到重用。李希烈死后,吴少诚等人起初推举陈仙奇统管军务,朝廷任命陈仙奇以后,不久又被吴少诚杀害,众人推举吴少诚主管留后事务。朝廷于是又授任他为申光蔡等州节度观察兵马留后,不久正式授任他为节度使。‎ 吴少诚擅长治理军政事务,勤俭无私,终日只是修葺城郭积聚粮食,不听从朝廷命令。贞元三年,判官郑常以及大将杨冀密谋赶走吴少诚来听命朝廷,试校书郎刘涉伪造数十份手诏,暗中发给大将,打算借吴少诚外出的机会,关闭城门来抵抗他。当吴少诚准备出城为宦官使者饯行时,郑常、杨冀等人谋划着准备行动,临近起事,被人告发,郑常、杨冀首先遇害。手下将领李嘉节等人各带假诏书请求处罚,吴少诚全部宽免了他们。他们的手下大将宋炅、曹齐逃回京城。‎ 十五年,陈许节度使曲环死了,吴少诚擅自出兵进攻抢掠临颍县,节度使留后上官况派兵前去救援,临颍镇节度使韦清与吴少诚相互勾结,(上官况派去的)救兵三千余人,全部被他们生擒捆绑而去。九月,又围困了许州。不久(皇上)发下诏命削除吴少诚的官爵,分派十六道兵马进击讨伐。十二月,官军在小溵河战败。第二年七月,韩全义在五楼行营驻军,被叛军偷袭,大败。韩全义与都监军使贾秀英、贾国良等人连夜逃跑,退驻到溵水筑城守备。汴宋、徐泗、淄青的兵马直趋陈州,在四面布列营寨。吴少诚军队在逼近溵水五六里处扎下营寨。韩全义等军队又退守陈州。吴少诚不久率军退回蔡州。顺宗即位后,加授吴少诚同中书门下平章事。元和初年,升任检校司空,依旧做平章事。元和四年十一月去世,时年六十岁,朝廷为他停止朝会三天,追赠司徒。‎ 滚动提升训练(三)‎ ‎  时间:60分钟   满分:75分 一、语言文字运用(18分)‎ ‎1.[2017·湖南六校联盟联考]下列各句中加点成语的使用,全部正确的一项是(3分)(  )‎ ‎①相较于享誉中外的佛教圣地九华山,紫蓬山似乎名不见经传,但谁又能想到,历史上,它曾是淮军将领的摇篮。‎ ‎②代沟之说,有相当的道理。不过,这条代沟如何沟通,只好潜移默化,子女对父母未便耳提面命。‎ ‎③小时候的张衡半夜数星星,说自己长大后要成为一名科学家,没想到一语成谶,后来的他成了汉代著名的天文历法大家。‎ ‎④从中国书法史看,“书以人名”似乎成为了一条重要规律。但不容置喙的是,真正的艺术作品的生命精神才是它的根本所在。‎ ‎⑤历史专业出身的石晓明,不愿像其他同学一样埋首钻研,皓首穷经,著书立说,而是毅然选择了与自己专业相去甚远的工作,实在让人匪夷所思。‎ ‎⑥《本草图经》中的年代较早的一些版画,因为出于各地画师之手,风格判若云泥,精粗详略也有差异,但总体水平较高。‎ A.①②⑤ B.③④⑤ C.①②⑥ D.③④⑥‎ 答案 A 解析 ①名不见经传:名字在古籍中没有记载,指人或事物没有什么名气。用在此处符合语境。②耳提面命:形容严格要求,殷切教诲。此处使用正确。③一语成谶:一句不好的话说中了。感情色彩不当。④不容置喙:指不容许别人插嘴说话。用在此处不妥,可改为“毋庸置疑”。⑤皓首穷经:指钻研儒家经典到老,也泛指勤奋苦读一生。使用正确。⑥判若云泥:高低差别就像天上的云彩和地下的泥土那样远,形容高低悬殊。本句用来说风格不同,不合适。‎ ‎2.[2017·肇东一中段考]下列各句中,没有语病、句意明确的一项是(3分)(  )‎ A.白皮书说,中国人民在南海的活动已有2000多年历史。中国最早发现、命名和开发利用南海诸岛及相关海域,最早并持续、和平、有效地对南海诸岛及相关海域行使主权。‎ B.土耳其击落俄罗斯战机,俄全力开动宣传机器,试图将土耳其与IS“暗通款曲”的种种罪名坐实,令他在国际舆论场上观感不佳。‎ C.虽然学校三令五申,要求学生自觉爱护学校环境,但是随手乱丢垃圾,人走关灯,损坏桌椅等现象依然十分严重,令人担忧。‎ D.在“大学生掏鸟16只被判10年半”一案中,官方披露的证据显示,不仅嫌疑人利用QQ群、百度贴吧出售猎物,还能准确介绍猎物名字、习性,“不识国家保护动物”的说法早已站不住脚。‎ 答案 A 解析 B项,语病一:中途易辙。应该将“土耳其击落俄罗斯战机”修改为“在土耳其击落俄罗斯战机后”作为后半句的状语。语病二:“令他在国际舆论场上观感不佳”句中代词“他”指代不明,应该改为“土耳其”。C项,“人走关灯”与“十分严重”搭配不当。D项,语序不当,关联词“不仅”应该调整到“嫌疑人”之后。‎ ‎3.[2017·中原名校质量考评]填入下面文段空白处的词语,最恰当的一组是(3分)(  )‎ 我们常常用“司空见惯”比喻某事常见、不足为奇。__①__不少人会纳闷:“司空”究竟是什么意思呢?其实,__②__了解了这个成语的来历,__③__不难理解了:唐朝刘禹锡被贬为苏州刺史时,应一个曾任司空官职的李绅的邀约赴宴,__④__请来歌女作陪。刘禹锡__⑤__当即赋诗“司空见惯浑闲事,断尽苏州刺史肠”,其中“司空”两字是唐代的一种官职,相当于清代的尚书。__⑥__后人就常常用其诗中的“司空见惯”来形容对怪事已经见惯,不觉奇怪了。‎ ‎①‎ ‎②‎ ‎③‎ ‎④‎ ‎⑤‎ ‎⑥‎ A 然而 既然 则 ‎/‎ 从而 因此 B 却有 如果 那么 况且 因而 ‎/‎ C 但是 只要 就 并 ‎/‎ 而 D ‎/‎ 只有 才 而且 还 由此 答案 C 解析 注重分析语句之间的关系。①句,从与前文的关系来看,是上一层意思的转折,可以填写转折连词;②③两句,从“纳闷”“了解”分析,两句话是条件关系,可以填写“只要……就”等;④句,从“邀约赴宴”和“请来歌女”分析,二者是并列关系,可以填写“并”“并且”之类的词语;⑤句,是对上文的结果的表述,可以填写表示顺承的词语,比如“就”“于是”“然后”等,从“当即”的角度分析,也可以不填写词语;⑥句,是对前文的总结,可以填写表示总结的词语等。‎ ‎4.[2017·南阳一中检测]填入下面一段文字横线处的语句,最恰当的一句是(3分)(  )‎ 创业创新是人类文明进步的不熄引擎,是植根于每个人心中具有顽强生命力的“种子”。推动发展,________。我国是世界上人口最多的国家,13亿勤劳智慧的中国人民中间,蕴藏着无穷的创造力。试想一下,如果13亿人的创新创造潜能充分释放出来,那将给经济社会发展带来什么样的变化。‎ A.靠社会生产力的解放,更要靠解放社会创造力 B.既要解放社会生产力,又要社会创造力的解放 C.不仅要解放社会生产力,更要解放社会创造力 D.解放社会生产力的同时,还要解放社会创造力 答案 C 解析 结合文段内容看,横线后的文字强调的是“无穷的创造力”“创新创造潜能”,上文也就应强调“创造力”。从句式角度看“不仅……更”表示递进关系,在语气上更贴近。‎ ‎5.[2017·重庆南开中学模拟]下图展现了人们在学习知识时的记忆规律,请把图片展现的内容写成一段话,要求内容得当,表述准确,语言连贯,不超过85个字。(6分)‎ 答:___________________________________________________‎ ‎___________________________________________________________‎ 答案 (示例)输入的信息经过注意,便成了短时记忆,但如果不及时复习,会被遗忘,若经过及时复习,就会变成长时记忆。‎ 解析 本题考查图文转换的能力。考生解答此题,先要认真阅读此框架,读懂框架关系,看懂箭头的指向。“输入的信息”可变为“短时的记忆”;“短时的记忆”有两种走向:“遗忘”和“长时的记忆”;变为“长时的记忆”要经过“复习”这个过程。厘清逻辑关系后,将这些信息用简洁明了的语言表述出来即可。‎ 二、文言文阅读(57分)‎ ‎(一)[2017·海南七校联盟联考]阅读下面的文言文,完成6~9题。(19分)‎ 刘师道,字损之,开封东明人。雍熙二年举进士,初命和州防御推官,历保宁、镇海二镇从事,凡十年。王化基、吕祐之、乐史荐于朝,擢著作佐郎,才一月,会考课,又迁殿中丞,出知彭州,加监察御史。转运使刘锡、马襄上其治迹,召归。会浦洛之败,奉诏劾白守荣辈,狱成,太宗奖其勤,面赐绯鱼。‎ 川峡豪民多傍户,以小民役属者为佃客,使之如奴隶,家或数十户,凡租调庸敛,悉佃客承之。时有言李顺之乱,皆傍户鸠集,请择傍户为三耆长迭主之,酬岁劳则授以官,诏师道使两川议其事。师道以为迭使主领则争忿滋多,署以名级又重增扰害,廷奏非便,卒罢之。改祠部员外郎,出为京东转运使。真宗嗣位,进秩度支。咸平初,范正辞荐其材堪长民,徙知润州。以漕事入奏,特迁司封,改工部郎中。擢枢密直学士,掌三班。俄擢权三司使,从幸澶渊。‎ 师道弟几道,举进士礼部奏名,将廷试,近制悉糊名较等,陈尧咨当为考官,教几道于卷中密为识号。几道既擢第,事泄,诏落其籍,永不预举。师道固求辨理,诏曹利用、边肃、阎承翰诣御史府推治之。坐论奏诬罔,责为忠武军行军司马,尧咨免所居官,为郓州团练副使。以郊祀恩,起为工部郎中、知复州,换秀州。‎ 大中祥符二年,以兵部郎中知潭州,迁太常少卿。师道敏于吏事,所至有声,吏民畏爱。长沙当湖、岭都会,剖烦析滞,案无留事。岁满,复加枢密直学士,换左司郎中,留一任。大中祥符七年,师道暴病卒,年五十四。‎ 师道性慷慨尚气,善谈世务,与人交敦笃。工为诗,多与杨亿辈酬唱,当时称之。‎ ‎(选自《宋史·刘师道传》,有删改)‎ ‎6.对下列句子中加点的词的解释,不正确的一项是(3分)(  )‎ A.会浦洛之败,奉诏劾白守荣辈 会:恰逢 B.署以名级又重增扰害 署:委任 C.师道固求辨理 固:固执 D.诏曹利用、边肃、阎承翰诣御史府推治之 诣:到 答案 C 解析 固:坚持。‎ ‎7.下列对文中加点词语的相关内容的解说,不正确的一项是(3分)(  )‎ A.考课:按一定的标准对各级官吏的政绩进行考核,并根据官吏的不同表现,区别不同等级,给予赏罚。‎ B.嗣位:嗣位指继承君位,我国封建王朝通常实行嫡长子继承制,君位由正妻所生最年长的儿子继承。‎ C.工部:中国封建时代中央官署名,为掌管营造工程、土地、户籍等事宜的机关,部长官称为工部尚书。‎ D.幸澶渊:文中指皇帝到澶渊。封建时代皇帝到某处,叫“幸”。妃、嫔受皇帝宠爱,叫“得幸”。‎ 答案 C 解析 “土地、户籍”为户部所掌管。‎ ‎8.下列对原文有关内容的概括和分析,不正确的一项是(3分)(  )‎ A.刘师道被人推荐,受到赏识。王化基等人向朝廷举荐他,他被提升为著作佐郎;刘锡等人上报他的政绩;范正辞举荐他可以担任地方长官。‎ B.刘师道恪尽职守。他奉命弹劾白守荣等人,结案后,皇帝给予他奖赏;他被朝廷派往处理川峡棘手事务,他分析实情予以上报。‎ C.刘师道因弟弟的事被贬官。他的弟弟在进士考试中作弊,他坚持申辩,要求朝廷派曹利用等人调查,结果他和考官陈尧咨都被贬官。‎ D.刘师道有治理才干,也有文才。他勤于政事,所到之处有名声,受人敬畏、爱戴;他善于写诗,常和杨亿等人唱和,当时受到好评。‎ 答案 C 解析 “要求朝廷派曹利用等人调查”错,应是朝廷派曹利用等人调查。‎ ‎9.把文中画横线的句子翻译成现代汉语。(10分)‎ ‎(1)几道既擢第,事泄,诏落其籍,永不预举。(5分)‎ 译文:_________________________________________________‎ ‎(2)长沙当湖、岭都会,剖烦析滞,案无留事。(5分)‎ 译文:_________________________________________________‎ 答案 (1)刘几道考中之后,事情泄露,(朝廷)下诏将他从名册中取消,永远不得参加科举。(得分点:既、落、预各1分,句意2分)‎ ‎(2)长沙是湖、岭一带的都市,(刘师道)处理繁难和拖延下来的事务,桌上没有遗留的。(得分点:剖〈析〉、烦〈或“滞”〉、案各1分,句意2分)‎ 解析 翻译时要有关键词意识,本题中的关键词有:既,已经;落,取消;预,参加。剖,处理;烦,繁难的事务;案,桌。‎ 参考译文:‎ 刘师道,字损之,开封东明人。雍熙二年考中进士,最初担任和州防御推官,历任保宁、镇海二镇从事,共十年。王化基、吕祐之、乐史向朝廷举荐他,提升为著作佐郎,刚刚一个月,恰逢考核,又升为殿中丞,出朝担任彭州知州,加官监察御史。转运使刘锡、马襄报告他的政绩,又被召回京城。恰逢浦洛战败,奉朝廷命令弹劾白守荣等人,结案,太宗奖励他的勤勉,当面赏赐绯衣与鱼符袋。‎ 川峡的豪强有很多依附他们的人家,以小民的身份服役隶属的成为佃户,役使他们如同奴隶,有的豪强有几十户佃户,凡是租税徭役或征收各种费用,都由佃户承担。当时有人说李顺造反,都是这种依附的民户聚集而成,请求朝廷挑选这些民户中的人作为三耆长轮流管理,有功劳就任命为官,朝廷下诏刘师道出使两川讨论这事。刘师道认为轮流派人管理会使争斗之事增多,委任职衔又使民户加重受害,当廷上奏认为不妥,最终停止这么做。改任祠部员外郎,出朝担任京东转运使。真宗继位,晋级为度支。咸平初年,范正辞推荐他可以担任地方长官,调任润州知州。入京报告漕运之事,特升为司封,改任工部郎中。提拔为枢密直学士,掌管三班。不久提拔为代理三司使,跟随皇帝到澶渊。‎ 刘师道的弟弟刘几道,参加进士考试已被礼部上奏名字,将要举行廷试,按新近的规定全都要将试卷糊封姓名再考评等级,陈尧咨担任考官,教刘几道在试卷中暗加记号。刘几道考中之后,事情泄露,(朝廷)下诏将他从名册中取消,永远不得参加科举。刘师道坚持要求申辩,(朝廷)下诏曹利用、边肃、阎承翰到御史台调查这件事。(刘师道)因言语虚假,被贬为忠武军行军司马,陈尧咨被免去所担任的官职,贬为郓州团练副使。因郊祀恩赐,起用为工部郎中、复州知州,改任秀州知州。‎ 大中祥符二年,凭着兵部郎中的身份任潭州知州,升任太常少卿。刘师道对政事很勤奋,所到之处很有名声,官吏百姓敬畏爱戴。长沙是湖、岭一带的都市,(刘师道)处理繁难和拖延下来的事务,桌上没有遗留的。任职期满,又加官枢密直学士,改为左司郎中,留任一期。大中祥符七年,刘师道突然发病去世,享年五十四岁。‎ 刘师道生性刚直崇尚气节,善于谈论时势,与人交往诚实厚道。善于写诗,常和杨亿等人唱和,当时受到好评。‎ ‎(二)[2017·洛阳一中段考]阅读下面的文言文,完成10~13题。(19分)‎ 康保裔,河南洛阳人。祖志忠,讨王都战没。父再遇,从太祖征李筠,又死于兵。保裔在周屡立战功,及再遇阵没,诏以保裔代父职,从石守信破泽州,又从诸将破契丹于石岭关,领登州刺史。寻知代州,移深州,领凉州观察使。真宗即位,召还,以其母老勤养,赐以上尊酒茶米。诏褒之,为高阳关都部署。契丹兵大入,诸将与战于河间,保裔选精锐赴之,会暮,约诘朝合战。迟明,契丹围之数重,左右劝易甲驰突以出,保裔曰:“临难无苟免。”遂决战。二日,杀伤甚众,蹴践尘深二尺,兵尽矢绝,援不至,遂没焉,时车驾驻大名,闻之震悼,废朝二日,赠侍中。以其子继英为六宅使、顺州刺史,继彬为洛苑使。继英等谢曰:“臣父不能决胜而死,陛下不以罪其拏幸矣,臣等顾蒙非常之恩!”因悲涕伏地不能起。上恻然曰:“尔父王事,赠赏之典,所宜加厚。”顾谓左右曰:“保裔父、祖死疆场,身复战没,世有忠节,深可嘉也。”保裔谨厚好礼,喜宾客,善骑射,弋飞走无不中。尝握矢三十,引满以射,筈镝相连而坠,人服其妙。屡经战阵,身被七十创。贷公钱数十万劳军,没后,亲吏鬻器玩以偿。上知之,乃复厚赐焉。‎ 方保裔及契丹血战,而援兵不至,唯张凝、李重贵率众策应,遇契丹兵交战。保裔为敌所覆,重贵与凝赴援,腹背受敌,自申至寅力战,敌乃退。当时诸将多失部分,独重贵、凝全军还屯,凝议上将士功状,重贵喟然曰:“大将陷没,而吾曹计功,何面目也!”上闻而嘉之。‎ ‎(节选自《宋史·康保裔传》)‎ ‎10.对下列句子中加点的词的解释,不正确的一项是(3分)(  )‎ A.及再遇阵没  及:趁着 B.寻知代州,移深州  寻:不久 C.继英等谢曰  谢:感谢 D.屡经战阵,身被七十创  被:遭受 答案 A 解析 及:等到。‎ ‎11.下列对文中加点的词语的相关解说,不正确的一项是(3分)(  )‎ A.诏,特指皇帝颁发的命令文告;诏令,文体名,古代帝王、皇太后或皇后所发命令、文告的总称。包括册文、制、敕、诏、策、令、玺书、教、谕等。‎ B.迟明,古代用于表示时辰的词语,意思是天亮的比较晚。古人最初是根据天色的变化将一昼夜划分为十二个时辰,各时辰各有其名,诸如平明、质明、平旦、旦明、食时、晡时、黄昏、人定等。‎ C.移,指调动官职,是平级调动。迁,也指调动官职,包括升级、降级、平级转调三种情况。为易于区分,人们常在“迁”字的前面或后面加一个字,升级叫迁升、迁授、迁叙,降级叫迁削、迁谪、左迁,平级转调叫转迁、迁官、迁调,离职后调复原职叫迁复。‎ D.左右,是古今异义词,文中指侍从,与“传以示美人及左右”一句中“左右”同义;现代汉语中,一般作方位词,左和右两方面;也可作实词讲,意思是支配,操纵。‎ 答案 B 解析 迟明,表示时辰的词语,天将亮的时候,不是“天亮得比较晚”;也不是十二时辰之一。‎ ‎12.下列对原文有关内容的概括与分析,不正确的一项是(3分)(  )‎ A.康保裔世代行伍出身,一心报效朝廷。其祖康志忠、其父康再遇先后战死边境,他代父职后,又随众将攻破契丹。宋真宗即位,召他回来,给予赏赐嘉奖。‎ B.康保裔抱定赴死信念抗击契丹。契丹大举入侵,双方约定次日早晨交战,契丹却提前至黎明层层包围宋军,康保裔不愿苟免于死,战至兵尽矢绝,最终阵亡。‎ C.康保裔战死沙场引起强烈反响。他的死讯传来,皇上深深震惊哀悼,停止朝会两天。又赐封其子官职,同时一再表示,康保裔是为王事而死,理应厚重褒奖。‎ D.康保裔品行端正,严谨厚道。他待人接物讲究礼貌,又擅长骑马射箭,射飞禽走兽无不中;在与契丹血战时,张凝、李重贵与他共同抗敌,敌军这才退去。‎ 答案 D 解析 本题考查归纳内容要点,概括中心意思的能力。在康保裔与契丹血战时,张凝、李重贵率领军队策应他,没有与他共同抗敌。‎ ‎13.把文中画横线的句子翻译成现代汉语。(10分)‎ ‎(1)贷公钱数十万劳军,没后,亲吏鬻器玩以偿。(5分)‎ 译文:_________________________________________________‎ ‎(2)凝议上将士功状,重贵喟然曰:“大将陷没,而吾曹计功,何面目也!”(5分)‎ 译文:_________________________________________________‎ 答案 (1)曾经借公家的十万钱慰劳军队,他牺牲后,亲属和部下卖掉器物珍玩来偿还。‎ ‎(2)张凝提议上书报告将士们立功的情况,李重贵深深叹息说:“大将都陷于敌阵战死了,我们这些人却想计较功劳,有什么脸面啊!”‎ 解析 本题考查理解并翻译文中的句子的能力。(1)重点词语:贷,借;劳,犒劳;“没”通“殁”,死亡;鬻,卖;以,目的连词,来。(2)重点词语:议,建议;上,上奏;状,情形;吾曹,我们;面目,脸面。‎ 参考译文:‎ 康保裔,是河南洛阳人。他的祖父康志忠,在讨伐王都时战死。他的父亲康再遇,在跟随太祖征讨李筠时,又死于战场。康保裔在周朝屡次立下战功,等到康再遇阵亡,(皇上)诏令让康保裔代替父亲之职,跟随石守信攻破泽州,又跟随各位将领在石岭关打败契丹,担任登州刺史之职。不久任代州知州,接着调任深州知州,担任凉州观察使。真宗即位之后,将他召回,因为他能勤勉地赡养年老的母亲,(皇上)赐给他家上好的酒、茶和米。还发诏令褒奖他,让他担任高阳关都部署。契丹兵大举入侵,各位将领与契丹兵在河间作战,康保裔遴选精锐兵力赶赴战场,恰逢天将黑,与诸将约定第二天早晨联合作战。次日黎明时,契丹把康保裔包围了好几重,康保裔的手下劝他换甲衣、骑快马冲出去,康保裔说:“面临灾难,不可苟且偷生。”于是与敌人决战。战斗进行了两天,杀死杀伤敌人很多,踏践起的尘土足有二尺深,手下的士兵全都用光了箭,但援兵还是没有来,于是就牺牲在那里。当时皇上的车驾停驻在大名,(皇上)听到康保裔牺牲的消息,震惊哀悼,停止上朝了两天,并赠予康保裔侍中之职。(皇上)还让康保裔的儿子康继英担任六宅使、顺州刺史,让康继彬担任洛苑使。康继英等人感谢(皇上)说曰:“我们的父亲不能在决战中胜利,而是战死了,陛下没有因此而对他的儿女治罪,就是大幸了,但是我们还是承蒙了皇上非同寻常的恩典!”于是悲伤流泪,匍匐在地不能起身。皇上伤感地说:“你们的父亲是为国而死的,按赠赏规定,是应该加以厚赏的。”回头对侍臣们说:“保裔的父亲、祖父都战死在疆场,他自己又在战斗中牺牲,世代都体现出忠节,非常值得嘉奖。”康保裔谨慎厚道,讲究礼节,喜欢结交宾客,善于骑马射箭,射飞禽走兽没有射不中的。曾经先放置好一把三十支一束的箭,拉满弓射去,那些箭的筈镝接连着落下,人们都钦服他的射术之妙。多次经历战阵,身上带着七十处伤口。曾经借公家的十万钱慰劳军队,他牺牲后,亲属和部下卖掉器物珍玩来偿还。皇上知道这件事后,就又对康保裔家优厚赏赐。‎ 正当康保裔与契丹人血战时,援兵却没有来,只有张凝、李重贵率领军队与他作策应,他们也遭遇契丹兵并与之作战。康保裔被敌人覆没时,李重贵与张凝赶去救援,却腹背受敌,从申时杀到寅时,才把敌人击退。当时各位将领多半丢失了自己的部队,只有李重贵与张凝保全部队回到驻地。张凝提议上书报告将士们立功的情况,李重贵深深叹息说:“大将都陷于敌阵战死了,我们这些人却想计较功劳,有什么脸面啊!”皇帝听说了这事,嘉奖了李重贵与张凝。‎ ‎(三)[2017·通化质检]阅读下面的文言文,完成14~17题。(19分)‎ 何攀,字惠兴,蜀郡郫人也。仕州为主簿。刺史皇甫晏为牙门张弘所害,诬以大逆。时攀适丁母丧,遂诣梁州拜表,证晏不反。故晏冤理得申。王濬为益州,辟为别驾。濬谋伐吴,遣攀奉表诣台,口陈事机,诏再引见,乃令张华与攀筹量进时讨之宜。濬兼遣攀过羊祜,面陈伐吴之策。攀善于将命,帝善之,诏攀参濬军事。及孙皓降于濬,而王浑恚于后机,欲攻濬。攀劝濬送皓与浑,由是事解。‎ 除廷尉。时廷尉卿诸葛冲以攀蜀士,轻之,及共断疑狱,冲始叹服。迁散骑侍郎。杨骏执政,多树亲属,厚封赏,欲以恩泽自卫。攀以为非,乃与石崇共立议奏之,奏曰:“今承洪基,此乃天授。至于班赏行爵,优于泰始革命之初,不安一也;今恩泽之封,优于灭吴之功,不安二也;今之开制当垂于后若尊卑无差有爵必进数世之后莫非公侯不安三也。臣等敢冒陈闻。窃谓泰始之初,及平吴论功,制度名牒,皆悉具存。纵不能远遵古典,尚当依准旧事。”帝不纳。‎ 后杨骏反,以豫诛骏功,封西城侯,邑万户,赐绢万匹。攀固让所封户及绢之半,余所受者分给中外宗亲,略不入己。征为扬州刺史,在任三年,迁大司农。转兖州刺史,加鹰扬将军,固让不就。太常成粲、左将军卞粹劝攀莅职,中诏又加切厉,攀竟称疾不起。及赵王伦篡位,遣使召攀,更称疾笃。伦怒,将诛之,‎ 攀不得已,扶疾赴召。卒于洛阳,时年五十八。‎ 攀居心平允,莅官整肃,爱乐人物,敦儒贵才。为梁、益二州中正,引致遗滞。巴西陈寿、阎义皆西州名士,并被乡闾所谤,清议十余年。攀申明曲直,咸免冤滥。攀虽居显职,家甚贫素,惟以周穷济乏为事。子璋嗣,亦有父风。‎ ‎(选自《晋书·何攀传》,有删改)‎ ‎14.对下列句子中加点的词的解释,不正确的一项是(3分)(  )‎ A.辟为别驾  辟:征召 B.当垂于后  垂:留传 C.以豫诛骏功  豫:参与 D.中诏又加切厉  切:恳切 答案 D 解析 切:急切。‎ ‎15.下列对文中画波浪线部分的断句,正确的一项是(3分)(  )‎ A.今之开制/当垂于后/若尊卑无差/有爵必进数世/之后莫非公侯/不安三也 B.今之开制当/垂于后若尊卑/无差/有爵/必进数世之后/莫非公侯/不安三也 C.今之开制/当垂于后/若尊卑无差/有爵必进/数世之后/莫非公侯/不安三也 D.今之开制当/垂于后若尊卑无差/有爵必进数世/之后莫非公侯/不安三也 答案 C 解析 结合句意,“当”为副词,应该之意,显然B、D停顿错误。“数世之后”不能断开,A、B、D均可排除。‎ ‎16.下列对原文有关内容的概括和分析,不正确的一项是(3分)(  )‎ A.何攀担任别驾之时,不仅奉王濬之命带着奏章前往中央,与张华商量伐吴事宜,还又前去拜访羊祜,面陈讨吴之策,他的才能赢得皇帝嘉许。‎ B.何攀与石崇共同弹劾杨骏,认为其赏赐封爵不应厚于本朝开国之时,不应高于对灭吴功臣的封赏,即使不能遵循远古典制,也应当依照先例。‎ C.何攀为官严肃,曾任廷尉、刺史、大司农等职,却坚决推辞了兖州刺史、鹰扬将军之职,后带病应召,是为篡位的赵王司马伦所迫,实属无奈之举。‎ D.何攀正直爱才,不仅任梁州、益州中正之时引荐被埋没的人才,而且还勇于辩明曲直,为刺史皇甫晏以及西州名士陈寿、阎义洗清了冤枉。‎ 答案 A 解析 与张华商量伐吴事宜不是王濬的命令。‎ ‎17.把文中画横线的句子翻译成现代汉语。(10分)‎ ‎(1)时廷尉卿诸葛冲以攀蜀士,轻之,及共断疑狱,冲始叹服。(5分)‎ 译文:_________________________________________________‎ ‎(2)攀固让所封户及绢之半,余所受者分给中外宗亲,略不入己。(5分)‎ 译文:_________________________________________________‎ 答案 (1)当时,廷尉卿诸葛冲因为何攀是蜀地人,所以很轻视他,后来一起审理疑案,诸葛冲这才为之叹服。(“以”“轻”“断”“狱”“始”各1分)‎ ‎(2)何攀坚持让出自己所受的封户和一半的绢物,其余受取的封赠全分给了直系、旁系的本宗亲戚,一点都没留给自己。(“固”“中外”“略不”各1分,句意2分)‎ 解析 本题的重点词语有:(1)以,因为;轻,轻视;断,审理;狱,案件;始,才。(2)固,坚持;中外,直系、旁系;略不,一点不。‎ 参考译文:‎ 何攀,字惠兴,是蜀郡郫地的人。他在州里任职主薄。上司刺史皇甫晏被牙门张弘所陷害,被诬告有大逆之罪。当时何攀正在守母丧期间,就到梁州上奏章,证明皇甫晏不会谋反。皇甫晏的冤情因此而得到平反。王濬在益州时,将他征辟为别驾。王濬计划攻打吴国,派遣何攀奉表去拜会台阁,口头陈述计谋,(皇帝)下诏两次引见,于是命令张华与何攀一起筹划进军攻打的时机。王濬还派何攀去拜访羊祜,面陈攻打吴国的计划。何攀善于奉命行事,皇帝认为他不错,下诏要何攀参与到王濬的军事工作中。后来,孙皓向王濬投降,而王浑为自己去得迟而恼火,想攻打王濬。于是何攀劝王濬把孙皓送到王浑那里,这件事就此解决。‎ 又任廷尉。当时,廷尉卿诸葛冲因何攀是蜀地人,所以很轻视他,后来一起审理疑案,诸葛冲这才为之叹服。转任散骑侍郎。杨骏掌朝中大权时,拉了很多亲戚当官,又大肆封赏,想要遍施恩泽以巩固自己的地位。何攀认为这样不对,于是和石崇一起提交议状上奏,奏章上说:“现在陛下继承宏业,是天意所授。而赏赐封爵,厚于武帝泰始革命初期,这是第一个令人不安的;今日施加恩泽的封赏,高于对灭吴功臣的封赏,这是第二个令人不安的;今日封赏的爵位和制度,应当传之于后世,如尊卑之位没有差别,有爵者都要晋升,那么数世之后都成了公侯,这是第三个令人不安的。臣等敢冒死向陛下提出建议。私下认为泰始初年,到平吴时的封赏制度和封赏名单,都保存完好。即使不能遵循远古的典制,也应当依照我朝旧制。”皇帝没有采纳。‎ 后来杨骏反,他又以参加铲除杨骏的功绩,被封为西城侯,食邑万户,赐绢万匹。何攀坚持让出自己所受的封户和一半的绢物,其余受取的封赠全分给了直系、旁系的本宗亲戚,一点都没留给自己。后征用为扬州刺史,在任三年,迁任大司农。又转任兖州刺史,加封鹰扬将军,他坚持推让不愿就职。太常成粲、左将军卞粹劝何攀就职,皇帝手诏又催得紧急严厉,何攀竟然称病不起。等到赵王司马伦篡位,派遣使者召见何攀,他更加称自己病势沉重。司马伦大怒,要杀了他。‎ 何攀不得已,带病接受召见。死于洛阳,享年五十八岁。‎ 何攀居心公平允当,做官时讲究严肃整齐,喜爱才能杰出的人,对文士厚重,爱惜人才。在梁、益二州任中正时,招引弃置未用的人才。巴西之地的陈寿、阎义都是西州的名士,而且都被乡里说坏话,被时政议论了十余年。何攀帮他们申明了曲直,他们都得以免于冤枉。何攀虽然身居要职,但家里非常清贫、简朴,还专门做周济穷人、救济有难处的人的事。他的儿子何璋继承了爵位,何璋也有他父亲的作风。‎ 考点十二 鉴赏古代诗歌的形象、语言和表达技巧 考点名片 考点内容 鉴赏文学作品的形象、语言和表达技巧。‎ 考查形式 ‎①唐宋诗词为考查重点;②体裁上以律诗,篇幅适中的双调词(分上下片)为主;③侧重鉴赏形象、语言、表达技巧及作者的思想感情;④一道客观题、一道主观题,两个小题,共11分。‎ 趋势分析 ‎①侧重考查形象、表达技巧和作者的思想感情。②北京卷有选择题,四选一题型,课标卷为五选二。③作品相对浅易,理解难度适中。‎ 题组1 形象 ‎1.阅读下面的小令,完成后面题目。‎ 沉醉东风·渔父 白朴 黄芦岸白渡口,绿柳堤红蓼滩头。虽无刎颈交,却有忘机友,点秋江白鹭沙鸥。傲杀人间万户侯,不识字烟波钓叟。‎ 下列对这首小令的分析与理解,不正确的两项是(  )‎ A.开头两句选用“黄”“白”“绿”“红”这样色彩鲜明的字眼,展现了渔父生活环境的优美,引发读者丰富的想象。‎ B.小令中的渔父虽然没有生死之交,却能以在秋江上自在来去、心地单纯的白鹭沙鸥为友,让人感到其心灵世界的洁净美好。‎ C.“烟波钓叟”就是诗人在这首小令中着力表现的渔父,因为渔父不识字,没文化,所以在万户侯面前骄傲不起来。‎ D.这首小令语言平实自然,风格清新质朴,着力塑造了白鹭沙鸥生活自由自在、行动无拘无束的形象。‎ E.诗人通过对自由自在的生活的描写,表现了作者不与达官贵人为伍,甘心淡泊宁静的生活情怀。‎ 答案 CD 解析 C项,对“傲杀人间万户侯,不识字烟波钓叟”的准确理解应是:渔父虽然不识字,但他自在美好的生活足以让人间的万户侯羡慕。D项,着力塑造白鹭沙鸥的形象不准确,着力塑造的是像白鹭沙鸥那样自由自在、行动无拘无束的渔父形象。‎ ‎2.[2017·新乡市模拟]阅读下面这首宋词,完成后面题目。‎ 西江月①‎ 苏轼 照野弥弥②浅浪,横空隐隐层霄③。障泥④未解玉骢骄,我欲醉眠芳草。‎ 可惜一溪风月,莫教踏碎琼瑶⑤。解鞍欹枕绿杨桥,杜宇一声春晓。‎ ‎[注] ①此词为苏轼被贬黄州时作。其春夜行蕲水边,过酒家,饮醉,乘月至一溪桥上,解鞍,枕臂醉卧少休。拂晓,见乱山攒拥,流水淙淙,宛如仙境,书此词于桥柱上。②弥弥:水波翻动的样子。③层霄:弥漫的云气。④障泥:马鞯,垂于马两侧以挡泥土。⑤琼瑶:美玉。这里形容月亮在水中的倒影。‎ 下列对本词的理解与赏析,不正确的两项是(  )‎ A.“障泥未解玉骢骄,我欲醉眠芳草”,写词人频临溪流,从马上下来,等不及卸下马鞯,即欲眠于芳草。既侧面描绘出月下溪景之美,又表现了词人的喜悦心情。‎ B.“可惜一溪风月,莫教踏碎琼瑶”一句运用了借代的修辞手法。琼瑶,本是美玉,这里代指月色,水月交辉,有如晶莹剔透的珠玉。此句传神地写出水月之静美。‎ C.“解鞍欹枕绿杨桥,杜宇一声春晓”一句用“解鞍欹枕”这个特写镜头表现出词人的随意豁达,而“杜鹃春晓”写出空山春晨的喧闹和生机。‎ D.本词写作者爱惜一溪风月,不让马踏碎溪月,解鞍酣眠直至春晓,描写了一个醉心自然、随性洒脱、乐观豁达的诗人形象。‎ E.此词所描绘的富有诗情画意的美景中,处处有“我”之色彩,景物成为塑造“我”的典型性格的凭据。‎ 答案 BC 解析 B项,是借喻,不是借代。C项,“喧闹和生机”不对,应为“表现了空山春晨的万籁俱寂”。‎ ‎3.阅读下面这首诗,然后回答问题。‎ 题齐安城楼 杜牧 呜轧江楼角一声,微阳潋潋落寒汀。‎ 不用凭栏苦回首,故乡七十五长亭。‎ 这首诗塑造了一个怎样的抒情主人公形象?请简要分析。‎ 答:___________________________________________________‎ ‎___________________________________________________________‎ 答案 这首诗塑造了一个宦游思乡的抒情主人公形象。他一直高踞城楼,俯视大江,凭栏回望,远眺通向乡关之路。一声角鸣,使他不由得蓦然惊醒,这才知天色已晚,夕阳已沉没于水天之际。‎ 解析 通读诗歌,我们可以想到一个一直高踞城楼,俯视大江,凭栏回首,远眺通向乡关之路的抒情主人公。从中可看出主人公对家乡的思念。‎ ‎4.阅读下面这首宋诗,回答问题。‎ 食笋 曾几①‎ 花事阑珊竹事初,一番风味殿春蔬。‎ 龙蛇戢戢②风雷后,虎豹斑斑雾雨余。‎ 但使此君常有子,不忧每食叹无鱼。‎ 丁宁下番③须留取,障日遮风却要渠。‎ ‎[注] ①曾几:南宋诗人,字吉甫,自号茶山居士,学识渊博,勤于政事。②戢戢(jíjí):牛羊角的样子。③下番:下一轮。‎ 颔联描绘了怎样的景象?请作简要赏析。‎ 答:___________________________________________________‎ ‎___________________________________________________________‎ 答案 颔联写春夏之交,在风雷雾雨之后,新笋破土而出,如龙角戢戢而动,如虎豹初露斑纹。诗人用比喻的手法将雨后春笋写得有声有色,动态十足,表现了对它的热爱,也给读者留下了想象的空间。‎ 解析 “怎样的景象”需要先找出诗中所描写的具体意象,然后对景象加以描述。‎ ‎5.阅读下面一首词,完成后面的题目。‎ 水调歌头[注]‎ 叶梦得 秋色渐将晚,霜信报黄花。小窗低户深映,微路绕欹斜。为问山翁何事,坐看流年轻度,拚却鬓双华?徙倚望沧海,天净水明霞。‎ 念平昔,空飘荡,遍天涯。归来三径重扫,松竹本吾家。却恨悲风时起,冉冉云间新雁,边马怨胡笳。谁似东山老,谈笑净胡沙!‎ ‎[注] 靖康之后,叶梦得随宋高宗南渡,深得高宗亲重。后高宗向金求和,叶梦得被调离长江前线,无所作为,于是上疏告老,隐居于湖州卞山石林谷。此词即作于其间。‎ 词的上片前四句写了什么样的景象?有什么作用?‎ 答:___________________________________________________‎ ‎___________________________________________________________‎ 答案 ‎ 作者选用秋色、秋霜、黄花、房屋(小窗低户)、微路等意象,描绘出秋季到来后词人隐居之处凄美、幽僻的景象。为后文抒发情感作铺垫。‎ 解析 本题考查鉴赏诗歌中的景物描写的能力。分析词的上片不难找出所写景物。古诗词中景物描写可以渲染气氛,衬托人物性格、心理,寄寓情感,深化中心。具体到本首词来看,上片的景物主要是为后文的抒情作铺垫的。‎ ‎6.阅读下面这首明诗,完成后面的问题。‎ 二月闻雁 孙承宗①‎ 几听孽鸟语关关②,尽罢虚弦③落照间。‎ 却讶塞鸿偏有胆,又随春信到天山。‎ ‎[注] ①孙承宗在天启初年奉命督师山海关、蓟辽、天津、登莱诸处军务,此诗为该时期之作。他的诗词创作风格直追稼轩气象,是明末著名的边塞和英烈诗人。②关关:指受伤离群的雁的悲鸣之声。③虚弦:空弦,指失群孽雁纷纷听到没有弓箭的空弦之声。‎ 请指出诗中“孽鸟”有什么特点,并分析诗人的用意。‎ 答:___________________________________________________‎ ‎___________________________________________________________‎ 答案 在飞回北方的途中受伤离群,惊弓丧胆。以此比喻打了败仗、胆小惧战的边关将领。‎ 解析 诗中的“孽鸟”是与塞鸿对照来写的,塞鸿“有胆”,孽鸟惊弓,其形象可以想见。‎ ‎7.阅读下面的诗歌,然后回答问题。‎ 泊歙浦 ‎[宋]方岳 此路难为别,丹枫似去年。‎ 人行秋色里,雁落客愁边。‎ 霜月倚寒渚,江声惊夜船。‎ 孤城吹角处,独立渺风烟。‎ 诗中写了哪些意象?营造了怎样的氛围?‎ 答:___________________________________________________‎ ‎___________________________________________________________‎ 答案 诗中描写了“丹枫”“秋雁”“霜月”“寒渚”“夜船”“孤城”等意象,营造出了凄清孤寂的氛围。‎ 解析 本题考查鉴赏诗歌意象的能力。把握诗歌的意象,是概括氛围特点的前提。这是一首表现游子思乡的诗。思乡的情感主要是借助对景物的描绘来体现的,时令为晚秋,景象萧瑟,这由诗中“丹枫”“雁”“霜月”“寒渚”等意象便可推知。另外,“丹”“霜”“寒”等修饰语和“夜船”“孤城”等意象更显凄凉与孤寂之境。‎ ‎8.阅读下面的诗歌,然后回答问题。‎ 寻春 王阳明 十里湖光放小舟,谩寻春事及西畴。‎ 江鸥意到[注]忽飞去,野老情深只自留。‎ 日暮草香含雨气,九峰晴色散溪流。‎ 吾侪是处皆行乐,何必兰亭说旧游?‎ ‎[注] 意到:估计(我)到近旁。‎ 颔联中“江鸥”的形象有何特征和作用?‎ 答:___________________________________________________‎ ‎___________________________________________________________‎ 答案 形象特征:活泼可爱与顽皮,充满情趣。(答“对人有疑虑”,也可)‎ 作用:写出了景物(环境)的生动特征,衬托出诗人欲亲近而不得的失落;表现了诗人对眼前春景的迷恋,对乡村生活的陶醉。‎ 解析 诗歌中的艺术形象往往带有诗人自己的影子,但有时也会借这种形象表达自己内心的一种感悟。要想准确理解艺术形象的特征:一要分析透形象的外在特征、环境特点和内在品性;二要联系诗人自身经历和所处社会环境,揣摩诗人所托之情,所言之志。对诗歌中形象的鉴赏,不要停留在其外形特点上,要挖掘事物内在的神韵、品格、精神。这首诗中的“江鸥”活动的背景是含“十里湖光”与“雨气”“草香”的日暮,环境温馨和谐,满是情趣,动作“忽飞去”满是调皮,体现了诗人的情趣。‎ ‎9.阅读下面这首宋词,然后回答问题。‎ 水调歌头·过岳阳楼作①‎ 张孝祥 湖海倦游客,江汉有归舟。西风千里,送我今夜岳阳楼。日落君山云气,春到沅湘草木,远思渺难收。徙倚阑干久,缺月挂帘钩。‎ 雄三楚,吞七泽,隘九州。人间好处,何处更似此楼头?欲吊沉累②无所,但有渔儿樵子,哀此写离忧。回首叫虞舜,杜若③满芳洲。‎ ‎[注] ①此词作于作者请求侍亲获准后,乘舟沿江东归途中。②累:无罪被迫而死。③杜若:一种芳草。‎ 上片写了怎样的景色?请用自己的话进行描述。‎ 答:___________________________________________________‎ ‎___________________________________________________________‎ ‎___________________________________________________________‎ 答案 夕阳斜照在广阔的洞庭湖面上,波光粼粼;湖中君山的暮霭云雾,四周萦绕;沅水、湘水相汇处的两岸草木,呈现出一片葱绿的春色;夜幕降临,一弯残月高挂天际。‎ 解析 可抓住上片中“湖海”“日落”“君山云气”“草木”“缺月”等意象,展开联想和想象,描摹景色。‎ ‎10.阅读下面这首词,完成后面的问题。‎ 南歌子 贺铸 疏雨池塘见,微风襟袖知。阴阴夏木啭黄鹂。何处飞来白鹭立移时。‎ 易醉扶头酒[注],难逢敌手棋。日长偏与睡相宜。睡起芭蕉叶上自题诗。‎ ‎[注] 扶头酒:醇厚浓烈易醉人之酒。‎ 这首词的上片描写了哪些意象?给我们描绘了一幅怎样的画面?‎ 答:___________________________________________________‎ ‎___________________________________________________________‎ 答案 疏雨、池塘、微风、夏木、黄鹂、白鹭。描绘了一幅恬静、幽美,充满生机的夏日风光图。‎ 解析 本题考查鉴赏古诗词中的形象的能力。描写的意象从词中不难找到,关键是要找全,不遗漏,否则容易失分。由“微风”“夏木”“黄鹂”“白鹭”等可以看出描绘了一幅生机勃勃的夏日图景。‎ 题组2 语言 ‎11.阅读下面这首诗,完成后面的问题。‎ 百嘉村见梅花 龚鼎孳[注]‎ 天涯疏影伴黄昏,玉笛高楼自掩门。‎ 梦醒忽惊身是客,一船寒月到江村。‎ ‎[注] 龚鼎孳(1615~1673):字孝升,崇祯七年进士。清兵入关后任太常寺少卿,康熙时官至刑部尚书。‎ 这是一首七言绝句,关于诗眼,有人认为是“惊”,有人认为是“客”,有人认为是“寒”。你认为呢?为什么?‎ 答:___________________________________________________‎ ‎___________________________________________________________‎ ‎___________________________________________________________‎ 答案 我认为是“寒”。用梅的疏影、玉笛、高楼等意象,表现了作者身为贰臣内心的痛苦寂寞,表达了对故国的思念。眼前的月亮着一“寒”字,正是他心境的写照,写出了百愁萦绕、无可奈何的凄凉。‎ 解析 首先明确诗眼,然后结合意象及诗人情感作答。诗眼一般能揭示整首诗的主旨,因此确定诗眼应从把握诗歌主旨入手。‎ ‎12.阅读下面一首唐诗,按要求答题。‎ 孤雁 杜甫 孤雁不饮啄,飞鸣声念群。‎ 谁怜一片影,相失万重云。‎ 望尽似犹见,哀多如更闻。‎ 野鸦无意绪,鸣噪自纷纷。‎ 古人有“一诗之眼”与“一诗之骨”的说法。“诗眼”,指诗中最能表现情感意味、精神内涵的“词(字眼)”;“诗骨”,指诗中处于核心地位、起统率作用的“句”。上面这首杜诗,其“诗眼”是________,其“诗骨”是____________________。‎ 答案 孤 飞鸣声念群 解析 根据题干说明,结合诗歌所表现的情感,可得出答案。‎ ‎13.阅读下面一首诗,然后回答问题。‎ 与夏十二登岳阳楼[注]‎ 李白 楼观岳阳尽,川迥洞庭开。‎ 雁引愁心去,山衔好月来。‎ 云间连下榻,天上接行杯。‎ 醉后凉风起,吹人舞袖回。‎ ‎[注] 乾元二年,李白流放途中遇赦,回舟江陵,南游岳阳而作此诗。‎ 诗中的“雁引愁心去”一句,有的版本写作“雁别秋江去”。你认为哪一句更妙?为什么?‎ 答:___________________________________________________‎ ‎___________________________________________________________‎ ‎___________________________________________________________‎ 答案 “雁引愁心去”更妙。①“雁引愁心去”运用了拟人手法,写出了李白流放遇赦的高兴心情。②这一句写大雁有意为诗人带走愁心,与下句写君山有情为诗人衔来好月,愁去喜来,互相映衬。③“引愁心”比“别秋江”更富有感情色彩,且更新颖。(言之成理即可)‎ 解析 古诗在流传过程中,同一首诗不同的版本有时有不同的用语。不同用语在该诗中可能都有一定的道理,理解鉴赏的角度不同,答案就会有所不同。这首诗的第三句有的版本作“别秋江”,两种用语都表达了流放遇赦的高兴心情。但“引愁心”用拟人手法更富感情色彩,与“别秋江”相比更胜一筹。‎ ‎14.阅读下面这首词,然后回答问题。‎ 调笑令 戴叔伦 边草,边草,边草尽来兵老。山南山北雪晴,千里万里月明。明月,明月,胡笳一声愁绝。‎ ‎“愁绝”一词为全词之“眼”,请简要分析词中包含了哪些“愁”?‎ 答:___________________________________________________‎ ‎___________________________________________________________‎ ‎___________________________________________________________‎ 答案 ①戍边士兵的思乡之愁。词中描绘“千里万里”的明月,既写出了戍卒离家之遥远,又寄予了戍卒的思乡怀人之情。②士兵长期戍边的孤独寂寞之愁。词中以边草枯萎来比喻戍卒之老,不仅写出了戍卒守边时间之长,也道尽了其人生的凄凉。‎ 解析 本词开头三句以边草起兴,感叹长期在边关征战的士兵的命运如同边草一样悲惨。士兵征战边关,年复一年,归期渺茫。他们看着边草渐渐变白枯干,深感青春消逝,年华老去。在这僻远广袤的边地,浓重的乡思涌上了心头。接下来词人的视角由边草转到白雪、明月。雪晴、月明的宁静景象,让人联想到征人伫立在苍茫的雪中的形象以及那排解不尽的边愁和绵长悠远的思乡情怀。‎ ‎15.阅读下面这首诗,完成后面的题目。‎ 晚望 ‎[清]郑珍 向晚古原上,悠然太古春。‎ 碧云收去鸟,翠稻出行人。‎ 水色秋前静,山容雨后新。‎ 独怜溪左右,十室九家贫。‎ 有人以为,尾联毁坏了全诗优美的意境,应该删去;也有人说,这两句恰是诗人匠心独运之处。你赞同哪一种意见?请结合全诗内容加以概括剖析。‎ 答:___________________________________________________‎ ‎___________________________________________________________‎ ‎___________________________________________________________‎ 答案 赞同后一种意见。前三联描绘春色之美和闲适之感,最后一联点出农村的穷苦,内心的沉重。作者运用“以乐写哀”(或“以乐景衬悲情”“对比”)的手法,抒发了满腔悲愤之情,也使感叹民生疾苦的主旨更为明确。‎ 解析 本题中“你赞同哪一种意见”好像是个开放型的题目(注意现今高考中的伪探究题),其实读懂诗歌就明白了诗人要表达的情感是很明确的——感叹民生疾苦。所以只能按后一种意见回答。“以乐写哀”的手法有点难度。‎ ‎16.阅读下面两首诗,然后回答问题。‎ 台城 韦庄 江雨霏霏江草齐,六朝如梦鸟空啼。‎ 无情最是台城柳,依旧烟笼十里堤。‎ 鹿柴 王维 空山不见人,但闻人语响。‎ 返景入深林,复照青苔上。‎ ‎《台城》中“鸟空啼”的“空”字历来为人所称道,你能品出其中的滋味吗?它与《鹿柴》中的“空”字所表现的意境是否相同?请加以赏析。‎ 答:___________________________________________________‎ ‎___________________________________________________________‎ ‎___________________________________________________________‎ 答案 这两个“空”字表现的意境不同。“鸟空啼”的“空”是“白白地”“徒劳地”的意思,既衬托出梦一般的景色,又陷入了诗人对历史兴衰的感慨:台城历经变迁,已经面目全非,但不解情的大自然,却一年一度景色依旧,连鸟儿也在无谓地啼鸣。这与杜甫的“隔叶黄鹂空好音”有异曲同工之妙。于乐景中注入哀情,反衬自然无情人有情,传达出诗人心灵深处的忧伤。《鹿柴》中的“空”是“空旷”之意,展现的是大山幽静、不闻人语的情境,表现出宁静幽深的境界。‎ 解析 解答本题主要考虑这两首诗的主旨。《台城》一诗主要是怀古伤今,《鹿柴》一诗则要表现的是一种恬淡、自然、幽静的境界。因此,两首诗中“空”字是不同的。‎ ‎17.阅读下面一首词,完成后面问题。‎ 西江月·夜行黄沙道中 辛弃疾 明月别枝惊鹊,清风半夜鸣蝉。稻花香里说丰年,听取蛙声一片。‎ 七八个星天外,两三点雨山前。旧时茅店社林边,路转溪桥忽见。‎ ‎“路转溪桥忽见”,“忽见”的是什么?是溪桥吗?并请就此简要谈一谈诗歌在用语上的特点。‎ 答:___________________________________________________‎ ‎___________________________________________________________‎ ‎___________________________________________________________‎ 答案 “忽见”的是“旧时茅店”。“旧时茅店”在“忽见”之前,更显夜行人的惊喜之情。诗歌用语有一定的跳跃性,为了情感表达、音节韵律等的需要,常运用颠倒词序等方式,以达到独特的效果。‎ 解析 “旧时茅店社林边,路转溪桥忽见”因为作者平时经常往来于黄沙道中,明明知道树林旁边有一个茅草小店,但此时因为是在夜里,却忽然不见了。可是,过了小溪上的石桥,再转个弯儿,那座旧时相识的茅店便突然出现在眼前。此处“忽见”的是“旧时茅店”。‎ 题组3 表达技巧 ‎18.阅读下面一首诗,完成后面的题目。‎ 送沈子福归江东 王维 杨柳渡头行客稀,罟师荡桨向临圻。‎ 惟有相思似春色,江南江北送君归。‎ 诗的三、四两句运用了什么表达技巧?试作简要分析。‎ 答:___________________________________________________‎ ‎___________________________________________________________‎ 答案 化虚为实。将抽象的情感化为可以感知的春色,形象感人。(回答“比喻的修辞手法”也可)‎ 解析 “惟有相思似春色”一句中,作者把心中的相思感情比作自然界的春色,相思无形,春色可感,化虚为实,具体可感。‎ ‎19.阅读下面一首诗,回答后面的问题。‎ 丁都护歌 李白 云阳上征去,两岸饶商贾。‎ 吴牛喘月时,拖船一何苦!‎ 水浊不可饮,壶浆半成土。‎ 一唱都护歌,心摧泪如雨。‎ 万人凿磐石,无由达江浒。‎ 君看石芒砀,掩泪悲千古。‎ 这首诗主要写纤夫之苦,为什么还要写“两岸饶商贾”?‎ 答:___________________________________________________‎ ‎___________________________________________________________‎ ‎___________________________________________________________‎ 答案 诗人把纤夫的生活放在两岸商贾云集、商业网点稠密的背景下,与巨商富贾们的生活形成鲜明对比,更加突出纤夫们的夏日拖船之苦。‎ 解析 “两岸饶商贾”表现了两岸商贾云集的繁华、热闹场面,这一场面恰恰与纤夫们辛苦拖船的画面形成对比。‎ ‎20.阅读下面两首唐诗,回答问题。‎ 夜上受降城闻笛 李益 回乐烽前沙似雪,受降城外月如霜。‎ 不知何处吹芦管,一夜征人尽望乡。‎ 春夜闻笛 李益 寒山吹笛唤春归,迁客相看泪满衣。‎ 洞庭一夜无穷雁,不待天明尽北飞。‎ 两首诗都写了景,但作用却不同,请结合诗句谈谈你的看法。‎ 答:___________________________________________________‎ ‎___________________________________________________________‎ ‎___________________________________________________________‎ 答案 前诗前两句写景(如霜的月光和月下雪一般的沙漠),用以烘托征人心境的孤独、悲惨、凄凉,为后面“尽望乡”作铺垫。后诗借大雁未等天亮就急切北飞来反衬诗人不能北归的遗憾和哀怨。‎ 解析 诗歌中景物描写往往具有烘托、反衬诗人情感的作用。答题时,从这几个角度,结合诗歌主旨分析即可。‎ ‎21.阅读下面一首诗,然后回答问题。‎ 秋思 张籍 洛阳城里见秋风,欲作家书意万重。‎ 复恐匆匆说不尽,行人临发又开封。‎ 王安石评张籍的诗说“看似寻常最奇崛”,本诗有一处成功的细节描写,正体现了王安石的评价。请你找出来,并作简要的分析。‎ 答:___________________________________________________‎ ‎___________________________________________________________‎ ‎___________________________________________________________‎ 答案 “行人临发又开封”,既照应了“意万重”,又紧承“复恐”,刻画出心有千言万语唯恐言之不尽的复杂微妙的心理,让人在看似平常的描写中体味到作者浓浓的乡思。‎ 解析 所谓细节描写是指文学作品中对人物动作、语言、神态、心理、外貌以及自然景观、场面气氛等细小环节或情节的描写。在本诗中,作者所写的“复恐匆匆说不尽,行人临发又开封”即为典型的细节描写,注意结合诗句加以分析。‎ ‎22.阅读下面这首元曲,然后回答问题。‎ ‎[中吕]山坡羊·叹世(其一)‎ 陈草庵 晨鸡初叫,昏鸦争噪,那个不去红尘闹?路遥遥,水迢迢,功名尽在长安道,今日少年明日老。山,依旧好;人,憔悴了!‎ 这首曲子使用了什么修辞手法?有什么作用?请结合全曲内容简要分析。‎ 答:___________________________________________________‎ ‎___________________________________________________________‎ ‎___________________________________________________________‎ 答案 对比。以“今日少年”与“明日老”、“山,依旧好”与“人,憔悴了”对比作结,指出追逐名利实际上是没有认识到生命的价值所在,是舍本逐末、迷失本性的行为,从而劝诫人们认识人生的真正意义,不要成了功名利禄的奴隶。‎ 解析 本题主要是对修辞手法及其表达效果的考查。结合有关修辞手法的知识,联系曲子的相关内容分析作答。此曲中,既有“今日少年”与“明日老”在时间上的对比,也有“山,依旧好”与“人,憔悴了”的对比。结合全曲来看,作者主要是劝诫人们不要一味地追求功名利禄而使自己出现对比中所说的结果。‎ ‎23.阅读下面一首宋词,完成后面的题目。‎ 洞仙歌[注]‎ 赵鼎 空山雨过,月色浮新酿。把盏无人共心赏。漫悲吟,独自捻断霜须。还就寝,秋入孤衾渐爽。‎ 可怜窗外竹,不怕西风,一夜潇潇弄疏响。奈何九回肠,万斛清愁,人何处,邈如天样。纵陇水、秦云阻归音,便不许时闲,梦中寻访?‎ ‎[注] 这首词是作者谪居岭南时所作。‎ 全词在写法上有什么特点?试简要分析。‎ 答:___________________________________________________‎ ‎___________________________________________________________‎ ‎___________________________________________________________‎ 答案 全词基本上采用铺叙的写法,不以剪裁工巧取胜,而以描写深刻细腻见长,按时间顺序叙述,以行动、心理描写推进,精细、鲜明、轻巧,含而不露。‎ 解析 这首词主要通过一系列生活细节的描述来表现抒情主人公的情感,这正是采用了铺叙的写法。注意结合原词加以赏析。‎ ‎24.阅读下面一首宋词,然后回答问题。‎ 柳梢青·送卢梅坡 刘过 泛菊①杯深,吹梅②角远,同在京城。聚散匆匆,云边孤雁,水上浮萍。‎ 教人怎不伤情?觉几度,魂飞梦惊。后夜相思,尘随马去,月逐舟行。‎ ‎[注] ①泛菊:饮菊花酒。②吹梅:吹奏《梅花落》。‎ 作者在表达感情时主要运用了什么手法?请结合全词内容具体说明。‎ 答:___________________________________________________‎ ‎___________________________________________________________‎ ‎___________________________________________________________‎ 答案 主要运用了比喻手法。上片用“云边孤雁,水上浮萍”的比喻来表现离别之苦,下片用“尘随马去,月逐舟行”的比喻来表现思念之切。(意思对即可)‎ 解析 本词所用手法有比喻、化用。题干要求回答的是“主要运用”的手法,答出“比喻”即可,要注意结合全词加以分析。‎ ‎25.阅读下面两首诗,回答后面的问题。‎ 盆池 韩愈 池光天影共青青,拍岸才添水数瓶。‎ 且待夜深明月去,试看涵泳几多星。‎ 小池 杨万里 泉眼无声惜细流,树阴照水爱晴柔。‎ 小荷才露尖尖角,早有蜻蜓立上头。‎ 简要说明《盆池》和《小池》在表现手法上有何不同。‎ 答:___________________________________________________‎ ‎___________________________________________________________‎ ‎___________________________________________________________‎ 答案 《盆池》取材小,而诗人从大处落笔,以小见大,用大手笔写小景致。《小池》则刻画精细,小荷蜻蜓浓缩为“盆景”,大中见小。‎ 解析 解答本题需对比鉴赏。一个从大处去写,“天影共青青”“涵泳几多星”;一个从小处去写,抓细节“蜻蜓立荷尖”。‎ ‎26.阅读下面这首宋词,完成后面的题目。‎ 临江仙·夜归临 苏轼 夜饮东坡醒复醉,归来仿佛三更。家童鼻息已雷鸣。敲门都不应,倚杖听江声。‎ 长恨此身非我有,何时忘却营营?夜阑风静縠纹平。小舟从此逝,江海寄余生。‎ 有人说“夜阑风静縠纹平”表面上看来只是一般的写景句子,其实却是作者主观世界和客观世界相契合的产物(即情景相融)。请作简要赏析。‎ 答:___________________________________________________‎ ‎___________________________________________________________‎ ‎___________________________________________________________‎ 答案 这一句描写的是深夜江上风平浪静的情景,此时的作者心与景会,神与物游,为如此静谧美好的大自然所陶醉。于是,他情不自禁地产生脱离世事困扰,寄情自然,实现精神解放(心灵自由)的遐想。眼前之景其实象征着作者所追求的那个宁静、安谧、自由的理想境界。(意思相近即可)‎ 解析 本题考查鉴赏诗歌表达技巧的能力。题目已经明确要求考生从情景交融的角度进行鉴赏,在答题时考生可以按照情景交融这种类型题的答题模式,结合词句具体内容作答。‎ 考点十三 评价古代诗歌的思想内容和作者的观点态度 考点名片 考点内容 评价内容主题,评价思想感情,评价作者的观点态度。‎ 考查形式 客观题(五选二题型)或主观简答题。分值5分或6分。独立命题或与表达技巧综合命题。‎ 趋势分析 独立命题或与表达技巧综合命题。‎ ‎1.阅读下面这首诗,完成后面问题。‎ 拟咏怀二十七首(其十八)‎ 庾信[注]‎ ‎[注] 庾信:南北朝时期诗人,辅佐梁元帝,出使西魏被扣留,西魏灭梁后被迫仕魏,后北周取代西魏又仕北周,本诗为仕周时期所作。‎ 下列对本诗的理解与赏析,不正确的两项是(  )‎ A.诗作前半篇写中夜操琴、书卷满床的情景,后半篇写白露明月、萤火飘流的秋色,构成凄清孤寂的意境。‎ B.“寻思”两句直抒胸臆,不仅有故国覆灭、封侯梦想破碎的叹惋,也有当下不能为国建功的自嘲。‎ C.“虽言”两句运用庄周梦蝶的典故,自己如同那个可以齐物我、一是非、在轻飘飘的达观世界中自适其志的庄周一样,梦为蝴蝶、忘怀自身,就可以摆脱时刻相随的愁思。‎ D.“残月”两句由“中夜”暗渡而来,表达出时光流逝而诗人却不能回国辅政尽忠的无限惆怅之情。‎ E.“露泣”两句由“新秋”而来,烘托出诗人凄凉哀伤、虚无烦乱的内心状态,用词精切,对仗工巧。‎ 答案 CD 解析 C项,“虽言”一句运用庄周梦蝶的典故,“虽言”“定自”两个虚词却是重点,虽然梦为蝴蝶、忘怀自身,就可以摆脱时刻相随的愁思,无奈自己绝对不是那个可以齐物我、一是非、在轻飘飘的达观世界中自适其志的庄周。通过反用典故,表明“愁”之无可消释。D项,“辅政尽忠”无中生有,此时梁已被灭;这里表达的是时光流逝而诗人却年年如故的悲哀之情。‎ ‎2.阅读下面的这首宋词,完成后面问题。‎ 浪淘沙·九日从吴见山①觅酒 吴文英②‎ 山远翠眉长,高处凄凉。菊花清瘦杜秋娘。净洗绿杯牵露井,聊荐幽香。  乌帽压吴霜,风力偏狂。一年佳节过西厢。秋色雁声愁几许,都在斜阳。‎ ‎[注] ①吴见山:吴文英词友,常有唱酬相和。②吴文英:宋代词人,号梦窗,一生未第,游幕终生。‎ 下列对这首宋词的理解,不正确的两项是(  )‎ A.“浪淘沙”是词牌名。原为唐教坊曲,又称《卖花声》。南唐李煜变为长短句,一般多用于中调、长调。后来由柳永、周邦彦等人演变为“浪淘沙慢”。‎ B.诗歌中的“一年佳节”指的是重阳节,重阳节在农历九月初九,一般有登高、赏秋、饮菊花茶、佩戴茱萸等习俗。它与除夕、清明节、中秋节统称中国传统四大祭祖的节日。‎ C.西厢,就是指西厢房。传统四合院中一般北面是正房,东西两面都是厢房,南面是倒座。东西南北都是房子,中间是天井。‎ D.本词表现出词人多种感情,既有对时光飞逝,自己已入暮年的感慨,又不乏羁旅在外的哀愁,还借助重阳这一主题寄托了思亲怀乡的苦闷。‎ E.“秋色”“雁声”和“斜阳”是词人对景思亲及其人生感慨,如周邦彦有“雁背夕阳红欲暮”之叹,李商隐有“夕阳无限好,只是近黄昏”的哀伤。‎ 答案 AB 解析 A项,通常以58字以内的短词为小令,如《十六字令》《如梦令》等。本词54个字,属于小令,可见《浪淘沙》多是小令。B项,“中秋节”表述错误,改为“中元节”。‎ ‎3.阅读下面这首唐诗,完成(1)~(2)题。‎ 柳州榕叶落尽偶题 柳宗元 宦情羁思共凄凄,春半如秋意转迷。‎ 山城过雨百花尽,榕叶满庭莺乱啼。‎ ‎(1)诗人心中的凄苦之情是如何表现的?请简要分析。‎ 答:___________________________________________________‎ ‎___________________________________________________________‎ 答案 诗人被远贬柳州,政治上遭受打击,看到雨后凋谢的百花、满庭榕叶,听到黄莺乱叫,油然而生的是宦途失意、羁旅孤凄的愁苦与哀思。‎ 解析 本题考查鉴赏诗歌情感的能力。鉴赏诗歌的情感:一要善于抓住关键词,尤其是直接点出情感和心理感受的词,如“宦情”“羁思”“凄凄”等;二要善于捕捉典型意象,如“百花尽”“榕叶满庭”“莺乱啼”等。‎ ‎(2)诗的三、四两句如何描写“春半如秋”?请作简要分析。‎ 答:___________________________________________________‎ ‎___________________________________________________________‎ 答案 从视觉、听觉和心理感受等多角度描写,看到山城柳州风雨过后百花凋零、榕叶满庭的景象,听到黄莺乱啼,虽是春天,诗人却感觉完全如同秋天。一个“乱”字,既是写莺啼,更是写诗人的心乱。诗人正是借莺的乱啼表现出他此时心意凄迷,恍如见到秋之景。‎ 解析 本题考查鉴赏诗歌表达技巧的能力。解答本题,应从修辞手法、表达方式和表现手法等角度分析,还可以从炼字的角度加以赏析,如“乱”字。‎ ‎4.阅读下面这首宋词,完成下面问题。‎ 浣溪沙 黄庭坚 一叶扁舟卷画帘,老妻学饮伴清谈,人传诗句满江南。‎ 林下猿垂窥涤砚,岩前鹿卧看收帆,杜鹃声乱水如环。‎ 上片写词人与老妻泛舟徜徉的雅趣。“雅”体现在哪里?‎ 答:___________________________________________________‎ ‎___________________________________________________________‎ 答案 ①词人卷起观赏的画帘,与老妻对坐;②与老妻边饮酒清谈,边观赏如画江山,“老妻学饮”一句,更是雅兴溢于笔端;③写诗填词,当然必不可少,“人传诗句满江南”一句,既道出了“清谈”的内容,又勾勒出了词人的翩翩风度。‎ 解析 本题考查鉴赏诗歌内容的能力。解答本题:一要留意典型意象,如“扁舟”“画帘”“诗句”;二要善于抓住诗中的主要事情,如诗人“卷画帘”,老妻“学饮”,还有“人传诗句”,这些都能体现“雅”。‎ ‎5.阅读下面这首唐诗,完成下面问题。‎ 吊万人冢 张 兵罢淮边客路通,乱鸦来去噪寒空。‎ 可怜白骨攒孤冢,尽为将军觅战功。‎ 这首诗表达了诗人的哪些情感?请简要概括。‎ 答:___________________________________________________‎ ‎___________________________________________________________‎ 答案 ①对战乱暂时结束的欣喜之情;②黑鸦聒噪带来的悲伤;③对惨死他乡的枉死者的同情;④对一心邀功的将军的愤慨。‎ 解析 本题考查鉴赏诗歌思想感情的能力。这首诗是战乱后诗人客游淮河地区时根据所见所感而描绘的一幅社会缩影图,控诉了统治者为一己私利而压迫人民的血腥罪恶。四句一句一事,首句“兵罢淮边”是叙事,“客路通”是表达喜悦之情;第二句写鸦“噪”凸显烦乱悲伤之情;第三句写“白骨攒孤冢”突出同情;第四句写“为将军觅战功”表达愤慨。‎ ‎6.阅读下面这首清词,完成下面问题。‎ 浣溪沙·红桥 ‎[清]王士祯 白鸟朱荷引画桡①,垂杨影里见红桥,欲寻往事已魂消。‎ 遥指平山②山外路,断鸿无数水迢迢,新愁分付广陵潮。‎ ‎[注] ①桡:船桨,此指船。②平山:指平山堂,扬州游览胜地。‎ 有人说,这首词写词人的怀古之幽思。请结合诗句简要分析。‎ 答:___________________________________________________‎ ‎___________________________________________________________‎ 答案 这首词写舟中观看红桥景色,抒发词人怀古之幽思。词人面对白鸟朱荷、绿柳红桥这优美之景,不禁思绪万千。“欲寻往事”,从古至今,这座名城发生过多少可歌可泣的事情啊?怎不令人黯然销魂!‎ 解析 本题考查鉴赏诗歌思想感情的能力。把握诗歌的思想感情:一要抓住诗中的意象,如“白鸟”“朱荷”“画桡”“垂杨”和“红桥”等,体会词人面对美景的万千思绪;二要留意表明情感和点明主旨的关键词句,如“欲寻往事已魂消”,便能体会到词人的幽思。‎ ‎7.阅读下面这首小令,完成(1)~(2)题。‎ 上小楼·隐居 ‎[元]任昱[注]‎ 荆棘满途,蓬莱闲住。诸葛茅庐,陶令松菊,张翰莼鲈。不顺俗,不妄图,清风高度。任年年落花飞絮。‎ ‎[注] 任昱,字则明,四明(今浙江宁波市)人。生卒年不详。与张可久、曹明善为同时人,少时好游历,一生不仕。‎ ‎(1)诗歌的题目是“隐居”,作者围绕“隐居”依次写了哪些内容?请结合全曲简要分析。‎ 答:___________________________________________________‎ ‎___________________________________________________________‎ 答案 ①作者首先交代了隐居的原因是仕途险恶,世道艰难(“荆棘满途”),因而在有如“蓬莱”仙境一样的风景优美之地隐居;②再写隐居的住所(“茅庐”)、周围的环境(“松菊”)和起居生活(“莼鲈”);③接着写隐居的原则是“不顺俗,不妄图,清风高度”;④最后写自己飘然出世、无意旁骛的隐居情怀。‎ 解析 本题考查鉴赏诗歌内容的能力。分析内容应立足于全诗,本篇起首二句,以“荆棘”与“蓬莱”‎ 两个并列复合词作借代,“荆棘”句,极言世路艰险,与“蓬莱”形成对照,使读者一开始对作者“隐居”产生理解与赞赏。下三句,用了三组历史典故,均与隐居密切关联。“茅庐”“松菊”“莼鲈”均为高风亮节的标志。于是,作者随之表明了自己追仪古贤的生活态度:“不顺俗,不妄图。”这种旷达超脱的“清风高度”,显示了隐者澄静、宁和的内心世界。‎ ‎(2)本曲最突出的表现手法是什么?请选择两三例分析作者通过这种手法抒发了哪些思想情感?‎ 答:___________________________________________________‎ ‎___________________________________________________________‎ 答案 主要的表现手法是用典:①诸葛亮在隆中隐居时,自比管仲、乐毅,作者以出山之前的诸葛亮自比,表达自己怀才不遇的心情;②陶渊明脱离官场、归隐田园时“三径就荒,松菊犹存”,作者借陶渊明之“松菊”表达自己清高自傲,超然独立的胸怀;③西晋张翰借口秋风起,思念家乡莼羹、鲈鱼,辞官归家,作者借此表达自己对故土田园的热爱和对隐居生活的自得其乐的惬意之情。‎ 解析 本曲用典恰当。全曲仅以一首一尾涉点尘世,其余均驰神走笔于隐居天地。‎ ‎8.阅读下面的宋诗,回答问题。‎ 秦淮夜泊(辛未正月赋)‎ 贺铸 ‎[注] ①朱箔:这里指红色的窗帘。②心旆:旆(pèi),泛指旌旗。这里指“心摇摇然如悬旌之无所终薄”(《史记·苏秦列传》),终薄:安顿,着落。‎ 尾联是如何表达作者思想情感的?请简要分析。‎ 答:___________________________________________________‎ ‎___________________________________________________________‎ 答案 ①尾联以直接抒情作结。眼前的景物虽美丽动人,但毕竟是在外漂泊的游子,加上正是暮色时分,人们都在家中,而诗人却独泊孤舟,抒发了淡淡的乡愁。②同时用典,“心旆正摇摇”出自《史记·苏秦列传》“寡人卧不安席,食不甘味,心摇摇然如悬旌而无所终薄”。诗人借此典故抒发了自己终无所托、没有着落的怅惘之情。(句中关键词用“惆怅”“迷惘”“伤感”“茫然”也可)‎ 解析 分析如何表达感情时要与手法捆绑在一起。“怜”直抒胸臆,抒发了在外漂泊的游子思乡之情,很好理解。但是下一句“心旆正摇摇”表明前一句不是强烈的乡愁,而是淡淡的心神不宁的惆怅迷惘之情。这点要从注释“终薄”的理解中找到答案,手法用典也是从注释中得出。‎ ‎9.阅读下面这首汉代古诗,完成(1)~(2)题。‎ 步出城东门 无名氏 步出城东门,遥望江南路。‎ 前日风雨中,故人从此去。‎ 我欲渡河水,河水深无梁。‎ 愿为双黄鹄[注],高飞还故乡。‎ ‎[注] 黄鹄:传说中的大鸟,一举千里,仙人所乘。‎ ‎(1)这首诗主要写了哪两件事?它们之间是什么关系?‎ 答:___________________________________________________‎ ‎___________________________________________________________‎ 答案 一是游子送友,二是游子欲归不能。在风雨中送别友人归去,更加衬托出游子的孤独寂寥,越发增添游子的思乡之情,表现出游子不能归乡的哀愁。‎ 解析 本题考查鉴赏诗歌内容的能力。概括诗中所述事件,要注意抓住叙事的要素,比如时间是“前日”,地点是“城东门”“江南路”,背景是“风雨中”,人物是“故人”和“我”,行为是送别友人、“还故乡”。由此可以概括出两件事:游子送友和游子欲归不能。写游子送友,是为了突出自己欲归不能的哀愁。‎ ‎(2)这首诗最后两句表达了诗人怎样的思想情感?运用了什么表现手法?请简要分析。‎ 答:___________________________________________________‎ ‎___________________________________________________________‎ 答案 本诗的最后两句表达了诗人想与友人同回故乡的迫切愿望,也暗含了不能与友人同回的惆怅之情。这两句运用想象,即诗人和故人能化作一双黄鹄,同回故乡,生动形象地表现了游子因现实的阻隔,想回故乡而不得的痛苦和哀愁。‎ 解析 本题考查鉴赏诗歌思想情感和表达技巧的能力。诗中最后两句运用了想象的表现手法,表达了诗人归乡的强烈愿望,思想情感和表现手法都比较容易分析得出。‎ ‎10.阅读下面这首唐诗,完成下面问题。‎ 蓝田溪与渔者宿 钱起 独游屡忘归,况此隐沦处。‎ 濯发清泠泉,月明不能去。‎ 更怜垂纶叟,静若沙上鹭。‎ 一论白云心,千里沧州趣。‎ 芦中野火尽,浦口秋山曙。‎ 叹息分枝禽,何时更相遇?‎ 本诗中“隐沦处”的美好体现在哪些方面?请简要分析。‎ 答:___________________________________________________‎ ‎___________________________________________________________‎ 答案 一是环境的“清”“明”“静”。如写“隐沦处”水清可以濯发,月明使人留恋,说明“隐沦处”的环境优美。二是和渔者同宿相谈甚欢,情趣相投,其乐融融。这里境合于心,人合于情,自然是美好的。‎ 解析 本题考查鉴赏古代诗歌内容和情感的能力。解答本题,首先,要理清整首诗的行文脉络:诗的前六句写诗人爱渔者的居住地,中间四句写与渔者宿的乐趣,最后两句写不忍与渔者分别。其次,要抓住关键词,如“清泠泉”“月明”“沙上鹭”“野火尽”“秋山曙”等,由此分析便可概括出答案。‎ ‎11.阅读下面这首宋词,完成后面问题。‎ 诉衷情近 柳永 雨晴气爽,伫立江楼望处。澄明远水生光,重叠暮山耸翠。遥认断桥幽径,隐隐渔村,向晚孤烟起。‎ 残阳里。脉脉朱阑静倚。黯然情绪,未饮先如醉。愁无际。暮云过了,秋光老尽,故人千里。尽日空凝睇。‎ 请简要分析词的下片抒发了词人什么样的情感,并赏析结句的表达效果。‎ 答:___________________________________________________‎ ‎___________________________________________________________‎ 答案 词的下片抒发了词人“秋光已尽”的迟暮之感和久望不见的怀人之悲。‎ 以词人“凝睇”的神态描写作结,情感表达含蓄蕴藉;以定格人物形象作结,戛然而止,言有尽而意无穷;与开头“伫立”相呼应,首尾圆合,结构完整;“空”字极富表现力,既有望而不见之意又写尽了内心的空落和无奈之感。‎ 解析 本题考查鉴赏诗歌思想感情和表现手法的能力。解答第一问,要注意抓关键词句,如“残阳”“黯然情绪”“愁无际”“秋光老尽,故人千里”等,这是词人悲秋所生的迟暮之感与身处异乡所生的怀人之悲的混合。解答第二问,需要从多个角度去思考,如描写手法、结构、炼字等,结合诗歌的内容和情感分析即可。‎ ‎12.阅读下面这首明诗,完成下面问题。‎ 赋得残月似新月 袁宗道 一钩曾挂暮霞里,半玦还悬晓雾中。‎ 醉起忽迷钟早晚,山行误认峰西东。‎ 从他乌历干支换,且喜蛾眉首尾同。‎ 安得人生也似月,苍颜皓首又如童。‎ 尾联的“安得”“如童”表达了作者怎样的情感?‎ 答:___________________________________________________‎ ‎___________________________________________________________‎ 答案 “安得”的意思是“哪里能够得到”,“如童”的意思是像童年一样,表达了作者积极的人生态度及对人生的美好愿望,即希望人生如月一样。新月与残月形态相同,而人生的暮年与童年虽形态不同,但人可以把握住现在,活得自在、洒脱。‎ 解析 首联“一钩曾挂暮霞里,半玦还悬晓雾中”直接点明题意,描写出缺月的形态。作者用“一钩”“半玦”两个词形象生动地描绘出新月与残月的特点,采用互文的形式将后者进行对照。“挂”“悬”二字写月增添了画面动感。尾联作者进一步抒发自己的感想,“皓首又如童”表达了作者对人生的美好愿望。‎ ‎13.阅读下面这首宋诗,完成(1)~(2)题。‎ 春游湖 徐俯 双飞燕子几时回?夹岸桃花蘸水开。‎ 春雨断桥人不度,小舟撑出柳阴来。‎ ‎(1)“双飞燕子几时回”一句在诗中有什么作用?整首诗表达了诗人怎样的情感?‎ 答:___________________________________________________‎ ‎___________________________________________________________‎ ‎___________________________________________________________‎ 答案 诗的开头巧用疑问,表现了诗人对春天到来的惊讶和喜悦,为全诗奠定了愉悦、明快的感情基调。自然引出下文对春天美景的描写和赞美。整首诗表达了作者对大自然的热爱和对美好春天的喜爱之情。‎ 解析 本题考查鉴赏诗歌的情感和表达技巧的能力。解答本题第一问,首先要理解诗句的内容,其次要把握蕴藏其中的情感,同时还要考虑其在结构上的作用。第二问,鉴赏诗歌的情感,要善于抓住诗中的主要意象,如“双飞燕子”“夹岸桃花”“春雨”等,由此体会诗人对春天的喜爱和赞美之情。‎ ‎(2)诗的三、四两句描写了哪几幅画面?这样写有什么妙处?请简要分析。‎ 答:___________________________________________________‎ ‎___________________________________________________________‎ ‎___________________________________________________________‎ 答案 一句一景,共两幅画面:一是雨后水涨,小桥被淹没,游人“不度”;二是柳阴深处,撑出小船,有人可以摆渡,继续游赏。这样写生动形象地表现了游人内心的波澜起伏:在观赏美好春景极其愉悦之时,突因断桥阻挡去路而失落,后又有小舟摆渡给人带来惊喜,一波三折,使得这次春游更富有情趣了。‎ 解析 本题考查鉴赏诗歌形象和表现手法的能力。解答此题,先要抓住诗中的主要意象,如“春雨断桥”“小舟”“柳阴”,再结合表现动作的词,如“不度”“撑出”,便可概括出两幅形象鲜明的画面,然后分析其中蕴含的情感,便可体味其妙处。‎ ‎14.[2016·武邑中学模拟]阅读下面这首宋词,完成问题。‎ 侧犯·咏芍药 姜夔 恨春易去,甚春却向扬州住。微雨,正茧栗梢头弄诗句。红桥二十四,总是行云处。无语,渐半脱宫衣笑相顾。‎ 金壶细叶,千朵围歌舞。谁念我、鬓成丝,来此共尊俎。后日西园,绿阴无数。寂寞刘郎,自修花谱。‎ 词的下片分别描写了芍药和词人怎样的形象?表达了词人怎样的情感?‎ 答:___________________________________________________‎ ‎___________________________________________________________‎ ‎___________________________________________________________‎ 答案 ①‎ 词的下片描写了正在盛开的芍药花,花朵金红,绿叶细密,人们在花丛中尽情地唱歌跳舞,刻画了芍药的美丽形象。②“鬓成丝”“自修花谱”等描写了词人年事已老,自觉迟暮而孤寂的形象。③词人通过对芍药花的盛开和人们的热闹的描写,对比自己的孤独和寂寞,表达词人的孤寂和悲凉之情。‎ 解析 本题考查赏析诗歌形象及情感的能力。概括人物、事物、景物等的形象,注重把握诗歌对这些内容的描写。比如词的下片对芍药的描写有两句:“金壶细叶”“千朵围歌舞”。第一句是正面描写,表现芍药花的美丽;第二句是侧面描写,通过人们的喜欢来表现芍药花的美丽。至于词人的情感,词中的“鬓成丝”“自修花谱”等表现了词人的孤寂、悲凉等。‎ ‎15.阅读下面这首元曲,完成(1)~(2)题。‎ ‎[南吕]四块玉·洞庭湖 马致远 画不成,西施女,她本倾城却倾吴。高哉范蠡乘舟去。那里是泛五湖?若纶竿不钓鱼,便索他学楚大夫[注]。‎ ‎[注] 楚大夫:楚人大夫文种,与范蠡同入越国辅佐勾践灭吴,后为勾践所杀。‎ ‎(1)第三句的两个“倾”意思是否一样?请简要分析。‎ 答:___________________________________________________‎ ‎___________________________________________________________‎ 答案 不一样。第一个“倾”的意思是“倾国倾城”,表现了西施美丽的容貌。第二个“倾”指“倾覆”,说明了吴国的灭亡是因为西施。‎ 解析 首先要理解“倾”这个字在句子中的含义,然后再解读出表达的情感。‎ ‎(2)最后三句表达了作者怎样的情感?请简要分析。‎ 答:___________________________________________________‎ ‎___________________________________________________________‎ 答案 最后三句用一个反问、一个假设,指出范蠡避祸远害是不得已之举,揭露了越王勾践的刻薄寡恩和残酷无情,表达作者对统治者的不满和愤懑之情。‎ 解析 本题考查鉴赏诗歌思想情感的能力。鉴赏诗歌的思想情感,需要抓住诗歌中表现情感的字、词,理解诗句的含义,在此基础上进行概括,同时要注意是否运用了表达技巧。此三句意思是:“难道真的是想泛舟在五湖上?如果他不是拿起钓竿去钓鱼,就会像那楚大夫一样落个被杀害的下场。”一个反问,一个假设,表达了作者对统治者的不满和愤懑。‎ ‎16.阅读下面这首清词,完成下面问题。‎ 杨柳枝 万树 不合临池起画楼,断烟疏雨叶飕飕。‎ 谁能数得垂杨柳?一叶垂杨一点愁。‎ 首句的“不合”表现了词中人物怎样的情感?请结合词句简要分析。‎ 答:___________________________________________________‎ ‎___________________________________________________________‎ 答案 表达了词中人物的苦闷和愁怨。“不合”在这里是“不应该”的意思,表达了词中人物对自然景色的无理责备。因为池塘的美景与杨柳的美丽一起进入词中人物的眼底,本来内心就充满愁思的词中人物更加忧愁,怎么能够受得了!‎ 解析 本题考查赏析诗歌情感的能力。赏析诗歌表达的情感,要注意把握诗歌中的语句、词语表述的内容以及特点。“不合”是不应该的意思,体现的是词中人物的无理,而这种无理恰恰表现了词中人物的情感,即通过对如此美景的抱怨表达人物的苦闷。‎ ‎17.阅读下面这首唐诗,完成(1)~(2)题。‎ 南征[注]‎ 杜甫 春岸桃花水,云帆枫树林。‎ 偷生长避地,适远更沾襟。‎ 老病南征日,君恩北望心。‎ 百年歌自苦,未见有知音。‎ ‎[注] 此诗是大历四年(769)春,杜甫由岳阳前往长沙途中所作。此时距他去世只有一年。‎ ‎(1)与《登高》相比,本诗的景与情关系有何不同?请简要分析。‎ 答:___________________________________________________‎ ‎___________________________________________________________‎ 答案 ①《登高》以哀景写哀情:借萧瑟凄凉的悲秋之景来抒发作者的孤苦悲凉之感。②本诗以乐景写哀情:借美妙迷人的春江之景来反衬作者愁苦悲哀、无以自遣的心境。‎ 解析 《南征》以明媚的江上春光开头,接着又让“偷生”“适远”的沾襟泪水把明朗欢快的气息抹洗得干干净净,诗人正于此不协调处展现自己内心深处的苦恼。《登高》则是诗人晚年在极端困窘的情况下写成的,那一天,他独自登上夔州白帝城外的高台,登高临眺,百感交集,望中所见,激起意中所触,萧瑟的秋江景色,引发了他身世飘零的感慨,渗入了他老病孤愁的悲哀。‎ ‎(2)本诗表达了作者什么样的复杂心情?请简要分析。‎ 答:___________________________________________________‎ ‎___________________________________________________________‎ 答案 ①羁旅悲愁:常年躲避战乱,漂泊偏远的地方,偷生落泪。②垂老多病之叹:作者已近垂暮,疾病缠身,孤苦伶仃,满怀悲叹。③心系朝廷:内心不忘君恩,总是北望长安。④苦无知音:多年自吟自叹,没有知音倾听理解。‎ 解析 这首诗反映了诗人死前不久极度矛盾的思想感情。这些情感着重体现在诗歌的后三联。“偷生”两句表现了诗人长年颠沛流离的羁旅悲愁之苦;“老病”两句表现了诗人已是年老多病之身,按理应当北归长安,然而命运却迫使他南往衡湘之苦;最后两句,表现了诗人一生苦吟,却不被人理解、赏识之苦。虽然如此,但“君恩北望心”一句表现出诗人渴望报效朝廷的情怀。‎ ‎18.阅读下面这首宋诗,完成问题。‎ 酒楼秋望 ‎[宋]华岳[注]‎ 西风吹客上阑干,万里无云宇宙宽。‎ 秋水碧连天一色,暮霞红映日三竿。‎ 花摇舞帽枝尤软,酒入诗肠句不寒。‎ 古往今来恨多少,一时收拾付杯盘。‎ ‎[注] 华岳:南宋著名爱国诗人。这首诗作于南宋晚期,国家正处于危难之中。‎ 与《过零丁洋》尾联相比,本诗尾联抒情方式有何不同?请简要分析。‎ 答:___________________________________________________‎ ‎___________________________________________________________‎ 答案 与《过零丁洋》尾联的直抒胸臆有不同。本诗尾联前句直接抒情(或直抒胸臆),将个人感情融入古往今来的历史之中,拓宽了情感的深度和广度;后句间接抒情,运用反语的手法,写诗人表面上姑且将心中的忧愤之情交付给菜盘和酒杯,实则是愤激之语,表达的感情更加激越。‎ 解析 《过零丁洋》尾联直抒胸臆,表现了诗人为国家安宁愿慷慨赴死的民族气节,使全篇由悲而壮,由郁而扬,形成一曲千古不朽的壮歌,激励和感召古往今来无数志士仁人为正义事业英勇献身。而《酒楼秋望》尾联充分表达了诗人报国无路(或山河破碎、收复无望)的无奈与忧愤之情,前半句直抒胸臆,后半句则运用了反语的手法。‎ ‎19.阅读下面这首元曲,完成问题。‎ ‎[双调]沉醉东风·重九 ‎[元]卢挚 题红叶清流御沟,赏黄花人醉歌楼。天长雁影稀,月落山容瘦。冷清清暮秋时候。衰柳寒蝉一片愁,谁肯教白衣送酒[注]?‎ ‎[注] “白衣送酒”的典故出自南朝宋檀道鸾《续晋阳秋》:“‎ 陶潜九月九日无酒,于宅边菊丛中摘盈把,坐其侧久,望见白衣至,乃王弘送酒也,即便就酌,醉而后归。”‎ 这首曲包含多种情感,请结合全曲进行赏析。‎ 答:___________________________________________________‎ ‎___________________________________________________________‎ 答案 ①陶醉于赏黄花之中的快乐。第一句写作者闲适地在红叶上题诗让流水带走作者的情义,观赏菊花后醉卧歌楼,表达了作者陶醉于赏黄花之中的快乐。②因秋景冷清油然而生的内心的孤独。看到万里长空的雁影稀疏,远山狭长清瘦,暮秋时节到处冷冷清清,作者内心的孤独冷清之情油然而生。③年华老去,无知己相伴的愁苦。这时节,无人送酒,一起解忧,年华老去,表达了作者无知己相伴的愁苦。‎ 解析 本题考查鉴赏诗歌情感的能力。概括时一定要参考注释,分析时要结合具体诗句。‎ ‎20.阅读下面这首诗,完成问题。‎ 群鹤咏①‎ ‎[南齐]萧道成 八风儛②遥翮③,九野弄清音。‎ 一摧云间志,为君苑中禽。‎ ‎[注] ①此诗作于诗人镇守淮阴时。诗人因其功勋卓著,名位日隆,颇受猜忌,为宋明帝所疑,被调进京城,担任黄门郎,便于监视。②儛:通“舞”。③翮(hé):本义为羽毛的根部,这里代称鸟翅。‎ 试分析此诗主要采用的抒情手法以及抒发的情感。‎ 答:___________________________________________________‎ ‎___________________________________________________________‎ 答案 托物言志或借鹤自喻。表达自己被召回之后志向受损,处处受束缚的苦闷不平之情,以及对往日那种无拘无束、胸存壮志的生活的无限怀念之情。‎ 解析 本题考查鉴赏诗歌表现手法和诗人情感的能力。答题时,可根据抒情方式的类型,结合注释,抓住诗歌关键词分析作答。由注释可知,诗人因其功勋卓著受到猜疑而被调入京都监视,才华得不到施展,心情郁闷感伤,而诗歌中所写的鹤的遭遇与自己相似,故从写作意图的角度分析,可知诗人采用托物言志的手法来表达自己被召回后的苦闷心情以及对过去自由、雄心壮志的怀念之情。‎ ‎21.阅读下面这首宋词,完成(1)~(2)题。‎ 踏莎行·庚戌中秋后二夕带湖篆冈小酌[注]‎ 辛弃疾 夜月楼台,秋香院宇。笑吟吟地人来去。是谁秋到便凄凉?当年宋玉悲如许。‎ 随分杯盘,等闲歌舞。问他有甚堪悲处?思量却也有悲时,重阳节近多风雨。‎ ‎[注] 庚戌:即宋绍熙元年(1190)。中秋后二夕:中秋节后的第二个晚上。篆冈:地名,当在带湖之侧。‎ ‎(1)词的上片四句中前两句描绘了怎样的景与情?在全词中有什么作用?请简要分析。‎ 答:___________________________________________________‎ ‎___________________________________________________________‎ ‎___________________________________________________________‎ 答案 描写了带湖秋夜的幽美景色,抒发了人们赏秋、乐秋之情。楼台为皎洁的明月所照亮,庭院里散发出秋花秋果的清香。赏景的人在美景下来来往往,也都是“笑吟吟地”。为下文作者抒发风雨悲秋之意做铺垫(蓄势)。‎ 解析 本题考査理解诗句内容、鉴赏作品的表达技巧的能力。需要在理解文意的基础上分析。“夜月楼台,秋香院宇”二句对起,以工整清丽的句式描绘出迷人的夜景:在清凉幽静的篆冈,秋月映照着树木荫蔽的楼台,秋花在庭院里散发着扑鼻的幽香。第三句“笑吟吟地人来去”,转写景中之人,十分自然。这七字除了一个名词“人”之外,全用动词或副词,衬以一个结构助词“地”,使得人物动态活灵活现,欢乐之状跃然纸上。秋景是如此令词人和他的宾客们赏心悦目,他不禁要想,为什么自古以来总有些人一到秋天就悲悲戚戚呢?很自然地引起下文的内容。‎ ‎(2)词的最后两句表达了作者什么样的思想感情?请结合全词加以分析。‎ 答:___________________________________________________‎ ‎___________________________________________________________‎ ‎___________________________________________________________‎ 答案 “思量却也有悲时,重阳节近多风雨。”这两句表面上写想来秋天也会使人悲伤,忧虑重阳节时多风多雨的天气。实际上“风雨”,一语双关(或比兴,或象征),“风雨”不仅指自然气候,更暗喻南宋国运日衰,政治形势之险恶。(或悲秋不仅仅为秋日自然风雨悲伤,更为南宋国运日衰、风雨飘零而悲)通过时节变化的描写反映作者对当时政局恶劣、国势衰微的忧虑之情,对国家、民族命运的悲叹。‎ 解析 结合作者的经历、时代背景及诗歌的内容综合理解。在上片“是谁秋到便凄凉”一个问句之后,作者又在下片着力地加上了一个意思更明显的反问,把自己本欲肯定的东西故意推到了否定的边缘。末二句突然作了一个笔力千钧的反跌:“思量却也有悲时,重阳节近多风雨。”这一反跌,跌出了此词悲秋的主题思想,把上面大部分篇幅所极力渲染的“不必悲”“有甚悲”等意思全盘推翻了。到此人们方知,一代豪杰辛弃疾也是在暗中悲秋的。他悲秋的理由是,重阳节快来了,那凄冷的风风雨雨将会破坏人们的幸福和安宁。“重阳节近多风雨”一句,化用北宋人潘大临咏重阳的名句“满城风雨近重阳”,这正是王国维《人间词话》所说的“借古人之境界为我之境界”。辛弃疾之所谓“风雨”,一语双关,既指自然气候,也暗喻政治形势之险恶。‎ ‎22.阅读下面这首词,完成问题。‎ 江城子·病起春尽 陈子龙[注]‎ 一帘病枕五更钟,晓云空,卷残红。无情春色,去矣几时逢?添我千行清泪也,留不住,苦匆匆。‎ 楚宫吴苑草茸茸,恋芳丛,绕游蜂。料得来年,相见画屏中。人自伤心花自笑,凭燕子,舞东风。‎ ‎[注] 陈子龙:明末文学家,本词大约作于南明王朝灭亡后一年。‎ 这首词表现了作者什么样的情感?请简要分析。‎ 答:___________________________________________________‎ ‎___________________________________________________________‎ 答案 ①伤春之情。作者“病起”发现春色已匆匆逝去,无法挽留,只能用“千行清泪”来表达对春天逝去的伤感。②亡国之恨、故国之思(对故国的悼念、怀念之情)。作者由“卷残红”想到故国的覆亡如同这无情春色一样逝去难留,不觉悲从中来,泪水潸然;楚宫吴苑曾经芳草丰茸,百花盛开,游蜂盘绕,但昔日春景来年只能在画屏中相见,更添故国之思和思而不得见的酸楚。(或作者想到楚宫吴苑长满丰茸的芳草,百花盛开,游蜂盘绕,而来年又会见到这画屏般秀丽的春色,可南明王朝灭亡,故国不在,只能独自伤心。)‎ 解析 对诗歌的思想内容和情感态度的把握要从以下几个方面入手:一要从作者所处的时代背景和个人经历入手分析;二要从作品描写的关键词入手品味;三要借助于诗歌中景物的特点来分析。本题结合注释可知,作者含有亡国之恨;再结合标题“病起春尽”以及“留不住,苦匆匆”等句去分析,应含有惜春之情。‎ ‎23.阅读下面这首宋词,完成问题。‎ 酒泉子 潘阆 长忆西湖。尽日凭阑楼上望:三三两两钓鱼舟,岛屿正清秋。‎ 笛声依约芦花里,白鸟成行忽惊起。别来闲整钓鱼竿,思入水云寒。‎ 词的下阕是如何表达作者的思想情感的?请简要分析。‎ 答:___________________________________________________‎ ‎___________________________________________________________‎ 答案 ①“笛声依约芦花里,白鸟成行忽惊起”借景抒情,通过描写从芦花荡里隐隐约约传送出来的渺远的笛声,忽然惊起栖息在芦花荡里的白鸟,从而衬托出环境的清幽、空旷、辽远,表达了作者对此境的喜爱和向往之情;②“闲整”“思入”两个动词把作者收拾鱼竿、急欲赴西湖垂钓的动作、心理形象地表现出来,衬托了作者忆西湖忆得不能再忍耐、亟想归隐湖上的心情。‎ 解析 下阕两句,继续写当日楼上见闻,前一句写声,“依约”是隐约、听不分明的意思,摹笛声渺茫幽远、似有若无的韵致;后句写形,用“忽惊起”‎ 状白鸟(即白鹭)翩然而逝、倏然而惊的形态,色彩明快,颇具情味,朴实的白描中透出空灵。“别来”二字将思路从回忆拉到现实。“闲整钓鱼竿”不仅应上阕之“钓鱼舟”,而且以收拾鱼竿、急欲赴西湖垂钓的神情,衬托忆西湖忆得不能忍耐、亟想归隐湖上的念头。词之下阕,营造出钓翁渔隐没的寥廓苍茫的背景,寓情于景,寄托了作者的“出尘”思想。‎ ‎24.阅读下面这首宋诗,完成问题。‎ 初冬道中 ‎[宋]曾巩 潦退蛟螭不可逃,溪潭清澈见秋毫。‎ 欲霜日射西山赤,渐冷天腾北极高。‎ 秀色更浓唯竹柏,孤根先动是蓬蒿。‎ 感时一抚青萍[注]叹,马踏西风气自豪。‎ ‎[注] 青萍:古宝剑名,泛指剑。‎ 这首诗表现了诗人怎样的志趣?请简要分析。‎ 答:___________________________________________________‎ ‎___________________________________________________________‎ 答案 溪潭清澈,写其品行纯洁;日射天腾,见其志向高远;竹柏后凋,明其坚守之意;青萍之叹,叙其壮志难酬;马踏西风,抒其凌云豪情。‎ 解析 根据诗歌中“景情”关系(借景抒情、情景交融)逐联分析。‎ ‎ [3年高考真题集训]‎ ‎1.[2016·全国卷Ⅰ]阅读下面这首唐诗,回答(1)~(2)题。‎ 金陵望汉江 李白 汉江回万里,派作九龙盘①。‎ 横溃豁中国,崔嵬飞迅湍。‎ 六帝沦亡后②,三吴不足观③。‎ 我君混区宇,垂拱众流安。‎ 今日任公子,沧浪罢钓竿④。‎ ‎[注] ①派:河的支流。长江在湖北、江西一带,分为很多支流。②六帝:代指六朝。③三吴:古吴地后分为三,即吴兴、吴郡、会稽。④这两句的意思是,当今任公子已无须垂钓了,因为江海中已无巨鱼,比喻已无危害国家的巨寇。任公子是《庄子》中的传说人物,他用很大的钓钩和极多的食饵钓起一条巨大的鱼。‎ ‎(1)诗的前四句描写了什么样的景象?这样写有什么用意?‎ 答:____________________________________________________‎ ‎____________________________________________________________‎ 答案 这四句描写了江水万流横溃、水势浩瀚、气势宏大的景象。作者以此为下文颂扬盛唐天下一家、国运兴盛积蓄气势,有利于突出诗的主旨。‎ 解析 诗的前四句寓情于景:汉江延绵曲折长达万里,分作九条支流就如同九条巨龙盘踞。江水四溢,泛滥于广大中原地区,波涛汹涌,迅疾奔流。写出了远去的汉江气势浩大,特别是三、四句用江水泛滥造成的巨大影响和损失来写前朝国运不兴,为歌颂当下盛世蓄势。‎ ‎(2)诗中运用任公子的典故,表达了什么样的思想感情?‎ 答:____________________________________________________‎ ‎____________________________________________________________‎ 答案 ①作者以水无巨鱼代指世无巨寇,表达了对大唐一统天下、开创盛世伟绩的歌颂;②作者自比任公子,觉得在太平盛世没有机会施展才干,不免流露出一丝英雄无用武之地的失落。‎ 解析 题目不仅指出了用典的表达技巧,而且还指出具体典故。答题时只要指出用典在主题情感上的表达作用即可。首先阅读注释,了解任公子的典故。典故中的人、事、物具有比喻意义:“任公子”‎ 代表贤臣良将,“垂钓”指平定叛乱,“巨鱼”比喻危害国家的巨寇,“很大的钓钩和极多的食饵”喻指费很大的力气。再联系诗歌语境,把握思想情感。六代的帝王沉寂沦亡之后,三吴已没有了昔日之盛,无足称赏。本朝圣明之君统一天下,垂衣拱手,无为而治。今天的任公子,已不需要沧海垂钓而罢竿了。然后结合背景,知人论世。当时的唐王朝正处在开元盛世之时,天下一统,海天清晏。诗中反用“任公子东海钓巨鱼”的典故,来表达作者江汉宁静、世无巨寇的社会理想。但盛世之下,诗人空有大才却无用武之地,不免徒增伤悲。‎ ‎2.[2016·全国卷Ⅱ]阅读下面这首唐诗,回答(1)~(2)题。‎ 丹青引赠曹将军霸①(节选)‎ 杜甫 先帝天马玉花骢②,画工如山貌不同。‎ 是日牵来赤墀下③,迥立阊阖生长风④。‎ 诏谓将军拂绢素,意匠惨淡经营中。‎ 斯须九重真龙出⑤,一洗万古凡马空。‎ ‎[注] ①曹将军霸:即曹霸,唐代著名画家,官至左武卫将军。②玉花骢:唐玄宗御马名。③赤墀:宫殿前的红色台阶。④阊阖:传说中的天门,这里指宫门。⑤斯须:一会儿。‎ ‎(1)如何理解曹霸画的马“一洗万古凡马空”?曹霸是怎样做到的?请简要分析。‎ 答:____________________________________________________‎ ‎____________________________________________________________‎ 答案 第一问:曹霸所画玉花骢神奇雄俊,如飞龙跃出,其他人画的“凡马”在此马面前都不免相形失色。‎ 第二问:曹霸先凝神构思,苦心布局,然后落笔挥洒,顷刻之间一气呵成。‎ 解析 本题考查理解、鉴赏诗歌语言的能力。第一问,“一洗万古凡马空”可翻译为“一下比得万代凡马皆成了平庸”。写出了曹霸所画玉花骢的神奇雄俊,与众不同。第二问,可结合“意匠惨淡经营中”“斯须九重真龙出”两句理解,这两句可翻译为“你匠心独运惨淡经营刻苦用功”“片刻间九天龙马就在绢上显现”,由此可见,曹霸画马首先凝神构思,苦心布局,然后落笔挥洒,一气呵成。‎ ‎(2)为了突出曹霸的高超画技,诗人作了哪些铺垫?请简要分析。‎ 答:____________________________________________________‎ ‎____________________________________________________________‎ 答案 ①画工如山貌不同:写曹霸要画的马已有众多画工画过,但画得都不成功。强调此马的雄俊非凡手可得,造成此马难画的印象;②迥立阊阖生长风:写真马昂头站立,给人万里生风之感,进一步点出画家要捕捉住此马飞动的神采尤其不易。‎ 解析 本题考查鉴赏诗歌结构技巧的能力。首先,对全诗进行整体理解把握,准确翻译诗歌语言,然后思考“作了哪些铺垫”。“画工如山貌不同”写多少画家画唐玄宗的御马玉花骢,都与原貌不同,更不能与其神似。用“生长风”形容玉花骢的神奇雄俊。‎ ‎3.[2016·全国卷Ⅲ]阅读下面的宋诗,回答(1)~(2)题。‎ 内宴奉诏作 曹翰①‎ ‎ 三十年前学六韬②,英名尝得预时髦③。‎ 曾因国难披金甲,不为家贫卖宝刀。‎ ‎ 臂健尚嫌弓力软,眼明犹识阵云④高。‎ 庭前昨夜秋风起,羞见盘花旧战袍。‎ ‎[注] ①曹翰(923~992),宋初名将。②六韬:古代兵书。③时髦:指当代俊杰。④阵云:战争中的云气,这里有战阵之意。‎ ‎(1)诗的颈联又作“臂弱尚嫌弓力软,眼昏犹识阵云高”,你认为哪一种比较好?为什么?请简要分析。‎ 答:____________________________________________________‎ ‎____________________________________________________________‎ 答案 观点一:作“弱”“昏”好。‎ ‎①“臂弱”“眼昏”表明作者承认自己年老体衰的客观事实,但强调即便如此,也还是能够去冲锋陷阵;②更强烈地表现出作者只要一息尚存,就不忘杀敌报国的刚毅精神。‎ 观点二:作“健”“明”好。‎ ‎①“臂健”“眼明”表明作者认为虽然岁月流逝,但身体依然强健,当然还可以冲锋陷阵,为国驱驰;②表现出作者心存随时准备杀敌报国的坚定信念,而忘记自己老之将至。‎ 解析 此题为开放性试题。若认为作“健”“明”好,则联系诗句,分三步解答即可:第一步,解释含义,“臂健”“眼明”指臂膀健硕、眼睛明亮;第二步,将其放在语境里理解,“尚嫌”“犹识”强调尽管岁月流逝,但身体强健,仍识得战阵;第三步,体会感情,全诗表达了作者随时准备杀敌报国的坚定信念。认为“弱”“昏”两字较好的作答也可分为以上三步:“弱”“昏”指体弱眼花,表达作者虽承认自己年老体衰,但依旧不忘杀敌报国的刚毅精神。‎ ‎(2)这首诗与辛弃疾的《破阵子(醉里挑灯看剑)》题材相似,但情感基调却有所不同,请指出二者的不同之处。‎ 答:____________________________________________________‎ ‎____________________________________________________________‎ 答案 ①曹诗写自己虽已年老,但报国之心犹存,重在表达“老骥伏枥,志在千里”的豪情;②辛词通过追怀金戈铁马的往事,表达英雄白首、功业未成的悲慨。‎ 解析 由“三十年前学六韬”可知作者此时年事已高;“不为家贫卖宝刀”表示作者一直把宝刀藏在身边,时刻准备为国效力;“尚嫌弓力软”“犹识阵云高”表明作者认为自己还能上阵杀敌。这首诗表达了作者虽然年事已高、但报国之心犹存的思想感情。辛弃疾《破阵子》的末句“可怜白发生”点出了全词的情感基调,以沉痛的笔调,表达了英雄白首、功业未成的悲慨。‎ ‎4.[2016·山东高考]阅读下面的元曲,回答(1)~(2)题。‎ 水仙子·舟中 孙周卿 孤舟夜泊洞庭边,灯火青荧对客船,朔风吹老梅花片。推开篷雪满天。诗豪与风雪争先,雪片与风鏖战,诗和雪缴缠。一笑琅然。‎ ‎(1)分析“诗豪与风雪争先,雪片与风鏖战,诗和雪缴缠”使用的两种修辞手法。‎ 答:____________________________________________________‎ ‎____________________________________________________________‎ 答案 比拟、排比。“诗豪与风雪争先”“雪片与风鏖战”,用“争先”“鏖战”把“诗豪”“风”和“雪”拟人化,“诗和雪缴缠”,用“缴缠”将“诗”拟物,把抽象的“诗”具象化,生动形象地描写风雪交加的壮美,表现作者迸发的诗情。“诗豪与风雪争先,雪片与风鏖战,诗和雪缴缠”构成排比句,描写了作者的诗情与风雪难分难解的关系,渲染了气氛。‎ 解析 “诗豪与风雪争先,雪片与风鏖战,诗和雪缴缠”三句,着重写了“朔风”陡至之后,作者看到的漫天风雪的情形,作者将风雪拟人化,用“豪”字,表现风雪的猛烈,用“鏖战”表现风雪的雄劲,用“缴缠”拟物化表现风雪的桀骜。这三句运用排比,增强气势,逐步加深对环境的烘托作用。漫天风雪,难分难解,景象开阔,风格豪迈。‎ ‎(2)结合作品,简要分析作者的情感变化。‎ 答:____________________________________________________‎ ‎____________________________________________________________‎ 答案 孤舟夜泊、青荧客船、朔风等,表现了作者的孤独之感、羁旅之思;漫天飞雪激发了作者的创作豪情,风雪鏖战,“诗豪”与风雪争先,“诗”又与雪缴缠,表现了作者啸傲孤独与风雪的豪迈气概;“一笑琅然”,抒发了作者战胜困境的快意和乐观旷达的情怀。‎ 解析 前三句,孤舟无伴,船外白茫茫一片,朔风劲烈,写出了作者的孤寂悲冷。“推开篷”一句,因为朔风骤起,却有了“雪满天”的全新发现,事出意外,惊喜顿生。最后“一笑琅然”,作者的豪情快意顿时将先前的孤寂悲冷一扫而光。‎ ‎5.[2016·天津高考]阅读下面的诗,按要求作答(1)~(3)题。‎ 登裴秀才迪小台 ‎[唐]王维 端居不出户,满目望云山。‎ 落日鸟边下,秋原人外闲。‎ 遥知远林际,不见此檐间。‎ 好客多乘月,应门莫上关。‎ ‎(选自《全唐诗》)‎ ‎(1)“满目望云山”句中“望”字一作“空”,你认为这两个字用哪个更好?请说明理由。‎ 答:____________________________________________________‎ ‎____________________________________________________________‎ 答案 “望”:照应题目中的“登台”,引出后面描写的景物。或“空”:①营造空旷的意境;②流露出超然的心态。‎ 解析 这是一道炼字赏析题,首先确定用哪个字更好,然后可从结构、内容等方面来说明理由。如认为“望”更好,可从与题目相照应、与后文的关系等方面说明理由;如认为“空”更好,可考虑从其所传达的意思、所传递的情感、所营造的意境等方面说明理由。‎ ‎(2)请结合诗句说明颔联采用了哪些表现手法?‎ 答:____________________________________________________‎ ‎____________________________________________________________‎ 答案 ①动静结合。落日与鸟,是动态描写;秋日原野,是静态描写。②寓情于景。通过描写秋原的空阔,表现出诗人闲适的心境。‎ 解析 常见的表现手法有借景抒情、寓情于景、动静结合、虚实结合、渲染、联想等。本诗颔联中,“落日”与“鸟”是动态描写,“秋日”和“原野”是静态描写,由此可知采用了动静结合的手法;“秋原人外闲”,用秋原的空阔来表现诗人的心境,可知还采用了寓情于景的手法。‎ ‎(3)你如何理解诗中的“闲”字?‎ 答:____________________________________________________‎ ‎____________________________________________________________‎ 答案 一个“闲”字,点出闲景、闲人、闲心,写出了闲境之美、闲适之情。‎ 解析 这道题问的是对一个字的理解,实际上是要求扣住“诗眼”进行分析。“你如何理解”,既可以写出你对诗中所写“闲”字的内涵的理解,也可以写自己由此诗生发的联想。‎ ‎6.[2015·全国卷Ⅰ]阅读下面这首唐诗,完成(1)~(2)题。‎ 发临洮将赴北庭留别①‎ 岑参 闻说轮台②路,连年见雪飞。‎ 春风不曾到,汉使亦应稀。‎ 白草通疏勒,青山过武威。‎ 勤王敢道远,私向梦中归。‎ ‎[注] ①临洮:在今甘肃临潭西。北庭:唐六都护府之一,治所为庭州(今新疆吉木萨尔北)。②轮台:庭州属县,在今新疆乌鲁木齐。‎ ‎(1)与《白雪歌送武判官归京》相比,本诗描写塞外景物的角度有何不同?请简要分析。‎ 答:____________________________________________________‎ ‎____________________________________________________________‎ 答案 本诗描写的边塞风光并非作者亲眼所见,而是出于想象。属于侧面描写。从标题可以看出,作者此时尚处于前往边塞的途中;开头“闻说”二字也表明后面的描写是凭听闻所得。‎ 解析 题目要求将此诗与《白雪歌送武判官归京》对比,对诗歌描写景物的角度进行分析。《白雪歌送武判官归京》是正面描写壮丽雪景的,而此诗中,一个“闻说”,告诉我们描写的对象是听说,是侧面的。由此进行分析,可以得到正确的答案。‎ ‎(2)诗的尾联表达了作者什么样的思想感情?对全诗的情感抒发有怎样的作用?‎ 答:____________________________________________________‎ ‎____________________________________________________________‎ 答案 第一问:表现了诗人虽有羁旅思乡之愁,却能以国事为重的爱国热忱。‎ 第二问:使得诗中的思乡之情不至流于感伤,也提升了全诗的格调。‎ 解析 题目要求对诗歌后两句所表达的情感及其在全诗中的作用进行分析,具体思考时,可以围绕作者的身份以及当时的环境进行。面对那种荒凉的环境,作者仍然是满怀报国之志的,这是作者英雄的一面;而对家乡的思念,又是人之常情。报国情怀重于思乡之愁,使诗歌多了一份豪迈,不至流于感伤,这种舍小为大的情怀也提升了全诗的格调。明确了这些,便可以得到准确的答案。‎ ‎7.[2015·全国卷Ⅱ]阅读下面这首唐诗,完成(1)~(2)题。‎ 残春旅舍 韩偓①‎ 旅舍残春宿雨晴,恍然心地忆咸京②。‎ 树头蜂抱花须落,池面鱼吹柳絮行。‎ 禅伏诗魔归净域,酒冲愁阵出奇兵。‎ 两梁③免被尘埃污,拂拭朝簪④待眼明。‎ ‎[注] ①韩偓(约842~923):字致尧,京兆万年(今陕西西安)人。这首诗是作者流徙闽地时所作。②咸京:这里借指都城长安。③梁:官帽上的横脊,古代以梁的多少区分官阶。④朝簪:朝廷官员的冠饰。‎ ‎(1)古人认为这首诗的颔联“乃晚唐巧句”,请指出这一联巧在哪里,并简要赏析。‎ 答:____________________________________________________‎ ‎____________________________________________________________‎ 答案 ①构思精妙,把“花须落”“柳絮行”这些常见的残春景象与“蜂抱”“鱼吹”联系起来,十分新奇;②用词巧妙,“抱”“吹”的使用虽然出人意料,却又显得非常自然。(意思答对即可)‎ 解析 本诗首联写到暮春时节作者流落他乡,寄居旅舍之内,下了一夜的雨,早晨刚放晴,残花满地。面对此情此景,作者恍然间忆起皇都长安。颔联两句承接首联的“残春”,描写了一幅晚春动态图景。春天将尽,残花纷纷飘落,柳絮在水面浮动,这样的景象在暮春时节比较常见,本无新奇之处。可是作者在这里却巧妙地将这些景物与“蜂”“鱼”联系起来,好像“花须落”“柳絮行”是“蜂抱”“鱼吹”的结果,构思十分新奇。动词“抱”“吹”的选用也非常巧妙,有点出人意料,可仔细品味之却又显得非常自然。蜂“抱”、鱼“吹”使得本来容易让人产生感伤情绪的暮春场景立刻变得鲜活了起来,颇显出几分生机。这让人不由得不佩服作者锤炼词句的高超技艺。这一联深受后人推崇,被认为写得工巧。所谓“乃晚唐巧句”见于明代许学夷的《诗源辩体》。‎ ‎(2)这首诗的后两联表达了作者什么样的感情?请简要分析。‎ 答:____________________________________________________‎ ‎____________________________________________________________‎ 答案 表达了作者内心孤寂愁苦,但仍忠于大唐、心系故国之情。通过参禅使自己平静,通过饮酒化解“愁阵”,表明他内心孤寂愁苦;避免染“尘埃”,整理朝冠期待“眼明”,表明他不愿依附奸佞,对大唐一片忠心。(意思答对即可)‎ 解析 作者客居旅舍,触景生情,回忆起皇都风物。记忆中的一切如此温馨,与现实形成了巨大反差,个人的境遇与国家的命运,让作者内心感到无比孤寂愁苦。颈联两句是对作者心境的描写。作者愁苦至极,想通过写诗抒写自己的心境。可这样做并不见效,心中郁结的愁思越积越深,如同重重愁阵横亘胸中,无奈之下,只有通过参禅、饮酒来化解愁苦,力图使自己内心归于平静。同颔联一样,这一联写得也非常出色,“禅伏诗魔”“酒冲愁阵”“出骑兵”,想象十分奇特。尾联收束全篇,表明了作者的希望。整理朝冠,不愿被“尘埃污”,期待“眼明”,表明作者不愿与朱温政权合作,希望大唐能够重新复兴,期待将来还能有机会为朝廷尽力尽职,不负朝冠。表达了作者虽然内心孤寂愁苦,但依然心系国运,对故国的无限眷恋之情。‎ ‎8.[2014·全国卷Ⅰ]阅读下面这首宋词,完成(1)~(2)题。‎ 阮郎归 无名氏①‎ 春风吹雨绕残枝,落花无可飞。小池寒渌欲生漪,雨晴还日西。‎ 帘半卷,燕双归。讳愁②无奈眉。翻身整顿着残棋,沉吟应劫③迟。‎ ‎[注] ①作者一作秦观。②讳愁:隐瞒内心的痛苦。③劫:围棋术语。‎ ‎(1)词上半阕的景物描写对全词的感情抒发起了什么作用?请结合内容分析。‎ 答:____________________________________________________‎ ‎____________________________________________________________‎ 答案 奠定了词的情感基调。春风吹雨,残红满地,词一开始就给人以掩抑低回之感;接下来写风雨虽停,红日却已西沉,凄凉的氛围非但没有解除,反而又被抹上了一层暗淡的暮色。‎ 解析 鉴赏景物描写的作用,首先要理解描写景物的词句,体悟景中蕴含的情感,指出景物描写的句子在结构、主旨表达、抒情方面的作用。‎ ‎(2)末尾两句表现了词中人物什么样的情绪?是如何表现的?请简要阐述。‎ 答:____________________________________________________‎ ‎____________________________________________________________‎ 答案 末尾两句表现了词中人物思绪纷乱、无法排遣的愁情,是通过人物自身的动作来表现的。回身整理残棋并想续下,借以转移愁情,可又因心事重重,以致犹豫不决,落子迟缓。‎ 解析 词中末尾两句,主要是动作描写,抓住所描写的动作的特点,结合上文景中所含之情,不难得出正确答案。分析时,首先要翻译词句,由动作中归纳、提炼体现词人情感的肢体语言。‎ ‎9.[2014·全国卷Ⅱ]阅读下面两首诗,完成(1)~(2)题。‎ 含山店梦觉作 ‎[唐]韦庄①‎ 曾为流离惯别家,等闲挥袂客天涯。‎ 灯前一觉江南梦,惆怅起来山月斜。‎ 宿渔家 ‎[宋]郭震②‎ 几代生涯傍海涯,两三间屋盖芦花。‎ 灯前笑说归来夜,明月随船送到家。‎ ‎[注] ①韦庄(约836~910):字端己,长安杜陵(今陕西西安东南)人。曾流离迁徙于汴洛、吴越等地。②郭震:字希声,成都人。生卒年及生平不详。‎ ‎(1)韦庄在诗中是用什么方法表现感情的?请简要分析。‎ 答:____________________________________________________‎ ‎____________________________________________________________‎ 答案 诗人是用衬托的方法来表现感情的。诗人虽然到处漂泊,但好像对此并不在意,认为这是“等闲”之事;而客中一觉梦醒,思家乡、念亲人的惆怅之情不禁油然而生。(意思答对即可)‎ 解析 韦庄是唐末乱世的漂泊者,他的行迹遍及大半个中国。首句写羁旅漂泊的身世已让他习以为常,其中“惯别家”体现了韦庄的无奈与辛酸。通过“等闲”一词可看出,诗人好像对此并不在意,认为这只是“等闲”之事而已。诗人长久漂泊江南,远离家乡,但思乡、思亲之情深埋心间,挥之不去。一觉醒来,面对无言的山月,这个孤独的游子倍感惆怅。‎ ‎(2)两首诗都写到“灯前”,这两处“灯前”各自表达了诗人什么样的感情?‎ 答:____________________________________________________‎ ‎____________________________________________________________‎ 答案 韦诗中,“灯前”表现了诗人旅途漂泊中的凄清、失神、怅惘之情;郭诗中,“灯前”表现了诗人住宿在渔家所感到的温暖、愉悦之情。(意思答对即可)‎ 解析 韦诗前两句,虽一再表白“流离”“惯别家”“客天涯”不过“等闲”,可客中一觉梦醒,孤苦凄清之情油然而生。把握韦庄“灯前”的感情,关键扣住“惆怅”一词,“惆怅”是诗人情感表达的转折点。‎ 郭诗首句写渔家世代与海为伴,二句写渔家简陋的生活环境,三句写一家人其乐融融,和谐幸福的场景,这让诗人这个游子倍感家的温暖。诗人虽是夜宿渔家,但其漂泊的孤独凄苦被渔家的温暖愉悦融化。把握郭震“灯前”的感情,关键扣住“笑说”一词,“笑说”透露出诗人的情感指向。‎ ‎10.[2015·天津高考]阅读下面这首诗,按要求作答(1)~(3)题。‎ 雨过至城西苏家[注]‎ ‎[宋]黄庭坚 飘然一雨洒青春,九陌净无车马尘。‎ 渐散紫烟笼帝阙,稍回晴日丽天津。‎ 花飞衣袖红香湿,柳拂鞍鞯绿色匀。‎ 管领风光唯痛饮,都城谁是得闲人。‎ ‎[注] 此诗作于宋哲宗元祐元年(1086),黄庭坚时任秘书省校书郎。是年,长期贬谪外放的苏轼被授予翰林学士、知制诰等要职。‎ ‎(1)诗中描写了春雨后的哪些景象?‎ 答:____________________________________________________‎ 答案 尘土涤净,紫烟渐散,雨过日丽,红花沾雨,柳色葱翠。‎ 解析 本小题考查的是诗歌的景象,抓住绘景的意象即可得出答案。‎ ‎(2)结合诗句说明颈联运用了哪些艺术手法。‎ 答:____________________________________________________‎ ‎____________________________________________________________‎ 答案 ①对仗,如“花飞”对“柳拂”,“红香湿”对“绿色匀”。②比拟,如“柳拂”。③从视觉、嗅觉、触觉等多角度(运用通感)进行描写,如“衣袖红香湿”。‎ 解析 本小题考查的是诗歌的表达技巧,诗歌的表达技巧可从修辞手法、表达方式、表现手法方面考虑。点出手法,并结合诗句分析即可。‎ ‎(3)全诗表达了诗人怎样的情感?‎ 答:____________________________________________________‎ ‎____________________________________________________________‎ 答案 雨后天晴访友喜悦之情;仕途上的踌躇满志或忙中偷闲的快乐。‎ 解析 本小题考查的是作者的情感。根据题目可知诗人去拜访苏轼,注释点明苏轼长期贬谪外放的苏轼被授予翰林学士、知制诰等要职。再结合诗歌描写的景物特征,即可知情感为高兴、忙中偷闲。‎ ‎11.[2015·江苏高考]阅读下面这首唐诗,回答(1)~(3)题。‎ 秋日题窦员外崇德里新居 刘禹锡 长爱街西风景闲,到君居处暂开颜。‎ 清光门外一渠水,秋色墙头数点山。‎ 疏种碧松通月朗,多栽红药待春还。‎ 莫言堆案[注]无余地,认得诗人在此间。‎ ‎[注] 堆案:堆积案头,谓文书甚多。‎ ‎(1)联系全诗,概括作者“开颜”的原因。‎ 答:____________________________________________________‎ ‎____________________________________________________________‎ 答案 朋友新居落成;周围景色优美;自己心情闲适;主人品味高雅;宾主志同道合。‎ 解析 根据标题“秋日题窦员外崇德里新居”可概括出朋友新居落成;根据“长爱街西风景闲”可概括出自己心情闲适;根据颔联可概括出周围景色优美;根据颈联可概括出主人品位高雅;根据“认得诗人在此间”可概括出宾主志同道合(都喜欢诗歌,都喜欢闲适)。‎ ‎(2)简要赏析颔联、颈联的写景艺术。‎ 答:____________________________________________________‎ ‎____________________________________________________________‎ 答案 选取景物,铺陈描摹(一渠水、数点山、碧松、红药);移步换景,富有层次(由远及近、由外而内);虚实结合,寓情于景(通月朗、待春还)。‎ 解析 两联选取“一渠水”“数点山”“碧松”“红药”等景物进行铺陈描摹;颔联写院外的远景,颈联写院内的近景;这两联前三句是实景,第四句是虚景。‎ ‎(3)尾联表达了作者什么样的情感?‎ 答:____________________________________________________‎ ‎____________________________________________________________‎ 答案 赞美恭维之意,羡慕向往之情,抒志趣相同之感。‎ 解析 根据“诗人在此间”(原来诗人在此间),可以表现出诗人对窦员外的恭维之意;根据“在此间”,可以表现出诗人的向往之情;诗人和窦员外都喜欢诗歌,诗人和窦员外都喜欢闲适,可以表现出他们志趣相同。‎ ‎12.[2015·广东高考]阅读下面这首诗,然后回答(1)~(2)题。‎ 早过大通驿①‎ 查慎行 ‎[注] ①大通驿:在安徽铜陵,大通河由此入长江,作者乘船途经此地。②荆关:五代后梁画家荆浩、关仝,二人擅长山水画。‎ ‎(1)第三联的“软”字在艺术表现上很有特色,请作赏析。‎ 答:____________________________________________________‎ ‎____________________________________________________________‎ 答案 “软”字一语双关,既是写风软,也是写江水之软。写出了微风轻柔,温软拂面,也写出了软风吹拂江水,使江水也变得温柔绵软,微波轻漾。展现了一幅风吹波澜、风和水暖的清晨美景,生动形象,具有艺术感染力。‎ 解析 本题考查诗歌鉴赏中的“炼字”。首先要分析“软”在诗句中的意思,然后从手法上进行赏析。手法上抓住双关的手法是关键。软,使……柔软,江水本窈深而显冷涩,但在习习晨风吹拂之下竟显得柔软,软字很形象地写出晨风轻轻吹送、江水微微波褶的情状,从中亦可见出诗人的早过大通驿的轻快心意。‎ ‎(2)诗题的“过”字在诗句中是如何体现的?请结合全诗简要分析。‎ 答:____________________________________________________‎ ‎____________________________________________________________‎ 答案 过即指经过,路过,诗中通过前三联的时间、景物的变化,移步换景,展现出作者清晨乘舟江上,经过大通驿时的轻松畅快的心情。首联写晨雾未散,朝阳未升,眼前一片迷蒙。颔联写江雾散去,远处的市镇依稀可辨,转眼移过了水湾。颈联写江风拂面,云朵飘过九子山间,写出了行舟之快。全诗通过作者在行舟之上,遥看岸上景物变化,体现出“过”之意。‎ 解析 考查学生对诗歌整体内容的把握。分析时要逐联赏析,结合“过”的诗意进行。①先从时间很快的角度来表现“过”:早雾刚刚消散,朝阳还未升起,没来及等景象展开就离开这个地方了;②再从行舟速度很快的角度来表现“过”:站在船上远远地通过柳树来辨识城市的方位,一霎眼功夫,眼前已经是满山枫树,到了另一处水湾。‎ ‎13.[2015·福建高考]阅读下面的诗歌,完成(1)~(2)题。‎ 秋夜纪怀 ‎[宋]陆游 北斗垂莽苍,明河①浮太清②。‎ 风林一叶下,露草百虫鸣。‎ 病入新凉减,诗从半睡成。‎ 还思散关③路,炬火驿前迎④。‎ ‎(选自《四库全书》本《放翁诗选》)‎ ‎[注] ①明河:银河。②太清:指天空。③散关:即大散关。④炬火驿前迎:举着火把到驿馆前迎接。‎ ‎(1)第二联写景精细,请简要分析。‎ 答:____________________________________________________‎ ‎____________________________________________________________‎ 答案 要点:①上句写微风穿过树林,叶子悄然落下;②下句写露水沾湿秋草,百虫鸣叫;③全联细致地写出了秋夜的寂静,营造出一种凄清的氛围。(其他看法,言之成理亦可)‎ 解析 答题时注意题干中的信息“第二联”“写景精细”,组织答案时应重点分析怎样写景。颔联状秋,写风过暗林,一叶飘零,露浸衰草,百虫嘶鸣,写得深沉落寞。‎ ‎(2)三、四两联抒发了诗人怎样的情怀?请简要分析。‎ 答:____________________________________________________‎ ‎____________________________________________________________‎ 答案 抒发了诗人的爱国情怀:诗人虽秋夜病卧,仍壮心不已,念念不忘昔年在大散关的战斗生活,还想重返战场,报效国家。(意思对即可)‎ 解析 前四句写景,后四句抒怀,写卧病,写吟诗,然而病中所思和诗中所吟的仍是难以忘怀的大散关,那铁马冰河的边防,那驿炬来迎的抗金前沿。全诗从状景起,以纪怀收,纪怀借状景为铺垫,状景因纪怀见精神。‎ ‎14.[2015·四川高考]阅读下面这首唐诗,然后回答(1)~(2)题。‎ 夏日游山家同夏少府 骆宾王 返照下层岑,物外狎招寻。‎ 兰径薰幽珮,槐庭落暗金。‎ 谷静风声彻,山空月色深。‎ 一遣樊笼累,唯馀松桂心。‎ ‎(1)请简要赏析“谷静风声彻,山空月色深”。‎ 答:____________________________________________________‎ ‎____________________________________________________________‎ 答案 “谷静风声彻,山空月色深”这两句用山谷的幽静、空旷衬托风声格外之响与月色分外之浓。视听兼具,动静结合,突出表现了山间空旷、静寂的美好景致。‎ 解析 题干要求“简要赏析 谷静风声彻,山空月色深 ”,这是考查学生鉴赏品味诗歌语言的能力。赏析的角度包括:修辞角度、描写角度、遣词造句角度、选材角度等等。简而言之,从内容和手法等角度入手分析。“谷静风声彻,山空月色深”,这两句主要是写景,故可以从写景的角度入手,“风声”立足于听觉,“月色”立足于视觉,可以从感官角度作答;同时用“谷静”“山空”“月色深”等景象描绘了一幅静谧的图景。‎ ‎(2)本诗表达了作者怎样的情感和志向?结合全诗简要分析。‎ 答:____________________________________________________‎ ‎____________________________________________________________‎ 答案 本诗表达了作者热爱自然之情,追求高洁坚贞、自由闲适之志。首联从夕照美景吸引作者游兴写起,点明访问山家之由;颔联写兰径和山家槐庭美景,幽兰散发阵阵香味,落日透过槐洒下斑驳光影,暗示山家主人为高洁隐士;颈联写山谷幽静空旷、风声之响、月色之浓,呈现出山间特有景致;尾联直接表达了诗人对自由闲适生活、坚贞高洁品格的追求。‎ 解析 题干问“本诗表达了作者怎样的情感和志向”,这是考查诗歌的情感。首先应到诗歌中找情感词,然后再找诗人笔下的景物,体会景物的特点。全诗紧扣“游”字,描写夏日暮晚时分的独特山景,表达了热爱自然之情,抒发了追求闲适、高洁之志。首联直接点题,侧面明旨“物外”。颔联承接上文,聚焦题中“山家”,突出“兰径薰”“槐庭落”的夏日景况,也暗示了诗人的志趣与追求。颈联转写山风、山月,一“彻”一“深”,照应其“静”“空”,空旷、静谧的意境自然形成。尾联应题,以议结情,“一遣”与“唯馀”‎ 的反差,揭示了一片“诗心”,主旨凸显。由此,全诗在营造山中夜色空旷、幽静的奇特意境中,含蓄地表达了诗人远离“樊笼”(尘世束缚)、亲近“松桂”(美好自然)的心愿,其厌恶尘世、热爱自然、追求高洁之情志,了了可见。‎ ‎15.[2015·山东高考]阅读下面的宋词,回答(1)~(2)题。‎ 卜算子 张元幹[注]‎ 风露湿行云,沙水迷归艇。卧看明河月满空,斗挂苍山顶。 万古只青天,多事悲人境。起舞闻鸡酒未醒,潮落秋江冷。‎ ‎[注] 张元幹,宋代爱国词人。‎ ‎(1)请对上片前两句中的“湿”“迷”二字分别作简要赏析。‎ 答:____________________________________________________‎ ‎____________________________________________________________‎ 答案 ①“湿”字,描写地面浓重的风露水气使行云也充满湿气而显得厚重凝滞,突出了外在环境的潮湿、阴冷,表现了作者凄凉和沉重的心情。②“迷”字,描写水面迷蒙的雾气使归舟迷失了航向,烘托出朦胧、迷茫的意境氛围,表现了作者内心的迷惘。‎ 解析 就此题看,“湿”是“打湿,沾湿”之意,既写出了夜深露重,突出了环境的潮湿、阴冷,又写出主人公待得时间长,从而表现出作者心事之重和内心的凄凉。“迷”是“迷失”之意,一方面写出秋江之上雾气迷蒙,月光迷离,烘托出朦胧、迷茫的氛围;另一方面表现了作者心中的迷惘。‎ ‎(2)“起舞闻鸡酒未醒,潮落秋江冷”表达了作者怎样的思想感情?请作简要分析。‎ 答:____________________________________________________‎ ‎____________________________________________________________‎ 答案 ①化用祖逖的典故,表现作者胸怀大志,而报国之志难以实现,内心悲愤无奈;②寓情于景,描写江潮的退落和秋江的冷寂,委婉地表达了作者面对国势衰退内心的悲凉,寄寓了浓郁的爱国情感。‎ 解析 “起舞闻鸡酒未醒,潮落秋江冷”的意思是:半醒半醉之间,闻鸡鸣而起舞,舞罢,沙河潮落,深夜的秋江一片冷寂。“起舞闻鸡酒未醒”借用祖逖的典故,表达报国之志,同时“酒未醒”也显示出作者志向难以实现的无奈。“潮落秋江冷”以景结情、含蓄委婉,通过描写江潮的退落和秋江的冷寂,表达出作者面对国势衰颓、壮志难酬的悲凉。‎ ‎[2年全国模拟重组]‎ ‎1.[2017·辽宁铁岭市联考]阅读下面一首诗歌,完成(1)~(2)题。‎ 金陵城西楼月下吟 李白 ‎[注] 谢玄晖:名谢朓,南齐著名诗人。‎ ‎(1)下列对这首诗的理解和分析,不正确的两项是(  )‎ A.“金陵夜寂凉风发,独上高楼望吴越”两句中“金陵”点明了地点,“凉风发”暗示了季节,“夜寂”概括了时间和环境,“望吴越”交代了登楼的目的。‎ B.“古来相接眼中稀”中的“相接”是从古至今,由人及己的意思;“眼中”在结构上有过渡作用,暗示后面将要写的内容。‎ C.“解道 澄江净如练 ,令人长忆谢玄晖”两句话中有“话”,意思是谢朓的诗我能理解,但今日我写此诗,又有谁能读懂呢?‎ D.作者笔触所及,广阔且悠远,看似信笔挥洒,未加经营,而读者细细品味,则会发现本诗以“愁情”为线索贯通前后,脉络清晰。‎ E.这首诗,诗人时而写自己行迹或直抒胸臆,时而描绘客观景物或赞美古人,给人一种明快、浪漫、清新的感觉。这就格外突出了诗中的抒情主线,使全诗浑然一体,愈见精美。‎ 答案 BE 解析 B项,“相接”精神相通,心心相印的意思。E项,诗歌体现出的是苍茫、悲凉、沉郁的基调,突出诗人的寂寞和忧愁。‎ ‎(2)古人评诗常用“诗眼”的说法,所谓“诗眼”往往是指一句诗中最精练传神的一个字,你认为这首诗第二联两句诗的“诗眼”分别是哪一个字?为什么?请简要赏析。‎ 答:____________________________________________________‎ ‎____________________________________________________________‎ 答案 诗眼分别是“摇”“滴”。凉风吹来,江水摇动,影子跟着摇动,让人感到城也摇动起来;秋月皎洁,露珠坠落,仿佛是从月亮上滴落下来似的。此二字化静为动,赋予画面动感,使静止的画面生动起来,情态逼真;又使本属平常的水、城、露、月等景物,呈现出不同寻常的情态,异趣横生。‎ 解析 “摇”“滴”两个动词用得尤其神奇。城是不会“摇”的,但“凉风发”,水摇,影摇,给你的幻觉,城也摇荡起来,月亮是不会“滴”露珠的,但“独上高楼”,凝神仰望秋月皎洁如洗,好像露珠是从月亮上滴下似的。“滴”与“摇”,使整个静止的画面飞动起来,使本属平常的云、水、城、露、月诸多景物,一齐情态逼露,异趣横生,令人浮想联翩,为之神往。这样的描写,不仅反映出浪漫主义诗人想象的奇特,也充分显示出他对大自然敏锐的感觉和细致的观察力,故能捕捉住客观景物的主要特征,“着一字而境界全出”。‎ ‎2.[2017·衡水重点中学联考]阅读下面宋词,完成(1)~(2)题。‎ 行香子 秦观 树绕村庄,水满陂塘。倚东风、豪兴徜徉。小园几许、收尽春光。有桃花红,李花白,菜花黄。‎ 远远围墙,隐隐茅堂。飏青旗、流水桥旁。偶然乘兴,步过东冈。正莺儿啼,燕儿舞,蝶儿忙。‎ ‎(1)下列关于这首宋词的赏析,不正确的两项是(  )‎ A.本词上阕定点取景,以村庄为中心描写四周景物,写出了词人所见的烂漫春光。‎ B.本词下阕与上阕相似,定点观察,写出了围墙、茅堂、桥、流水等意象,表达了作者喜爱之情。‎ C.本词着力表现了清新自然、绚丽多彩、充满生机的农家田园景象。‎ D.正如词人“倚东风、豪兴徜徉”,本词主要表现了词人意兴豪迈、旷达洒脱的情怀。‎ E.这首词一反秦观其他词中常有的那种哀怨情调,色彩鲜明,形象生动,给人以轻松愉快的美的享受。‎ 答案 BD 解析 本题主要考查诗歌内容与情感的分析与鉴赏能力,也涉及诗歌艺术手法的考查。主要是要准确解读诗歌内容。B项,下阕移步换景,以行踪为线索由近到远,动静相间,风光如画。D项,结合整首词的内容,“豪兴”是说明词人游兴正浓,“徜徉”是闲庭信步,与下阕“偶然乘兴”相呼应,主要表现词人享受田园风光时内心的快意与轻畅。“意兴豪迈”“旷达洒脱”概括不准确。‎ ‎(2)请赏析这首词的语言特色。‎ 答:____________________________________________________‎ ‎____________________________________________________________‎ 答案 ①通俗生动,朴素清新,质朴自然。“树绕村庄,水满陂塘”如同口语,生动自然。写景如“桃花红,李花白,菜花香”,色彩简单鲜明又显出农家本色。②用词准确。“倚东风、豪兴徜徉”,“豪兴”点明心情,“倚”“徜徉”则写出诗人怡然自得的神态。“莺儿啼,燕儿舞,蝶儿忙”,“啼”“舞”“忙”与三个动物的活动相对应,写出了春天的生命活力。③句式整齐,格调轻快。本词句式都是三、四字句式,轻快流畅。(要点+实例)‎ 解析 本题考查诗歌语言的分析与鉴赏能力。要结合诗词大意感知诗歌的语言特点。先用一些概括性的词语点出,再结合诗句作具体分析。诗歌的语言特点一般有:清新自然、朴实无华、华美绚丽、明白晓畅、委婉含蓄、雄浑豪放、放笔婉约、简练生动、抑扬顿挫等。‎ ‎3.[2017·成都五校联考]阅读下面这首诗,完成(1)~(2)题。‎ 云居寺①高顶 王褒②‎ ‎[注] ①云居寺:在北周都城长安附近的山上,一说此山是终南山。②王褒(513~576):先后在南朝的梁和北朝的西魏、北周为官,才名极高。‎ ‎(1)下列对本诗的赏析,不正确的两项是(  )‎ A.这首诗虽然只有二十个字,但是大阖大开,气魄雄伟,景象堪称壮丽。‎ B.前两句“云已合”与“日犹晴”几字,突出了中峰到山顶的天气的变化。‎ C.这是一首五言诗,前两句运用对比的手法,因而给人工整匀称的美感。‎ D.最后两句写登临绝顶后看到的美妙景象,顿觉眼界开阔,心胸畅达。‎ E.本诗良好的艺术效果是通过绘声绘色、细致入微的静态描摹获得的。‎ 答案 CE 解析 C项,此句既用了对比,突出了中峰和山顶的天气不同,又用了对偶,而对偶的作用是“给人工整匀称的美感”。E项,不是“绘声绘色、细致入微的静态描摹”,而是写出了登山过程中的实际感受。‎ ‎(2)前两句诗所写的景象和表达的心情有何变化?请结合诗句具体分析。‎ 答:____________________________________________________‎ ‎____________________________________________________________‎ 答案 前两句所写的景象由攀登到山腰时的云雾四合写到登上顶峰时的云开天晴,心情由塞到开。第一句“中峰云已合”说是攀登到半山腰,已是云雾弥漫,不辨东西,写出了山之高峻,也写出了攀登之艰难。但下一句“绝顶日犹晴”,写历尽艰难,登上山顶,竟是丽日悬天,晴空如洗,光华四布,此时心情也豁然开朗,艰辛的攀登后到达天地间的壮美之境,自有“柳暗花明又一村”的欢愉感。‎ 解析 读诗题“云居寺高顶”,有作者主观感受的词就一个“高”字;读诗,云雾缭绕是高,可这只是中峰,“绝顶日犹晴”也是因为高,才知山顶与山腰气象迥然不同。“邑居随望近,风烟对眼生”,登高望远,眼界开阔,心胸畅达,一路的艰辛化为登山后的喜悦,想一路白云相伴原是自己已登仙境,自是妙不可言。虽是要求解答前两句诗,但真要解透,不可忽视后两句,要整体把握,方可明白真意。‎ ‎4.[2017·河南洛阳市调研]阅读下面这首宋词,完成(1)~(2)题。‎ 倦行芳慢 ‎[宋]王雱 露晞向晚,帘幕风轻,小院闲昼。翠径莺来,惊下乱红铺绣。倚危墙,登高榭,海棠经雨胭脂透。算韶华,因循过了,清明时候。‎ 倦游燕,风光满目,好景良辰,谁共携手?恨被榆钱①,买断两眉长斗。忆高阳②,人散后,落花流水人依旧。这情怀,对东风,尽成消瘦。‎ ‎[注] ①榆钱:榆树早春未生叶时先开花,果实不久成熟,名榆荚,形状似钱而小,色白成串,俗呼榆钱。②高阳:《史记·郦生列传》:“郦生食其者,陈留高阳人也。……县中皆谓之狂生。”他见刘邦时,自称 高阳酒徒 。 高阳 之人即指游燕时的狂朋怪侣。‎ ‎(1)下列对这首宋词上阕内容的鉴赏,不准确的两项是(  )‎ A.傍晚时分,初夏的凉风拂过,吹动了重重帘幕,主人公在这寂寂的环境中独处。‎ B.“翠径莺来,惊下乱红铺绣”写黄莺飞来,惊动枝上的花瓣纷纷下落,表现了词人内心烦躁凌乱。‎ C.“海棠经雨胭脂透”中“胭脂透”写经雨的海棠已经开放到最鲜艳最鼎盛的时刻,也暗示着盛极而衰、即将凋落的暮春信息。‎ D.上阕最后三句略略点明题意,这清明时节美丽的春光又轻易流逝,有一种“无计留春住”的叹惋。‎ E.这首词由景及情,上片景中有情,下片以情带景,笔锋细腻,用语婉媚,韵致翩翩,堪称诗人王雱的孤篇力作。‎ 答案 AB 解析 A项,上阕起三句描述了一个具体环境,时间是暮春白昼,地点是闲静小院,氛围沉寂清幽。B项“翠径莺来,惊下乱红铺绣”写雨停雨霁,黄莺飞来,枝上的花瓣仿佛被惊动而缤纷下落,落红点缀绿径,色彩斑斓,犹如织铺地,故曰“铺绣”‎ ‎。这里体现词人观察细密,用笔工致,使暮春景物变活,极具匠心,表现了暮春的凋零残败而又清新红艳。虽清新红艳但终将凋零,与下文情感一致。因此说体现了词人“内心烦躁凌乱”不准确。‎ ‎(2)结合下阕内容,具体赏析诗人有哪些伤春意绪?‎ 答:____________________________________________________‎ ‎____________________________________________________________‎ 答案 ①春来懒事游宴。虽然“好景良辰,风光满目”,只因无人携手同乐,对游宴之事就意懒情倦了。②一春之不欢,春愁之长之深。自有榆钱以来,所“买”得者是“两眉长斗”,因愁苦而双眉紧锁。③因循岁月,蹉跎时光。酒侣星散,又值“落花流水”的春暮,这一年又是这样。④结尾三句写自己因春消瘦,总收全文。‎ 解析 本题考查分析诗歌内容和作者情感的能力。关键是要吃透诗词大意,在理解诗词大意的基础上可依次提炼答案。下阕以“倦游燕”起,说春来懒事游宴。虽然时是“好景良辰”,景是“风光满目”,只因无人携手同乐,对游宴之事就意懒情倦了。“谁共”二字反诘,以下再用“恨”字承接,进一步形容春愁之深。“恨被榆钱,买断两眉长斗”,本意只是说一春常愁中。因“钱”之称而得“买”字意,又榆钱早春即见,几与春光同起讫。“买断”即买尽,自有榆钱以来,所“买”得者是“两眉长斗”,则其一春之不欢,至此已曲折写出。以下“忆高阳,人散后”,似转仍承,承上“游燕谁共携手”意。“这情怀,对东风、尽成消瘦”,以说一春之愁,比“买断两眉长斗”又进一步,总收全文。‎ ‎5.[2017·云南曲靖中学月考]阅读下面的诗歌,完成(1)~(2)题。‎ 眼中 元好问[注]‎ ‎[注] 元好问:金末元初著名诗人,金亡后拒绝仕元。‎ ‎(1)下列对本诗的理解,最恰当的两项是(  )‎ A.作者“拥被不眠”主要是因为夜晚天气寒凉。‎ B.“衣冠今日是何年”写出了时间的流逝。此句意蕴丰富,与《桃花源记》中“问今是何世,乃不知有汉,无论魏晋”有相似的意蕴。‎ C.“枯槐聚蚁”使用了典故,指的是南柯梦中的槐安国蚁民。在此用以比喻弱国微名。‎ D.“华颠”即华山之巅。此句流露出诗人有远隔尘世,要到那青山深处、华山之巅的打算。‎ E.元好问“诗似杜甫”,其诗歌风格沉郁苍凉。全是批判意识与悲怆情怀相融合,使诗作增加了历史深度。‎ 答案 CE 解析 A项,作者“拥被不眠”,主要是因为“时势纷然”,忧国忧民。B项,“衣冠今日是何年”是言国家破亡,改朝换代,与《桃花源记》中的句子表现人们与世隔绝,不知时间流逝的意思完全不同。D项,“华颠”指白头、年老。‎ ‎(2)全诗表达了诗人哪些思想感情?请简要分析。‎ 答:____________________________________________________‎ ‎____________________________________________________________‎ 答案 ①“眼中时事亦纷然,拥被寒窗夜不眠”写时事纷然,诗人寒夜不眠,进而想到骨肉分离,追问“衣冠今日是何年”表达了亡国之痛。②“枯槐聚蚁无多地”携百姓流离失所,无立身之地,表达了对百姓的同情和悲悯。③“秋水鸣蛙自一天”“何处青山隔尘土,一庵吾欲送华颠”写诗人以“秋水鸣蛙”相比,只愿自守一片天地,想要寻找与世隔绝的青山,度过余生,表达了归隐之情。‎ 解析 题干考查诗歌的情感,答题关键词是“结合全诗”,考生要从意象、意境、手法角度去分析。从首联的“事纷然”“夜不眠”想到骨肉分离,又有追问“今日是何年”的亡国之痛。还有“无多地”及“枯槐聚蚁”的典故,写出百姓的苦难和作者深度同情。最后几句有对自然的向往及归隐之心。‎ ‎6.[2017·豫北名校联盟精英对抗赛]阅读下面这首唐诗,完成(1)~(2)题。‎ 野望 ‎[唐]杜甫 ‎[注] ①三城戍:西山三城的堡垒,三城,与吐蕃临界,为蜀边要塞。②南浦,泛指送别之地。万里桥,在成都杜甫草堂的东边。③涓埃:细流与微尘,比喻微小。‎ ‎(1)下列对本诗的理解,不正确的两项是(  )‎ A.首联由景入题,远望“西山白雪三城戍”,近看“南浦清江万里桥”,景象开阔辽远。‎ B.颈联写自己“迟暮多病”“未报圣朝”,含蓄地批判了皇帝昏庸无能,荒淫误国,抒发了自己报国无门的悲叹之情。‎ C.尾联照应题目——“野望”,抒发了作者深为民不堪命,世事日转萧条和自己年老多病,功业未就的隐忧。‎ D.前两联写景,后两联抒情,景是情的基础,情是景的生发,情景交融,浑然一体。‎ E.诗人虽身居草堂,然心忧家国,诗歌内容深广,意境雄浑,感情深沉,表情达意抑扬跌宕。‎ 答案 BD 解析 B项,无批判和报国无门之意,应该是“功业未就”;D项,颔联是叙事和抒情。‎ ‎(2)颔联蕴含了诗人哪些思想情感?请结合具体诗句加以赏析。‎ 答:____________________________________________________‎ ‎____________________________________________________________‎ 答案 ①忧国之情:“海内风尘”指安史之乱的连年战火,表现了诗人对国家局势的担忧。②对诸弟的思念:战乱频仍,诸弟分离,长期阻隔,诗人对亲人的无限思念之苦跃然纸上。③漂泊之悲:诗人流落天涯,一身飘零,“一身遥”蕴含无限人世之悲,流离之苦。‎ 解析 颔联两句由战乱推出怀念诸弟,自伤流落的情思。“风尘”指安史之乱导致的连年战火。杜甫四弟:颖、观、丰、占。只杜占随他入蜀,其他三弟都散居各地。此时“一身遥”客西蜀,如在天之一涯。诗人怀念家国,不禁“涕泪”横流。真情实感尽皆吐露,不由人不感动。‎ ‎7.[2017·华中师大一附中模拟]阅读下面这首宋词,完成(1)~(2)题。‎ 水调歌头·游览①‎ 黄庭坚 瑶草②一何碧,春入武陵溪③。溪上桃花无数,枝上有黄鹂。我欲穿花寻路,直入白云深处,浩气展虹霓。祗恐花深里,红露湿人衣。‎ 坐玉石,倚玉枕,拂金徽④。谪仙何处,无人伴我螺杯。我为灵芝仙草,不为朱唇丹脸,长啸亦何为?醉舞下山去,明月逐人归。‎ ‎[注] ①此词大约写于被贬官时期。②瑶草:仙草。③武陵溪:出自陶渊明《桃花源记》。④金徽:瑶琴。‎ ‎(1)下列对这首宋词上阕内容的赏析,不正确的两项是(  )‎ A.“瑶草一何碧”运用比兴手法,赞美瑶草(仙草)像碧玉一样可爱,激起读者兴趣,把读者引进仙境中去。‎ B.“溪上桃花无数,枝上有黄鹂”承接“春入武陵溪”,表现了“桃花源”仙境的清丽澄净,幽静脱俗。‎ C.“我欲穿花寻路”三句表达了作者对此境的喜爱,满怀喜悦规划着自己的行程,渴望有更美的发现。‎ D.“祗恐花深里,红露湿人衣”故意正话反说,说仙境里花深露重,怕会打湿衣服,实际表现了词人对仙境的深深喜爱和依恋。‎ E.此词为春行纪游之作,词人采用幻想的镜头,描写神游“桃花源”的情景,反映了他出世、入世相冲撞的人生观。‎ 答案 CD 解析 C项,“我欲穿花寻路”三句虚写想象、以虚衬实,写词人幻想能找到一个自由施展才能的理想世界,含蓄地表示对现实的不满。D项“祗恐花深里,红露湿人衣”承接前三句,写词人担心仙境里花深露湿,沾湿人衣,既写现实的游览,又曲折地表现他对纷乱人世的厌倦但又不甘心离去的矛盾。所以D错。‎ ‎(2)结合下阕内容,简要分析词人表达了怎样的思想情感?‎ 答:____________________________________________________‎ ‎____________________________________________________________‎ 答案 ①词人想象“坐玉石、倚玉枕、拂金徽(弹瑶琴)”表现他的志行高洁、与众不同。②接着说李白不在,无人陪他饮酒,是说他缺乏知音,感到异常寂寞,同时曲折地表达出他对现实的不满。③词人说自己到仙境探索是为了得到灵芝仙草,不是为了得到像“朱唇丹脸”一样的桃花,即使得不到又何必长啸呢?以象征性的语言说明自己不必去为得不到功名利禄而忧愁叹息。④最后两句描写词人醉后摇摇晃晃,下山翩翩起舞,明月伴他归去的形象,表现他想逃避现实却又不甘心如此的矛盾心理及孤寂心境。‎ 解析 本题考查分析、鉴赏诗歌内容与作者思想情感的能力。关键是要吃透诗词大意。注意词体的一个特点是讲究铺叙,基本上一个句号对应一个内容层次。学生在理解诗词大意的基础上可依次提炼答案。“仙草”即开头的“瑶草”,“朱唇丹脸”指第三句“溪上桃花”。苏轼咏黄州定惠院海棠诗云:“朱唇得酒晕生脸,翠袖卷纱红映肉。”花容美艳,大抵略同,故这里也可用以说桃花。这两句是比喻和象征的语言,用意如李白《拟古十二首》之四所谓“耻掇世上艳,所贵心之珍”。“长啸亦何为”意谓不必去为得不到的功名利禄而忧愁叹息。‎ ‎8.[2017·资阳市诊断性考试]阅读下面这首诗,完成(1)~(2)题。‎ 公安县怀古①‎ 杜甫 ‎[注] ①本诗作于大历三年(公元768年)深秋。是年夏,吐蕃进犯灵武、邠州,京城震动。杜甫因好友严武去世,便离开蜀地到荆楚一带的公安县暂住。②此二句主要写了三国时刘备君臣的关系及吕蒙的战绩。‎ ‎(1)下列对本诗的理解,正确的两项是(  )‎ A.这是一首五言古体诗。全诗八句四联,对仗工整,颇显诗人的诗律功底。‎ B.首联构思精巧,用笔着力表现出“山野”之“旷”,“江水”之“深”。‎ C.颔联中,“催”字将深秋季节里昼短夜长的自然情状形象生动地展现出来。‎ D.颈联“用典”,主要表现出了刘备君臣的和睦关系以及吕蒙的赫赫战功。‎ E.整首诗层次清晰明了,首叙古迹形胜,次写想象之景,后直抒怀古情思。‎ 答案 CD 解析 A项,“五言古体诗”判断有误,本诗应为“五言律诗”,属近体诗。B项,作为怀古诗,“野旷吕蒙营,江深刘备城”重点表现的不是自然景观,而是此地的历史,即“吕蒙营”“刘备城”。E项,“次写想象之景”判断有误,应为“眼前之景”。‎ ‎(2)本诗与杜甫的《咏怀古迹》(其三)在思想情感上有何异同?请简要分析。‎ 答:____________________________________________________‎ ‎____________________________________________________________‎ 答案 ①相同点:诗人在两首诗中均流露出了对古人(王昭君、吕蒙、刘备)的缅怀、敬仰之情以及自己怀才不遇、壮志未酬的无奈伤悲。②不同点主要表现在诗人对古人缅怀敬仰的具体情感。《咏怀古迹》(其三)中,诗人主要表达对王昭君凛然勇毅的家国大义的敬佩;《公安县怀古》中,诗人主要表达对像“刘、关、张、孔”等君臣和睦关系的向往。(意思对即可)‎ 解析 本题考查古代诗歌对比鉴赏以及评价诗歌的思想内容和作者的观点态度的能力。比较杜甫在两首怀古诗中的异同,需要对两首诗有比较准确的把握。从体裁上而言,两首诗都是怀古诗,都是因为诗人来到了名人故地而自然生发的感慨。《咏怀古迹》(其三)是诗人对王昭君的缅怀,想到昭君生于名邦,殁于塞外,去国之怨,难以言表。因此,主题落在“怨恨”‎ 二字,作者既同情昭君,也感慨自身的怀才不遇。本诗是作者对曾经在公安县建立功业的刘备、吕蒙等人的缅怀,表达对他们能建功立业的人生的羡慕和刘备君臣的和睦关系的向往。‎ ‎9.[2017·牡丹江一中期考]阅读下面一首唐诗,完成(1)~(2)题。‎ 金陵怀古 许浑 ‎[注] ①玉树:陈后主所制乐曲《玉树后庭花》。②景阳兵合:隋军攻陷金陵,直逼景阳宫外,陈后主束手就擒,陈朝灭亡。③禾黍:出自《诗经》,周大夫行役过故宗庙宫室之地,看见到处长着禾黍,感伤王都颠覆,因而作了《黍离》一诗。④洛中,即洛阳,洛阳多山。‎ ‎(1)下列对本诗的理解,不正确的两项是(  )‎ A.金陵是孙吴、东晋和南朝的宋、齐、梁、陈六朝古都,隋唐以来,由于政治中心的转移,无复六朝的金粉繁华。金陵的盛衰沧桑,成为许多后代诗人寄慨言志的话题。‎ B.首联写南朝最后一个小朝廷,在陈后主所制乐曲《玉树后庭花》的靡靡之音中覆灭。“残”和“空”,从文化生活和军事设施两方面反映陈朝的腐败,一文一武,点染出金陵城一片没落不堪的景象。‎ C.颔联描写金陵的繁荣盛况。“松楸”,坟墓上的树木。诗人登高而望,看到荒冢上远近高低尽是挺拔的松楸,六朝的宫殿高高低低。‎ D.颈联概括世间的风云变幻。这里,“拂”字、“吹”字写得传神,“亦”字、“还”字写得含蓄。石燕和江豚,是传说里面神奇怪诞的动物,唐汝询说:“千官有冢,六代无宫,惟馀石燕江豚,作雨吹风而已。”在这里烘托一种神秘莫测的浪漫主义气氛。‎ E.尾联照应开头,抒发了诗人的感慨。“惟有青山似洛中”,就是说今日的金陵只有青山和洛阳相似,其余的一切都和六朝相同。江山不改,世事多变,令人感慨万千。‎ 答案 CE 解析 C项,颔联描写金陵的衰败景象,“松楸”,坟墓上的树木。诗人登高而望,远近高低尽是松楸荒冢,残宫禾黍,南朝的繁荣盛况,已成为历史的陈迹。E项,“惟有青山似洛中”,就是说今日的金陵除去山川地势与六朝时依然相似,其余的一切都大不一样了。江山不改,世事多变,令人感慨万千。‎ ‎(2)请对第三联中“拂”“吹”两个字做简要赏析。‎ 答:____________________________________________________‎ ‎____________________________________________________________‎ 答案 “拂”字是“掠过”之意,写石燕掠过云朵带来晴雨变化,“吹”为“鼓动”之意,写江豚在江面上兴风鼓浪,把江上自然的风云晴雨的变化写作由石燕、江豚带来,用石燕、江豚象征历史上叱咤风云的英雄人物。‎ 解析 “拂”字、“吹”字写得传神。“拂云”描写石燕掠雨穿云的形象,“吹浪”表现江豚兴风鼓浪的气势。“江豚”和“石燕”象征历史上叱咤风云的人物,如尾联所说的英雄。这两句通过江上风云晴雨的变化,表现人类社会的干戈起伏和历代王朝的兴亡交替。‎ ‎10.[2017·河南新乡市模拟]阅读下面这首清诗,完成(1)~(2)题。‎ 钱塘观潮 施闰章 ‎[注] 鸱夷:皮袋,这里借指潮神伍子胥。据《吴越春秋》等记载,春秋吴国大夫伍子胥因劝谏吴王夫差而被疏远、赐死。伍子胥临死时,嘱咐家人把他的眼睛挖出来或者把头割下来悬挂在南城门上,以便看到吴国的灭亡。吴王大怒,下令用鸱夷把他的尸体包裹起来,投入钱塘江。后来伍子胥化为钱塘江潮潮神。‎ ‎(1)下列对本诗的理解与赏析,不正确的两项是(  )‎ A.首联“海色雨中开,涛飞江上台”表明作者是在雨中观潮,潮水来时,在迷蒙的雨点和潮水冲击下,海色随潮水展开。潮来时浪涛飞向钱塘江上的观潮台,极写潮水声势之大。‎ B.颔联“声驱千骑疾,气卷万山来”同时运用了比拟、夸张、视听结合等修辞手法,写出了钱塘江潮仿佛驱赶着千军万马,又仿佛将千万座山席卷而来,气势惊心动魄。‎ C.“绝岸愁倾覆”中运用一个“愁”字,通过写岸边峭壁被潮水拍打时生怕被冲垮,写出了诗人对潮水冲垮堤岸的担忧之情。‎ D.“轻舟故溯洄”一个“故”字,写江中弄潮儿故意在潮水上涨时随着潮头起伏腾跃,在水中回旋,写出了弄潮儿的高超本领。“故”与“愁”两字的对举富有生趣。‎ E.前三联写观潮所见之景象,诗人运用多种表现手法,极力描写潮声之大,水势之猛,写实中渗透想象,描摹形象生动,体现了作者非凡的语言驾驭能力。‎ 答案 BC 解析 B项,“视听结合”不是修辞手法。C项,这里的“愁”字是拟人的手法,通过写岸边峭壁被潮水拍打时生怕被冲垮,写出了潮水的气势。‎ ‎(2)诗歌的尾联对表现诗歌的主题有何作用?请简要分析。‎ 答:____________________________________________________‎ ‎____________________________________________________________‎ 答案 ①最后一联,把钱塘江的滔天巨浪及其排山倒海的不凡气势,想象成是伍子胥充满遗恨的冤魂兴起的。②借以江潮的怒态象征伍子胥的“遗恨”之大之强。③表达了作者对伍子胥充满深深的同情、惋惜与慨叹。④深化了诗歌的思想主题。‎ 解析 史载,春秋时楚国人伍子胥,其父伍奢、兄伍尚都被楚平王杀害,他逃奔吴国,先佐吴王阖闾伐楚国,后又佐吴王夫差(阖闾之子)打败越国。最终,夫差听信伯嚭的谗言逼迫伍子胥自杀,并下令将其尸首装入“鸱夷革”抛进江中。后来传说伍子胥怨恨吴王夫差,死后驱水为涛,因而,钱塘江潮又有“子胥潮”之称。施诗尾联发诗人观潮之后的感慨,由观潮联想到伍子胥及其传说,为伍子胥的冤死而鸣不平,笔墨转换自然,耐人寻味。‎ ‎11.[2017·重庆永川中学模拟]阅读下面的宋词,完成(1)~(2)题。‎ 西江月·阻风三峰下 张孝祥①‎ 满载一船秋色,平铺十里湖光。波神②留我看斜阳,放起鳞鳞细浪。‎ 明日风回更好,今宵露宿何妨?水晶宫里奏霓裳,准拟③岳阳楼上。‎ ‎[注] ①张孝祥:南宋初词人。这首词是因船行至洞庭湖畔的黄陵庙下为风浪所阻而作。作者在写给友人的信中提道:“某离长沙且十日,尚在黄陵庙下,波臣风伯亦善戏矣。”②波神:水神。③准拟:准定。‎ ‎(1)下列对词中语句的理解,不正确的两项是(  )‎ A.一、二两句点明了行船的季节,描写了洞庭湖上风浪未起时的景色。‎ B.三、四两句有主观想象色彩,“鳞鳞细浪”实际上是指滚滚波涛。‎ C.五、六两句写明日风向一转便可顺风行船,今晚露宿又有什么关系呢?‎ D.七、八两句写作者期盼到岳阳楼上听水下传来的《霓裳》曲。‎ E.全词以景入情,情景交融,静中有动,动静结合,想象丰富,具有浓厚的浪漫色彩,是婉约派的代表作。‎ 答案 DE 解析 D项所述内容对文本理解不正确。本诗七、八两句收结,别具情味。“霓裳”,即《霓裳羽衣曲》,是唐代比较流行的一首舞曲。“岳阳楼”,在湖南省岳阳市城西,面临洞庭湖。第七句写江中波涛发出的一阵阵声响,就像水府在演奏美妙悦耳的音乐。这种生动的比喻表现出词人独到的想象。第八句则表达了词人内心的愿望:当行舟到达岳阳时,一定要登上岳阳楼眺望洞庭湖雄伟壮阔的自然风光。E项,“婉约派的代表作”错误,这首词想象丰富,笔力雄放,以及体现出的词人达观豪迈的胸怀等,都可判断属于豪放派。‎ ‎(2)在这首词中,作者是以怎样的胸怀对待风波险阻的?举出两处具体进行赏析。‎ 答:____________________________________________________‎ ‎____________________________________________________________‎ 答案 ①达观、豪迈的胸怀。②如“波神留我看斜阳,放起鳞鳞细浪”,明明是行船受阻,却戏谑为波神留我欣赏斜阳的景观;如“明日风回更好,今宵露宿何妨”的不以受阻为意;如“水晶宫里奏霓裳,准拟岳阳楼上”,既已受阻,不如就此欣赏眼前的风光,听水波演奏的美妙乐曲等。‎ 解析 解答此题时,要有整体阅读的意识,在读懂全词的基础上针对问题作答。要仔细体味词作的语言,如“明日风回更好”,写他期待风向回转,天气变好,及时登程的心情。“今宵露宿何妨?”“何妨”,犹言“有什么关系呢”,实际上是无可奈何的话,但也表现了他在迫不得已的情况下“露宿”时的旷达胸襟。尤其结尾两句前一句写一阵阵江中波涛的声响,就像水府在演奏美妙悦耳的音乐,想象生动新颖,显示出作者旷达的胸怀、乐观的态度和高雅的生活情趣。‎ ‎12.[2017·河北承德一中检测]阅读下面这首唐诗,完成(1)~(2)题。‎ 古风(其十)‎ 李白 ‎[注] ①鲁连:鲁仲连,战国齐人,他说服魏与赵合力抗秦。②平原:即平原君,赵国重臣。‎ ‎(1)下列对本诗的理解,不正确的两项是(  )‎ A.三、四句与曹植诗句“大国多良材,譬海出明珠”都运用比喻的方式表达赞誉之情。‎ B.“后世仰末照”句,感叹鲁仲连的功绩如同就要落山的太阳一样将被后人渐渐遗忘。‎ C.本诗最后两句,以“澹荡人”与开头的“倜傥生”相呼应,意在表明诗人的志趣。‎ D.李白在诗中盛赞了鲁仲连的高风亮节,并把他引为“同调”,内容显豁,感情深挚。‎ E.本诗雄浑壮丽的语言,缠绵哀怨的气韵,足以诠释李白高雅华贵的精神气质和追求。‎ 答案 BE 解析 B项,“末照”指余辉,“后世仰末照”指的是后世的人们依旧敬仰他的功绩。E项,这首诗直书其事,直抒胸臆,可说是最为质朴的写法,“雄浑壮丽的语言,缠绵哀怨的气韵”错误。‎ ‎(2)有人评此诗:“此托鲁连起兴以自比。”结合诗句,谈谈李白借鲁仲连表达了自己怎样的人生理想。‎ 答:____________________________________________________‎ ‎____________________________________________________________‎ 答案 ①辅弼天下,建功立业。结合诗句“却秦振英声,后世仰末照”。②不慕名利,功成身退。结合诗句“意轻千金赠,顾向平原笑”。(结合诗句具体分析略)‎ 解析 此诗开篇首句运用起兴的手法,引出倜傥高妙的鲁连的形象,再以出海明月的比喻表达了对鲁连如明月般光芒万丈的才华的赞赏,接着两句诗阐述鲁连说服魏赵,合力抗秦的丰功伟绩,以及鲁连淡泊名利、功成身退的高风亮节,令后世仰慕。由此引出作者的人生理想。‎ ‎13.[2017·河北名校联盟联考]阅读下面这首词,完成(1)~(2)题。‎ 满江红·别大名亲旧[注]‎ ‎(元)许衡 河上徘徊,未分袂、孤怀先怯。中年后、此般憔悴,怎禁离别。泪苦滴成襟畔湿,愁多拥就心头结。倚东风、搔首谩无聊,情难说。‎ 黄卷内,消白日。青镜里,增华发。念岁寒交友,故山烟月。虚道人生归去好,谁知美事难双得。计从今、佳会几何时?长相忆。‎ ‎[注] 此词为作者赴朝廷征召时辞别亲友所作,时年46岁。‎ ‎(1)下列对本首词上阕的理解,不正确的两项是(  )‎ A.上阕前三句写词人与亲友依依惜别,在还未分别的时候就感受到分别后孤独无依的忧惧与悲愁。‎ B.“中年后、此般憔悴,怎禁离别”一句,作者面对即将到来的长途充满忧惧,担心路途遥远,艰难困苦太多。‎ C.“泪苦滴成襟畔湿,愁多拥就心头结”正面描写自己身心两方面由离别引起的极度哀愁和痛苦。‎ D.“倚东风、搔首谩无聊,情难说”是说词人站在春风中因痛苦而无所寄托,内心没什么好说的而显得“难说”。‎ E.中国古典诗词历来不乏对离情别绪的描写,许衡的这首《满江红》避开了一般的浮泛描写,处处从个人的真实经历出发,表现出了自己独特的生活感受。‎ 答案 BD 解析 本题考查对诗歌内容的分析与鉴赏能力。上阕是从不同角度、不同具体内容反复铺叙渲染离别之苦。B项,联系自己的身世遭遇,说中年本不适合漂泊,因而此时的离别就更加痛苦。D项“倚东风、搔首谩无聊,情难说”是说内心极其痛苦不知如何表达心中的感情,与柳永的“执手相看泪眼,竟无语凝噎”相类似。“内心没什么好说的”不准确。‎ ‎(2)比较阅读柳永《雨霖铃》下片和本词的下片,分析二者所蕴含的思想情感有何不同之处。‎ 答:____________________________________________________‎ ‎____________________________________________________________‎ 答案 柳词下阕着重写想象中别后的凄楚情景,烘托其伤感之意、思念之情。“多情自古伤离别,更那堪冷落清秋节”,自己离别比古人、常人都痛苦;更痛苦的是酒醒后会是在“杨柳岸、晓风残月”凄凉清寒之处;而且从今往后漫长孤独的日子里,即使有良辰美景也无人陪伴,无心欣赏。‎ 许词下片是对“难说”之“情”的具体诉说,离别之苦中含有人生迟暮及矛盾徘徊之感。下片开始四句写大好的青春年华消磨在黄卷里,青镜里也添了不少白发,对比之中有一种沉重的人生迟暮之感。他怀念恬淡悠然的隐居生活:在寒冷的日子里有好友可交往,出门来有故乡的山水云月可赏游,这是一种对心灵自由的追求。因应召而离别,使他陷入了一种无法解脱的人生困境:自由恬淡的归隐生活虽好但君命难违,只能无奈感叹“美事难双得”。正是这种种难言的隐衷使词人在离别之际更加感伤和哀痛。‎ 解析 本题考查的是诗歌内容和思想情感的分析、鉴赏能力以及比较阅读能力。关键是要吃透两首词的相关内容及其思想情感。柳词主要是写男女之间儿女情长的离别之苦;而许词是亲友之间的离别,其中夹杂着中年人生的种种感触。将课内词作与课外词作作比较阅读,此种题型要注意。‎ ‎14.[2017·湖南省六校联盟联考]阅读下面一首宋诗,完成(1)~(2)题。‎ 水村闲望 俞紫芝①‎ ‎[注] ①俞紫芝,字秀老,金华人。官场失意后,蛰居水村,寄情于山水。②赊:语气词。③绷:布幅。‎ ‎(1)对作品赏析不恰当的两项是(  )‎ A.首联抓住水村的特点,先写停泊在沙洲上的船,再写隔岸烟雾迷朦中的野草,远近明暗搭配和谐。‎ B.颔联描绘翠鸟闲居荷叶之下,鹭鸶栖息于芦花丛中,这样一幅清幽冷寂的画面,传达出诗人闲适的心境。‎ C.颈联采用动静结合的手法来写景,酒旗飘飘为动景,渔屋酒家为静景,一动一静,动中有静,更增情味。‎ D.尾联“无人写得寄京华”一句,写诗人不禁想画一幅《水村闲望》画,表明他官场失意后的愤懑之情。‎ E.诗的前三联写景,由沙滩到隔岸,然后转到河内,再移到岸上,跳跃变化大,给人以目不暇接的感受。‎ 答案 BD 解析 B项,颔联描绘的是一幅幽静的画面,并非清幽冷寂。D项,尾联表达了诗人鄙弃官场、寄情山水的情怀,并没有官场失意的愤懑。‎ ‎(2)有人说,整首诗的前三联都是对水村景物的生动刻画,但末句提到“京华”,显得有些突兀,你是如何看待这一点的呢?请简要阐述。‎ 答:____________________________________________________‎ ‎____________________________________________________________‎ 答案 这种说法不妥。诗人在前三联描绘出一幅恬淡静谧的深秋水村图,呈现出一种安宁朦胧的意境,从而表现了作者悠然闲适的心绪和寄情山水的情怀。尾联写可惜这一幅栩栩如生的水墨画,没有人画了寄到京华去,去劝说那些朝廷中人远离官场,归返自然,表达了诗人鄙弃浑浊官场的情怀。前后情怀是一致的,不显突兀。‎ 解析 这首诗的前三联写景,由沙滩到隔岸,然后转到河内,再移到岸边,最后延伸到河外,虽然跳跃变化很大,却给人以目不暇接的异样感受。最后,这位“红颜弃轩冕,白首卧松云”的诗人,不禁想劝一劝那些可怜的身处“朱门务倾夺”中的人们,切莫“就中奔走费光阴”,自己多想画一幅《水村闲望》的写意画,再题上这首诗送给他们。可见,前三联重在写景,尾联重在抒情。前三联借景抒情,尾联则直抒胸臆,前后贯通,情怀一致。‎ ‎考点十四 默写常见的名句名篇 考点名片 考点内容 ‎①所考名句一般不超出新课标规定的初高中必背古代诗文篇目范围;②个别年份涉及课外常见名句。尤其是出自《论语》《孟子》《荀子》等文化经典的名句;③句中多有易错字、易混字。‎ 考查形式 填空题,在具体语境中填空,强调理解,5分。‎ 趋势分析 ‎①全国卷以必背古代诗文篇目为考查范围;②部分自主命题试卷依据本省市自定篇目考查;③全国卷将延续初中名句2空、高中名句3空的题量比例;④部分自主命题试卷仍会有“任选”“限选”等要求。‎ ‎1.补写出下列句子中的空缺部分。‎ ‎(1)《氓》中写女子回忆小时候与男子嬉戏玩耍的快乐场景的句子是:______________,______________。‎ ‎(2)《登飞来峰》中富于哲理,借景抒情,表达自己锐意改革的远大政治抱负的诗句是:______________,______________。‎ ‎(3)______________,夜泊秦淮近酒家。(杜牧《泊秦淮》)‎ 答案 (1)总角之宴 言笑晏晏 (2)不畏浮云遮望眼 自缘身在最高层 (3)烟笼寒水月笼沙 ‎2.补写出下列名篇名句中的空缺部分。‎ ‎(1)王维在《使至塞上》中用“______________,______________”两句对沙漠中的典型景物进行了细致的刻画。‎ ‎(2)人们常说,“活到老,学到老”,荀子《劝学》篇中的“______________”一句印证了这句话。‎ ‎(3)《蜀道难》中运用夸张修辞方法,写出秦蜀之间崇山峻岭、不可逾越的句子是“____________,____________”。‎ 答案 (1)大漠孤烟直 长河落日圆 (2)学不可以已 (3)西当太白有鸟道 可以横绝峨眉巅 ‎3.补写出下列名篇名句中的空缺部分。‎ ‎(1)《劝学》中用“吾尝终日而思矣,不如须臾之所学也;______________,______________”来说明学习的作用。‎ ‎(2)《蜀道难》中“______________,______________”两句化用了张载《剑阁铭》中“形胜之地,匪亲勿居”的句子,借古说今,暗寓了蜀道之难自古而然。‎ ‎(3)《醉翁亭记》中体现全文核心及醉翁命名之意的句子是:“______________,在乎山水之间也。”‎ 答案 (1)吾尝跂而望矣 不如登高之博见也 (2)所守或匪亲 化为狼与豺 (3)醉翁之意不在酒 ‎4.补写出下列句子中的空缺部分。‎ ‎(1)《离骚》中用香草做比喻说明自己遭贬黜是因为德行高尚的两句是:“______________,______________。”‎ ‎(2)《离骚》中,屈原通过加高帽子和加长佩带的做法,表明要使自己品格更加高洁的两句是“______________,______________”。‎ ‎(3)在《逍遥游》中,庄子引“汤之问棘”,说鹏背翼巨大,“______________”,穿越云气,背负青天到南海。‎ 答案 (1)既替余以蕙兮 又申之以揽茝 (2)高余冠之岌岌兮 长余佩之陆离 (3)抟扶摇羊角而上者九万里 ‎5.补写出下列句子中的空缺部分。‎ ‎(1)白居易在《琵琶行》中,既交代秋天的背景又蕴含离别之意的句子是:“______________,______________。”‎ ‎(2)《钱塘湖春行》一诗中,借助莺燕的活动传达了春天来临的信息并透露出诗人喜悦之情的诗句是:“______________,______________。”‎ ‎(3)辛弃疾在《永遇乐·京口北固亭怀古》中写刘裕驰骋疆场所向披靡的一句是:“______________。”‎ 答案 (1)浔阳江头夜送客 枫叶荻花秋瑟瑟 (2)几处早莺争暖树 谁家新燕啄春泥 (3)气吞万里如虎 ‎6.补写出下列名篇名句中的空缺部分。‎ ‎(1)柳宗元《小石潭记》中描写的小石潭虽然环境景物幽美静穆,但氛围过于冷清,让人感到“______________,______________”,不可久留。‎ ‎(2)白居易在《琵琶行》中用“______________,______________”来表现琵琶女内心的怨愤,即使在演奏间歇仍能使读者感受至深。‎ ‎(3)苏轼《念奴娇·赤壁怀古》,表现诗人旷达之情的一句是:______________。‎ 答案 (1)凄神寒骨 悄怆幽邃 (2)别有幽愁暗恨生 此时无声胜有声 (3)一尊还酹江月 ‎7.补写出下列句子的空缺部分。‎ ‎(1)《离骚》中用“______________,余不忍为此态也”两句来直接表白自己宁死也不与小人们同流合污的心志。‎ ‎(2)《赤壁赋》中描写诗人泛舟江上随意飘荡自由来去的情景的两句是:“______________,______________。”‎ ‎(3)《渔家傲·秋思》中用“______________,______________”两句,点出了“塞下”的地域性的特点,并以“异”字领起全篇,为下片怀乡思归之情埋下了伏线。‎ 答案 (1)宁溘死以流亡兮 (2)纵一苇之所如 凌万顷之茫然 (3)塞下秋来风景异 衡阳雁去无留意 ‎8.补写出下列句子中的空缺部分。‎ ‎(1)郦道元《三峡》中,从色彩、情态入手描绘三峡潭水春冬美景的句子是:____________,____________。‎ ‎(2)荀子在《劝学》中认为君子最终能超过一般人,是因为君子:____________。‎ ‎(3)《送东阳马生序》中“______________,______________”两句描述了自己早年穿着旧衣服出入于那些衣着光鲜的同学之间,而心中毫无羡慕之情的状态。‎ 答案 (1)(则)素湍绿潭 回清倒影 (2)善假于物(也)‎ ‎(3)余则缊袍敝衣处其间 略无慕艳意 ‎9.补写出下列名句名篇中的空缺部分。‎ ‎(1)杜甫《登高》中的“______________,______________”两句抒发了漂泊异乡、年老多病的复杂情感。‎ ‎(2)唐诗常常以与“秦”相关的词语指代长安及其附近地区。王勃《送杜少府之任蜀州》中“______________”一句里也有类似的指代。‎ ‎(3)杜牧在《阿房宫赋》议论部分中批判秦统治者穷奢极欲,以“______________,______________”两句揭露其疯狂掠夺财富而大肆挥霍的暴行。‎ 答案 (1)万里悲秋常作客 百年多病独登台 (2)城阙辅三秦 (3)奈何取之尽锱铢 用之如泥沙 ‎10.补写出下列句子中的空缺部分。‎ ‎(1)《永遇乐·京口北固亭怀古》中写统治者妄想追求霍去病那样的筑坛祭天功业,结果吃了败仗,张皇南逃,狼狈不堪的两句是:“______________,______________。”‎ ‎(2)李白《蜀道难》中“______________”一句,写出了蜀道旁崖谷雷鸣的奇险场景。‎ ‎(3)周敦颐在《爱莲说》中表达自己喜爱莲花的高洁品性的两句是:“____________________,______________。”‎ 答案 (1)封狼居胥 赢得仓皇北顾 (2)砯崖转石万壑雷 (3)予独爱莲之出淤泥而不染 濯清涟而不妖 ‎11.补写出下列句子中的空缺部分。‎ ‎(1)《师说》以为子择师和自己不从师作对比,韩愈直接点明自己的态度,认为这样做,最终导致的结果是“____________,____________”。‎ ‎(2)李商隐《锦瑟》一诗中明确表达了自己的惆怅苦痛,对当时之情为之哀婉不已的一句是:______________。‎ ‎(3)白居易《琵琶行》中琵琶女演奏琵琶过程中,声虽“暂歇”,却产生更好效果的两句是:______________,______________。‎ 答案 (1)小学而大遗 吾未见其明也 ‎(2)只是当时已惘然 ‎(3)别有幽愁暗恨生 此时无声胜有声 ‎12.根据要求,补写出下列句子中的空缺部分。‎ ‎(1)《诗经·卫风·氓》中写女主人公回忆儿时与氓两小无猜、快乐说笑的两句是:______________,______________。‎ ‎(2)《阿房宫赋》中描写秦王的“套房”多、女人多、财宝多,结果在楚人的一把火里化为焦土,令人叹惋。文中揭示造成这种悲剧根源的一句是:______________。‎ ‎(3)晋·傅玄说的“近朱者赤,近墨者黑”,指出了环境对人的影响;而《爱莲说》中的“______________,______________”则表明了人可以不受环境的影响。‎ 答案 (1)总角之宴 言笑晏晏 (2)秦爱纷奢 (3)莲之出淤泥而不染 濯清涟而不妖 ‎13.补写出下列句子中的空缺部分。‎ ‎(1)《师说》中韩愈认为“弟子不必不如师,师不必贤于弟子”的原因是:“______________,______________。”‎ ‎(2)人生常常面临各种各样的抉择。当“义”和“生”不能两全时,我们应当像孟子所说的那样有所取舍,“______________”。‎ ‎(3)苏轼在《赤壁赋》中极写曹操志得意满,趾高气扬的气势,他站在船头“______________,______________”。‎ 答案 (1)闻道有先后 术业有专攻 (2)舍生而取义者也 (3)酾酒临江 横槊赋诗 ‎14.补写出下列句子中的空缺部分。‎ ‎(1)曹操《观沧海》中提及诗歌作用的句子是:______________。‎ ‎(2)李白《行路难》抒发自己终有一天会实现自己凌云壮志的句子是:______________,______________。‎ ‎(3)《次北固山下》中蕴含着积极乐观的生活哲理、与“沉舟侧畔千帆过,病树前头万木春”有异曲同工之妙的诗句是:______________,______________。‎ 答案 (1)歌以咏志 (2)长风破浪会有时 直挂云帆济沧海 (3)海日生残夜 江春入旧年 ‎15.补写出下列名篇名句中的空缺部分。‎ ‎(1)《氓》中“______________,______________”两句,以河流有岸、湿地有边比喻凡事都有底线,暗示女主人公对负心男子的忍耐已经到了极限。‎ ‎(2)《劝学》中,写螃蟹具备良好的外在条件,却因内心浮躁,只能寄居的句子是“______________,______________”。‎ ‎(3)《虞美人》中,李煜用比喻这样描写自己的愁情:______________。‎ 答案 (1)淇则有岸 隰则有畔 (2)蟹六跪而二螯 非蛇鳝之穴无可寄托者 (3)恰似一江春水向东流 ‎16.补写出下列句子中的空缺部分。‎ ‎(1)用《出师表》中的话来说,“先帝”三顾茅庐是为了“______________”,诸葛亮为了报答“先帝”的知遇之恩,“______________”。‎ ‎(2)《虞美人》中“______________”直接表现出李煜思念故国而又悔恨交加的复杂心境。‎ ‎(3)杜牧在《阿房宫赋》中,用“______________,______________”表现秦人搜刮钱财时斤斤计较,但使用起来却视如敝屣的状态。‎ 答案 (1)咨臣以当世之事 遂许先帝以驱驰 (2)故国不堪回首月明中 (3)奈何取之尽锱铢 用之如泥沙 ‎17.请根据要求补写出下列句子中的空缺部分。‎ ‎(1)《离骚》中,屈原重申自己始终以“好修”为乐之后,用“______________,岂余心之可惩”两句来表明要坚持理想,即使遭受极刑也不改变的决心。‎ ‎(2)晏殊《浣溪沙》(一曲新词酒一杯)中,被誉为“天然奇偶”的千古佳句是“______________,______________”。‎ ‎(3)白居易《琵琶行》中“______________,______________”两句,既写出了作者等人对琵琶女的盛情相邀,也写出了琵琶女的矜持娇羞。‎ 答案 (1)虽体解吾犹未变兮 (2)无可奈何花落去 似曾相识燕归来 (3)千呼万唤始出来 犹抱琵琶半遮面 ‎18.补写出下列句子中的空缺部分。‎ ‎(1)杜牧《阿房宫赋》中,展示出“长廊”和“檐牙”的特点的两句是“______________,______________”。‎ ‎(2)《卫风·氓》中“______________”短短数字,便刻画出氓看起来忠厚老实的形象。‎ ‎(3)《离骚》里屈原面对背直追曲的时代虽然感到忧郁苦闷、走投无路,却也表示自己“______________,______________”,这表明了他不愿苟合取容的高尚情操。‎ 答案 (1)廊腰缦回 檐牙高啄 (2)氓之蚩蚩 (3)宁溘死以流亡兮 余不忍为此态也 ‎19.补写出下列句子中的空缺部分。‎ ‎(1)《念奴娇·赤壁怀古》中,描写江水汹涌澎湃、波浪翻卷如雪的一句是:______________。‎ ‎(2)龚自珍在《己亥杂诗》中形象地表达了自己虽然辞官,但仍会关心国家的前途和命运且代落花立意,倾吐心曲,表现崇高的献身精神的句子是“____________,____________”。‎ ‎(3)苏轼在《赤壁赋》中以月亮作比,描写世间万物变化的规律的句子是:“______________,______________。”‎ 答案 (1)卷起千堆雪 (2)落红不是无情物 化作春泥更护花 (3)盈虚者彼 而卒莫消长也 ‎20.补写出下列句子中的空缺部分。‎ ‎(1)韩愈《早春呈水部张十八员外》一诗中,与杜甫的“好雨知时节,当春乃发生。随风潜入夜,润物细无声”有异曲同工之妙的诗句是:“______________,______________。”‎ ‎(2)“______________,______________”是《琵琶行》全诗的主旨,更是诗人与琵琶女感情的共鸣。‎ ‎(3)柳宗元《小石潭记》中“佁然不动,______________”两句,描写了小鱼由静而动的活泼情景,生动形象。‎ 答案 (1)天街小雨润如酥 草色遥看近却无 (2)同是天涯沦落人 相逢何必曾相识 (3)俶尔远逝 ‎21.补写出下列句子中的空缺部分。‎ ‎(1)李白的《蜀道难》中“______________,______________”两句,融会了五丁开山的神话,刻画了蜀道的高危难行。‎ ‎(2)苏轼的《赤壁赋》中“______________,______________”两句,运用对比手法慨叹生命短促、外物永恒。‎ ‎(3)李煜的《虞美人》中直接抒发亡国之恨的句子是“小楼昨夜又东风,______________”。‎ 答案 (1)地崩山摧壮士死 然后天梯石栈相钩连 (2)哀吾生之须臾 羡长江之无穷 (3)故国不堪回首月明中 ‎22.补写出下列句子中的空缺部分。‎ ‎(1)《岳阳楼记》中能体现岳阳楼长联上“范希文两字关情”的句子是“______________,______________”。‎ ‎(2)杜甫的《登高》中,诗人用“______________,_____________”分别从时间和空间维度抒发自己的多重愁苦,被宋代学者罗大经认为“十四字之间含有八意,而对偶又极精确”。‎ ‎(3)庄子在《逍遥游》结尾中列举出三类人的“无所待”才是真正的逍遥的句子是(任答一类人即可):______________________。‎ 答案 (1)先天下之忧而忧 后天下之乐而乐 (2)万里悲秋常作客 百年多病独登台 (3)至人无己,神人无功,圣人无名(任选一句)‎ ‎23.补写出下列句子中的空缺部分。‎ ‎(1)《离骚》中屈原用制作衣服的材料表示自己品行高洁的句子是:______________,______________。‎ ‎(2)论及“学”与“思”的问题,孔子认为“学而不思则罔,思而不学则殆”,而荀子在《劝学》中则说“___________,___________”。‎ ‎(3)《观刈麦》中点出贫妇人生活如此困窘的原因是:______________。‎ 答案 (1)制芰荷以为衣兮 集芙蓉以为裳 (2)吾尝终日而思矣 不如须臾之所学也 (3)家田输税尽 ‎24.补写出下列句子中的空缺部分。‎ ‎(1)《道德经》提及的“善人,不善人之师;不善人,善人之资”,与《论语》中的“____________,____________”有异曲同工之妙。‎ ‎(2)荀子在《劝学》中指出:“青,取之于蓝,而青于蓝。”这与韩愈《师说》中“______________,______________”的观点是相似的。‎ ‎(3)《使至塞上》中,诗人借两种自然景物自况,写出了飘零之感的两句是:“______________,______________。”‎ 答案 (1)择其善者而从之 其不善者而改之 (2)(是故)弟子不必不如师 师不必贤于弟子 (3)征蓬出汉塞 归雁入胡天 ‎25.补写出下列句子中的空缺部分。‎ ‎(1)杜甫在《春望》中,用“______________,______________”两句表现了诗人在国破家亡、离乱伤痛之外又因为衰老更增添了一层悲愁。‎ ‎(2)《赤壁赋》写作者与友人于扁舟举杯共饮的句子是“____________,____________”。‎ ‎(3)《三峡》中有一句话把三峡春冬季节山水草木的秀丽景色概括无遗,这句话是:“______________。”‎ 答案 (1)白头搔更短 浑欲不胜簪 (2)驾一叶之扁舟 举匏樽以相属 (3)清荣峻茂 ‎26.补写出下列句子中的空缺部分。‎ ‎(1)《庄子·逍遥游》中,上承对蝉和斑鸠嘲笑的大鹏的叙写,下启对朝菌、寒蝉等的论说的句子是:“______________,______________。”‎ ‎(2)李白《蜀道难》中用“______________,______________”这两句,写出了山路极其曲折、迂回的情形。‎ ‎(3)王维以“大漠孤烟直,长河落日圆”突出体现了边塞的壮美,范仲淹在《渔家傲·秋思》中运用相似的景物渲染了边塞的悲凉,这句词是“____________,____________”。‎ 答案 (1)小知不及大知 小年不及大年 (2)青泥何盘盘 百步九折萦岩峦 (3)千嶂里 长烟落日孤城闭 ‎27.补写出下列句子中的空缺部分。‎ ‎(1)莫听穿林打叶声,____________。(《定风波》)‎ ‎(2)《离骚》中表明作者保持清白为正道而死,也是以古贤为榜样的两句(表明自己追慕古代圣贤,宁死不失正义):____________,____________。‎ ‎(3)《陋室铭》中,虚实结合写室主人交往之雅的句子是:“______________,______________。”‎ 答案 (1)何妨吟啸且徐行 (2)伏清白以死直兮 固前圣之所厚 (3)谈笑有鸿儒 往来无白丁 ‎28.补写出下列句子中的空缺部分。‎ ‎(1)“秋”之一字,自古便动人情肠,它是《氓》诗中弃妇眼里“桑之落矣,______________”的哀婉凄凉,是《登高》诗中杜甫面对“风急天高猿啸哀,______________”的秋景心中涌起的悲凉与哀愁,而白居易与友人分别时更是在《琵琶行》的开篇即借“______________”的秋景来渲染那道不尽的离愁与伤怀,国人“伤春悲秋”的传统也许正源于古典诗歌中那浓厚的秋意与悲凉。‎ ‎(2)苏轼在《赤壁赋》中写了“客”悲伤的原因,其中因与天地相比自身显得渺小的语句是:“______________,______________。”‎ ‎(3)杜甫在《登高》中由高到低,写其所见所闻,渲染秋江景物特点的句子是:“______________,______________。”‎ 答案 (1)其黄而陨 渚清沙白鸟飞回 枫叶荻花秋瑟瑟 (2)寄蜉蝣于天地 渺沧海之一粟 (3)风急天高猿啸哀 渚清沙白鸟飞回 ‎29.补写出下列句子中的空缺部分。‎ ‎(1)文天祥在《过零丁洋》中用“____________”一句,表达了国事的艰难。‎ ‎(2)《氓》中写男子变化无常,三心二意的句子是:____________,____________。‎ ‎(3)《逍遥游》中“______________,______________”,指出顺应天地万物之性、驾驭六气的变化才是真正的逍遥游。‎ 答案 (1)山河破碎风飘絮 (2)士也罔极 二三其德 (3)若夫乘天地之正 而御六气之辩 ‎30.补写出下列句子中的空缺部分。‎ ‎(1)《邹忌讽齐王纳谏》一文中,描述齐威王纳谏后,群臣纷纷进谏的场面的句子是:“____________,____________。”‎ ‎(2)《送东阳马生序》中,揭示精神追求战胜物质贫困的句子是:______________,______________。‎ ‎(3)李煜的《相见欢》中,描写庭院凄清的句子是:______________。‎ 答案 (1)群臣进谏 门庭若市 (2)以中有足乐者 不知口体之奉不若人也 (3)寂寞梧桐深院锁清秋 ‎ [3年高考真题集训]‎ ‎1.[2016·全国卷Ⅰ]补写出下列句子中的空缺部分。‎ ‎(1)《荀子·劝学》指出,蚯蚓虽然身体柔弱,却能“__________,__________”,是用心专一的缘故。‎ ‎(2)在《出师表》开头,诸葛亮向后主指出,先帝刘备过早去世,“__________,__________”,正是危急存亡之时。‎ ‎(3)在《永遇乐(千古江山)》中,辛弃疾回顾了元嘉年间的那次北伐,宋文帝刘义隆本希望能够“__________”,但由于行事草率,最终却“__________”。‎ 答案 (1)上食埃土 下饮黄泉 ‎(2)今天下三分 益州疲弊 ‎(3)封狼居胥 赢得仓皇北顾 解析 易错字:(1)埃;(2)弊;(3)胥,赢,仓皇。‎ ‎2.[2016·全国卷Ⅱ]补写出下列句子中的空缺部分。‎ ‎(1)《孟子·鱼我所欲也》中表示,生是我希望得到的,义也是我希望得到的,但“__________,__________”。‎ ‎(2)李白《蜀道难》中“__________,__________”两句,以感叹的方式收束对蜀道凶险的描写,转入后文对人事的关注。‎ ‎(3)杜牧《阿房宫赋》中以“__________,__________”描写阿房宫宫人的美丽,她们伫立远眺,盼望皇帝临幸。‎ 答案 (1)二者不可得兼 舍生而取义者也 ‎(2)其险也如此 嗟尔远道之人胡为乎来哉 ‎(3)一肌一容 尽态极妍 解析 易错字:(1)得兼;(2)嗟,哉;(3)妍。‎ ‎3.[2016·全国卷Ⅲ]补写出下列句子中的空缺部分。‎ ‎(1)《左传·曹刿论战》中记载,鲁庄公十年,齐国入侵。曹刿求见国君献策,但他的乡人质疑道:“__________,__________?”‎ ‎(2)严格地说,浔阳并非绝对没有音乐,只是声音单调繁杂,实在难以入耳。白居易《琵琶行》中“__________,__________”两句表达了这样的意思。‎ ‎(3)在《赤壁赋》的开头,苏轼写自己与朋友泛舟赤壁之下,朗诵《诗经·陈风》中的《月出》篇,即文中所谓“__________,__________”。‎ 答案 (1)肉食者谋之 又何间焉 ‎(2)岂无山歌与村笛 呕哑嘲哳难为听 ‎(3)诵明月之诗 歌窈窕之章 解析 易错字:(1)焉;(2)嘲哳;(3)窈窕。‎ ‎4.[2016·北京高考]在横线上填写作品原句。‎ ‎(1)陆游在诗中称西村为“小桃源”,使人联想到《桃花源记》,其中有:______________,______________,有良田美池桑竹之属;______________,______________。‎ ‎(2)追忆往事,是陆游诗歌中常有的内容,如《书愤》一诗中“______________,______________”一联,就是对抗金历史的回忆。‎ ‎(3)《西村》是一首律诗,中间两联是对仗的。杜甫《登高》中也有两联是对仗的,请写出其中一联。______________,______________。‎ 答案 (1)土地平旷 屋舍俨然 阡陌交通 鸡犬相闻 (2)楼船夜雪瓜洲渡 铁马秋风大散关 (3)无边落木萧萧下 不尽长江滚滚来 (或:万里悲秋常作客 百年多病独登台)‎ 解析 易错字:(1)俨,阡陌;(2)渡;(3)萧萧(作客)。‎ ‎5.[2016·山东高考]补写出下列句子中的空缺部分。‎ ‎(1)《庄子·逍遥游》描写迁徙南溟的大鹏击水之广、飞升之高的句子是“__________,__________”。‎ ‎(2)杜甫《登岳阳楼》颔联“__________,__________”,描写了洞庭湖浩瀚壮阔的景色,千古传诵。‎ ‎(3)李煜《虞美人》词中“__________?__________”两句,以江水比喻自己的愁绪,抒发亡国之君无尽的痛苦之情。‎ 答案 (1)水击三千里 抟扶摇而上者九万里 ‎(2)吴楚东南坼 乾坤日夜浮 ‎(3)问君能有几多愁 恰似一江春水向东流 解析 易错字:(1)抟;(2)坼,乾坤;(3)恰似。‎ ‎6.[2016·天津高考]补写出下列名篇名句中的空缺部分。‎ ‎(1)良将劲弩守要害之处,______________。(贾谊《过秦论》)‎ ‎(2)既窈窕以寻壑,______________。(陶渊明《归去来兮辞》)‎ ‎(3)______________,下有冲波逆折之回川。(李白《蜀道难》)‎ ‎(4)______________,乾坤日夜浮。(杜甫《登岳阳楼》)‎ ‎(5)竹杖芒鞋轻胜马,谁怕?______________。(苏轼《定风波》)‎ 答案 (1)信臣精卒陈利兵而谁何 (2)亦崎岖而经丘 (3)上有六龙回日之高标 (4)吴楚东南坼 (5)一蓑烟雨任平生 解析 易错字:(1)何;(2)崎岖;(3)标;(4)坼;(5)蓑。‎ ‎7.[2016·上海高考]默写。‎ ‎(1)非独贤者是有心也,人皆有之,______________。(《孟子·告子上》)‎ ‎(2)______________,孤城遥望玉门关。(王昌龄《从军行》)‎ ‎(3)楚天千里清秋,______________。(辛弃疾《水龙吟·登建康赏心亭》)‎ ‎(4)夕阳下,酒旆闲,______________。(马致远《【双调】寿阳曲·远浦帆归》)‎ ‎(5)______________,身世浮沉雨打萍。(文天祥《过零丁洋》)‎ ‎(6)知足不辱,______________。(《老子》第四十五章)‎ ‎(7)奇文共欣赏,______________。(陶渊明《移居》)‎ ‎(8)山一程,水一程,身向榆关那畔行,______________。(纳兰性德《长相思》)‎ 答案 (1)贤者能勿丧耳 (2)青海长云暗雪山 (3)水随天去秋无际 (4)两三航未曾着岸 (5)山河破碎风飘絮 ‎(6)知止不殆 (7)疑义相与析 (8)夜深千帐灯 ‎8.[2016·江苏高考]补写出下列名句名篇中的空缺部分。‎ ‎(1)朝搴阰之木兰兮,______________。(屈原《离骚》)‎ ‎(2)不宜妄自菲薄,______________,以塞忠谏之路也。(诸葛亮《出师表》)‎ ‎(3)男女衣着,悉如外人。______________,并怡然自乐。(陶渊明《桃花源记》)‎ ‎(4)地崩山摧壮士死,______________。(李白《蜀道难》)‎ ‎(5)______________,蓝田日暖玉生烟。(李商隐《锦瑟》)‎ ‎(6)故国神游,多情应笑我,______________。(苏轼《念奴娇·赤壁怀古》)‎ ‎(7)______________,于我如浮云。(《论语·述而》)‎ ‎(8)心事浩茫连广宇,______________。(鲁迅《无题》)‎ 答案 (1)夕揽洲之宿莽 (2)引喻失义 (3)黄发垂髫 (4)然后天梯石栈相钩连 (5)沧海月明珠有泪 (6)早生华发 (7)不义而富且贵 (8)于无声处听惊雷 ‎9.[2016·浙江高考]补写出下列名篇名句的空缺部分。‎ ‎(1)______________,筋骨之强,______________,下饮黄泉,用心一也。(《荀子·劝学》)‎ ‎(2)夫以秦王之威,______________,辱其群臣。______________,独畏廉将军哉?(司马迁《史记·廉颇蔺相如列传》)‎ ‎(3)客喜而笑,____________。肴核既尽,____________。(苏轼《赤壁赋》)‎ ‎(4)______________,封狼居胥,______________。(辛弃疾《永遇乐·京口北固亭怀古》)‎ ‎(5)昨夜雨疏风骤,______________。______________,却道海棠依旧。(李清照《如梦令·昨夜雨疏风骤》)‎ 答案 (1)蚓无爪牙之利 上食埃土 (2)而相如廷叱之 相如虽驽 (3)洗盏更酌 杯盘狼籍 (4)元嘉草草 赢得仓皇北顾 (5)浓睡不消残酒 试问卷帘人 ‎10.[2015·全国卷Ⅰ]补写出下列句子中的空缺部分。‎ ‎(1)在《离骚》中,屈原诉说自己曾因佩戴蕙草而遭到贬逐,也曾被加上采摘白芷的罪名,但他坚定地表示:“________________,________________”。‎ ‎(2)王维《使至塞上》中“________________,________________”一联,写了到达边塞后看到的奇特壮丽风光,画面开阔,意境雄浑。‎ ‎(3)苏轼《念奴娇(大江东去)》中“________________,________________”两句,收束了对赤壁雄奇景物的描写,引起后面对历史的缅怀。‎ 答案 (1)亦余心之所善兮 虽九死其犹未悔 ‎(2)大漠孤烟直 长河落日圆 ‎(3)江山如画 一时多少豪杰 ‎11.[2015·全国卷Ⅱ]补写出下列句子中的空缺部分。‎ ‎(1)《庄子·逍遥游》指出,“________________,________________”,就像倒在堂前洼地的一杯水,无法浮起一个杯子一样。‎ ‎(2)白居易《琵琶行》中“________________,________________”两句,写的是演奏正式开始之前的准备过程。‎ ‎(3)杜牧《赤壁》中“________________,________________”两句,设想了赤壁之战双方胜败易位后将导致的结局。‎ 答案 (1)(且夫)水之积也不厚 则其负大舟也无力 (2)转轴拨弦三两声 未成曲调先有情 (3)东风不与周郎便 铜雀春深锁二乔 ‎12.[2014·全国卷Ⅰ]补写出下列句子中的空缺部分。‎ ‎(1)屈原在《离骚》中表现自己同情百姓的苦难生活,并因此流泪叹息的名句是“____________,____________”。‎ ‎(2)李白《蜀道难》中“____________,____________”两句写山势高险,即便是善飞的黄鹤、轻捷的猿猴都很难越过。‎ ‎(3)杜甫在《春望》中借花鸟以抒发自己悲愤情感的名句是“____________,____________”。‎ 答案 (1)长太息以掩涕兮 哀民生之多艰 ‎(2)黄鹤之飞尚不得过 猿猱欲度愁攀援 ‎(3)感时花溅泪 恨别鸟惊心 ‎13.[2014·全国卷Ⅱ]补写出下列句子中的空缺部分。‎ ‎(1)《庄子·逍遥游》中以“朝菌”和“蟪蛄”为例来说明“小年”一词的两句是“____________,____________”。‎ ‎(2)李白《行路难(金樽清酒斗十千)》一诗经过大段的反复回旋,最后境界顿开,用“‎ ‎____________,____________”两句表达了诗人的乐观和自信。‎ ‎(3)在《赤壁赋》中,苏轼用“____________,____________”两句概括了曹操的军队在攻破荆州后顺流东下时的军容之盛。‎ 答案 (1)朝菌不知晦朔 蟪蛄不知春秋 (2)长风破浪会有时 直挂云帆济沧海 (3)舳舻千里 旌旗蔽空 ‎14.[2015·安徽高考]补写出下列名篇名句中的空缺部分。‎ 甲:问君西游何时还?畏途巉岩不可攀。①____________,②____________。③____________,④____________。⑤____________,⑥____________,使人听此凋朱颜!(李白《蜀道难》)‎ 乙:①悬泉瀑布,____________,清荣峻茂,良多趣味。(郦道元《三峡》)‎ ‎②____________,其为惑也,终不解矣。(韩愈《师说》)‎ ‎③几处早莺争暖树,____________。(白居易《钱塘湖春行》)‎ ‎④蓬山此去无多路,____________。(李商隐《无题》)‎ ‎⑤____________,歌窈窕之章。(苏轼《赤壁赋》)‎ ‎⑥想当年,金戈铁马,____________。(辛弃疾《永遇乐》)‎ 答案 甲:①但见悲鸟号古木 ②雄飞雌从绕林间 ③又闻子规啼夜月 ④愁空山 ⑤蜀道之难 ⑥难于上青天 乙:①飞漱其间 ②惑而不从师 ③谁家新燕啄春泥 ‎④青鸟殷勤为探看 ⑤诵明月之诗 ⑥气吞万里如虎 ‎15.[2015·广东高考]补写下列句子中的空缺部分。‎ ‎(1)____________,何时可掇?____________,不可断绝。(曹操《短歌行》)‎ ‎(2)____________,____________;雁阵惊寒,声断衡阳之浦。(王勃《滕王阁序》)‎ ‎(3)昨夜闲潭梦落花,____________。江水流春去欲尽,____________。(张若虚《春江花月夜》)‎ ‎(4)夫夷以近,____________;险以远,____________。(王安石《游褒禅山记》)‎ 答案 (1)明明如月 忧从中来 (2)渔舟唱晚 响穷彭蠡之滨 (3)可怜春半不还家 江潭落月复西斜 (4)则游者众 则至者少 ‎16.[2015·湖北高考]补写出下列名篇名句中的空缺部分。‎ ‎(1)人不知而不愠,____________?(《论语》)‎ ‎(2)君子博学而日参省乎己,____________。(《荀子·劝学》)‎ ‎(3)涉江采芙蓉,____________。(《古诗十九首》)‎ ‎(4)母、孙二人,更相为命,____________。(李密《陈情表》)‎ ‎(5)三径就荒,____________。(陶渊明《归去来兮辞》)‎ ‎(6)岩扉松径长寂寥,____________。(孟浩然《夜归鹿门歌》)‎ ‎(7)群山万壑赴荆门,____________。(杜甫《咏怀古迹》)‎ ‎(8)遥岑远目,献愁供恨,____________。(辛弃疾《水龙吟·登建康赏心亭》)‎ 答案 (1)不亦君子乎 (2)则知明而行无过矣 (3)兰泽多芳草 (4)是以区区不能废远 (5)松菊犹存 (6)惟有幽人自来去 (7)生长明妃尚有村 (8)玉簪螺髻 ‎17.[2015·湖南高考]古诗文默写。‎ ‎(1)沅有芷兮澧有兰,思公子兮未敢言。____________,观流水兮潺湲。(屈原《湘夫人》)‎ ‎(2)或取诸怀抱,____________;或因寄所托,____________。(王羲之《兰亭集序》)‎ ‎(3)千呼万唤始出来,犹抱琵琶半遮面。____________,____________。(白居易《琵琶行(并序)》)‎ 答案 (1)荒忽兮远望 (2)悟言一室之内 放浪形骸之外 (3)转轴拨弦三两声 未成曲调先有情 ‎18.[2015·山东高考]补写出下列句子中的空缺部分。‎ ‎(1)《论语》记载的孔子言论中,用星辰作比喻,形象地表述“为政以德”社会效果的句子是“____________,____________”。‎ ‎(2)陶渊明《归去来兮辞》描写归乡途中轻舟快风的两句“____________,____________”,表达了作者弃官归乡的畅快心情。‎ ‎(3)杜甫五律《旅夜书怀》的颔联“____________,____________”,描绘的景象雄浑阔大,反衬了作者孤苦漂泊的悲怆心情。‎ 答案 (1)譬如北辰 居其所而众星共(拱)之 (2)舟遥遥以轻飏 风飘飘而吹衣 (3)星垂平野阔 月涌大江流 ‎19.[2015·天津高考]按序号在对应横线上补写空缺部分。‎ 诗人笔下的“秋”气象万千。它是王勃眼里“①____________,秋水共长天一色”的寥廓多姿,是杜甫笔下“玉露凋伤枫树林,②____________”的沉雄富丽,是苏轼文中“纵一苇之所如,③____________”的浩渺烟波。它有刘禹锡“晴空一鹤排云上,便引诗情到碧霄”的超拔豪放,有柳永“念去去,千里烟波,④____________”的绵绵愁绪,有辛弃疾“⑤____________,尽西风,季鹰归未”的慷慨悲情,更有毛泽东词中“鹰击长空,鱼翔浅底,⑥____________”的勃勃生机。‎ 答案 ①落霞与孤鹜齐飞 ②巫山巫峡气萧森 ③凌万顷之茫然 ④暮霭沉沉楚天阔 ⑤休说鲈鱼堪脍 ⑥万类霜天竞自由 ‎20.[2015·重庆高考]补写出下列句子中的空缺部分。‎ ‎(1)陶渊明《归去来兮辞》中描写拄着拐杖出去走走,随时随地休息的一句是“____________”。‎ ‎(2)杜牧《阿房宫赋》描写渭水、樊川水流平缓的两句是“____________,____________”。‎ ‎(3)辛弃疾《永遇乐·京口北固亭怀古》写宋文帝刘义隆草率出师北伐,结果落得北望敌军而惊慌失措的三句是“____________,____________,____________”。‎ 答案 (1)策扶老以流憩 (2)二川溶溶 流入宫墙 ‎(3)元嘉草草 封狼居胥 赢得仓皇北顾 ‎[2年全国模拟重组]‎ ‎1.[2017·河北调研]补写出下列句子中的空缺部分。‎ ‎(1)《荀子·劝学》中用“致千里”者却非“利足”,“绝江河”者却非“能水”来证明“____________,____________”的观点。‎ ‎(2)王湾在《次北固山下》中,用“____________”一句描写潮涨江阔的情景。‎ ‎(3)李贺在《雁门太守行》中用“____________,____________”两句从听觉和视觉两方面渲染了战争悲壮、残酷的气氛。‎ 答案 (1)君子生非异也 善假于物也 (2)潮平两岸阔 ‎(3)角声满天秋色里 塞上燕脂凝夜紫 ‎2.[2017·泸州诊断性考试]补写出下列句子中的空缺部分。‎ ‎(1)《岳阳楼记》中“______________,______________”两句通过写花草的香味和颜色,刻画了春天来临时洞庭湖的景物特点。‎ ‎(2)《阿房宫赋》通过一个人的意愿,也就是千万人的意愿这一议论,对比人心事理,进而发出了如下质问“______________,______________”,以此揭露统治者的贪婪、挥霍无度。‎ ‎(3)______________,血色罗裙翻酒污。(白居易《琵琶行》)‎ 答案 (1)岸芷汀兰 郁郁青青 (2)奈何取之尽锱铢 用之如泥沙 (3)钿头银篦击节碎 ‎3.[2017·江西省联考测试]补写出下列句子中的空缺部分。‎ ‎(1)《逍遥游》中,庄子指出,列子虽然御风而行,也没有汲汲求福,但是“此虽免乎行,__________”,不算真正的逍遥游。‎ ‎(2)《破阵子·为陈同甫赋壮词以寄之》中,辛弃疾以“______________,______________”两句描写了战斗的紧张激烈。‎ ‎(3)《琵琶行》中,借助对听者反应的体现来侧面体现琵琶声动听的一句是“______________”。‎ 答案 (1)犹有所待者 (2)马作的卢飞快 弓如霹雳弦惊 (3)东船西舫悄无言 ‎4.[2017·山西大学附中模块诊断]补写出下列句子中的空缺部分。‎ ‎(1)苏轼在《念奴娇·赤壁怀古》中以“______________,______________,______________”三句话生动形象地写出了周瑜在赤壁之战中从容娴雅,沉着应战,指挥若定,轻松自如将敌人消灭的儒将风度。‎ ‎(2)《永遇乐·京口北固亭怀古》中写宋武帝刘裕叱咤风云,驰骋疆场的名句是“‎ ‎____________,____________”。‎ 答案 (1)羽扇纶巾 谈笑间 樯橹灰飞烟灭 (2)金戈铁马 气吞万里如虎 ‎5.[2017·安徽安庆联考]补写出下列句子中的空缺部分。‎ ‎(1)杜牧的《阿房宫赋》描写了秦宫内奢靡的生活场景,他写道:远处仿佛“雷霆乍惊”,那是宫车驶过;车声渐行渐远,最终“________________”。‎ ‎(2)在《阿房宫赋》中,杜牧提出了秦国如果爱惜六国的人才,“____________”,安享天下这一观点。‎ ‎(3)现实社会中,不尊师不虚心求教者常有之,对这种常妄自尊大、师心自用的人,我们可以引用《论语》中“____________,____________”两句来进行劝诫。‎ 答案 (1)杳不知其所之也 (2)则递三世可至万世而为君 (3)三人行 必有我师焉 ‎6.[2017·四川资阳诊断性考试]补写出下列句子中的空缺部分。‎ ‎(1)杜甫在《登高》中,借用“落叶”与“江水”抒发时光易逝、壮志未酬的感伤与失落的两句是“_____________,____________”。‎ ‎(2)《逍遥游》中,看到大鹏鸟经过一系列的准备才能“图南”之后,蜩与学鸠通过形象地描述自己在林中飞行和休息的样子来嘲笑大鹏鸟的句子是“________________,__________________”。‎ ‎(3)苏轼在《念奴娇·赤壁怀古》中通过描写周瑜装束儒雅,从容镇定之间就将曹军打得“________________”的风采,概括出了整个战争的胜利场景。‎ 答案 (1)无边落木萧萧下 不尽长江滚滚来 (2)我决起而飞 抢榆枋而止 (3)樯橹灰飞烟灭 ‎7.[2017·保定摸底考试]补写出下列句子中的空缺部分。‎ ‎(1)沙漠到处结冰,空中阴云密布,岑参《白雪歌送武判官归京》中的“______________,______________”两句将这种景象展现得淋漓尽致。‎ ‎(2)杜甫《登高》中“______________,______________”两句凸显了夔州秋天的典型特征,诗人在写景的同时,抒发了韶光易逝、壮志难酬的悲怆。‎ ‎(3)《出师表》中,诸葛亮希望后主不要自己看轻自己的句子是:“______________。”‎ 答案 (1)瀚海阑干百丈冰 愁云惨淡万里凝 (2)无边落木萧萧下 不尽长江滚滚来 (3)不宜妄自菲薄 ‎8.[2017·辽宁铁岭联考]补写出下列名篇名句中的空缺部分。‎ ‎(1)白居易《琵琶行(并序)》中,“______________,______________”两句用比喻的修辞、对比的手法正面描写音乐。‎ ‎(2)庄子《逍遥游》中“______________,______________”两句,运用了比喻和夸张的修辞手法来写鹏鸟之大。‎ ‎(3)杜甫《茅屋为秋风所破歌》中,“______________,______________”两句诗通过正面和侧面结合的手法描写雨之大和雨之疾。‎ 答案 (1)大弦嘈嘈如急雨 小弦切切如私语 (2)背若泰山 翼若垂天之云 (3)床头屋漏无干处 雨脚如麻未断绝 ‎9.[2017·广东中山统测]补写出下列名篇名句中的空缺部分。‎ ‎(1)在杜牧《阿房宫赋》中,文末总结秦国灭亡的教训,并借古讽今,以“______________,亦使后人而复哀后人也”两句告诫今人如果不从秦的灭亡中引以为鉴,仍要重蹈历史覆辙。‎ ‎(2)庄子在《逍遥游》中引用“______________,______________”两句,说明船和水的关系,阐明了世间万事万物总会有所凭借的道理。‎ ‎(3)《诗经·氓》中一句“______________,______________”是女主人回忆当年恋爱时的情景,这与成语“青梅竹马”的意境相仿。‎ 答案 (1)后人哀之而不鉴之 亦使后人而复哀后人也 (2)且夫水之积也不厚 则其负大舟也无力 (3)总角之宴 言笑晏晏 ‎10.[2017·东北育才学校模拟]补写出下列名篇名句中的空缺部分。‎ ‎(1)《爱莲说》中比喻君子美名远扬的语句是:______________。‎ ‎(2)李白《蜀道难》中在“使人听此凋朱颜”之后进一步突出蜀道山之高危、壁之险绝的两句是“________________,____________”。‎ ‎(3)《离骚》中屈原表明人各有各的乐趣,而他穷其一生追求美政的语句是:______________,______________。‎ 答案 (1)香远益清 (2)连峰去天不盈尺 枯松倒挂倚绝壁 (3)民生各有所乐兮 余独好修以为常 ‎11.[2017·广东五校联考]补写出下列名篇名句中的空缺部分。‎ ‎(1)______________,往来而不绝者,滁人游也。(欧阳修《醉翁亭记》)‎ ‎(2)杜甫的《登高》中,“______________,______________”这两句分别从听觉和视觉这两方面写夔州秋天的典型特征,声音哀怨凄惨,色彩冷艳清淡,渲染了悲凉的气氛。‎ ‎(3)《劝学》中强调空想不如学习的一句是“____________,____________”。‎ 答案 (1)伛偻提携 (2)风急天高猿啸哀 渚清沙白鸟飞回 (3)吾尝终日而思矣 不如须臾之所学也 ‎12.[2017·河南河北名校省际联考]补写出下列句子中的空缺部分。‎ ‎(1)诸葛亮《出师表》中,作者直言“益州疲弊”,自身条件很差,地少将寡,民穷地荒,进而大声疾呼“______________”。大有危在旦夕之势,如不救亡存国,将会出现国破身亡的惨局。‎ ‎(2)白居易《琵琶行》中,写琵琶女沦落天涯后,华年不再,青春已去,只有在梦里回味往昔的“荣光”,梦醒时分,跌回现实中来,无限伤感的两句是:“______________,______________。”‎ ‎(3)孔子在《论语》中论述当别人不了解甚至误解自己时,应当采取的正确态度,表明个人修养的语句是“____________,____________”。‎ 答案 (1)此诚危急存亡之秋也 (2)夜深忽梦少年事 梦啼妆泪红阑干 (3)人不知而不愠 不亦君子乎 ‎13.[2017·江西宜春市四校联考]补写出下列名篇名句中的空缺部分。‎ ‎(1)醉里挑灯看剑,梦回吹角连营。______________,五十弦翻塞外声,沙场秋点兵。‎ ‎(2)______________,渺沧海之一粟。‎ ‎(3)画图省识春风面,______________。‎ ‎(4)杜牧在《阿房宫赋》中,直言阿房宫的宫殿楼阁占地之广,状阿房宫楼阁之高的句子是“_______________,________________”。‎ 答案 (1)八百里分麾下炙 (2)寄蜉蝣于天地 (3)环珮空归夜月魂 (4)覆压三百余里 隔离天日 ‎14.[2017·中原名校第三次质量考评]补写出下列句子中的空缺部分。‎ ‎(1)刘禹锡《酬乐天扬州初逢席上见赠》中运用典故表达对友人的思念,感慨世事沧桑,人事全非的两句诗是“______________,______________”。‎ ‎(2)《赤壁赋》从不变的角度描述人与万物的关系的句子是:____________,____________。‎ ‎(3)《邹忌讽齐王纳谏》中,齐王下令能受上赏的人是“______________”。‎ 答案 (1)怀旧空吟闻笛赋 到乡翻似烂柯人 (2)自其不变者而观之 则物与我皆无尽也 (3)群臣吏民能面刺寡人之过者 ‎15.[2017·大庆一中阶段测试]补写出下列句子中的空缺部分。‎ ‎(1)《出师表》中诸葛亮劝刘禅对宫中、府中官员的赏罚要坚持同一标准的句子:____________,____________。‎ ‎(2)在《师说》中韩愈以反问的形式表达了无论对方年长年少,只要懂得道理,都值得向他学习的句子是:“______________,______________?”‎ ‎(3)《蜀道难》中“______________,______________”两句,写出了长期以来秦蜀不通往来的现状,极言出行之难。‎ 答案 (1)陟罚臧否 不宜异同 (2)吾师道也 夫庸知其年之先后生于吾乎 (3)尔来四万八千岁 不与秦塞通人烟 ‎16.[2017·湖北八校联考]补写出下列句子中的空缺部分。‎ ‎(1)范仲淹在《渔家傲·秋思》中表达自己和征人们想家却又不甘无功而返的矛盾心理的句子是“______________,_____________”。‎ ‎(2)李白《蜀道难》中直接描写剑阁地势险要的句子是“______________”。‎ ‎(3)杜甫《登高》中道出诗人郁积心中的凄苦之情、难以排遣的国运之恨的句子是“________________,________________”。‎ 答案 (1)浊酒一杯家万里 燕然未勒归无计 (2)剑阁峥嵘而崔嵬 (3)艰难苦恨繁霜鬓 潦倒新停浊酒杯 ‎17.[2017·湖南六校联盟联考]补写出下列句子中的空缺部分。‎ ‎(1)辛弃疾《永遇乐·京口北固亭怀古》中,诗人认为此时的北伐没有做好准备,只会像刘义隆当年“______________”一样的结果。‎ ‎(2)庄子《逍遥游》中“______________,______________”两句,表明宋荣子看淡了世间荣辱,不因为外界评价而兴奋或颓丧。‎ ‎(3)《岳阳楼记》中“______________,______________”两句写月光与水波一起荡漾,闪烁着金光,月光的倒影沉浸在水底,宛如一块美玉。‎ 答案 (1)赢得仓皇北顾 (2)(且)举世誉之而不加劝 举世非之而不加沮 (3)浮光跃金 静影沉璧 ‎18.[2017·南昌联考]补写出下列名篇名句中的空缺部分。‎ ‎(1)屈原在《离骚》中以神仙借指楚怀王,抱怨楚王荒唐,不体恤民情的诗句是“______________,______________”。‎ ‎(2)李白《行路难》中有两句诗运用典故写出了诗人有为于当世的理想,其中运用伊尹的典故的诗句是“______________”。‎ ‎(3)韩愈在《师说》中将巫医乐师百工之人与士大夫之族从师的态度进行对比后提出“______________,______________”的质疑来批评不愿从师的士大夫。‎ 答案 (1)怨灵修之浩荡兮 终不察夫民心 (2)忽复乘舟梦日边 (3)今其智乃反不能及 其可怪也欤 ‎19.[2017·石家庄质检]补写出下列名篇中的空缺部分。‎ ‎(1)在《赤壁赋》中,苏轼看到月下美景,于是纵酒放歌,其中写船桨之美的句子是“______________”。‎ ‎(2)“文章憎命达”是杜甫对李白的评价,而他自己又何曾不是这样呢?在《登高》一诗中,“______________,______________”这两句诗从时空两方面着笔写出了他的悲愁和孤独。‎ ‎(3)韩愈在《师说》中,用简洁生动的语言,凝练地概括出“士大夫之族”不愿从师的荒谬心态的两句是:“______________,______________。”‎ 答案 (1)桂棹兮兰桨 (2)万里悲秋常作客 百年多病独登台 (3)位卑则足羞 官盛则近谀 ‎20.[2017·哈尔滨三中模拟]补写出下列句子中的空缺部分。‎ ‎(1)屈原的《离骚》中“______________”一句表现了诗人因选择道路时没有看清楚的悔恨之情。‎ ‎(2)庄子在《逍遥游》中表达“风聚集的力量不雄厚,它承载巨大的翅膀便力量不够”的意思的句子是“______________,______________”。‎ ‎(3)孟子在《生于忧患,死于安乐》一文中,以“______________,______________”两句深刻地揭示出饱经磨难对于一个人成才的重要。‎ 答案 (1)悔相道之不察兮 (2)风之积也不厚 则其负大翼也无力 (3)所以动心忍性 曾益其所不能 滚动提升训练(四)‎ ‎  时间:80分钟   满分:108分 一、语言文字运用(20分)‎ ‎1.[2017·安徽模拟]下列各句中,加点的成语使用不恰当的两项是(3分)(  )‎ A.每天清晨取蜂蜜一至两勺,用温水冲服,空腹饮用,长此以往不仅能滋阴润肺,而且可排毒养颜。‎ B.法国大昆虫学家法布尔,把科学和文学巧妙地结合起来,用富有诗意的笔触给人们描绘了一个绚丽多姿、光怪陆离的昆虫世界。‎ C.没有强大的创新设计、生产制造能力,国家实力的提升就无从谈起,民族复兴的宏伟蓝图也只能是空中楼阁。‎ D.雍正在位13年,竟写下了约4000万字的朱批,就连现在的专栏作家、网络写手也难以望其项背。‎ E.在《群英会蒋干中计》中,罗贯中运用生动细致的动作、神态描写,为我们塑造了一个胸无城府却又自作聪明、十分迂腐可笑的蒋干形象。‎ 答案 AE 解析 A项,长此以往:指长久这样下去,多指不好的情况。B项,光怪陆离:形容现象奇异,色彩繁杂。C项,空中楼阁:指悬于半空之中的城市楼台,也比喻虚构的事物或不现实的理论、方案等。D项,望其项背:能够望见别人的颈项和脊背,表示赶得上或比得上,多用于否定句中。E项,胸无城府:形容待人接物坦率真诚,褒义词。‎ ‎2.[2017·洛阳一中模拟]依次填入文段空白处的词语,最恰当的一组是(3分)(  )‎ ‎5月8日,福建泰宁“大型泥石流”,制造了一场极其可怕的灾难,__①__会让我们经历苦痛,__②__我们的希望不会沉沦,__③__会升腾。__④__从灾难发生那一刻起,我们都在为那些灾难中的生命祈祷,__⑤__这场灾难产生如何可怕的破坏力,我们也都有理由相信,随着我们所有人的力量向同一个方向聚集,爱心与力量__⑥__能够在最快的时间内传递到灾区。‎ ‎①‎ ‎②‎ ‎③‎ ‎④‎ ‎⑤‎ ‎⑥‎ A 尽管 可是 即使 也 不管 都 B 虽然 但是 反而 因为 无论 就 C 尽管 可是 反而 也 不管 就 D 虽然 但是 即使 也 无论 都 答案 B 解析 ①②两句应是转折关系,③是反向递进,④表原因,⑤⑥是条件关系。‎ ‎3.[2017·江西模拟]下面的句子中,没有语病的一项是(3分)(  )‎ A.第五届“创新作文大赛”颁奖典礼,是设在颇具文艺范的市图书馆举行的,现场揭晓了“构思之星”“最佳团队”等获奖名单。‎ B.坚定不移地以利用和保护环境为基础的循环经济之路,切实提高人民的生活水平,全面建成小康社会,是可持续发展的最终目标。‎ C.今年春晚表现了“你我中国梦,全面建小康”的主题,尤其是弘扬传统美德的歌舞小品集中体现了社会主义核心价值观。‎ D.互联网应建设成“护联网”,一方面要联合起来删除不健康的网页,另一方面要培养孩子正确使用、识别、传播网络信息的能力。‎ 答案 C 解析 本题考查辨析病句的能力。A项,结构混乱,“设在……图书馆”和“在……图书馆举行”两种句式杂糅,可删去“设”或者“举行”;B项,成分残缺,第一个分句缺少谓语动词,可在“坚定不移地”后加上“走”;D项,语序不当,应改为“识别、使用、传播”。‎ ‎4.[2017·江淮十校联考]在下面一段文字横线处补写恰当的语句,使整段文字语意完整连贯,内容贴切,逻辑严密,每处不超过15个字。(5分)‎ 人类只有一个地球,__①__,国际社会日益成为一个你中有我、我中有你的命运共同体。__②__,反对冷战思维,倡导树立双赢、多赢、共赢的新理念,呼吁摒弃你输我赢、赢者通吃的旧思维,彰显的正是中国智慧、中国价值。中国坚持维护国际公平正义,不觊觎他国权益,不嫉妒他国发展,__③__,绝不会拿自己的核心利益做交易,坚守的正是中国立场、中国态度。‎ 答:①______________________________________________‎ ‎②_____________________________________________________‎ ‎③_____________________________________________________‎ 答案 ①各国共处一个世界 ②中国倡导人类命运共同体意识 ③也绝不放弃我们的正当权益 解析 ①处应该将“一个地球”和“国际社会”关联在一起;②处主语是“中国”,横线后是“反对”,结合后文中的“倡导”与“呼吁摒弃”可知,此空应该填“中国的立场是怎样的”一类内容;③处表达的应是“对我国利益的坚守”一类的内容。‎ ‎5.[2017·东莞模拟]下面是某单位“迎春联谊”活动的初步构思框架,请把这个构思写成一段话,要求内容完整,表述准确,语言连贯,不超过85个字。(6分)‎ 答:____________________________________________________‎ ‎____________________________________________________________‎ 答案 本次“迎春联谊”活动计划分别在室内和室外进行。室内活动首先由领导致辞,然后自由选择参加棋牌、品茗或乒羽活动。室外活动是在农庄垂钓和采摘蔬果,晚上在烧烤场开篝火晚会。(内容完整给2分;表述准确给2分;语言连贯给2分。如有其他答案,只要符合要求,可酌情给分;不完整的酌情扣分)‎ 解析 本题是一道表文转换题。首先要分清“迎春联谊”活动由“室内”和“室外”两项内容组成;其次再分别表述每项活动的具体内容。‎ 二、古诗文阅读(88分)‎ ‎(一)[2017·福州模拟]阅读下面的文言文,完成6~9题。(19分)‎ 王猛,琅邪临沂人。五岁而父清遇害。陈文帝军度浙江,访之,将加夷灭。母韦氏携之遁于会稽,遂免。及长,勤学不倦,博涉经史,兼习孙、吴兵法。以父遇酷,终陈文帝之世不听音乐,蔬食布衣,以丧礼自处。宣帝立,乃始求位。‎ 太建初年,释褐鄱阳王府中兵参军,再迁永阳王府录事参军。诏随大都督吴明彻略地,以军功封应阳县子,累迁太子右卫率,徙晋陵太守。威惠兼举,奸盗屏迹,富商野次,云“以付王府君”。郡人歌之。‎ 时孔范、施文庆等相与比周,害其梗直,议将出之而未有便。会广州刺史马靖不受征,乃除王猛都督东衡州刺史,领始兴内史,与广州刺史陈方庆共取靖。猛至,即禽马靖送建邺,进爵为公,加先胜将军、平越中郎将、大都督,发广、桂等二十州兵讨岭外荒梗,所至皆平。‎ 祯明二年,诏授镇南大将军、都督二十四州诸军事,寻命徙镇广州。未之镇,而隋师济江,猛总督所部赴援。时广州刺史临汝侯陈方庆、西衡州刺史衡阳王陈伯信并隶猛督府,各观望不至。猛使高州刺史戴智烈、清远太守曾季远各以轻兵就斩之而发其兵。及闻台城不守,乃举哀素服,藉稿不食。因勒兵缘江拒守,以固诚节。‎ 及审陈后主不死,乃遣其部将辛昉驰驿赴京师归款。隋文帝大悦,谓昉曰:“猛怀其旧主,送故情深,即是我之诚臣。保守一方,不劳兵甲,又是我之功臣。”即日拜辛昉开府仪同三司,仍诏猛与行军总管韦洸便留岭表经略。‎ 猛母妻子先留建邺,因随后主入京,诏赐宅及什物甚厚,别赉物一千段,及遣玺书劳猛。仍讨平山越,驰驿奏闻。时文帝幸河东,会猛使至,大悦。杨素贺因曰昔汉武此地闻喜用改县名王猛今者告捷远符前事于是又降玺书褒赏以其长子王缮为开府仪同三司。猛寻卒于广州,隋文帝闻而痛之。遣使吊祭,赠上开府仪同三司,封归仁县公。命其子王缮袭,仍授普州刺史。葬关中,谥曰成。‎ ‎(节选自《南史·王猛传》)‎ ‎6.对下列句子中加点的词的解释,不正确的一项是(3分)(  )‎ A.乃除王猛都督东衡州刺史  除:免去 B.因勒兵缘江拒守 缘:沿着 C.猛母妻子先留建邺 妻子:妻子儿女 D.别赉物一千段 赉:赏赐 答案 A 解析 除:任命。‎ ‎7.下列对文中画波浪线部分的断句,正确的一项是(3分)(  )‎ A.杨素贺/因曰/昔汉武此地闻喜/用改县名/王猛今者告捷/远符前事/于是又降玺书褒赏/‎ B.杨素贺因曰/昔汉武此地闻喜/用改县名/王猛今者告/捷远符前事/于是又降玺书褒赏/‎ C.杨素贺/因曰/昔汉武此地闻喜用/改县名/王猛今者告捷/远符前事/于是又降玺书褒赏/‎ D.杨素贺因曰/昔汉武此地闻喜用/改县名/王猛今者告/捷远符前事/于是又降玺书褒赏 答案 A 解析 结合句意,“闻喜”即听到喜讯,应停顿,排除B、C。“告捷”不可拆分,排除B、D,故答案为A。‎ ‎8.下列对原文有关内容的概括和分析,不正确的一项是(3分)(  )‎ A.王猛历经凶险,勤学自律。他五岁时父亲遇害,幸亏母亲韦氏带他出逃才避免了被诛杀,他勤奋学习,因父亲遭遇凄苦终陈之世不肯享乐。‎ B.王猛功勋显赫,政绩卓著。他不仅凭借军功被封为应阳县子,后又屡次升迁,开疆拓土,而且在任晋陵太守时,恩威并施,使富商遍地,百姓人人歌颂。‎ C.王猛审时度势,竭尽忠诚。他曾斩杀怀有异心的将领,在台城失守后穿上孝服不肯进食;也曾派部将向文帝表达忠君之意,深得文帝信赖,文帝几次下发玺书奖赏他。‎ D.王猛去世,尽享哀荣。王猛去世后,文帝感到很悲痛,派人吊祭,并赠谥,他的儿子也受到父荫。‎ 答案 A 解析 原文表述是“终陈文帝之世不听音乐”。‎ ‎9.把文中画横线的句子翻译成现代汉语。(10分)‎ ‎(1)时孔范、施文庆等相与比周,害其梗直,议将出之而未有便。(5分)‎ 译文:_________________________________________________‎ ‎(2)及审陈后主不死,乃遣其部将辛昉驰驿赴京师归款。(5分)‎ 译文:_________________________________________________‎ 答案 (1)当时孔范、施文庆等人交互结党营私,陷害梗直的人,商议将要把王猛调出而没有合适的机会。(比周、梗直、出之各1分,句意2分)‎ ‎(2)等到弄清楚了陈后主没有死,(王猛)就派他的部将辛昉驾乘驿马疾行到京城投诚。(审、驰驿、归款各1分,句意2分)‎ 解析 关键词有:比周,结党营私;梗直,梗直的人;出之,把他调出。审,弄清楚;驰驿,驾乘驿马疾行;归款,投诚。‎ 参考译文:‎ 王猛,是琅琊临沂人。五岁的时候父亲王清遇害。陈文帝的军队渡过浙江,访察他们,准备加以诛灭。他的母亲韦氏带着他逃到了会稽,于是得以免祸。长大以后,勤学不倦,博览经史,兼学孙子、吴起兵法。因为父亲的遭遇太苦,他在整个文帝时期始终不听音乐,吃蔬菜粗粮,穿粗布衣裳,以丧礼自处。宣帝即位,才开始求官。‎ 太建初年,初入官场担任鄱阳王府中兵参军,再调任永阳王府录事参军。诏令他跟随大都督吴明彻去夺取土地,因为军功封为应阳县子,逐步升迁为太子右卫率,调任晋陵太守。威恩兼用,奸人盗贼匿迹,富商遍地,人们都说“应该归功于王府君”。郡中的人们歌颂他。‎ 当时孔范、施文庆等人交互结党营私,陷害梗直的人,商议将要把王猛调出而没有合适的机会。正巧广州刺史马靖不服从征调,于是便任命王猛为都督东衡州刺史,兼始兴内史,与广州刺史陈方庆共同攻取马靖。王猛到后,马上就捉住了马靖送交建邺,晋升爵位为公,加官先胜将军、平越中郎将、大都督,调发广、桂等二十州兵讨伐岭外蛮荒地区,所到之处都予以平定。‎ 祯明二年,下诏任命他为镇南大将军、都督二十四州诸军事,不久改镇广州。还没到任,而隋朝的军队渡过长江,王猛总督所辖部队赶赴增援。当时广州刺史临汝侯陈方庆、西衡州刺史衡阳王陈伯信都隶属于王猛都督府,各自观望不到。王猛派高州刺史戴智烈、清远太守曾季远各带轻兵前去杀死了他们而调发了他们的军队。等听到台城失守,便举哀穿上孝服,坐在草垫子上待罪不食。于是率兵,沿长江拒守,以固守忠诚的节操。‎ 等到弄清楚了陈后主没有死,(王猛)就派他的部将辛昉驾乘驿马疾行到京城投诚。隋文帝非常高兴,对辛昉说:“王猛怀念他的旧主,送别故人情义深厚,就是我的忠诚大臣。保守一方,不用耗费兵力,又是我的功臣。”当天任命辛昉开府仪同三司,仍然诏令王猛与行军总管韦洸就留在岭南经管。‎ 王猛的母亲妻子儿女先是留在建邺,然后随着后主入京,诏令赐给宅院和各种物品很多,另外赠予布帛一千段,并且派人带着诏书慰劳王猛。他讨平了山越,通过驿马上奏朝廷。当时文帝视察河东,正好王猛的使者到来,十分高兴。杨素庆贺,于是说:“从前汉武帝在这里听到喜讯,因而改了县名,王猛现在告捷,和从前的事远远相合。”于是又下发诏书奖赏,封他的长子王缮为开府仪同三司。王猛不久死在广州,文帝听到消息非常痛心,派遣使者前往吊祭,追赠开府仪同三司,封为归仁县公。让他的儿子王缮承袭爵位,仍然授给他普州刺史。王猛埋葬在关中,谥号为成。‎ ‎(二)[2017·成都外国语学校月考]阅读下面这首诗,完成10~11题。(11分)‎ 岁暮归南山①‎ 孟浩然 北阙②休上书,南山归敝庐。‎ 不才明主弃,多病故人疏。‎ 白发催年老,青阳③逼岁除。‎ 永怀愁不寐,松月夜窗虚。‎ ‎[注] ①该诗写于唐玄宗开元十六年,四十岁的孟浩然来长安考进士落第之后。②北阙:皇宫北面的门楼,汉代尚书奏事和群臣谒见都在北阙,后因用作朝廷的别称。③青阳:指春日。‎ ‎10.下列对这首诗的分析和鉴赏,不正确的两项是(5分)(  )‎ A.首联字面上说“北阙休上书”,表达自己归隐田园,不再“上书”的坚决意志。‎ B.颔联中“不才”既是谦词,又兼含了有才不被人识、良骥未遇伯乐的感慨。‎ C.本诗颔联写出因为自己生病,故友疏远、明主见弃的凄凉之景。‎ D.尾联“催”“逼”二字,形象地表达出诗人不遇明主白发早生的愤慨和激愤。‎ E.此诗系诗人归隐之作,在辗转反复、心绪极端复杂的情况下表达了一腔怨悱之情。‎ 答案 AD 解析 A项,“表达自己归隐田园,不再上书的坚决意志”错误,实际上表达的正是“魏阙心常在,金门诏不忘”的情意。“南山归敝庐”本非所愿,不得已也。D项,“表达出诗人不遇明主白发早生的愤慨和激愤”错误,是表现诗人不愿以白衣终老此生而又无可奈何的复杂感情。‎ ‎11.高步瀛在《唐宋诗举要》中评价《岁暮归南山》一诗时说:“结句意境深妙。”请结合诗句简要分析。(6分)‎ 答:____________________________________________________‎ ‎____________________________________________________________‎ 答案 寒冷的冬夜,一轮冷月透过松树,将清冷的光辉照进窗内,给诗人以清虚空寂之感。(紧扣“意境”,描述意境,)这句诗以景结情,语意双关,(指出手法,)既是对眼前清冷、寂寥的实景的客观描写,也是诗人求仕不得后内心愁苦与无望的真实写照,含蓄委婉,极有韵致。‎ 解析 也正是由于诗人陷入了不可排解的苦闷之中,才使他“永怀愁不寐”,写出了思绪萦绕,焦虑难堪之情态。“松月夜窗虚”,更是匠心独运,它把前面的意思放开,却正衬出了怨愤的难解。看似写景,实是抒情:一则补充了上句中的“不寐”,再则情景浑一,余味无穷,那迷蒙空寂的夜景,与内心落寞惆怅的心绪是何等相似!“虚”‎ 字更是语涉双关,把院落的空虚,静夜的空虚,仕途的空虚,心绪的空虚,包容无余。紧扣“深妙”,分析意境的含意与作用即可。‎ ‎(三)[2017·福州八县市联考]阅读下面这首唐诗,完成12~13题。(11分)‎ 蜀先主庙 刘禹锡 天下英雄气,千秋尚凛然。‎ 势分三足鼎,业复五铢钱①。‎ 得相能开国,生儿不象贤②。‎ 凄凉蜀故妓,来舞魏宫前。‎ ‎[注] ①五铢钱:汉武帝时货币为王莽废止,光武帝时恢复。②象贤:学习先祖的贤才。‎ ‎12.下列对本诗的赏析不恰当的两项是(5分)(  )‎ A.这首诗是凭吊古迹的咏史诗,主要目的在于讥贬刘禅。诗的首联写刘备在世时是叱咤风云的英雄,有力地衬托了后文刘禅的昏庸不才。‎ B.颔联咏功业,对仗难度大,却自成巧思。上句化用孙楚《为石仲容与孙皓书》中“自谓三分鼎足之势,可与泰山共相终始”表现刘备的英雄气概;下句化用“汉末童谣 黄牛白腹,五铢当复 ”表现刘备的雄心壮志。‎ C.颈联说人事,转接之间,富于变化。这一联嘲讽刘备长于择相,短于教子,从而导致嗣子不肖,后业不继。两句词意互相抗衡、声情顿挫美妙。‎ D.尾联感叹后主不肖。刘禅降魏后,被迁到洛阳;一次司马昭赐宴,命原蜀宫歌伎现场歌舞,而刘禅对此喜笑自若。尾联化用此意,表现刘禅不惜先业。字里行间,渗透着作者对于刘备身后事业消亡的无限嗟叹之情。‎ E.全诗前一半写盛德,后一半写业衰,在盛衰对比中,道出一个古今兴亡的教训。诗人咏古抒怀,借此警策开元盛世之后昏庸无能的唐朝统治者。‎ 答案 AC 解析 A项,主要在于称颂刘备;C项,无“嘲讽”之意。颈联意用刘备的长于任贤择相,与他的短于教子致使嗣子不肖相对比,叹惜刘备功业不能卒成。‎ ‎13.这首诗首联写得“突兀劲挺”,请仔细揣摩分析,说说其有何妙处。(6分)‎ 答:____________________________________________________‎ ‎____________________________________________________________‎ 答案 首联妙处:①境界雄阔绝伦。“天下”“千秋”等字眼写出了“英雄气”至大无垠,万古长存。这样遣词用语显示诗人吞吐日月、俯仰古今的胸臆。②用典不着痕迹。“天下英雄”四字暗用曹操对刘备说过的“今天下英雄,唯使君与操耳”,刘诗仅添一“气”字。③意在言外,旨意深远。“尚凛然”写出千秋之后先主之庙堂尚且威势逼人,凸显其当年叱咤风云的英雄气,寄托了诗人的敬仰之情。(意思对即可)‎ 解析 首联“天地英雄气,千秋尚凛然”,高亢入云,突兀挺拔。细品诗意,其妙有三:一、境界雄阔奇绝。“天地”两字囊括宇宙,极言“英雄气”之充塞六合,至大无垠;“千秋”两字贯串古今,极写“英雄气”之万古长存,永垂不朽。遣词结言,又显示出诗人吞吐日月、俯仰古今之胸臆。二、使事无迹。“天地英雄”四字暗用曹操对刘备语:“今天下英雄,惟使君与操耳”(《三国志·蜀志·先主传》)。刘禹锡仅添一“气”字,便有庙堂气象,所以纪昀说:“起二句确是先主庙,妙似不用事者。”三、意在言外。“尚凛然”三字虽然只是抒写一种感受,但诗人面对先主塑像,肃然起敬的神态隐然可见;其中“尚”字用得极妙,先主庙堂尚且威势逼人,则其生前叱咤风云的英雄气概,自不待言了。‎ ‎(四)[2017·广西桂林中学模拟]阅读下面这首小令,完成14~15题。(11分)‎ 驻马听·吹 白朴 裂石穿云,玉管宜横清更洁。霜天沙漠,鹧鸪风里欲偏斜。凤凰台上暮云遮,梅花惊作黄昏雪。人静也,一声吹落江楼月。‎ ‎14.下面对作品的理解或赏析,不恰当的两项是(5分)(  )‎ A.“驻马听”是这首小令的题目,作者白朴与关汉卿等被誉为“元曲四大家”。‎ B.作者描绘了戍边将士吹箫抒怀的情景和自己听闻乐声时产生的丰富想象。‎ C.起句别致,先以比喻描绘其声,再言其声缘于何物,作品基调奇特、浓烈。‎ D.以“霜天沙漠,鹧鸪风里欲偏斜”比喻乐声的意境及其使闻者动情的魅力。‎ E.“一声吹落江楼月”,以夸张手法引出想象的世界,曲终而意韵不绝,妙笔!‎ 答案 AB 解析 A项,“驻马听”是这首小令的曲牌。B项说作者听闻“戍边将士吹箫抒怀”,没有根据;“吹箫”亦不妥,作品中写道“玉管宜横清更洁”,可见描绘的是笛子的吹奏效果。‎ ‎15.为什么说“梅花惊作黄昏雪”形象地显现了乐声的艺术魅力?(6分)‎ 答:____________________________________________________‎ ‎____________________________________________________________‎ 答案 满树梅花竟然闻笛声而惊落,飘飘洒洒如黄昏时的雪花。笛声使梅花有了人的情感,确乎有非同一般的魅力。(言之成理即可。)‎ 解析 “梅花惊作黄昏雪”,作者运用通感的手法,借助想象和比喻,立体地再现了悠扬清雅的笛曲。这里还有另外一层意思,古曲“梅花落”是感伤离别的曲子,作者听到这样美丽的笛声,这样凄哀的曲子,不由自已想起家乡,想起自己的亲人,勾起离别之痛,这里含蓄地表达了作者的思乡之情,在着力夸张渲染衬托吹笛人的高超技术和笛声优美的同时,将感情更深入了一个层次,令这首词让人读来更为感动,达到了感情和艺术上的统一。‎ ‎(五)[2017·福建师大附中质检]阅读下面的宋词,完成16~17题。(11分)‎ 喜迁莺·晋师胜淝上①‎ 李纲②‎ 长江千里,限南北,雪浪云涛无际。天险难逾,人谋克壮,索虏岂能吞噬!阿坚百万南牧,倏忽长驱吾地。破强敌,在谢公处画,从容颐指。‎ 奇伟!淝水上,八千戈甲,结阵当蛇豕。鞭弭周旋,旌旗麾动,坐却北军风靡。夜闻数声鸣鹤,尽道王师将至。延晋祚,庇烝民,周雅③何曾专美!‎ ‎[注] ①晋师胜淝上:指中国历史上著名的以少胜多的“淝水之战”。前秦的苻坚以百万之师犯晋,东晋谢安指挥八千军士在淝水抗击,并大获全胜。②李纲:北宋末、南宋初的抗金名臣。③周雅:指《诗经》中赞颂周宣王战功的诗篇。‎ ‎16.下列对本词的理解,不正确的两项是(5分)(  )‎ A.本词开篇写景,描写长江白浪涛涛,奔腾千里,是阻隔南北的天然界线。‎ B.“结阵当蛇豕”中的“蛇豕”意思是毒蛇和野猪,在这里比喻凶残的敌人。‎ C.“破强敌”三句写谢安指挥若定,从容不迫,对部下颐指气使,十分严厉。‎ D.“夜闻数声鸣鹤”中的“鹤”所表达的意境与“黄鹤一去不复返,白云千载空悠悠”(唐·崔颢《黄鹤楼》)的“鹤”相同。‎ E.“延晋祚,庇烝民”写东晋在淝水取得胜利,使国运延续,人民得到庇护。‎ 答案 CD 解析 C项“对部下颐指气使,十分严厉”有误,“颐指”在这里是形容谢安神情淡定,从容指挥的样子。D项,意境不同。‎ ‎17.前人评这首词:“于宋廷有良史之鉴也。”请结合全词,简要分析作者写“淝水之战”有何用意。(6分)‎ 答:____________________________________________________‎ ‎____________________________________________________________‎ 答案 作者写“淝水之战”是为了借古讽今。词中写东晋凭借长江天险的地理优势和任用有才能的将士谋划作战,最终以少胜多,成功击退了敌人的侵略,借此劝谕南宋统治者要鼓足信心,因为南宋有同样的地理优势,也不乏抗金的人才,只要借鉴历史经验,坚决抗击金兵,就一定能以弱胜强,抵御外辱。‎ 解析 ‎ 本题考查古代诗歌对比鉴赏以及评价诗歌的思想内容和作者的观点态度的能力。作者主要篇幅描述了淝水之战晋胜秦败的过程及其值得借鉴的历史意义:强大的敌人并不可怕,是可以被打败的。他写这首词,意在讽谕高宗以古为鉴,须知少可以胜多,弱可以胜强,强敌不足畏,全在“人谋克壮”。应痛下决心,北伐中原,收复失地,作者的用心是很明显的。‎ ‎(六)名篇名句默写(25分)‎ ‎18.[2017·湖南五市十校联考]补写出下列名篇名句中的空缺部分。(5分)‎ ‎(1)成语“沧海一粟”由苏轼《赤壁赋》中的“______________,______________”演化而来。‎ ‎(2)《阿房宫赋》中,“______________,______________”两句运用倒置式的暗喻,以璀璨晶亮的明星来比喻纷纷打开的妆镜,既贴切又形象。‎ ‎(3)诸葛亮在《出师表》中,劝谏后主,也是劝谏后人不要看轻自己的句子是:“______________。”‎ 答案 (1)寄蜉蝣于天地 渺沧海之一粟 (2)明星荧荧 开妆镜也 (3)不宜妄自菲薄 ‎19.[2017·太原调研]补写出下列句子中的空缺部分。(5分)‎ ‎(1)李商隐的《锦瑟》中,“______________”一句化用庄子的典故来表现诗人沉迷在美好的境界之中。‎ ‎(2)岑参《白雪歌送武判官归京》中表现送别时诗人依依不舍之情的诗句是“______________,______________”。‎ ‎(3)诸葛亮的《出师表》中,“______________,______________”两句写出了在国家危难之时大臣们的具体做法。‎ 答案 (1)庄生晓梦迷蝴蝶 (2)山回路转不见君 雪上空留马行处 (3)(然)侍卫之臣不懈于内 忠志之士忘身于外 ‎20.[2017·山西五校联考]补写出下列句子中的空缺部分。(5分)‎ ‎(1)《闻王昌龄左迁龙标遥有此寄》一诗中融情入景,借明月表达飘零之感、离别之恨的诗句是“______________,______________”。‎ ‎(2)苏轼《赤壁赋》中“______________,______________”紧承“余音袅袅,不绝如缕”,把洞箫的哀音表现得形象真切。‎ ‎(3)李煜的《虞美人》中“______________”这一问句包含了宇宙的永恒和人生的短暂无常之意,表达了作者的满腔愁绪。‎ 答案 (1)我寄愁心与明月 随君直到夜郎西 (2)舞幽壑之潜蛟 泣孤舟之嫠妇 (3)春花秋月何时了 ‎21.[2017·四川巴蜀黄金大联考]补写出下列句子中的空缺部分。(5分)‎ ‎(1)屈原《离骚》中,作者以“蛾眉”比拟自己的美德,以“众女”肆意造谣中伤比拟朝臣对自己的造谣中伤的两句是“______________,______________”。‎ ‎(2)李商隐《锦瑟》中,“______________,______________”两句,从“追忆”中醒来,知道那梦已经远去,并且当时就不甚分明,流露出无可奈何的情怀。‎ ‎(3)苏轼《赤壁赋》中,客人以眼前的山川形式发问“______________”,紧接着,追述了曹操破荆州、迫使刘琮投降的往事。‎ 答案 (1)众女嫉余之蛾眉兮 谣诼谓余以善淫 (2)此情可待成追忆 只是当时已惘然 (3)此非孟德之困于周郎者乎 ‎22.[2017·曲靖一中检测]补写出下列句子中的空缺部分。(5分)‎ ‎(1)李煜《虞美人》中,运用想象手法,抒发物是人非的感慨的句子是:______________,______________。‎ ‎(2)范仲淹在《渔家傲·秋思》中,表达人们不甘无功而返的句子是:______________。‎ ‎(3)李商隐《锦瑟》中,化用典故,表示美好愿望终如烟云、可望而不可即的诗句是“______________,______________”。‎ 答案 (1)雕栏玉砌应犹在 只是朱颜改 (2)燕然未勒归无计 (3)沧海月明珠有泪 蓝田日暖玉生烟 第三部分 论述类文本阅读 考点十五 理解文中重要概念和句子的含意 考点名片 考点内容 与文章的核心内容密切相关的词语,表达功能特别强烈的词语,理解上容易发生偏差的词语的含义;能点明主旨的语句或能显示脉络层次的关键性语句,在文中起重要作用的中心句、过渡句、总结句,内涵较为丰富而且具有提示性或引导性的语句,比较含蓄的有深层含义的语句,结构比较复杂、对理解文意有直接影响的语句。‎ 考查形式 ‎①全国卷、天津卷等为单选题;②其他省份试卷另有填空题、简答题。‎ 趋势分析 ‎①题型稳定,全国卷、天津卷等仍将采用单选题形式;②部分自主命题试卷仍会综合采用单选题、填空题、简答题的形式。‎ 一、阅读下面的文字,完成文后1~2题。‎ 大数据时代 ‎“大数据时代”的说法并不新鲜,早在2010年,美国数据科学家维克托·迈尔·舍恩伯格在《大数据时代》一书中就系统地提出,以前,一旦完成了收集数据的目的之后,数据就会被认为已经没有用处了。比如,在飞机降落之后,票价数据就没有用了;一个网络检索命令完成之后,这项指令也已进入过去时。但如今,数据已经成为一种商业资本,可以创造新的经济利益。‎ 数据能够成为一种资本,与移动互联网有密切关系。随着智能手机、平板电脑等移动数码产品的“白菜化”,WIFI信号覆盖的无孔不入,越来越多的人不再有“在线时间”和“不在线时间”之分,只要他们愿意,便可几乎24小时一刻不停地挂在线上;在线交易、在线支付、在线注册等网络服务的普及固然方便了用户,却也让人们更加依赖网络,依赖五花八门的网上平台。‎ 大数据时代的科技进步,让人们身上更多看似平常的东西成为“移动数据库”,如带有存储芯片的第二代银行卡、信用卡,带有芯片读取功能的新型护照、驾驶证、社保卡、图书证,等等。在一些发达国家,官方为了信息录入方便,还不断将多种“移动数据库”的功能组合成一体。‎ 数字化时代使得信息搜集、归纳和分析变得越来越方便,传统的随机抽样被“所有数据的汇拢”所取代,基于随机抽样而变得重要的一些属性,如抽样的精确性、逻辑思辨和推理判断能力,就变得不那么重要,尽可能汇集所有数据,并根据这些数据得出趋势和结论才至为关键。简单说,以往的思维决断模式是基于“为什么”,而在“大数据时代”,则已可直接根据“是什么”来下结论,由于这样的结论剔除了个人情绪、心理动机、抽样精确性等因素的干扰,因此,将更精确,更有预见性。‎ 不过,一些学者指出,由于“大数据”理论过于依靠数据的汇集,那么一旦数据本身有问题,在“只问有什么,不问为什么”的模式下,就很可能出现“灾难性大数据”,即因为数据本身的问题,而做出错误的预测和决策。‎ 斯坦福大学专家特来沃尔·哈斯蒂也指出,“大数据”的理论是“在稻草堆里找一根针”,而面临的问题是“所有稻草看上去都挺像那根针”。而乔治·梅森大学专家瑞贝克·高尔丁则提出“数据提供者造假”的危险,在“大数据时代”变得更有害,因为“大数据”理论建立在“海量数据都是事实”的基础上,但人们无法控制数据提供者和搜集者本人的偏见和筛选。‎ 由于“大数据”炙手可热,数据的流失、泄露和私下买卖也成为噩梦,全球各地不时发生的个人信息被盗,可被看作“大数据时代”对个人生活的伤害,而“维基泄密事件”则提醒强力部门和各国政府,“大数据”的魔力同样会对强者构成威胁。‎ ‎(《国际先导报》2015年7月5日,有删节)‎ ‎1.下列选项中,不属于“危险大数据”内容的一项是(  )‎ A.“所有数据的汇拢”使随机抽样中的精确性、逻辑思辨和推理判断能力变得不重要。‎ B.如果过于依靠数据的汇集,一旦数据本身有问题,就会可能导致“灾难性大数据”出现。‎ C.“大数据”理论需要海量数据都是以事实作基础,但人们无法控制数据都是真实的。‎ D.“大数据”一旦流失泄露和私下买卖,不但对个人生活,甚至对各国政府都会带来伤害。‎ 答案 A 解析 ‎ 一般论述类文章中的重要概念主要是一些术语,其内涵多数是临时的、具体的、不具备普适性;其对概念的内涵揭示,也不一定是下定义的标准语言形式。概念的外延,是指具有概念所反映的特有属性的全部事物,即概念确指的对象的范围。对概念的外延考查,通常是对某事物所包含的对象的判定,或是分别对若干对象做出是否包含在内作判定。善于把握概念的适用范围、使用对象、感情色彩、轻重程度等,多侧面、多角度、多层次地理解。设题一般为客观选择,考查科学术语或说明对象的内容特征。A项是从“大数据能够使抽样更精确,更有预见性”来表述的,而不是“危险大数据”的概念。‎ ‎2.根据原文提供的信息,下列对“大数据时代”的阐述,不符合原意的一项是(  )‎ A.带有存储芯片的第二代银行卡、信用卡、新型护照、驾驶证、社保卡、图书证等固然给你带来方便,但数据的流失、泄露和被人私下买卖,也可能成为你的噩梦。‎ B.“移动互联网”提供了数据传输的网络服务,赋予了“大数据”的商业资本价值。‎ C.“灾难性大数据”的出现,是因为收集者不负责任地“只问有什么,不问为什么”。‎ D.“大数据时代”,根据“是什么”来下的结论剔除了个人情绪、心理动机等因素的干扰,因此,与传统的随机抽样相比更精确,更有预见性。‎ 答案 C 解析 理解文中重要概念的含义一般答题模式是先找出考查的概念所在的具体语段及语句;然后运用层次分析的方法,找准概念所涉及内容的位置和范围;再根据语境,理解概念的含义,并整合相关信息来作答。“灾难性大数据”的出现是因为数据本身有问题,更可能是提供者先造假。‎ 二、阅读下面的文字,完成文后题目。‎ 有人会说,幸福这个东西很难说,好像是很主观的感觉,很难有统一的标准。确实是这样,每个人对幸福的理解是不一样的。但是,你若深入地问为什么会不一样,其实还是有标准的。一个人对幸福的理解,从大的方面来说,其实是体现了价值观的,就是你究竟看重什么。‎ 古希腊哲学家亚里士多德曾经说过:幸福是我们一切行为的终极目标,我们做所有的事情其实都是手段。一个人想要赚钱赚得多一点,这本身并不是目的,他是为了因此可以过上幸福的生活。有人可能就要反驳了:我不要那么多钱,也可以幸福。比如说我读几本好书,就会感到很幸福。其实对后一种人来说,读书就是他获得幸福的手段。‎ 对于什么是幸福,西方哲学史上主要有两种看法、两个派别。一派叫作“快乐主义”,其创始人是古希腊哲学家伊壁鸠鲁。近代以来,英国的一些哲学家,如亚当·斯密、约翰·穆勒、休谟对此也有所阐发。这一派认为,幸福就是快乐。但什么是快乐?快乐就是身体的无痛苦和灵魂的无烦恼。身体健康、灵魂安宁就是快乐,就是幸福。他们还特别强调一点,人要从长远来看快乐,要理智地去寻求快乐。你不能为了追求一时的、眼前的快乐,而给自己埋下一个痛苦的祸根,结果得到的可能是更大的痛苦。另一派叫作“完善主义”。完善主义认为,幸福就是精神上的完善,或者说道德上的完善。他们认为人身上最高贵的部分,是人的灵魂,是人的精神。你要把这部分满足了,那才是真正的幸福。这一派的代表人物是苏格拉底、康德、黑格尔等,包括马克思,他们强调的是人的精神满足。‎ 这两派有一个共同之处,那就是,都十分强调精神上的满足。如伊壁鸠鲁强调,物质欲望的满足本身不是快乐,物质欲望和生命本身的需要是两码事。生命需要得到满足那是一种快乐,但是超出生命需要的那些欲望反而是造成痛苦的根源。约翰·穆勒则强调,幸福就是快乐,但是快乐是有质量和层次的区别的。一个人只有各种快乐都品尝过了,他才知道哪一种快乐更深刻、更持久、更强烈、更美好。‎ 在中国哲学里,我感觉,道家比较接近“快乐主义”,尤其是庄子强调生命本身的快乐,还强调精神自由的快乐,与天地精神相往来的快乐。儒家比较接近“完善主义”,儒家认为人生的理想境界、最高的享受就是道德上的完善。‎ 也有哲学家认为,幸福是根本不可能的。最典型的就是德国哲学家叔本华。他说人是受欲望支配的,欲望就意味着匮乏,你缺什么往往就对什么有欲望,而匮乏意味着痛苦。所以,欲望没有满足的时候你是痛苦的,但是欲望满足以后,人是不是就快乐了呢?非也。欲望满足以后是无聊。叔本华说,人生就像钟摆一样,在痛苦和无聊之间摇摆,幸福是不可能的。‎ 如果我们仅仅从满足身体的、物质的欲望层面来理解的话,幸福确实是不可能的。但是如果我们超越欲望层面来看幸福,这个观点就不成立了。比如你非常爱读书,你渴望去读那些好书,你知道一些好书在等着你读,那个时候你会痛苦吗?你不会。读完了以后你会无聊吗?不会。你感到丰富了自己的精神,你会因此快乐。这就进一步说明,我们谈幸福问题,一定要超越纯粹欲望的层面,要从价值观角度去谈。‎ ‎(摘编自周国平《幸福的哲学》)‎ 下列有关“幸福”的表述,不符合原文意思的一项是(  )‎ A.有人说,幸福好像是很主观的个人感觉。每个人对幸福的理解不尽相同,对幸福的认识也就很难有统一的标准。‎ B.按照古希腊哲学家亚里士多德的观点,幸福是一切行为的终极目标,我们做各种事情其实都是获得幸福的手段。‎ C.亚当·斯密、约翰·穆勒一派认为幸福是身体无痛苦和灵魂无烦恼,而黑格尔等人乃至马克思的主张则与之相反。‎ D.西方“完善主义”认为,幸福就是精神上的完善,人们在满足自身灵魂、精神的需求后才能感受到真正的幸福。‎ 答案 C 解析 本题属于概念理解题。C项错误原因是无中生有,“黑格尔等人乃至马克思的主张则与之相反”错误。由原文第四段首句“这两派有一个共同之处,那就是,都十分强调精神上的满足”可见此选项错误。‎ 三、阅读下面的文字,完成文后题目。‎ 社交网络让我们更近了吗 于洋 渴望交流是人的本能,交流的形式在不同的时代有不同的特征。过去的交流是鱼腹鸿雁,千里传信;现代的交流是书信电话,片语真心;网络时代的交流是面对电脑一遍遍地刷屏,等待对方的回复。‎ 网络丰富了人们的交流形态,极大地满足了人们对交流和沟通的渴望。社交网络的出现让网络的功能有了更大的扩展。每个人都可以以自己为中心,建立一个庞大的社交群,在动辄以万计数的好友和“粉丝”中,我们以为可以最大程度地对抗孤单、抱团取暖。‎ 全球最大的社交网站Facebook注册用户数接近9亿;我国当前最大的社交网站人人网今年注册用户将超过2亿;截至去年年底,我国的微博用户有2.5亿。一个庞大的社交网络正在慢慢张开,越来越多的人被粘到这张大网中。‎ 按照“六度空间”理论,我们最多通过6个人,便可以认识世界上任何一个陌生人。在社交网络里,你可以找到你多年不见的老友,可能被一个你喜欢的明星关注;只要你愿意,你甚至可以和一国总统成为“好友”。社交网络这种联络的作用让很多人觉得异常温暖,人类沟通的愿望也让社交网站承担了更多情感的使命。‎ 然而,看似熟络的社交网络,却经常让人感受到刺骨的寒冷。一个网名叫“走饭”的女孩在发表一条微博后结束了自己的生命。几天后,人们知道她的名字叫马洁,是一个“90后”的南京高校学生。翻看她的微博,你会发现她一直在网上“求救”,只是没有人听到。‎ 我们不能把一个年轻生命的终结归罪于网络,但不得不怀疑,网络到底有没有让我们的关系变得亲密,我们对网络的情感期待是不是太高?‎ 在网络上,我们有很多好友,却没有几个见过面的;加入了很多圈子,却没有几个真正有兴趣的;每天都在线上碰到,却没有几个会打招呼的。社交网络给了我们联系,却未必给我们交流;拉近了我们的距离,却未必增加我们的亲密;激发了我们社交的天性,却可能磨平了我们沟通的能力。‎ 牛津大学进化人类学教授罗宾·邓巴有一个有趣的“邓巴数”理论,即人的大脑新皮层大小有限,提供的认知能力只能使一个人维持与大约150人的稳定人际关系。也就是说,无论你在社交网络上有多少好友和“粉丝”,你最多能频繁交流的也就150人左右。‎ 幸福感来自社交的质量而不是数量,来自于沟通的深度而不是频率。技术让人际关系变得越来越扁平和肤浅。我们简化了社交的流程,不再需要各种繁文缛节。但这种简化的结果是,我们只希望让人们看到自己最好的一面、最幸福的瞬间,人们在网络上把自己包装成一个快乐天使,变成一个隐藏自己真实情感的虚拟人。‎ 提高人类的幸福感是技术最大的责任和价值,我们应该正确对待新技术带来的便利,在使用社交网络沟通时也关注真实世界的交流,正确驾驭技术带来的变革,而不是让技术编织的网络掌控生活。‎ 网络连接不等于人际交往,沟通便利不代表关系的亲密,技术发展不意味着思想的进步。纸张比键盘更有热度,现实生活比虚拟社区更加阳光,真实的交流比数字的流动更温暖人心。技术发展的重要性不言而喻,但人与人之间真诚和真实的交流有更重要的意义。‎ 下列关于“社交网络”的表述,正确的一项是(  )‎ A.社交网络进一步丰富了人们的交流形式,极大地满足了人们对交流和沟通的渴望。‎ B.全球最大的社交网站Facebook注册用户接近9个亿;我国当前最大的社交网站人人网的注册用户也超过2.5亿。‎ C.在社交网络里,我们必定找得到多年不见的老友,也会被自己喜欢的某个明星关注;只要愿意,我们甚至可以和国家领导人成为“好友”。‎ D.社交网络给了我们联系,却不能给我们交流;拉近了我们的距离,却不能增加我们的亲密;激发了我们社交的天性,却磨平了我们沟通的能力。‎ 答案 A 解析 本题考查对文字概念及句子的理解能力。B项,张冠李戴,原文第三段表述为“我国当前最大的社交网站人人网今年注册用户将超过2亿;截至去年年底,我国的微博用户有2.5亿”。C项,说法绝对,原文第四段表述为“你可以找到你多年不见的老友,可能被一个你喜欢的明星关注”。D项,说法绝对,第七段为“……未必给我们交流……未必增加我们的亲密……可能磨平了我们沟通的能力”。‎ 四、阅读下面的文字,完成后面问题。‎ 骚扰电话该谁管?‎ 最近,网上流行这样一句话:“每天叫醒我的,不是闹钟,也不是梦想,而是骚扰电话。”虽有戏谑与夸张成分,但未尝不是人们深受骚扰电话之害的真实写照。‎ 理财推销、发票开具、中奖兑换、房屋租售、辅导培训,骚扰电话五花八门,商业推广与电信诈骗混为一体,让人感觉“接不完,不胜烦”。‎ 骚扰电话到底有多少?今年3月,百度发布的《中国互联网安全白皮书》显示,2015年,全国骚扰电话总量为948亿条,较2014年上涨57%。这么大的数量,即使设套诈骗的概率有限,最后被骗用户的绝对数也不少。最不可思议的是,很多人手机上显示,一些被安全软件标注了上千次甚至上万次的骚扰电话还能继续拨打,岂非怪事?‎ 客观地看,骚扰电话的界定确实不容易。以常见的骚扰号码400+电话为例,除非含有色情、暴力、淫秽等明显违法内容,否则很难认定哪些电话是骚扰、哪些内容属诈骗。更有人认为,电话通信是公众权利,在不能判定某个号码涉嫌违法的情况下,运营商没有权力擅自停止服务。这种似是而非的“付费就能使用论”,好像有一定道理,但听起来不免让人心塞。对每年数以几百亿计的骚扰电话,难道只能听之任之?‎ 据知情人透露,目前骚扰电话分工精细,已经形成了完整的产业链条。一些所谓高科技软件公司开发出的系统,既能批量外呼,还能语音自动群呼,更能随意更改主叫号码,为骚扰电话推波助澜;有的地方电信运营商为了追求利润,滥用透传技术,出售通道帮不法分子建立呼叫中心,提供交换机等设备支持,更助长了骚扰电话的气焰。有媒体报道,在这条黑色利益链背后,每通话1分钟,运营商能获得五六分钱的收益。‎ 打击骚扰电话,离不开电信运营商的主动作为。对个别地方运营商参与骚扰电话产业链牟利问题,各大运营商总部应严格行业规范,定期开展网络安全普查,清除内部害群之马。对授权出售常见骚扰电话号段的代理商家,切实担负起监督责任,在电信号源、号码发放、资质审核、投诉处理等环节都不能放松。在甄别骚扰电话的技术上,积极与相关互联网公司开展合作,不断改造升级屏蔽软件,让相关软件更有效。‎ 打击骚扰电话,需要创新监管方式,加大处罚力度。在这方面,一些国家的做法值得借鉴。比如,美国联邦贸易委员会开设专门网站,为电信用户申请“别打我电话”服务,把那些申请保护的电话编入“全国别打电话名录”,推销公司如果拨打列入该名录的电话,将被处以1.1万美元的罚款。德国法律规定,乱打骚扰电话者,将根据情节,除被处以数额不等的罚款外,还将面临最长3年的监禁。‎ 这几年,为打击骚扰电话,我国有关部门先后多次开展“清网行动”,查处了不少伪基站,取得了一定成效。但从根本上管住骚扰电话,还需要相关部门共同协作。电信运营商应当尽到管理责任,工信、工商、金融、公安等相关部门建立应急协调机制,提高骚扰电话的拨打成本,加大对违法分子的处罚力度,为饱受骚扰的电信用户讨回一个清静。‎ 下列关于“骚扰电话”的表述,不正确的一项是(  )‎ A.骚扰电话包括理财推销、发票开具、中奖兑换、房屋租售、辅导培训等电话,商业推广与电信诈骗混为一体。‎ B.因为骚扰电话每年巨大的拨打数量,即使设套诈骗的概率有限,最后被骗用户的绝对数也不少。‎ C.很多人手机上显示,一些被安全软件标注了上千次甚至上万次的骚扰电话还能继续拨打。‎ D.骚扰电话无法屏蔽或打击,每年有数以几百亿计的骚扰电话被拨打,人们只能听之任之。‎ 答案 D 解析 本题考查对文中句子的理解能力。D项原文第四段末尾为“对每年数以几百亿计的骚扰电话,难道只能听之任之?”,且文章后面有各国打击骚扰电话的行动和措施,并非完全“只能听之任之”。‎ 五、阅读下面的文字,完成问题。‎ 多通道和自主的审美是数码图形艺术审美中的一大特征。迈可尔·海姆把人们在虚拟数码图形艺术中的活动方式概括为模拟、远程展示、身体完全沉浸、身临其境、互动、人造性、网络化的交往等七个方面。我们可以从中得出两个结论:一是人们在虚拟视景中的活动是全方位、多通道的。这七个方面的活动几乎涉及了人类身体的所有部位。二是人们在虚拟视景中的活动是完全自主的,享有最高的审美活动权限。多通道和自主的审美方式成为虚拟视景艺术审美与传统艺术审美的主要区别,开拓了人类审美活动的新视野,必将促进人类的全面发展。‎ 第一,多通道参与的审美。这种崭新的交互方式大大拓展了审美的体验方式。人类的整个意识活动离不开身体的眼、耳、鼻、舌、身各个部位的分工合作,它们分别对应于视觉、听觉、嗅觉、味觉和触觉。眼视、耳听是人类几千年以来欣赏艺术的两大主要审美通道,如绘画以视觉为主导;音乐以听觉为主导;电影、电视或戏曲则为两者的综合。在虚拟现实艺术审美领域,全方位多通道的交互方式大大突破了这一局限。除了常规的眼、耳方式,鼻、舌、身都可以实现共同参与。虽然嗅觉、味觉和触觉只占人类从外界获得信息总量的5%左右,尤其是触觉,主要靠皮肤、肌肉和体内器官与外界直接或间接接触所产生的痛、冷或热等感觉的交互审美方式,在传统艺术的审美领域中几乎没有用武之地。在虚拟现实的艺术领域,触觉反而成为交互审美方式的主要通道,这势必大大丰富人类的审美感受。‎ 第二,数码图形艺术是一个可以完全自主的审美梦工厂。由于真实的个人生存从来就是建立在他的有限性上,真实的人际交往,从来也是建立在彼此的有限性上的。但是,人们对自身完善、发展的要求是无限的,这就形成了人的现实存在的有限性与人对自身发展要求的无限性之间的矛盾。人有各种主观愿望,如改善自身或自然状况,追求完美的理想人生境界等。但由于人类个体力量的局限,这些愿望始终无法实现或难以实现。于是产生了通过求助他人或借助其他方式达到实现理想的愿望。虚拟数码图形艺术往往是这种情形下所能求助的最佳方式。‎ 虚拟视景中的各种交互活动都是属于安全的冲突、无害的危险和不必担心的死亡,只要相关的虚拟数码图形艺术软件的设计条件和交互功能允许,体验者可以任何方式,任何目的,扮演任何角色,体验任何方式的生活。总之,人间的喜、怒、哀、乐,生、老、病、死都可以在这里集中而短暂地得到体验,并且找到了一种能够让自己打开心扉的方式,让人性中的黑暗与光明、善良与险恶、残忍与仁慈都在这里得到实时的剖析,在长期、不断的交互体验与审美欣赏过程中,身心、性情有可能因此而改变,其审美活动也在这一过程中不知不觉地得到了落实。这些都是其他的审美体验方式所无法达到的。观看电影、电视、戏曲或小说,欣赏者也有某种程度的角色“代入”感,但欣赏者意念中的代入角色只能被动地跟随情节的进展而进展,没有自主余地,更谈不上与视景中的其他角色互动(交互)。由此可见,虚拟数码图形艺术才是一个可以完全自主的真正审美“梦工厂”。‎ ‎(摘编自《美学研究》,有删改)‎ 下列不是围绕关于“虚拟视景艺术审美与传统艺术审美的区别”表述的一项是(  )‎ A.多通道和自主的审美方式成为虚拟视景艺术审美与传统艺术审美的主要区别,开拓了人类审美活动的新视野。‎ B.眼视、耳听是人类几千年以来欣赏艺术的两大主要审美通道,如绘画以视觉为主导;而音乐则以听觉为主导。‎ C.触觉在传统艺术的审美领域中几乎没有用武之地。但在虚拟现实的艺术领域,触觉却成为交互审美方式的主要通道。‎ D.观看电影、电视、戏曲或小说,欣赏者没有自主余地,而虚拟视景中的其他角色可以互动,实现完全自主的真正审美。‎ 答案 B 解析 考核概念的题目的命题点主要集中在概念的内涵、外延,主要的错误是范围不当、偷换概念、无中生有、强拉因果、曲解文意、答非所问等。命题的方式经常是对文中相关概念的要点进行组合,或对文中的句子进行转换,或是改变文中句子之间的关系。此题B项答非所问,B项只是讲到传统艺术的特点,没有提到虚拟视景艺术的特点。‎ 考点十六 筛选并整合文中的信息 考点名片 考点内容 客观题要求能对照材料选出符合题干要求的选项,主观题要求能筛选出与题目有关的语句,进行简要表述。‎ 考查形式 ‎①全国卷、天津卷等为单选题;②其他省份试卷另有填空题、简答题。‎ 趋势分析 ‎①题型稳定,全国卷、天津卷等仍将采用单选题形式;②部分自主命题试卷仍会综合采用单选题、填空题、简答题的形式。‎ 一、阅读下面的文字,完成后面1~2题。‎ 音乐成功的秘诀:越和谐越受欢迎 运气、努力、才华、机遇……加上悦耳的和弦、耐人寻味的旋律和恰到好处的时间,这些就是歌曲征服人类大脑的秘诀。‎ 西班牙《趣味》月刊2月号文章称,谱写一首成功、隽永且富有感染力的歌曲的基本公式是:使用“IVviIV”4个和弦,音乐的速度为每分钟120拍,采用二声部合唱,加入一两个不和谐的音符,选择四四拍节奏。‎ 文章称,有的音乐能够迅速登上排行榜,有的却在电台播送了几次就销声匿迹了,这其中并非仅靠运气好坏。从科学的角度来看,有些歌曲注定就会大红大紫。英国布里斯托尔大学的人工智能专家特尔·德比耶研发出一款能够预测一首歌曲能否成功的软件。‎ 德比耶的软件基于对最近一个世纪以来英国的各项音乐大奖名单和音乐排行榜榜单的详细分析。要想评估一首新歌是否有机会成功,要考虑到23项特征:有些比较显而易见,例如长度和速度;有些则比较复杂,例如主旋律和伴舞性。总之,德比耶的软件能够精确地评估一首新曲能否为大众所喜爱和接受。‎ 德比耶发现,人们对音乐的喜好在不断发展,并在研发软件时考虑到这一点。例如,在速度方面,20世纪80~90年代最受欢迎的是慢歌,但2000年之后快歌打了翻身仗。‎ 美国南加州大学的另一项研究显示,如果为主唱加上伴唱,那么歌曲登上音乐排行榜的机会将大大增加。在乐器方面,如果采用一两件乐器或者5件以上乐器,成功几率比较高。大多数歌曲都采用3至5件乐器,而打破乐器数量的常规,也能让歌曲脱颖而出。使用吉他、钢琴和电子合成器的乐曲效果并不好。反之,电子合成器和音色纯净的吉他的组成往往能够获得成功。‎ 文章称,为什么同样是由音符组成,有的歌曲就悦耳动听,有的歌曲就难以入耳?美国明尼苏达大学通过研究发现,这取决于所有音符在弹奏过程中是否具有和谐性。神经生物学家戴尔·珀维斯指出,人们之所以偏好和谐的音乐,是因为人类的语言就具有这种特性。从生物学角度出发,我们会受到类似于语言的音乐的吸引。‎ 人们可能都有过这样的经历:听到一首朗朗上口的歌曲之后,自己便不由自主地哼唱起来,甚至于一整天脑海中都回荡着这首歌,挥之不去。这种现象叫作“耳虫现象”。辛辛那提大学的詹姆斯·凯拉里斯表示,“耳虫现象”产生的原因是这段音乐持续刺激大脑皮层的听觉中枢,这种不由自主的音乐记忆超出了我们的控制范围。不过幸运的是,英国雷丁大学的研究显示,要想摆脱“耳虫”的骚扰,最简单的方法就是嚼口香糖。‎ 调查显示,超过75%的人依靠听音乐提振情绪,50%的人为了集中注意力而听音乐。在听音乐时,如果是一曲欢快的乐曲,听者的脸上往往会露出微笑;如果是一首悲伤的歌曲,听者也会感到心情沉重。但这并不意味着,我们要提振情绪的时候不能选择悲伤的音乐,因为美国俄亥俄州立大学的研究员戴维·休伦发现,虽然看似矛盾,但在很多时候,一首催人泪下但旋律优美的歌曲更能令人士气倍增。他通过研究发现,听者在听到忧伤的音乐时,血液中的催乳素水平提高,从而使紧张的情绪放松下来。‎ 文章称,美国斯坦福大学的一项神经成像研究显示,音乐能够刺激大脑皮层,进而引起人们的关注。广告业者似乎深谙其道,几乎所有的广告片都有背景音乐。美国北卡罗来纳州立大学的达维德·埃纳尔认为,描写雄心壮志、离情别绪、痛苦迷乱的音乐往往能够抓住消费者的心。‎ ‎1.下列关于原文内容的表述,不正确的一项是(  )‎ A.运气、努力、才华、机遇、悦耳的和弦、耐人寻味的旋律和恰到好处的时间等,这些就是歌曲征服人类大脑的秘诀。‎ B.从科学的角度来看,有的音乐能够迅速登上排行榜,有的却在电台播送了几次就销声匿迹了,这其中并非仅靠运气好坏,人们是能够预测一首歌曲能否成功的。‎ C.德比耶的软件基于对最近一个世纪以来英国的各项音乐大奖名单和音乐排行榜榜单的详细分析,能够精确地评估一首新曲能否为大众所喜爱和接受。‎ D.研究显示,如果为主唱加上伴唱,采用一两件乐器或者5件以上乐器,用电子合成器和音色纯的吉他组合,那么歌曲就能登上音乐排行榜。‎ 答案 D 解析 本题考查学生筛选信息和概括信息的能力。题干问的是“不正确的一项是”,选项D曲解文意。原文第六段首句是“那么歌曲登上音乐排行榜的机会将大大增加”,D项是“就能”,“必然”代替了“或然”。‎ ‎2.根据原文内容,下列理解和分析不正确的一项是(  )‎ A.调查显示,超过75%的人依靠听音乐提振情绪,50%的人为了集中注意力而听音乐。欢快的乐曲,往往让人露出微笑;悲伤的歌曲,听者也会感到心情沉重。‎ B.美国戴维·休伦发现,听者在听到忧伤的音乐时,血液中的催乳素水平提高,从而使紧张的情绪放松下来,因此,提振情绪的时候不一定不能选择悲伤的音乐。‎ C.美国斯坦福大学的一项神经成像研究显示,音乐能够刺激大脑皮层,进而引起人们的关注。广告业者似乎深谙其道,所以所有的广告片都有背景音乐。‎ D.美国北卡罗来纳州立大学的达维德·埃纳尔认为,描写雄心壮志、离情别绪、痛苦迷乱的音乐大多能够抓住消费者的心。‎ 答案 C 解析 本题考查学生筛选信息和概括信息的能力。C项中“所以所有的广告片都有背景音乐”扩大了范围。原文最后一段中的“广告业者似乎深谙其道,几乎所有的广告片都有背景音乐”用的是“几乎”。‎ 二、阅读下文,完成后面1~2题。‎ 警惕“生态癌症”——蓝藻 蓝藻是地球上最早出现的生物之一,亿万年来,蓝藻默默地为地球提供着氧气,是目前地球大气圈的主要缔造者之一,在地球生物多样性的形成过程中,起着关键的作用。蓝藻具有双重性,一个是光合作用,因为它带有叶绿素,从这个角度看它是一种植物;另外它比较微小,是一种细菌。所以它是植物性和细菌性的结合体。蓝藻虽然能进行光合作用,但它和植物的不同点更多,所以目前大部分学者还是主张将蓝藻从植物中分离出来,列入原核生物。蓝藻是生命力最旺盛的生物之一,它能存在于90 ℃高温的温泉中,也能存在于冰山里,它的细胞外有一层很厚的胶质,可以抵抗外界的冷热刺激。蓝藻家族成员众多,既有“好蓝藻”,也有“坏蓝藻”。好蓝藻如国家一级重点保护植物发菜,而著名的蛋白质补充剂螺旋藻,则是螺旋藻属的著名成员。“坏蓝藻”的典型是微囊藻属的蓝藻,正是它们,将河水与湖水染成蓝绿色的“水华”,并散发出阵阵恶臭。‎ 蓝藻爆发主要存在于静止的水体中,中国湖泊富营养化问题严重,生长蓝藻明显。一说起水体的富营养化,人们通常会立即联想到工业污染。工业污染大家比较容易发现,而农业污染则不太为公众熟悉,可也比较严重。此外,生活污染也加快了富营养化。蓝藻需要的营养物质比较多,一般水里的氮都比较多,其他营养也都够用,就是磷比较缺。一旦水体中出现大量的磷,蓝藻的爆发就自然而然了,所以磷是一种具有决定性意义的因素。其次,适宜的温度也是引起蓝藻大量繁殖的一个很重要因素。最常见的蓝藻在亚热带地区的25~35 ℃温度下繁殖较多。而今年的蓝藻大爆发同样也有着温度的促进,大气变暖是肯定的促因之一。‎ 蓝藻中的微囊藻、鱼腥藻及颤藻会产生微囊藻毒素。这些毒素通常存在于蓝藻的细胞膜里,所以蓝藻活着的时候没事,待其死亡腐败时,细胞膜破裂,藻毒素就释放到了水里。动物实验表明微囊藻毒素可能让小鼠致癌,但这一情况对人体的影响目前还不清楚。在目前已经检定出的微囊藻毒素中,微囊藻毒素-LR是最常见、毒性也最强的一种,因而世界卫生组织将微囊藻毒素-LR作为饮用水质量测定标准的项目之一。另外,蓝藻腐烂时会释放出异臭。但研究尚未发现它们对人体会产生什么实质性的伤害。‎ 有人说蓝藻是“生态癌症”,一旦患上就很难清除。对于蓝藻的治理,应急办法很多,效果却一般。一旦蓝藻爆发,打捞是应急处理的第一步,但打捞漂浮在水面的蓝藻只能产生眼不见为净的效果。大部分蓝藻存在于水中和水底淤泥里,打捞不上来。另一种应急方法是“物理吸纳法”,往水里添加黏土,可以让蓝藻吸附在黏土颗粒上,令其沉入水底。但这也没法治本,因为蓝藻还在水底生存,还会继续爆发。化学方法也可以应用于蓝藻去除工作,比如在水中投放杀藻剂或絮凝剂。不过,这些化学药剂可能对湖泊产生污染,因此对化学药剂有着很高的要求。此外,生态治理上更高级的办法还有利用微生物直接侵袭蓝藻细胞,这一方法非常有效,缺点是价格昂贵。‎ ‎1.下列表述完全符合原文意思的一项是(  )‎ A.蓝藻腐烂时释放出的异臭气味会不会对人体产生实质性的伤害,我们目前还不太清楚。‎ B.蓝藻会产生微囊藻毒素,蓝藻死亡腐败时,细胞膜破裂,藻毒素就释放到了水里。‎ C.微囊藻毒素可能让小鼠致癌,对人体健康也有害,世界卫生组织将微囊藻毒素-LR作为饮用水质量测定标准的项目之一。‎ D.25~35 ℃温度下各种蓝藻繁殖较多。今年我国各地的蓝藻大爆发与大气变暖关系密切。‎ 答案 A 解析 B项,原文说部分蓝藻会产生毒素,此处扩大了范围。C项,原文说微囊藻毒素“对人体的影响目前还不清楚”。D项,原文是“最常见的蓝藻”在亚热带地区的25 ℃到35 ℃下繁殖较多,而不是“各种蓝藻”。‎ ‎2.根据原文提供的信息,下列推断不正确的一项是(  )‎ A.磷对蓝藻具有决定性意义,因此我们如果能控制水体中磷的排入,就可以在一定程度上控制蓝藻的爆发。‎ B.只要我们严格控制工业废水排入湖泊,就一定能够控制住蓝藻灾害。‎ C.水体不流动加速了水体的富营养化,如果保持水体的流动,相信也可以缓解蓝藻成灾的局面。‎ D.蓝藻被称为“生态癌症”,但加强环境预警,严格控制污染源,对湖泊进行水源修复,消灭蓝藻灾害并不是不可能的。‎ 答案 B 解析 见原文第二段,工业污染只是造成湖泊富营养化的一个因素,此外还有农业污染和生活污染,只严格控制工业废水排入湖泊,还不能完全阻止湖泊富营养化,也就不一定能够控制住蓝藻灾害。‎ 三、阅读下面的文字,完成后面问题。‎ 人和其他动物所不同的是具有学习的能力。人的行为方式并不固执地受着不学而能的生理反应所支配。所谓“学”就是在出生之后以一套人为的行为方式作模型,把本能的那一套方式加以改造的过程。学的方法是“习”。“习”是指反复地做,靠时间中的磨炼,使一个人惯于一种新的做法。因之,学习必须打破个人今昔之隔。这是靠了我们人类的一种特别发达的能力,时间中的桥梁——记忆。在动物的学习过程中,我们也可以说它们有记忆,但是它们的“记忆”是在简单的生理水准上。一个小白老鼠在迷宫里学得了捷径,它所学得的是一套新的生理反应。和人的学习不相同的是,它们并不靠一套象征体系的。人固然有很多习惯,在本质上是和小白老鼠走迷宫一般的,但是他却时常多一个象征体系帮他的忙。所谓象征体系中最重要的是“词”。我们不断地在学习时说着话,把具体的情境抽象成一套能普遍应用的概念,概念必然是用词来表现的,于是我们靠着词,使我们从特殊走上普遍,在个别情境中搭下了桥梁;又使我们从当前走到今后,在片刻情境中搭下了桥梁。从这方面看去,一个动物和时间的接触,可以说是一条直线的,而人和时间的接触,却比一条直线来得复杂。他有能力闭了眼睛置身于“昔日”的情境中,人的“当前”中包含着从“过去”‎ 拔萃出来的投影,即时间的选择累积。‎ 一个依本能而活动的动物不会发生时间上阻隔的问题,它的寿命是一连串的“当前”。但是人却不然,人的“当前”是整个靠记忆所保留下来的“过去”的累积。如果记忆消失了,我们的“时间”就可以说是阻隔了。‎ 人有此能力是事实,人利用此能力,发展此能力,还是因为他“当前”的生活必须有着“过去”所传下来的办法。人的学习是向一套已有方式的学习,唯有学会了这套方式才能在人群中生活下去。这套方式并不是每个人个别的创制,而是社会的遗业。小白老鼠并不向别的老鼠学习,每只老鼠都得自己在具体情境里,从“试验错误”的过程中,得到个别的经验,它们并不能互相传递经验,互相学习。人靠了他的抽象能力和象征体系,不但累积了自己的经验,而且可以累积别人的经验。上边所谓那套传下来的办法,就是社会共同经验的累积,也就是我们常说的文化。文化是依赖象征体系和个人的记忆而维持着的社会共同经验。这样说来,每个人的“当前”,不但包括他个人“过去”的投影,而且是整个民族“过去”的投影。历史对于个人并不是点缀的饰物,而是实用的、不能或缺的生活基础。人不能离开社会生活,就不能不学习文化。文化得靠记忆,不能靠本能,所以人在记忆力上不能不力求发展。我们不但要在个人的今昔之间构筑桥梁,而且在社会的世代之间也得构筑桥梁,不然就没有了文化,也没有了我们现在所能享受的生活。‎ ‎(摘编自费孝通《乡土中国》)‎ 下列关于原文内容的表述,不正确的一项是(  )‎ A.人类的学习是依靠记忆能力,经过反复磨炼,改造自身那些不学而能的生理反应,适应新方法的过程,就是要打破个人的今昔之隔。‎ B.人类的学习过程和小白老鼠走迷宫本质上是一样的,凭借记忆打破今昔之隔,从而学得一套新的生理反应,找到新捷径,养成新习惯。‎ C.人类的学习有别于动物,依靠“词”的帮助,通过时间的选择累积,可以在个别的、片刻的情境中获得普遍的、对于未来的认知。‎ D.人类的学习就是累积自己的、别人的乃至社会共同的“过去”,沟通个人和社会的今昔,是“当前”生活不可或缺的基础。‎ 答案 B 解析 本题考查理解文意、筛选并整合文中的信息的能力。B项,“人类的学习过程和小白老鼠走迷宫本质上是一样的”说法有误,原文第一段中间是“人固然有很多习惯,在本质上是和小白老鼠走迷宫一般的”,选项的说法张冠李戴,与小白老鼠走迷宫本质上一样的是“人类的习惯”,并非是“人类的学习过程”。‎ 四、阅读下面的文字,完成问题。‎ 美国种族问题痼疾何以难消 只有充分的经济和社会权利保障,“人人生而平等”才能不再是写在纸面上的理念。‎ 近日,包括华人在内的美国亚裔在全美多个城市举行游行示威,抗议纽约市华裔前警员梁彼得因在执勤过程中误杀黑人而被定罪。‎ 暂且不论梁彼得案种种技术性细节,本案之所以在社会层面引起剧烈反应,根源在于美国深刻的种族矛盾。在多数抗议者看来,此前发生的警察枪击黑人案件中,频繁出现了涉案白人警官“全身而退”的情况,而梁彼得却被陪审团定为重罪,这说明相比白人警察,同属少数族裔的梁彼得成为全美反警察暴力、警察与非洲裔社区对立的牺牲品。《纽约时报》评论称,梁彼得被判罪名成立,“触动了在纽约亚裔群体中长期存在的一种不满情绪,他们将此次发生的事件视为一个无力反抗的边缘社群遭恶劣对待的又一个例子”。白宫就此案的表态也承认,在全美多地,不同肤色的种族和执法机关之间存在着深深的不信任。‎ 种族问题在美国是一个广泛存在的问题,当前梁彼得案的舆论发酵,只是美国社会这个痼疾的另一种显现。非洲裔、亚裔等美国少数族裔无论在经济上还是在政治上都属弱势群体,与美国主流社会存在着相当大的阶层差异,这是美国种族矛盾“一点就着”的根本原因。近年来,弗格森非洲裔青年布朗遭枪杀、纽约非洲裔男子被白人警察“扼喉”致死、巴尔的摩骚乱、得克萨斯聚会风波等一系列涉及种族问题的事件频频撕裂美国社会,全球舆论为之哗然。正如弗吉尼亚州前州长蒂姆·凯恩所指出,今日之美国社会虽然没有像内战时期那样决裂,“但每个天天看新闻的人都知道,美国仍然处于深深的社会分裂之中”。‎ 美国主流政治难以有效解决种族问题带来的种种严峻挑战,理应成为外界观察美国政治的一个重要方面,尽管美国政府总是乐于在国际舞台上戴副“人权捍卫者”的面具,但其自身在种族问题上的无所作为,却在这个面具上写满“虚伪”二字。今天的美国,一方面主流社会对社会构成变化的不适应感与忧虑感在加重;另一方面主流社会自身出现尖锐对立,自由派与保守派在基本价值观方面显现出裂痕。建立在主流精英基础之上的美国政治体制,深受这种忧虑感与分裂的制约,无力真正弥补社会裂痕,难以从政策层面切实缓解种族矛盾,政治正确成为政治回避的冷酷现象一再出现。就目前的报道来看,梁彼得案背后,政府廉租房楼梯间没有照明与惨剧发生直接相关。这样的细节理应得到更多重视。‎ 在纽约前市长迈克尔·布隆伯格看来,种族矛盾激发社会性事件,“反映出长期以来民众因贫困和缺乏经济流动性产生的失望”。任何一个社会的基本稳定不仅取决于开放包容的社会文化,更取决于对社会公平的有效保障。只有充分的经济和社会权利保障,“人人生而平等”才能不再是写在纸面上的理念,针对少数族裔的系统性歧视才可能得以消除。‎ ‎(《人民日报》2016年2月23日)‎ 对于“梁彼得案”,下列分析不正确的一项是(  )‎ A.梁彼得因在执勤过程中误杀黑人而被定罪,根据美国频繁出现的涉案白人警官“全身而退”的情况,多数抗议者认为梁彼得是不应该被定重罪的。‎ B.梁彼得被陪审团定为重罪,这说明相比白人警察,同属少数族裔的梁彼得成为全美反警察暴力、警察与非洲裔社区对立的牺牲品。‎ C.梁彼得案的舆论发酵反映了种族问题在美国是一个广泛存在的问题,原因在于少数族裔与主流社会存在相当大的阶层差异。‎ D.梁彼得案说明今日之美国社会虽然没有像内战时期那样决裂,“但每个天天看新闻的人都知道,美国仍然处于深深的社会分裂之中”。‎ 答案 D 解析 原文第四段指出,“当前梁彼得案的舆论发酵,只是美国社会这个痼疾的另一种显现”,它只是一个个例,好多事件都说明这个问题。D项是以偏概全。‎ 五、阅读下面的文字,完成后面题目。‎ 作为生活智慧在中国传统中的具体体现,“家本位”文化在传统社会中发挥着极为重要的作用。中国既没有西方那种严格意义上的宗教,也不用“上帝造人”解释人的本源,所以,对于“我是谁,我从哪里来,将要到哪里去”这类“终极关怀”的问题,中国人就以家族制度为基础的“家本位”文化来回应。林语堂曾指出:“使种族稳定的文化因素之一首先是中国的家族制度。这种制度有明确的定义和优良的组织系统,使得人们不可能忘记自己的宗系。这种不朽的社会组织形式,被中国人视为珍宝,比任何其他世俗的财产都宝贵,甚至含有一种宗教的意味。向祖先表示崇拜的各种礼仪,更加增强了它的宗教色彩。对这套东西的意识也已经深深地扎根在中国人的心灵之中。”‎ 就终极关怀而言,家族已起到应有的作用,但以往的中国家族研究告诉我们,家族还有保障族人生存和在族内扶贫济困的作用,以解决族人的生存问题。若有能力,它也会兴办家族的学堂,解决族人的受教育和发展问题。从加强凝聚力从而保障其发挥作为社会组织的作用来说,以族人间天然的血缘联系组织起来的家族显然是成本最低的,这使家族成为农耕社会的相对封闭的乡村社区中最基本的组织形式,也使家族成为族人在建立自己关系网以获取生存和发展的资源、构建生存和发展的保障时的首选。显然,作为一个属于“私”领域的社会组织,家族之所以“不朽”,是因为它能满足个人的根本需求,而且从中获得资源时成本相对较低。更重要的是,它提供了一个亲情的环境来满足个人对情感的需要,这是其他任何社会组织无法比拟的。‎ 自西周以“敬天法祖”理念创立宗法制度,将祭祀权与继承权结合在一起,创建了诸侯臣服天子的大一统帝国以来,家族经历了由贵族化向平民化的转变,成为中国城乡社会生活不可或缺的组织,特别是在明清两代,家族成为中国社会最基本的社会单位。但自清末以来,中国农村以“敬宗收族”为目的的宗族制受到强烈冲击。由于士绅阶层的衰落和劳动力的流动,家族及其所属家庭与中央政权的联系被削弱,从而增加了当时中国社会结构的不稳固性。精英向大城市的流动削弱了家族的基础,使家族不能承担原有的社会功能。另外,国家政权建设的推进和越来越多的在农村地区立足的新型社会组织也在不断削弱原本属于家族的权势,最终可以看到的是,中国农村以“敬宗收族”为目的的宗族制全面走向衰落。1949年,中国共产党领导的新民主主义革命的胜利导致以“敬宗收族”‎ 为目的的家族与宗法制度又一次发生重大变化。不过,我们在调查中发现,家族观念与家族活动并没有随着这样巨大的社会与政治变迁而简单消失,而是以其自身的种种特质卷入新的社会与政治形态。家族或宗族的深厚基础在于,作为一种社会组织,它形成于以血缘为基础的先赋的社会关系。20世纪80年代,中国农村中的政治意识形态开始淡化,基层政权在农村的组织作用有所削弱,在不少地方的农村中出现家族组织公开恢复活动或重建的现象。这是因为,以血缘关系为主的家族关系网络在中国社会一直存在。‎ ‎(摘编自王善华、孙飞宇《“社会底蕴”:田野经验与思考》)‎ 下列关于原文内容的表述,不正确的一项是(  )‎ A.中国传统社会,儒教、道教都不是严格意义上的宗教,回应“我是谁,我从哪里来,将要到哪里去”这类“终极关怀”问题的是“家本位”文化。‎ B.有明确的定义和优秀的组织系统的中国家族制度是一种不朽的社会组织形式,被中国人视为宝贵的财富。‎ C.向祖先表示崇拜的各种礼仪,给“家本位”文化涂上了宗教色彩,从而使其含有一种宗教的意味。‎ D.家族既可以保障族人的生存,又能在一定程度上解决族人的发展问题。‎ 答案 C 解析 本题考查筛选并整合文中的信息。C项是对原文第一段后面“这种不朽的社会组织形式……甚至含有一种宗教的意味。向祖先表示崇拜的各种礼仪,更加增强了它的宗教色彩”的误解。‎ 考点十七 分析文章结构、归纳内容要点、概括中心思想 考点名片 考点内容 把现代文阅读中的整体内容或其中某一部分内容用几句简明的话扼要地表达出来。‎ 考查形式 ‎①全国卷、天津卷等为单选题;②其他省份试卷另有填空题、简答题。‎ 趋势分析 ‎①题型稳定,全国卷、天津卷等仍将采用单选题形式;②部分自主命题试卷仍会综合采用单选题、填空题、简答题的形式。‎ 一、阅读下面的文字,完成后面问题。‎ 当“中式教育”遇上英国土壤 黄培昭 英国广播公司有关中国老师在英国教学经历的纪录片本周播放了最后一部分,那些接受“中式教育”的英国学生最终在各个科目中得到很好的成绩。不过,这期间引发的各种争议值得人们再度思考。‎ 中国老师在英国教学过程中遇到的许多尴尬,实际上是在两个文明和两套社会体系坐标下,不同教育理念、思想、模式等发生碰撞后产生的结果。英式教育,大体可称为“快乐教育”,其推崇的观念在于:教育的艺术,是唤醒孩童天生好奇心并在未来满足它的艺术;而好奇心本身的鲜活及益处,与内心的满足及快乐成正比。笔者曾到英国的一些中小学采访,还参加过家长会,发现学生们笑意盈盈的时候居多,少见眉头紧锁、满脸严肃。‎ 中国乃至东亚的教育,非常重视和强调扎实的基础知识,讲究学生的“勤学苦练”。最常说的就是,“天将降大任于斯人也,必先苦其心志,劳其筋骨……”学校都在这一原则指导下施教,因而基础教育水平极高。学生们吸收了丰富的基本知识,掌握了应试技能,这都是英国学生所缺乏的,英国学生的数学等基础学科水平普遍较差,甚至要低于中国一两个年级。在英国,时常听闻有些学生遇到考试神经紧张,甚至还需要考前心理舒压。看来,快乐也是有代价的。‎ 不能否认的是,貌似以“玩乐”为主的英式教育,对培养学生思考能力、创新能力、实践能力以及想象力等大有裨益,这也对学生进入高等教育后进行学习、科研潜力的爆发进行了铺垫。笔者发现,英国中小学更注重艺术、文学等课程的设置,而不认为苦练加减乘除等计算技巧很有需求。支撑这一观念的人认为,在计算机广为普及的今天,已没必要再让人脑来做最基本的运算活动。相反,艺术、文学能够启迪人的心智,激发学生们的潜能,唤醒他们心中的审美等,有利于学生的长远发展,使他们进入大学后,在学习能力与职业竞争力等方面,能够后来居上。‎ 打个不恰当的比喻,英国教育就是“放养”模式。他们认为若在“圈养”的框架下,学生们中规中矩,束手束脚,虽然营养丰富,但不得不面对未来“单飞”时能力匮乏的窘境。不过,“放养”也是有规矩的。英国教育对学生综合素质的要求并不低:社会实践,各种集训,访学实习,野外生存等,不一而足。这些活动使得学生们从小就接触实践,实现了与社会的对接。‎ 当然,正像“鞋子合不合脚,自己穿了才知道”,中英教育发端于各自国土,符合本国国情社情。双方教育的着力点和着重点有差异,并不存在教育本身孰优孰劣的问题。如果英国一味照搬中国的教育模式,即便如这次取得了好成绩,但“水土不服”就会引发纪录片中校长那样的抵触情绪;如果把英国教育模式移植到中国,显然也根本行不通。看来,相互借鉴、有机融合,彼此取长补短,逐步填平文化和教育理念的鸿沟,才是应取态度。乐于学习而不“夜郎自大”,不盲目妄议、指摘和贬损对方,才是教育的应有之道。‎ ‎(摘自2015年8月21日《人民日报》)‎ 下列理解和分析,不符合原文意思的一项是(  )‎ A.英式教育是“快乐教育”,英国的一些中小学学生少有眉头紧锁、神经紧张的,总是笑意盈盈的。‎ B.中国的教育,很是重视和强调扎实的基础知识,基础教育水平很高,中国学生在基本知识和应试技能的把握上要优于英国学生。‎ C.好奇心本身的鲜活及益处与内心的满足及快乐是成正比的,英式教育较为注重对孩童天生好奇心的唤醒。‎ D.英式教育绝不是完全式“放养”,它在社会实践、各种集训、访学实习、野外生存等活动方面有着相当高的要求。‎ 答案 A 解析 本题考查归纳内容要点,概括中心意思的能力。文章第三段末尾说“在英国,时常听闻有些学生遇到考试神经紧张,甚至还需要考前心理舒压”,可见并非“总是笑意盈盈的”。曲解文意。‎ 二、阅读下面的文字,完成后面问题。‎ ‎《周易》借助于具体的形象符号,启发人们把握事物的抽象意义,崇尚一种观物取象、立象尽意的思维方式。《周易·系辞传》:“夫象,圣人有以见天下之赜,而拟诸其形容,像其物宜,是故谓之象。”卦象是《周易》的骨骼,舍象则无《易》。借助卦象,并通过象的规范化的流动、联结、转换,具象地、直观地反映所思考的客观对象的运动、联系,并借助六十四卦系统模型,推断天地人物之间的变化,这种思想方式渗透到中医和中国古代科技之中。道家庄子主张“得鱼而忘筌”,“得意而忘言”(《庄子·外物》),魏晋玄学家王弼提出“得意在忘象,得象在忘言”(《周易明象》)的命题,表明了中国思维穿透语言,领略语言背后之象,进而穿透形象而领略其背后之意蕴的特点。‎ 中国儒释道三家都主张直觉地把握宇宙人生之根据和全体。儒家的道德直觉、道家的艺术直觉、佛家的宗教直觉,都把主客体当下冥合的高峰体验推到极致。中国哲学认为,对于宇宙本体,不能依靠语言、概念、逻辑推理、认知方法,而只能靠直觉、顿悟加以把握。‎ 道家认为,心灵的虚寂状态最容易引发直觉思维。因此,人们要尽可能地摆脱欲望、烦恼的困扰,保持心境的平和、宁静。而要使直觉思维真实呈现,则离不开默思冥想的“玄览”。老子主张“涤除玄览”。“涤除”即否定、排开杂念,“玄览”即深入静观。这是在高度精神修养的前提下才具备的一种思维状态。庄子主张“心斋”“坐忘”。“心斋”即保持心境的虚静纯一,以便直接与道契合。“坐忘”即心灵空寂到极点,忘却了自然、社会,甚至忘却了自己的肉身和智慧,物我两忘,浑然冥同大化之境。‎ 儒家孔子的“默而识之”,孟子的“不虑而知”“不学而能”的良知良能,荀子的“虚壹而静”“大清明”,张载的“大其心则能体天下之物”,朱熹的“豁然贯通焉”“众物之表里精粗无不到,吾心之全体大用无不明”,陆九渊的“吾心”与“宇宙”的冥契,王阳明的“致良知”,都是扬弃知觉思虑,直接用身心体验宇宙终极的实在,达到对道德本体之契合的一种境界或方法。‎ 佛家更是强调一种精神性的自得和内心的体验,彻见心性之本源。禅宗的参究方法是不立文字,教外别传,直心而行,无念为宗,触类是道,即事而真。不执着外物,种种言行纯任心性之自然。禅宗的顿悟成佛,排除语言文字、逻辑思维工具,主体直接契入客体(人生的本性或宇宙的实相),与客体融合为一。这种思维活动的过程与结果是只可意会而不能言传的,有赖于每个人自己体悟,别人只能暗示、启发,而不能替代。‎ 超越逻辑,祛除言诠,止息思维,扫除概念,排斥分析、推理诸理性思维活动,精神内敛,默然返照,当下消解了主客、能所、内外、物我的界限,浑然与天道合一。这是一种思维状态,即“众里寻他千百度,蓦然回首,那人却在灯火阑珊处”,当下得到了对于生活和生命,对于自然世界和精神世界之最深邃的本质的一种整体的、综合的洞悉。但这种状态实际上是在多次反复的理性思维的基础上产生的,没有理性思维的铺垫,这种灵感或悟性就不可能出现。‎ ‎(选自张岱年、方克立主编《中国文化概论》)‎ 下列理解和分析,符合原文意思的一项是(  )‎ A.中国哲学认为,要把握宇宙本体,应当以直觉、顿悟为主,以语言、概念、逻辑推理、认知方法为辅。‎ B.道家代表人物老子主张“涤除玄览”,庄子主张“心斋”“坐忘”,他们提倡排除杂念,保持心灵的虚寂。‎ C.禅宗参悟修行的过程,直心而行,纯任心性之自然,这是只可意会而不可言传的,他人无从启发和替代。‎ D.儒学大家张载、陆九渊主张抛弃直觉思虑,而用身心体验宇宙终极的实在,达到身心与道德本体的契合。‎ 答案 B 解析 依据文章思路,找到选项在原文的位置,顺藤摸瓜,确定筛选范围。仔细对照,正确判断。找准选项有关内容在原文的位置后,要把原文与选项对照,辨明正误。A项,原文第二段末句为“不能依靠语言、概念、逻辑推理、认知方法”;C项,原文倒数第二段末尾为“别人只能暗示、启发,而不能替代”;D项,原文第四段后面为“扬弃知觉思虑”。故选B。‎ 三、阅读下面的文字,完成问题。‎ 陶渊明自耕自食的田园生活虽然远离了尘世恶浊,却也要承担肢体的病衰、人生的艰辛。田园破败了,他日趋穷困,唯一珍贵的财富就是理想的权利。于是,他写下了《桃花源记》。田园是“此岸理想”,桃花源是“彼岸理想”。终点在彼岸,一个可望而不可即的终点,因此也可以不把它当作终点。‎ ‎《桃花源记》用娓娓动听的讲述,从时间和空间两度上把理想蓝图与现实生活清晰地隔离开来。这种隔离,初一看是艺术手法,实际上是哲理设计。‎ 就时间论,桃花源中人的祖先为“避秦时乱”而躲进这里,其实也就躲开了世俗年代。“不知有汉,无论魏晋。”时间在这里停止了,历史在这里消失了,这在外人看来是一种可笑的落伍和背时,但刚想笑,表情就会凝冻。人们反躬自问:这里的人们生活得那么怡然自得,外面的改朝换代、纷扰岁月,究竟有多少真正的意义?于是,应该受到嘲笑的不再是桃花源中人,而是时间和历史的外部形式。这种嘲笑,对人们习惯于依附着历史寻找意义的惰性,颠覆得惊心动魄。‎ 就空间论,桃花源更是与人们所熟悉的茫茫尘世切割得非常彻底。这种切割,并没有借用危崖险谷、铁闸石门,而是通过另外三种方式。第一种方式是美丑切割。这是一个因美丽而独立的空间,在进入之前就已经是岸边数百步的桃花林,没有杂树,“芳草鲜美,落英缤纷”,那位渔人是惊异于这段美景才渐次深入的。这就是说,即便在门口,它已经与世俗空间在美丑对比上“势不两立”。第二种方式是和乱世切割。这是一个凭着祥和安适而独立的空间,独立于乱世争逐之外。和平的景象极其平常又极其诱人:良田、美池、桑竹、阡陌、鸡犬相闻、黄发垂髫……这正是历尽离乱的人们心中的天堂。但一切离乱又总与功业有关,而所谓功业,大多是对玉阶、华盖、金杖、龙椅的争夺。人们即便是把这些耀眼的东西全都加在一起,又怎能及得上桃花源中的那些平常景象?因此,平常,反而有了超常的力度,成了人们最奢侈的盼望。很多人说,我们也过着很平常的生活呀。其实,即使是普通民众,也总是与试图摆脱平常状态的功利竞争有着千丝万缕的联系,因此都不是桃花源中人。桃花源之所以成为桃花源,就是在集体心理上不存在对外界的向往和窥探。外界,被这里的人们切除了。没有了外界,也就阻断了天下功利体系。这种自给自足的生态独立和精神独立,才是真正的空间独立。第三种方式可以说得拗口一点,叫“不可逆切割”。桃花源的独自美好,容不得异质介入。那位渔人的偶尔进入引动传播,而传播又必然导致异质介入。因此,陶渊明选择了一个更具有哲学深度的结局——桃花源永久地消失于被重新寻找的可能性之外。桃花源中人虽不知外界,却严防外界,在渔人离开前叮嘱“不足为外人道也”‎ ‎。渔人背叛了这个叮嘱,出来时一路留下标记,并且终于让执政的太守知道了。但结果是,太守派人跟着他循着标记寻找,全然迷路。更有趣的是,一个品行高尚的隐士闻讯后也来找,同样失败。陶渊明借此划出一条界限,桃花源并不是一般意义上的隐士天地,那些以名声、学识、姿态相标榜的“高人”,也不能触及它。这个“不可逆切割”,使桃花源表现出一种近似洁癖的冷然。陶渊明告诉一切过于实用主义的中国人,理想的蓝图是不可以随脚出入的。在信仰层面上,它永远在;在实用层面上,它不可逆。‎ ‎(节选自余秋雨《重峦叠嶂间的田园》)‎ 下列表述,不符合原文意思的一项是(  )‎ A.桃花源在集体心理上不存在对外界的向往和窥探,也因此阻断了天下功利体系,形成了自给自足的生态独立和精神独立。‎ B.桃花源是理想的蓝图,是真正意义上的隐士天地,只有真正的隐士才能找到它。‎ C.桃花源再也找不到了,《桃花源记》的这一结局具有更深的哲学意味:理想的蓝图是不可以随脚出入的。‎ D.桃花源中人不会向往和窥探外面的世界,他们不知外界,却严防外界。‎ 答案 B 解析 B项,“是真正意义上的隐士天地,只有真正的隐士才能找到它”说法错误,原文结尾表述是“桃花源并不是一般意义上的隐士天地,那些以名声、学识、姿态相标榜的 高人 ,也不能触及它”。‎ 四、阅读下面的文字,完成后面题目。‎ 微电影也可负载大主题 牛梦笛 普通观众一般将筷子兄弟的《老男孩》作为微电影的代表,这部当时还被称为“网络短片”的作品讲述了小人物的梦想故事,贴近普通年轻人的生活视角,既具备当下网络文化的一系列特征,也有着基本的电影品质和完整的人物故事链条。自《老男孩》之后,此类网络视频有了新的概念与定义,成为一种在互联网平台上传播能力较强的艺术作品。‎ 短,是微电影显著的特征。这个特征赋予其独特的叙事方式和灵活的展示手法,并使拍摄变得更为简易。近年来,中国微电影产业的总产值已达700亿元左右,具有艺术、商业价值的年产量达2万部。据预测,未来3年内,总产值将超过1000亿元。它的兴盛,开始牵动大众文化产品的发展态势,也折射出了自身在创作和推广上的问题。‎ 首先,不少打着“微电影”旗号的网络视频存在依靠软色情、软暴力来增加点击量的现象,内容低俗。其次,不少与微电影相关的创投和赛事活动流于形式,成为商业机构和一些地方的门面活动,很少产生具有影响力的作品和导演。还有一些批评者担忧,微电影正在成为一个无所不包的概念,进而消解微电影应有的艺术品质,让这一独特的影视形态在艺术上过早凋谢。‎ 一般来说,内容上的泥沙俱下,是一个新的艺术类型在初创时期难以避免的情况。在加强管理和引导的同时,我们必须认识到:“微”时代的影视产品,也可以利用其独特的叙事方式和创意,消化大的主题,在赢得市场的同时,承担起大众教化和艺术探索的功能。‎ 艺术性、公益性已经成为优秀微电影的重要标识。央视公益广告《筷子篇》用5分钟微电影的形式,以中国最常见的饮食工具“筷子”为切入点,以小见大,传递了感恩、明礼、睦邻、关爱等中华传统美德,表达出“教会感恩,学会分享”的含义,浓缩了全球华人最深邃的情感。影片虽短,但它所蕴含的意义深刻入心。微电影《再不爱就晚了》曾3天火爆网络获得了500万的点击量,这个呼吁年轻人关爱父母的短片,让不少年轻人看了之后潸然泪下,想到自己渐渐长大,然而父母却一天天老去,心里很不是滋味。短短的一个电影,却承载了孝老敬亲这个巨大的伦理内核。‎ 微电影也可以通过其独特的方式,承载起普通人的中国梦。近年来,一批由农民、建筑工人、医护人员等特定群体自导自演的微电影备受关注。虽然这些影片未必有其他微电影那么高的艺术水准,但故事都取材于真实经历,感情真挚,也特别能够打动观众,从而通过网络向观众传递了核心价值观,促进不同群体之间的相互理解和包容互信。这当中的真实情感和文化情怀,展示了所有艺术作品万变不离其宗的成功之道;这些题材和拍摄方式,也为微电影承载社会价值提供了更多可能。因此,充分认识微电影的作用,做到扬长补短,“物”尽其用,是当前创作界的一道重要课题。‎ ‎(《光明日报》2016年3月9日2版)‎ 下列理解和分析,不符合原文意思的一项是(  )‎ A.近年来,中国微电影产业的总产值已达700亿元左右,这一数值还在不断攀升,可以肯定这一艺术形式具有广阔的发展前景。‎ B.一般来说,一个新的艺术类型在初创时期难以避免内容上泥沙俱下的情况,微电影也难逃这一魔咒,所以我们要加强管理和引导。‎ C.所有艺术作品万变不离其宗的成功之道是作品当中的真实情感和文化情怀,微电影也不例外,例如《筷子篇》传递了中华传统美德。‎ D.当前创作界应该充分认识微电影的作用,做到扬长补短,规避其在创作上内容低俗、推广上流于形式等问题,“物”尽其用。‎ 答案 A 解析 回答本题,先在文中分别找到备选项内容的对应语段,采用比对方法,在此基础上判断正误。原文第二段是“近年来,中国微电影产业的总产值已达700亿元左右,具有艺术、商业价值的年产量达2万部。据预测,未来3年内,总产值将超过1000亿元”,选项A中“这一数值还在不断攀升”“可以肯定”表述绝对化,“还在”表示既成事实;同时,不能因产值单一数据的可能提升肯定微电影有“广阔的发展前景”。故选A。‎ 五、阅读下面的文字,完成问题。‎ 中医药学伴随中国古老的社会文明的产生、发展而产生、发展。从自然环境中采集药物,运用从生活、生产实践尤其是同疾病作斗争的经验,而探索、总结和积累医疗技艺和方法,是中医药学的总体特点。同汉字因保留了原始的象形文字的特征,因而与世界各国文字相比独具特色一样,中医药学也因为直接从生活、生产实践中来,而同样独具特色。最能体现中医药学的哲学建树的是其整体恒动观念和辨证论治原则。‎ 天人一体观是中国自古就有的“天人相应”论,在中医学上的运用和反映包括两个方面:一是认为自然界是个整体,而人则是自然界中的一分子;二是认为人自身也是个有机整体。‎ 整体恒动观是指在认为人是个有机整体的前提下,人体的脏腑器官、气血津液,通过经络系统有机地联系起来,在正常的生理活动中相互依存、相互制约,而在患病过程中则又互为因果、相互制约。而这一切,又都是建立在充满朴素物本主义(唯物主义)和辩证法的阴阳五行学说的基础之上的。‎ 理论上融医理、哲理和伦理为一体的特征,乃中医药学的一大特色。所谓“悬壶济世,治病救人”就集中体现了这个特点。“凡大医治病,必当安神定志,无欲无求,先发大慈恻隐之心,誓愿普救含灵之苦,若有疾厄来求救者……皆如至亲之想。亦不得瞻前顾后,自虑吉凶,护借身命。见彼苦恼,若己有之,深心凄怆,勿避险恶,昼夜寒暑,饥渴疲劳,一心赴救,无作功夫形迹之心。如此可为苍生大医,反此则是含灵巨贼。……夫为医之法,不得多语调笑,谈谑喧哗,道说是非,议论人物,炫耀声名,訾毁诸医,自矜己德,偶然治瘥一病,则昂头戴面,而有自许之貌,谓天下无双,此医人之膏肓也。”孙思邈在《大医精诚》中所说的这段话,把中医药学融医理、哲理和伦理为一体的理论特征,表述得淋漓尽致。‎ 诊治上的辨证论治原则,是对相应时代所不断发展着的物本主义和辩证法在医疗实践上的运用。“辨证”就是在“四诊”即望神观形、听声嗅味、察言问病和按诊切脉的基础上,在整体观念、四诊和参等理论指导下,将四诊所得的资料和信息,进行综合分析,从内外征兆方面入手,了解病变的部位、原因和性质,探明并抓住病根,然后审因论治。具体方法包括“八纲辨证”“脏腑辨证”和“热病辨证”。‎ 临床上集医疗、护理、预防和养生为一体的特点,也充满深刻的哲理。诸如未病先防、既病防变,治病求本,急则治标、缓则治本,三因制宜和布阵用药中的“八略”立法、“八阵”制方等等,无一不渗透着辩证法。而在具体选药配方中的辨析药性、熟知药味、明晓归经、炮制适度、配伍确当、药量相宜、煎煮得法等等,也都无一不是对辩证法等哲理的灵活运用。‎ 中医药学所蕴含的哲理远不止这些,诸如自然观、物质观、运动观、整体观、联系观、时空观、矛盾观、系统观、调控观、生命观、疾病观、治疗观、认知观等众多方面,无一不在。‎ ‎(摘编自宋定国《国学纵横·中医药学之哲学建树》)‎ 下列理解和分析,不符合原文意思的一项是(  )‎ A.从自然环境中采集药物,借助生活、生产实践经验来探索、总结和积累医疗技艺和方法,这是中医药学的总体特点,也是其与众不同的特点。‎ B.汉字因保留了原始的象形文字特征而独具特色,中医药学则因直接从生活、生产实践中来而独具特色,其原因从根本上说是一样的。‎ C.中医药学“治病求本”,但并非一概而论,而是“急则治标、缓则治本”,对急病采用治标的方法,对缓病采用治本的方法,这也是辩证法在中医临床上的运用。‎ D.中医药学伴随中国古老社会文明的产生、发展而产生、发展,其哲学建树以朴素的物本主义和辩证法为基础。‎ 答案 C 解析 本题考查归纳内容要点、概括中心意思的能力。C项“对急病采用治标的方法,对缓病采用治本的方法”错误,原文倒数第二段是“未病先防、既病防变,治病求本,急则治标、缓则治本”,这可以理解为对紧急病症应当是先“治标”,病情缓和稳定后再“治本”,最后达到解决根本问题的效果。‎ 考点十八 分析概括作者在文中的观点态度 考点名片 考点内容 在归纳内容要点、概括中心意思的基础上分析概括作者对事物所持的看法、作者在文中所表现出的思想倾向和感情倾向,包括肯定与否定、爱与憎、褒与贬等。‎ 考查形式 ‎①全国卷、天津卷等为单选题,②其他省份试卷另有填空题、简答题。‎ 趋势分析 ‎①题型稳定,全国卷、天津卷等仍将采用单选题形式;②部分自主命题试卷仍会综合采用单选题、填空题、简答题的形式。‎ 一、阅读下面的文字,完成文后问题。‎ 培厚“精神新土层”,迎接“新集体生活”‎ 去年年底,腾讯公布官方数据显示,用户每天在微信平台上平均阅读5.86篇文章。以每篇1500字计,一个月下来就是一本200多页的书。这样一本“书”,恐怕难以扭转“中国人不读书”的印象,也不足以涵养丰盈的精神生活。不仅仅是在中国,美国埃默里大学教授马克·鲍尔莱恩对于“把时间都花在了社交网站”上的年轻人,也忧心忡忡。为了提醒埋头手机者保留一个“与历史、与艺术、与公民理念相遇”的生命空间,他的书名就叫《最愚蠢的一代》。‎ 作家博尔赫斯说:如果有天堂,那一定是图书馆的模样。爱书之人,可能都幻想过这样一座图书馆:没有容积的限制、跨越时空的区隔,“摊开你的手掌,无限在此收藏”。网络不就是这样吗?飞速发展的搜索技术、不断扩大的社交平台,海量的信息、无界的交流,只需在巴掌大的屏幕上敲击几下就尽在掌握。从丰富性的角度看,互联网绝不该成为文化的沙漠,而应是比博尔赫斯的想象更辉煌的“天堂”。‎ 今天的“新集体生活”更加开放、更加自由、个性更加鲜明。在网络的土壤中,即便是荒腔走板的歌声,也可能因触动隐秘的心弦而得到共鸣。只是,这样的个性张扬,虽为文化土层的培育提供了更大空间,却也“既有繁花,亦生稗草”,让互联网文化显得颇为芜杂。微信公号不乏“揭秘”“爆料”“有染”等吸引眼球的劲爆词汇,更有“30岁以前要明白”“中国人转起来”等或浓或淡的心灵鸡汤……这些内容尽管无伤大雅,却也难免让人担心它们背后日渐贫血的心灵。‎ 面对泥沙俱下的文化洪流,尤其需要澄清自己的文化水源,才能于沙中淘到金。无论有多么庞大的库存,你能接触到的如果只是你知道或者愿意知道的那一小部分,那与其说是看到了世界,不如说只是验证了自己。从这个角度看,那些无法深究的八卦、那些不知所云的感悟、那些似是而非的谣传、那些低俗庸俗的段子,不是丰富了你的生活,反而是把你阻隔在了一个伟大的宝库之外。‎ 互联网的丰富,正源于其海纳百川。网络时代独有的文化,可说是一种“新集体文化”。知识之海、观念之海,在每个“集体人”的参与中更显无涯。无数网友的标记、评论和打分,形成了影片、书籍、音乐的“标准”;网站中,许多专业人士分享的知识、经验、见解,形成了对一个问题的多角度考察。而如博客、微博、朋友圈,离开了成千上万用户的参与更是毫无意义。互联网,网聚人的力量;新集体文化,同样网聚每个人的文化。‎ 今天的中国人,尚未走出自身“三千年未有之大变局”,又一脚踏进信息时代的洪流,与全世界一起走进“旷古未有之大时代”。近百年来生活方式的变化,可能比之前上千年更大。‎ 从人类文明进程看,这样的时刻,往往伴随着文化活力的迸发。灿烂的文化成就,都生长于全新生产生活的土壤。正是从这样的角度,我们有着怎样的文化素养,不仅仅决定着我们能收获什么。告别肤浅,告别短促,走出心灵之“宅”,克服精神之“惫”,培厚新集体文化的精神土层,才能给历史真正留下属于我们时代的文化遗产。‎ ‎(选自《人民日报·人民观点》,有删改)‎ 对本文观点的理解不正确的一项是(  )‎ A.去年底,我国国民的阅读量平均每人每日阅读近6篇,一个月电子的阅读量就等于200多页一本书,这大大丰富了国民的精神生活,但并不能就说国民精神境界得到了提高。‎ B.网络中的搜索技术、社交平台,海量信息与无界交流,都可以随意掌握在人们手中,正可谓“摊开你的手掌,无限在此收藏”,互联网或许可成为人们文化精神中的“天堂”。‎ C.微信公号为吸引人们的眼球,常用些劲爆的词汇,增添些五花八门的内容,这恰好印证了互联网的丰富多彩与其海纳百川的特点,人们对此无须过分担忧,但都需引起注意。‎ D.对于当今的中国人来说,在尚未走出自身困境的同时,又踏进纷繁浩瀚的信息时代,这不仅仅是个挑战,更是一个机遇,随着生活方式的变化,可以着力促进文化活力的迸发。‎ 答案 A 解析 本题考查分析概括作者在文中的观点态度的能力。A项中“这大大丰富了国民的精神生活”与原文第一段“恐怕难以扭转 中国人不读书 的印象,也不足以涵养丰盈的精神生活”矛盾。‎ 二、阅读下面的文字,完成问题。‎ 如果说工业3.0要解决的是生产效率与消费效率之间的矛盾,那么,以“互联网+”为特征的工业4.0则很可能会打破先生产后消费的传统思维,甚至会让生产与消费之间的鸿沟逐步消除。‎ 在工业4.0阶段,传统产业与互联网是“互联网+”,而不再是“+互联网”。一个“+”位置变化耐人寻味。过去,无论信息化带动工业化还是二者深度融合,都是“+互联网”概念,即传统产业是主体,互联网只是工具。‎ 工具的最大特点是被动。再好的工具,只有被利用才有价值。工具化是工业3.0阶段互联网的主要特征。在3.0阶段,互联网作为具有革命性的工具,的确可以扩大和提升信息交流的空间和速度,从而让传统产业不仅生产效率继续有所提高,而且使得消费效率获得极大提升。特别是网络销售平台的建立,让消费过程变得更加高效、便捷。如果说以蒸汽机和电气化为代表的工业1.0和2.0所运用的是力学原理,解放的是体力,解决的是产能,那么以信息化为代表的3.0运用的则是数字手段,延伸的是人类的眼睛和耳朵,主要解决的是生产效率和消费效率之间的矛盾。在这一时期,互联网仍然是工具,因此传统产业的基本形态并没有因互联网的加入而改变。‎ 在工业4.0阶段,互联网已经不再是传统意义上的信息网络,它更是一个物质、能量和信息互相交融的物联网,传递的也不仅仅是传统意义上的信息,它还可以包括物质和能量的信息。互联网自身的演进导致了它角色的变化。某种意义上讲,今后的互联网已不再是一般意义上的工具,它会上升为矛盾主体,从设计、生产、销售到售后的全流程对传统产业进行改造。传统产业则可能变为被“+”的对象。‎ 互联网的去工具化从百度、腾讯等互联网巨头纷纷主动涉足传统制造业已经初现端倪。而互联网对传统制造业带来的颠覆在发达国家也已经出现。‎ 美国的某款新能源汽车,由于运用了物联网概念,已经取消了传统的4S店商业模式,不仅销售不需要,甚至保养、维修也不再需要4S店。因为发达的物联网络已经让生产者能掌握售出汽车的各种数据,甚至轮胎该更换或者哪个零件该出问题,厂家比用户还清楚,可以随时随地上门服务。‎ 率先提出工业4.0概念的德国走得更远。如今在德国一些城市的街头,停着不少有特殊标识的汽车,只要是会员,就可以用一张卡片把任意一辆车开走,办完事停在路边走人即可,而你驾驶车辆的所有数据会及时传回车厂,相关费用从你的账户扣除。这一模式未来很可能颠覆传统汽车的销售模式,很多人今后可以不必再买车;而类似奔驰、宝马这类传统制造企业,也很可能会演变成汽车服务提供商……‎ 据悉格力集团为应对小米手机的挑战,也开始生产手机了。但是,如果格力还是用传统制造思维去生产智能手机,就可能还会停留在3.0甚至2.0时代。‎ 根据原文内容,下列理解和分析不正确的一项是(  )‎ A.区别“互联网+”与“互联网”的主要依据,是互联网在解决生产与消费的矛盾中,究竟处于主体地位还是被动地位。‎ B.在“互联网+”阶段,厂家可通过物联网掌握售出产品的各项数据,根据需要随时随地主动地为用户提供上门服务。‎ C.在4.0时代,传统产业可能变为“+”的对象,类似奔驰、宝马这类传统制造企业,很可能会演变成汽车服务提供商。‎ D.反思应对小米手机挑战的过程,格力集团只有停止用传统制造思维去生产智能手机,才能真正步入工业4.0时代。‎ 答案 D 解析 本题考查分析概括作者在文中的观点态度的能力。D项,无中生有,原文最后一段说“如果格力还是用传统制造思维去生产智能手机,就可能还会停留在3.0甚至2.0时代”,不能由此推断D项的内容。‎ 三、阅读下面的文字,完成问题。‎ ‎“学而优则仕”传统之功过说 两千余年来,“学而优则仕”作为以学致仕的信条被读书人奉行不渝。尤其是隋唐科举制度形成以后,“学而优则仕”的信条与科举制度融为一体,互为里表,成了士子生活的金科玉律。‎ ‎“学而优则仕”传统在历史演化中对中国社会产生过积极影响。它确立了学问作为政府取吏的标准。以学取士将大部分饱读儒家经典的读书人吸引到官员队伍中,保证了政府运作始终处于接受过儒家道德教训的文吏手中。历代草莽英雄出身的开国皇帝不得不接受叔孙通的名言“儒者难与进取,可与守成”,并视为治国要诀,对书生保有相当的尊重。文吏统治造就了“士”作为无冕之王的优越地位,也促成了“士为四民之首”的观念。《三国演义》塑造了名士祢衡裸体痛骂曹操而为曹操所宽宥的形象,近代文化名人章太炎以大勋章作扇坠在袁世凯的总统府门前大诟袁氏包藏祸心,而被袁氏所容忍,个中原因固不止一端,但有一点可以肯定,士子对世道民心的巨大影响,无论是治世英雄,还是乱世奸雄,都不能不有所忌惮。另一方面,读书人坚守位卑不忘忧国的信条,以天下为己任,希望将平生所学贡献于国家民族,都与学优而仕传统有关。‎ 中国历史上,所谓“贵族”在很大程度上是一个文化概念,并不是全由血统决定。对社会各等级的人而言,通过以科举制度为体现的“学优而仕”途径跻身于士大夫阶级之后,可以加入孟子所说的“劳心者”之列,由“治于人”而变为“治人”,从而由“贱”入“贵”,成为“贵族”。正是由于学优而仕传统为读书人提供了改变自己命运的出路,整个中国社会各等级之间的划分才不像印度种姓制度那般僵死。中国数千年的传统文化并没有创造出多少“平等”观念,西方基督教世界的信众以信教而为自己争得了平等地成为上帝仆人的权利,而中国的士子们则由学优而仕获得了参与政治的平等权。‎ ‎“学而优则仕”传统对中国社会的负面影响,从根本上是源于以“仕”为“学”之鹄的这个既定前提。在“家”“国”一体的宗法专制时代,以“学”而至仕途,最终结局只能是以学问服务于帝王的“家天下”,“学成文武艺,货与帝王家”成为士子们的必然归宿。在帝王“家天下”附庸关系的等级网中,主仆关系的确立意味着对主子的物质和精神的依附,于是,以帝王之是非为是非,成为越两千余年不变的通例。被“学而优则仕”信念和科举功名诱入帝王彀中的天下的士子们,在主子面前只能“人主未命而唯唯,未使而诺诺”;在“食君俸禄,为君分忧”的附庸伦理支配下,为帝王的“家天下”的长治久安耗尽心力,以便在等于帝王将相家谱的所谓青史上留取功名,博取“忠”“贤”“能”的赞辞。‎ ‎(删改自《“学而优则仕”传统之功过说》)‎ 根据原文内容,下列理解和分析不正确的一项是(  )‎ A.即使草莽英雄出身的开国皇帝也明白“守江山治天下”,必须倚靠读书人,尊重士人,文吏治国是他们的统治要诀。‎ B.中国历史上的“贵族”阶层,对社会各等级的人是开放的,读书人通过读书改变自身命运,社会也在流动中保持了活力。‎ C.面对狂傲不羁的名士,那些历史上的英雄和奸雄都相当容忍,有所忌惮,原因就在于“士”阶层有着无冕之王的优越地位。‎ D.在科举制度已废除百年的今天,“逢进必考,择优录取”成为国家选聘公务员的严格规定,可以看作“学优而仕”传统具有巨大合理性的体现。‎ 答案 C 解析 本题考查分析概括作者在文中的观点态度的能力。“原因就在于……”说法不准确,原文第二段后面是“个中原因固不止一端”,说明历史上英雄和奸雄容忍名士的原因还有很多。‎ 四、阅读下面的文字,完成问题。‎ 中庸之道是中国文化的骨髓,作为一种方法论,它已经深深渗透到了与中国文化有关的每一个元素和成分之中,成为构成普遍的文化心理和社会心理的核心要素之一。每个置身于中国文化视野中的社会成员,无论你愿不愿意,承不承认,你都无法摆脱那与生俱来的中庸的思维模式和价值观。因此,正确地认识中庸之道,并加以合理地应用,既是一种智慧,也是一种无可回避的文化责任。‎ 中庸思想起源很早。《尚书》中就已经记载了很多关于古代圣王执中、行中的例子。在《尚书·大禹谟》中,有被宋儒称为“十六字心传”的那一著名的箴言:“人心惟危,道心惟微,惟精惟一,允执厥中。”《尚书·洪范》记载中就有中道的思想:“无偏无颇,无偏无党,王道荡荡。”由此,我们可以把《尚书》中强调“执中”的政治智慧看作是中庸之道的思想源头。“中庸”一词,语出《论语·雍也》。孔子说:“中庸之为德也,其至矣乎!民鲜久矣。”意思是,中庸乃至高的道德修养境界,长久以来,很少有人能够做得到了。孔子这样推许的“中庸”,其含义到底是什么?朱熹解释说:“不偏之谓中,不易之谓庸。中者天下之正道,庸者天下之定理。”“中庸者,不偏不倚,无过不及,而平常之理,乃天命所当然,精微之极致也。”这样,中庸一词的含义是:中,是一种凡事都追求不偏不倚、无过不及的最为恰当的状态;庸,则是说这样做是不可更易的常理。另一种解释来自汉代的郑玄。他注解说:“中庸者,以其记中和之为用也。庸,用也。”意思是,中庸,就是中道之运用。参照许慎《说文解字》的说法,“中,内也;上下通也”“庸,用也”,则“中庸”的本义就是中道的运用,郑玄的解释更符合中庸的本意。‎ 不管怎样,中道本身便是极高极难的标准。这一点,也可以通过孔子的其他言论来参证,如“过犹不及”,意思就是凡事都要尽可能追求最为恰当合理的处置,不能过,也不能不及。打个比方,比如勇敢,过了就是鲁莽,不及就是懦弱,只有恰如其分的勇敢,才符合勇敢概念本身。事实上,中庸本来是一种道德修养境界,但是孔子把它普泛化了,在这种普泛化的过程中,中庸也自然而然地转换为一种方法论了。一件事我们可以尽可能追求最优解,但是每件事都要追求最优解,实在是太难了。这样一个最优解,怎样才能做到呢?孔子的教导是,“吾有知乎哉?无知也。有鄙夫问于我,空空如也。我叩其两端而竭焉”“执其两端,而用其中于民”。“叩其两端”或“执其两端”,就是指从情的两种极端的状态入手,认真推敲研究,寻求和两种极端完全不同的最优解。这种“叩其两端”的处理方式,也着实不容易。‎ 为什么孔夫子要悬置这样一个大家都很难做到,却又不得不努力去做的高标准?缘由大约有三。其一,儒家以祖述尧舜,倡明先王之道为己任。在这个过程中,他们很容易把先王之道理想化。中道的思想作为尧舜禹汤、文武周公等古代圣王的成功秘诀,被儒家抬高到理想化的程度,更是情理之中的事。其二,儒家的中庸之道是古代农业社会大环境下的产物。在农业社会中,生产方式和生活方式的单一性,以及由单一性而必然存在的重复性,乃是儒家倡导凡事追求最优解思想的社会背景。其三,农业社会生产和生活的慢节奏也为儒家凡事追求最优解的理想化方案提供了时间上的可能性。‎ 根据原文内容,下列理解和分析正确的一项是(  )‎ A.正确认识并合理应用中庸之道,既体现人们的智慧,又反映人们的文化责任感。‎ B.中庸之道作为一种方法论,已经成为构成普遍的文化心理和社会心理的核心要素。‎ C.中庸之道是古代农业社会环境下的产物,根本原因是当时生产和生活方式的单一性。‎ D.儒家凡事追求最优解的理想化方案,长此以往便促成了农业社会生产生活的慢节奏。‎ 答案 A 解析 解答此题,可将各选项表述同文本相关内容进行比较。B项错误在于句末应该加上“之一”;C项错误在于“根本原因”,属于无中生有;D项错误在于因果关系颠倒。‎ 五、阅读下面的文字,完成问题。‎ 中国传统文化有两大支柱,一是儒家,二是道家。一般认为,儒家比较强调伦理学,重视道德修养;道家则强调智慧的觉悟和解脱。两者的不同,可以用三个简单观念来加以分辨。‎ 第一,儒家的思考方式是以人类为中心,所以肯定我们要尊重及帮助别人,让人类社会可以永续发展。儒家采取的路线是从政治上改革。‎ 道家认为以人为中心去思考问题,必定徒劳无功,必须改变思维模式,超越人类本位。而超越人类本位,首先必须顺其自然,尽量避免人为的造作,因为人为的造作越多,麻烦越多。譬如“天下皆知美之为美,斯恶已。皆知善之为善,斯不善已”,定出真、善、美的标准以后,就会有不真、不善、不美出现;反之,如果还没有标准,每个人都可以开心自在,不用刻意做好事,因为没有所谓的好事可做;不用担心有没有面子,因为所要做的只是活着而已。所以,人世间的一切都是相对的,道家的思想是要我们设法排除人类本位的想法,敞开眼界与心胸,从整个宇宙来看一切。只有不受时间与空间的拘束,心灵才可能自由逍遥。‎ 第二,儒家以“天”为至高存在,凸显历史背景;道家以“道”为至高存在,展现宇宙视野。任何一派哲学对于宇宙的真相或本体都必须有所论断。中国的传统思想是以“天”作为宇宙的最后根源。古人称帝王为“天子”,更是充分证明“天”在古人心目中是至高主宰。儒家承前启后,继承了这一观念,把“天”当作最高存在。孔子两次遇到困境,都把“天”抬出来,如说:“天之未丧斯文也,匡人其如予何?”‎ 道家则不同,道家以“道”代替“天”,“天”则被降格为和“地”并称,“天地”并称指的主要是自然界,自然界本身保持一种均衡状态,问题也远比人类社会少。然而,自然界虽然自给自足,毕竟不是最后的根源。道家认为宇宙最后的根源是“道”,“道生一,一生二,二生三,三生万物”。“道”孕育万物,是一切的起始与归宿;“道”存在于万物之中,却又超越万物。道家由“道”取代“天”的地位,很多西方学者据此认为,这是中国古代最具革命性的思想。‎ 第三,儒家期盼“天人合德”,从向善到择善再到至善;道家则希望“与道合一”。“天人合德”的“德”是善的德行,亦即人要行善,要不断地修养德行。“与道合一”则代表人要成为有道者或行道者,觉悟了“道”,人的生命境界整个就不一样了。‎ 过去认为,有三种人学习道家会比较有心得。一是年长的人,二是失意的人,三是非常聪明的人。从秦汉到唐宋,中国历代的文人,许多都喜欢道家,他们的作品所用的语汇,他们的生命所展现出的情调,与崇尚儒家的文人截然不同。像苏东坡在《前赤壁赋》提到的:“唯江上之清风,与山间之明月,耳得之而为声,目遇之而成色,取之无尽,用之不竭。”这显然是道家对大自然的欣赏,可以说是敞开心灵与自然沟通,不像儒家主要界定在人的社会中。‎ 但是,人活在这个世界上,也不能没有儒家作为指引,因为每一个人都是从家庭出发,然后进入社会,因而必须设法实践人与人之间适当的关系。如果离开儒家,可能会面临不知如何安顿自己,以及不知如何与人相处的问题。况且,如果大家都走道家的路,这个社会要交给谁来担当呢?由此观之,儒家和道家在社会的功用上,是有点分工合作的意味。‎ ‎(节选自傅佩荣《国学的天空》,有删改)‎ 下列对文章有关内容的理解和分析,正确的一项是(  )‎ A.中国的传统思想是以“天”作为宇宙的最后根源,“天”在古人的心目中就是至高主宰。‎ B.面对乱世,儒家采取的路线是从政治上改革,儒家有勇于担当的精神,这比道家强。‎ C.中国历代文人都喜欢道家,他们的生命所展现出的情调,与崇尚儒家的文人截然不同。‎ D.人活在世上,不能没有儒家作为指引,否则,会面临不知如何安顿自己等诸多问题。‎ 答案 A 解析 本题考查分析概括作者在文中的观点态度的能力。B项,“面对乱世,儒家采取……”说法属于无中生有;C项,“中国历代文人都喜欢道家”说法范围扩大,原文倒数第二段是“许多都喜欢道家”;D项,“会面临不知如何安顿自己等诸多问题”说法错误,原文最后一段是“可能会面临……”,该项把可能性说成肯定了。‎ ‎ [3年高考真题集训]‎ 一、[2016·全国卷Ⅰ]阅读下面的文字,回答1~3题。‎ 殷墟甲骨文是商代晚期刻在龟甲兽骨上的文字,是商王室及其他贵族利用龟甲兽骨占卜吉凶时写刻的卜辞和与占卜有关的记事文字。殷墟甲骨文的发现对中国学术界产生了巨大而深远的影响。‎ 甲骨文的发现证实了商王朝的存在。历史上,系统讲述商史的是司马迁的《史记·殷本纪》,但此书撰写的时代距商代较远;即使公认保留了较多商人语言的《尚书·盘庚》篇,其中亦多杂有西周时的词语,显然是被改造过的文章。因此,胡适曾主张古史作为研究对象,可“缩短二三千年,从诗三百篇做起”。甲骨文的发现,将商人亲手书写、契刻的文字展现在学者面前,使商史与传说时代分离而进入历史时代。特别是1917年王国维写了《殷卜辞中所见先公先王考》及《续考》,证明《史记·殷本纪》与《世本》所载殷王世系几乎皆可由卜辞资料印证,是基本可靠的。论文无可辩驳地证明《殷本纪》所载的商王朝是确实存在的。‎ 甲骨文的发现也使《史记》之类的历史文献中有关中国古史记载的可信性增强。因为这一发现促使史学家们想到,既然《殷本纪》中的商王世系基本可信,司马迁的《史记》也确如刘向、扬雄所言是一部“实录”,那么司马迁在《史记·夏本纪》中所记录的夏王朝与夏王世系恐怕也不是向壁虚构。特别是在20世纪20年代疑古思潮流行时期,甲骨文资料证实了《殷本纪》与《世本》的可靠程度,也使历史学家开始摆脱困惑,对古典文献的可靠性恢复了信心。‎ 甲骨文的发现同时引发了震撼中外学术界的殷墟发掘。“五四运动”促使中国的历史学界发生两大变化:一是提倡实事求是的科学态度,古史辨派对一切经不住史证的旧史学的无情批判,使人痛感中国古史上科学的考古资料的极端贫乏;二是历史唯物主义在史学界产生了巨大影响。1925年王国维在清华国学研究院讲授《古史新证》,力倡“二重证据法”,亦使中国历史学研究者开始注重地下出土的新材料。这些历史因素对近代考古学在中国的兴起具有催生作用。1928年秋,当时的中央研究院历史语言研究所开始发掘殷墟,其最初的目的乃是继续寻找甲骨。而第二次发掘时,已从主要寻找甲骨变成了对整个遗址所有遗存的科学发掘。‎ 甲骨文的发现还大大加速了对传统的中国文字学的改造。汉代以后中国的文字学家崇尚许慎的《说文解字》,传统的文字学主要是《说文》学;但由于北宋以来金石学的发展,特别是对金文的研究,已不断地用商周古文字对《说文》的文字学进行补充。到了清代,对金石学的研究进一步深入,使《说文》的权威性受到了较大的冲击。甲骨文的发现提供了汉字的早期形式,其构成离小篆甚远,多有象形、会意文字,令当时学者眼界大开。《说文》以小篆为本解释字源的理论难以维持,从此中国文字学就进入了一个新的时期。‎ ‎(摘编自朱凤瀚《近百年来的殷墟甲骨文研究》)‎ ‎1.下列关于原文内容的表述,不正确的一项是(  )‎ A.殷墟甲骨文是商代后期王公贵族占卜吉凶时写刻在龟甲或兽骨上的文字,它的发现对中国学术界产生了深远的影响。‎ B.在殷墟甲骨文发现之前,人们只能从有限的文献记载中了解到中国历史上存在过一个商王朝,然而这些文献却并非成于商代。‎ C.由于缺少成于商代的文字史料,因此从稳妥的角度出发,胡适认为古史研究大致可从西周时代开始进行。‎ D.1917年王国维写的《殷卜辞中所见先公先王考》及《续考》,证明了《史记·殷本纪》所载内容的真实性。‎ 答案 D 解析 “证明了《史记·殷本纪》所载内容的真实性”错。原文第二段末尾:“证明《史记·殷本纪》与《世本》所载殷王世系几乎皆可由卜辞资料印证,是基本可靠的。论文无可辩驳地证明《殷本纪》所载的商王朝是确实存在的。”‎ ‎2.下列理解和分析,不符合原文意思的一项是(  )‎ A.在20世纪20年代疑古思潮流行时期,一些历史学家对《世本》的可靠性将信将疑,认为其中记载的一些内容恐怕是虚构的。‎ B.旧史学的研究既缺少实事求是的科学态度,又缺乏科学的考古资料,因而它受到古史辨派的无情批判。‎ C.王国维的“二重证据法”让中国历史学研究者认识到,在考证古史时不仅要注重历史文献的记载,也要重视地下出土的新材料。‎ D.许慎的《说文解字》没有利用汉字的早期形式,而主要依据小篆来研究古文字,这使它在解释字源方面存在着一定的不足。‎ 答案 B 解析 由原文倒数第二段开头“一是提倡实事求是的科学态度,古史辨派对一切经不住史证的旧史学的无情批判,使人痛感中国古史上科学的考古资料的极端贫乏”可见,该选项强加因果。另外,旧史学的研究 ‎“缺少实事求是的科学态度”,无中生有。‎ ‎3.根据原文内容,下列说法不正确的一项是(  )‎ A.《尚书·盘庚》明显是后人改造过的文章,由此看来,尽管其中保留了许多商人语言,但是仅凭此篇仍不足以证实商王朝的存在。‎ B.若想证实司马迁在《史记·夏本纪》中记录的夏王朝与夏王世系的客观存在,还要依靠地下出土的新材料。‎ C.第二次殷墟发掘的目的发生了改变,是因为历史语言研究所认识到,除了甲骨之外,遗址的其他遗存也可以作为研究中国历史的材料。‎ D.直至殷墟甲骨文被发现,学者们探究先民的造字之法才有所凭依,从此中国的文字学就进入了一个新的时期。‎ 答案 D 解析 “直至殷墟甲骨文被发现,学者们探究先民的造字之法才有所凭依”不合文意,由原文最后一段可知,汉代以后,学者们探究先民的造字之法主要凭依小篆;北宋以来,凭依金文对《说文》的文字学进行了补充。‎ 二、[2016·全国卷Ⅱ]阅读下面的文字,回答1~3题。‎ 人们常说“小说是讲故事的艺术”,但故事不等于小说,故事讲述人与小说家也不能混为一谈。就传统而言,讲故事的人讲述亲身经历或道听途说的故事,口耳相传,把它们转化为听众的经验;小说家则通常记录见闻传说,虚构故事,经过艺术处理,把它们变成小说交给读者。‎ 除流传形式上的简单差异外,早期小说和故事的本质区别并不明显,经历和见闻是它们的共同要素。在传媒较为落后的过去,作为远行者的商人和水手最适合充当故事讲述人的角色,故事的丰富程度与远行者的游历成正比。受此影响,国外古典小说也常以人物的经历为主线组织故事。《荷马史诗》《一千零一夜》都是描述某种特殊的经历和遭遇,《堂吉诃德》中的故事是堂·吉诃德的行侠奇遇和所见所闻,17世纪欧洲的流浪汉小说也体现为游历见闻的连缀。在中国,民间传说和历史故事为志怪类和史传类的小说提供了用之不竭的素材,话本等古典小说形式也显示出小说和传统故事的亲密关系。‎ 虚构的加强使小说和传统故事之间的区别清晰起来。小说中的故事可以来自想象,不一定是作者亲历亲闻。小说家常闭门构思,作品大多诞生于他们离群索居的时候。小说家可以闲坐在布宜诺斯艾利斯的图书馆中,或者在巴黎一间终年不见阳光的阁楼里,杜撰他们想象中的历险故事。但是,一名水手也许要历尽千辛万苦才能把在东印度群岛听到的事带回伦敦;一个匠人漂泊一生,积攒下无数的见闻、掌故和趣事,当他晚年坐在火炉边给孩子们讲述这一切的时候,他本人就是故事的一部分。传统故事是否值得转述,往往只取决于故事本身的趣味性和可流传性。与传统讲故事的方式不同,小说家一般并不单纯转述故事,他是在从事故事的制作和生产,有深思熟虑的讲述目的。‎ 就现代小说而言,虚构一个故事并非其首要功能,现代小说的繁荣对应的是故事不同程度的减损或逐渐消失。现代小说家对待故事的方式复杂多变,以实现他们特殊的叙事目的。小说家呈现人生,有时会写到难以言喻的个人经验,他们会调整讲故事的方式,甚至将虚构和表述的重心挪到故事之外。在这些小说家笔下,故事成了幌子,故事之外的附加信息显得更有意味。19世纪末期以来,小说家对小说故事性的破坏日趋强烈。这时,一个故事的好坏并不看它的“成色”如何,而是取决于讲故事的方式。契诃夫曾经把那些不好好讲故事的小说家称为“耍弄蹩脚花招的人”,但这种花招的大量出现也有其内在的合理性——他们要摆脱陈旧的故事模式,摆脱虚假的因果关系和矫揉造作的戏剧冲突,甚至摆脱故事本身。现代小说家认为,传统的故事模式早已失去了弹性和内在活力,也失去了起初的存在价值,那些千百年来一直在给小说提供养料的故事模式已经成为制约想象力的障碍之一。‎ ‎(摘编自格非《塞壬的歌声》)‎ ‎1.下列关于原文内容的表述,不正确的一项是(  )‎ A.讲故事的人不一定是小说家,小说家在讲故事的时候,不像传统的故事讲述者那么依赖亲身经历和耳闻目睹的事。‎ B.传统故事和早期小说的本质差异在于,前者是故事的口耳相传,后者则是由作家创作加工后的游历见闻。‎ C.17世纪的欧洲流浪汉小说和部分中国古典小说,或在叙述形式方面,或在素材来源方面,都受到了传统故事的影响。‎ D.当小说家越来越依靠想象力虚构故事的时候,小说和传统故事在内容来源方面的差异使它们之间的关联不再像过去那么紧密。‎ 答案 B 解析 曲解文意,“传统故事和早期小说的本质差异”错,文章第二段开头的表述是“早期小说和故事的本质区别并不明显”。‎ ‎2.下列理解和分析,不符合原文意思的一项是(  )‎ A.水手在伦敦讲东印度群岛的所见所闻,匠人在火炉边讲自己的人生经历,他们讲的故事各有特点,但同属于传统故事模式。‎ B.传统的故事讲述者大多会讲述那些为听众喜闻乐见的事,小说家则会根据自己的写作意图审慎构思,创作新的故事。‎ C.现代小说不太注重一个故事如何来讲,因为故事情节已不再是现代小说最重要的因素,人们更注意故事之外的附加意味。‎ D.现代小说家不喜欢传统故事模式,视它为绊脚石,是因为他们觉得这种故事模式显得僵化古板,已经不能促进小说艺术的发展。‎ 答案 C 解析 由文章第四段“一个故事的好坏并不看它的 成色 如何,而是取决于讲故事的方式”可知,现代小说重视的是一个故事如何来讲,而非“不太注重”。此外“故事情节已不再是现代小说最重要的因素”与“现代小说不太注重一个故事如何来讲”之间也不存在因果关系。‎ ‎3.根据原文内容,下列说法不正确的一项是(  )‎ A.传统的故事讲述人如果把自己的故事记录下来,进行加工整理,就能形成一种和早期小说接近的文字,有些讲述人也会成为小说家。‎ B.现代小说家尝试用新的方式讲故事,会削弱小说的故事性,这将降低小说对虚构的依赖,小说的个人表达功能却会因此得到强化。‎ C.契诃夫不大认可“不好好讲故事的小说家”,对他们的做法评价不高,由此可知当时这股写作潮流与他的创作理念相悖。‎ D.现代小说的发展加剧了故事在小说中的衰变,与此同时,随着现代传媒的不断发展,传统的故事讲述方式也可能消亡。‎ 答案 B 解析 根据文章第四段内容,“这将降低小说对虚构的依赖”有误,应是“甚至将虚构和表述的重心挪到故事之外”。‎ 三、[2016·全国卷Ⅲ]阅读下面的文字,完成1~3题。‎ 文学中有历史。当今历史学家大都认为,没有什么文献资料不是史料,不但文学作品,即如佛经、道藏、信札、家谱、账本、碑铭等也无一不是,而且随着史学研究领域的拓展,史料范围还在不断扩大。从“三言二拍”里可以看到晚明市井生活的真实面貌,这对于研究社会史的人几乎是一个常识。陈寅恪以诗证史,也为大家所熟悉。但在“五四”以前,史料范围并非如此宽泛,文学作品在大多数史学家眼里也并非史料,有些文献到底属于文学还是史学,一两千年来都没有一致的看法。神话传说就是如此,其中相当突出的例子是《山海经》。‎ 神话传说是文学,史前时代,无文字可征,只有传说,暂当历史。三皇五帝至今未曾坐实,但“炎黄子孙”已经成为口头语,甚至成为历史共识。新的传说还会不断产生,能否成史颇为可疑,但以神话传说研究历史,却是一种重要的方法。在历史上,《山海经》究竟应归于文学还是史学,曾是死结。王国维《古史新证》说:“而疑古之过,乃并尧、舜、禹之人物而亦疑之,其于怀疑之态度及批评之精神不无可取,然惜于古史材料未尝为充分之处理也。”这些古史材料就包括《山海经》《穆天子传》等文献。在《汉书·艺文志》里,《山海经》列于数术类。此后该书在目录学里的角色转换过几次,《隋书·经籍志》将《山海经》列于史部地理类,也就是将它看成史书了。‎ 历史是讲真实的,《山海经》一般被视为荒诞不经,连司马迁写《史记》都不敢采用。虽然《山海经》里平实的山川地理内容应归于史部,但其中大量的神话故事却显然有悖信史,所以清人编《四库全书》,言其“侈谈神怪,百无一真,是直小说之祖耳”,将其改列于子部小说家类。这个死结直到“五四”以后才大致解开。解开的途径有二:一是将《山海经》分而治之,不把它看作一部成于一人一时之书,神话归神话,历史归历史;二是神话中也有历史的成分在,仍可以之证史或补史。分而治之者,以为《山海经》中的《五藏山经》是比较雅正的部分,谭其骧就写了《〈五藏山经〉的地域范围》一文,分析《山经》写作时的地理知识水平。将历史成分发掘出来的,自然以王国维用《山海经》来印证甲骨文中殷商先王亥为最明显的例子。‎ 上面说的是介于文学与史学之间的文献,至于纯粹的文学作品,当然也能从中发掘史料。但发掘史料是一回事,把整个作品当成真史就很可虑了。《红楼梦》反映了清代前期的历史现实没有错,可是如果过分坐实到具体历史人物身上,就未免失之穿凿了。戏说之类当然是文学,但读者观众往往误以为是历史。如中俄签订《尼布楚条约》,张诚、徐日昇当时担任与俄国谈判的翻译,工作是以拉丁语作为中介的,而电视剧《康熙王朝》中他们说的却是俄语,观众看到这个情节时被误导也就难以避免了。‎ ‎(摘编自周振鹤《历史中的文学与文学中的历史》)‎ ‎1.下列关于原文内容的表述,不正确的一项是(  )‎ A.在当今历史学界,历史学家的研究领域不断地扩展,各种体裁的文学作品都有可能成为他们研究历史的资料。‎ B.古代的史学家选取史料的范围比较狭窄,他们并未广泛采用“以诗证史”或将小说用于社会历史研究之类的方法。‎ C.王国维在《古史新证》中认为,有些历史学家如果能充分利用史料,就不会“疑古”,怀疑尧、舜、禹等人物的真实性。‎ D.历代学者对《山海经》有不同认知,《隋书·经籍志》把它列入史部,视为史书,王国维则把它作为古史材料看待。‎ 答案 C 解析 此题主要考查理解文中语句含义的能力。选择表述不正确的,是这类题一以贯之的考法。本题难度不大,需要学生理解选项含义,并回到原文作对照分析。找准语境和细致比对是关键。C项“就不会 疑古 ,怀疑尧、舜、禹等人物的真实性”错误。原文第二段“而疑古之过,乃并尧、舜、禹之人物而亦疑之,其于怀疑之态度及批评之精神不无可取,然惜于古史材料未尝为充分之处理也”的表述,表明王国维并非讲不该疑古,而是讲疑古过分。‎ ‎2.下列理解和分析,不符合原文意思的一项是(  )‎ A.很多人认为《山海经》的记载荒唐夸张,与真实的历史差别较大,司马迁也持这种观点,因此《史记》并不采用《山海经》。‎ B.《四库全书》的编者认为,《山海经》所记的神话传说并无真实可言,不宜归入史部,而应列入子部小说家类。‎ C.谭其骧和王国维利用《山海经》研究历史的方法不同,前者是将神话和历史分而治之,后者则从神话中发掘史料。‎ D.电视剧《康熙王朝》对历史事件和历史人物进行了虚构,其中部分情节与历史事实有出入,不能从这类作品中发掘史料。‎ 答案 D 解析 此题主要考查理解句子甚至语段的含义、筛选并整合文中信息的能力。要求选择不符合原文意思的项。本题难度适中,需要学生理解语句、语段含义,筛选信息,对照分析。D项“其中部分情节与历史事实有出入,不能从这类作品中发掘史料”错误,与这个选项对应的原文最后一段的段首句为“至于纯粹的文学作品,当然也能从中发掘史料”。‎ ‎3.根据原文内容,下列说法不正确的一项是(  )‎ A.即使在科学技术如此发达的今天,也会产生新的传说,这些传说将来会不会成为研究这个时代的史料也未可知。‎ B.“五四”‎ 之前,很多涉及历史的神话传说之所以没有成为广泛使用的史料,是因为这些作品在史学和文学归类问题上存在争议。‎ C.在历史研究中,当代学者会把文学作品作为史料看待,在他们看来,《三国演义》和《水浒传》的艺术手法差异并不重要。‎ D.文学作品能否成为史料,取决于历史学家的眼光,而历史学家对文学与史学关系的认识在一定程度上受制于当时的学术背景。‎ 答案 B 解析 要学生筛选信息,并作理解、综合分析与对照,然后做推断。B项“是因为这些作品在史学和文学归类问题上存在争议”错误,属于因果不当,原文是“这个死结直到 五四 以后才大致解开。解开的途径有二:一是……分而治之……二是神话中也有历史的成分在,仍可以之证史或补史”,换言之即五四之前未对其分而治之。‎ 四、[2015·全国卷Ⅰ]阅读下面的文字,完成1~3题。‎ 宋代的农业、手工业、商业在唐代的基础上又有了新的发展,特别是商品经济出现了空前的繁荣。在此背景下,宋代的货币流通和信用进入迅速发展时期,开创了古代金融的新篇章。‎ 宋代在信用形式和信用工具方面都呈现出新的特点。信用形式有借贷、质、押、典、赊买赊卖等多种形式。借贷分为政府借贷和私人借贷。政府借贷主要表现为赈贷的形式,在紧急情况下通过贷给百姓粮食或种子的方式,帮助他们渡过困境。私人借贷多为高利贷,它可以解决社会分化和“钱荒”带来的平民百姓资金严重不足的问题,满足特殊支付和燃眉之急的需要。质、押是借贷的担保形式,由质库、解库等机构经营。质属于动产担保,它必须转移动产的占有;押属于不动产担保,通常将抵押物的契约交付债权人即可。债务人违约时,债权人可用变卖价款优先受偿。典作为不动产转移的一种形式是在宋代形成和发展起来的。其特点是典权人向出典人支付典价后,在典期内就占有了出典人典产的使用权和收益支配权,出典人也不必向典权人支付利息。宋代的商业贸易非常发达,但存在着通货紧缩现象,故赊买赊卖行为也很普遍,几乎生产、流通、消费领域的所有物品都能进行赊买赊卖。从实际效果看,它解决了军需、加强了流通,更重要的一点,它对束缚生产流通扩大和发展的高利贷构成了冲击。‎ 随着社会经济的发展,宋代商业贸易对货币的要求越来越高,但是社会中货币供给和流通状况不尽理想,表现为货币流通区域的割据性、货币供给数量的有限性,以及大量流通的铜铁钱细碎和不便携带的特性,其结果是抑制了经济发展。为了解决这类问题,在高度发达的造纸和印刷技术保障下,通过民间自发力量的作用和官府的强制推行,宋代社会陆续出现了诸如茶引、盐引、交子、关子和会子等新型纸质信用工具。茶引、盐引要求相关人员先用粮草或现钱的付出作为取得的条件,然后凭此类纸质信用工具异地兑取现钱或政府专卖货物。这些信用工具的使用,除了可发挥信用功能外,也使得政府和商人在专卖货物领域能够共同获利,既有利于商人从政府专卖的货物中分得一份利益,又有利于政府实现增加收入、补给军需等目标。早期的交子、关子、会子要求相关人员先交纳现钱作为取得的条件,然后再根据需要持交子、关子、会子到指定的地区兑取现钱。这类信用工具携带方便且具有汇票性质,可以保障大宗交易、跨地区交易货款的顺利结算。它们的使用,弥补了货币的不足,节省了货币流通需求量。此后这种交子、关子、会子逐步发展为纸币。可见,宋代新型信用工具的大量使用,是社会经济发展史中最具标志性意义的新生事物,它缓解或解决了交换过程中的诸多不便与矛盾,从而在很大程度上促进了经济发展。‎ ‎(摘编自王芳《宋代信用的特点与影响》)‎ ‎1.下列关于原文内容的表述,不正确的一项是(  )‎ A.宋代的信用进入迅速发展时期,借贷、质、押、典、赊买赊卖等信用形式的产生是宋代金融的一个新特点。‎ B.宋代的政府借贷基本上是赈济性借贷,主要目的是帮助百姓渡过困境,因此与私人借贷相比,政府借贷的利率要低得多。‎ C.在宋代,债务人可以用不动产的契约或动产作为担保,向债权人借贷。在债务人不偿还债务时,债权人可用变卖价款优先受偿。‎ D.赊买赊卖的信用形式在一定程度上解决了宋代通货紧缩带来的资金不足的问题,缓解了生产、流通、消费领域中的诸多矛盾。‎ 答案 A 解析 “信用形式有……多种形式”是宋代金融的一个新特点,文中无信息表明“借贷、质、押、典、赊买赊卖”都是在宋代产生的。‎ ‎2.下列理解和分析,不符合原文意思的一项是(  )‎ A.在商品经济发展的推动下,宋代的信用工具不断创新,出现了茶引、盐引、交子、关子和会子等信用工具。‎ B.各类新型纸质信用工具最初是由宋代政府发行的,其发行目的是为了解决货币流通区域的割据性等多方面的问题。‎ C.茶引、盐引等信用工具的使用,可以使一些商人取得茶、盐等货物的专卖凭证,从政府专营的物品中分得一部分利益。‎ D.宋代的造纸术和印刷术高度发达,这为交子、关子和会子等新型信用工具的产生提供了技术条件。‎ 答案 B 解析 “各类新型纸质信用工具最初是由宋代政府发行的”错误,原文第三段前面是“通过民间自发力量的作用和官府的强制推行”。‎ ‎3.根据原文内容,下列理解和分析不正确的一项是(  )‎ A.质库、解库是进行押物、放款、收息的机构。唐宋时期随着社会经济的日益发展,质库、解库也随之兴盛。‎ B.在宋代,出典人将房产押给典权人后,可以获得一笔典价,且不必支付利息。在典期内,典权人不但享有房屋的使用权,同时还拥有出租权。‎ C.虽然早期的交子具有汇票性质,可以克服金属货币不便携带的缺点,保障商品交易中货款的顺利结算,但是它还没有发展成为纸币。‎ D.宋代各种信用形式和信用工具对当时的经济发展都起到非常积极的作用,同时也为此后各个朝代提供了借鉴。‎ 答案 D 解析 说法过于绝对,不能说“宋代各种信用形式和信用工具”都有积极作用,借贷中的私人借贷多为高利贷,高利贷对生产流通扩大和发展有阻碍。‎ 五、[2015·全国卷Ⅱ]阅读下面的文字,完成1~3题。‎ 艺术品的接受在过去并不被看作是重要的美学问题,20世纪解释学兴起,一个名为“接受美学”的美学分支应运而生,于是研究艺术品的接受成为艺术美学中的显学。‎ 过去,通常只是从艺术家的立场出发,将创作看作艺术家审美经验的结晶过程,作品完成就意味着创作完成。而从接受美学的角度来看,这一完成并不说明创作已经终结,它只说明创作的第一阶段告一段落,接下来是读者或观众、听众的再创作。由于未被阅读的作品的价值包括审美价值仅仅是一种可能的存在,只有通过阅读,它才转化为现实的存在,因此对作品的接受具有艺术本体的意义,也就是说,接受者也是艺术创作的主体之一。‎ 艺术文本即作品对于接受者来说具有什么意义呢?接受美学的创始人、德国的伊瑟尔说艺术文本是一个“召唤结构”,因为文本有“空白”“空缺”“否定”三个要素。所谓“空白”,是说它有一些东西没有表达出来,作者有意不写或不明写,要接受者用自己的生活经验与想象去补充;所谓“空缺”,是语言结构造成的各个图像间的空白,接受者在阅读文本时要把一个个句子表现的图像片断联结起来,整合成一个有机的图像系统;所谓“否定”,指文本对接受者生活的现实具有否定的功能,它能引导接受者对现实进行反思和批判。由此可见,文本的召唤性需要接受者呼应和配合,完成艺术品的第二次创作。正如中国古典美学中的含蓄与简洁,其有限的文字常常引发出读者脑海中的丰富意象。‎ 接受者作为主体,他对文本的接受不是被动的。海德格尔提出“前理解”,即理解前的心理文化结构,这种结构影响着理解。理解不可能是文本意义的重现,而只能是文本与“前理解”的统一。这样,文本与接受就呈现出一种相互作用的关系:一方面文本在相当程度上规定了接受者理解的范围、方向,让理解朝它的本义靠拢;另一方面,文本不可能将接受者完全制约住、规范住,接受者必然会按照自己的方式去理解作品,于是不可避免地就会出现误读或创造。从某种意义上说,理解就是误读,创造也是误读,不要希望所有的接受者都持同样的理解,也不要希望所有的理解都与艺术家的本旨一致,那样并不意味着艺术作品的成功。‎ 文本一经产生就成为历史,它所表达的思想感情、所反映的生活,都只能是过去的,而理解总是现在进行时。当我们接受历史上的艺术作品时,我们当然可以设身处地想象古人的生活,体验古人的思想感情,但我们毕竟是现代人,只能按照我们现在的心理文化结构去理解古人。当然,任何理解都只能是个体的理解,但个体毕竟是与群体相通的,所以个体的理解中也有普遍性。理解作为现实的行为具有通向实践的品格,艺术品正是通过理解走向现实,并在生活中发挥作用的。不是别的,正是理解擦亮了艺术品的生命之光。‎ ‎(摘编自陈望衡《艺术是什么》)‎ ‎1.下列关于原文内容的表述,不正确的一项是(  )‎ A.在过去,艺术品的接受并不属于美学的研究范围,而当接受美学诞生以后,关于艺术品的接受的研究就成为艺术美学中的一门显学。‎ B.在接受美学诞生以前,人们一般的认识是:整个创作过程就是艺术家的审美经验不断结晶的过程,艺术品一旦形成,创作也就大功告成。‎ C.接受美学认为,艺术品在艺术家手中产生出来,这只是艺术创作的第一阶段,读者、观众、听众对艺术品的接受是艺术创作的继续。‎ D.通过读者、观众和听众的接受,艺术作品的价值才从一种可能的存在转化为现实的存在,从这个意义上说,接受也属于艺术创作的一部分。‎ 答案 A 解析 “在过去,艺术品的接受并不属于美学的研究范围”错,原文首句说的是“艺术品的接受在过去并不被看作是重要的美学问题”,这句话的意思是,艺术品的接受属于美学的研究范围,只是并不重要。‎ ‎2.下列理解和分析,不符合原文意思的一项是(  )‎ A.文本之所以是一个“召唤结构”,一个原因就是它有一些内容有意不写或不明写,需要接受者用自己的生活经验与想象去补充。‎ B.文本的“否定”要素具有对接受者所生活的现实加以否定的功能,这种功能是通过接受者接受文本并对现实进行反思和批判而实现的。‎ C.“前理解”是接受者在理解文本以前的心理文化结构,由于接受者对文本的接受不是被动的,所以这种结构会影响接受者对文本的理解。‎ D.作品被艺术家创作出来以后就成为历史,作品是通过接受者的理解而存活于现实,并发挥作用的。从这个意义上说,作品的生命力存在于理解之中。‎ 答案 C 解析 因果关系不当,见原文第三段。‎ ‎3.根据原文内容,下列理解和分析不正确的一项是(  )‎ A.中国古典美学中强调的含蓄和简洁可以说是艺术作品召唤性的体现,含蓄的美在于从有限中表现无限,简洁的美在于以少胜多,以简驭繁。‎ B.理解就是误读,创造也是误读,理解距离艺术作品的本义越远,就越是具有创造性,正如《西厢记》之于《莺莺传》、《金瓶梅》之于《水浒传》。‎ C.文本在一定程度上规定了接受者理解的范围和方向,所以即使我们今天阅读历史上的艺术作品,也可以在相当程度上了解古人的生活,体验古人的思想感情。‎ D.作为接受者的个体毕竟生活在群体之中,其思维和观念与群体是相通的,因此接受者们对于同一文本的理解即使千姿百态,也不可能完全没有同一性。‎ 答案 B 解析 “理解距离艺术作品的本义越远,就越是具有创造性”的说法于文无据,对应原文第四段。‎ 六、[2014·全国卷Ⅰ]阅读下面的文字,完成1~3题。‎ 悲剧产生于社会的矛盾、两种社会力量的冲突。冲突双方分别代表着真与假、善与恶、新与旧等对立的两极,却总是以代表真、善、新等美好的一方的失败、死亡、毁灭为结局,他们是悲剧的主人公。因为他们的力量还比较弱小,还无法与强大的旧势力或邪恶力量抗衡,正义的要求不能实现,于是形成了悲剧。古希腊学者亚里士多德指出,悲剧描写了比现实中更美好同时又是“与我们相似的”人物,通过他们的毁灭“引起怜悯和恐惧来使感情得到陶冶”,即产生净化的作用。‎ 然而,悲剧不仅表现冲突与毁灭,而且表现抗争与拼搏,这是悲剧具有审美价值的最根本的原因。鲁迅说过:“悲剧将人生的有价值的东西毁灭给人看。”这种毁灭是抗争、拼搏以后的毁灭,抗争与拼搏体现了人的一种精神。古希腊神话中普罗米修斯为了人类从天上盗取火种,触怒了主神宙斯,被锁在高加索山崖上,每日遭神鹰啄食肝脏,但普罗米修斯毫不屈服,最后坠入深渊。罗丹的大理石雕塑《马身人首》中,人臂绝望地扑向一个它所抓不到的目标,而马足则陷于尘土不能自拔,表现出人性与兽性的冲突,象征着灵与肉的斗争,具有强烈的悲剧性。可以说,没有抗争就没有悲剧,冲突、抗争与毁灭是构成悲剧的三个主要因素。‎ 悲剧的审美价值的载体只能是文学艺术。因为人生有价值的东西、美好事物的毁灭是令人伤悲的,因此现实中的悲剧不能作为直接的审美对象来欣赏,否则人就是泯灭了人性的人了。现实中的悲剧只能激起人的同情、义愤,迫使人采取严肃的伦理态度和实践行动。民主革命时期,在演出歌剧《白毛女》的过程中,曾多次出现扮演地主黄世仁的演员被打甚至险遭枪击的事件,这是人们以实际的道德评价代替了审美活动。现实的悲剧只在客观上具有悲剧的审美性质,它们必须以文学艺术的形式表现出来,才能成为欣赏的对象,美学上所谓的“以悲为美”才能实现。‎ 悲剧成为审美对象只能以文学艺术的形式出现,原因在于它需要建立悲剧事件与人的心理距离。不仅遥远的时间会使过去的现实悲剧的悲惨因素淡化,就是很近的时间间隔也可以使人不陷入现实。这里还有一个空间的间隔,悲剧艺术展现的毕竟是一个人们不熟悉或有点陌生的空间,这就使人们不容易介入其中,而能够客观、超然地看待。当然,在欣赏中审美主体可以“审美地”加入悲剧冲突,体验悲剧客体的巨大和狂暴、悲剧主体的抗争和悲痛,从而感受到强烈的震撼和刺激,获得悲剧感和审美愉悦。‎ 悲剧表现的不是人生的欢乐或全然的幸福,而是悲剧主体对待痛苦和死亡的方式,这是人类社会和人类活动中十分重要、严肃的一面。悲剧在表现对伟大和崇高的人的摧毁的同时,更表现出无法摧毁的人的伟大和崇高。‎ ‎(摘编自王晓旭《美的奥秘》)‎ ‎1.下列各项中,其性质不属于原文所论悲剧的一项是(  )‎ A.在梁山伯与祝英台的故事中,祝英台女扮男装外出求学,为追求爱情自由,面对封建势力的巨大压力,拒绝委曲求全,最后触碑殉情,化成蝴蝶。‎ B.在甲午海战中,清军致远舰在中弹累累、舰身倾斜、弹药耗尽的情况下,开足马力,冲向日本吉野舰,最后被鱼雷击中,沉入海中,200多名官兵壮烈殉国。‎ C.在电影《狼牙山五壮士》中,五位八路军战士为了掩护大部队撤退及当地群众安全转移,阻击了3000多名日寇的多次进攻,弹尽粮绝之后,跳下悬崖。‎ D.老舍笔下的祥子,纯朴善良,勤劳能干,有着骆驼般坚韧的精神,在饱受旧社会、旧制度的沉重打击之后,沦为自甘堕落的行尸走肉。‎ 答案 B 解析 甲午海战是史实,而不是文艺作品中的事件。原文倒数第二段所论:悲剧的审美价值的载体只能是文学艺术,悲剧成为审美对象只能以文学艺术的形式出现。因此,B项性质不属于原文所论悲剧。‎ ‎2.下列理解,不符合原文意思的一项是(  )‎ A.在悲剧冲突中,代表真、善、新等美好的一方总是以失败、死亡、毁灭为结局,他们是悲剧的主人公,即悲剧主体,而其对立面则是悲剧客体。‎ B.在罗丹的《马身人首》雕塑中,人首和人臂是人、灵和人性的象征,马身和马足则是兽、肉和兽性的象征,兽性和人性的矛盾构成了人间的悲剧。‎ C.当悲剧以文学艺术的形式出现,悲剧事件与观众或读者之间就会具有一定的心理距离,这样人们就不至于获得悲剧感,从而不至于介入悲剧冲突之中。‎ D.悲剧主体的死亡意味着肉体力量的失败,却并不意味精神力量的失败,所以说悲剧在表现伟大和崇高的人被摧毁的同时,更表现出人的无法摧毁的伟大和崇高。‎ 答案 C 解析 根据选文第四自然段的内容可知,C项中所说的“这样人们就不至于获得悲剧感”不符合原文意思。‎ ‎3.根据原文内容,下列理解和分析不正确的一项是(  )‎ A.亚里士多德认为悲剧具有“净化”作用。他所说的“净化”,不是指受众在生理上的发泄,如呼喊、哭泣等,而是指他们道德、精神和情感的提升。‎ B.悲剧在表现冲突与毁灭的同时,也表现抗争与拼搏,因此双方力量越是悬殊,主体的抗争越是艰难,所体现的精神就越强大,悲剧的审美价值也越高。‎ C.在歌剧《白毛女》的演出过程中,扮演地主黄世仁的演员被激愤的观众殴打的事件,说明人们的实际道德评价是不可能把现实的悲剧作为审美对象的。‎ D.人们之所以喜欢欣赏悲剧,是因为悲剧会引起人的悲伤、畏惧、怜悯,使人在强烈的痛苦中获得一种快感,所谓“以悲为美”的意思全在于此。‎ 答案 D 解析 “所谓 以悲为美 的意思全在于此”不正确,参见第三段末句“以悲为美”的论述。‎ ‎[2年全国模拟重组]‎ 一、[2017·湖南五市十校联考]阅读下面的文字,完成1~3题。‎ 在亚洲的很多国家,人们发现白桦、杜仲和杨梅树皮的衍生物还具有抗癌的功效。另外,树皮也可以作为大气污染的生物指示器。最近,澳大利亚和美国组成的研究团队对90种树木的树皮特征进行了测定。研究负责人罗塞尔教授指出,他们发现树皮与木质部和叶片存在有趣的关联,如木材密度与内树皮密度成正比,树皮含水量与木质部导管直径成正比,与叶片大小也成正比;树皮厚度增大可以很明显地增大树干的结实程度与储水能力,这是人们首次较为全面地认识树皮的生理生态学意义。‎ 那么,关于树皮,我们到底知道它的哪些秘密呢?‎ 树皮,生命的守护者。‎ 在易着火的生态系统中,较厚的树皮可以保护树干,如热带稀树大草原(易发生火灾的区域)的树种相对于热带森林树种往往拥有相对较厚的树皮。但也有例外,如易脱落树皮的桉树有较薄的树皮,但其树皮能够进行明显的光合作用,从而补偿了树皮薄带来的抗火能力较差的劣势。‎ 一些树的树皮长有尖刺,用来保护树皮不受动物的侵害。但树皮除了通过长刺和变硬来吓跑小动物之外,有些树还会像动物那样生有警戒色的斑纹,如笔筒树树皮上像人眼的环;紫薇树带有斑点的树皮离远看来像猎豹,猛一看还真让人有点生畏。‎ 树皮光合作用,关键时候才“出手”。‎ 那些不能进行光合作用的树皮,主要对树干起到支撑和保护的作用,而能够进行光合作用的树皮则在植物的呼吸、养分输送和碳同化吸收中发挥作用。这些生理学功能会随着树体的增大而发生明显的改变,即使对同一树种的不同部位也是这样,如山仓子和阴香从基部萌生的嫩枝树皮光滑且呈现绿色,相比于成熟的树皮光合作用的效率更高。对于成熟的红椎,树皮能够进行微弱的光合作用,但当进入一定年龄的时候,基本上丧失了光合能力,树皮的形态也发生了很大的变化,呈片状脱落。有研究发现红椎树皮光合能力的终止增强了叶片的光合能力,或许这暗示着树皮与叶片生理上的协同调节。‎ 树皮,与环境相契共生。‎ 在对中国北部樟子松树皮的研究中,科学家发现由于环境不同树皮也存在明显差异。如因为所处环境的差异,樟子松分为褐皮、黄皮、红皮和黑皮四种主要类型,其中黄皮的树皮率(指树皮的体积占树皮和木质部总和的比例)最小。‎ 随着树体的增大,几乎所有树种的树皮率都会减少。而随着树木高度的增加,一些树种的树皮率会增加。与此同时,树皮也会影响环境。通过树干光合作用或呼吸作用,树皮会改变气象微环境。‎ 树皮,彩虹般的美丽。‎ 在墨西哥热带干旱森林,罗塞尔教授发现了很多五颜六色的树皮。彩虹桉,顾名思义,就是树皮如彩虹一样绚丽多彩的一种桉树。这种桉树有一个特性,当缺水或养分不足时,色彩就会变得暗淡;而当水分充足时,它就会毫不吝啬自己的姿色,向人们展示大自然的美妙。‎ 作为生态环境中不可或缺的一员,树皮还在不经意间发挥其他的生物功能,如水杉的树皮可以为蚂蚁提供安乐窝。可见,树皮多姿多彩的功能和价值是大自然的慷慨馈赠,值得人们不断探索和发现。‎ ‎(选自《百科知识》,有删改)‎ ‎1.对于“树皮与木质部和叶片存在有趣的关联”的分析,不符合原文意思的一项是(  )‎ A.内树皮密度与木材密度成正比例关系。‎ B.树皮含水量与叶片大小成正比,与木质部导管直径也成正比。‎ C.树皮具有抗癌的功效,也可以作为大气污染的生物指示器。‎ D.树皮厚度增大,可以增大树干的结实程度与储水能力。‎ 答案 C 解析 C项原文首句说“白桦、杜仲和杨梅树皮的衍生物还具有抗癌的功效”,并不代表所有的树皮都具有抗癌的功效。另外,C项本身不是对“树皮与木质部和叶片存在有趣的关联”的分析。‎ ‎2.下列表述,不符合原文意思的一项是(  )‎ A.笔筒树树皮上像人眼的环,紫薇树树皮离远看来像猎豹的斑点,其他树树皮上的尖刺,也能保护树皮不受动物的侵害。‎ B.樟子松分为褐皮、黄皮、红皮和黑皮四种主要类型,这种分类主要是其所处环境差异造成的。‎ C.红椎树皮光合能力的终止增强了叶片的光合能力,这可能暗示着树皮与叶片生理上的协同调节。‎ D.较厚的树皮可以保护树干免遭火灾,而较薄的树皮抗火能力较差,起不到保护树干的作用。‎ 答案 D 解析 原文第四段末尾说,有些较薄的“树皮能够进行明显的光合作用,从而补偿了树皮薄带来的抗火能力较差的劣势”。‎ ‎3.根据原文内容,下列理解和分析不正确的一项是(  )‎ A.能进行光合作用的树皮,对植物的呼吸有作用;不能进行光合作用的树皮,对于树干的保护有作用。‎ B.树皮率会随着树体的增大而减少,随着树木高度的增加而增加。‎ C.由于环境不同树皮也存在明显差异,在一定条件下,树皮又反过来影响环境。如通过树干光合作用改变气象微环境。‎ D.树皮除自身的生理生态功能之外,还发挥着其他的生物功能,树皮的秘密奥妙无穷,我们了解的还只是冰山一角。‎ 答案 B 解析 B项说法不严密。原文倒数第四段表述为“随着树体的增大,几乎所有树种的树皮率都会减少。而随着树木高度的增加,一些树种的树皮率会增加”,“几乎所有”和“一些树种”不能代表全部。‎ 二、[2017·福建联考]阅读下面的文字,完成1~3题。‎ 陋劣之中有至好——谈审丑快感 童庆炳 人们不难发现,在艺术创作中,丑成了一个重要的描写对象。照理,丑的东西只能使人厌恶,何以某些丑的东西进入艺术作品,倒给人们带来美感呢?“嘴甜心苦,两面三刀,上头一脸笑,脚下使绊子,明是一盆火,暗是一把刀”的王熙凤,其卑污的灵魂难道不让人恨得咬牙切齿吗?为什么读者在手捧《红楼梦》之际,又会产生“骂凤姐,恨凤姐,不见凤姐想凤姐”的心理呢?或者说生活丑怎么会变成艺术美呢?这实际上是一个审丑快感问题。‎ 丑作为一种艺术内容是怎样引起我们的快感的呢?这可以从以下三点加以说明:第一,丑是一种背景,用来衬托美的丽质。无论在生活中还是在艺术中,美的东西都不是孤立的存在。美的东西总是同丑的东西相比较而存在、相斗争而发展的。心理学的实验证明,对比效应是人感知事物的一大特征。高个子在矮个子旁边显得更高;白色在黑色包围中显得更白;健美置身于病态旁显得更健美;崇高与卑劣相比较显得更崇高;美与丑相对照显得更美。‎ 在我国的古代诗歌中,把丑作为一种背景,用以衬托美的写法,是屡见不鲜的。如刘禹锡的“沉舟侧畔千帆过,病树前头万木春”,以“沉舟”的死静来衬托“千帆”竞发的生动,以“病树”的病态来衬托“万木”争春的壮美。生动因有死静相与共而显得更生动,壮美因有病态相陪伴而显得壮美。这就是说,丑还是丑,但丑作为陪衬而成为美的条件,成为美的一个源泉。这也就是艺术家在自己的作品中把阴影掺入光明,把滑稽丑怪置于优美崇高之旁的一个重要原因。‎ 第二,丑往往比美更能揭示内在的真实,更能激发深刻的美感。丑的对象,其外在的形态对审美感官具有阻拒性,它不会顺利地给人们带来快感。但它却具有一种吸引力,而且促使人们从对象的外在表象中解脱出来,而去关注与追寻对象内部蕴含的意味,这样,丑的对象就给人带来一种更深刻、更震撼人心的美感。而艺术家也就利用“丑”这个特性,用以表现人们的内心世界里最深邃的东西。‎ 法国著名雕塑家罗丹有一件《欧米哀尔》的雕塑,所表现的是一个年老的风尘女子。‎ 面容的憔悴,肌肉的萎缩,皮肤的皱纹,表情的悲哀,都令人感到这是一位再丑不过的女人。然而这件雕塑却比无数的美女雕塑更成功。有的评论家在《欧米哀尔》面前惊呼:“啊,丑得如此精美!”实际上,罗丹的这件雕塑吸引人之处,是超越外在形态的更加深刻的美学意义:罪恶的社会把一个人美好的青春和幸福给毁灭了。正是这种内在蕴含使《欧米哀尔》显得精美,并给人带来了心灵的震动和深刻的美感。‎ 第三,对丑的揭露、谴责和批判,是令人痛快的。艺术既是对客观的反映,同时又是主体的创造。在这种创造中,艺术家的审美理想是一道美的光亮,它可以刺穿丑,使其丑态毕露而被征服。对丑的征服会使人产生胜利感而激起愉悦之情。车尔尼雪夫斯基早就说过:“在滑稽中丑态是使人不快的;但是,我们是这样明察,以至能够了解丑之为丑,那是一件愉快的事情。我们既然嘲笑丑态,就比它高明。”这种以美裁判丑从而激起快感的说法,是解释审丑快感的一种最流行最重要的说法。这是因为,“丑的现象本身不会令人高兴。但是痛斥他却真是一件快事啊!这是对丑的谴责,在美的理想之光照射下使人目眩,让丑的劣迹在美面前原形毕露”。在《红楼梦》中,王熙凤的所作所为,只能引起我们的厌恶。但对她的心狠手辣、无恶不作的揭露、谴责,却使人无比痛快。由此可见,艺术创作中以美裁判丑,是审丑快感产生的一个重要原因。‎ ‎(本文有删改)‎ ‎1.下列有关“审丑快感”的表述,不正确的一项是(  )‎ A.心理学实验证明,对比效应是人们感知事物的一大特征,审丑快感的产生离不开这种对比效应;从这个意义上讲,丑可以成为美的一个源泉。‎ B.丑的对象的外在形态虽然不会顺利地给人们带来快感,但它能促使人们关注与追寻对象内部蕴含的意味,并给人们带来更深刻、更震撼人心的美感。‎ C.艺术源于生活而又高于生活,生活中丑的东西进入艺术作品中能够给人们带来“审丑快感”,这种快感完全取决于艺术家的主体创造。‎ D.在艺术家审美理想之光的照射下,丑的东西会原形毕露,这种对丑的征服会让人产生胜利感,从而也就产生了审丑快感。‎ 答案 C 解析 C项说法不对。原文最后一段前面“艺术既是对客观的反映,同时又是主体的创造”。‎ ‎2.下列各项中,不适合用来证明“丑是一种背景,用来衬托美的丽质”这一观点的一项是(  )‎ A.求美则不得美,不求美则美矣。求丑则不得丑,不求丑则有丑矣。(《淮南子》)‎ B.癫狂柳絮随风舞,轻薄桃花逐水流。(杜甫《漫兴·其五》)‎ C.贵珠出乎贱蚌,美玉出乎丑璞。(葛洪《抱朴子》)‎ D.滑稽丑怪作为崇高优美的配角和对照,要算是大自然给予艺术的最丰富的源泉。(雨果《〈克伦威尔〉序》)‎ 答案 B 解析 柳絮、桃花都是比喻乘风乱舞、随波逐流的势利小人;这两句都是写“丑”,表现了世人对现实的不满。‎ ‎3.根据原文内容,下列理解和分析不正确的一项是(  )‎ A.在真正的艺术作品中,丑的对象能转化为美,让人产生兴味盎然、无比震撼的审丑快感,这种快感是美的对象所不能给予的。‎ B.美与丑是对立中的存在,两者是相反相成的。离开丑孤立地去求美,得不到美;相反,若能把丑置于美之旁,那么美就在对比中显露出来了。‎ C.罗丹的《欧米哀尔》以丑的、病态的形象为题材,其目的不是为了展览丑,而是为了激起欣赏者心灵的震动和深刻的美感。‎ D.“以美裁判丑从而激起快感”之所以成为美学界一种最流行最重要的说法,是因为它较为合理地解释了艺术创作中审丑快感产生的原因。‎ 答案 A 解析 “这种快感是美的对象所不能给予的”理解不当,“审美快感”与“审丑快感”是非常复杂的美学问题,二者很难区分优劣。‎ 三、[2017·清远模拟]阅读下面的文字,完成1~3题。‎ 中国文化的美丽精神 宗白华 诗哲泰戈尔曾说过:“世界上还有什么事情,比中国文化的美丽精神更值得宝贵的?中国文化使人民喜爱现实世界,爱护备至,却又不致现实得不近情理!他们已本能地找到了事物的旋律的秘密。不是科学权力的秘密,而是表现方法的秘密。这是极其伟大的一种天赋。”‎ 他的这几句话里,包含着极精深的观察与意见,值得我们细加思考。‎ 先谈“中国人本能地找到了事物的旋律的秘密”。东西方古代哲人,都曾仰观俯察探求宇宙的秘密。但希腊及西洋近代哲人倾向于拿逻辑的推理、数学的演绎、物理学的考察去把握宇宙间质力推移的规律,一方面满足理知了解的需要,另一方面导引西洋人去控制物力,发明机械,造福民生。西洋思想最后所获得的是科学权力的秘密。中国古代哲人却是拿着“默而识之”的观照态度,去体验宇宙间生生不已的节奏。即泰戈尔所谓旋律的秘密。《论语》上载:子曰:“天何言哉?四时行焉,百物生焉,天何言哉!”四时的运行,生育万物,对我们展示着天地创造性的旋律的秘密。一切在此中生长流动,具有节奏与和谐。老子也从他高超严冷的眼里观照着世界的旋律,他说:“致虚极,守静笃,万物并作,吾以观复!”活泼的庄子也说他“静而与阴同德,动而与阳同波”,他把他的精神生命体合于自然的旋律。荀子歌颂着天地的节奏:“列星随旋,日月递照,四时代御,阴阳大化,风雨博施,万物各得其和而生,各得其养而成。”‎ 中国古代哲人找到了宇宙旋律的秘密,并且把这获得的至宝,渗透进我们的现实生活,使我们生活在礼与乐里,创造着社会的秩序与和谐。我们又把这旋律装饰到我们日用器皿上,使形而下之器表现着形而上之道(即生命的旋律)。中国古代艺术特色表现在日用器皿的各种图案花纹里,而中国最光荣的绘画艺术,也还是从商周铜器图案、汉代砖瓦花纹里脱胎出来的呢!‎ 再谈“中国人喜爱现实世界,爱护备至,却又不致现实得不近情理”。在新石器时代,我们制作了玉质的日用器皿,后来把它们作为我们政治上、社会上及精神人格上美丽的象征物。在青铜器时代我们竭尽当时的艺术技能把日用器皿,如烹饪的鼎、饮酒的角等等,制精制美,使它们成了天地境界的象征。我们赋予最现实的器具以崇高的意义,优美的形式,使它们成了天地境界的象征。我们赋予最现实的器具以崇高的意义、优美的形式,使它们不仅仅是我们役使的工具,而且是可以同我们对语、同我们情思往还的艺术境界。‎ 但我们也曾得到过科学权力的秘密。火药同指南针这两项发明到了西洋人手里,成就了他们控制世界的权力——路上霸权与海上霸权,中国自己倒成了这霸权的牺牲品。‎ 我们发明火药,却用它来制造奇巧美丽的烟火,使人们在新年里享受平民式的欢乐。我们发明指南针,并不曾向海上获取霸权,却让风水先生勘定我们庙堂、居宅及坟墓的方位与走向,使我们生活中顶重要的“住”,能够选择优美适宜的自然环境。‎ 中华民族很早发现了宇宙旋律及生命节奏的秘密,以和平的音乐的心境爱护现实,美化现实,却轻视了科学征服自然的权力,使我们不能摆脱贫弱的地位,在生存竞争剧烈的时代,受人欺侮,我们的灵魂粗野了,卑鄙了,怯懦了,也现实得不近情理了。中国文化的美丽精神正面临着极大的问题。‎ ‎(选自《艺境》,1946年,有删节)‎ ‎1.关于“中国人本能地找到了事物的旋律的秘密”,下列表述不符合文意的一项是(  )‎ A.中国古代哲人仰观俯察宇宙秘密的目的,并不像西方哲人那样想获得“科学权力的秘密”。‎ B.发现宇宙秘密的能力是中国人所独有的,被泰戈尔认为是中国人的一种极其伟大的天赋。‎ C.中国古代哲人找到的“事物的旋律的秘密”不是科学权力的秘密,而是表现方法的秘密。‎ D.中国古代哲人把找到的宇宙旋律的秘密渗透进现实生活,创造着社会的秩序与和谐。‎ 答案 B 解析 “发现宇宙秘密的能力是中国人所独有的”不符合文意,参见原文第三段。‎ ‎2.下列各项不能体现“中国人喜爱现实世界,爱护备至,却又不致现实得不近情理”的一项是(  )‎ A.四时运行和万物生育,展示着天地创造性的旋律的秘密,中国人本能地找到了这种秘密。‎ B.新石器时代制作了玉质器皿,后来我们把它们作为政治、社会和精神人格上美丽的象征物。‎ C.我们将崇高意义与优美形式赋予现实的器具,使它们已不再仅仅是被役使的工具。‎ D.我们没有用火药和指南针来获取霸权,而用来享受新年的平民式的欢乐和适宜的自然环境。‎ 答案 A 解析 属于对“中国人本能地找到了事物的旋律的秘密”的阐述,本小题参见倒数第四段。‎ ‎3.下列理解和分析,符合原文内容的一项是(  )‎ A.西方哲人探求宇宙万物的目的是了解与控制,因此他们根本就体会不到探求过程中的真谛与乐趣。‎ B.庄子把他的精神生命体合于自然的旋律,而荀子歌颂天地的节奏,他们对宇宙旋律的体验大相径庭。‎ C.中国文化的美丽精神体现在“中国文化使人民喜爱现实世界,爱护备至,却又不致现实得不近情理”。‎ D.在重视科学征服自然的权力的同时,我们更应坚持中国文化的美丽精神,否则,我们便不能摆脱贫弱的地位。‎ 答案 C 解析 A项“因此他们根本就体会不到探求过程中的真谛与乐趣”属无中生有;B项“他们对宇宙旋律的体验大相径庭”与原文不符;D项“更应坚持中国文化的美丽精神”并不是我们“摆脱贫弱的地位”的途径。‎ 四、[2017·江淮十校联考]阅读下面的文字,完成1~3题。‎ 用知识产权撬动知识创新 滕朝阳 ‎①回溯人类文明史,专利制度促进了创新知识迅速公共化,同时也为技术和市场的结合提供了制度保证。从专利制度诞生早于工业革命发生的时序来看,二者之间应该存在极其重要的因果关联。时至今日,包括专利权在内的知识产权在世界范围内普遍受到法律保护。为树立尊重知识、崇尚科学和保护知识产权的意识,营造鼓励知识创新和保护知识产权的法律环境,世界知识产权组织将每年的4月26日定为“世界知识产权日”。‎ ‎②与西方国家相比,我国的知识产权保护工作起步较晚。知识产权进入人们的观念,被确立为一种制度在经济社会生活中发挥作用,不过是近三十年来的事。我国已经建立了门类比较齐全的知识产权法律体系,取得的知识产权成果也令世界刮目相看。但侵犯知识产权的行为还不少,“山寨产品”还受到不少消费者认可,尊重知识、尊重创造还没有深入人心,保护知识产权就是保护民族创新能力还没有形成广泛共识和自觉行动。‎ ‎③知识创新是充满高风险、需要高投入、具有高难度的智力活动。很多人能熟练应用旧知识,只有极少数人能在知识存量的基础上发展出新知识,而知识创新的成果恰恰是人类福利增进和社会进步的源泉。只有尊重、保护创造性智力成果,才能激发人们的创新热情,并为持续创新奠定基础。相反,如果人人都可以搭便车,从事知识创新的人就无法得到足够的补偿与激励,持续创新就会丧失动力。‎ ‎④‎ 漠视知识产权,就将被假冒伪劣包围,在这样的环境中生活,很难有获得感和幸福感。但人们对知识产权的重视,还没有达到应有的广度和深度。事实上,即使是权利人,对知识产权的保护也往往不够重视。作家莫言在一篇文章中,提到几年前一群作家跟一家网站打版权官司:“按说这是一件正大光明的事情,但是很多人说风凉话:有这么多的国家大事,你们不去关心;有这么多的弱势群体,你们不去关怀;你们这帮作家——人类灵魂的工程师,就关心自己的版税,还诉诸公堂,占用媒体宝贵的版面和时间。”这是一种非常典型的情境,从一个侧面反映了对知识产权的态度。‎ ‎⑤我们在保护知识产权方面取得了明显进步,但还很不尽如人意。有的企业负责人坦言“已经不愿意申请专利了”,因为“不申请专利还好,一申请专利反而被剽窃得更快”,于是“核心技术方面绝不再向外透露”。在专利制度建立之前,技术进步都是由技术机密所驱动,而这种传承方式会极大地阻碍了人们之间的知识共享。知识创新成果的产权保护不力,就会回到“机密保护”的状态,这既会抑制自己的创新,也不利于引进最好的技术。‎ ‎⑥在互联网时代,知识产权保护面临更复杂的环境,相关法律制度还有待于进一步完善。在根本上,还是要确立这样一种理念:侵犯知识产权不仅可耻,而且违法!只有保护好创造性智力成果,投身知识创新的人才会越来越多,创新驱动才会变成广阔的现实,社会才能获得不竭的前进动力。‎ ‎(2016年第8期《半月谈》,有删节)‎ ‎1.下列关于原文内容的表述,不正确的一项是(  )‎ A.从专利制度诞生早于工业革命发生的时序来看,知识产权与知识创新之间应该存在极其重要的因果关联。‎ B.作家莫言提到的几年前一群作家跟一家网站打版权官司的这一事例,就是一种非常典型的权利人重视知识产权的情境。‎ C.要激发人们的创新热情,并为持续创新奠定基础,就必须尊重、保护创造性智力成果。如果人人都不尊重、保护创造性智力成果,持续创新就会丧失动力。‎ D.如果知识创新成果的产权保护不力,创新成果就会回到“机密保护”的状态,这既会抑制自己的创新,也不利于引进最好的技术。‎ 答案 A 解析 A项,见第一段文字,关键是对第二句话中的“二者之间”的理解,“二者之间”根据前后文意,且承前指代,应是“专利制度(或知识产权保护)与知识创新”。B项,一群作家跟一家网站打版权官司的事例,反映的恰恰是权利人(即作家)对知识产权的重视。C项,将“只有……,才……”这一句式的表达变成了“要……,就必须……”的句式进行表达。D项,按照文意,这里“回到 机密保护 的状态”的应是知识创新成果,而不是“知识创新成果的产权”。因而,此处理解是正确的。‎ ‎2.下列理解和分析,不符合原文意思的一项是(  )‎ A.从人类文明史来看,专利制度促进了创新知识迅速公共化,同时也给技术和市场的结合提供了制度保证。为此,世界知识产权组织才将4月26日定为“世界知识产权日”。‎ B.知识创新这种智力活动充满高风险、需要高投入、具有高难度,而知识创新的成果又恰恰是人类福利增进和社会进步的源泉。‎ C.在保护知识产权的专利制度建立之前,技术进步都是由技术机密所驱动,而这种传承方式会极大地阻碍人类知识的共享。‎ D.近三十年来,知识产权正在逐渐进入中国人的观念里,并被确立为一种制度在我国的经济社会生活中发挥着作用。‎ 答案 A 解析 A项,见原文第①段。“为此”的“此”指的是“为树立尊重知识、崇尚科学和保护知识产权的意识,营造鼓励知识创新和保护知识产权的法律环境”,而非“从人类文明史来看,专利制度促进了创新知识迅速公共化,同时也给技术和市场的结合提供了制度保证”。B项,此项涉及对整个第③段语意的整体把握与理解。C项,见第⑤段,“专利制度”之前加“保护知识产权”只是让语意更明晰,“人们之间的知识共享”与“人类知识的共享”之间等义。D项,见第②段,变动语序,但语意未改变。‎ ‎3.根据原文内容,下列说法不正确的一项是(  )‎ A.我国有的企业负责人坦言“已经不愿意申请专利了”,是因为“不申请专利还好,一申请专利反而被剽窃得更快”,这说明我们在保护知识产权方面还很不尽如人意。‎ B.如果漠视知识产权,人们就将在被假冒伪劣包围的环境中生活,因而也很难有获得感和幸福感。‎ C.“山寨产品”在我国受到不少消费者认可,这说明尊重知识、尊重创造还没有深入人心,保护知识产权还没有形成广泛共识和自觉行动。‎ D.时至今日,包括专利权在内的知识产权在世界范围内普遍受到法律保护。我国也已经建立了门类比较齐全的知识产权法律体系,知识产权已深入人们的观念。‎ 答案 D 解析 A项,见第⑤段第一句“我们在保护知识产权方面取得了明显进步,但还很不尽如人意”,一个“但”字,说明句意的重点在“还很不尽如人意”上,后面是举例进行说明。B项,见第④段,“就将”说明此句是一假设复句,这里前一分句加上因果连词“如果”不影响语意。“在这样的环境中生活”一句,本身就含有条件、原因之意,故加上“因而”不改变原意。C项,见第②段的末句,“ 山寨产品 还受到不少消费者认可”是一种社会现象,后面两句是对这种现象的引申分析即指出原因。D项,“知识产权已深入人们的观念”错误。见原文第②段“我国也已经建立了门类比较齐全的知识产权法律体系”,但不能说明“知识产权已深入人们的观念”,仅仅说明“知识产权进入人们的观念”。‎ 五、[2017·四川质检]阅读下面的文字,完成1~3题。‎ 传承传统文化要有新思维 池建宇 文化是一个民族的灵魂和血脉。中华优秀传统文化是中华民族的集体记忆和精神家园,是我们民族的生命力、凝聚力的重要载体。根据时代的新进步、新进展,对中华优秀传统文化的内涵不断加以补充、拓展、完善,增强其影响力和感召力,对增强民族自尊心,使全国人民始终保持奋发有为的精神状态具有重要意义。‎ 在弘扬中华优秀传统文化时,必须贴近生活、贴近群众、贴近人民。要充分发挥好传统文化宣传者、帮助者、创造者的作用,努力为人民提供最需要的文化产品和文化服务,这样才能真正赢得群众,达到宣传、教育、引导群众的目的。要深入社区、深入群众,要使中华文化走入社区和社会,改造和发展具有浓郁民族特色的民间风俗礼仪,开展丰富多样、健康有益的民间民俗文化活动,让传统文化不仅保存在纸上、珍藏在博物馆里,而且真正走进人们的日常生活,保持中华民族共有的精神记忆和文化传承。‎ 传统文化不等于教条、枯燥乏味,而是恰恰要更加鲜活,更加实在。做好新时期的宣传思想文化工作,要根据时代的变化和人民群众的欣赏特点,积极探索宣传文化活动形式创新,提高传播艺术水准,善于运用先进技术手段和现代传播技巧,特别是互联网和多媒体新技术,改造文化传统生产经营传播模式,实现题材体裁、风格流派、表现手法的多样化,努力实现艺术精湛、制作精美。中华民族五千年的文明积累了极为丰富的文化遗产,要加强规划,加大投入,特别是运用现代科技手段,认真做好文化典籍整理工作,切实保护我们的文化瑰宝;在新时代,通过多种多样的科技手段,让深藏在博物馆、科研机构里的传统文化瑰宝走出来,让更多的人真正接触、了解、学习。让群众在参与活动中受到熏陶,启迪人生,陶冶情操。同样,也使更多的人了解传统文化,喜爱传统文化,成为优秀传统文化的承载者和传播者。‎ 中华传统文化蕴藏于那些经典文本之中,具体化为各种传统文化知识,以各种技术技能存在于现代人的日常生活中。五千年中华文明积淀下来的文化,塑造了中国人的精神品格和精神世界,是中华民族文化的基石,也是社会进步的必要元素。对于弘扬中华优秀传统文化,基础教育有着不可替代的作用。因此,在基础教育里面,要将我国最美好的传统文化融入每个人的血液里。要使中华优秀传统文化成为小学、中学的重要课程,切实做好中小学生的传统文化教育,各学科课程都要结合学科特点融入中华优秀传统文化的内容。教师要加深对优秀传统文化的学习、理解,从而对学生起到表率作用。此外,还可以在全国中小学生中广泛开展典籍诵读等多种丰富多彩的学生活动,让阅读传统经典成为习惯。‎ 在传承和弘扬中华优秀传统文化的同时,要坚持使用先进正派、健康向上的文化去抵制落后腐朽、低俗庸俗的文化糟粕。坚持古为今用、推陈出新,有鉴别地加以对待,有扬弃地予以继承,既不能片面地讲厚古薄今,也不能片面地讲厚今薄古。‎ ‎(摘自《光明日报》,有删改)‎ ‎1.下列关于原文内容的表述,不正确的一项是(  )‎ A.在弘扬中华优秀传统文化时,要让传统文化走出博物馆,深入群众中去。‎ B.在弘扬中华优秀传统文化时,要让教条、枯燥的传统文化变得鲜活、实在。‎ C.在弘扬中华优秀传统文化时,要从娃娃抓起,在中小学中进行推广和普及。‎ D.在弘扬中华优秀传统文化时,要善于运用现代的传播手段,要有现代思维。‎ 答案 D 解析 本题考查理解文意,筛选并整合文中信息的能力。D项,“要有现代思维”过于绝对,原文是说“传承传统文化要有新思维”,而不是“现代思维”,且强调“既不能片面地讲厚古薄今,也不能片面地讲厚今薄古”。‎ ‎2.下列理解,不符合原文意思的一项是(  )‎ A.随着社会的进步,对传统文化的内涵也就需要不断地补充完善,以适应时代的发展。‎ B.要传承文化就要让文化真正地走进人们的日常生活,就必须深入社区、深入群众。‎ C.要传承文化首先要保护好文化,这就要求必须运用现代的科技手段来整理文化典籍。‎ D.基础教育对传承传统文化将起到至关重要的作用,所以要让传统文化成为重要课程。‎ 答案 C 解析 本题考查归纳内容要点,概括中心意思的能力。C项,“必须运用现代的科技手段来整理文化典籍”过于绝对,文中这一做法是针对“使传统文化更加鲜活”提出的合理建议,并不是必要条件,且文章强调“传承传统文化要有新思维”,而不是强调“保护好文化”。‎ ‎3.根据原文内容,下列理解和分析不正确的一项是(  )‎ A.积极地创新宣传文化活动的形式,有鉴别地继承传统文化,增强其影响力和感召力将具有重要的意义。‎ B.传承传统文化要开展丰富多样、健康有益的文化活动,要让群众参与到活动中,成为承载者和传播者。‎ C.传承传统文化离不开中小学的基础教育,学校要开展各类活动,教师也要加强对传统文化的学习理解。‎ D.传承和弘扬中华传统文化可以增强民族的自尊心,所以要坚持运用马列主义等先进正派的思想来传播。‎ 答案 A 解析 本题考查分析概括作者在文中的观点态度的能力。A项,原文第一段中为“对中华优秀传统文化的内涵不断加以补充、拓展、完善,增强其影响力和感召力……具有重要意义”。‎ 滚动提升训练(五)‎ ‎  时间:60分钟   满分:73分 一、现代文阅读(18分,每小题3分)‎ ‎(一)[2017·衡水中学摸底]阅读下面的文字,完成1~3题。‎ 灾异与人事 葛剑雄 中国自古以农立国,对水旱灾害相当敏感。加上中国东部主要农业区受季风气候影响,大小灾害频繁,成为统治者和民众经常性的威胁。现实的需要使中国古代对天文、气象、物候的记载和研究相当重视,并注意考察人类活动与自然变化的关系,形成了一些独特的观念,其中之一即天人感应或天人合一。‎ 时下流行的看法都将天人合一解释为人类与自然的和谐相处,更多的是反映了时人的愿望,是对传统观念一种积极的、但也是实用主义的解释。尽管原始的天人合一观念的确包含了这样的内容,却并不是它的主体。所谓“天人合一”,是指天意决定人事,而天意是通过天象或灾异来显示的。君主是“天子”,由天意确定,也代表天命。所以君主如有失德,或治理不当,或人事有悖于天意,必定会受到天象的警告或灾异的惩罚。正因为如此,从最古老的史书开始,天象和灾异都是不可或缺的记载。《二十四史》中大多有《五行志》《天文志》《灾异志》,但所记内容无不与朝代兴衰、天下治乱、君主贤愚、大臣忠奸相一致。凡国之将兴,天子圣明,大臣贤能,则风调雨顺,紫气东来,吉星高照;反之则灾异频仍,天象错乱。‎ 在这种观念的主导下,一旦出现罕见的天象或异常的气候,如日蚀、太阳黑子、流星、陨石、星宿异位、地震、山崩、水旱灾害等,皇帝就要换上素色服装,不吃荤腥辛辣,不近女色,迁居偏僻清静的场所,反省自己的过失,征求臣民的意见。有的皇帝还会下“罪己诏”,公开承认错误,宣布改弦更张的政策。有的虽没有具体措施,却会请求上天千万不要与天下百姓为难,一切罪责由自己担当。为了通达上天,感动诸神,往往还要在正常的祭祀典礼以外,按需要举办各类额外的祭祀或祈求仪式。‎ 这样做一般不会错,因为皇帝总有失德之处,政府的举措不会十全十美,臣民们平时不敢说的话,不便提的意见也可趁机上达天听。即使是昏君暴君,在上天的警告面前也不得不有所收敛。在专制集权体制下,只有这样的机会才能给最高统治者一点警诫。何况举办这些活动一般花不了多少人力物力,一旦渡过难关,就能博得万民称颂,名垂史册。万一无效,也是天威莫测,天意难违,君臣都已尽了人事。对处理人与自然的关系也不无积极作用,因为人能敬畏天命天意,总会格外小心谨慎。‎ 不过“天人合一”也是一把双刃剑,奸臣小人也能利用天象灾异做手脚。因为对天象的解释在人,也没有什么标准答案,所以大权在握的人完全可以随心所欲将矛头指向政敌异己,或者趁机按自己的意愿改变政策。正人君子和书呆子往往泥古不化,只知援引古代圣君贤臣的先例,只会侈谈修身进德,不屑采取具体措施,或者不知如何应对灾情。极端的做法就是听天由命,毫无作为,以为只要绝对敬畏,专修人事,灾害不除自灭。如唐玄宗时发生严重蝗灾,一部分大臣的意见竟然是不应消灭,只能听任蝗虫自生自灭,才符合天意,皇帝只要深刻自省,定能得到上天的宽恕。幸而唐玄宗听从宰相姚崇的建议,下令灭蝗,才没有造成更大的灾情。‎ 现在面对异常气候和严重灾情,自然又会想到天人合一的观念。我希望在运用和阐述时能实事求是,尊重历史,采取辩证而积极的态度。‎ ‎1.下列关于原文内容的表述,不正确的一项是(  )‎ A.以农立国的特点,使中国对水旱灾害相当敏感,中国东部主要农业区受季风气候影响,大小灾害频繁,成为统治者和民众经常性的威胁。‎ B.将天人合一解释为人类与自然的和谐相处,是一种积极的,也是实用主义的解释,这一解释反映了人们的愿望。‎ C.天象和灾异在中国的古代史书里都有所记载,如“二十四史”中《五行志》《天文志》《灾异志》所记内容与朝代兴衰、天下治乱、君主贤愚、大臣忠奸相一致。‎ D.天人合一的观念使得皇帝在出现罕见的天象或异常的气候时,往往会采取一些措施,反省自己的过失,征求臣民的意见。‎ 答案 B 解析 B项表述错误,原文第二段首句将“天人合一”解释为人与自然的和谐相处,是时下流行的说法,反映的是时人的看法。选项将意义扩大了。‎ ‎2.下列理解和分析,不符合原文意思的一项是(  )‎ A.中国“天人感应”或“天人合一”的观念的形成是因为中国以农立国,大小灾害频繁,需要考察人类活动与自然变化的关系。‎ B.天象或灾异显示天意,君主如有失德,或治理不当,或人事有悖于天意,也会受到天象的警告或灾异的惩罚。‎ C.在专制集权体制下,君主举行的祭祀或祈祷仪式能够给臣民们一些说话的机会,他们平时不敢说的话、不便提的意见也可趁机上达天听。‎ D.皇帝举办一些活动对处理人与自然的关系会有积极作用,因为人能敬畏天命天意,总会格外小心谨慎。‎ 答案 C 解析 C项“君主举行的祭祀或祈祷仪式”错误,从原文第三、四段看,臣民的意见能上达天听的应该是皇帝举行的其他的一些活动,如征求臣民的意见,下“罪己诏”等,而不是在祭祀等活动中。‎ ‎3.根据原文内容,下列理解和分析不正确的一项是(  )‎ A.天意决定人事是天人合一的基本意思,罕见的天象或异常的气候,如日食、流星、陨石、地震、山崩、水旱灾害等,都是天意对人的惩戒。‎ B.天人合一的观念,有它的积极作用,因为皇帝总有失德之处,政府的举措不会十全十美,即使是昏君暴君,在上天的警告面前也不得不有所收敛。‎ C.“天人合一”是一把双刃剑,奸臣小人可以利用天象灾异做手脚,而泥古不化的正人君子和书呆子往往在灾异面前不屑采取具体措施,或者不知如何应对灾情。‎ D.现在面对异常气候和严重灾情,自然又会想到天人合一的观念。作者希望在运用和阐述这一观念时能实事求是,尊重历史,采取辩证而积极的态度。‎ 答案 A 解析 A项“是天意对人的惩戒”错误,原文中罕见的天象或异常的气候的出现被古人认为是对统治者的惩戒。‎ ‎(二)[2017·哈尔滨六中月考]阅读下面的文字,完成4~6题。‎ 中国茶史 郑培凯 ‎①上古时代,茶在中国的植物图谱中已经出现,但是最早时,茶属于药品,或者属于菜蔬,一直到了唐代,随着茶叶的广泛种植和行销到了游牧民族地区,茶才正式成为中国人的日常饮用之物。这时候,陆羽创立了完整的茶叶科学体系,规范了饮用方法,包括提出了“茶有真香”的核心观念。根据一些古籍记载,战国时候,四川一带已经有饮用茶的习惯,秦灭蜀后,将之带出来,这里也是古茶树的发源地之一,符合“南方有嘉木”的说法。‎ ‎②到了三国魏晋时代,浙江和江南普遍种茶,饮茶人也增加,不再属于贵族专利,扩展到士大夫阶层,用以待客。当时也做成饼,叶片大汁不能黏合的就用米汤去黏合,喝的时候先去研磨,然后用沸水冲泡,还没有形成唐时那种复杂精美的饮用法。不过当时长江流域尤其是中下游,已经很普及饮用茶了,包括对器物和水都有讲究,但是饮用方式还比较古朴,茶处理如同蔬菜,放在水里煮喝,加各种香料与佐料,基本上就像蔬菜汤。属于实用阶段。唐之后,茶饮不再是实用主义,而是上升到了精神领域,这就成就了“饮茶之道”。茶之流行,除了交通和社会原因,也包括禅教大兴,在参禅过程中,为了提神不寐,也为了打坐,很多寺庙推广喝茶。当时禅宗影响很大,又影响到了民间,渗透特别广泛。‎ ‎③宋代茶书和茶人的世界首先在宫廷,当时宫廷的饮茶习惯非常发达,制作茶的技术比之唐代还要复杂。先是龙凤团,后来发展到石乳、白乳,再后来又有小龙团,以及各种密云龙、瑞云祥龙,越来越精细,层出不穷。当时的点茶手法是水和茶要用得恰当,比例均匀,否则就表面的沫饽不匀。还有斗茶法,没有水痕的最佳。为了达到效果,建立了一套新的系统,包括茶叶制作、茶叶击拂、茶叶品饮、器物优略,都形成了仪式和系统。‎ ‎④明代的士大夫阶层讲究品茶,与品茗环境和制茶都有很大联系,构成了一种发达的品茗体系,所以明朝成为中国茶的复兴时代。品茶的情趣,一方面是恢复了唐宋赏茗器的乐趣,对茶饮的程序和器物的雅洁再三致意,不因为明使用紫砂壶为主的相对简单的品茗体系,就不欣赏器物了,不对茶器物有追求了。另一方面,着重性灵世界,追求品茶所带来的心灵的修养的提升,期待有和谐之境界。‎ ‎⑤清代基本上延续了明朝的饮茶方式,有两件事情值得一提。一是茶碗越来越少,到了最后就成了基本使用青花杯,或者白瓷杯,紫砂壶成了最主要的泡茶工具;二是福建工夫茶的出现,导致了小紫砂壶的流行,这都是明清的茶事重点。但是随着清中期后民生的凋敝,整个的品茗雅趣开始走向没落,走了下坡路。尤其是1890年之后,基本上没有人有心思提及品茗雅事了。这之后,战乱频繁,革命事起,品茗之趣长期无人提及,结果现在很多中国人觉得茶道是日本的国粹,与中国文化无关,这也是历史失落太久的缘故。大多数中国百姓用大杯冲泡茶,倒是也符合质朴之道。‎ ‎(选自《醒狮国学》)‎ ‎4.下列关于原文内容的表述,不正确的一项是(  )‎ A.茶在上古时代的中国植物图谱中已经出现,古籍记载的战国时期四川一带已经有了饮用茶的习惯等可以说明蜀地是古茶树的发源地之一。‎ B.三国魏晋时期在长江中下游地区已经普遍饮茶,随着茶叶的广泛种植以及行销到游牧民族等,茶在唐朝时期真正成为国人日常饮用之物。‎ C.宋代宫廷的点茶和斗茶是烹茶的方式,但因有着一定的系统和仪式而成为精神领域的享受,这说明整个宋代的茶书和茶人的世界在宫廷。‎ D.清代中期后,民生凋敝,整个品茗雅趣走向没落,基本没人提及品茗雅趣了,随着时间的推移,现在很多中国人认为茶道是日本的国粹。‎ 答案 C 解析 扩大范围。“整个宋代的茶书和茶人的世界在宫廷”错误,文中第三段首句有“宋代茶书和茶人的世界首先在宫廷”,可以判断出随后的茶书、茶人的世界应该扩大到整个社会。‎ ‎5.下列理解和分析,不符合原文意思的一项是(  )‎ A.在茶饮上升到精神领域陆羽有着重要的贡献,他创立了完整的茶叶科学体系,规范了饮用的方法,提出了“茶有真香”的核心观念。‎ B.饮茶开始时是属于贵族专利,后来饮茶扩展到士大夫阶层,但饮用方式比较古朴,茶处理如同蔬菜,没有唐代复杂精美的饮用法。‎ C.唐代茶的流行除了社会原因和交通外,还与禅教的兴盛有关,很多寺院推广喝茶来提神不寐,因禅宗的影响很大,最终就影响了民间。‎ D.现在大多数中国百姓都是用大杯冲泡茶叶,这很符合古代的质朴之道,但因历史原因,我们祖先创造的茶道已与中国文化无关了。‎ 答案 D 解析 胡乱联系。“我们祖先创造的茶道已与中国文化无关了”错误,原文第五段后面说“现在很多中国人觉得茶道是日本的国粹,与中国文化无关”,意思是因为中国的茶道长期无人提及,造成一些人认为茶道就是日本的而不是中国的错误观念。‎ ‎6.根据原文内容,下列理解和分析不正确的一项是(  )‎ A.茶最初被认为是菜蔬等,所以饮茶的方式与后来大不相同,十分有生活气息,在三国魏晋时就是放在水里煮,再加香料和佐料。‎ B.唐代的制茶技术没有宋代复杂,到明代制茶技术有了更高的发展,构成了一种发达的品茗体系,使明代成为中国茶的复兴时期。‎ C.明朝品茶的情趣不仅是品味茶叶香味,更重要的还有通过品茶带来的心灵的修养的提升,当然在饮茶时还会像唐代一样赏茶具。‎ D.清代在品茶上有着自己的特点,如茶碗变得更加小,基本使用青花瓷、白瓷杯,紫砂壶成为主要泡茶工具,福建工夫茶出现等。‎ 答案 B 解析 延伸过度。“明代制茶技术有了更高的发展”错误,原文第四段的意思是明代成为中国茶的复兴时代,主要因为士大夫阶层讲究品茶,构成了一种发达的品茗体系,而不是因为制茶技术的发展。‎ 二、古代诗文阅读(35分)‎ ‎(一)文言文阅读(19分)‎ ‎[2017·湖北沙市中学考试]阅读下面的文言文,完成7~10题。‎ 袁枢,字机仲,建之建安人。幼力学,尝以《修身与写赋》试国子监,周必大、刘珙皆期以远器。试礼部,词赋第一人,教授兴化军。‎ 为礼部试官。常喜诵司马光《资治通鉴》,苦其浩博,乃区别其事而贯通之,号《通鉴纪事本末》。参知政事龚茂良得其书,奏于上,孝宗读而嘉叹,以赐东宫及分赐江上诸帅,且令熟读,曰:“治道尽在是矣。”‎ 他日,上问袁枢何官,茂良以实对,上曰:“可与寺监簿。”于是以大宗正簿召登对,即因史书以言曰:“臣窃闻陛下尝读《通鉴》,屡有训词,见诸葛亮论两汉所以兴衰,有戒,垂法万世。”遂历陈往事,自汉武而下至唐文宗,偏听奸佞,致于祸乱。上曰:“朕不至与此曹图事帷幄中。”枢谢曰:“陛下之言及此,天下之福也。”‎ 兼国史院编修官,分修国史传。章悖家以其同里,宛转请文饰其传,枢曰:“吾为史官,书法不隐,宁负乡人,不可负天下后世公议。”时相赵雄总史事,见之叹曰:“无愧古良史。”‎ 权工部郎官,累迁兼吏部郎官。两淮旱,命廉视真、杨、庐、和四郡。归陈两淮形势,谓:“两淮坚固则长江可守,徒知备江,不知保淮,置重兵于江南,委空城于淮上,非所以戒不虞。瓜洲新城,专为退保,金使过而指议,淮人闻而叹嗟。谁为陛下建此策也?”‎ 迁大理少卿。通州民高氏以产业事下大理,殿中侍御史冷世光纳厚赂曲庇之,枢直其事以闻,人为危之。上怒,立罢世光,以朝臣劾御史,实自枢始。诏权工部侍郎。因论大理狱案请外,有予郡之命,既而贬两秩,寝前旨。光宗受禅,叙复元官。‎ 擢右文殿修撰、知江陵府。江陵濒大江,岁坏为巨浸,民无所托。楚故城楚观在焉,为室庐,徙民居之,以备不虞。种木数万,以为捍蔽,民德之。开禧元年,卒,年七十五。‎ 创作《易传解义》及《辩异》《童子问》等书藏于家。‎ ‎(选自《宋史·袁枢传》,有删改)‎ ‎7.对下列句子中加点词的解释,不正确的一项是(3分)(  )‎ A.治道尽在是矣         道:方法 B.宁负乡人,不可负天下后世公议 负:对不起 C.诏权工部侍郎 权:代理 D.种木数万,以为捍蔽,民德之 德:思想 答案 D 解析 德:名词用作动词,感激。‎ ‎8.下列对文中加点词语的相关内容的解说,不正确的一项是(3分)(  )‎ A.国子监,是中国古代自明朝以后的中央官学,为中国古代教育体系中的最高学府,又称国子学或国子寺。‎ B.礼部,中国古代官署,南北朝北周始设。隋唐为六部之一。历代相沿。管理全国学校事务及科举考试还有藩属和外国之往来事。‎ C.累迁,表示官职多次调动。在古代,超迁、擢、陟、拔等表示官职的升迁,而谪、黜、窜、左迁等则表示降职贬官。‎ D.《通鉴纪事本末》为中国第一部纪事本末体史学著作。“纪事本末体”兼有纪传、编年二者优点,使“数千年事迹经纬明析”。‎ 答案 A 解析 “自明朝以后”有误,应是“自隋朝以后”。‎ ‎9.下列对原文有关内容的概括和分析,不正确的一项是(3分)(  )‎ A.袁枢文才突出,学识渊博。他少年时努力学习,曾经以《修身与写赋》参加国子监考试,后又参加礼部考试,得词赋科第一。周必大、刘珙看好他的前程。‎ B.皇帝关心提拔袁枢,袁枢借史事劝诫皇帝。袁枢积极进言,敢于弹劾权贵,御史冷世光违法乱纪,袁枢弹劾他,皇帝生气,立即罢免了冷世光。‎ C.袁枢为人正直,修史秉笔直书。他负责修国史,章悖是他的同乡,向他委婉请求修饰自己的传记,被袁枢严词拒绝,皇帝知道后赞赏了袁枢。‎ D.袁枢为民办实事,体恤百姓。他在江陵任职,江陵靠近长江,每年遭受水患,百姓遭殃;他建草房安置百姓,种树木防备水灾,百姓感激他。‎ 答案 C 解析 “皇帝知道后赞赏了袁枢”错误,见原文第四段末句,应是宰相赵雄赞赏袁枢。‎ ‎10.把文中画横线的句子翻译成现代汉语。(10分)‎ ‎(1)遂历陈往事,自汉武而下至唐文宗,偏听奸佞,致于祸乱。(5分)‎ 译文:_________________________________________________‎ ‎(2)上怒,立罢世光,以朝臣劾御史,实自枢始。(5分)‎ 译文:_________________________________________________‎ 答案 (1)(袁枢)于是依次陈述古代的事,从汉武帝以后到唐文宗,偏听邪恶人的话,造成祸乱。‎ ‎(2)皇帝生气,立即罢免了冷世光,以朝廷官员的身份弹劾御史,实际上是从袁枢开始的。‎ 解析 (1)“历”,依次;“陈”,陈述;“奸佞”,形容词作名词,奸佞的人。“历”“陈”“奸佞”各1分,大意2分。‎ ‎(2)“罢”,罢免;“以朝臣”,凭借朝臣的身份;“实”,实际上。“罢”“以朝臣”“实”‎ 各1分,大意2分。‎ 参考译文:‎ 袁枢,字机仲,建宁府建安县人。少年时努力学习,曾经用《修身与写赋》在国子监应试,周必大、刘珙都用远大前程期许他。参加礼部考试,考词赋第一名,担任兴化军教授。‎ ‎(袁枢)担任礼部试官。经常喜爱诵读司马光的《资治通鉴》,苦恼它盛大广博,于是辨别它里面的事件而贯穿在一起,称为《通鉴纪事本末》。参知政事龚茂良得到他的书,献给皇帝,孝宗读了嘉奖赞叹,拿来赐给太子以及沿江各将帅,而且命令他们熟读,说:“治国的方法都在这里。”‎ 后来有一天,皇帝问袁枢现任什么官,龚茂良按实回答,皇帝说:“可以任他为寺监簿。”于是凭着大宗正簿的身份上朝对答皇帝的询问,就根据史书进言说:“我私下听说陛下曾读《通鉴》,多次有教导的话,看到诸葛亮谈论两汉兴衰的原因,有告诫,流传示范万世。”于是依次陈述古代的事,从汉武帝以后到唐文宗,偏听邪恶人的话,造成祸乱。皇帝说:“我不至于和这些人在朝廷策划国事。”袁枢道歉说:“陛下的话说到这里,是天下人的福气。”‎ 兼任国史院编修官,分工负责国史传记。章悖因为是袁枢的同乡,委婉地请求袁枢用文辞修饰他的传记,袁枢说:“我是史官,记载史事的原则是不隐瞒,宁愿对不起老乡,不可以对不起天下后世的公正的评论。”当时宰相赵雄总管修史的事,见到这事感叹说:“不愧于古代的优秀史官。”‎ ‎(袁枢)任工部郎官,历升吏部郎官。两淮干旱,(皇上)令袁枢巡视真、扬、庐、和四郡。回朝后陈述两淮的形势,说:“两淮坚固那么长江可守得住,今天只知道防备长江,不知道保住淮河,置重兵于江南,把空城放弃在淮河流域,这不是警惕不测事件的做法。瓜洲是新城,专门为退保之地,金使路过对其指点、议论,淮人听说感叹不已。谁为陛下定的这个策略?”‎ 他升任大理少卿。通州百姓高氏因为产业的事被关进大理狱,殿中侍御史冷世光接受巨额贿赂偏袒包庇高氏,袁枢原原本本地把事情报告皇帝,大家替袁枢担心。皇帝生气,立即罢免了冷世光,以朝廷官员的身份弹劾御史,实际上是从袁枢开始的。朝廷下诏让袁枢代理工部侍郎。由于议论大理寺案件请求调外地就职,有给他郡守的任命,接着贬官两级,停止以前的任命。光宗继位,恢复原来的官职。‎ 提拔为右文殿修撰、江陵府知府。江陵靠近长江,每年被冲垮成一片汪洋,百姓无处安身。楚国的旧城楚观还在,建草房,迁移百姓居住在那里,以防备意外之事。种树木几万株,作为屏障,百姓感激他。开禧元年,去世,享年七十五岁。‎ 创作《易传解义》和《辩异》《童子问》等书藏在家里。‎ ‎(二)古代诗歌阅读(11分)‎ ‎[2017·吉林重点高中联考]阅读下面这首词,完成11~12题。(11分)‎ 菩萨蛮 ‎[唐]温庭筠 宝函①钿雀②金鸂鶒③,沉香阁上吴山碧。杨柳又如丝,驿桥春雨时。  画楼音信断,芳草江南岸④。鸾镜⑤与花枝,此情谁得知?‎ ‎[注] ①宝函:华美的梳妆盒。②钿雀:雀饰的金钗。③鸂(xī)鶒(chì):水鸟名,又称紫鸳鸯。这里是金钗上的装饰。④芳草江南岸:化用“王孙游兮不归,春草生兮萋萋”(《楚辞·招隐士》)。⑤鸾(luán)镜:背上镌刻有鸾凤图案的妆镜。‎ ‎11.下面对这首晚唐词的理解,不正确的两项是(5分)(  )‎ A.“宝函钿雀金鸂鶒”触物生情,女子看到金钗上成双成对的“鸂鶒”时内心充满了一丝喜悦,表现了她内心与之相似的愿望。‎ B.“杨柳又如丝,驿桥春雨时”虚实结合,由眼前回忆当年春雨潇潇时和情人驿桥边依依惜别的情景。‎ C.“画楼音信断,芳草江南岸”两句实写窗外春景之繁盛、游子无信,反衬内心之思念愁苦之情。‎ D.“鸾镜与花枝,此情谁得知?”中“枝”与“知”谐音双关,女子感慨自己的相思之情无人理解,只有妆镜与花枝知道。‎ E.这首词由景到物,由景到情,自今忆昔,又由昔至今,看似散乱不连,实则脉络暗通,婉转绵密,情韵悠然。‎ 答案 AC 解析 “宝函钿雀金鸂鶒”,写女子晨起梳妆,她看到成双成对的“金鸂鶒”,与自己眼前的孤单寂寞恰好相反并形成对比反衬,内心应充满了悲愁苦闷。所以A错。同时结合下文,本词主要表现了女子的相思苦闷,“喜悦”与全词格调不合。C项亦实亦虚,暗用了“王孙游兮不归,春草生兮青青”的典故,写爱人远出不归,音信断绝。‎ ‎12.请简要分析这首词的情感流程。(6分)‎ 答:____________________________________________________‎ ‎________________________________________________________________________‎ 答案 女子晨起化妆,饰物上成双的鸳鸯触起她内心的伤感孤独(1分);于是移目向外,试图排遣,沉香阁外吴山青翠碧绿(1分);杨柳又轻细如丝,使她回想起当年春雨潇潇时和情人驿桥边依依惜别的情景,女子沉浸在回忆的悲伤中(1分);眼前芳草茂盛萋萋,而远去之人无音讯到画楼,女子由回忆回到现实,内心有着春归人不归的隐痛和更加强烈的孤独(1分)。无限自伤自怜之情使她窥镜自照,感叹自己的心事鸾镜和花都懂得,自己的心事有谁能知道呢?伤感自己如花的青春在无尽的等待中渐渐逝去(2分)。‎ 解析 本词中的艺术形象即女子,她内心的情感变化即词的情感流程。所以关键是细读诗句,读懂诗意,逐层提炼概括。词体的一个特点是讲究铺叙,基本上一个句号对应一个内容层次。学生在理解诗词大意的基础上可依次提炼答案。“杨柳又如丝,驿桥春雨时”,是由眼前到回忆,由虚到实,时空交替。“鸾镜与花枝”,“枝”谐音“知”,解读时要注意。‎ ‎(三)名篇名句默写(5分)‎ ‎13.[2017·湖北枣阳开学考试]补写出下列句子中的空缺部分。(5分,每空1分)‎ ‎(1)《醉翁亭记》中以色彩鲜明的语言,描绘春夏之景的句子是:______________,______________。‎ ‎(2)《逍遥游》中,对天空的颜色成因进行了探寻,并发出了疑问的两句是:____________?____________?‎ ‎(3)《诗经·卫风·氓》中写女子在无奈下与男子约定婚期的句子是:将子无怒,______________。‎ 答案 (1)野芳发而幽香 佳木秀而繁阴 (2)其正色邪 其远而无所至极邪 (3)秋以为期 三、语言文字运用(20分)‎ ‎14.[2017·张家口五校联测]下列各句中加点成语的使用,全都正确的一项是(3分)(  )‎ ‎①八达岭野生动物世界发生老虎咬人事件后,网上沸沸扬扬,老虎咬人事件到底是游客本身的责任还是园方的责任一度成为舆论热点。‎ ‎②因为98个样本(北京奥运会的60个样本和伦敦奥运会的38个样本)药检结果呈阳性,所以国际奥委会或许因兴奋剂丑闻将举重项目从奥运中连根拔起。‎ ‎③经过四年艰苦谈判,哥伦比亚政府与该国最大反政府武装惺惺相惜,24日在古巴首都哈瓦那宣布达成最终全面和平协议,结束长达半个多世纪的武装冲突。‎ ‎④魏雪女士多年来始终致力于公益慈善事业,在她的感召下,很多人也如过江之鲫,帮助农村贫困地区发展基础教育,开展了“华萌班”等公益项目。‎ ‎⑤2011年8月31日,华山景区对外免费开放。如今,一年多过去了,记者27日实地探访发现,虽然免费开放,游客却寥寥无几,还不如收费时人多。‎ ‎⑥为了在《百家讲坛》讲述《清十二帝疑案》,阎崇年翻阅了大量历史资料,精心研究,可谓沙里淘金,最终赢得了观众的认可。‎ A.①②④ B.②④⑤‎ C.①⑤⑥ D.③④⑥‎ 答案 C 解析 ①沸沸扬扬:像沸腾的水一样喧闹,多形容议论纷纷。②连根拔起:比喻彻底铲除或消灭。不合语境。③惺惺相惜:聪明人怜惜聪明人,泛指性格、才能或境遇等相同的人互相爱重、同情。对象不当。④过江之鲫:赶时髦的人很多。为贬义词。此句褒贬不当。⑤寥寥无几:非常少。⑥沙里淘金:既比喻从大量的材料中选择精华,也形容费力大而成效少。此句用第一义。‎ ‎15.[2017·佛山一中段考]填入下面文段空白处的词语,最恰当的一组是(3分)(  )‎ ‎(1)由于古生物学工作者不容易弄清楚蜥螈到底是两栖动物还是爬行动物,__①‎ ‎__过去把它放在爬行动物中,__②__现在又把它放在两栖动物中。‎ ‎(2)如果我们的散文拒绝精神的攀沿,__③__拒绝起码的思想含量,而始终徘徊于低层次的琐碎生活,__④__散文还会有它沉甸甸的分量吗?‎ ‎(3)古代劳动人民的诗歌创作__⑤__表现了他们对现实的认识和爱憎,而且表现了他们质朴的艺术感受,__⑥__逐渐形成了现实主义与浪漫主义完美融合的特色。‎ ‎①‎ ‎②‎ ‎③‎ ‎④‎ ‎⑤‎ ‎⑥‎ A 尽管 而 甚至 所以 可能 ‎/‎ B 既然 于是 和 ‎/‎ 固然 进而 C 以致 而 甚至 那么 不仅 从而 D 虽然 进而 又 ‎/‎ 不但 也 答案 C 解析 (1)①处“以致”与句首“由于”构成因果关系,②处前后的情况形成反差,填表转折关系的“而”比较合适。(2)从“拒绝精神的攀沿”到“拒绝起码的思想含量”,语意程度加深,③处填表递进关系的“甚至”合适,④处与句首“如果”构成假设关系,填“那么”合适。(3)⑤处根据“而且表现了……”可知,选填的是表递进关系的关联词,排除A、B,⑥处根据语意可知“形成了现实主义与浪漫主义完美融合的特色”是结果,填“从而”合适,因此排除D,选C。‎ ‎16.[2017·江西铅山、横峰联考]下面的句子中,没有语病的一项是(3分)(  )‎ A.高速公路上交通事故的主要原因是司机违反交通规则或操作不当造成的,交通部门要加强安全宣传,提高司机的安全意识。‎ B.食醋富有氨基酸、钙、磷、铁和维生素B等成分,因此具有美容功效,皮肤吸收之后,可改善营养缺乏,促使皮肤美白细腻。‎ C.能否使课堂教学回归教育的本真,是课堂教学效率能否得到提高的关键,也是引导学生成为有诗意追求的人的关键。‎ D.傅园慧在晋级里约奥运女子100米仰泳决赛后的一段采访视频给奥运带来了一股表情包的“洪荒之力”,很多网友纷纷表示这小姑娘太搞笑啦。‎ 答案 D 解析 A项,句式杂糅。“原因是……造成的”错误。B项,搭配不当。缺少与“改善”搭配的词语。C项,两面对一面。‎ ‎17.[2017·吉林白城一中质检]在下面一段文字横线处补写恰当的语句,使整段文字语意完整连贯,内容贴切,逻辑严密。每处不超过15个字。(5分)‎ 中国人讲究家国观念,欲治其国,__①__。事实上,一个人的责任、担当与情感,总是按照家庭伦理、由近及远扩散开去。很难想象,一个不疼爱自己孩子的人,会对这个社会产生认同感和归属感;不能相信,一个对自己家人缺少担当的人,__②__。__③__,才能谈得上爱社会和国家。‎ 答:①__________________________________________________‎ ‎②_____________________________________________________‎ ‎③_____________________________________________________‎ 答案 ①必先治其家 ②会对这个国家产生责任感和使命感 ③只有首先爱家人和家庭。‎ ‎18.[2016·石家庄质检]观察下列图示,整合图示信息,紧扣“赢”字的构件和含义,写一段100字左右的文字。(6分)‎ 答:____________________________________________________‎ ‎____________________________________________________________‎ 答案 ①“赢”字由“亡”“口”“月”“贝”“凡”构成。②“亡”表示危机意识,“口”表示沟通能力,“月”表示时间概念,“贝”表示取财有道,“凡”表示平常心态。③危机意识、沟通能力、平常心态、时间观念加上取财有道,是成为真正赢家的必备要素。‎ 第四部分 文学类文本阅读 小说阅读 考点十九 小说的情节 考点名片 考点内容 重要作家的精短小说情节的概括提炼,情节安排的作用。‎ 考查形式 或客观题,或主观题。①用简明的语言概括故事情节;②××情节在小说中起什么作用?‎ 趋势分析 对情节的概括提炼和对故事情节安排的作用分析是主要命题点。‎ 一、阅读下面的文字,完成后面的题目。‎ 泥人张 冯骥才 ‎①手艺道上的人,捏泥人的“泥人张”排第一。而且,有第一,没第二,第三差着十万八千里。‎ ‎②泥人张大名叫张明山,咸丰年间常去的地方有两处,一是东北城角的戏院大观楼,一是北关口的饭馆天庆馆。坐在那儿,为了瞧各样的人,也为捏各样的人。去大观楼要看戏台上的各种角色,去天庆馆要看人世间的各种角色。这后一种的样儿更多。‎ ‎③那天下雨,他一个人坐在天庆馆里饮酒,一边留神四下里吃客们的模样。这当儿,打外边进来三个人。中间一位穿得阔绰,大脑袋,中溜个子,挺着肚子,架势挺牛,横冲直撞往里走。站在迎门桌子上的“撂高的”一瞅,赶紧吆喝着:“益照临的张五爷可是稀客、贵客,张五爷这儿总共三位——里边请!”‎ ‎④一听这喊话,吃饭的人都停住嘴巴,甚至放下筷子瞧瞧这位大名鼎鼎的张五爷。当下,城里城外气最冲的要算这位靠着贩盐赚下金山的张锦文。他当年由于为盛京将军海仁卖过命,被海大人收为义子,排行老五,所以又有“海张五”一称。但人家当面叫他张五爷,背后叫他海张五。天津卫是做买卖的地界儿,谁有钱谁横,官儿也怵三分。‎ ‎⑤可是手艺人除外。手艺人靠手吃饭,求谁?怵谁?故此,泥人张只管饮酒、吃菜,西瞧东看,全然没把海张五当个人物。‎ ‎⑥但是不一会儿,就听海张五那边议论起来。有个细嗓门儿的说:“人家台下一边看戏,一边手在袖子里捏泥人。捏完拿出来一瞧,台上的嘛样,他捏的嘛样。”跟着就是海张五的大粗嗓门儿说:“‎ 在哪儿捏?在袖子里捏?在裤裆里捏吧!”随后一阵笑,拿泥人张找乐子。‎ ‎⑦这些话天庆馆里的人都听见了。人们等着瞧艺高胆大的泥人张怎么“回报”海张五。一个泥团儿砍过去?‎ ‎⑧只见人家泥人张听赛没听,左手伸到桌子下边,打鞋底抠下一块泥巴。右手依然端杯饮酒,眼睛也只瞅着桌上的酒菜,这左手便摆弄起这团泥巴来,几个手指飞快捏弄,比变戏法的刘秃子还灵巧。海张五那边还在不停地找乐子,泥人张这边肯定把那些话在他手里这团泥上全找回来了。随后手一停,他把这泥团往桌上“叭”地一戳,起身去柜台结账。‎ ‎⑨吃饭的人伸脖一瞧,这泥人张真绝了!就赛把海张五的脑袋割下来放桌上一般。瓢似的脑袋,小鼓眼,一脸狂气,比海张五还像海张五,只是只有核桃大小。‎ ‎⑩海张五在那边,隔着两丈远就看出捏的是他。他朝着正走出门的泥人张的背影叫道:“这破手艺也想赚钱,贱卖都没人要。”‎ ‎⑪泥人张头都没回,撑开伞走了。但天津卫的事没有这样完的。‎ ‎⑫第二天,北门外估衣街的几个小杂货摊上,摆出来一排排海张五这个泥像,还加了个身子,大模大样坐在那里。而且是翻模子扣的,成批生产,足有一二百个。摊上还都贴着个白纸条,上边使墨笔写着:贱卖海张五。‎ ‎⑬估衣街上来来往往的人,谁看谁乐。乐完找熟人来看,再一块乐。‎ ‎⑭三天后,海张五派人花了大价钱,才把这些泥人全买走,据说连泥模子也买走了。泥人是没了,可“贱卖海张五”这事却传了一百多年,直到今儿个。‎ ‎1.下列对小说有关内容和艺术特色的鉴赏,最恰当的一项是(  )‎ A.第④段极力写海张五的不可一世,运用反衬手法,借海张五嚣张跋扈、不可一世的样子衬托出泥人张身处乱世洁身自好的高尚品格。‎ B.估衣街上来来往往的人看到“贱卖海张五”就“谁看谁乐”,是因为泥人张将海张五捏得活灵活现,而且对他的五官丑陋之处都进行了夸张式表现。‎ C.第⑧段运用了神态、动作描写,生动形象地表现了泥人张捏泥人的精湛手艺和对海张五的蔑视心理。‎ D.“泥人张”的做法让我们看到了他善于观察模拟人物,并且心灵手巧、艺高人胆大,但做事不够圆滑。‎ 答案 D 解析 A项,衬托出泥人张不向权贵低头的高尚品格。B项,“谁看谁乐”,是因为海张五是城里城外气最冲的,看到他被捉弄人们拍手称快。C项,没有神态描写。‎ ‎2.请概括小说的主要情节。‎ 答:____________________________________________________‎ ‎____________________________________________________________‎ 答案 为瞧各样人,捏各样人,泥人张常去天庆馆饮酒;海张五张狂出场,泥人张毫不理会,海张五及手下人拿泥人张找乐子,泥人张捏完核桃大的海张五脑袋扬长而去;第二天北门外估衣街几个小杂货摊贱卖海张五泥像;海张五花大价钱把街市上卖的泥人连同泥模子全买走了。‎ 解析 概括小说的主要情节,其实就是概括故事发展的各个阶段的主要内容。本文的情节发展历经了四个阶段:①②段为故事的开端,③~⑪段为故事的发展,⑫⑬段为故事的高潮,⑭段为故事的结局。‎ ‎3.海张五被“贱卖”的可笑结局给了我们怎样的启示?请结合现实回答。‎ 答:____________________________________________________‎ ‎____________________________________________________________‎ 答案 为人处世不可嚣张跋扈,目空一切,否则只会惹来不必要的麻烦。在现实生活中,我们为人处世应该低调、谦和、容忍他人。只有每个人都切实地提高个人素质,我们的社会才能更加和谐。‎ 解析 结合文本首先从海张五既丢人又“丢”钱的可笑结局中,提炼出做人蛮横无理没有修养必然招致被羞辱的结果。然后联系现实,从建立和谐社会的高度认识如何做人做事。‎ 二、阅读下面的文字,完成1~3题。‎ 我没有病 侯发山 他推开虚掩的门,走到老板桌前,坐在皮椅上晃动了两下,然后端起茶杯喝了一口温热的茶水,很满足地“呀”了一声,感觉好极了。‎ 桌上的电话忽然响起来,把他吓了一跳。他没有去接。电话固执地响着。他这才拿起听筒,只听有人说道:“局长,下午开会不?”他皱了一下眉头,不耐烦地说:“不开会!整天开会,有什么好开的?”说罢挂了电话。‎ 这时有人敲门。他应答了一声,只见一个上了年纪的人进来了。他说:“大爷,你有什么事?”‎ 老人怔了一下,忙点头哈腰地说:“局长,我来报销医疗费,跑了好多趟了,财务上不给报销……我来一趟不容易,你看能不能给报了?都好几年了。”说罢掏出了一沓子单据。有这事?岂有此理!他说:“大爷,总共多少钱?”‎ 老人颤抖着手把单据递给他:“局长,不多,只有四百多块。”他拿起桌子上的笔“刷刷”签了几个字:“同意报销!”老人千恩万谢地走了。‎ 老人前脚走,后脚就又进来一个西装革履的中年人。中年人看了他一眼,疑惑地说:“你是新来的局长?”他点点头:“是啊,你有什么事?”‎ 中年人讪笑了一下,掏出一个鼓鼓囊囊的包递过去,说:“局长,听说局里要改造办公楼,这是一点小意思。等我把工程承包到手……”他没有接红包,气呼呼地说:“闭嘴!我要吃有吃,要穿有穿,要你的钱有何用?你真有钱去捐给四川灾区的孩子。”‎ 中年人手足无措,不知道他是不是在作秀。‎ 他用手指着门,厉声说道:“你给我走,别说没工程,就是有工程也不给你这号人……”他知道,工程一旦给这种人做,绝对是豆腐渣。‎ 中年人悻悻而去。‎ 这时候电话又响了起来,他拿起听筒,电话那端传来一个娇滴滴的声音:“哥哥,你答应给妹子装修房子,怎么还不行动啊?”他不动声色地说:“你是谁?”“我是 睡美人 舞厅的红红啊!你把妹子给忘了?上次我过生日,你送我一条金项链,你忘了?”他冷冷一笑,然后悄悄说道:“我老婆在这儿呢,惹恼了她,有你的好日子过……”‎ 他刚挂断电话,又一个电话打了进来:“大哥,今晚我请客,在老地方, 醉仙楼 酒店……”他打断对方的话,说:“我不去,说不去就不去,因为什么?因为我知道吃人的嘴短,拿人的手软……”说罢他就挂断了电话。对方肯定有求于他,要不然不会叫他的。‎ 可是,电话又响了起来。他恨恨地看了电话机一眼,伸手把线给拔了,顿时,电话成了哑巴。这时候,突然闯进来几个医护工作人员,上前把他给制服了。‎ 其中一个医生说:“想不到你跑到了这里,走,跟我们回去。”他挣扎着:“我没有病,我没病……”‎ 这时候,办公室套间的门开了,一个睡眼惺忪、大腹便便的家伙踉跄着走了出来,酒气熏天,口齿不清地说:“干、干吗?跑到我的办公室来闹什么?真、真是有病!”‎ ‎(选自《小说选刊》2016.9,有删改)‎ ‎1.下列对小说有关内容和艺术特色的鉴赏,最恰当的一项是(  )‎ A.老人怔了一下,忙点头哈腰地说:“局长……都好几年了。”老人进屋后“怔了一下”是因为他一下子认出刚来的局长原来是老熟人。‎ B.“桌子上的电话忽然响起来,把他吓了一跳……他这才拿起听筒……”作者这样写只是为了说明“他”平时就非常讨厌接电话。‎ C.本文的细节描写十分传神,作者通过一系列的细节描写,使读者对主人公的身份产生怀疑。而文章结尾更是出人意料,发人深思。‎ D.小说中的“他”只是一名精神病患者,作者写他是为了讽刺医院的不负责任,任由病人偷偷溜出来;那个“睡眼惺忪,大腹便便的家伙”是一个病入膏肓的家伙,作者用他来隐喻社会中的病态现象。‎ 答案 C 解析 A项,老人怔了一下的原因是,“他”客气地问了一声:“大爷,你有什么事?”让老人感到少有的温暖。B项,纵观全文,作者这样写的主要目的是为了暗示主人公的真实身份。D项,小说中的“他”‎ 坚持社会公平与正义,是一个灵魂健康的人。‎ ‎2.请简要分析小说是如何通过奇特的情节设置悬念的,这样写有何好处?‎ 答:____________________________________________________‎ ‎____________________________________________________________‎ 答案 ①小说开头并没有交代“他”的身份,而是写“他”来到办公室安排工作,接待来访的“上了年纪的人”,怒斥企图行贿的“中年人”,接听电话,制造了“他”是不是局长的悬念。小说最后才点明“他”不是局长,而是“医护工作人员”在到处寻找的“病人”。‎ ‎②这样设置悬念的好处:使故事扑朔迷离,情节曲折;使主人公这个人物形象更具传奇色彩,激发读者的阅读兴趣。‎ 解析 这篇小说在情节的安排上有以下特点:一是故事情节一波三折,这样写能引人入胜,扣人心弦,增强了故事的戏剧性和可读性;二是结尾出人意料。解答时要纵观全文,思考小说是如何通过奇特的情节设置悬念的,这样写有何妙处。‎ ‎3.小说结尾“干、干吗?跑到我的办公室来闹什么?真、真是有病!”在文中有何作用?请联系全文,从故事情节和内容两方面作答。‎ 答:____________________________________________________‎ ‎____________________________________________________________‎ 答案 (1)在情节上:①呼应了题目“我没有病”,引发读者思考谁才是真正有病的人;②办公室真正的主人现身,进一步解开了读者对主人公真实身份的疑惑。‎ ‎(2)在内容上:揭示了小说的主题,表达了作者对社会上丑恶的人和事的辛辣讽刺,抒发了作者对当今社会的无限忧虑之情。‎ 解析 解答时,一要结合具体语境分析其表层含意,二要结合全文分析其深层含意。考生在答题时只要抓住这一点就能归纳出本题的答案。‎ 三、阅读下面的文字,完成1~3题。‎ 幸存者 ‎[美]休·B·卡夫 熬到第三个饥饿的夜晚,诺尼把眼睛盯在那条狗身上。在这座漂流的冰岛上,除了高耸的冰山之外,没有任何的血肉——就剩他们两个了。‎ 在那次撞击中,诺尼失去了他的雪橇、食物、皮衣,甚至他的尖刀。他只救起了心爱的猎犬——尼奴克。如今,一人一狗被困在冰岛上,保持着一定的距离,虎视眈眈地注视着对方。‎ 诺尼以往对尼奴克的宠爱绝对是真实的,真实得如同此刻的饥饿、夜晚的寒冷以及那只受伤的脚上咬啮着的痛苦。然而家乡的人在荒年不也屠杀他们的狗来果腹吗?不是吗?他们甚至想都不想一下就做了。‎ 他告诉自己,当饥饿到了尽头一定要觅食,“我们二者之中注定要有一个被对方残杀,”诺尼想,“所以……”‎ 他无法徒手捕杀那只狗。尼奴克的凶悍有力远胜于他。此刻,他急需一件武器。‎ 他脱下手套,把腿上的绷带拆下来。几个星期前,他伤了自己的腿,用一些绳索和三片铁板绑成了绷带。‎ 他跪在地上,把一片铁板插入冰地的细缝里,并且使劲地用另一片铁板在上面磨擦。‎ 尼奴克聚精会神地看着他。诺尼仿佛能感觉到那炯炯的眼神正放出愈发炽烈的光芒。‎ 他继续工作,并且企图使自己忘记它的目的。那片铁板现在已经有一面的刃了,并且越磨越锋利。太阳升起时他刚好完成了工作。‎ 诺尼将那新磨的尖刀从冰地里拔出来,用拇指抚拭着刀刃。太阳光从刀面反射过来,几乎使他一时眼花目眩。诺尼把自己完全变得残酷起来,“到这儿来,尼奴克!”他轻轻地叫着。‎ 狗疑惑地看着他。“过来,快!”他唤着。‎ 尼奴克走近了一点。诺尼在它的眼神中看到恐惧。从它沉滞的喘息和蹒跚、笨重的脚步可以得知它的饥饿和痛楚。他的内心开始哭泣了。他痛恨自己,但又不得不狠下心来。‎ 尼奴克越来越近了,保持着它的警戒。诺尼感到喉间一股浓重的呼吸,他看出它那两只眼睛好似两股痛苦哀怨的井泉。‎ 现在,就是现在!快攻击它!‎ 诺尼跪倒在地上的身体因一阵激烈的哽咽而颤抖着。他突然唾骂起那把尖刀,疯狂地把它往远处掷去。他空着双手,颠踬地向狗爬去,终于倒在雪地里。‎ 狗发出凶狞的咆哮,环绕着他的身体走动。诺尼现在充满了恐惧。‎ 掷出那把刀子之后,他已没有任何防备。诺尼现在虚弱得毫无力气反抗。他的性命就好像悬在尼奴克面前的一块肉,而它的眼中充满了饥饿。‎ 狗绕着他徘徊,并且开始从后面匍匐前进,诺尼现在听到那饥饿的喉咙发出咕噜咕噜的吞咽唾液的声音。‎ 他闭上眼睛,祈求着这次的攻击不要太痛苦,他感觉到它的爪子踏上他的腿,尼奴克渐热的喘息逼近他的脖子,一股强烈的气流聚集在他的喉头。然后,他感觉到一条热热的舌头在轻轻地舔着他。‎ 诺尼睁开眼睛,怀疑地注视着尼奴克。他伸出一只手臂把它和自己紧紧地抱在一起,悲伤地开始呜咽哭泣……‎ 一小时之后,一架飞机从南方起飞,上面一位年轻的驾驶员沿着海岸巡逻,他往下注视着那片漂流的浮冰,在冰山的正上方盘旋,此时,他看到一道刺眼的闪光。‎ 那是阳光在某件物体上反射的光芒。他的好奇心渐渐升起,他降低了高度,沿着冰山盘旋。此时,他发现冰山的阴影之中一堆黑色的影子,从形状上来看似乎是人类,仿佛那影子有时还分成两个。他把飞机降落在水边,开始巡查,发现了那两个影子,一个人和一只狗。那个男人已经昏迷不醒,但确信还活着,那只狗呜呜地在一旁哀鸣,已经虚弱得不能移动了。‎ 至于那道引起驾驶员注意的光芒,就是那把磨得雪亮的尖刀。它挺直地插在不远的雪地上,在风中微微地颤动着……‎ ‎(有改动)‎ ‎1.下列对小说有关内容和艺术特色的鉴赏,最恰当的一项是(  )‎ A.诺尼的心理变化过程是,身陷冰岛的恐惧无助——饥饿之极萌生杀狗求生之念——面对尼奴克眼神的痛苦犹豫——面对尼奴克痛苦哀怨的眼睛扔掉尖刀。‎ B.结尾雪地上挺直插着的尖刀,给人以强烈的视觉冲击力。原本是杀狗凶器却成了救人工具的情节转换,凸显了人性中的善良、宽容、美好和庄严。‎ C.小说有明暗两条线索,明线是“尖刀”,从磨制到抛弃,再到反射光线使主人被救的过程中,它始终贯穿全文;暗线就是尼奴克从凶狠到温柔的变化。‎ D.小说给我们展示了一种和谐美好的情景——人萌发了善念,扔掉利刀,选择死亡;狗收起凶相,忠诚以待,轻舔主人:人与狗相拥相偎。小说闪耀着友善和谐的思想光芒。‎ 答案 A 解析 B项,很难凸显人性中的宽容一面。C项,暗线就是主人公诺尼心理的变化。D项,小说的主旨应该是抑恶扬善、坚守忠诚。‎ ‎2.小说是如何在情节上巧妙设置悬念来吸引读者的?‎ 答:____________________________________________________‎ ‎____________________________________________________________‎ 答案 小说的前半部分,人会不会杀死狗的悬念吸引着读者;中间部分,狗会不会吃掉人的悬念吸引着读者;最后,人和狗会是什么样的结局的悬念吸引着读者。作者层层设置悬念,增加了作品的吸引力,使读者不得不看下去。‎ 解析 该题考查对小说情节技巧的分析。关键是把三次悬念的内容说准,还有这样的安排对行文有何作用,从文章的内容来看,悬念是层层深入的,都具有吸引读者继续阅读的兴趣。‎ ‎3.小说的结尾有何特点和作用?‎ 答:____________________________________________________‎ ‎____________________________________________________________‎ 答案 被扔掉的凶器,成为救人的工具。结尾的艺术空白,言有尽而意无穷,有一种冲击人心的张力,更能突出主旨。‎ 解析 “空白”是关键词,结合文本分析结尾的特点与作用。一般而言,“空白”的艺术也称“留白”‎ 艺术,即为意韵无穷,同时对文章主旨更有突出作用。‎ 四、阅读下面的小说,回答文后问题。‎ 小公务员之死 ‎[俄]契诃夫 一个美好的晚上,一位心情美好的庶务官伊凡·德米特里·切尔维亚科夫,坐在剧院第二排座椅上,正拿着望远镜观看轻歌剧《科尔涅维利的钟声》。他看着演出,感到无比幸福。但突然间,他的脸皱起来,眼睛往上翻,呼吸停住了……他放下望远镜,低下头,便……阿嚏一声!他打了个喷嚏,无论何时何地,谁打喷嚏都是不能禁止的。庄稼汉打喷嚏,警长打喷嚏,有时连达官贵人也在所难免。人人都打喷嚏。切尔维亚科夫毫不慌张,掏出小手绢擦擦脸,而且像一位讲礼貌的人那样,举目看看四周:他的喷嚏是否溅着什么人了?‎ 但这时他不由得慌张起来。他看到,坐在他前面第一排座椅上的一个小老头,正用手套使劲擦他的秃头和脖子,嘴里还嘟哝着什么。切尔维亚科夫认出这人是三品文官布里扎洛夫将军,他在交通部门任职。‎ ‎“我的喷嚏溅着他了!”切尔维亚科夫心想,“他虽说不是我的上司,是别的部门的,不过这总不妥当。应当向他赔个不是才对。”‎ 切尔维亚科夫咳嗽一声,身子探向前去,凑着将军的耳朵小声说:‎ ‎“务请大人原谅,我的唾沫星子溅着您了……我出于无心……”‎ ‎“没什么,没什么……”‎ ‎“看在上帝份上,请您原谅。要知道我……我不是有意的……”‎ ‎“哎,请坐下吧!让人听戏嘛!”‎ 切尔维亚科夫心慌意乱了,他傻笑了一下,开始望着舞台。他看着演出,但已不再感到幸福。他开始惶惶不安起来。幕间休息时,他走到布里扎洛夫跟前,在他身边走来走去,终于克制住胆怯心情,嗫嚅道:‎ ‎“我溅着您了,大人……务请宽恕……要知道我……我不是有意的……”‎ ‎“哎,够了!……我已经忘了,您怎么老提它呢!”将军说完,不耐烦地撇了撇下嘴唇。‎ ‎“他说忘了,可是他那眼神多凶!”切尔维亚科夫暗想,不时怀疑地瞧他一眼。“连话都不想说了。应当向他解释清楚,我完全是无意的……这是自然规律……否则他会认为我故意啐他。他现在不这么想,过后肯定会这么想的!……”‎ 回家后,切尔维亚科夫把自己的失态告诉了妻子。他觉得妻子对发生的事过于轻率。她先是吓着了,但后来听说布里扎洛夫是“别的部门的”,也就放心了。‎ ‎“不过你还是去一趟赔礼道歉的好,”她说,“他会认为你在公共场合举止不当!”‎ ‎“说得对呀!刚才我道过歉了,可是他有点古怪……一句中听的话也没说。再者也没有时间细谈。”‎ 第二天,切尔维亚科夫穿上新制服,刮了脸,去找布里扎洛夫解释……走进将军的接待室,他看到里面有许多请求接见的人。将军被围坐在其中,他已经开始接受呈文了。询问过几人后,将军抬眼望着切尔维亚科夫。‎ ‎“昨天在 阿尔卡吉亚 剧场,如果大人还记得的话,”庶务官开始报告,“我打了一个喷嚏,无意中溅了您……务请您原谅……”‎ ‎“什么废话!……天知道怎么回事!”将军扭过脸,对下一名来访者说:“您有什么事?”‎ ‎“他不想说!”切尔维亚科夫脸色煞白,心里想道,“看来他生气了……不行,这事不能这样放下……我要跟他解释清楚……”‎ 当将军接见完最后一名来访者,正要返回内室时,切尔维亚科夫快步跟上去,又开始嗫嚅道:‎ ‎“大人!倘若在下胆敢打搅大人的话,那么可以说,只是出于一种悔过的心情……我不是有意的,务请您谅解,大人!”‎ 将军做出一副哭笑不得的样子,挥一下手。‎ ‎“您简直开玩笑,先生!”将军说完,进门不见了。‎ ‎“这怎么是开玩笑?”切尔维亚科夫想,“根本不是开玩笑!身为将军,却不明事理!既然这样,我再也不向这个装腔作势的人赔不是了!去他的!我给他写封信,再也不来了!真的,再也不来了!”‎ 切尔维亚科夫这么思量着回到家里。可是给将军的信却没有写成。想来想去,怎么也想不出这信该怎么写。只好次日又去向将军本人解释。‎ ‎“我昨天来打搅了大人,”当将军向他抬起疑问的目光,他开始嗫嚅道,“我不是如您讲的来开玩笑的。我来是向您赔礼道歉的,因为我打喷嚏时溅着您了,大人……说到开玩笑,我可从来没有想过。在下胆敢开玩笑吗?倘若我真开玩笑,那样的话,就丝毫谈不上对大人的敬重了……谈不上……”‎ ‎“滚出去!!”忽然间,脸色发青、浑身打战的将军大喝一声。‎ ‎“什么,大人?”切尔维亚科夫小声问道,他吓呆了。‎ ‎“滚出去!”将军跺着脚,又喊了一声。‎ 切尔维亚科夫感到肚子里什么东西碎了。什么也看不见,什么也听不着,他一步一步退到门口。他来到街上,步履艰难地走着……他迷迷糊糊地回到家里,没脱制服,就倒在长沙发上,……咽了气。‎ ‎1.下列对小说有关内容和艺术特色的鉴赏,最恰当的一项是(  )‎ A.小说灵活地运用了语言、肖像、动作等多种描写手法来塑造主人公切尔维亚科夫这一典型形象。‎ B.小说的主人公切尔维亚科夫向长官前后进行过多达四次的“道歉”,而且一次比一次显得卑怯与悲戚。‎ C.小说喜剧性的结局既表现了作家对思想庸俗、生活委琐的小市民的“哀其不幸”与“怒其不争”,也堪称其对黑暗、罪恶的制度的控诉,具有深刻的社会意义。‎ D.小说从批判市侩意识的角度观察和反映生活,写出了丧失人格意识的奴性心理对于强权和暴力的恐惧和服从。‎ 答案 D 解析 A项,小说没有运用肖像描写,有心理描写。B项,小说中切尔维亚科夫向长官布里扎洛夫将军前后进行过五次(或六次)道歉。C项,小说的结局是切尔维亚科夫的死,这非但不是喜剧性的,反而是悲剧性的。‎ ‎2.小说中画横线部分的情节是否可以删去?为什么?请简要分析。‎ 答:____________________________________________________‎ ‎____________________________________________________________‎ 答案 不可以。①切尔维亚科夫的妻子是小说中唯一的配角,当切尔维亚科夫内心惶恐不安地把自己的失态告诉妻子时,她最终给予丈夫的建议还是再次去向长官道歉,她的言行无疑推动了小说情节的发展。②切尔维亚科夫的妻子的反应在某种程度上与前者的卑怯形成了对比,但是在她的思想意识中,仍然有根深蒂固的奴性心理和等级观念,从而凸显了当时俄国社会中切尔维亚科夫式心理的普遍性,进而深化了小说的主题。③这段情节在侧面烘托了小说主人公切尔维亚科夫因循守旧、惶恐忧郁的性格特征。‎ 解析 首先仔细阅读画线文字,把握两人的对话内容,然后分别从情节、人物、主题三方面思考作答。‎ ‎3.对于小公务员之死,有人认为这完全归咎于黑暗的俄国社会,也有人认为这源于小公务员的奴性心理和等级观念,还有人将其归因于小公务员自身的“自卑情结”,你认同以上哪一种说法?结合小说内容谈谈自己的观点。‎ 答:____________________________________________________‎ ‎____________________________________________________________‎ 答案 观点一:我认为小公务员之死的根本原因在于黑暗的俄国社会。①小说中的布里扎洛夫将军老头从头到尾,并未因为庶务官“一不小心”喷在他秃头顶与脖子上的喷嚏发出一声指责;倒是庶务官自己不厌其烦、胆战心惊地连连道歉,最后终于被吓死。②作家在这里巧妙地揭露出:沙皇专制统治严格的等级制度与残酷的高压政策是这场悲剧的制造者与罪魁祸首。归根结底,这是另一种意义上的“他杀”。③这说明,在这种高压政策面前,人们生存在地位与权势分明的氛围中,恐怖与压迫的毒瘤已经深深地污染、毒害了当时俄罗斯各阶层人们的心灵。‎ 观点二:我认为小公务员之死源于其自身的“自卑情结”。①从小说情节发展看,小公务员有着很强的“自卑情结”,这也是他的性格特征之一。小公务员生活的环境使他很早就产生了根深蒂固的低人几等的自卑心理。②在他眼里,那些上层社会的达官贵人有着神圣不可侵犯的威严,他们的一举一动、一言一行,都代表着一种意志、命令和心理威胁,并对他产生一种现实的压抑感。当他想摆脱这种压抑获得个性满足的自尊时,却受到了现实的巨大遏制与反弹,因为上层社会不容纳他,所以,当他向将军道歉,却遭到生硬而傲慢的拒绝时,他的心理受到严重的挫伤。③‎ 可他要求获得自尊的愿望,使他要找回心理平衡,定心看戏,于是,他要再次道歉,然而随之而来的打击则更加大了他心境的倾斜度,使他完全处在幻想状态中,不能按照生活的逻辑进行思维,而运用想象的逻辑进行判断,用想象代替现实做出一系列主观臆想的判断甚至做出“穿上新制服,刮了脸”去赔礼道歉的荒唐举动。这正是作者对病态社会使人性扭曲、心理变态的揭露。‎ 解析 观点明确(围绕题干中涉及的一个或几个观点展开论述)。注意结合作品内容,尤其是时代背景不能忽略,这是人物生存的土壤,可以帮助我们更好地理解人物的性格成因。‎ 考点二十 小说的环境 考点名片 考点内容 环境描写的作用。‎ 考查形式 或客观题,或主观题。①本文第×段的景物描写有什么作用?②本文第×段的景物描写运用了什么描写手法?有什么作用?‎ 趋势分析 对环境描写的手法和环境描写作用的分析是主要命题点。‎ 一、阅读下面的文字,完成文后题目。‎ 我去寻找路小丫了 闫玲月 雪下得这么大,娘今晚会早些回家吧。路小丫张开嘴巴呵着凉凉的小手,一团白雾在手心里飘散。炉火仿佛失去了动力,火苗微弱地摇摆着,路小丫连忙添进一铲子煤面。炉内腾起一条火龙,迸发出生命的热量,把她的脸蛋烤得红扑扑的。‎ ‎“吱扭”一声,门开了,一股冷气裹挟着一个雪人进来了。路小丫赶忙拿起毛巾为雪人打扫满身的雪花,雪人眨眼间变成了亲切的娘。娘问小丫饿坏了吧,小丫摇头。娘把裂满血口子的手伸向胸前,掏出一个热乎乎的烤红薯说,快趁热吃吧!路小丫接过烤红薯,一缕诱人的香气扑鼻而来,勾起她的食欲。‎ 路小丫细心剥去外皮,让它露出烤得焦黄的薯身,递到娘的嘴边说,娘饿了,娘先吃。娘轻轻咬了一小口,笑眯眯地说,娘的小丫真懂事。红薯又被送到路小丫嘴边,小丫张开嘴巴高兴地吃起来。‎ 绵软甜香的烤红薯温暖着路小丫的童年时光。娘不是亲娘,路小丫是烤红薯的娘在路边发现的。那个冬天真冷,冻得路小丫都快哭断气了,娘一狠心,捡回她的半条命,取名路小丫。娘没旁的亲人,就把路小丫当成亲闺女养活。娘没别的本事,以烤红薯为生。路小丫幼时睡觉总喜欢佝偻着小身子往娘的怀里钻。懂事后她对娘说,娘身上有一股好闻的香味。娘笑了,这丫头,娘又没钱搽脂抹粉,哪来的香味儿。后来她才懂得娘身上的香味儿就是经久不散的薯香。路小丫每晚在散发着阵阵薯香的娘的怀里安然入睡,一睡就是八年。‎ 八年后的一个下午,路小丫家里来了一个女人,女人与娘说了好久,不断用手帕擦泪。女人朝路小丫伸出嫩白的双手,路小丫躲到娘的身后。娘脸上挂着泪珠,笑着说,去吧孩子,这是你的亲妈,你们两个就像一个模子刻出来似的。路小丫舍不得离开娘,两步一回头,娘说,走吧孩子,那个家才有属于你的幸福生活。娘说完,把一扇大门死死关上。女人要带路小丫上车时,娘从后面气喘吁吁地追了上来,递给小丫一个小包袱,说,娘能给你的只有这么多了。路小丫抱紧小包袱,一路上不曾松手,不必打开她也知道,里面是娘亲手烤的红薯。‎ 来到原本属于自己的家,房子里的一切在路小丫眼里都是陌生的、新鲜的,包括那个让自己称呼她为“妈妈”的漂亮女人。‎ 妈妈说,路小丫这个名字太土气了,得先改个名字,爸爸姓陆,不如就叫陆小雅吧,虽然只取谐音,意义上却有了天壤之别。陆小雅被高价送入一所重点学校,同学们只知道她叫陆小雅,路小丫的名字随着她的记忆淡去了。‎ 陆小雅的妈妈发誓要把女儿培养成如她当年一样出色的舞蹈演员。陆小雅每天在妈妈的监督下练功,妈妈就是她的老师,严厉的眼神,严厉的话语,陆小雅找不到娘那样的温暖。‎ 陆小雅清楚地记得,那次,她在一家大商场外闻到一股久违的香味,就不由自主地一路追寻,终于在拐角处见到一个烤红薯的,陆小雅发现新大陆似的惊喜万分,抓起一个烤红薯就吃,才吃两口就被随后赶来的妈妈一巴掌打掉在地,妈妈说,真丢人,这是咱们吃的东西么!‎ 陆小雅每天按部就班地生活着,上学练功、吃饭睡觉都在妈妈的关注中。陆小雅朋友很少,话语很少,没有谁见她开怀大笑过,妈妈却说女孩子矜持点好,这才是大家闺秀的风范。有一天矜持的陆小雅突然失踪了,如同一只断线的风筝不知去向,只给妈妈留了一张字条,上面写着“我去寻找路小丫了”。‎ ‎(选自《羊城晚报》,有删改)‎ ‎1.下列对小说有关内容和艺术特色的鉴赏,最恰当的一项是(  )‎ A.小说中“一股冷气裹挟着一个雪人进来了”与娘“掏出一个热乎乎的烤红薯”形成冷暖对比,更加有力地突出了娘对路小丫的疼爱。‎ B.在寒冷的冬夜,母女二人吃烤红薯的这一幕,生动地表现了母女情深,也为最后写“陆小雅”的出走设置了悬念。‎ C.“娘说完,把一扇大门死死关上”,这是因为娘不希望路小丫的亲妈看到她为路小丫烤红薯。‎ D.在亲妈那里,“陆小雅”找不到娘那样的温暖,是因为她还不能适应新的环境,还沉浸在对娘的思念之中,也不能明白亲妈的期盼,误解了亲妈的严厉。‎ 答案 A 解析 B项,不是设置悬念,是作铺垫。C项,娘死死地关上门,主要是因为她不希望路小丫看到她伤心难过。D项,不是因为她不明白亲妈的期盼,而是这个家庭没有娘那样的温暖。‎ ‎2.文章第一段的环境描写有什么作用?‎ 答:____________________________________________________‎ ‎____________________________________________________________‎ 答案 ①交代故事发生的时间、地点;②渲染一种寒冷而又温馨的氛围;③具有象征意义,象征路小丫微弱而又顽强的生命力;④寄托了作者对路小丫的命运的美好祝愿。‎ 解析 回答这类题时,一要弄清环境的特征,二要明确环境描写与人物刻画之间的关系,三要理解环境描写与表现小说主题之间的关系,四要弄清环境描写与小说情节发展之间的关系。‎ 二、阅读下面的文字,完成1~3题。‎ 瞎子 莫泊桑 看见初升的太阳便觉得衷心喜悦,这种喜悦到底是怎么回事?为什么降到大地来的这片光明会如此这般使我们感到幸福?天空蔚蓝,田野碧绿,房舍雪白;我们喜洋洋的眼睛畅饮着这些鲜艳的色彩,把它们化成我们心中的快乐。于是我们一心只想跳舞、奔跑、歌唱,在思想上感到轻松愉快。‎ 门洞底下的那些瞎子处在永恒的黑暗之中,早已漠然无动于衷,在这个新的欢乐气氛中,也仍旧是安安静静地待着。只是时时刻刻吆喝身边的狗,叫它们安静,不明白为什么它们老想蹦蹦跳跳。‎ 这样的人我曾经见过一个,他过着难以想象的最残酷的苦难生活。‎ 他是一个乡下人,父亲是诺曼底的一个农庄主人。父母在世的时候,总算还有人照看他,他感觉苦痛的只是他那可怕的残疾。可是两老一去世,残酷的生活就开始了。有一个姐姐收留了他,然而农庄里的人待他却像待一个白吃饭的穷鬼,每顿饭都要怪他吃得太多,叫他懒虫、饭桶。尽管他的姐夫把他那份遗产夺到自己手里,可是连汤也舍不得给他多喝,给他的也就是刚刚够而不至于饿死的那么一点。‎ 几年里情况就是这样。不过他什么事也不能做,再加上老是冷冰冰的不声不响,最后惹恼了他的亲戚们,于是他成了受气包,成了一种供人虐待折磨的小丑,一种牺牲品,专供周围那些粗胚子发泄他们的兽性,惨无人道地取乐。‎ 凡是他的失明使人想到的残忍的恶作剧,都被想出来了。为了叫他吃了东西付出代价,他的吃饭时间变成了邻居们散心、他受罪的时刻。‎ 他的姐夫因为总这样养着他,心里有气就动手打他,不停地抽他的嘴巴,看见他躲躲闪闪或是举手还击时的那种瞎费气力的样子,不禁笑了起来。从此又有了新的玩法,就是打耳光。那些长工、短工、女仆高兴起来就给他一巴掌,打得他眼皮直眨巴。他不知道往哪儿躲,只好不停地伸着胳膊阻挡别人的攻击。‎ 最后他被逼着去要饭。赶集的日子他被带到大道边上,一听见有脚步声或是车轮声,就伸着帽子结结巴巴地叫喊:“求求您,行个好吧。”‎ 可是乡下人是不喜欢乱花钱的,一连几个星期,他一个铜子也带不回来。‎ 有一年冬天,地面盖满了雪,天冷得出奇。可是他的姐夫一大早就把他带到很远很远的一条大路上去求乞,一整天都把他撂在那里。到了晚上,他的姐夫当着那些雇工的面说自己没有找着他。随后他又说:“用不着担心,一定是有人因为他冷把他带走了。丢不了。明天早上他一定会回来喝汤的。”‎ 第二天,他没有回来。‎ 他的亲戚们在一个星期里假装着到处打听他的消息,到处找他。他们甚至还哭了几声。‎ 那一年冬天十分冷,很迟才解冻。一个星期日,农民们上教堂做弥撒,发现一大群乌鸦在平原上空不停地盘旋,然后像一阵黑乎乎的雨点集中落在同一个地方,一会儿飞走,一会儿又飞回来。‎ 接下来的一个星期里,这些乌鸦还在那里,它们像一片乌云似的浮在天空,简直可以说四面八方的乌鸦都聚集在这里了。它们常常落到亮闪闪的雪地上,在上面铺下一片怪里怪气的黑点子。顽固地搜寻着。‎ 一个小伙子跑去看看它们究竟在干什么,这才发现了瞎子的尸体,已经支离破碎,被吃掉了一半。他那双无光的眼睛已经不见了,让乌鸦啄走了。‎ 现在我遇到有太阳的日子感到舒畅快乐的时候,就不禁要想到这个可怜虫,心里泛起一种凄凉的回忆和莫名其妙的悲哀,是啊,他在世上是这样命苦,以至于见过他的人听说他惨死,反倒感到一阵轻松。‎ ‎1.下列对小说有关内容和艺术特色的鉴赏,最恰当的一项是(  )‎ A.小说开篇写到光明给正常人带来了喜悦和幸福,人们用眼睛畅饮着天空、田野、房舍的鲜艳色彩;作者只是想给小说营造一种轻松愉快的氛围,其实这与小说后面的感情基调是不吻合的。‎ B.瞎子什么事也不能做,并且老是冷冰冰地不声不响,因此惹恼了他的亲戚们,如果他能有一技之长,或者活泼些,就不会被亲戚抛弃。‎ C.无论是农庄的人,还是瞎子的亲戚,虽然他们身体没残疾但都不是正常的人,他们都在欺负瞎子,他们想出各种残忍的恶作剧来折磨瞎子,这些都不应是正常人对待残疾人的做法,这群人的同情心已经泯灭。‎ D.在一个十分寒冷的冬天的早晨,瞎子的姐夫故意把瞎子带到很远很远的一条大路上去求乞,把他撂在那里一个星期,一个星期后还是乌鸦最先发现瞎子已经死了,瞎子是可怜的,他感觉不到人世间的温暖。‎ 答案 C 解析 A项,开篇的描写是为了引出瞎子的故事,起衬托作用;B项,有道理,但文中并没有这方面的暗示;D项,时间上有错,文中只是说“一个星期日”农民们看见乌鸦落在瞎子死的地方,并没有说是瞎子死后的第几个星期日。‎ ‎2.文章的第一段能删去吗?为什么?‎ 答:____________________________________________________‎ ‎____________________________________________________________‎ 答案 不能删去。第一段渲染了一种欢快、愉悦的气氛,与下文瞎子的命运形成鲜明的对比。‎ 解析 文章的第一段细致地描述了“看见初升的太阳”的喜悦,这看似与“瞎子”没有什么关系,但联系下文内容——瞎子的艰难处,就可看出这两者的联系:对比。‎ ‎3.小说运用近似夸张的手法描写乌鸦,“像一阵黑乎乎的雨点集中落在同一个地方”“像一片乌云似的浮在天空”,这样写有什么作用?‎ 答:____________________________________________________‎ ‎____________________________________________________________‎ 答案 这是环境描写,渲染了一种凄凉悲惨的气氛,突出了“瞎子”悲惨的命运:活着受人嘲弄、歧视、虐待,死后被乌鸦吃掉。这样写,加重了小说的悲剧色彩。‎ 解析 对乌鸦的描写集中在倒数第三、四段,营造了一种阴沉、凄凉的气氛,就是在这种气氛中人们发现了瞎子的尸体。这一环境衬托,让读者更加感受到了瞎子命运的悲惨。‎ 三、阅读下面的文字,完成后面的试题。‎ 百羊川 赵文辉 百羊川坐落在豫北沿儿茄庄村后面的山坡上,别以为真容得下百只羊撒欢,豫北不好找策马扬鞭的场地,更别说在山上。百羊川才一亩几分地,居然平平坦坦,就像山水画上按了一枚印章。这可是枚好印章。茄庄的坡地靠天收,没有机井,山又是个旱山,一秋不下雨,坡上还真收不了几把米。唯有百羊川旱涝保收,越旱小米还越香!老辈人迷信说,百羊川是神田,其实是这块田占对了山脉,下面一定是一根水脉。因水质特别,加上土是黑红黑红的胶土,长出的谷穗又肥又实,碾出的小米喷香喷香,黏度好。明朝潞王落魄于此,一尝便不能相忘,并且年年上贡茄庄小米。又在茄庄修了一座望京楼天天眺望天子,以表忠心。这不过是一段野史,无从考证,倒是当年从豫北走出去的那位副部长,因为爱吃茄庄小米,要把百羊川的主人提拔成公社书记,却是千真万确的。‎ 这主人就是水伯。水伯的祖上就有过要被提拔的经历,说是提一个县令,祖上没去,依然布衣老农,守了下来,就一直守到了水伯这一辈。水伯不稀罕什么公社书记,他只稀罕百羊川的秋天,风吹嫩绿一片,最后变成满坡金黄。农闲的时候,水伯在屋前屋后堆积草粪,坑是上辈人挖好的,水伯只管把青草、树叶、秸秆一股脑儿填下去,再压上土浇上大粪,沤成肥壮的松软的草粪,一担一担挑上百羊川。水伯从祖上接下这个活,一直干到了现在。茄庄的大人小孩都知道,百羊川的小米一直到今天还这么好吃,都是沾了草粪的光。‎ 水伯家的小米每年秋后都有人开着小车来买,买的人多,米少,买主常常为此吵嘴。后来干脆提前下订金,再后来就比价,比来比去,一斤小米的价钱比别人家的竟高出几倍。水伯的儿子受人指点,把“茄庄小米”注册了商标,进城开起了门市部,兼卖一些土特产。几年之后在城里置了房,又要接水伯去。水伯确实老了,锄头也不听使唤了,好几次把谷苗当成稗子锄起来。儿子要留下来照看百羊川,水伯不放心,进城前一再关照:“山后的草肥,多割点沤粪。这几年村里掀房的多,给人家拿盒烟说点好话,老屋土咱都要了,秋后翻地撒进去, 老屋的土,地里的虎 ,百羊川离不开这些!”千叮咛万嘱咐,水伯才离开了茄庄。‎ 儿子却不老实在茄庄侍弄谷子,三天两头往城里来。水伯很不放心,问:“你来了,谁看着百羊川?”儿子说:“雇了村里的光棍老面,老面多老实,叫他给地里上十车粪保证不会差一锨,老面又是种地的老把式,爹你还有啥不放心的?”水伯信了儿子的话,不再为难儿子。再说腿脚也真不中用了,下个楼都要人搀着。有时想回去看看百羊川,又一想自己的腿脚,也就罢了。‎ 这一天,楼下忽然响起一声吆喝:“茄庄小米!谁要?”‎ 水伯的心一阵痒痒,他知道又是一个冒充者。但他知道这冒充者一定是茄庄一带的,他很想去揭穿他,又不忍让他太难堪。家里没有其他人,水伯就强撑着下了楼,问卖小米的:“哪儿的小米?”‎ ‎“哪儿的?还用问?百羊川的!”‎ 水伯笑了,说:“别说瞎话了,我是百羊川的水伯!”几个正买小米的妇女一听,扔下装好的小米走了。卖小米的很恼火,瞪水伯:“你百羊川的咋了?还不跟我的小米一个样,都是化肥喂出来的?”水伯还是笑着说:“你可不能瞎说,百羊川的小米,没喂过一粒化肥,我还不知道?”卖小米的收拾好东西,推着车往外走:“哼,百羊川才一亩几分地,能产多少小米?撑死不过一千多斤!你儿子一年卖十几万斤茄庄小米,莫非你百羊川能造小米?把陈小米用碱搓搓,又上色又出味儿,哄死人不赔命。哼!”‎ 想再问,卖小米的已走远,水伯愣在那里。‎ 水伯一人搭乘中巴回到茄庄,见人就问:“我儿子真的在卖假小米?”被问的人都摇头,说不清楚,问你儿子吧。水伯明白了,踉踉跄跄爬上百羊川。正是初冬,翻耕过的百羊川蒙了一层细霜,一小撮一小撮的麦苗拱出来。麦垄上横着几只白色化肥袋子,阳光一照,泛出刺眼的光,直逼水伯。水伯嗓子里一阵发腥,哇地一口,把一片鲜红喷向了初冬的百羊川,接着扑通一声倒了下去。这时除了一只山兔远远地窥视着水伯,初冬的山坡再无半只人影。‎ 百羊川静极了。‎ ‎(选自《小小说选刊》,有删改)‎ ‎1.下列对小说的思想内容与艺术特色的分析和鉴赏,最恰当的一项是(  )‎ A.有人要把水伯提拔成公社书记,而水伯坚持在百羊川种地,说明水伯为人忠厚老实,认为自己只会种小米,能力不足以做公社书记。‎ B.水伯的儿子也是小说重点刻画的人物,作者对他爱恨交加,既赞扬他有经商头脑,又批评他为了金钱,生产、贩卖假小米的行为。‎ C.水伯家的小米因为好吃,很多人都抢着买,小米的价格因此比别人家的高出几倍,这个细节暗讽水伯家趁机哄抬物价,唯利是图。‎ D.“百羊川静极了”一句十分耐人寻味,环境的寂静衬托出水伯因儿子造假,郁愤而死的凄凉情景,引发读者对重建诚信人格的深刻反思。‎ 答案 D 解析 A项,“能力不足以做公社书记”,文中是“不稀罕”;B项,“赞扬他有经商头脑”错误;C项,“水伯家趁机哄抬物价,唯利是图”错误。‎ ‎2.小说第一段描写了哪些内容?试简要分析其作用。‎ 答:____________________________________________________‎ ‎____________________________________________________________‎ 答案 ①交代百羊川的自然环境,设置人物活动的背景。②交代百羊川的“水脉”“水质”“胶土”,分析百羊川生产出优质小米的原因,为下文写百羊川的小米供不应求、商人造假做铺垫。③以明朝潞王的故事侧面衬托百羊川小米的高质量,为下文写百羊川的小米供不应求、商人造假做铺垫。④写百羊川的主人被提拔做公社书记,侧面反映百羊川小米的高质量,并为下文留下悬疑。‎ 解析 环境是小说的三要素之一,其作用首先从本身分析,交代人物活动的背景,同时起到烘托、渲染的作用。然后从其他的要素分析,从人物的角度是暗示人物的心理和人物的命运,从情节的角度是推进情节的发展,从主旨的角度是暗示文章的主旨,或者升华主旨。本题首先答出描写的内容,然后分析作用。作用要结合环境描写的特征和文章的开头进行分析。‎ 四、阅读下面的文章,完成后面问题。‎ 第二种失败 麦家 第九次冲锋被击溃下来的时候,他悲愤得像一头因重创而恐怖、因恐怖而咆哮的困兽,禁不住仰天号叫了一声。这粗壮的悲鸣,似雄狮的怒吼,歇斯底里,撕心裂肺。‎ 这是一个有风有阳光的日子。就像以往许许多多个美丽的清晨一样,山雾袅袅,轻风送爽,小鸟鸣唱,晨曦把半个山头映得霞光四射,整个是一派充满诗情画意的景致。但是,突然的,这往日的宁静和美妙很快被粉碎了。‎ 是战争打扰了它!‎ 他奉命在天黑前拿下山头。他在八点钟组织了第一次冲锋,然后是第二次、第三次、第四次……一次又一次冲上去,一次又一次被打下来。‎ 现在,绛红的太阳跌落在山顶的另一边,四射着早晨一样的金色光芒,但照耀的已全然不是同一个山坡。这是一个卧满尸体、烈焰升腾的山坡,远看,像开满了杜鹃,又像布满了红旗,既悲惨,又壮丽。山坡上犹如惨遭浩劫一般的褴褛,破败不堪。他看一眼还在继续跌落的太阳,心想,太阳走得真快啊。‎ 山坡下,士兵们又一次在整队集合。他要组织第十次冲锋。‎ 队伍终于横出来了。站在他面前的,只是四十几个衣衫不整的伤兵和哀兵。‎ 同志们……他觉得自己的声音也是很陌生的,又哑又黏,像喉咙里堵满了稠血,敌人还在山上,我们还冲不冲锋!‎ 冲!‎ 他觉得听到了四百个人的喊声,好像山坡上的尸体也在喊。‎ 太阳在继续西沉。他抬头看看落日,又回头看看队伍,知道自己现在只能做最后一次努力了。我再也输不起了,他想。‎ 第十次冲锋开始了。士兵们猫着腰,吼叫着,疯狂地直往山顶扑去。枪声紧密。呼啸的子弹如同雨点一般泼下来,中弹的士兵一个个倒在了大地的怀抱里。‎ 他冲在队伍的前面,时而匍匐,时而跃进,那挥舞手枪的样子,就像在指挥千军万马。可士兵们却一个个在和他告别。‎ 突然,他一下子跪倒在距离堑壕十几米远的山坡上,热乎乎的血瞬间从几个黑暗的窟窿里奔涌不息。子弹钻进他大腿了!‎ ‎“我冲不上去了!”他悲愤地吼叫着。‎ 他看见几个战士飞快地越过了他。这叫他振奋,他拉开嗓门大喊:冲!给我冲!往上冲!‎ 不一会儿,所有没有扑倒的战士都越过他,冲到了堑壕外沿。‎ 终于,一个熟识的身影突然领先跃进了堑壕。很快,第二个也跟着上去了。三、四、五……七……九,他激动了,又一次挣扎着想站起来。可两条腿简直像被灼热的焦土熔化了。他站不起来!‎ 渐渐地,厮杀声开始冷落了下来,而这时一声突然的枪响,似乎像是打出了一个句号。从此,山顶如同死光了人,再没有发出一点声音。‎ 他知道,胜负已经摊牌。‎ 谁胜?谁负?他激动得要死,也害怕得要死。他希望山顶上马上跃出一张他熟悉的脸,或者一个声音。是的,捷报应该是他的。他已经经不起打击了。他努力尽了,也损失尽了,接下来应该请他收获和交代了。‎ 终于,山顶上突然颤颤巍巍地站出了一面旗帜。那是一面褴褛得失去了真实和原貌的旗帜,但他几乎不用看,只是用鼻子嗅了一下,就觉得脑浆飞溅……‎ 那是敌人的旗帜!‎ 山坡上,静静的。‎ 还有人吗?他回头扫视山坡:山坡上,只有一具具静卧的尸体。对着无数尸体,他也忍不住吼叫起来:还有人吗?谁还能冲锋?‎ 只有山谷的回音:谁还能冲锋……还有人吗……‎ 一阵风过,他打了一个寒战,同时也清醒了。恍若一场梦醒来,他接着又看见了辉煌的落日,和满山满坡的尸体,还有那面迎风招摇的旗帜。‎ 难道我这样趴着等着来人把我抬下去,或者拖上去?他想。‎ 不,我不想作为一个伤兵被人抬回去,也不能做俘虏,被他们拖上去。他自言自语道,我要爬上去,我没死,我应该爬上去,爬上去决一死战。‎ 这叫宁死不屈。‎ 他一拱一拱地爬起来了,夕阳的光辉照耀着他,像照耀着一片熠熠发光的金属。他丑陋的身躯如同一条断尾的蚯蚓,只是不息地、顽强不屈地蠕动在焦土和血泥之中,直逼山顶,直逼那面狰狞的旗帜。‎ 堑壕就在眼前,这是多么激动人心!‎ 接下去,他只要来一个翻滚就上了山顶了。此时此刻,他需要的是拿生命作最后一搏的勇气和毅力。但是,他的目光又看到了那面狰狞可怖的旗帜。旗帜似一个威风的巨人傲然凌立,孤独中透露出一股杀气腾腾的神气。它是对方胜利的铁证。看着它,他的冲动和勇气顿时损失了大半。他想,你一个没腿的伤兵爬上去又能怎样?让对方再次享受屠杀的快乐?‎ 他对自己说,我被打垮了。‎ 与其上去让他们屠杀,让他们享受屠杀的快乐,我宁可自杀……‎ 于是,枪口顶在了太阳穴上。是右边的太阳穴。‎ 是的,他是不害怕死的。他害怕的是失败和失败以后的一切。‎ 晚霞如血,光辉抹在他身上,他感到了光辉的压力。其实,作为军人,最重要的是看你关键时候是不是有那种大无畏的、不怕死的、宁死不屈的精神和风度,他想。既然败局已定,那么还有什么可留恋的?‎ 于是,他开始下达此生的最后一道命令:开枪!‎ 枪声拔地而起,直插云霄。‎ 一个小时后,增援部队赶到,不费一枪一弹便冲上山顶。当他们冲上山顶后,发现阵地上唯一的活物是一个奄奄一息的伤兵,他双手紧紧拥抱着旗杆,连松手的力气都没。‎ ‎(有删节)‎ ‎1.下列对这篇小说思想内容与艺术特色的分析和鉴赏,最恰当的一项是(  )‎ A.文章把他的号叫比作“雄狮的怒吼”,把他比作“一头暴怒的、歇斯底里的狮子”,生动地写出了他暴怒的情态,着意表现他对敌人强烈的愤慨、誓与敌人拼到底的决心。‎ B.文章对第十次冲锋时的场景描写,既有对主人公语言、动作的细腻刻画,也有对战士群体冲锋的“面”的描绘,点面结合的写法,增强了文章的画面感、真实感。‎ C.第十次冲锋结束后,山顶终于竖起了一面旗帜,“褴褛得失去了真实和原貌的旗帜”“他几乎不用看”“只是用鼻子嗅了一下”等描写为下面的故事情节设置悬念。‎ D.他有军人的风骨,宁肯壮烈地死,也不肯屈辱地活,所以当败局已定时,他毫不犹豫地向自己举起了手枪,增援部队赶到时,他已全军覆没,点明了“第二种失败”的含意。‎ 答案 B 解析 A项,应该是表现他的悲愤、恐怖、暴怒等情感以及拼到底的决心;C项,“为下面的故事情节设置悬念”错,应该是为他的“第二种失败”埋下伏笔;D项,“败局已定”是他的感受,而不是事实,“第二种失败”不是指全军覆没,而是指他惧怕失败以及失败后所带来的后果,最终被自己内心的恐惧打败。‎ ‎2.小说中画横线部分的景物描写有什么作用?请简要分析。‎ 答:____________________________________________________‎ ‎____________________________________________________________‎ 答案 ①突出战事惨烈。与前文中宁静美丽的画面形成对比,从侧面展现了战事的惨烈。②推动情节发展。日薄西山的画面描写,突出了时间的推移,暗示下文即将开始的第十次冲锋。③以一天结束时的“黄昏落日”暗示主人公最终失败的结局。‎ 解析 本题考查鉴赏环境描写的能力。小说中的环境描写通常对人物形象、故事情节、作品主题起着一定的作用,具体作用还需结合描写的内容来分析。文中画横线句子描写的是战争过后山坡上惨烈、血腥的场景,从整体上来说,与上文宁静的环境形成对比;从细节上来说,对太阳的描写大有深意,早晨到黄昏的时间推移暗示冲锋一直在进行,并还将继续,落日又暗示最后失败的结局。‎ ‎3.文章的结尾既出人意料,又在情理之中。作者这样安排结尾有什么意图?请结合全文谈谈你的看法。‎ 答:____________________________________________________‎ ‎____________________________________________________________‎ 答案 ①从主题上看,他死在自己的枪下,败给了自己内心的脆弱,性格上的悲剧能更好地深化人往往被自己打败(第二种失败)的主题;②从人物形象上看,用奄奄一息仍紧抱旗杆的伤兵与害怕面对失败而开枪自杀的他形成对比,反衬了他的脆弱,展现了作为英雄的他的另一面,丰富了人物形象;③从文章结构上看,小说悲剧式的结尾与开头的悲剧氛围相呼应,首尾圆合,结构严谨;④从表达效果上看,本应是胜利者的“他”竟然在还没有看清山顶的景象时就选择了开枪自杀,引人深思,令人回味。‎ 解析 应注意结合文本自身,从不同角度挖掘。这种欧·亨利式结尾的作用,也需要从小说的主旨、人物形象、故事情节,以及文章的结构作用、手法作用、表达效果等角度思考,当然这些角度不一定每个试题都完全具备。‎ 考点二十一 小说的人物形象 考点名片 考点内容 小说人物的内在情感、思想品质、人物性格。‎ 考查形式 或客观题,或主观题。①文中的×××是一个怎样的人物?②文中的×××有哪些优秀的品质?③文中的×××性格特征怎样?请简要概括。‎ 趋势分析 对人物描写角度的考查或可成为主要命题点。‎ 一、阅读下面的文字,完成后边的问题。‎ 东家 秦悦 东家不像个东家。‎ 一脸疲倦,从粥厂踱回,一碗稀粥,一个黑不溜秋的馒头,吃完,倒杯酒,滋——滋——,那个香甜,那个美妙,仿佛困顿瞬间灰飞烟灭。‎ 东家,富甲一方,闻名遐迩,善饮,常饮康百万酒。‎ 以前,与一帮文人雅士聚,东家说“吴酒一杯春竹叶”“李白斗酒诗百篇”“劝君更尽一杯酒”“桃李春风一杯酒”,还说怀素醉酒书《自叙帖》,书圣王羲之、草圣张旭醉而行云流水挥洒经典。‎ 饥馑,可怕的饥馑来了。东家没了闲情逸致,诗情画意,有的,是雄心,打败饥馑这只魔鬼的万丈豪情。‎ 但,每晚饭后东家的一杯酒,雷打不动。听着东家的陶醉声,下人们大咽口水,心中,一股温暖在奔腾:灾荒,不怕,马上会过去。好日子,不远了!‎ 酒中乾坤大。东家躺下,片刻,沉沉进入了梦乡。‎ 次日,醒来,东家袖手,低头,沉思,慢慢地踱向粥厂,若不是那身半新不旧的长衫,咋看咋像个饥民。‎ 大灾之年,一碗稀粥,对于饥肠辘辘的灾民,就是美味佳肴;一碗粥,能救活生命垂危的饥民,胜过琼浆玉露。东家思绪如烟,摇曳,飘逸。‎ 多少天来,没饿死一人。东家心中满是欣慰灿烂。‎ 东家的粥厂,最大,最红火。‎ 消息长了翅膀,早飞向四面八方。路上,络绎不绝的灾民蹒跚而来。因饿,他们走得比蚂蚁还慢,还缓,摇摇晃晃,像风中瑟瑟发抖的枯叶。‎ 每天,东家的粥厂,热气腾腾,芳香四溢,与饥馑、与死亡顽强而热烈地叫板。‎ 可是,这段时间,管家却形容枯槁,时不时,陷入沉思,长吁短叹。随着时间的推移,管家的忧郁,更重了,更浓了,有时,喃喃自语,没人听清,也顾不上听。其实,他心中有一股火,左冲右突,差点憋不住,想喷发出来,但最终,还是压了下去。‎ 当如血的夕阳又一次不堪重负轰然坠落地平线,东家慢慢地踱回去,吃饭,然后,品酒。管家进来,慌慌张张,吞吞吐吐,唠唠叨叨。东家听明白了,听明白了的东家,很淡泊,很平静,如一池水,水波不兴。‎ 东家神色凝重,一字一板,掷地有声。那是,海一样的胸襟,山一样的气度!管家哑了,默默退出,可是,眼前晃动的,是东家消瘦的脸颊,单薄的身材。瘦而高的东家,如一节长长的竹竿,硬硬地重重地抽打在管家的心上。‎ 失血苍白的日子,让人麻木。没有人清楚,日子的车轮沉重而缓慢地转了多少圈。‎ 始终,管家的心,五味杂陈,潮潮的,热热的。有时,他忧心忡忡,愁肠百结;有时,他无怨无悔,悲壮慷慨。‎ 小麦,珍珠似的小麦,运来了一车又一车。‎ 玉米,金粒一样的玉米,运来了一车又一车。‎ 还有,高粱,大豆,红薯……‎ 都说,东家有个聚宝盆,从盆里取粮,永远也取不完……‎ 粥厂,可爱的粥厂,在多灾多难中,擎起了一面鲜艳的生命之旗。成千上万的灾民,幸运地活了下来。‎ 在望眼欲穿中,大雨终于来了。欢快的雨水,急不可耐地扑向龟裂的大地。无数声音呐喊:老天爷!老天爷!此时的东家,青枝绿叶,绽放出久违的喜悦和激动。‎ ‎“牲口卖了,地卖了,首饰卖了,房契卖了,铺子卖了……”管家再也忍不住了,压抑长久的秘密,呼啸而出,砸得饥民个个像木桩,呆呆的。‎ 饥民跪倒一大片,向东家叩首,泪水雨水交织在一起。他们用最真诚的方式表达心声……‎ 一个个秘密,争先恐后从他嘴里蹦出,再也拦不住,挡不住。‎ 天旱时,东家停了酒的酿造;已酿成的,运往外省,换成了大量的粮食。那时,有官商找上门,让东家造酒,负责销售,说饥荒时没人造,造,可获暴利。东家怒发冲冠,严词拒绝,叹息,谁能做如此暴殄天物泯灭良心的事?!‎ 说着,说着,管家成了泪人儿,哽咽不已。‎ 泣不成声的管家再也讲不出那个藏在心里差点发酵的秘密:东家饮的,不是自制的酒,而是水!‎ ‎1.下列对这篇小说思想内容与艺术特色的分析和鉴赏,最恰当的一项是(  )‎ A.文章开头东家从粥厂回来后,一碗稀粥,一个馒头,吃完,“滋——滋——”喝酒的细节描写,表现出他悠闲自在的生活态度。‎ B.文中写东家的粥厂“与饥馑、与死亡顽强而热烈地叫板”,运用了拟人的修辞手法,生动形象地展示出当时粥厂红红火火的经营状态。‎ C.文中将东家比作长长的竹竿,“硬硬地重重地抽打在管家的心上”,以生动的比喻表现出东家身材的单薄和处境的艰难。‎ D.文中写小麦、玉米、高粱、大豆、红薯等一车车运来,表现了东家实力雄厚,灾难到来时能够从容应对,突出东家形象的高大。‎ 答案 C 解析 A项,“悠闲自在”理解不是十分准确,这一细节描写,一定程度上表现了东家的悠闲自在,而更多表现出的是他希望以自己的行为让大家相信灾荒很快就会过去,坚定对生活的信心。B项,“生动形象地展示出当时粥厂红红火火的经营状态”分析不当,当时粥厂为了救济灾民,压力很大。D项,“表现了东家实力雄厚,灾难到来时能够从容应对”分析不当,由原文“牲口卖了,地卖了,首饰卖了,房契卖了,铺子卖了……”可知,东家是拿出了全部的家当来应对饥荒,他应对得并不从容,这样写也不是为了表现他实力雄厚。‎ ‎2.作者用了较多笔墨写管家,这样写有什么作用?‎ 答:____________________________________________________‎ ‎____________________________________________________________‎ 答案 ①管家是事情的见证人,他知道东家所做的一切,作者不直接写东家的所作所为,而通过管家之口叙述东家所做的一切,使小说极具艺术感染力。②作者对管家进行描写,写管家被秘密折磨着,这就为小说制造了悬念,引发读者的阅读兴趣。③借管家之口在文末揭示出东家的秘密,更衬托出东家形象的高大。‎ 解析 本题要求对管家这一形象的作用进行分析。具体思考时,可以结合文章的具体内容,从其在情节发展中的作用、其对主要人物东家所起的衬托作用等方面进行分析。‎ ‎3.分析东家这一人物的形象特点。‎ 答:____________________________________________________‎ ‎____________________________________________________________‎ 答案 ①有闲情雅致,善饮酒,喜欢与文人雅士聚谈。②充满济世救民的情怀,灾难来临,他倾尽自己的家产让灾民得以存活。③坚守大义,不发不义之财,官商找上门让他造酒获暴利,他严词拒绝。甚至连他的“饮酒”也是自我安慰,实际是自制的水,更显出他救世的情怀。‎ 解析 本题考查欣赏作品人物形象的能力。本题要求对东家这一人物的形象特点进行分析,具体解答时,可以根据文中对东家的描写,概括出答案要点。开头写他对酒的感情,以及饮酒时的情态,可得出答案①;主体部分写他救济灾民的经历,可概括出答案②;文章倒数第三段写他拒绝官商酿酒的要求,可概括出答案③。‎ 二、阅读下面的文字,完成问题。‎ 下坡或者上坡 从县政府往东走,是一条长长的下坡路。当然也可以这样说,从国税局往西走,是一条长长的上坡路。两种说法,指的是同一条道路。人间的事情就是这样,同一个事物,至少会有两种不同的说法。乍一听,好像是矛盾的,是相反的,仔细琢磨,其实是一回事儿。立场不同,视角不同,说法自然就不会相同嘛。‎ 从县政府到国税局之间那条长长的下坡路,几天前发生了一场交通事故,把我和我的朋友老周都牵扯进去了。‎ 老周曾经是我的同事,两个人在同一间办公室里坐对面,关系相当密切。十几年前“下海”热的时候,老周义无反顾地下去了,开办了一个律师事务所。如今,老周的事业红红火火,连奥迪都开上了。我呢,还窝在原先的那个部门。当然也“进步”了,不过跟老周相比,我的“进步”可以忽略不计。‎ 老周不忘旧交,忙里偷闲,会主动来电话,约我出去小聚。去的是老地方,农家菜馆。‎ 那个黄昏跟往常一样,老周亲自开车来接我。跟往常不一样的是,在那个下坡路的中间地段,我们发现前面路边上围了一群人。老周把车停下了。‎ 老周说:“肯定是交通事故。”说完,老周打开车门走了出去。我坐着没动。我不想管这种闲事,弄不好会管出一身麻烦的。这样的例子我见到的不少,听到的更多。原本是一种美德,可很多人都被自己的美德弄得焦头烂额……‎ 我正胡思乱想着,老周回到车上,从公文包里拿出纸和笔,匆匆地写着什么。‎ 我说:“老周,别管闲事。”老周没有抬头,边写边说:“不管不行,弄不好有生命危险。”我知道自己拖不住老周的后腿。他认准要做的事,谁反对都不行。当年“下海”的时候就是这样。‎ 我心里有点儿不踏实,跟老周一起下了车。我看见柏油路上躺着一个白发的老人,脑袋上有血,手上也有。跟我想象的一样,肇事司机逃走了。没逃走的是一群围观的看客。事不关己,看看又何妨呢?何况,看过以后,他们至少会捡到一个话题,跟别人说来说去,还可以发几声感慨,加几句议论,甚至还可以骂骂娘。‎ 老周走到白发老人身边,扶起了他的上半身,把纸和笔都递给他,说:“老人家,你签个字,证明我不是肇事的司机。签完字,我送你上医院。”老人的神志还算清醒,他看了老周一眼,点点头,接过笔,在纸上写下了自己的名字。我注意到,老人的手有些颤抖,字写得歪歪扭扭,而且,还把手上的血蹭到那张“证明”上去了。‎ 这时候围观的人群里发出不同的声音。‎ 一个声音说:“救人要紧,签哪门子字呢?”又一个声音说:“快点吧,出了人命谁负责?”还有一个声音说:“人心不古,人心不古哇。”老周一声不吭。我不知道他在心里想什么。我倒是有些愤愤不平。一大群人没一个肯动手帮忙的,现在倒说起风凉话了。‎ 我们把老人送进了医院。还好,伤势不算太重。医生说,需要住院观察一下。恰好,老人的儿子也赶到了,住院的事情,让他去安排吧。‎ 老人的儿子是个穿着皮夹克的中年人,脸色很阴,似乎有些愤怒。他没去安排住院的事,而是瞪着眼睛走到我们身边,说:“怎么回事儿?”老周把前后的经过跟他说了一遍。“皮夹克”沉默不语,他的表情很怪,似乎不相信老周的话,又似乎是将信将疑。屋子里的空气僵硬了。老周轻轻叹了口气,把“证明”拿出来给他看。“皮夹克”低着头看“证明”,看了很久很久,才从牙缝里挤出了几个冷冰冰的字:“那就,谢谢啦。”说完,一转身走了,连老周的姓名都没想起问一下。老周不是肇事司机,他心里是不是特别失望呢?‎ 我对“皮夹克”非常不满。什么人呀?人家帮了你父亲,你就用这种态度来对人家?老周长长吁了一口气,突然在我的胸脯上擂了一拳,大声说:“走,我们到富丽华大酒店,一醉方休!”富丽华是本地最高档的酒店,我和老周还从来没有一起去过呢。‎ 我很纳闷儿,老周怎么变得如此兴奋呢?‎ ‎1.下列对这篇小说思想内容与艺术特色的分析和鉴赏,最恰当的一项是(  )‎ A.小说中的“我”反对老周救人,反映了“我”的世故与冷漠,说明本质上“我”与车祸现场的“看客”没有什么区别。‎ B.小说描写老人在“证明”上签字时的情形,说明老人可能伤得很重,突出抢救老人的紧迫性,但是围观者的一番议论是不应该出现的。‎ C.小说描写“皮夹克”的行为,既验证了老周坚持让老人签“证明”行为的正确性,也批评了国人的某种阴暗心理的劣根性。‎ D.小说没有明确表明“我”的身份,而是通过对“我”的言行及处事方式的描写,暗示“我”是县政府机关的干部。‎ 答案 C 解析 A项,“我”只是提醒老周别管闲事,以免惹出麻烦,后来的救人行动“我”都全程参与其中,与袖手旁观说风凉话的“看客”还是有着本质区别的;B项,老人可能伤得很重,突出抢救老人的紧迫性,自然会引出围观者的一番议论。D项,“我”的县政府机关干部身份是通过对机关的位置和路的下坡上坡的描写中表露出来的,“我”的言行和处事方式不能表明这种身份。‎ ‎2.小说中的老周具有怎样的性格?请简要分析。‎ 答:____________________________________________________‎ ‎____________________________________________________________‎ 答案 ①忙里偷闲,约“我”出去小聚,体现出重情义,遇到交通事故主动上前体现热心助人。②不在意围观者的态度,体现敢作敢为,有社会责任感。③让老人签字“证明”体现理智冷静,善于保护自己。‎ 解析 概括分析小说中人物性格的特点,一定要结合小说刻画人物的手段和方法进行。在小说创作中,为了刻画人物性格,作者会调动各种手段和各种描写方法;那么,在阅读小说时分析概括人物的性格特征也要从这些方面入手。特别要注意典型的情节、细节描写。‎ ‎3.小说描写了老周救助车祸中受伤的老人又坚持让老人在证明他不是肇事司机的字条上签名的故事。有人赞同老周的做法,认为老周救助他人时注意保护自己;也有人反对老周的做法,认为老周救助他人不应有附加条件。你的看法呢?请就你认同的一种观点加以探究。‎ 答:____________________________________________________‎ ‎____________________________________________________________‎ 答案 赞同老周:①老周既要救人,又设法避免惹上麻烦,这是社会环境逼他如此,老周这样做,既善良又有理性。②老周既救了人,又没惹上麻烦,不论手段如何,结果是最理想的。‎ 反对老周:①抢救生命第一,容不得犹豫、拖延;②见义勇为、舍己救人是中华民族的传统美德,不应该有附加条件。‎ 解析 首先亮明观点。注意观点:一要尽量贴近大众主流,符合时代审美价值观;二要选择自己有话说有较充分理由的观点,不能一味求新求异。列举理由时要源于文本,不能剥离文本,强拉硬扯。‎ 三、阅读下面的文字,完成问题。‎ 侯琢玉 叶孤 侯七家住泗州城郊外,原名侯小宝,因在家中排行老七,所以人们都称他侯七。由于爹妈死得早,加之六个哥哥姐姐也不管侯七的死活,侯七便如孤儿一般在泗州城游荡。侯七虽识字不多,倒也读过几天私塾,在泗州城晃荡,眼明手快,能说会道,不久,便在城中一品轩古董店谋了个伙计的差事。‎ 侯七干活卖力,也很聪明,嘴油舌滑,一品轩老板董玉轩对他甚是喜欢、看重,数年间将他的工钱涨了几倍。而侯七也没有辜负董玉轩的栽培,将一品轩打理得井井有条。老板常常擦拭着玉,自言自语:“嗯,是块好玉……”有时也微微皱眉。‎ 侯七在一品轩一干就是十多年,从一个晃荡的毛头小子干成了三十多岁的大青年。十几年间,侯七倒也虚心好学,除了干好自己伙计该干的事情外,还旁敲侧击地向老板董玉轩讨教学习古董知识。董玉轩何等聪明,知道一品轩之所以能在泗州城屹立数十年不倒,全靠自己对古董行业几十年浸淫的专业判断。如果侯七得到自己真传,势必到哪儿都有立足之地。董玉轩有两子,但一个留了洋,一个当了兵,竟无一子肯跟随他做古董生意。董玉轩也想过将自己的一身技艺找个传人,侯七也曾是他的考虑对象。但董玉轩精明如斯,想到自己一旦倾囊相授,侯七艺满必会离开一品轩,另谋高就,或是另起炉灶,这样自己十几年的栽培也就付之东流了。因此,侯七只学到了些皮毛。侯七也明白董玉轩不会轻而易举地把技艺传给自己,十几年来,他也攒了一笔钱,时刻准备着自己当老板。‎ 一日,店里有一老妇来卖一块玉璧,这玉璧呈椭圆形,头大尾小,通体圆润,光泽明亮。侯七一看就知道这是一块上了年代的真品好玉,一问价,老太太竟要价不菲,说这玉璧是明末长平公主之物,要五百块大洋,侯七虽吓了一跳,但想到这玉璧有几百年历史,又是宫廷之物,如果将之买下然后再伺机出手,说不定可以大赚一把,那时自己另起炉灶就大有希望了。于是经过一番讨价还价,侯七终于用自己攒了十几年的三百块大洋,悄悄将玉璧买下,老板董玉轩自是瞒住不说。‎ 趁一日得闲,侯七悄悄揣玉璧至山阳城,找到了古董界泰斗萧老,萧老一见玉璧顿时两眼放光,侯七见了心中一喜,能让古董界泰斗两眼发出光芒的自然不会是差的玩意了,这回自己多半赚大了。‎ 萧老双眼光芒放完,方道:“的确是一块好玉,应该是明末之玉。”侯七听毕心中又是一喜,自己这个三脚猫竟没有看走眼,这玉璧果然是一块明代的好玉。其实侯七当时并没有十分的把握,完全靠多年的感觉,想不到这老妇竟没有说谎。‎ 侯七正要问这块玉璧价值几何之时,突见萧老将玉璧贴在唇鼻之间,仔细闻了起来,嗅吮再三,萧老兴奋道:“此玉隐约有一股华贵之气!”侯七连忙兴奋道:“卖的人说此玉璧乃长平公主贴身玉璧。”同时他也对萧老佩服不已,连这华贵之气也能闻出来,不愧是古董界的泰山北斗。‎ 萧老一听,恍然大悟道:“难怪,难怪!”继而又叹道:“不过,可惜的是此玉乃长平公主死后下葬时用过的,虽玉质名贵,但裹污秽之气,已经不值钱了。”侯七大惊。他不甘心地问道:“那这玉璧到底能值几个钱?”萧老叹道:“最多三十大洋。”不可能……不可能……我花三百大洋买的玉怎么可能呢……侯七神情恍惚地出了萧宅。萧老在后面又道一句:“如若不信,回去后你用温热毛巾覆于玉璧上,片刻再闻闻你就明白了。”‎ 心灰意冷的侯七回到泗州城一品轩,悄悄拿来热毛巾裹住玉璧,一会儿侯七再拿起玉璧一闻,果然闻到一股臭味。侯七又怒又恼,一把将玉璧摔了个稀巴烂。‎ 侯七后悔已晚,但更后悔自己技不精湛,假如出去开店,迟早得倒闭。侯七下定决心要死心塌地跟董玉轩好好学好好干。‎ 二十年后,管理一品轩数年的侯七已是泗州城数一数二的古董行家。‎ 董玉轩的两个儿子,老大留洋娶了个洋妞定居法兰西再也不回来了,这可把董玉轩气得够呛。而更让董玉轩伤心欲绝的是,老二当兵参加了革命党,被砍了头。董玉轩一病不起。‎ 董玉轩临死的时候告诉侯七,二十年前,那个老妇卖玉,是自己安排的,目的就是让侯七留在一品轩。还说:“一个古董行家没有几十年成不了行家。这人啦,就跟这玉一样……”‎ 侯七也明白了老板的良苦用心,不经过几番磨砺,又怎么能成为今日开了几间分店的侯琢玉呢!‎ ‎(选自《小小说大世界》,有删改)‎ ‎1.下列对小说的内容和艺术特色的分析和鉴赏,最恰当的一项是(  )‎ A.小说善于使用对比手法刻画人物,如侯七偷买玉璧前后行事的变化,正表现了他的成熟过程;而写董玉轩的二子则反衬侯七的虚心好学。‎ B.董玉轩喜欢、看重侯七,但又精明如斯,不肯轻易地倾囊相授,正是作品要表现的重点内容——人性的自私和猜疑。‎ C.董玉轩前后两次对玉生发感慨,既首尾照应,又赋予文本丰富深刻的象征意蕴,有力表现了主题。‎ D.小说情节多处使用陡转技法,如萧老在鉴赏玉璧中“的确是一块好玉,应该是明末之玉”的赞叹,而结尾师傅的临终告白既是出人意料,又在情理之中。‎ 答案 C 解析 A项,对董玉轩二子的叙写不是为了反衬侯七,而是交代董玉轩选择爱徒接班、必定要磨砺他的缘由;B项,“不肯轻易地倾囊相授”,是担心“自己十几年的栽培也就付之东流了”,重点表现出人性自私和猜疑的一面”言之过重。D项,萧老在鉴赏玉璧中“的确是一块好玉,应该是明末之玉”的赞叹不是陡转,“可惜的是此玉乃长平公主死后下葬时用过的”这一叹才是陡转。‎ ‎2.结合文本,分点概括侯七的形象特点。‎ 答:____________________________________________________‎ ‎____________________________________________________________‎ 答案 ①身世可怜,识字不多;②眼明手快,能说会道;③聪明能干,虚心好学;④有事业心,但急于求成;⑤知错就改,踏实肯干,自我磨砺。‎ 解析 本题考查分析人物形象的能力。解答此题,要回到原文,可从原文中筛选出对侯七的评价,如“侯七便如孤儿一般在泗州城游荡”“侯七虽识字不多”“眼明手快,能说会道”“干活卖力,也很聪明”“没有辜负董玉轩的栽培”等;还可从原文中筛选出侯七的行为,他时刻准备自己做老板,偷偷买玉璧,但道行不够;遭受打击后死心塌地跟董玉轩好好学好好干。由这些即可概括出答案。‎ ‎3.这篇小说叙写平凡人平凡事,却给人丰富的人生启示,请结合文本加以探究。‎ 答:____________________________________________________‎ ‎____________________________________________________________‎ 答案 ①成功来自经年累月的积累和自身的反复磨砺。如侯七从小伙计成为数一数二的古董行家,历经了几番“琢磨”“摔打”,正如“几十年浸淫专业”的师傅所言,“没有几十年成不了行家”。②做事要脚踏实地、专心向学,不可急于求成。如侯七瞒着师傅偷偷买玉自讨苦吃,终于明悟“死心塌地跟董玉轩好好学好好干”。③做人应光明磊落、安守本分,不可投机取巧。如侯七瞒着师傅偷偷买玉自讨苦吃是教训,而董玉轩用“欺骗手段”留住爱徒虽用心良苦但也令人深思。④学海无涯,唯勤奋扎实、不断努力进取方可有成。如侯七从知皮毛、“三脚猫”到古董行家的转变即告诉我们这一道理。‎ 解析 从文本的情节中,发现、提取人生感悟,一般可从人物经历的是非、成败、正反等几方面挖掘。提炼启示时采用从个案到一般规律的方法,所有启示必须源于文本。‎ 四、阅读下面的文字,完成1~3题。‎ 信仰 邓石岭 下午两点半,下着细雨,天阴得有点发冷。我刚打开办公室的门,他也跟进来了。我沏了一杯茶,坐定,自顾喝茶,没理他。他没有像以往毫不客气地在沙发上坐下来,而是局促地站在屋子中间,抖瑟着。我没叫他坐,他也不坐。‎ 他姓蔡,70多岁了,是上访专业户,我们都叫他蔡老头。办公室的人陆陆续续地来上班了,看见他,都乐了,都说,蔡老头,又从北京回来了呀?见到某某了吗?某某指的是中央的一个大人物。他说,回来了。又给每个人都敬上烟。抽的是大中华。老头今天有喜事了?我不问,其他人也不问,都去忙自己的事,把老头一个人撂在屋中间。他有话自然会说。他果然就说了,他说,解决了,我的问题解决了。我们都将头抬了起来,你看着我,我看着你,又都一齐看着蔡老头。‎ 老头高兴了,自个儿搬了个凳子,坐了下来,猛吸了一口烟,说,解决了,我的问题解决了。‎ 我说,这么说,他们认定你是1940年入党的了?‎ 老头说,不,他们说我是1949年入党的。‎ 一屋子的人都笑了起来,几个人差点笑出了眼泪。只有老头不笑,像看着怪物一样看着我们。我们也像看着怪物一样看着他。‎ 蔡老头真是个怪物。就为到底是哪一年入党的问题,十几年来,他一直在上访。本来,不管是1940年还是1949年入党的,他都享受离休待遇。可是他固执得很,死认定自己是1940年入的党,十几年里,他跑广州跑北京,工资都倒贴进去了。这一次,他去北京,是去找一个和自己一起出生入死的战友。蔡老头说,是真金,就不怕烈火烧。‎ 我对他说,这么说,蔡老头,这一次你没有找到你那位战友?‎ 不,找到了。老头说,他也说我是1940年入的党,我确实是1940年入的党。就是那一年,日本鬼子的炸弹炸伤了我的左腿。老头说着就卷起他的裤腿,把那碗口大的疤指给我们看。‎ 老头的固执劲又来了。屋子里的几个人又都笑了起来。按惯例,老头接下来会呈上他那一叠厚厚的材料。那一叠材料里有上至中央下至村委会的公章,老头每到一个地方申诉,就一定叫人家盖上公章,明明那公章一点也不管用,可老头信它。那一叠材料里,就有我亲自给他盖上的十九个公章。我拿出公章来,准备给他盖上第二十个公章。‎ 可这次老头没带材料来,见我们都笑,老头张开的嘴又闭上了。我说,蔡老头呀,你这次来,到底想干什么呀?老头搓搓手,说,嘿,我要干什么呀?突然又说,你们都是党员吧?办公室里几个年轻小伙子打趣道,蔡老头,党员又有什么用呀?你还是先弄清自己是哪一年入党的吧。老头激动起来了,满脸通红,牙齿咯咯地响。他大声说,我告诉你,小子,我是1940年入党的,我是老党员了。入党没用,干啥有用?你说说,你说说。泡歌厅、搞腐败,这些就有用了吗?你们这些人,还是共产党员吗?‎ 老头几乎要跳起来了。他说,我这次去北京,我那位战友死了,他死了,我突然想开了,他干了一辈子革命,死了还将遗体捐献给国家,我还在哪一年入党的问题上争什么,我还算个共产党员吗?老头突然“啪”地一巴掌打在自己脸上。‎ 清脆的一巴掌,像打在办公室每个人的脸上。大家都不说话,老头脸色发紫,我脱下一件衣服,披在他身上。‎ 我的心里,流着长长的泪。‎ ‎1.下列对小说有关内容的分析和概括,最恰当的一项是(  )‎ A.小说善于运用环境描写,作品开篇的“下着细雨,天阴得有点发冷”就营造了一种沉重、压抑的氛围为后面蔡老头最终也没能证明自己1940年入党的悲剧奠定了伤感的基调。‎ B.这篇小说故事情节简单有趣,就是把一个场景生动地描绘出来,语言平实且生活化,通过语言表现人物的性格特征和心理变化,富有感染力,叙述也条理清晰,特色鲜明。‎ C.小说采用了多种手法来塑造主人公形象。通过对蔡老头的动作、语言和神态描写,以及其他形象的侧面烘托,准确传神地体现了蔡老头的心理活动和感情变化,从而使该形象更具感染力。‎ D.“我”是蔡老头故事的见证者,也是蔡老头的同情者和支持者,所以“我”曾亲自在他的申诉材料上盖上十九个公章,并准备给他盖第二十个公章。‎ 答案 C 解析 A项,“沉重、压抑”用词过重,“为悲剧奠定了伤感的基调”表述不当。B项,“有趣”用词欠妥,小说的感情基调虽然不是压抑沉重,却也在情节表述上容不得“有趣”二字;D项,“‎ 同情者和支持者”不当。‎ ‎2.文章第一段中“他没有像以往毫不客气地在沙发上坐下来,而是局促地站在屋子中间,抖瑟着”,体现了人物怎样的心理?‎ 答:____________________________________________________‎ ‎____________________________________________________________‎ 答案 体现了蔡老头看到老战友死了之后还把遗体捐献给国家,深受感动,为自己斤斤计较哪一年入党的小事而深深自责、内疚、不安的心理。‎ 解析 人物的动作反映人物的心理,结合文本倒数第三段,可知蔡老头一反往日的毫不客气,表现出不好意思,反映了人物内心的自责、不安与内疚。‎ ‎3.根据小说内容,请你简要分析蔡老头的形象特点。‎ 答:____________________________________________________‎ ‎____________________________________________________________‎ 答案 ①蔡老头为了一个入党的时间问题而上访十九次,体现了他的固执执着;②从蔡老头认识到错误后的动作言行中可以看出他的纯朴正直;③蔡老头对一些人(包括一些党员)“泡歌厅、搞腐败”的愤恨,体现了他的嫉恶如仇;④最后蔡老头深刻检讨,并对党表示忠诚,反映了他的坚持信仰。‎ 解析 从塑造人物形象方面看,可概括为以下几点,首先是对入党时间的上访次数之多,表现了他固执的性格;其次是前后思想的变化及描写,表明他思想纯正的特性;再次是对一些不好的社会现象的痛恨之情;最后从他思想的深刻转变,表现了他对信仰的坚持及对党的忠诚。‎ 考点二十二 小说的艺术技巧 考点名片 考点内容 小说的表达方式(主要是描写手法)、修辞手法、表现手法、结构特色、材料安排等。‎ 考查形式 或客观题,或主观题。①文中运用了什么表现方法来塑造形象的?②文中用了何种表达方式?它是如何为作者表情达意服务的?③某某段在文中有何作用?请简要分析。④某句运用了什么手法?在文中有何作用?‎ 趋势分析 描写手法的作用和结构安排的作用或为主要命题点。‎ 一、阅读下面的文字,完成问题。‎ 小山村 沈祖连 小山村,树绿水清,开门见山,山路弯弯,早有鸟儿啁啾,晚有山雾缭绕。虽然远离城市,缺乏城里的物质文明,可他们也一代代地繁衍了下来。‎ 小山村是和谐的。小山村有一个杂货店,这就是城里的百货商场、超市;小山村有个肉摊,这就是城里的菜市场;小山村有一个小酒馆,这就是城里的饭店酒家;小山村有间小屋,小孩在这里认字,这就是城里的学校;小山村还有一个卫生室,这就是城里的医院。而我所讲的故事,就是在这个卫生室里发生的。‎ 医生的拿手技术是治疗各种疼痛,凡腰痛腿痛手脚痛及各种无名肿痛,经他治疗,没有不好的,这是他祖上传下来的绝技。与其说医生的医术高明,不如说是医生的药物独特。凡此种种疼痛,医生总要使用一种很独特的草药,这种草药叫“一粒珍珠”,也叫“一粒金丹”。刚从土里挖出时,呈银白色,就像一颗颗珍珠,而经太阳一晒,便慢慢变成金黄色,活脱脱一颗颗金丹。看不出这小物竟有神奇功能。病人痛得咧着嘴来,经过一番拨弄,多是笑着走出去的。‎ 据说医生的先祖当年游历海南,在五指山遇到奇人,才得此偏方。到了医生手上,已传了四代。几代人都有着极好的口碑,为人解痛,不图不取,一家人始终住着那低矮小瓦房。不过小瓦房也没什么不好,小山村里的人全都住这种小瓦房。‎ 当然,作为小山村独家医院,只凭一个单方是不行的,见天有几个这样的病人?多数是感冒发热伤风咳嗽,于是,医生也就附设了内科外科、儿科妇科。每天不管看病抓药的人怎样,医生总是有条不紊地工作。他在门口设个排队处,那排队方式竟也独特,每人一块瓦片,或正方形或长方形或不规则形,上面也用瓦片写着一个号。瓦片做笔,瓦片做纸,写出的号码倒也清晰可辨。每次进来一个人,只要你拿出瓦片,那号码是不会错的,依顺序来,不挤不乱。来的都是本村本乡的人,再急也得排队,除非别人主动让你,否则还真不好意思插队。‎ 这天来了辆小轿车,贼黑贼黑的,一直开到了卫生室门口。车里下来一个年轻人,再打开右边的门,扶出另外一个人。被扶的是个上了年纪的男人,头发都变白了。看他一手支着腰胯,一定是痛得不轻。医生正在给村人看病。门外集着一堆手拿瓦片的村人。来人自然没有瓦片。坐在最外边的黎三问,是来看病吗?‎ 是啊,不看病跑来干什么?‎ 是的,不看病来这儿干什么。说得平常,可村人都不大喜欢这种大大咧咧的样子。黎三随手递给他一块瓦片,他却不要,挤到前面,先是掏出烟,顺手抽出一支,塞到医生嘴中,随手打着火机递过去,不由你不抽。一口喷出来的白烟,使得整个屋都香了起来。医生说,啥烟,这么香?‎ 香吗?那就留给你慢慢抽。那人将那包烟放到了桌上,告诉你,大中华,三块五一支。‎ 啊?那可不敢要啊。‎ 那算什么?我们路远,先帮个忙,让我们看吧。‎ 医生稍显为难地看了看外边手持瓦片的村人。村人见来人也不多,就一个,也就默许了。‎ 大概一刻钟,看好了,那人将一张大票留在桌上,问,够了吗?‎ 要不了这么多,我找钱给你。‎ 不用找了。说着便扶着男人往外走。那人走了,秩序又恢复了正常。‎ 过了几天,那人又来了。照样不用瓦片,照样留下一包好烟,照样先看,照样给了一张大钱。只是在走时,向医生要了这里的电话。‎ 好几天没见那人来了。这天有人跑来叫医生到大队部去接电话。医生丢下了正在看的病人,出去了。好一会儿才回来,跟村人说,真对不起,我有点事得到城里一趟,明天回来。说着收拾东西,匆匆出门。村人便只好将手里的瓦片放下。反正也没啥大病,明天就明天吧。‎ 到了第二天,医生真的回来了,是那辆贼黑贼黑的小车送回来的,于是瓦片又派上了用场。又过了十来天,那辆贼黑贼黑的车又来了,是那个开车的单独来的。医生看看手拿瓦片的村人,虽然眼里掠过了一丝内疚,还是上了那车一溜烟地走了。从此,医生十天半月也不回来一次,回来也是匆匆地小住一夜,第二天又走了。村人也再不用瓦片了。‎ 半年之后,小山村里出现了一幢小洋楼,那是医生家的。‎ 小洋楼面对小瓦房,鹤立鸡群,自成风景。只是村人每每路过,那眼睛总是斜视的。‎ ‎(有删改)‎ ‎1.下列对这篇小说思想内容与艺术特色的分析和鉴赏,最恰当的一项是(  )‎ A.文章开头两段交代了小山村的环境,突出小山村质朴、和谐的特征,同时点明故事发生的地点,引起下文。‎ B.文中“与其说医生的医术高明,不如说是医生的药物独特”,含有对医生的讽刺,为下文医生背弃医德做铺垫。‎ C.每天来治疗疼痛的没几个人,“于是,医生也就附设了内科外科儿科妇科”,从中可以看出医生追求利益的心理。‎ D.开贼黑贼黑的小车的年轻人,办事不按规矩,对于其搞特权的行为,村民虽然十分不满却又无可奈何。‎ 答案 A 解析 B项,“含有对医生的讽刺,为下文医生背弃医德做铺垫”说法错误,药物独特并不是对医生的讽刺;C项,当时村民多数是感冒发热伤风咳嗽,医生附设内科外科儿科妇科是为村民考虑,并没有刻意追求利益。D项,“村民虽然十分不满却又无可奈何”不合文意,文中说村民“‎ 见来人也不多,就一个,也就默许了”。‎ ‎2.小说的结尾很有深意,请简要分析。‎ 答:___________________________________________________‎ 答案 ①“小洋楼”和“小瓦房”形成鲜明的对比,突出医生的变化;②村人每每路过眼睛是斜视的,表明医生被村人鄙视;③通过居所的对比和村人的态度,含蓄地表达了作者的态度,升华主旨;④和小说开头交代的小山村的环境形成对照,发人深省。‎ 解析 本题考查赏析作品的内涵,领悟作品的艺术魅力的能力。题干要求分析小说结尾的“深意”,文章结尾段落所写的是医生盖了小洋楼和村里人对医生的态度,从“小洋楼”与“小瓦房”的鲜明对比中可看出医生的变化,即医生变得富有,却丧失了医德。从村里人对医生的态度中可看出作者的态度,即对医生进行批判,对使医生发生改变的现代人的价值观、物质欲望进行批判。明确了这些,便不难得出答案。‎ ‎3.小说中“瓦片”多次出现,有何作用?请结合文章内容谈谈你的看法。‎ 答:___________________________________________________‎ 答案 ①“瓦片”是医生看病排号的凭证;②“瓦片”是小山村人质朴、诚信、守规矩的象征;③“瓦片”失去作用象征着医生医德的丧失;④“瓦片”命运的变化构成文章的暗线,使文章的结构严谨;⑤“瓦片”是在“一张大票”面前失去作用的,引发人们的思考,深化文章的主旨。‎ 解析 本题考查对作品进行个性化阅读和有创意的解读的能力。题干要求分析“瓦片”的深意和作用,从表面上看“瓦片”是村民排队看病的凭证,深入挖掘可知它象征“质朴、诚信、守规矩”,然后分析“瓦片”在串联文章结构、暗示文章主旨和表现人物性格方面的作用即可。‎ 二、阅读下面的文字,完成问题。‎ 知音 余显斌 雪,很大,夜很静。一把火,从他房后烧起,一眨眼间,席卷了整个茅屋。他跑出来,随着他的,只有一把二胡。‎ 他没有回头,即使回头,也看不见什么,因为他是盲人。风吹来,浑身很冷。在风里,他一步步走了,最终,变成一粒黑点,消失在天边。‎ 从此,他漂流异乡。‎ 陪伴他的,是一把破旧的二胡,小镇村庄,一路行来。二胡声,在他走过的地方流泻,如一声声低低的诉说,细细的,蛛丝一样。‎ 夜里,他歇宿在破庙里,草堆后,静穆地坐着,一双盲眼,一动不动,望着虚空。手指颤动,一缕月光水色,从琴弦上淌出,闪着波纹,扩散着,荡漾着。‎ 他走过的地方,要一点剩饭,或者两个冷馒头。‎ 一般的,他只吃一半,另一半,放在自己寄宿的地方,草堆旁,或者是破庙里。第二天走时,留在那儿。‎ 大家都说,这个人,穷讲究,不吃隔夜东西。‎ 他没说什么,摇头叹息。要饭时,仍多要些,拿回寄宿的地方。剩下一些,放在那儿。有时,要少了,他不吃,把要来的东西都放那儿。‎ 这日,一个雪天,他头晕眼花,倒了下去。醒来时,一个女孩的声音,清脆地响起,醒了,你终于醒了。‎ 他点头,慢慢坐起来,很是感激。无物感谢,就拿起二胡,闭着眼,手指颤动,一支乐曲,婉约流淌。‎ 曲子停止了,一切都静静的。‎ 过了很久,女孩醒悟过来,赞叹,你的二胡拉得真好啊,我去告诉师傅,你就跟着我们杂技团吧。说完,女孩一阵风,跑了。‎ 不一会儿,女孩进来了,坐下。‎ 他一笑,道,不收吧?是啊,一个杂技团要一个拉破二胡的干啥啊?‎ 你别急,我再求求师娘。女孩说。‎ 他笑笑,在女孩离开后悄悄走了,一步一步,走向流浪的远方。二胡音,仍如水,随他流淌。时间,也在二胡声中流淌。‎ 他在乞讨和流浪中,慢慢老去。‎ 一日,在一个破庙里,他摸着个人,睡在那儿,奄奄一息。显然,是饿的。他忙拿出讨要的馒头,喂他吃下。两个冷馒头下肚,那人有了精神气,坐起来。那夜,没有旁人,只他俩。他坐在神案前,手指轻弹,两滴乐音溅下,闪着晶亮的光。然后,二胡音悠扬,在静静的夜空响起,一会儿如一缕花香,拂过人心;一会儿如一丝轻风,浮荡如纱。‎ 那人静静听着,罢了,哑着嗓子一声长叹,是《月夜鸟鸣》吧,真是人间一绝!‎ 他笑笑,眨眨已盲的眼,和衣躺下,道,睡吧,明天,还要讨饭呢。‎ 那人,也睡下。‎ 以后,他拉二胡,挣点小钱,养活两人,因为那人也是盲人。夜里,坐在破庙里,他拉二胡,那人听。在奔波中,一天一天,他走向生命的尽头。那天,他吐了几口血,靠在一个草堆旁,对那人说,你不是想得到《松风流水》的乐谱吗?今天,我给你拉。‎ 你——怎么知道?那人惊问。‎ 你眼睛也看不见东西;右手食指有弦痕,是拉二胡的;在这个人世,能欣赏我二胡的,只有两人,一个是个女孩,另一个是我的弟子。他道,脸上有一丝温馨。‎ 师父!那人跪下,不再哑着嗓子,流着泪喊。‎ 他点头,微微一笑,你多次向我讨要《松风流水》的音谱,又悄悄一把火烧了我的茅屋,不就是想逼我带着乐谱逃走,你好中途盗取吗?哎,世间最好的乐谱不在纸上,在心中。这些年,你跟在后面,我知道。没说破,是想让你跟着吃苦,时间长了,就领会了我当年的话。‎ 你留下饭菜,也是给我的?那人哽咽着问。‎ 你脸皮薄,不讨要,会饿死的。他仍一脸平静。‎ 说完,二胡音流出,始如蚊痕,继如流水,最后,如一地灿烂春光。‎ 音乐越来越低,流入地下,渺无音痕。‎ 二胡落下,他也倒下。‎ 你知道是我,为什么不恨我啊?那人抱着他,号啕大哭。‎ 你是我的弟子,我的——知——音。他说,带着一丝笑,咽了气。‎ 那人跪下,恭敬地叩下头去。然后,拿起二胡。月夜里,二胡音如水,波光闪闪,流泻一地。‎ ‎1.下列对这篇小说思想内容与艺术特色的分析和鉴赏,最恰当的一项是(  )‎ A.小说题目为“知音”,从文中我们可以得知,瞎子师傅的知音很少,只有他弟子一人,所以他明知是弟子放火烧了自己的房子,但他也不记恨弟子。‎ B.房子起火后,瞎子师傅只带着一把二胡跑出来,说明他家非常贫穷,没有别的贵重物品。‎ C.本篇小说结尾交代了茅屋起火的原因,以及弟子一直跟随瞎子企图得到乐谱的情节,看上去有点突兀,所以后文作者借瞎子之口进行解释。‎ D.文章结尾,弟子跪在师傅身旁,恭敬地叩头,此时此刻,弟子才懂得师傅的良苦用心,但师傅已死,此时回报师傅的只有传承师傅的绝技。‎ 答案 D 解析 A项,“只有他弟子一人”错,小女孩也是瞎子的知音;B项,“说明他家非常贫穷,没有别的贵重物品”无中生有;C项,本篇小说结尾看似突兀,实则前文已经为这个结尾作了铺垫。如文章开头“雪,很大,夜很静”的描写,为后文表明茅屋是被点燃的作了铺垫;瞎子师傅要饭时总是“吃一半,留一半”,为后文弟子一直跟随他作了铺垫;弟子在漂泊途中第一次见到师傅时故意哑着嗓子,与真相大白时不再哑嗓子,前后呼应。‎ ‎2.本文二胡声音悠扬、动听,作者是如何描写二胡声音的?‎ 答:___________________________________________________‎ 答案 ①侧面描写,小女孩赞叹盲人师傅二胡拉得好。弟子哑着嗓子说,《月夜鸟鸣》真是人间一绝!这都表明师傅拉的二胡声悠扬、动听。②比喻,“一会儿如一丝轻风,浮荡如纱”,突出二胡声音轻柔。③通感,“一缕月光水色,从琴弦上淌出,闪着波纹,扩散着,荡漾着”,生动形象。‎ 解析 本题要求分析“如何描写二胡声音的”,所以回答时首先要找出描写二胡声音的文字。然后分析描写的手法、角度及其作用。‎ ‎3.本文描写了两位瞎子,他们是师徒关系,结合文本探究他们是怎样的人,请结合文本简析。并就其中一位谈谈给你的启示。‎ 答:___________________________________________________‎ 答案 师傅是一个技艺高超的人,从小女孩的赞叹,弟子说《月夜鸟鸣》真是人间一绝,和文中多处对音乐的描写等情节可以看出。师傅是一个以德报怨的人。弟子放火烧了瞎子的茅屋,瞎子不但没有记恨弟子,反而在流浪途中宁可自己挨饿也要给弟子留一半吃的。‎ 弟子是一个为达目的不择手段、但最终能认识错误悔改的人。弟子为了得到瞎子的乐谱,放火烧了瞎子的茅屋,并一直跟随瞎子寻找机会,但最终被师傅感动,跪倒在师傅身旁,认识到了自己的错误。‎ 解析 探究人物形象特点需结合其人在文本中的相关描写,在对人物的性格把握中完成人物形象的概括。结合人物形象谈启示,从对人物自身经历的把握上,用正确的价值观去衡量,并从正反两方面提取感悟,归纳成文。‎ 三、阅读下面的文字,完成问题。‎ 完美的猎物 ‎[加拿大]莫洛迪·坎贝尔 上了年纪的老妇人可以说是最完美的猎物。‎ 萨莎按捺不住内心的窃喜,但表面上却装出十分尊敬的样子:欠着身子饶有兴趣地倾听着老妇人说话。‎ ‎“你喜欢猫吗?呃、呃……刚才你说你叫什么来着?”‎ 萨莎乐不可支,强忍着不动声色。“我喜欢猫,你养猫了吗?”她飞快地回答,同时扫了一眼这套公寓,屋内的高档家具登时映入了她的眼帘。‎ ‎“正打算养,”老妇人颤巍巍地说,“我找钟点工就是为了帮我照料猫咪。”‎ 萨莎朝老妇人笑了笑。她确信自己很有把握做这个活——她以前从未失过手。白发苍苍,不爱打扮,衣着朴素……这样的老家伙们最容易摆平。萨莎要的薪酬比那些中介机构的开价低得多——只要给现金就行,有了现金干什么事情都方便。‎ 老妇人还在那里喋喋不休地唠叨着她的猫,萨莎的眼睛却一个劲地在房间里扫来扫去。昂贵的家具,高雅的雕塑,餐柜上摆放着银光闪闪的蜡烛台,那八成都是些纯银的。毫无疑问,主人的卧室里肯定还有许多珠宝和大量的现金,像她这样的老家伙一般都不大喜欢银行卡。萨莎沾沾自喜,想象着即将到手的果实。寻找贵重物品的藏匿之处对萨莎来说是小菜一碟。‎ ‎“亲爱的,再加点茶?”‎ 萨莎欣然应允,笑容可掬。老妇人拿茶壶的那双手青筋毕露,满是皱纹的手背上布满了深褐色的老年斑。萨莎灵巧地迎上茶杯,让茶壶里的茶水恰到好处地倒入她的杯中。‎ ‎“天哪!”老妇人失声咕哝了一句。‎ ‎“莫蒂菲夫人,没什么事吧?”‎ ‎“哦,没什么,”她显得很尴尬,“实在对不起,我需要去……”,说着,她摇晃着身体从沙发上站起来,拖着沉重的步伐朝卧室奔去。‎ ‎“去卫生间了。”萨莎自言自语地嘀咕着。她一边品茶,一边微笑,即将来临的财运使她非常开心。这些老家伙们从来都懒得查看证明、介绍信什么的,这绝对有机可乘。呵呵,这些老人们都是些菜鸟。‎ 萨莎靠在长沙发上闭上眼睛,接下来的事情对她来说易如反掌。‎ ‎20分钟过后,穿着外套的埃尔维拉·莫蒂菲从卧室走出来,直接来到躺在沙发上的萨莎·萨罕斯卡身边,她仍然步履艰难,但脸上却带着古怪的笑容。‎ 老妇人从萨莎身上飞快地摘下三根金项链、一对厚实的金耳环,还有金手镯和金戒指,她动作如此熟练,令人瞠目结舌。她把这些战利品统统塞进外套里面装有拉链的那个贴身口袋里。‎ 那双枯枝般的手指紧接着又在萨莎的手袋里寻找,翻出了她的钱包。‎ 竟然有800美元!这足以让我们过上一段好日子,老妇人喃喃自语。最近账单上的各种费用都涨得相当厉害。‎ 老妇人把沙发后面的一个包打开,从里面拿出一个长长的蓝色热水瓶,十分小心地把茶壶、茶杯、小奶壶里所有的东西都倒进瓶里,把每件茶具都用事前准备好的布袋套好。整理好这一切以后,她拉上包上的拉链,起身站了起来。‎ 老妇人的眼睛最后在房间里打量着。临时找一套这样的公寓十分方便,房主及其家人一般总是在这个时候离家外出度周末。‎ 沙发上的姑娘仍然沉睡不醒。挑选这样的目标就意味着机会和成功,因为她或他们绝不会去报警。这些年轻人总是自以为是,其实很愚蠢。即使她真的敢去报警,也说不出个所以然来,因为这个年龄的老妇人看上去都差不多,年轻人绝对分辨不出谁的头发更白,谁的皱纹更多。‎ 老妇人弯下腰来注视着自己的猎物,摇了摇头。这些年轻人真的都是一些菜鸟!‎ ‎(选自《译林》)‎ ‎1.下列对这篇小说思想内容与艺术特色的分析和鉴赏,最恰当的一项是(  )‎ A.小说故事情节紧凑,悬念层出不穷,萨莎想要诈骗的对象“老妇人”竟然是个手段更加高明的反诈骗专家,真是螳螂捕蝉黄雀在后。‎ B.“老妇人”的骗术十分高明,她和“萨莎”的一番对话就使“萨莎”放松警惕,放心地在沙发上入睡,可谓是精于此道,深谙别人心理。‎ C.“老妇人”收拾好了所有的用具,并且都用事前准备好的布袋套好,而最后眼睛还在房间里打量,这可能是在检查是否漏出马脚,可见她具有极高的警惕性。‎ D.“老妇人”发现“萨莎”的钱包里居然有800美元,不禁喃喃自语,十分惊喜,可见其偷盗的主要原因是物价飞涨,不得已而为之。‎ 答案 C 解析 A项,老妇人是个诈骗惯犯,习惯于黑吃黑,不是所谓的反骗专家,而且事件也称不上是螳螂捕蝉黄雀在后,是魔高一尺道高一丈。B项,萨莎是被老妇人茶水中的药剂所迷倒,不是自己放心入睡。D项,偷盗的主要原因是物价飞涨无中生有,因果关系不当。‎ ‎2.小说对萨莎进行描写时运用了哪些描写手法?其作用是什么?‎ 答:___________________________________________________‎ 答案 ①语言描写:如“我喜欢猫,你养猫了吗?”“莫蒂菲夫人,没什么事吧?”这表现了萨莎的见风使舵和善于察言观色。②神态描写:“表面上却装出十分尊敬的样子:欠着身子饶有兴趣地倾听着老妇人说话”表现出萨莎伪装出的谦恭和内心的虚伪;“她一边品茶,一边微笑”表现了萨莎自认为即将得手时内心的自鸣得意。③动作描写:萨莎说话的同时扫了一眼这套公寓,“眼睛却一个劲地在房间里扫来扫去”,流露出她的诈骗盗窃的本能反应;“萨莎靠在长沙发上闭上眼睛”,乐不可支,体现其放松警惕,自以为胜利在望,扬扬自得。④心理描写:“即将来临的财运使她非常开心”“这些老家伙们从来都懒得查看证明、介绍信什么的,这绝对有机可乘”,表现出萨莎对自己的骗术十分有信心,并且为此而沾沾自喜。‎ 解析 基本思路是:点明手法——举例说明——表达效果,描写手法的表达效果侧重人物性格这一角度,组织答案时注意分条表述。结合文本可从语言、神态、动作、心理描写等方面进行考虑,再结合相应的描写分析其作用。‎ ‎3.文中“菜鸟”这个词的含义是什么?有人认为这篇小说以“菜鸟”为题更好,请联系文本谈谈你的观点和理由。‎ 答:___________________________________________________‎ 答案 菜鸟即初学者,新手,生手。指刚进入某些圈子,对某些事物操作不熟悉,能力不高的人。‎ ‎(示例一)“完美的猎物”合适:“完美的猎物”,猎物直接体现小说的主题,点明是一起诈骗事件;有利于情节的展开,老妇人和萨莎都把对方当成了自己的诈骗对象即猎物,并且对他们而言都是最“完美”的那一种;题目与文中的首尾照应,圆和如一,突出了中心。原题目合适。‎ ‎(示例二)“菜鸟”合适:题目新颖,引起读者阅读兴趣;有利于凸显主题,老妇人和萨莎都认定对方是菜鸟,围绕这个中心,巧妙斗智,而魔高一尺道高一丈,老妇人成为了最后胜者;“菜鸟”一词贯穿始终,是全文的线索。因此,以“菜鸟”为题是合适的。‎ 解析 本题是探究题,重点考查对作品进行个性化阅读和有创意的解读的能力。答案不求唯一,但要言之有理。解答时要认真分析文章,在把握文章主题思想的基础上,结合文本分析,阐发自己的观点态度。‎ 四、阅读下面的文字,完成问题。‎ 我讲最后一个故事 裘山山 不知是谁提议的,每人讲一个故事。‎ 当这个提议摆上桌面时,桌前坐着的八个人都已经喝得差不多了。‎ 简班长叫简单,如今早已是简处长了,他醉意浓浓地说,我可是天天看报的,大小报都看,从上班看到下班,谁要剽窃报上的现成故事,我马上就能知道,罚他喝酒!‎ 大家一个个说着自己的故事,引来一阵阵笑声。简单说,严亮,你讲讲吧,让我们也了解了解你。他一边说一边看着米晓岚,米晓岚若无其事的样子,但看得出,她的内心并不轻松。‎ 严亮和米晓岚上高中时就挺要好的,上大学后依然情深意长。没想到严亮从军医大毕业后,一下分到了西藏。两人就此分了手。‎ 孙家杰见严亮不讲,就说,这样吧……没等他说完话,张平均自顾自地说,有一天晚上我和几个朋友在自己的酒楼喝酒,喝高兴了,就站起来大声宣布,今晚所有的客人都免单!这下可好,欢声雷动,夸我是个豪爽的老板……‎ 严亮说,比起你们讲的这些故事,我的生活实在太平淡了。我们团驻守的那个地方,叫察隅,海拔跟成都差不多,一年四季都能看到绿色,还有花。‎ 米晓岚插话说,察隅怎么会和成都海拔一样呢?差得太远了。‎ 严亮看她一眼说,你怎么知道?‎ 这是他俩今晚第一次搭话。严亮说,当然,也有很多不一样的地方。我们那儿有一种毒蚊子,它只要在你手上叮一下,你整条胳膊都会肿,半个月才能好。有一天我忽然想到一个问题,蝎子煮熟了吃可以消毒,要是生吃会不会更好些呢?我就决定试试。‎ 简班长说,你小子胆子还挺大。他注意到,米晓岚抬起头来,有些担心地看着严亮。‎ 严亮笑笑说,我找来一个卫生员,让他观察我的情况。结果吃下去后,没有任何中毒反应。我这才大胆地用到临床上,当天就能止疼止痒,三天就能消肿。把我们团长高兴的,给我记了一功。‎ 简单说,严亮,我认为你讲的这个吃蝎子的故事,是今晚最精彩的故事。‎ 大家哄地笑起来,纷纷说,同意!同意!‎ 一个细细的声音突然响起,说,我还没讲呢,你们怎么就评出最精彩的了?‎ 米晓岚笑笑说,我来讲最后一个吧。‎ 米晓岚说,我认识一个女人,读中学时爱上了班上的一个男生。男生也很爱她。他们彼此说了许多山盟海誓的话。后来,男生大学毕业分到了西藏。在家人的坚决反对下,她只好和他分手了,其实坦率地说,家人不反对,她自己也缺乏勇气,也害怕面对西藏……‎ 可是分手后,她怎么也忘不了他,怎么也无法开始新生活。去年暑假,女人终于决定去西藏找那个男生。她想也许见到他,发现他还爱着自己,自己也还爱着他,她就有勇气面对西藏了。‎ 女人坐飞机到了邦达机场,一下飞机,她就被高原反应折磨得像要死掉一样。到分区后得知,去他那个边防团的路被洪水冲断了,而且断了不只一处,正在抢修。她只好住在分区的招待所等。在招待所,她见到了许多要去那个边防团探亲的家属,不少人还带着孩子。那些日子,她听到了太多关于军人家属的故事,那些故事让她非常具体地明白了当一个军人妻子的艰辛和不易,她有些害怕了。‎ 分区司令员请沿线的地方政府协助,先把这些女人送到道路中断的地方,女人们步行走过塌方处后,再由下一段路的地方政府接上,继续往前走。就这样,一段一段地往前移。可是最后来接应她们的拖拉机无论怎么挤,也挤不下所有人。护送她们去的分区干事非常为难。政委的家属见状说,我回分区慢慢等着吧。这时,那个女人拦住了政委的家属,说,还是我下去吧,我不是家属,没道理挤这个座位的,我只是去看一个朋友。她和那些女人分开后,返回了分区,又返回了成都……‎ 严亮直直地看着米晓岚,好像被她的故事定住了似的。米晓岚艰难地笑笑,说,我承认,我没有勇气面对,所以我就……半途而废了。‎ 严亮忽然说,不,不是这样的,这个故事还有个结尾。结尾是,当那些女人到达时,所有的丈夫们都等在路口,他们呼啦一下冲上去,和自己满身风尘的妻子孩子紧紧地拥抱在一起,泪流满面。作为医生,我当时也站在那里等她们。当我看到这一场景时,忽然想,幸好我没结婚。‎ 苗娜忽然大声说,不是说好了不许讲伤心故事的吗?‎ 她的眼泪和话同时涌出。‎ 没有人说话。‎ ‎(选自《北京文学》,有删改)‎ ‎1.下列对这篇小说思想内容与艺术特色的分析和鉴赏,最恰当的一项是(  )‎ A.严亮和米晓岚是一对从中学时就要好的恋人,但严亮从军医大毕业分到西藏之后,两人却分手了,分手的原因是米晓岚缺乏面对西藏的勇气。‎ B.严亮之所以说他们团驻守的那个地方海拔跟成都差不多,一年四季都能看到绿色,还有花,是因为想在同学面前显得体面一些而故意说的。‎ C.小说以“每人讲一个故事”开篇,构思非常巧妙,通过“讲故事”的方式展现了人物之间的关系,“讲故事”也是全文的线索,串联全文,使文章结构严谨。‎ D.严亮看到战友与他们的家属泪流满面地紧紧拥抱在一起的时候,想到自己没有结婚,暗自庆幸自己可以不用承受他们的痛苦。‎ 答案 C 解析 A项,“米晓岚缺乏面对西藏的勇气”这是他们两人分手的原因之一,除此之外还有家庭的反对等。B项,严亮这样说并没有想故意“显得体面”的意思。D项,“暗自庆幸……痛苦”错误,严亮庆幸自己没结婚,是因为他觉得米晓岚不用因他而承受像战友的家属一样的痛苦。‎ ‎2.这篇小说运用了哪些艺术手法?请结合全文简要分析。‎ 答:___________________________________________________‎ 答案 ①铺垫。本文标题为“我讲最后一个故事”,但在米晓岚“讲最后一个故事”之前却用了不少的篇幅写其他人,这是为“我讲最后一个故事”做铺垫,为故事高潮的出现起到了推动作用。②对比。“处长”“老板”的“城市”生活与军人严亮的边疆生活形成对比,聚会开始时充满笑声的热闹气氛与最后一个故事讲完后“没有人说话”的寂静沉重的氛围形成对比,通过对比,升华了文章的主题。③以小见大。本文截取的只是军人返乡与同学聚会的一个小小场景,但就在这样一个小场景之中,通过军人对爱情的抉择,展示出他们平常外表下所蕴含的令人动容的真情与品格。‎ 解析 本题考查分析作品的主要表现手法的能力。考生要在通读全文,明白全文“写了什么”的基础上,进一步思考本文“是如何写的”。从标题与全文内容的安排来看,“我讲最后一个故事”是文章的标题,其内容出现在文章后半部分,那么前半部分的内容就是为后半部分的内容做铺垫的;纵观文章前后内容,可以看出“故事”反映的生活是不同的,聚会前后的气氛也是不同的,明显使用了对比手法;从本文选取的故事发生的场合与所表现的主题来看,属于用“小”场合表现“大”主题。答题时不要忘了答出艺术手法的表达效果。‎ ‎3.在去边防团的途中,米晓岚为什么选择放弃?她的这一举动让你感受到了什么?请结合全文谈谈你的看法。‎ 答:___________________________________________________‎ 答案 第一问:①最后来接应的拖拉机上位子有限,为了给其他“真正的”军人家属留下与亲人见面的机会,她最终选择了放弃。②缺乏勇气面对高原反应、恶劣环境以及对成为军人家属感到害怕,她最终选择了放弃。‎ 第二问:①感受到了米晓岚的真诚善良和她对其他军属的爱心和敬意。②感受到了她对爱情选择的无奈,以及生活的残酷。③感受到了自始至终从未“放弃”的军嫂们的不凡——平常人的爱情、家庭,在驻守边疆的军人和他们的亲人眼里,竟是如此艰难不易。④感受到了身为军人无悔选择背后的悲壮与崇高。‎ 解析 第一问结合文本中女主人公米晓岚的经历和心理活动描写即可得出。第二问需要我们以一个相对客观的角度,从米晓岚和军嫂们的身上进行发掘。注意体会前面的故事都是为最后米晓岚和严亮的故事做铺垫,小说中心就是借此表现守卫边疆的军人和军嫂的艰苦奉献精神生活,所以做题时要懂得把握这一基本点。‎ 考点二十三 探究文本意蕴、人文精神 考点名片 考点内容 文本所蕴含的思想意义,流露出的感情等多种内容。‎ 考查形式 ‎①小说表达了怎样的主题?②作品对你有何启示?③小说告诉了我们什么道理?④文章意蕴深刻,请结合文本探究。‎ 趋势分析 更趋向于考查文本给人的生活启示。‎ 一、阅读下面的文字,完成问题。‎ 女法官的泪水 奚同发 刘大爷走进那间两面流水檐的平房时,索小叶已在那个办公室当了七年的小镇法庭庭长。与以往不同的是,刘大爷面对热情让座的索小叶却摇头摆手说:不坐,不坐,小叶法官,跟我出来一下嘛!‎ 行!女法官说着双手整理一下头发,把放在桌头的大盖帽拿起来,戴端正,跟刘大爷出门。左腿跨出门槛,右腿还在门里,眼前的情形让她大吃一惊。院里站着一个个树桩似的小伙子,少说也有三四十人,个个身强力壮,而且不像往常庄稼人那样穿得松松垮垮。‎ 刘大爷说话时嗓音有些沙哑,态度却异常坚决:小叶法官,你可不能走呀。索小叶一头雾水:我到哪儿去啊?刘大爷说,乡亲们都知道了,都在传,你要调走哩。他们找到我,让我一定把你留下。我知道,你三十多岁,要相亲哩,咱这儿有的是好小伙。你看,刘大爷用手一指院子说,这是我从各村找来的,你随便挑哪个都行……‎ 索小叶的脸“腾”一下红了,一时间不知如何开口,心里翻江倒海,五味涨潮,双眼噙上泪花子……‎ 原来,几天前索小叶与外地来的同学在镇上小饭馆吃饭,同学给她介绍对象,并说对方有能力把她调到市里。这事被乡亲听到,就传开了。‎ 索小叶当法官的小镇,依山天成,其权力范围有很大一部分延伸到山的深处。加上经济不发达,穷乡僻壤,几乎没有一个法官可以干三年以上。索小叶却在这里工作了三年、四年、五年、六年、七年,村民们已习惯了有事就找索法官。针对村民的官司,索小叶尽量庭外调解,以减少当事人的花费;就是开庭,也常常是带着书记员把法庭搬到农家的小院或炕头。断案环境一变,案子办得顺利多了。农村乡邻,鸡零狗碎,哪有什么解不开的结呀!‎ 七年中,索小叶调走的机会多的是。她不走,有一个最重要的原因,她不会跟别人说,也不能跟别人说。‎ 索小叶大学毕业后被分配到市中级人民法院,从书记员做起,由于兢兢业业的工作态度和后来参加考试的优异成绩,很快成为审判员。当法官,是她儿时的梦,农村的童年给她留下了太深的烙印。邻居上访被警察打死在派出所里,虽然行凶者得到应有的惩罚,可判给他家的经济赔偿,一直因为派出所缺少经费而不能执行。一位身躯如弓的老人,从此踏上漫漫的上访路,一床黑乎乎的被卷外裹塑料布走南过北,让人心酸。当时年幼的索小叶咬牙,立志长大要做一名法官,虽然如愿了,但她的理想很快被破坏了……‎ 那是一个让她感到无比屈辱的过程。看完卷宗,经历几年的职业训练早已能平静如水的她怒气冲天地把材料摔在案头:严惩!脑海里飞快地以各条款法律量刑。这时电话响了,是市委副秘书长打来的。对方暗示她这个案子不能判有罪,而且说这不是他本人的意思,是谁谁的意思。她的大脑乱起来,第一次面临艰难的选择:一边是法律的公正;另一边将影响到她未来的发展,甚至是这个案子后她的直接命运。‎ 最终让她崩溃的是她无法回避的制度。虽然想顶住,但由于院长的压力,三人的合议庭两位陪审员以二对一的比例,压倒她的意见。作为审判长,她不得不第一次流着泪去写判决书,几次把笔摔了,又不得不捡回来,这是她的职责,不仅要写,还要找理由,把这个她本来反对的结果写得有理有据。作为一名法官,自己竟然受到法律的伤害……最终选择离开,在得知小镇的情况后,她毫不犹豫地来了。在这个小天地里,她才找到了当法官的感觉。‎ 怎么可能走?她能到哪里去?‎ 望着刘大爷,索小叶哽咽着说:都回吧,让大伙……都回吧……‎ 老人像做错了事似的,结舌说,看……不上,再,另寻……‎ 她强忍着泪说:回吧,让大家回吧……我不走……‎ 说出最后一句最关键的话,她摇摇手回到办公室,顺手关了门。腿发软,背倚门,泪水肆无忌惮地奔泻而下,已听不清小院里的人是如何离去的……‎ 这是索小叶当法官后第二次流泪。‎ 果真一诺千金,她在小镇上生活了近二十年,一直没有出嫁,49岁因病而逝。临终时她说:真的舍不了小镇,就让小镇作为我最后的家吧!‎ 此前,小镇从来没有立碑的习惯,但是大家在进镇的路口为她立了一块两米多高的石碑。石碑正面,因大家争执不休,不得不写成“闺女姐姐妈妈”三个并列竖排称谓,下面才是“法官索小叶”,但是老人们还是玩了个心眼儿,前两个字写得稍大,由于每年清明都有人用金粉为字一层层上色,进镇的人远远地就可以看见两个金光闪闪的大字——闺女。‎ ‎(选自《北方文学》,有删改)‎ ‎1.下列对这篇小说思想内容与艺术特色的分析和鉴赏,最恰当的一项是(  )‎ A.为了让索小叶安心在小镇工作,刘大爷费尽心思地找来许多小伙子让索小叶相亲,索小叶被深深地感动,因而答应大家不离开小镇。‎ B.作者善于通过神态描写表现人物心理,以形写神,如“脸‘腾’一下红了……双眼噙上泪花子”就写出了索小叶当时的羞涩和感动。‎ C.索小叶调走的机会很多,她不想走的原因是以前自己在市中级人民法院判案时得罪了上级领导,她怕这件丑事被乡亲们知道。‎ D.小说通过典型情节塑造人物形象,如“立碑”的情节就体现了索小叶在村民特别是老人心目中的重要地位,突出索小叶为人民服务的形象特点。‎ 答案 B 解析 A项,“索小叶被深深地感动,因而答应大家不离开小镇”错,索小叶本来就不想调离小镇。C项,“不想走”是因为“在这个小天地里,她才找到了当法官的感觉”,且“得罪了上级领导,她怕这件丑事被乡亲们知道”曲解文意。D项,“在村民特别是老人心目中的重要地位”说法不准确,小说结尾“老人们还是玩了个心眼儿,前两个字写得稍大”是为了突出老人们的可爱,其实在所有村民的心目中,索小叶的地位都是一样重要。‎ ‎2.主人公索小叶是可敬的,但她的一生也让人觉得可怜,请结合小说内容,探究索小叶“可敬”和“可怜”的原因。‎ 答:___________________________________________________‎ 答案 “可敬”的原因:①为了真正实现自己当法官的梦想,竟然一辈子坚守在偏僻的小镇;②冒着牺牲自己的前途的危险,顶住压力,坚决捍卫法律的公正;③她真诚对待小镇乡民,为小镇乡民服务,赢得了乡亲们的尊敬。‎ ‎“可怜”的原因:①作为审判长,面对错误的审判结果,她却不得不违心而无奈地履行自己的职责,身为以法律为武器伸张正义的法官,反而被法律伤害;②为了一个承诺,索小叶牺牲了自己的终身幸福,且英年早逝。‎ 解析 本题考查探究能力。“可怜”与“可敬”都是总体性的评价,实质上考查考生对小说主人公的精神品质的综合评价与分析。“可敬”的角度是从索小叶的精神品质来说的,“可怜”的角度是从索小叶的人生幸福问题(或个人问题)来说的。所以,答原因时,一定要结合文本内容,从有利于观点的角度进行分析,做到自圆其说。‎ 二、阅读下面的文字,完成后面试题。‎ 命若琴弦 史铁生 ‎①老少二人都赤着上身,各自拎了一条木棍探路。这正是说书的旺季。老瞎子领着小瞎子一个村子一个村子紧走。老瞎子心里算定:弹断一千根琴弦的日子就在这个夏天了,说不定就在前面的野羊坳。‎ ‎②老瞎子说书开头常是这么几句:“自从盘古分天地,三皇五帝到如今,有道君王安天下,无道君王害黎民。轻轻弹响三弦琴,慢慢稍停把歌论,歌有三千七百本,不知哪本动人心。”‎ 于是听书的众人喊起,老的要听董永卖身葬父,小的要听武二郎夜走蜈蚣岭,女人们想听秦香莲。这是老瞎子最知足的一刻,他不慌不忙地喝几口水,待众人的吵嚷声鼎沸,便把琴弦一阵紧拨,唱道:“今日不把别人唱,单表公子小罗成。”或者:“茶也喝烟也吸,唱一回哭倒长城的孟姜女。”满场立刻鸦雀无声。‎ ‎③这天晚上,在野羊坳说书。书正说到紧要处——“罗成回马再交战,大胆苏烈又兴兵。苏烈大刀如流水,罗成长枪似腾云,好似海中龙吊宝,犹如深山虎争林。又战七日并七夜,罗成清茶无点唇”。老瞎子把琴弹得如雨骤风疾,字字句句唱得铿锵。小瞎子听见那小妮子兰秀站在离他不远处尖声细气地说笑,却心猿意马,手底下早乱了套数。‎ ‎④野羊岭上有一座小庙,离野羊坳村二里地,师徒二人就在这里住下。小瞎子蹶着屁股烧火。老瞎子坐在一旁淘米,凭着听觉他能把米中的砂子捡出。老瞎子终于开了腔:“听我一句话,保准对你没坏处。以后离那妮子远点儿。”‎ ‎⑤一早起,小瞎子病了。一连好几天,老瞎子无论是烧火、淘米,还是给小瞎子挖药、煎药,总想那张药方和琴弦。弹断最后一根就可以去抓药了,然后就能看见这个世界:他无数次爬过的山,无数次走过的路,无数次感到过温暖和炽热的太阳,梦想着的蓝天、月亮和星星。心里总在说:“我非要最后看一眼不可。”晚上仍一个人到野羊坳去说书。终于弹断了最后一根琴弦。他几乎是连跑带爬地上了野羊岭,回到小庙里。‎ ‎⑥“明天我就去抓药。你就先留在这儿,我用不了十天就回。”‎ ‎⑦老瞎子早忘了兰秀儿的事。“吃的、喝的、烧的全有。你要是病好利索了,也该学着自个儿出去说回书。行吗?”‎ ‎⑧“行。”小瞎子觉得有点对不住师父。‎ ‎⑨蛇皮剥开了,老瞎子从琴槽中取出一张叠得方方正正的纸条。小瞎子把那药方放在手里摸了一会儿,也有了几分肃穆。“你师爷一辈子才冤呢。他本能弹够一千根,可他记成了八百。要不然他能弹断一千根。”天不亮老瞎子就上路了。‎ ‎⑩回到野羊坳时已经是冬天。小瞎子已经走了些日子,村里人都说是在兰秀嫁到山外去的那天。老瞎子心里便一切全都明白。若不是还想着他的徒弟,老瞎子就回不到野羊坳。那张他保存了五十年的药方原是一张无字的白纸。请了多少个识字而又诚实的人帮他看,人人都说那果真就是一张无字的白纸。“您别死,再活几年,您就能睁眼看一回了。”说这话时他还是个孩子。他师父久久不言语,最后说:“记住,人的命就像这琴弦,拉紧了才能弹好,弹好了就够了。”‎ ‎⑪在深山里,老瞎子找到了小瞎子。“师父,您现在看得见了?”小瞎子伸手去摸师父的眼窝。老瞎子把他的手抓住。“记住,得弹断一千二百根。”“一千二?”“把你的琴给我,我把这药方给你封在琴槽里。”老瞎子现在才弄懂了他师父当年对他说的话——咱的命就在这琴弦上。‎ ‎⑫莽莽苍苍的群山之中走着两个瞎子,一老一少,一前一后,两顶发了黑的草帽起伏攒动,匆匆忙忙,像是随着一条不安静的河水在漂流。无所谓从哪儿、到哪儿去,也无所谓谁是谁……‎ ‎(选自史铁生的《命若琴弦》,有删改)‎ ‎1.下列对这篇小说思想内容与艺术特色的分析和鉴赏,最恰当的一项是(  )‎ A.小说多处描写老瞎子为治好自己的眼睛不辞辛劳奔波说书,到后来发现“药方”只是一张白纸时,顿时感到自己的生命之弦骤然断裂,一切都失去了意义。‎ B.小说第⑤段细致描写了老瞎子弹断最后一根琴弦前的心理活动,表现了老瞎子对治好眼睛的渴望,为后文发现“药方”实为一张白纸而蓄势。‎ C.小说的构思巧妙,它让我们看到了一个瞎子艺人的一生,也看到了众多底层人民的生命状态——人生的残酷与命运的无奈。‎ D.这篇小说,以瞎子艺人追求睁眼看到光明为内容,倾诉了残疾艺人们内心对命运不公的愤慨和不平之气。‎ 答案 B 解析 A项“一切都失去了意义”有误。正是意识到生命的意义不在结果而在过程,老瞎子才决定将这个“药方”继续传给徒弟。C项“众多底层人民”不准确,小说所反映的人生的苦难和命运的无奈并不只针对底层人,而是所有人都有可能的际遇。D项“愤慨和不平之气”有误,而是对命运的接受和奋斗。‎ ‎2.小说的最后,老说书人已经知道所谓的“药方”根本不存在,为什么他还要将这个“药方”‎ 继续传给徒弟?请结合小说的主题和现实生活,谈谈你的理解。‎ 答:___________________________________________________‎ 答案 ①老瞎子终其一生翻山越岭说书谋生,他相信这一切的奔忙辛苦都是值得的,就在于弹断一千根琴弦能用药方去治眼睛。可见这“药方”是老瞎子困苦生活的目标和希望,是生活的依托。虽然最终没能治好眼睛,但老瞎子明白是这“药方”支撑他一直活得充实,所以他认为有义务把生活的希望传给下一代。②小说的题目是“命若琴弦”,意在告诉人们,生命就像琴弦,只有“拉紧了”,让自己忙碌着、奔波着,生命才有意义。意义本在于过程,而无所谓“从哪儿来,到哪儿去,谁是谁。”或者:老瞎子的师父说“人的命就像这琴弦,拉紧了才能弹好,弹好了就够了。”传达出了小说的主题,即生命的意义在于忙碌厚实的过程,目标是什么,能否实现,并不重要。③现实生活常常是无奈的,甚至是苦难的。要在心中给自己树立一个目标,让生活充满奋斗的原动力,让自己成为一个内心明亮的人。哪怕最终不能实现,也能让生命中的每一天都充满意义。找到一个理由,为了更好地活着。‎ 解析 注意从文章标题、人物性格特征和文章主旨的角度分析,然后结合现实谈自己的看法。小说的解读注意五位一体的原则,即“主题”“环境”“情节”“人物”“标题”五位要同时解读,分析一种要从其余的几个角度进行分析。分析情节安排的合理性,首先答情节的内容是相辅相成,还是矛盾的对立,然后这些情节在人物的塑造、环境的再现、主旨的表达以及和标题的照应的角度进行分析。‎ 三、阅读下面的文字,完成问题。‎ 苦梅 马卫 苦梅又叫乌梅,个小,肉少,汁酸。如果不是没有吃的,孩子们是不会去吃苦梅的。一个姑娘却偏偏取了这么个名儿,不过也难怪,山区农村生活苦嘛。苦梅一出生,就和饥饿结伴,要不是后来包产到户,苦梅一家的日子就更难过了。‎ 苦梅二十岁的时候,成了新媳妇儿,“新媳妇儿,憨憨人儿,不说话儿,给糖走人儿。”儿歌嘛,小孩子嘴甜,诳糖吃。到了夜深人静,客人们离开,苦梅才发现,进新房的怎么不是先前相亲的那位男人呢?‎ 男人一脸的木讷,呆呆地站着,半天才嗫嚅出来,以前相亲的,是他表哥;自己丑,所以找表哥代替。事已至此,多说无益,何况,男人家拿了钱,给家里砌了砖房,让哥娶上了嫂子。拿妈的话说就是——长得帅,能吃么?‎ 男人没有啥手艺,除了干地里的活,干不了其他。可是粮食价格低,除去农药、化肥、种子,赚不到啥钱。就这样过了七八年,孩子也读小学了,家里仍然一贫如洗。‎ 苦梅对老公说,这样下去,不是办法,一辈子也致不了富。‎ 男人却安于现状,不缺吃不缺穿,还要求啥呢?可是经不住苦梅天天纠缠,最后男人让步了,苦梅和她的表姐进城,一个去给人帮厨,一个当超市营业员。‎ 苦梅在超市,极认真。别人忙里偷闲玩手机,她没有。别人喜欢小喇叭广播,讲奇闻怪事,苦梅不知道。苦梅就像个机器人,陀螺似地转,特别热心给人介绍商品,帮老人购物。‎ 苦梅的勤奋和本分,换来了一个领班的岗位。不比不知道,一比吓一跳,当上了领班,工资竟然是营业员的倍数,而且轻松多了。所以,苦梅工作更加认真。‎ 苦梅出事,是在一个周末。‎ 那天,她在商场巡视,检查工作,发现有一位叫巧芋的营业员偷吃话梅。如果偷吃几个,谁也不会发现,商品有一定的损耗率。但是规定是死的,营业员绝不能吃和拿商场的商品。一旦发现,轻则扣工资,补损失;重则开除,或送派出所。‎ 苦梅没有当场揭露巧芋,女人家谁不爱面子呢?‎ 把巧芋叫到办公室,给她倒上水,才说了自己看到的事。没想到,巧芋一百二十个不承认。她说:提奸捉双,拿贼拿赃。你既然看到了我偷吃话梅,为啥不当场拿住?现在,你是污蔑我!我和你没完。‎ 苦梅没想到好心没有好报,气得粗气红脸:这次就算了,如果我再拿住了你,你走人!‎ 苦梅只能这样说了。她知道,这些营业员,差不多全是乡下来的,拿着低薪,还要养家糊口,难呢。只要巧芋改了,这事就算了。‎ 苦梅和表姐合住的出租屋,表姐回来一般要一点左右。那晚,苦梅值晚班,回家时快十一点了,还在夜摊上吃了碗米粉。就在苦梅开门的时候,突然蹿出一位蒙面男人,把她踹倒在地,给她几拳,几脚,嘴上说:臭婆娘,懂事点,不要管得太宽。这次放过你,要是你再多嘴,把你脸划了!苦梅虽然来自农村,有一把力气,可是相对于男人,那是蚍蜉撼树。‎ 蒙面人走了,半天苦梅才起来,回到屋内,见腿上身上的瘀痕,忍不住哭了起来。不用想,这事肯定是巧芋干的,听说,她一人在城里,常和些不三不四的男人鬼混。所以,巧芋特别好打扮,身上也不差钱,上班极不认真。‎ 表姐回来的时候,见苦梅这副样子,问了半天,苦梅才嘟咙出来。‎ 苦梅并不怕巧芋,是怕巧芋勾搭上的野男人。为了安全,苦梅辞职了,经理挽留她,她坚决回家。孩子见她回来,高兴地溜到她怀里,就不离开。老公见她回来,嘿嘿直乐。‎ 伤不重,养了一周,就复了原。‎ 苦梅是个闲不住的人,经过三年的打工生活,她明白一个道理:要得富,开店铺。‎ 苦梅开了村里第一家超市,当然不能和城里的比,不过两间屋大,主卖副食和生活必用品。不用请人,自己和老公亲自干。货自己去进,晚上做台账。‎ 乡里人虽然计较价钱,但绝不会偷东西。一个月下来,挣的比她在城里打工还多一倍。‎ 苦梅家渐渐富了,有了彩电,有了冰箱,还给老公买了摩托,进货也方便。至于田和地,只种谷子、蔬菜,剩的种树。‎ 她家没有种其他树,只种苦梅。人们不太理解,苦梅也不解释。乡下人都知道,苦梅味酸、涩、平,敛肺、润肠、生津、解酒,良药呢。村里人常来讨用,苦梅一概白送,尤其那些爱喝醉酒的男人们,他们嘴上还要占便宜:苦梅,吃你呢!‎ 苦梅的脸上,全是笑。嘴上说:吃我?酸死人!‎ ‎1.下列对这篇小说思想内容与艺术特色的分析和鉴赏,最恰当的一项是(  )‎ A.“长得帅,能吃么?”是苦梅的母亲在苦梅婚前对她的告诫,这说明她母亲早就知道苦梅的丈夫长得丑,但她为了儿子,只能委屈女儿。‎ B.男人诚实,只会干地里的活,没有其他手艺,又安于现状,这一形象与妻子苦梅形成了鲜明对比,突出了苦梅的勇于开拓、心灵手巧。‎ C.本文使用第三人称展开文章的叙述,故事娓娓道来,对情节的描述客观真实,好像读者就是故事发生发展的见证人,更易引起共鸣。‎ D.文章结尾,苦梅说“吃我?酸死人!”使用夸张的手法,意在强调自己种的乌梅未到成熟季节,果实的酸度比较高,又酸又涩,非常难吃。‎ 答案 C 解析 A项,“母亲早就知道苦梅的丈夫长得丑”于文无据,母亲并不一定早就知道丈夫丑。B项,“与妻子苦梅形成了鲜明对比,突出了苦梅的勇于开拓、心灵手巧”不准确。其一,二人形象并不相反,所以不是对比;其二,小说情节没有明显地体现苦梅的“勇于开拓、心灵手巧”。D项苦梅的话,其实只是她对醉酒男人调笑之语的巧妙堵截。‎ ‎2.小说结尾写到“她家没有种其他树,只种苦梅”,她只种苦梅的原因是什么?请结合全文分析。‎ 答:__________________________________________________‎ 答案 ①她的名字是苦梅,从心理上对苦梅有一种亲近感。②苦梅是一味良药,送给需要的人,给人帮助,也可以招徕更多客户。③苦梅这种树寄寓着主人公对人生的感悟和思考。‎ 解析 这是一道探究题,思考时要注意从多种角度发掘“苦梅”(树)和苦梅(人)之间的联系。既然题干要求找出“只种苦梅”的原因,必然二者有解不开的关联,比如情感上的、价值上的,以及感悟情怀上的等等。‎ ‎3.小说题目是“苦梅”,请结合小说内容分析它的妙处。‎ 答:__________________________________________________‎ 答案 ①一语双关,既指乌梅,又指名叫“苦梅”的姑娘,两者相得益彰。②隐喻人物个性。苦梅味酸、涩、平,可隐喻主人公人生的不幸,而其敛肺、润肠、生津、解酒,可隐喻主人公善良的品格。③暗示小说主题:人即使命运不好,也不能失去善良和真诚,而要努力去改变现状。④‎ 能引人思考,激发读者阅读兴趣。以“苦梅”为题,能让人产生了解内容的欲望:何为苦梅?苦梅与人物与主题有何关系?‎ 解析 小说的题目,运用很巧妙。分析时从外在表现形式,从小说主题,人物的形象等角度去看。“苦梅”,一语双关,既是一种树,又是小说的主人公。苦梅的人物形象也和苦梅的特点有相似之处。从小说的主题上看,苦梅命运不好,但凭借她的善良和努力,最终有了一个好的结局。苦梅虽苦,但其敛肺、润肠、生津、解酒,终能被人接受,“苦梅”和文章要表达的主题一致。‎ 四、阅读下面的文字,完成问题。‎ 追风筝的人 ‎[美]卡勒德·胡赛尼 斗风筝比赛是阿富汗古老的冬日风俗。规则是放起你的风筝,割断对手的线。若有风筝被割断,那些孩子就拼命地追逐那个随风飘扬的风筝。对追风筝的人来说,最大的奖励是捡到最后掉落的那只风筝,那是无上的荣耀。每个斗风筝的人都有助手,我的助手——兼仆人与朋友——是哈桑。‎ 那年冬天的一个夜里,距比赛还有四天,爸爸对我说:“我觉得今年你也许能赢得比赛!”爸爸随口一说,却在我脑海中埋下了一颗种子:好办法!让他看看,他的儿子终究非同凡响。‎ 街上新霁的积雪银光闪闪,天空蓝得无可挑剔。哈桑和我走出大门,雪花反射出白晃晃的光芒,照得我睁不开眼。街上,风筝斗士们在做最后的准备。街道传来欢声笑语,各处屋顶挤满了看客。我朝自家的屋顶看去,爸爸挥挥手,我不知道他究竟是跟我还是跟哈桑打招呼。‎ ‎“我们得开始了。”哈桑说。‎ 突然间我想放弃。爸爸在屋顶上看着我,他的眼光像太阳那样热得令人发烫。‎ ‎“我有点不想放风筝了。”我说。‎ ‎“今天是个好日子。”哈桑说。‎ 我试图让眼光离开我们家的屋顶。哈桑上前一步,低声说了一句让我有些吃惊的话。“记住,阿米尔少爷,没有鬼怪,只是个好日子。”哈桑一眼看穿我。‎ ‎“没有鬼怪。”我低声说,出乎意料的是我竟然觉得好些。‎ 不到一分钟,我的风筝扶摇直上。一个钟头后,空中已经挂着至少四十只风筝,如同纸制的鲨鱼,巡游搜猎食物。寒冷的微风吹过我的头发。哈桑在我身旁,他的手掌被线割得鲜血淋漓。第一批被挫败的风筝断了线,像流星那样划过苍天,拖着闪亮的尾巴,散落在临近的街区,给追风筝的人带来奖赏。‎ 我偷眼望向爸爸,寻思他眼下在想些什么。风筝纷纷坠下,而我的仍在翱翔。我双眼不时瞟向爸爸,紧紧盯着他的羊毛衫。‎ 下午三点,阴云密布,太阳躲在它们后面,影子开始拉长。只剩下六只风筝了,我的是其中之一。我双腿发痛,脖子僵硬。但看到风筝一只只掉落,心里的希望一点点增大。‎ 一只蓝风筝在过去那个钟头里大开杀戒。现在,只剩下我的红风筝和那只蓝风筝了。局势紧张得如同我流血的手拉着的线。人们纷纷顿足尖叫:“干掉它!干掉它!”爸爸会不会也在欢呼呢?但我所能听到的是脑袋里血液奔流的声音。‎ 一阵风拉升了我的风筝,我占据了有利的位置。“干掉它!干掉它!”的欢呼越来越响,仿佛罗马人对着斗士高喊:“杀啊!杀啊!”。‎ ‎“快赢了,阿米尔少爷!”哈桑兴奋得直喘气。那一刻来临了,我合上双眼,松开拉线的手。寒风将风筝拉高,线又在我手指上割开一个创口,接着……‎ 不用听人群欢呼,我也知道我赢了!我看到爸爸站在屋顶边缘,双拳挥舞,高声欢呼。我体验到有生以来最棒的一刻。‎ ‎“安拉保佑,我们等会儿再庆祝吧。现在,我要去帮你追那只蓝风筝。”哈桑撒腿就跑。‎ ‎“哈桑,”我大喊,“把它带回来!”‎ 他踢起阵阵雪花,飞奔到街道的拐角处。他突然停下来,转身,双手放在嘴边,说:“为你,千千万万遍!”然后露出一脸微笑,消失在街角之后。‎ 我收回风筝后,寻找每条通道都没有哈桑的踪迹。我正在担心时,天就快黑了。突然听到前面传来一阵声响,这是一条僻静的死胡同。只见哈桑站在末端,拳头紧握,双腿微微张开。在他身后,摆着那只蓝风筝。那是我打开爸爸心门的钥匙。‎ 三个邪恶的大男孩挡住了哈桑的去路。我觉得自己无法呼吸。他们逼近哈桑。‎ ‎“把风筝给我!”其中一个挥舞着拳头。‎ 哈桑眼里流露出恐惧,可是他摇摇头。“这是阿米尔少爷的风筝。”‎ ‎“你这个白痴!把风筝给我们。”‎ 哈桑弯腰捡起一块石头。他们一愣,退后一步。‎ ‎“这是你最后的机会了!”‎ 哈桑高举那只抓着石头的手。‎ 我张开嘴,几乎喊出来。但我没有,我浑身麻木。‎ 他们动手了,哈桑扔出石块,击中了其中一个的额头。他们一拥而上……‎ 有种温热的东西从我眼角流淌下来。我眨眨眼,看见自己依旧咬着拳头,咬得很紧,从指节间渗出血来。我回到一个空无一人的市场,斜倚在一家小店铺紧闭的门前。‎ 十五分钟后,哈桑回来了,手里拿着那只蓝风筝。‎ ‎“你到哪里去了?我在找你。”我艰难地说,仿佛在吞嚼一块石头。‎ ‎(节选自《追风筝的人》,有删改)‎ ‎1.下列对小说思想内容与艺术特色的分析和鉴赏,最恰当的一项是(  )‎ A.小说以第一人称的无限视角展开,不仅真实生动地向读者展现了阿富汗浓郁的异域风情,而且深刻揭示了家庭、友情、人性的丰富内涵。‎ B.斗风筝比赛,既是小说的主要情节,同时也成为了人物活动的重要背景,表现了阿富汗人民的勇敢与好斗。‎ C.小说情节波澜起伏,高潮部分哈桑忍受坏孩子的侮辱追回风筝,揭示了标题的含意。“风筝”既是“我”打开父亲心门的钥匙,也表现了哈桑的勇敢,更象征着无上的荣耀以及每个人心中的成功梦想。‎ D.小说刻画的人物形象栩栩如生,运用了肖像、语言、神态、动作描写,尤其是心理描写,生动揭示了“我”复杂的内心世界。‎ 答案 C 解析 A项“无限视角”错,应该是有限视角。B项“好斗”牵强。D项小说刻画哈桑性格没有运用肖像描写。‎ ‎2.请结合文本分析小说中“风筝”的内涵,并谈谈小说给你的启示。‎ 答:__________________________________________________‎ 答案 内涵:①对“我”来说,是用以向父亲证明自己的东西(或答“是打开父亲心门的钥匙”);②对每个风筝斗士来说,是勇敢与荣誉的象征(或答“风筝象征着每个人心中的成功梦想”);③对哈桑来说,是证明自己与少主人阿米尔的坚固友谊的物品。‎ 启示:①每个人心中都有一只梦想的“风筝”,要勇敢地去追属于自己的梦想。②不计较地付出,是人性最美的花。‎ 解析 对“风筝”内涵的解读要从不同角度出发,从“风筝”的象征意义进行分析。结合文本可知,在“我”的眼中“风筝”不仅是一件赢得比赛的玩具,更是打开父亲心门的钥匙,“我”通过赢得比赛,可以向父亲证明自己的非同凡响;而对于每一位斗风筝的人而言,赢得风筝比赛,并追得风筝,是一种无上的荣誉,一种成功的梦想。对哈桑而言,为阿米尔少爷追回被割断的风筝,更加证明了自己对阿米尔少爷的忠贞和友谊。谈小说给你的启示时,一方面要结合小说内容;另一方面要选择自己最擅长的角度,上升到一定的高度。‎ ‎3.“为你,千千万万遍”,哈桑的这句真诚的话语让人动容,代表着友情的一种高尚境界。请结合小说内容及生活体验,谈谈你对这句话的理解。‎ 答:__________________________________________________‎ 答案 ①这句话是阿米尔打败所有竞争者后,哈桑去帮他捡剪断的竞争者的风筝时说的。②“为你,千千万万遍”是哈桑对阿米尔的一句誓言,表现了他对朋友的忠诚和真挚的友情。③一个人对另一个人说“为你,千千万万遍”,支撑他的是其誓死不改的忠诚和信仰。④当今社会不乏利益至上者,更显友情之珍贵。友情是最纯洁最真挚的情感,不夹杂世俗功利。每个人都渴望这种友情。‎ 解析 注意结合文本分析,答题时尽量分点。首先从语句含义角度理解,答出背后体现出的人物个性特征。谈对这句话的理解时,要结合生活体验和社会现实,比如以当今社会人情友情的凉薄,对比哈桑的忠诚勇敢,答出自己的认识和理解。‎ ‎ [3年高考真题集训]‎ 一、[2016·全国卷Ⅰ]阅读下面的文字,回答1~4题。‎ 锄 李锐 拄着锄把出村的时候又有人问:“六安爷,又去百亩园呀?”‎ 倒拿着锄头的六安爷平静地笑笑:“是哩。”‎ ‎“咳呀,六安爷,后晌天气这么热,眼睛又不方便,快回家歇歇吧六安爷!”‎ 六安爷还是平静地笑笑:“我不是锄地,我是过瘾。”‎ ‎“咳呀,锄了地,受了累,又没有收成,你是图啥呀你六安爷?”‎ 六安爷已经记不清这样的问答重复过多少次了,他还是不紧不慢地笑笑:“我不是锄地,我是过瘾。”‎ 斜射的阳光明晃晃地照在六安爷的脸上,渐渐失明的眼睛,给他带来一种说不出的静穆。六安爷看不清人们的脸色,可他听得清人们的腔调。但是六安爷不想改变自己的主意,照样拄着锄把当拐棍,从从容容地走过。‎ 百亩园就在河对面,一抬眼就能看见。一座三孔石桥跨过乱流河,把百亩园和村子连在一起。这整整一百二十亩平坦肥沃的河滩地,是乱流河一百多里河谷当中最大最肥的一块地。西湾村人不知道在这块地上耕种了几千年几百代了。几千年几百代里,西湾村人不知把几千斤几万斤的汗水洒在百亩园,也不知从百亩园的土地上收获了几百万几千万斤的粮食,更不知这几百万几千万斤的粮食养活了世世代代多少人。但是,从今年起百亩园再也不会收获庄稼了。煤炭公司看中了百亩园,要在这块地上建一个焦炭厂。两年里反复地谈判,煤炭公司一直把土地收购价压在每亩五千块。为了表示绝不接受的决心,今年下种的季节,西湾村人坚决地把庄稼照样种了下去。煤炭公司终于妥协了,每亩地一万五千块。这场惊心动魄的谈判像传奇一样在乱流河两岸到处被人传颂。一万五千块,简直就是一个让人头晕的天价。按照最好的年景,现在一亩地一年也就能收入一百多块钱。想一想就让人头晕,你得受一百多年的辛苦,流一百多年的汗,才能在一亩地里刨出来一万五千块钱呐!胜利的喜悦中,没有人再去百亩园了,因为合同一签,钱一拿,推土机马上就要开进来了。‎ 可是,不知不觉中,那些被人遗忘了的种子,还是和千百年来一样破土而出了。每天早上嫩绿的叶子上都会有珍珠一样的露水,在晨风中把阳光变幻得五彩缤纷。这些种子们不知道,永远不会再有人来伺候它们,收获它们了。从此往后,百亩园里将是炉火熊熊、浓烟滚滚的另一番景象。‎ 六安爷舍不得那些种子。他掐着指头计算着出苗的时间,到了该间苗锄头遍的日子,六安爷就拄着锄头来到百亩园。一天三晌,一晌不落。‎ 现在,劳累了一天的六安爷已经感觉到腰背的酸痛,满是老茧的手也有些僵硬。他蹲下身子摸索着探出一块空地,然后,坐在黄土上很享受地慢慢吸一支烟,等着僵硬了的筋骨舒缓下来。等到歇够了,就再拄着锄把站起来,青筋暴突的臂膀,把锄头一次又一次稳稳地探进摇摆的苗垅里去。没有人催,自己心里也不急,六安爷只想一个人慢慢地锄地,就好像一个人对着一壶老酒细斟慢饮。‎ 终于,西山的阴影落进了河谷,被太阳晒了一天的六安爷,立刻感觉到了肩背上升起的一丝凉意。他缓缓地直起腰来,把捏锄把的两只手一先一后举到嘴前,轻轻地啐上几点唾沫,而后,又深深地埋下腰,举起了锄头。随着臂膀有力的拉拽,锋利的锄刃闷在黄土里咯嘣咯嘣地割断了草根,间开了密集的幼苗,新鲜的黄土一股一股地翻起来。六安爷惬意地微笑着,虽然看不清,可是,耳朵里的声音,鼻子里的气味,河谷里渐起的凉意,都让他顺心,都让他舒服。银亮的锄板鱼儿戏水一般地,在禾苗的绿波中上下翻飞。于是,松软新鲜的黄土上留下两行长长的跨距整齐的脚印,脚印的两旁是株距均匀的玉茭和青豆的幼苗。六安爷种了一辈子庄稼,锄了一辈子地,眼下这一次有些不一般,六安爷心里知道,这是他这辈子最后一次锄地了,最后一次给百亩园的庄稼锄地了。‎ 沉静的暮色中,百亩园显得寂寥、空旷。六安爷喜欢这天地间昏暗的时辰,眼睛里边和眼睛外边的世界是一样的。他知道自己正慢慢融入眼前这黑暗的世界里。‎ 很多天以后,人们跟着推土机来到百亩园,无比惊讶地发现,六安爷锄过的苗垅里,茁壮的禾苗均匀整齐,一棵一棵蓬勃的庄稼全都充满了丰收的信心。没有人能相信那是一个半瞎子锄过的地。于是人们想起六安爷说了无数遍的话,六安爷总是平稳固执地说,“我不是锄地,我是过瘾”。‎ ‎(有删改)‎ ‎1.下列对小说相关内容和艺术特色的分析鉴赏,最恰当的两项是(  )‎ A.小说开头寥寥几句对话,六安爷这个勤劳而孤僻的老农形象已经跃然纸上,同时,他与村人的分歧也开始显露,并为下文情节发展埋下了伏笔。‎ B.西湾村人与煤炭公司“惊心动魄的谈判”,是小说中隐约可见的叙事背景,也是深刻的社会背景,巧妙地将六安爷的个人感受跟时代的变化联结起来。‎ C.小说中写到百亩园将要变成焦炭厂,往日的田园风光将会被“炉火熊熊、浓烟滚滚”的景象所取代,深化了作者关于生态问题的思考及小说的环保主题。‎ D.关于六安爷锄地的描写生动而富有诗意,传达了六安爷在百亩园劳作时惬意舒畅的感觉,这样的写法强化了小说所表达的人与土地分离的悲凉感。‎ E.综合全文来看,六安爷的“平静固执”,说明他作为一个老人,一方面已经饱经沧桑,看透世事变迁;另一方面也难免思想保守、无法与时俱进。‎ 答案 DB 解析 A项,说六安爷的性格“孤僻”,于文无据。C项,“深化了作者关于生态问题的思考及小说的环保主题”的后半部分错,“环保”并不是本文着力表现的主题。E项,“另一方面也难免思想保守、无法与时俱进”曲解文意,六安爷的“平静固执”某种层面上是对工业文明的一种排斥,但更多的是对土地的眷恋与坚守。‎ ‎2.小说以“锄”为标题,有什么寓意?请结合全文简要分析。‎ 答:___________________________________________________‎ 答案 ①锄作为一种农具,象征六安爷的人生和精神;②锄喻示劳动者与土地的亲密关系;③锄意味着传统的农业生产和生活方式;④锄作为一种劳作行为,蕴含着六安爷对土地的热爱,又暗含着他对土地的告别。‎ 解析 本题考查欣赏作品的形象,赏析作品的内涵,领悟作品的艺术魅力的能力。标题的寓意要从不同的角度分析:从内容上看,“锄”既是实物农具,象征着作为农民的六安爷的人生和其精神寄托;也是一种动作,是六安爷锄地的动作,饱含着六安爷对土地的深情,对土地被征用的不舍;从结构上看,“锄”联系着人物与土地,提示了两者的关系;从主题上看,意味着农业文明,传达出面对新时代发展的冲击,土地的使用该何去何从的思考。‎ ‎3.小说较为夸张地连续使用“几万”“几百万”之类的词语描述百亩园的历史,这样写的作用是什么?请简要分析。‎ 答:___________________________________________________‎ 答案 ①强调百亩园是西湾村人安身立命的物质基础;②将百亩园抽象为一种生活方式的象征;③与下文百亩园的一朝被毁构成鲜明尖锐的对比。‎ 解析 小说连续使用“几万”“几百万”之类的词语描述百亩园的历史,显然是为了突出百亩园的悠久历史及其对西湾村的贡献之大;与后文西湾村人为了眼前利益而放弃祖业形成对比,反差强烈;突出表现了小说的主题和作者的情感态度。‎ ‎4.“我不是锄地,我是过瘾”这句话,既是理解六安爷的关键,也是理解小说主旨的关键。请结合全文进行分析。‎ 答:___________________________________________________‎ 答案 六安爷层面:①六安爷用这句话来回应村人的劝阻,由此能感受到他温和而又固执的性格特征;‎ ‎②百亩园即将不复存在,六安爷的眼睛也快要失明,他要过在百亩园劳作的“瘾”,由此能体会到他内心的隐痛。‎ 小说主旨层面:①在大地上劳作是一种“瘾”,即劳动者的精神需要;②随着传统的农业生产、生活方式的结束,耕种的意义只剩下“过瘾”,令人叹惋又发人深思。‎ 解析 由“‘我不是锄地,我是过瘾’这句话,既是理解六安爷的关键,也是理解小说主旨的关键”可知,命题者的意图就是让考生分析这句话对人物形象的塑造与主题表现两方面的作用。在人物形象塑造上,通篇文章中,六安爷反复说这句话,表现了六安爷的性格特征和内心的隐痛。在主题表现上,这句话旨在引起人们对农民与土地分离这一社会现象的反思。‎ 二、[2016·全国卷Ⅱ]阅读下面的文字,完成1~4题。‎ 战争 ‎[美]迈尔尼 ‎1941年9月,我在伦敦被炸伤,住进了医院。我的军旅生涯就此黯然结束。我对自己很失望,对这场战争也很失望。‎ 一天深夜,我想给一位朋友打电话。接线生把我的电话接到了一位妇女的电话线上,她当时也正准备跟别人通话。‎ ‎“我是格罗斯文诺8829,”我听见她对接线生说,“我要的是汉姆普斯特的号码,你接错了,那个倒霉蛋并不想跟我通话。‎ ‎“哦,我想是。”我忙插嘴。‎ 她的声音很柔和,也很清晰,我立刻喜欢上了它。我们相互致歉后,挂上了话筒。可是两分钟后,我又拨通了她的号码,也许是命中注定我们要通话,我们在电话中交谈了20多分钟。‎ ‎“你干吗三更半夜找人说话呢?”她问。‎ 我跟她说了原因,然后反问“那么你呢?”‎ 她说她老母亲睡不好觉,她常常深夜打电话与她聊聊天。之后我们又谈了谈彼此正在读的几本书,还有这场战争。‎ 最后我说:“我有好多年没这样畅快地跟人说话了。”‎ ‎“是吗?好了,就到这里吧,晚安。祝你做个好梦。”她说。‎ 第二天整整一天,我老在想昨晚的对话情形,想她的机智、大方、热情和幽默感。当然还有那悦耳的口音,那么富有魅力,像乐曲一样老在我的脑海里回旋。到了晚上,我简直什么也看不进。午夜时,格罗斯文诺8829老在我脑海里闪现。我实在难以忍受,颤抖着拨了那个号码。电话线彼端的铃声刚响,就马上被人接起来。‎ ‎“哈罗?”‎ ‎“是我,”我说,“真对不起,打扰你了,我们继续谈昨晚的话题,行吗?”‎ 没说行还是不行,她立即谈起了巴尔扎克的小说《贝姨》。不到两分钟,我们就相互开起玩笑,好像是多年的至交。这次我们谈了45分钟。午夜时光和相互的不认识,打破了两人初交时的拘谨。我提议彼此介绍一下各自的身份,可是她婉言谢绝了。她说这会把事情全弄糟,不过她留下了我的电话号码。我一再许诺为她保留,直到战争结束。于是她说了一些她的情况,17岁时她嫁给一个自己不喜欢的男人,以后一直分居。她今年36岁,唯一的儿子在前不久的一次空袭中被炸死了,年仅18岁。他是她的一切。她常常跟他说话,好像他还活着。她形容他像朝霞一样美,就跟她自己一样。于是她给我留下了一幅美丽的肖像。我说她一定很美,她笑了,问道:“你怎么知道的?”‎ 我们越来越相互依赖,什么都谈。我们在大部分话题上看法相似,包括对战争的看法,我们开始读同样的书,以增加谈话的情趣。每天夜晚,不管多晚,我们都要通一次话。如果哪天我因事出城,没能通话,她就会埋怨说她那天晚上寂寞得辗转难眠。‎ 随着时间的推移,我愈来愈渴望见到她。我有时吓唬她说我要找辆出租车立刻奔到她跟前。可是她不允许,她说如果我们相见后发现彼此并不相爱,她会死掉的。整整12个月,我是在期待中度过的。我们的爱情虽然近在咫尺,却绕过了狂暴的感情波澜,正平稳地驶向永恒的彼岸。通话的魅力胜过了秋波和拥抱。‎ 一天晚上,我刚从乡间赶回伦敦,就连忙拿起话筒拨她的号码。一阵嘶哑的尖叫声代替了往日那清脆悦耳的银铃声,我顿时感到一阵晕眩。这意味着那条电话线出了故障或者被拆除了。第二天仍旧是嘶哑的尖叫。我找到接线生,请求他们帮我查查格罗斯文诺8829的地址,起先他们不理睬我,因为我说不出她的名字。后来一位富有同情心的接线小姐答应帮我查查。‎ ‎“当然可以。”她说,“你好像很焦急。是吗?嗯,这个号码所属的那片区域前天夜里挨了炸弹,号码主人叫……”‎ ‎“谢谢,”我说,“别说了,请你别说了。”‎ 我放下了话筒。‎ ‎(沈东子译,有删改)‎ ‎1.下列对小说相关内容和艺术特色的分析鉴赏,最恰当的两项是(  )‎ A.小说以“1941年9月,我在伦敦被炸伤”开头,不仅是为了交代故事发生的时间地点,更是为了强调这是作者的一段亲身经历。‎ B.“我有好多年没这样畅快地跟人说话了”,话中有话,既委婉地表达了“我”对女主人公的喜爱之情,又为两人进一步交往作了铺垫。‎ C.得知事情真相时“我”只说了句“别说了,请你别说了”,就放下了话筒,这看似不合常情的表现,背后传达的却是难以言说的悲哀。‎ D.接线生的失误让两人相识,心灵的需要让他们相恋,无情的轰炸让他们永别,小说情节既在意料之外,又在情理之中,设计自然而又精巧。‎ E.小说不仅描写了战时一对普通恋人的悲欢离合,也以真实的笔触,描绘了一幅世界反法西斯战争的历史画卷,表现了民众的必胜信念。‎ 答案 DC 解析 本题考查对小说相关内容和艺术特色的分析鉴赏能力,涉及情节结构、人物语言、主题等。A项,“更是为了强调这是作者的一段亲身经历”错误。“我在伦敦被炸伤”是为下文故事情节的发展做铺垫。B项,“又为两人进一步交往作了铺垫”不恰当,主要是“表达了‘我’对女主人公的喜爱之情”,引出下文的情节。E项,小说只是写了“我”的一段经历,“描绘了一幅世界反法西斯战争的历史画卷”夸大其词。“表现了民众的必胜信念”错误,应是表现对战争的谴责与反思。‎ ‎2.小说中的女主人公有哪些性格特点?请简要分析。‎ 答:___________________________________________________‎ 答案 ①大方热情、机智幽默,懂得及时化解生活矛盾;②乐观向上、热爱生活,战争和不幸都不能阻止她对美好生活和爱情的追求;③善良真诚、理性克制,有责任感,关心母亲,思念儿子,真诚待“我”。‎ 解析 本题考查分析小说人物形象特点的能力。分析人物形象时先用关键词点出人物形象的主要特点,然后结合文本分析。从“她的机智、大方、热情和幽默感。当然还有那悦耳的口音,那么富有魅力,像乐曲一样老在我的脑海里回旋”,能看出她大方热情、机智幽默。从在战争中仍然坚持读书,能看出她乐观向上,热爱生活。从“她说她老母亲睡不好觉,她常常深夜打电话与她聊聊天”和“唯一的儿子在前不久的一次空袭中被炸死了,年仅18岁。他是她的一切。她常常跟他说话,好像他还活着”,能看出她关心母亲,思念儿子。从她婉言谢绝“我”提议“介绍一下各自的身份”的要求,“她说这会把事情全弄糟,不过她留下了我的电话号码”,能看出她善良真诚、理性克制,有责任感,真诚待“我”。‎ ‎3.小说以“电话”为枢纽连接人物、安排情节,这样处理有什么作用?请简要分析。‎ 答:___________________________________________________‎ 答案 ①一个电话将两人命运连在一起,偶然与必然交错,凸显了战争背景,强化了戏剧性情节;②主人公言行主要通过电话聊天呈现出来,便于透露人物心声,使人物形象更真实;③电话交流的限制性给小说留下较多空白,丰富了人物与主题的想象空间。‎ 解析 本题考查分析小说线索的作用的能力。因为接线生接错电话,两人戏剧性地联系在一起,进而命运相连。两人从未见面,只能通过电话交流,因而电话聊天透露出人物的心声,表现人物形象。电话交流的限制性又丰富了读者对人物与主题的想象空间。‎ ‎4.小说写的只是战争中的一个小故事,却用了“战争”‎ 这样一个大题目,你认为这样处理合适吗?请结合全文,谈谈你的观点。‎ 答:___________________________________________________‎ 答案 观点一:合适。①小故事冠以大题目,对比鲜明,强化了艺术张力;②战争是故事发生的契机与悲剧的根源,是小说构思的基础;③小说写的虽是爱情故事,但主题却是对战争的“失望”与反思。‎ 观点二:不合适。①小故事冠以大题目,故作高深,不符合写作的一般原则;②小说的艺术感染力源自战争中的爱情,而不是战争;③小说情节设置以小人物的坚强与不幸为主干,战争只是引起情节变化的背景。‎ 解析 本题为探究性题目,考查对作品进行个性化阅读和有创意的解读的能力。回答时应在联系全文内容的基础上,抓住题干中的条件和要求来分析。小说所写的情节只是战争中的一个小故事,却用了“战争”这样一个大题目,这或是别有深意,或是考虑失当。可就其中一个角度,从标题的艺术效果、主题等方面结合文本分析。‎ 三、[2016·全国卷Ⅲ]阅读下面的文字,完成1~4题。‎ 玻璃 贾平凹 约好在德巴街路南第十个电杆下会面,去了却没看到他。我决意再等一阵,踅进一家小茶馆里一边吃茶一边盯着电杆。旁边新盖了一家酒店,玻璃装嵌,还未完工,正有人用白粉写“注意玻璃”的字样。‎ 吃过一壶茶后,我回到了家。妻子说王有福来电话了,反复解释他是病了,不能赴约,能否明日上午在德巴街后边的德比街再见,仍是路南第十个电杆下。第二天我赶到德比街,电杆下果然坐着一个老头,额头上包着一块纱布。我说你是王得贵的爹吗,他立即弯下腰,说:我叫王有福。‎ 我把得贵捎的钱交给他,让给娘好好治病。他看四周没人,就解开裤带将钱装进裤衩上的兜里,说:“我请你去喝烧酒!”‎ 我谢绝了。他转身往街的西头走去,又回过头来给我鞠了个躬。我问他家离这儿远吗,他说不远,就在德巴街紧南的胡同里。我说从这里过去不是更近吗,老头笑了一下,说:“我不走德巴街。”‎ 他不去德巴街,我却要去,昨日那家茶馆不错。走过那家酒店,玻璃墙上却贴出了一张布告——‎ 我被酒店此举感动,很快想到王有福是不是撞了玻璃受的伤呢,突然萌生了一个念头:既然肯赔偿,那就是他们理屈,何不去法院上告,趁机索赔更大一笔钱呢?我为我的聪明得意,第二天便给王有福打电话,约他下午到红星饭店边吃边谈。‎ 红星饭店也是玻璃装修。我选择这家饭店,是要证实他是不是真的在酒店撞伤的。他见了我,肿胀的脸上泛了笑容,步履却小心翼翼,到了门口还用手摸,证实是门口了,一倾一倾地摇晃着小脑袋走进来。‎ ‎“我没请你,你倒请我了!”他说。‎ ‎“一顿饭算什么!”我给他倒了一杯酒,他赶忙说:“我不敢喝的,我有伤。”‎ ‎“大伯,你是在德巴街酒店撞伤的吗?”‎ ‎“你……那酒店怎么啦?”‎ ‎“这么说,你真的在那儿撞的!”‎ ‎“这……”‎ 老头瓷在那里,似乎要抵赖,但脸色立即赤红,压低了声音说:“是在那儿撞的。”一下子人蔫了许多,可怜得像个做错事的孩子。‎ ‎“这就好。”我说。‎ ‎“我不是故意的。”老头急起来。“我那日感冒,头晕晕的,接到你的电话出来,经过那里,明明看着没有什么,走过去,咚,便撞上了。”‎ ‎“你撞伤了,怎么就走了?”‎ ‎“哗啦一声,我才知道是撞上玻璃了。三个姑娘出来扶我,血流了一脸,把她们倒吓坏了,要给我包扎伤口,我爬起来跑了。我赔不起那玻璃呀!”‎ ‎“他们到处找你哩。”‎ ‎“是吗?我已经几天没敢去德巴街了,他们是在街口认人吗?”‎ ‎“他们贴了布告……”‎ 老头哭丧下脸来,在腰里掏钱,问我一块玻璃多少钱。‎ 我嘿嘿笑起来。‎ ‎“不是你给他们赔,是他们要给你赔!”‎ ‎“赔我?”‎ ‎“是赔你。”我说,“但你不要接受他们的赔偿,他们能赔多少钱?上法院告他们,索赔的就不是几百元几千元了!”‎ 老头愣在那里,一条线的眼里极力努出那黑珠来盯我,说:“你大伯是有私心,害怕赔偿才溜掉的,可我也经了一辈子世事,再也不受骗了!”‎ ‎“没骗你,你去看布告嘛!”‎ ‎“你不骗我,那酒店也骗我哩,我一去那不是投案自首了吗?”‎ ‎“大伯,你听我说……”‎ 老头从怀里掏出一卷软沓沓的钱来,放在桌上:“你要肯认我是大伯,那我求你把这些钱交给人家。不够的话,让得贵补齐。我不是有意的,真是看着什么也没有的,谁知道就有玻璃。你能答应我,这事不要再给外人说,你答应吗?”‎ ‎“答应。”‎ 老头眼泪花花的,给我又鞠了下躬,扭身离开了饭桌。‎ 我怎么叫他,他也不回头。‎ 他走到玻璃墙边,看着玻璃上有个门,伸手摸了摸,没有玻璃,走了出去。‎ 我坐在那里喝完了一壶酒,一口菜也没吃,从饭馆出来往德巴街去。趁无人理会,我揭下了那张布告:布告继续贴着,只能使他活得不安生。顺街往东走,照相馆的橱窗下又是一堆碎玻璃,经理在大声骂:谁撞的,眼睛瞎了吗?!‎ 我走出了狭窄的德巴街。‎ ‎(有删改)‎ ‎1.下列对小说相关内容和艺术特色的分析鉴赏,最恰当的两项是(  )‎ A.“约好在德巴街路南第十个电杆下会面”,是对地下斗争题材影视作品的模仿,为后文悬念丛生的情节做出铺垫。‎ B.发现王有福正是受伤的路人后,“我”劝他到法院上告酒店,寻求更多赔偿,因为“我”不仅热心帮助朋友,也有打官司的经验。‎ C.王有福不情愿承认自己误撞酒店玻璃受伤,主要是因为妻子有病,家庭生活很困难,害怕酒店追究责任,让他赔偿损失。‎ D.“我”经过照相馆时,见经理面对碎玻璃大骂,这一细节暗示此地这类纠纷不少,王有福担心的“投案自首”之事是经常发生的。‎ E.玻璃墙伤人事件的背后,交织着伦理观念、法治观念、诚信意识等不同理念的矛盾、困惑与冲突,是转型期中国社会的一面镜子。‎ 答案 EC 解析 A项,“是对地下斗争题材影视作品的模仿”不合文意。B项,“因为‘我’不仅热心帮助朋友,也有打官司的经验”错误,应该是“我”认为酒店管理失误导致王有福受伤,要求赔偿正当合理。D项,“暗示此地这类纠纷不少”正确,但“王有福担心的‘投案自首’之事是经常发生的”于文无据。‎ ‎2.“我”在小说中的主要作用是什么?请简要分析。‎ 答:___________________________________________________‎ 答案 ①讲述故事:小说故事是由“我”叙述出来的,真实可信;②推进情节:“我”是事件的参与者,由于“我”的提议,情节得以发展变化;③衬托人物:小说主人公王有福的性格,由于“我”的存在而更加鲜明。(意思答对即可)‎ 解析 ‎ 本文采用第一人称的叙述方式,使小说显得真实亲切,拉近了与读者的距离,同时便于作者抒发感情。正是“我”约王有福见面,才推动了情节的发展。主人公王有福的人物形象正是通过“我”的所见所做所感才一点点地展现出来,“我”的存在也凸显了王有福的思想性格特点,使他的形象更加鲜明。‎ ‎3.小说中的王有福有哪些性格特点?请简要分析。‎ 答:___________________________________________________‎ 答案 ①性情谦卑,甚至有点窝囊:见了晚辈,也弯腰鞠躬,说话谦和;②胆小怕事,有些狡黠:撞了玻璃偷偷溜掉,别人问起也不敢承认;③有点固执,但不失本分善良:怀疑酒店诚意,承认自己责任,不愿借机发财。‎ 解析 王有福的性格特点可通过以下内容进行概括:①“他立即弯下腰”“又回过头来给我鞠了个躬”,体现出王有福“性情谦卑,甚至有点窝囊”,因为“我”是晚辈。②“你……那酒店怎么啦?”“这……”“我不是故意的”,面对“我”的追问,王有福躲躲闪闪,含糊其词,体现出他“胆小怕事,有点狡黠”。③从“你大伯是有私心,害怕赔偿才溜掉的,可我也经了一辈子世事,再也不受骗了”以及文本倒数第七段王有福让我转交赔玻璃的钱等话中可以看出他“有点固执,但不失本分善良”。‎ ‎4.是否状告酒店,“我”与王有福的态度不同。你更认同谁的态度?请结合全文,谈谈你的观点。‎ 答:___________________________________________________‎ 答案 观点一:认同王有福的态度。①王有福受伤与酒店管理有关,但他是有行为能力的成年人,应负一定责任;②王有福害怕赔偿溜走,逃避责任在先,索赔的理由不够正当充分;③王有福害怕被骗而拒绝索赔,在当时的情况下,未尝不是理性的选择。‎ 观点二:认同“我”的态度。①酒店失误导致王有福受伤,要求赔偿正当合理;②王有福放弃赔偿是担心被骗,说明他缺乏法律意识,更应进行法律启蒙;③王有福式的宽容是对不良行为的纵容,有害无益。‎ 解析 首先亮明自己的观点,答题时,既可以认同“我”的态度,也可以认同王有福的态度;然后根据小说的具体内容进行分析。(1)认同王有福的态度。理由也可从以下角度作答。①环境角度:酒店虽有错,但王有福也有责任。②人物角度:王有福害怕赔偿,偷偷溜走,逃避责任在先,索赔理由不正当。③主旨角度:王有福害怕被骗而拒绝索赔的心理,这也是一种社会现象。(2)认同“我”的态度。①环境角度:酒店有错,导致王有福受伤,状告合情合理。②人物角度:王有福胆小怕事,说明他缺乏法律意识,更应鼓励他状告酒店。③主旨角度:社会中有很多王有福这样的人物,他的宽容只会让社会纵容酒店的不良行为。‎ ‎[2年全国模拟重组]‎ 一、[2017·福建师大附中期中考试]阅读下面的文字,完成1~3题。‎ 红孩儿 陈国凡 兵荒马乱之时,山高林密之地,县市交界之处,常有盗匪出没。婺州的尖峰山就是这样一去处。不知打何时起,有盗匪啸聚尖峰山,打家劫舍。官府动了不少心思进行围剿,却终究奈何不得,盗匪便势力日壮,气焰愈狂。‎ 匪首人称红孩儿。因他极喜《西游记》里的红孩儿,人小鬼大,武艺高强,便以其名自称。有关他因何沦入匪道的,倒有一说。‎ 红孩儿的父亲本是婺城一张姓大员外,因老来得子,极宠红孩儿,视为宝贝疙瘩,含在嘴里怕化了,捧在手中怕掉了,对其百依百顺。一大家子人终日围着红孩儿转,欢笑不断,却不料大祸临头。‎ 红孩儿突然失踪,家人正哭爹喊娘四处寻找,却收到了尖峰山匪首陈麻子的绑票,限张员外三日之内凑齐三千两白银,否则撕票。真是祸从天降,张员外急火攻心,身体本有大恙,竟一命呜呼了。张家群龙无首,各路冤家、债主趁机上门惹事、讨债。原来张家空有架子,平素结怨甚多,外债甚大。张家顿时乱作一团,只好以家财抵债,却再无赎回红孩儿的余钱。‎ 真是背到家了,奶奶的!陈麻子气急败坏,本想杀了红孩儿泄愤,却见襁褓中的红孩儿肥嘟嘟的,极爱笑,很可爱,加之没有子嗣,陈麻子动了恻隐之心,想把红孩儿当亲身骨肉,好生抚养。不想红孩儿日夜啼哭,陈麻子只好把红孩儿的母亲接到山寨。当然,事先约法三章,断不能告知红孩儿身世,否则,母子性命不保。自此,红孩儿在尖峰山茁壮成长,且练就一身武艺。‎ 春去秋来,陈麻子病逝。临死前立下遗嘱,要军师全力辅佐红孩儿。‎ 红孩儿遂成了匪首,时年16岁。‎ 红孩儿虽年幼,却聪明过人,悟性极高,山寨很快走上正轨,漂亮地干了几票后,红孩儿一下子树立了威信,连老军师也对其服服帖帖,恭恭敬敬。两年后,老军师过世,红孩儿愈发成熟,完全能独当一面了。‎ 红孩儿干票有其特色。‎ 其一,不打家劫舍,就专干绑票。绑票不寻老人,不要姑娘,专绑孩子。专绑孩子,也不是随便一见孩子就绑票,他会细细寻觅,专对那些自幼被过分溺爱的纨绔子弟下手。这些富贵、官宦之家不差钱,舍得下血本赎回。‎ 其二,红孩儿干票从不害人性命。被绑上山的一干人等,若遇到家变等缘由,不能被赎回,留或不留,悉听尊便。留,欢迎;不留,则即刻送其下山,还给予盘缠和生活开销。‎ 其三,劫富济贫,对百姓秋毫无犯。红孩儿还说服家境特别困难的百姓子弟上山抚养,传授武艺、识文断字,一样不少。几年下来,贫穷子弟能文能武,还不用受山下官商的欺凌,故极少有想下山的。‎ 百姓皆称善。富人、官府却恨之入骨,曾多次联合进剿,均大败而还。尖峰山地势险要、易守难攻,防御工事环环相扣,戒备到位,处处设防。若非智取,实难攻陷。‎ 母亲归天,红孩儿是个大孝子,决定接下来的几天,山寨要隆重举办后事。官商清楚,这是剿灭红孩儿的绝好时机。‎ 果然,夜深的山寨疏于防范,被官兵攻破,红孩儿不幸被生擒。‎ 毕竟还是孩子啊,不够老练。婺州知府很是得意。‎ 大堂问审,红孩儿不卑不亢,侃侃而谈。‎ ‎“我只是将世上的不义之财,还之于民。我何时杀人越货,危害一方?我何罪之有?”‎ 官老爷们被问得哑口无言,虚汗淋漓,只好暂停问审,将红孩儿收押牢房。‎ 不日,皇帝大赦天下。婺城知府顺应民意,释放了红孩儿,但不许再回尖峰山。知府承诺不滥杀无辜,澄清吏治,改善民生,努力开拓婺州新局面。‎ 那几年,婺州官不逼,民不反,太平无事。‎ 故事还没完呢。‎ 婺州知府将升迁,临行前再三嘱咐红孩儿:“今后若实在难以活命,就再上尖峰山吧。”‎ ‎“为何?”红孩儿大为不解。‎ 知府没答,摇摇头叹口气,一拂袖,走了。‎ ‎(选自2015年《小说月刊》第10期,有删改)‎ ‎1.下列对这篇小说思想内容与艺术特色的分析和鉴赏,最恰当的一项是(  )‎ A.开篇简要介绍当时的社会现状和尖峰山的特殊位置,意在表明盗匪产生的必然性,为红孩儿的出现张本,暗示红孩儿得天时、地利、人和。‎ B.小说以概述为主,涵盖的事件多,又受制于篇幅的局限,于是情节运行加快,其中细节描写的缺失,影响了小说的艺术表现力。‎ C.红孩儿被收监,只有死路一条,但突遇皇帝大赦,知府也顺应民意,红孩儿获得释放,这时小说情节发生逆转,出乎意料。‎ D.发生在红孩儿与知府之间的恩怨情仇令读者唏嘘不已,其中暗含了在当时大人物生杀予夺而小人物难以掌握自己命运的社会现实。‎ 答案 C 解析 A项,“红孩儿得天时、地利、人和”分析有误,属无中生有。B项,“细节描写的缺失……”分析有误,小说中有语言、心理描写,还有结尾处的动作、神态描写,都是细节描写。D项,“唏嘘不已”分析有误,小说中的知府在攻破山寨,擒获红孩儿后,并未使用酷刑;后因皇帝大赦,知府又顺应民意,释放了红孩儿;在知府将升迁时,他还不忘提醒红孩儿,这些并非让人感到遗憾或悲哀。‎ ‎2.小说在刻画红孩儿这个形象时,突出了他的哪些性格特征?请简要分析。‎ 答:___________________________________________________‎ 答案 ①聪颖过人。漂亮地干了几票后,树立了威信,连老军师也服服帖帖,恭恭敬敬。②坚持原则。干票时严守规矩,专干绑票,不害性命,劫富济贫,深得民心。③‎ 不够老练。山寨隆重举办其母丧事时,疏于防范,被官兵攻陷,不幸被生擒。④不卑不亢。大堂问审时,理直气壮,正气凛然,让官老爷们哑口无言。‎ 解析 其实本题是变相的考查人物形象。解答此题,阅读全文,逐段逐句地筛选文章信息,并加以概括。人物性格分析注意从小说的情节入手,通过对人物的语言、动作、心理等描写或其他的侧面描写进行分析总结。“漂亮地干了几票后,树立了威信,连老军师也服服帖帖,恭恭敬敬”“干票时严守规矩,专干绑票,不害性命,劫富济贫,深得民心”“山寨隆重举办其母丧事时,疏于防范,被官兵攻陷,不幸被生擒”,然后分条概括。‎ ‎3.小说原稿的结尾是这样的——“新官来了,会推倒一切,啥事都能发生啊。”知府痛心疾首道。原稿和修改稿的两个结尾,你更欣赏哪一个?请结合全文,谈谈你的看法。‎ 答:___________________________________________________‎ 答案 更欣赏本文结尾。①审美意蕴上,充满了含蓄美。知府没答,显得高深莫测,而他的动作、神态给人留下了审美的空间和回味的余地。②人物形象上,使人物形象更加丰满。知府费尽心机收擒了红孩儿,在皇帝大赦时释放了他,知府将升迁时又再三叮嘱,而叮嘱的原因却秘而不宣,这显示出知府老练的一面;同时与红孩儿的不够老练形成鲜明的对比。③思想内容上,升华了小说主题。小说通过叙写红孩儿被擒被放的经历,揭示了“官逼民反、官不逼民不反”的主题,但知府知而不言,为读者留下了想象的空间,使主题更加深刻。‎ 解析 这是一道探究的题目,既涉及文章的内容,又要拓展,首先立足文本分析结尾。明确观点,主要围绕人物塑造、情节发展、主旨表达等方面分析。本题主要就小说的结尾设题,明确观点,结合小说内容分析即可。向外延伸主要是针对文本提出自己的看法或对自己的启示,答题时首先明确自己的观点,然后结合生活中的事例进行论证,论证要围绕自己的观点进行,做到中心突出、简单明了。‎ 二、[2017·唐山、保定联合调研]阅读下面的文字,完成1~3题。‎ 腊梅 吴连广 腊梅嫁给顺子时,村里的年轻小伙子都当八路走了。腊梅就问顺子:“顺子,人家都当八路打小鬼子去,你咋没去?”顺子低着头,小声地说:“俺爹俺娘的身子骨一天不如一天,不能下地干庄稼活儿,俺去当八路打小鬼子,爹娘咋办?”‎ 顺子说的是实情,爹娘虽然年岁不大,只有五十多岁,可都一身的病,别说下地干庄稼活儿,就连走路都困难。八路军和地方政府也看到了这些,也就没有动员顺子参军。腊梅听顺子的话是真话,也就没多说什么。‎ 第二年一开春,腊梅有了身孕,挺着肚子操持着家务,伺候着病病歪歪的公婆。忽然有一天,顺子对腊梅说:“明天,俺要到二姑家去一趟有点事,过几天就回来。”‎ 顺子走后,腊梅就听说,辽南支队来村里招收新兵。腊梅越想越觉得顺子走得蹊跷,早不走晚不走,偏这个时候到三十里外的二姑家去。第二天,腊梅做好够公婆一天吃的饭菜就走了。她要去找回顺子,让他参加辽南支队打小鬼子。‎ 腊梅一见到在二姑家的顺子,一句话也不说,拉着顺子就走,顺子不敢不跟腊梅走。走了一段路程,腊梅哭了,她望着低着头的顺子说:“打小鬼子,不是哪一个人的事,你不去,他也不去,小鬼子什么时候才能打走!”顺子一下子抱住腊梅激动地说:“腊梅,俺也想参加八路军打小鬼子,可俺舍不得你和肚子里的孩子,还有俺爹俺娘。”两行清澈的泪水从顺子的脸上滚落下来,顺子理了一下腊梅被山风吹乱的头发,接着说:“上了战场,就不能当孬种,可子弹不长眼睛,死了就死了,咱们的孩子就没爹了。”‎ 顺子穿上灰色的军装走了,再也没有回头。腊梅站在村口的老槐树下,一直到看不到走远的部队。初秋腊梅生了一个大胖儿子,她也听说辽南支队开赴前线作战的消息,后来这支黑土地上壮大的部队,转战东北抗日战场上,奋勇杀敌,让日本关东军非常头痛,无数关东军的尸体倒在黑土地上。‎ 腊梅像盼星星盼月亮似的,盼望着把小日本早一天打回老家去,顺子就会回来。可小日本还没打走,东北战场上的战事还很激烈,就传回顺子回来了的消息。当腊梅飞似的跑到村口的大槐树下,望着一群人抬着担架走来,腊梅心里“咯噔”了一下,她不知道顺子是啥模样,她预感到了那不堪入目的一瞬。‎ 人群渐渐地近了,腊梅再次飞奔过去。担架上抬着的是顺子,顺子双腿被炮弹炸飞了。走的时候,是一个双腿如飞的男人,回来,却把一双腿留在战场上了。腊梅哭了,泪水像雨点似的,砸在地上了。她擦了擦眼泪笑了,无论怎样总算回来了。‎ 自从顺子回来以后,脾气非常暴躁,总是和腊梅发脾气,有时还骂腊梅,拿起东西就砸腊梅。腊梅并不在意顺子对她的不公,依旧细心照料着顺子。那天,顺子又发脾气了,拿起喝水的碗砸在腊梅的头上,一股鲜血顺着脸流了下来,腊梅望着顺子苦苦笑了笑说:“顺子,俺知道你想赶俺走,不想拖累俺,俺知道,俺不走,俺是你的女人,不管你是什么样子,这一辈子俺都是你的女人!”‎ 顺子哭了……‎ ‎(选自《小小说选刊》2015年第3期)‎ ‎1.下列对小说相关内容和艺术特色的分析鉴赏,最恰当的一项是(  )‎ A.小说通过抗日战争时期一对青年男女腊梅和顺子的故事,歌颂了两个青年人在民族解放事业上的牺牲精神。‎ B.男主人公顺子因家庭困难逃避参军,几经波折后,才在妻子腊梅的帮助下改变了落后思想,在最后毅然从军。‎ C.村里招收新兵,顺子为了躲避从军,谎称有事躲到了二姑家里,直到腊梅追到二姑家,顺子才极不情愿地回来。‎ D.小说语言质朴,流畅简洁,情节生动,跌宕起伏,往娓娓的叙述中展现了小说的主旨,结尾更是动人心弦。‎ 答案 A 解析 B项,“落后思想”错,文中顺子说“俺也想参加八路军打小鬼子”,可见他也有积极抗战的思想;C项,“顺子才极不情愿地回来”不准确,文中没有明确写出顺子回来时的态度,从后文他毅然参军来看,此处回来不能是“极不情愿的”;D项,“跌宕起伏”不准确,小说节奏较为舒缓。‎ ‎2.小说开头“腊梅嫁给顺子时,村里的年轻小伙子都当八路走了”的叙述,有什么作用?请简要分析。‎ 答:___________________________________________________‎ 答案 ①交代故事发生的背景是在抗日战争时期。②交代小说的主要矛盾,暗示腊梅和顺子在是否参军上存在分歧。③为下文腊梅劝说顺子参军的情节作铺垫。‎ 解析 本题考查分析作品结构、概括作品主题的能力。小说开头一般的作用为开宗明义、点题、引出话题、总领全文、交代故事背景、烘托氛围、为下文作铺垫、交代时间、地点、人物、事件等。结合本题,开头第一句话交代了故事发生的背景是抗日战争时期;与下一句腊梅和顺子的对话紧密相连,交代了小说的主要矛盾,暗示了二人存在分歧,为下文腊梅劝说顺子参军的情节作铺垫。‎ ‎3.腊梅有哪些性格特征?请简要分析。‎ 答:___________________________________________________‎ 答案 ①思想进步。看到别的年轻人都去打鬼子,想方设法让顺子去从军。②吃苦耐劳。有了身孕还操劳家务,伺候公婆。③忠于爱情。顺子负伤回家后,不离不弃。‎ 解析 本题考查鉴赏作品形象、领悟作品艺术魅力的能力。解答本题可以从腊梅的语言、动作、行为入手。“腊梅就问顺子:‘顺子,人家都当八路打小鬼子去,你咋没去?’”“她望着低着头的顺子说:‘打小鬼子,不是哪一个人的事,你不去,他也不去,小鬼子什么时候才能打走!’”说明她思想进步,积极动员丈夫参军;“第二年一开春,腊梅有了身孕,挺着肚子操持着家务,伺候着病病歪歪的公婆”说明她吃苦耐劳;“腊梅望着顺子苦苦笑了笑说:‘顺子,俺知道你想赶俺走,不想拖累俺,俺知道,俺不走,俺是你的女人,不管你是什么样子,这一辈子俺都是你的女人!’”说明她忠于爱情。‎ 三、[2017·信阳调研测试]阅读下面的文字,完成1~3题。‎ 最后一次蹲守 朱卫国 公墓,秋叶纷纷。‎ 老梁身穿保洁工作服,挨着一辆垃圾车,他一边观察着不多的扫墓人,一边清理小径的落叶。时间过去一年了,他又在这里等她,如果这次等不到,这将是老梁职业生涯的一个遗憾,再过几天,他就正式退休了。‎ 今天是她儿子十周岁的生日,这么重要的日子,她肯定会来,一个母亲唯一的儿子长眠在此,那么孤单,她一定熬不了的,明知道来看儿子会有危险,但还是要来。‎ 老梁赌她这几天一定会来,并且预测她更加憔悴与绝望了。一年前,就在此地,他和她近在咫尺,尽管她如此乔装打扮,如此谨慎小心,他还是认出了她。他应该扔掉手中的扫帚,一跃而上,结束多日的蹲守,但他没有,他犯了一个职业大错,让一个女逃犯从眼前平静地离开了。‎ 这是老梁单独执行的任务,尽管没有受到处分,但蹲守必须继续。‎ 一年了,她凌乱不堪的眼神让老梁无法平静。那是怎样的一副面容啊,一个三十四岁的女人,却如一片枯黄干涩的落叶,凄惶憔悴,不可终日,痛失爱子的折磨已经深肌入骨。是的,她真是一个可怜的女子,老梁看见她眼中浑浊的泪水,像一根雨线倾注而下,被极力克制的身子仍剧烈抖动着,那是一位悲伤无助的母亲在逝儿墓前的哭泣。‎ 此前,他曾无数次熟悉过她的照片与资料,他不敢相信之前风华动人的她竟凋零成如此模样。她跪在儿子的墓前,轻轻抚摸着孩子的相片,完全忘却了身边可能存在的危险,此刻,她已丢掉警惕与防范,回到了纯粹的母亲身份,她凝视着儿子定格的面容,很久很久。渐渐地,她脸上露出了轻微的笑意,或许,她想到了:人间与天堂的距离不过是一块墓碑。‎ 老梁没有冲上去,设计好的种种抓捕细节已经忘得无影无踪,他甚至不忍心靠近去打扰她,不知怎的,心底有一种力量拉着他继续隐藏,他看着她哭完后摆出了祭品,有巧克力、玩具,然后,她平静地离开了墓园。‎ 在很长一段时间里,老梁并不后悔放走了她。‎ 一阵秋风把老梁从思绪中拉回,他紧了紧上衣。瑟瑟寒意让墓园更显寂寥,远处的暮霭中开始闪现零星的灯火。公墓里人影寥寥,夜晚的帷幕即将拉起。‎ 他赌这几天她一定会来的。‎ 他再一次在脑海中回溯起她的经历。原本她有一个可爱的儿子,尽管被丈夫抛弃了,母子俩还可以相依为命的。她发誓要好好带大儿子,她的确做到了,成了一位不折不扣的女强人,赚了很多钱,如果儿子没有突发那场疾病,一切都是美好的。医生的那一句“我们已经尽力了”,带走了她所有的希望。‎ 绝望的她执拗地认为医生治死了她的儿子,她要报复,要把医院的大门炸掉,要给医院好看。她私买了一些炸药,藏在特意在郊外空旷地方租来的屋子里,哪知骤起的高温让炸药自爆,炸伤了偶尔从这里经过的路人。看到有关爆炸的新闻后,她开始了逃匿生涯。‎ 老梁承认自己容易感情用事,这是职业大忌,他反省过。他一次次地想象着她的逃匿生活,那是怎样的一种折磨啊,永远只能是地下人,无法见到阳光。慢慢地,老梁开始后悔自己的所为,他觉得自己就是罪人,是他让她在继续着不幸与痛苦。是的,抓住她,让她有一个结束,也有一个新的开始,才是对她最好的怜悯。‎ 她仿佛彻底消失了,很长一段时间,都发现不了她的踪影。但老梁坚信,她一定还会在她儿子生日前后出现在儿子的墓地,毕竟,她是一个孩子的母亲。‎ 秋雨似有似无,老梁在落叶间凝神注视。她一定会来的,他坚信。他再次预想种种抓捕细节,甚至安排好了在外围配合的同事。他想,这将是他最后一次蹲守。‎ 他的预测没有错。下午五点五十,一个人影闪了过来,往十九号墓地走去,是她,就是她,老梁一眼认出来了,并给外围的同事发出了信号。他假装推动垃圾车,继续观察她的动静。他心想,该结束了。‎ 她从包里掏出一大堆零食,一样样放在儿子墓前,轻轻抚摸着儿子的相片,温情地说着:“儿子,妈妈来看你了。”‎ 老梁拿着扫帚边扫边靠近她,以便距离合适后成功抓捕,只有四米了,三米,两米,必须要行动了,他向同事发出了行动信号。‎ 突然,她转过身,对他平静地说:“我知道你们是警察,让我再看一会儿孩子吧,我进去后,孩子就看不到妈妈了。”她化了精致的妆容,仍掩不住满脸的风霜。‎ ‎“好吧。”他说。‎ 半个小时后,她缓缓起身,最后回头看了一下儿子,对老梁说:“是该结束了。”‎ ‎“嗯,是该重新开始了。”他给她戴上了手铐。‎ ‎(选自《小小说月刊》,2016年7期,略有删改)‎ ‎[注] 《中华人民共和国刑法》规定:非法制造、买卖、运输、邮寄、储存枪支、弹药、爆炸物的,处三年以上十年以下有期徒刑;情节严重的,处十年以上有期徒刑、无期徒刑或者死刑。‎ ‎1.下列对这篇小说有关内容与艺术特色的分析和鉴赏,最恰当的一项是(  )‎ A.文章第二段交代了老梁以保洁身份蹲守的内容,渲染了紧张气氛:“他又在这里等她”设置悬疑,“正式退休”交代了老梁如果这次不能抓捕“她”会造成遗憾的原因。‎ B.“是该结束了”一句说明“她”对儿子的过世和自己的逃亡生活有了清醒、正确的看法,“她”再也不愿过逃亡的生活,“她”要接受法律的惩罚,为自己的错误做法赎罪。‎ C.“她”对儿子的爱是整个悲剧发生的根本原因,因为爱儿子“她”把儿子的过世迁怒于医院,后来购买炸药,最终因炸药自爆炸伤路人,最终使“她”走上逃亡的道路。‎ D.“她”平静地对老梁等人说“我知道你们是警察”,说明“她”是一个聪明的女子,之前就认出去年要抓捕“她”的老梁,但爱儿子心切的“她”还是冒险去祭奠儿子。‎ 答案 B 解析 A项,“渲染了紧张气氛”错误,这段话主要交代故事发生的原因,有一点悬疑,但不是紧张氛围。C项,“‘她’对儿子的爱是整个悲剧发生的根本原因”错误,悲剧的原因应该是“她”偏执的性格使然,不是因为“她”爱孩子这件事。D项,“认出去年抓捕自己的老梁”错误,文中没有说“她”认出老梁是去年抓捕他的人,前文有老梁第一次抓捕“她”时没有冲上去的内容。‎ ‎2.小说中老梁的回忆交代了哪些重要内容?请结合小说简要分析。‎ 答:___________________________________________________‎ 答案 ①交代了“她”的身世和犯法的原因。“她”被丈夫遗弃,与儿子相依为命,但儿子因为生病过世,“她”迁怒医院,要炸毁医院大门,结果“她”私自购买的炸药自爆,炸伤了行人。②交代了老梁没有抓“她”的具体原因。蹲守墓地的老梁看到“她”的憔悴和悲伤,不忍心打扰“她”,最后没有抓住“她”。③交代了老梁猜测“她”一定会到墓地的原因。“她”十分爱自己的儿子,不忍心在儿子生日时还让他“孤单”,所以“她”一定会冒着危险在儿子生日的时候到墓地去祭奠儿子。‎ 解析 本题考查概括故事情节,分析作品结构,把握文章思路的能力。老梁的回忆里,有“她”的不幸遭遇,有“她”的偏激行为,有老梁一直没有抓捕“她”的原因,结合文本进行分析逐条总结即可。‎ ‎3.小说最后一段中,老梁对“她”说“是该重新开始了”这句话既是理解人物老梁的关键,也是理解小说主旨的关键。请结合全文进行分析。‎ 答:___________________________________________________‎ 答案 人物层面:①一年前,蹲守抓捕犯人的老梁看到“她”憔悴的面容、痛失爱子的悲伤,没有打扰给儿子祭奠的“她”,结果让“她”平静地离开墓园,体现了老梁的怜悯之心。②一年后,老梁蹲守公墓,发誓要把“她”逮捕归案,给“她”一个重新做人的机会,表现了老梁对自己错误的清醒认识和对法律的正确理解。‎ 主旨层面:①“法网恢恢,疏而不漏”,任何人不论在任何前提下做坏事都要受到法律的惩罚。②法律无情人有情。③违反法律后的隐藏逃匿不是解决问题的办法,犯人只有受到法律惩罚后才有重新开始的机会。‎ 解析 本题考查体会重要语句的丰富含意,品味精彩的语言表达艺术的能力。“是该重新开始了”包含了两方面的意思。老梁既是对“她”说的,也是对自己说的。“她”重新开始,是因为“她”触犯了法律,必须受到法律的制裁,重新做人;老梁自己重新开始,是因为老梁曾经因为怜悯“她”的身世遭遇而没有抓捕“她”,这是对法律的违背,现在老梁对自己的错误有了正确的认识。题目要求从这句话中理解老梁形象,则可以看出老梁的怜悯之心和公正之心。从小说主旨上分析,老梁从人情到法律的观念的转变,正体现了法律的公正。只要触犯了法律,就应该受到制裁,如果人情代替了法律,就违背了法律的威严,就会导致社会的混乱。而且如“她”一样的逃亡,虽然身体是自由的,但心灵却永远不会自由,法律会提供让“她”重新做人的机会,也是救赎“她”的唯一办法。‎ 四、[2017·保定联考]阅读下面的文字,完成1~3题。‎ 保护人 ‎[法]莫泊桑 玛兰做梦也没想到会有这么好的官运!‎ 有天早上,他从报上看到从前一位同学新近当了议员。玛兰重新成了他那位同学呼之即来、挥之即去的朋友。‎ 不久议员摇身一变当了部长,半年后玛兰就被任命为行政法院参事。‎ 起初,他简直有点飘飘然了。为了炫耀,他在大街上走来走去,仿佛别人只要一看见他,就能猜到他的身份。后来,出于一种有权势而又宽宏大量者的责任感,他油然萌生一股压制不住要去保护别人的欲望。无论在哪里遇到熟人,他都高兴地迎上去,不等人家问,就连忙说:“您知道,我现在当参事了,很想为您出点力。如有用得着我的地方,请您甭客气,尽管吩咐好了。我在这个位置上,是有权力的。”‎ 一有机会,他对任何人都主动给予无限慷慨的帮助。他每天都要给人写十封、二十封、五十封介绍信,他写给所有的官吏。他感到幸福,无比幸福。‎ 一天早上,他准备去行政法院,屋外已经下雨了。‎ 雨越下越大。他只好在一个房门口躲雨。那儿已有个老神父。在当参事前,他并不喜欢神父。自一位红衣主教在一件棘手事情上客气地向他求教以后,他对他们也尊敬起来。他看看神父,关切地问:“请问您到哪一区去?”‎ 神父有点犹豫,过了一会儿才说:“我朝王宫方向去。”‎ ‎“如果您愿意,神父,我可以和您合用我这把伞。我到行政法院去。我是那里的参事。”‎ 神父抬起头,望望他:“多谢,我接受您这番好意。”‎ 玛兰接着说:“您来巴黎多半是为散心吧。”‎ 神父回答:“不,我有事。”‎ ‎“哦!是件重要的事吗?如果您用得着我,尽管吩咐好了。”‎ 神父好像挺为难。吞吞吐吐地说:“啊!是一件无关紧要的私事……一点小误会。您不会感兴趣的。是……是一件内部的……教会方面的事。”‎ ‎“哎呀,这正属行政法院管。您尽管吩咐我好了。”‎ ‎“先生,我也正要到行政法院去。您心肠真是太好了。我要去见勒尔佩、萨翁两位先生。说不定还得见珀蒂帕先生。”‎ ‎“哎呀,他们都是我最要好的朋友,刮刮叫的同事。我都恳切地去替您托托关系。包在我身上好了。”‎ 神父嘟囔着说了许多感恩的话。‎ 玛兰高兴极了。“哼!您可碰到了一个千载难逢的机会,神父。瞧吧,瞧吧,有了我,您的事情解决起来一定非常顺利。”‎ 他们到了行政法院。玛兰把神父领进办公室,请他坐在火炉前,然后伏案写道:“亲爱的同事:请允许我恳切地向您介绍德高望重的桑蒂尔神父,他有一件小事当面向您陈述,务请鼎力协助。”‎ 他写了三封信,那受他保护的人接了信,千恩万谢地走了。‎ 这一天平静地过去了。玛兰夜里睡得很好,第二天愉快地醒来,吩咐仆人送来报纸。他打开报纸念道:‎ ‎“有个桑蒂尔神父,被控告做过许多卑鄙龌龊的事……谁知他找到一位叫玛兰的行政法院参事做他的热心的辩护人,该参事居然大胆地替这个披着宗教外衣的罪犯,给自己的同事们写了最恳切的介绍信……我们提请部长注意该参事令人不能容忍的行为……”‎ 他一下就蹦起来去找珀蒂帕。‎ 珀蒂帕对他说:“唉!您简直疯了,居然把那老阴谋家介绍给我。”‎ 他张皇失措地说:“别提了……您瞧……我上当了……他这人看上去那么老实……他耍了我……卑鄙可耻地耍了我。我求您,求您设法狠狠地惩办他一下,越狠越好。我要写信。请您告诉我,要惩办他,得给谁写信?……对,找总主教!”‎ 他突然坐下来,伏在珀蒂帕的桌上写道:“总主教大人:我荣幸地向阁下报告,最近有一个桑蒂尔神父欺我为人忠厚,用尽种种诡计和谎言陷害我。受他花言巧语哄骗,我竟至于……”‎ 他把信封好,扭转头对同事说:“您看见了吧,亲爱的朋友,这对您也是个教训,千万别再替人写介绍信了。”‎ ‎(据郝运译文删改)‎ ‎1.下列对这篇小说有关内容与艺术特色的分析和鉴赏,最恰当的一项是(  )‎ A.由于同学的帮助,玛兰才当上了行政法院参事。因此他无论在哪里遇到熟人,都主动向对方提供帮助,这是他回报的方式。‎ B.在当参事前,玛兰并不喜欢神父,但是在一位红衣主教向他请教以后,“他对他们也尊敬起来”。这样描写达到了照应上文的目的。‎ C.玛兰被珀蒂帕训斥后,急于为自己辩解,并马上归罪于桑蒂尔神父。这足以看出他似乎很想保护别人,但实际上更关心自己的利益。‎ D.桑蒂尔神父起初并不想用“一件无关紧要的小事”麻烦玛兰,因此他回应玛兰的请求时吞吞吐吐,这种神情表现了他内心的犹豫。‎ 答案 C 解析 A项,不是“回报的方式”,而是炫耀的方式。B项,不是“照应上文”,而是引起下文。D项,除了“犹豫”外,应该还有不安和恐惧,或说受宠若惊。‎ ‎2.小说中的玛兰是一个什么样的形象?请简要分析。‎ 答:___________________________________________________‎ 答案 ①自私,趋炎附势,见风使舵;②伪善,爱慕虚荣,自高自大;③天真,热心,却没有原则。‎ 解析 本题考查欣赏作品的形象。解答本题要从全文出发,把握住人物行为、语言、心理和细节描写,从而全面归纳概括出人物性格的特点。本题误区在于答案要点不全,表述语言不规范。做好此题,一定要抓住人物的语言动作。从玛兰成为“行政院参事”之后的一系列动作和语言,可看出他爱慕虚荣、自高自大、伪善的特点;而帮助老神父这件事可看出他热心、天真、没有原则;面对报纸的报道和珀蒂帕的批评所表现的动作和语言,可看出他自私、趋炎附势、见风使舵的特点。‎ ‎3.小说后半部分引用了报纸上的一段报道,作者这样写对情节安排有哪些作用?‎ 答:___________________________________________________‎ 答案 ①补充故事,集中揭示人物之间的矛盾关系,使情节的内在逻辑更加合理;②推动情节发展,为下文玛兰的言行提供依据,使小说进入高潮;③给读者留下更多的想象空间,强化情节平中见奇的效果。‎ 解析 本题考查分析作品结构和主要表现手法的能力。解答本题需从小说整体入手,重点把握这一情节设置对故事发展的推动作用,达到出人意料的效果,揭示了小说人物形象的本质,同时也达到了对主人公玛兰讽刺的效果。对情节的作用理解把握往往不能从整体入手,这样就无法理解和概括出情节设置对故事发展和人物刻画的作用。‎ 五、[2017·鹰潭模拟]阅读下面的文字,完成1~3题。‎ 父亲没有赴约 ‎[美]罗伯特·诺格斯 这个故事发生在风景如画的国家丹麦的一个小客栈里。这种客栈通常供应游客食物和饮料。这儿的人都讲英语。我和父亲因为生意上的事,也因为旅游来到了这样的客栈,过着愉快的时光。‎ ‎“我希望母亲和我们一起在这儿。该多好啊!”我说。‎ ‎“如果你母亲来这里,带着她去附近旅游一定非常惬意!”父亲说。‎ 年轻时他曾经在丹麦旅游参观。我问:“您自那次旅游后离开此地到现在有多长时间了?”‎ ‎“哦!大约三十年。我依稀记得路途上曾经到过这个客栈。”他朝周围看了看,沉浸在回忆的气氛中。‎ ‎“那是多么美好的日子……”突然他沉默了,我看见他的脸变得异常苍白。随着他的视线望去我发现一个太太手里拿着一托盘饮料站在一群顾客面前。看得出她从前也许很漂亮。但现在发胖了,头发显得有些零乱。我问父亲:“您认识她么?”‎ ‎“从前认识。”他说。‎ 这位太太来到我们桌前,问:“要饮料吗?”‎ ‎“她变得太多太多了。感谢上帝她没有认出我。”父亲轻声低语,手里拿着手帕做了个鬼脸。“在遇到你母亲前我曾经认识她。”他继续说,“那时我还是个学生,到这里来旅游。她当时是个年轻可爱的少女,温文尔雅、妩媚动人。我们疯狂地相爱了。”‎ ‎“母亲知道此事吗?”我突然忿忿不平地说。‎ ‎“当然知道。”父亲焦虑地看着我,轻声说。我能感觉到他此时的窘迫。‎ 我说:“爸,您大可不必……”‎ ‎“假如你母亲在这儿,她将告诉你这一切。我不想让你为此操心。那时我对她和她的家庭来说完全是个外国人。当时我的生活完全依赖你爷爷。如果她跟我结婚,她不会有任何前途。所以她的父亲竭力反对我们。当我写信告诉父亲我想跟她结婚时,你爷爷便拒绝提供哪怕是一分钱的援助。于是我不得不返回故乡。但是临走前我们见过一次面,我告诉她我必须回美国去借些钱,几个月后回来便跟她结婚。”‎ ‎“我们知道,”他继续说,“她的父亲可能会拦截我们的来往信件。所以我们决定我将简单地给她寄一个纸条,告诉她我们见面的时间和地点,在那里我们将举行婚礼。然后我就回美国贷了款并写信告诉她见面的事。她收到信后复函说,‘届时我将如期而至。’可是她没有去。后来我了解到她在约定日期两周前和当地的一个客栈老板结婚了。她没有等到我们预定的时刻。”‎ 接着,父亲说:“感谢上帝她没有赴约。回家后我遇到了你母亲,我们过得非常幸福。我们常为这个年轻时的骑士故事说笑寻开心。我提议将来你把此事写成文字。”‎ 那位太太拿着啤酒出现在我们面前。‎ ‎“你是从美国来的吗?”她问我。“是的。”我说。‎ 她微笑着说:“哦,美国,令人神往的地方。”‎ ‎“是的,你的许多同胞都去了美国,你考虑过此事吗?”‎ ‎“不是,不是现在。”她说,“很久以前我曾经想过。但最后我还是留在了这里。留在这里挺不错的。”‎ 喝罢啤酒我们离开客栈。我问父亲:“爸,您给她的信上的日期是怎么写的?”他停下脚步,掏出一信封在上面写了几个字。“像这样,”他说,“12/11/13,就是说1913年12月11日!”‎ ‎“不!”我惊呼,在丹麦和其他任何欧洲国家不是那个日期。在这些地方,人们按日、月、年的顺序写日期。所以你写的日期不是12月11日,而是11月12日!”‎ 父亲用手捂住脸。“哦!她到了那里!”他惊叫道,“只因为我没有赴约,她才跟别人结的婚。”他沉默了片刻。“还好!”他说,“我衷心祝愿她幸福。实际上看来她似乎确实如此。”‎ 当我们总结此事时我突然说:这真是件幸运的事,否则不会遇上我母亲。‎ 父亲双手放在我肩膀上,温和地看着我,微笑地看着我说:“我是双倍的荣幸,小伙子,不然的话,我既不会遇上你母亲更不会遇上你!”‎ ‎1.下列对这篇小说思想内容与艺术特色的分析和鉴赏,最恰当的一项是(  )‎ A.小说没有直接描写“父亲”和“母亲”之间的爱情,而是通过描写“父亲”与一个丹麦女孩之间美丽的误会,侧面表达了“父母亲”之间美好的爱情。‎ B.文中的父亲由于文化传统方面的不同,没有和美丽的丹麦女孩走到一起,后来又遇到了新的爱情,父亲的行为虽有遗憾,但是他的做法也无可厚非。‎ C.小说的结局,既在意料之外,又在情理之中,父亲没有赴约制造了美丽的错误,小说作者颇具艺术匠心的构思,使小说情节一波三折,悬念迭出。‎ D.小说注重于细微处写入,以前美丽的丹麦女孩,但现在发胖了,头发显得有些零乱,说明“她”经受了爱的痛苦,不注重容颜,也不相信所谓爱情。‎ 答案 A 解析 B项“无可厚非”不当,C项“一波三折,悬念迭出”错误,D项“不注重容貌,也不相信所谓爱情”错误。‎ ‎2.本文以“父亲没有赴约”为题有什么好处?请结合全文进行分析。‎ 答:___________________________________________________‎ 答案 ①设置悬念,引发读者兴趣;②与文末呼应,暗示父亲的爱情结局:因与丹麦女孩赴约未成,成就了与母亲的一段美好姻缘;③点出父亲对第一段爱情的遗憾和欣慰:由于两国人民在文化上的一点分歧,导致两人错过了约定的日子,因文化差异造成一场爱情美丽的误会而遗憾,为她有良好的归宿感到欣慰;④侧面表达父亲对爱情的忠贞形象,和表达厮守一生的却只有一个,对双方来说,这本身就是一种爱的荣幸的主题。‎ 解析 本题为赏析小说的标题,题目为“父亲没有赴约”‎ ‎,一、设置悬念,引起阅读兴趣;二、小说的情节相关内容,与父亲的爱情结局相呼应;三、突出了父亲对妻子的忠贞;四、凸显对爱情忠贞不二的主旨。并结合文章相关内容进行分析与总结。‎ ‎3.根据你对小说的理解,请简要分析父亲这个人物形象。‎ 答:___________________________________________________‎ 答案 ①对爱情忠贞,热烈;②前阶段:痛苦,遗憾;③后阶段:幸福,荣幸;④理性,稳重。‎ 解析 鉴赏人物形象,解答时要抓住相关情节内容和人物描写方法分析概括。本文主要情节是“巧遇”,描写方法主要是语言、动作、神态描写。如:“我不得不返回故乡。但是临走前我们见过一次面,我告诉她我必须回美国去借些钱,几个月后回来便跟她结婚”表现父亲对爱情执着、忠贞;他沉默了片刻,“还好!”他说,“我衷心祝愿她幸福,实际上看来她似乎确实如此。”表现了父亲的理性、稳重。当然,从文本的内容来看,父亲的形象还有前后阶段的不同,前阶段是痛苦和遗憾,后阶段是幸福与荣幸。‎ 第四部分 滚动提升训练(六)‎ ‎  时间:70分钟   满分:78分 一、语言文字运用(20分)‎ ‎1.[2017·河南摸底]下列各句中加点成语的使用,全都正确的一项是(3分)(  )‎ ‎①在城镇化的时代背景下,拯救濒危的传统工艺美术品,促进传统工艺美术产业的振兴已经迫在眉睫,不能再拖延下去了。‎ ‎②以人民利益为重是我党的宗旨,各级领导干部应该坚持以民为本的原则,上行下效,全力推动社会各项事业的持续健康发展。‎ ‎③适当运用古词语,可以使语言表达既简洁鲜明,又古朴典雅,但是如果理解有误,使用不当,反而会贻笑大方。‎ ‎④正在加速发展的山西制造业,要形成一个轻重均衡的结构,不仅需要高科技产品,也需要老百姓日常使用的下里巴人。‎ ‎⑤在南海问题上,我们奉劝菲律宾尽早回到谈判解决问题的轨道上来,也奉劝域外势力及时收手,不要试图浑水摸鱼。‎ ‎⑥日前,美国游泳队巴西里约热内卢奥运会选拔赛在美国奥马哈闭幕,美国游泳队在本届奥运会上的参赛阵容水落石出。‎ A.①④⑥ B.②④⑤‎ C.①③⑤ D.②③⑥‎ 答案 C 解析 本题考查正确使用词语(包括熟语)的能力。①迫在眉睫:形容事情已到眼前,情势十分紧迫。②上行下效:上面的人怎么做,下面的人就学着怎么干。多指不好的事。“坚持以民为本”是好事,所以用错对象。③贻笑大方:指让内行人笑话。④下里巴人:比喻通俗的文学艺术。用错对象。⑤浑水摸鱼:比喻趁混乱时机攫取不正当的利益。⑥水落石出:水落下去,石头就露出来,比喻真相大白。不合语境。‎ ‎2.[2017·江西联考]填入下面文段空白处的词语,最恰当的一组是(3分)(  )‎ 当然,表面的公正不等于公正。__①__,__②__表面功夫都不做足,内在的真诚性__③__是值得怀疑的。__④__,议员在议会中打架,__⑤__只是作秀,但也有可能是全力为选民争取利益;__⑥__假如他们只顾着打呼噜,连表现一下的念头都没有,那就一定是置选民利益于不顾。‎ ‎①‎ ‎②‎ ‎③‎ ‎④‎ ‎⑤‎ ‎⑥‎ A 然而 如果 一定 那么 虽然 ‎/‎ B 可是 ‎/‎ 肯定 也许 即使 那么 C 但是 如果 大概 比如 也许 而 D ‎/‎ 一旦 似乎 也许 那么 可是 答案 C 解析 ‎ 本题考查语言表达连贯的能力。解答连贯题最重要的方法就是根据上下文意进行推断,选择合适的关联词。“表面功夫”“内在的真诚性”两者之间是条件,结合“怀疑”,此处应该是表示推断,故用“大概”。“议员打架”是举例,“作秀”是一种推断,故用“也许”。‎ ‎3.[2017·银川一中月考]下列各句中,没有语病的一句是(3分)(  )‎ A.作为网络熟人社会的“朋友圈”,已渗透到生活的各个角落,但因传播环境的封闭,它的自我纠错力很弱,产生了各种谣言充斥网络。‎ B.网络之所以有无尽的吸引力,我想一个主要的原因就在于它能隐蔽你的身份,生活在一个虚幻迷离的世界里。‎ C.中国遵循合作共赢的理念,就全球治理机制和合作模式提出了具体可行、既有继承又有创新的“好点子”并付诸实践。‎ D.“国之交在于民相亲,民相亲在于心相通。”在新的历史起点上,中英教育交流合作迎来了更丰富的发展空间和前景。‎ 答案 C 解析 A项,句式杂糅。“产生了各种谣言充斥网络”可改为“产生了各种谣言”或“各种谣言充斥网络”。B项,中途易辙,应为“使你生活在一个虚幻迷离的世界里”。D项,搭配不当,“更丰富的”与“发展空间和前景”不搭配。‎ ‎4.[2015·安徽高考]请根据上下文,在材料的空格处补写恰当的内容。要求:语意连贯,表达简明,每处不超过12个字。(5分)‎ 文学形象如同绘画形象,一样是心中的形象,一样全凭虚构,一样先要用心来看。写人、写物、写环境,必须看得逼真,直至看到细节,方能落笔。__①__,绘画是片断静止的文学。文学用文字作画,所有的文字都是色彩;__②__,所有的线条色彩都是语言。画非画,文非文,画亦文,文亦画。我画,不过再现一句诗,一阕词,一段散文而已;站在画面上的千姿百态的树,全是感动过我的不同境遇中的人物,或者全是我自己;淌过纸表的流水,不论舒缓、激荡,还是迷茫虚渺,更是我一时真切的情绪,这与写作时的心又有何异?‎ 答:①__________________________________________‎ ‎②______________________________________________‎ 答案 ①文学是绵延不断的画面 ②绘画是用笔墨写作 解析 第①处后面说“绘画是片断静止的文学”,结合前文“文学形象如同绘画形象”推测,可填与“文学是绵延不断的画面”类似的句子;第②处由前文“文学由文字作画,所有的文字都是色彩”,结合后文“所有的线条色彩都是语言”推测,可填与“绘画是用笔墨写作”类似的句子。‎ ‎5.[2016·宝鸡模拟]参照下面巴金先生的图像《沉思》,结合你对巴金的了解,刻画你心目中巴金的形象。要求:①使用第二人称,侧重肖像描写;②至少运用一种修辞手法;③语言简明、连贯、得体;④不超过100字。(6分)‎ 答:________________________________________________‎ 答案 你,巴金先生,眉头紧锁,面容消瘦,表情严肃,那或是因失去妻子充满的无限怀念,那或是因丢失小狗包弟还在痛悔万分;你,嘴唇紧闭,是深思自己讲真话的人生;你,雪白的头发,如黄河流水般泻下,是为人正直的体现。‎ 解析 做这一类试题,一定要注意审题,要落实每一条写作要求。如描写肖像,巴金的眉头紧锁,可以反映出他在沉思,可展开合理想象与描写,分层表达。‎ 二、现代文阅读(9分,每小题3分)‎ ‎[2017·甘肃定西联考]阅读下面的文字,完成6~8题。‎ 新丝绸之路经济带的国家战略分析 丝绸之路的历史,可以追溯到汉武帝派遣张骞出使西域之前数千年。张骞到达中亚后,发现那里已经大量使用中国的竹制品和纺织品。西汉使团凿开亚、欧、非三大洲的通道,被德国地理学家李希霍芬命名为“丝绸之路”。实际上,就功能而言,还可以叫“茶叶之路”“瓷器之路”“欧亚使道”,而且地理上的具体路线也不止一条。历史上,开辟丝绸之路绝非仅仅出于贸易目的,当时的中原王朝为了巩固北方边界的安全,在信息极端闭塞的情况下,凭借传闻与使团的勇气和信念,搭建起连接东西文明的桥梁。这个过程既有偶然,更是必然。丝绸之路尽管曲折,但仍然是连接亚欧大陆最便捷的通道,极大地滋养了东西文明的交流,但囿于技术条件、自然条件与政治因素的限制,它难以承载大规模的物质转运的任务。随着航海技术的进步,丝绸之路被效率更高的海运所取代。‎ ‎2013年9月,习近平总书记完整阐述了新丝绸之路经济带的构想,这一构想既与古老的丝绸之路一脉相承,又充分体现了时代特点。在历史坐标系上,我们可以更清楚地认识新丝绸之路经济带构想的内涵。‎ 在新技术条件下,丝绸之路具备复兴的客观条件。古老的陆路运输技术不足以承载产生规模效益的运输量。但是铁路和公路运输技术的发展,极大地降低了陆地运输的成本。据测算,从我国连云港到荷兰鹿特丹,如果通过丝绸之路,运输距离可比海运缩短9000多公里,时间缩短近一个月,运费节约近四分之一。此外,古丝绸之路必须避开山地与沙漠,路线选择范围有限,经济、社会效益不高。而今天,我们的技术水平已经能把铁路修到世界屋脊。因此,与古丝绸之路相比,新丝绸之路覆盖的面积将更广,路线更密集,也更发达,从而可以在更广泛的区域内把资源与市场串联起来。新丝绸之路是在新技术条件下,对古老的交通通道的复兴与拓展。‎ 新丝绸之路构想充分兼顾了国际、国内两方面的战略需求。从国际角度看,丝绸之路两端是当今国际经济最活跃的两个主引擎:欧洲联盟与环太平洋经济带。丝绸之路沿线大部分国家处在两个引擎之间的“塌陷地带”‎ ‎,发展经济与追求美好生活是该地区国家与民众的普遍诉求。这方面的需求与两大经济引擎通联的需求叠加在一起,共同构筑了丝绸之路复兴的国际战略基础。从国内角度看,我国当前的发展需要兼顾地区平衡,并着力开拓新的经济增长点。复兴丝绸之路能带动经济实力较为薄弱的西部地区,有望形成新的开放前沿。‎ 新丝绸之路设想兼顾政治、经济、安全乃至文化利益的均衡发展。中亚地区处于地缘战略要冲,又是东西文明的交汇点,更是近年恶名昭彰的宗教极端势力的发源地。新丝绸之路构想以经济合作为先导与基石,以政治合作为前提与推进手段,以促进文化交流、化解安全风险为重要目标,是具有前瞻性的综合战略规划。‎ ‎(摘编自《人民论坛·学术前沿》)‎ ‎6.根据文意,下列关于“丝绸之路”的表述,不符合原文意思的一项是(  )‎ A.“丝绸之路”也可以称为“茶叶之路”“瓷器之路”“欧亚使道”,它是中原王朝搭建起的连接东西文明的桥梁。‎ B.由德国科学家命名的“丝绸之路”促进了东西文明的交流,张骞出使西域之前,中亚人已经通过这条路进行竹制品和纺织品的贸易活动。‎ C.“丝绸之路”是连接亚欧大陆最便捷的通道,十分曲折,难以承载大规模的物质转运的任务。‎ D.开辟“丝绸之路”既有偶然,更是必然,不仅出于贸易目的,也为了巩固北方边界的安全。‎ 答案 B 解析 据原文可知,中亚人的贸易在张骞出使西域之前,而德国地理学家李希霍芬命名的“丝绸之路”(即“这条路”)是指西汉使团凿开的亚、欧、非三大洲的通道。‎ ‎7.下列各项,不属于表述“新丝绸之路经济带构想”内涵的一项是(  )‎ A.新丝绸之路是在新技术条件下,对古老的交通通道的复兴与拓展,两端是欧洲联盟与环太平洋经济带。‎ B.丝绸之路具备复兴的客观条件,我们的技术水平已经能使新丝绸之路覆盖的面积更广,路线更密集,也更发达。‎ C.新丝绸之路设想兼顾政治、经济、安全乃至文化利益的均衡发展,是具有前瞻性的综合战略规划。‎ D.新丝绸之路的构想需要兼顾地区平衡,着力开拓新的经济增长点,进而带动经济实力较为薄弱的西部地区。‎ 答案 D 解析 张冠李戴。我国当前的发展需要兼顾地区平衡,并着力开拓新的经济增长点。‎ ‎8.下列理解和分析,不符合原文内容的一项是(  )‎ A.古老的丝绸之路滋养了东西文明的交流,而新丝绸之路的构想与它一脉相承,而且充分体现了时代特点。‎ B.政治合作是新丝绸之路构想的前提与推进手段,而促进文化交流、化解安全风险则是它的重要目标。‎ C.新丝绸之路构想的国际战略基础是丝绸之路沿线大部分国家对发展经济与追求美好生活的普遍诉求。‎ D.古代丝绸之路被海运所取代,新丝绸之路却可能因为在运输距离和运输时间上优于海运,承载能够产生规模效益的运输量。‎ 答案 C 解析 以偏概全。“国际战略基础”还包括“两大经济引擎通联的需求”。‎ 三、古诗文阅读(35分)‎ ‎(一)[2017·桂林中学月考]阅读下面的文言文,完成9~12题。(19分)‎ 袁盎者,楚人也,字丝。父故为群盗,徙处安陵。高后时,盎尝为吕禄舍人。及孝文帝即位,盎兄哙任盎为中郎。‎ 绛侯①为丞相,朝罢趋出,意得甚。上礼之恭,常自送之。袁盎进曰:“陛下以丞相何如人?”上曰:“社稷臣。”盎曰:“绛侯所谓功臣非社稷臣社稷臣主在与在主亡与亡方吕后时诸吕用事擅相王刘氏不绝如带是时绛侯为太尉主兵柄弗能正。吕后崩,大臣相与共畔诸吕,太尉主兵,适会其成功,所谓功臣,非社稷臣。丞相如有骄主色,陛下谦让,臣主失礼,窃为陛下不取也。”后朝,上益庄,丞相益畏。已而绛侯望袁盎曰:“吾与而兄善,今儿廷毁我!”盎遂不谢。‎ 及绛侯免相之国,国人上书告以为反,征系清室,宗室诸公莫敢为言,唯袁盎明绛侯无罪。绛侯得释,盎颇有力。绛侯乃大与盎结交。‎ 袁盎常引大体慷慨。宦者赵同以数幸,常害袁盎,袁盎患之。盎兄子种为常侍骑,持节夹乘,说盎曰:“君与斗,廷辱之,使其毁不用。”孝文帝出,赵同参乘,袁盎伏车前曰:“臣闻天子所与共六尺舆者,皆天下豪英。今汉虽乏人,陛下独奈何与刀锯余人载!”于是上笑,下赵同。赵同泣下车。‎ 上幸上林②,皇后、慎夫人从。其在禁中,常同席坐。及坐,郎署长布席,袁盎引却慎夫人坐。慎夫人怒,不肯坐。上亦怒,起,入禁中。盎因前说曰:“臣闻尊卑有序则上下和。今陛下既已立后,慎夫人乃妾,妾主岂可与同坐哉!适所以失尊卑矣。且陛下幸之,即厚赐之。陛下所以为慎夫人,适所以祸之。陛下独不见‘人彘’乎?”于是上乃说,召语慎夫人。慎夫人赐盎金五十斤。‎ 然袁盎亦以数直谏,不得久居中,调为陇西都尉。‎ ‎(节选自司马迁《史记》,有删改)‎ ‎[注] ①绛侯:即周勃,刘邦的开国大臣。②上林:秦汉时皇家苑囿。‎ ‎9.下列对文中画波浪线部分的断句,正确的一项是(3分)(  )‎ A.绛侯所谓/功臣非社稷臣/社稷臣主在与在/主亡与亡/方吕后时/诸吕用事擅相/王刘氏不绝如带/是时绛侯为太尉主兵/柄弗能正 B.绛侯所谓功臣/非社稷臣/社稷臣主在与在/主亡与亡/方吕后时/诸吕用事/擅相王/刘氏不绝如带/是时/绛侯为太尉主/兵柄弗能正 C.绛侯所谓功臣/非社稷臣/社稷臣主在与在/主亡与亡/方吕后时/诸吕用事擅相/王刘氏不绝如带/是时/绛侯为太尉/主兵/柄弗能正 D.绛侯所谓功臣/非社稷臣/社稷臣主在与在/主亡与亡/方吕后时/诸吕用事/擅相王/刘氏不绝如带/是时绛侯为太尉/主兵柄/弗能正 答案 D 解析 本题考查文言文断句的能力。做断句题应首先在通读全文的基础上理解需要断句部分的大致意思,其次要先易后难,寻找标志性词语、虚词及固定句式等。还应根据句子成分进行断句,确保断后的句子是完整的。做此题把握两个关键点:①文章涉及的是汉朝刘氏天下,故“王刘氏”的划分是错误的,排除A、C两项。②“主兵柄”是个动宾短语,意为“掌握兵权”,意义完整,不应断开。故排除B项。‎ ‎10.下列对文中加点词语的相关内容的解说,不正确的一项是(3分)(  )‎ A.古人的名和字之间是有联系的,有的名与字含义相同或相近,比如诸葛亮,字孔明,“亮”与“明”的字义十分相近;有的则含义相反,如袁盎,盎是充盈的意思,字“丝”,则是少的意思。‎ B.孝文帝即汉文帝,“孝文”是谥号。谥号是古代帝王、诸侯、贵族、大臣、后妃等具有一定地位的人死后,根据其生平事迹与品德修养所给予的称号,有褒有贬,如范仲淹的“文正”是褒,周厉王的“厉”是贬。‎ C.坐姿有多种:“常同席坐”这个“坐”与我们现在的坐姿是相同的,但是“箕踞”的坐姿则是两脚张开,两膝微曲地坐着,形状像箕。是一种轻慢傲视对方的姿态。如“轲自知事不就,倚柱而笑,箕踞以骂”。‎ D.皇后,皇帝的正妻。在后宫的地位就如同天子,是众妃子之主。古代皇后所居的寝宫多位于后宫正中央,因此皇后寝宫称为“正宫”或“中宫”。“中宫”“正宫”也就成了皇后的别称。‎ 答案 C 解析 本题考查识记古代文化常识的能力。古人有多种坐姿,这里“席坐”应指“以膝居地,小腿平置于地,臀部贴于脚后跟”。‎ ‎11.下列对原文有关内容的概括和分析,不正确的一项是(3分)(  )‎ A.袁盎出身卑微,父亲曾当过强盗,吕后掌权时,他还是寄身吕禄门下的家臣,孝文帝时靠哥哥保任他为中郎。‎ B.袁盎直言敢谏,他曾在孝文帝面前指出周勃的过错,要文帝坚守君臣礼节,致使周勃对他有怨气,但在周勃被无辜下狱时,他又替周勃辩说,使周勃获释,两人又成了好朋友。‎ C.袁盎有些行为可以说公私兼顾。宦官赵同受文帝宠幸,常常暗中伤害袁盎,袁盎对他很头痛。于是找机会当面羞辱了赵同,文帝只是笑一笑,让赵同下车。‎ D.一次,孝文帝带着皇后和慎夫人到上林苑。袁盎拉慎夫人入座,慎夫人很生气,不肯入座,皇帝也很恼怒,起身进入内室。后来经袁盎解释原因,皇上和慎夫人原谅了他。‎ 答案 D 解析 本题考查归纳内容要点、概括中心意思的能力。D项,“袁盎拉慎夫人入座”错,原文倒数第二段前面说“袁盎引却慎夫人坐”,“却”的意思是“后退”,是说袁盎认为慎夫人是妾,不能和皇后平起平坐,所以把慎夫人的座位往后拉。‎ ‎12.把文中画横线的句子翻译成现代汉语。(10分)‎ ‎(1)已而绛侯望袁盎曰:“吾与而兄善,今儿廷毁我!”盎遂不谢。(5分)‎ 译文:_______________________________________________‎ ‎(2)臣闻天子所与共六尺舆者,皆天下豪英。今汉虽乏人,陛下独奈何与刀锯余人载!(5分)‎ 译文:_______________________________________________‎ 答案 (1)过了不久,绛侯(周勃)怨恨袁盎说:“我与你的兄长(袁哙)有交情,现在你小子却在朝廷上诽谤我(或说我坏话)!”袁盎也不向他谢罪(或也不理睬他)。‎ ‎(2)我听说与天子共乘高大车舆的人,都是天下的英雄豪杰。如今汉王朝虽然缺乏人才,陛下为什么单单要和宦官(受过刀锯切割的人或受过阉割的人)同坐一辆车呢!‎ 解析 本题考查理解并翻译文中句子的能力。重点考查文言实词、文言虚词、文言句式的翻译。(1)望:埋怨、责备;廷:名词作状语,在朝廷上;谢:谢罪。(2)共:共同、一起;独:单单;载:坐车。‎ 参考译文:‎ 袁盎,是楚国人,字丝。他的父亲原先聚众为盗贼,后来搬迁定居在安陵。高后当政之时,袁盎曾经做过吕禄的家臣。等到孝文帝登上了帝位,袁盎的兄长袁哙保举他做了中郎。‎ 绛侯(周勃)担任丞相,退朝之后每每急步而出,很是得意。皇上待之以礼,对他非常恭敬,常常亲自送他。袁盎进谏说:“陛下认为丞相是什么样的人?”皇上说:“他是国家的重臣。”袁盎说:“绛侯是通常所说的功臣,并不是国家的重臣。国家的重臣应该是与主上共治国事,存亡与共。当年吕后当政的时候,吕氏诸人掌权,擅自相互称王,以致刘家的天下就像丝带一样,几乎快要断绝。在这个时候,绛侯当太尉,掌握兵权,却不能匡正朝纲。吕后逝世,大臣们一起共同反对吕氏诸人,太尉掌握兵权,又恰好遇到那个成功的机会,所以他是通常所说的功臣,而不是国家的重臣。丞相如果对皇上表现出骄傲的神色,而陛下却谦虚退让,臣下与主上都违背了礼节,我私下认为陛下这样不可取。”以后在上朝的时候,皇上逐渐威严起来,丞相也逐渐敬畏起来。过了不久,绛侯(周勃)怨恨袁盎说:“我与你的兄长(袁哙)有交情,现在你小子却在朝廷上诽谤我(或说我坏话)!”袁盎也不向他谢罪(或也不理睬他)。‎ 等到绛侯被免除了丞相的职位,封国中有人上书朝廷,告发他谋反,于是绛侯被召进京,囚禁在监狱中,皇室宗亲以及诸位公卿都不敢替他说话,只有袁盎出来证明绛侯无罪。绛侯得以被释放,袁盎出了不少力。绛侯于是与袁盎倾心结交。‎ 袁盎常常称引些有关大局的道理,说得慷慨激昂。宦官赵同屡屡受到皇上的宠幸,常常暗中伤害袁盎,袁盎为此感到忧虑。袁盎的侄儿袁种担任常侍骑,手持符节护卫在皇帝左右,(袁种)劝说袁盎说:“您和他相斗,应在朝廷上侮辱他,使他所毁谤的话不起作用。”孝文帝出巡,赵同陪同乘车,袁盎伏在车前,说:“我听说与天子共乘高大车舆的人,都是天下的英雄豪杰。如今汉王朝虽然缺乏人才,陛下为什么单单要和宦官同坐一辆车呢!”于是皇上大笑,让赵同下车。赵同流着眼泪下了车。‎ 皇上驾临上林苑,窦皇后、慎夫人跟从。她们在宫中的时候,常常同席而坐。这次,等到就座的时候,郎署长布置席位,袁盎把慎夫人的座位拉后了一些。慎夫人生气,不肯就座。皇上也动了怒,站起身来,回到宫禁中。袁盎就上前劝说道:“我听说尊贵和卑下有次序,那样上下才能和睦。如今陛下既然已经确定了皇后,慎夫人只不过是个妾,妾和主上怎么可以同席而坐呢!这样恰恰失去了尊卑的分别了。再说陛下宠爱她,就重赏她。陛下宠幸慎夫人之举,其实恰好成了祸害她的根由。陛下难道没有听说过‘人彘’之祸吗?”皇上这才高兴,召来慎夫人,把袁盎的话告诉了她。慎夫人赐给袁盎黄金五十斤。‎ 但是袁盎也因为多次直言劝谏,不能长久地留在朝廷,被调任陇西都尉。‎ ‎(二)[2017·全国百校联盟联考]阅读下面这首宋词,完成13~14题。(11分)‎ 添字采桑子 李清照 窗前谁种芭蕉①树,阴满中庭。阴满中庭,叶叶心心,舒卷有余情。  伤心枕上三更雨,点滴霖霪②。点滴霖霪,愁损③北人④,不惯起来听。‎ ‎[注] ①芭蕉:多年生草本植物,叶大、成椭圆形,开白花,果实似香蕉。②霖霪:指雨点绵绵不断,滴滴嗒嗒不停。③愁损:因发愁而损伤身体和精神。④北人:北宋灭亡,词人从故乡山东济南被迫流落到江南,故称“北人”。‎ ‎13.下列对本词的理解和赏析,不正确的两项是(5分)(  )‎ A.“窗前谁种芭蕉树”意思是:在这个国破家亡的时候,人人都栽种芭蕉,以寄托自己的情思。‎ B.词人将“点滴霖霪”组成迭句,不但从音韵上造成连绵悄长的效果,而且烘托了清新明快的气氛。‎ C.词的上片描述芭蕉树的“形”与“情”,下片写夜听雨打芭蕉声。本词写芭蕉,由视觉形象转化为听觉形象,进一步将愁引向深处。‎ D.这首词借吟咏芭蕉抒发了怀恋故国、故土之幽情。上片首句借芭蕉以寄情思,表达了在失意、无奈与痛苦中煎熬的词人对幸福生活的向往。‎ E.这首词将芭蕉与雨联系在一起,就多了几分悲凉凄清。静寂的雨夜,词人辗转难眠,听窗外苦雨,绵长的愁绪,随着雨打芭蕉的声音,更重更深。‎ 答案 AB 解析 A项,“窗前谁种芭蕉树”实际上意思是:在这个国破家亡的时候,不知是谁栽种的芭蕉,以寄托自己的情思。诗句意思被曲解了。B项,词人将“点滴霖霪”组成迭句,不但从音韵上造成连绵悄长的效果,而且烘托了悲凉凄绝的气氛。‎ ‎14.有人评论:“下片结句用‘愁损北人,不惯起来听’煞住,看似平淡,实极深刻。”试结合李清照生平作简要分析。(6分)‎ 答:________________________________________________‎ 答案 从字面上看,“起来听”似乎纯系由于“北人不惯”,但这里的“北人”,实际上应解作“流离之人”“沦落之人”,因此,这种“不惯”也就绝不只是水土气候上难以适应的不惯,而是一种飘零沦丧的异乡之感。‎ 解析 上片描述芭蕉树的“形”与“情”。芭蕉树长在窗前,但却能够“阴满中庭”,这就间接地写出了它树干的高大,枝叶的繁茂,树冠的伸展四垂。接着,词人将描写范围缩小到芭蕉树的细部——蕉叶和蕉心。蕉心卷缩着,蕉叶舒展着,这一卷一舒,像是含情脉脉,相依相恋,情意无限深挚绵长。词人通过长夜孤枕难眠、雨打芭蕉之声的单调连绵和客居异乡的飘零处境,抒发了深沉浓重、凄凉难耐的思国怀乡之情。‎ ‎(三)名篇名句默写(5分)‎ ‎15.[2017·西北狼教育联盟联考]补写出下列句子中的空缺部分。(5分)‎ ‎(1)《爱莲说》中与“近朱者赤,近墨者黑”形成对比,表现莲高洁品质的句子是“________________,________________”。‎ ‎(2)《离骚》中,屈原表明即使没有人了解自己也无所谓,只要自己内心美好就可以的句子是“________________,________________。”‎ ‎(3)李白在《蜀道难》中,写出了剑阁地势险要,易守难攻的特点的句子是“______________,一夫当关,万夫莫开”。‎ 答案 (1)予独爱莲之出淤泥而不染 濯清涟而不妖 ‎(2)不吾知其亦已兮 苟余情其信芳 (3)剑阁峥嵘而崔嵬 四、文学类文本阅读(14分)‎ ‎16.[2017·湖南五市十校共同体联考]阅读下面的文字,完成(1)~(3)题。(14分)‎ 去王家村 夏鲁平 去年夏天,家乡发了一场大水。我负责大哥那个村的包保工作,又让我顺便包保王家村。我的主要工作任务是,调查一下还有多少受灾扶贫项目没有得到很好的落实,群众目前需要解决的问题是什么。‎ 来到王家村,我暂时住在村部。我特意向村长王东明打听那个溺水孩子父亲的情况。王东明告诉我,那个孩子的父亲叫王宝贵,是个聋哑人,一直是村里的贫困户。‎ 他告诉我,王宝贵是村里的老大难,水灾前,村里收到上面拨的扶贫款,盖了两间房屋,其中一间给了王宝贵。房子盖完了,王宝贵却不满意,嫌房子盖到了村子里的低洼处。当时村里能盖房子的地方也只能是这里,没办法的事,我们怎么解释都说不通,王宝贵的老婆倒不是聋哑人,我们就跟他老婆讲,王宝贵却在一旁火冒三丈,上来就抓我衣领子。做了好事人家还不领情,你说我这心里该是什么滋味?这几年,大家都奔着钱使劲儿,我猜想这聋哑人挣不到钱,心里不痛快,见到谁,都看不顺眼,有什么办法?这次闹水灾,村里很多人家房屋都不能住人了,王宝贵家的房子完好无损,这回他才看到了自家房子的好处。‎ 不过,通过这次水灾,也的确看出王宝贵家房址有些问题,全村的水全往他家院子里流,他家的院子成了河道。‎ 洪水到来时,上游的水裹卷着死猪死鸡直往下冲,冲过来的还有门板、家具、柴草垛,柴草垛上居然还趴着一个人,不管怎么呼喊,还是被无情的大水冲走了。这时我们惊奇地看见王宝贵站在自家窗台上张望了一阵,往腰上系了一根粗绳子,跳到洪水中,大家以为王宝贵被洪水吓疯了,急于逃生,但我们都想错了。跳到洪水中的王宝贵拼命游到他家院子里那棵老榆树跟前,爬上了老榆树,解开腰上的粗绳子,捆在树干上,他又扶着绳子游回窗台,撅着屁股爬回屋里。‎ 王东明说,王宝贵往那棵大榆树上拴上绳子不长时间,洪水里又漂来一堆柴草垛。柴草垛上蹲着一个三十多岁的女人,她被吓傻了,不喊也不叫,直看着站在房顶上的人群呆愣,柴草垛漂进王宝贵家的院子,正好被那根粗绳拦住,不走了,柴草垛一点点散开,变成了零星几块,又顺着绳子底下漂走。王宝贵老婆在窗台上喊那女人,让那女人拽住绳子,王宝贵从窗口伸出一个木杆,那女人趴在绳子上,一只手攥住木杆,用另一只手和胳膊挽住绳子,王宝贵用劲把女人拽进屋里。刚救完这个女人,紧接着又漂过来一根圆木,圆木上趴着一个男人,那男人倒是灵巧,漂到绳子跟前,伸手抓住绳子,一点点挣扎着爬进了王宝贵家的窗台。有了这男人,王宝贵也多了一个帮手,只要有人漂过来,就被他们拽进屋里,就这么忙活了不知多长时间,王宝贵救出了五六个人。当然也有没救成功的,我们亲眼看见有一块门板漂过来,门板上趴着一个十几岁的小男孩儿,漂进王宝贵家院子的时候,洪水忽然出现旋涡,门板偏离了原有的方向,朝着榆树那边靠去,王宝贵老婆喊,抓绳子,快抓绳子。那男孩儿慢了半拍,伸手没有抓住绳子,又顺着洪水漂走了,王宝贵急了,他冲出窗台欲跳入水中追赶那块门板,被他老婆从后面一把抱住,王宝贵也许急红了眼,回手给了他老婆一巴掌,他老婆的手松开了,那门板却漂远了,即使王宝贵跳入水中,也追不上了,他就眼睁睁看着那男孩儿,沮丧得要命。也就从这时开始,王宝贵不再站在窗台上救人,他拽住绳子,把半个身子探入水中,只要漂过来一个人,他就能及时搭救。当时我们都看傻眼了,想不到这个木呆呆的王宝贵竟有这么大的勇气和胆量。‎ 后来洪水退去,那些被王宝贵搭救的人,有的来看过他,有的根本没来,对于那些来与不来的人,王宝贵全没理会,经历了这么一场大水灾,什么事都不算事了。‎ ‎(选自《光明日报》,有删改)‎ ‎(1)下列对这篇小说思想艺术特色的分析和鉴赏,最恰当的一项是(4分)(  )‎ A.小说擅长人物性格描写,尤其重视细节描写,如王宝贵救人的场面,凸显人物丰富复杂的内心世界。‎ B.小说以村长王东明的叙述为线索,描写了王宝贵这一质朴的农民在灾害面前无所畏惧的性格特征。‎ C.小说善于使用对比手法刻画人物,王宝贵的美好形象就是在与村长王东明一系列言行的鲜明对比中,逐渐凸显出来的。‎ D.小说在人物性格描写方面还注重人物心理的细腻刻画,使人物形象更加丰满。‎ 答案 B 解析 A项,“凸显人物丰富复杂的内心世界”不恰当。C项,不是对比手法,而是先抑后扬的手法。D项,“还注重人物心理的细腻刻画”这一说法不恰当。‎ ‎(2)小说先写了村长介绍王宝贵家庭情况以及盖房子的事,这样处理有什么好处?请简要分析。(4分)‎ 答:________________________________________________‎ 答案 ①采用了欲扬先抑的手法。‎ ‎②表达了村长对王宝贵的不满之情,侧面表现王宝贵的脾气暴躁,为下文王宝贵在灾害中的勇敢表现张本。‎ ‎③丰富了人物形象,深化了文章的主题。‎ 解析 ‎ 分析文段作用可从三个角度去思考:一从内容本身所写。就是看该句段写了什么内容,对表现主旨、抒发情感等有哪些作用。二从上下文关联。即这些句段与材料在上下文起到了什么样的关联作用。这是思考的主体部分。这时的答案应具体。如答“过渡”,必须写出由什么内容过渡到什么内容,“照应”是与哪些内容相照应。三从结合表现手法所起的作用考虑结构作用。‎ ‎(3)作品中的王宝贵有哪些性格特征?请简要分析。(6分)‎ 答:________________________________________________‎ 答案 ①脾气暴躁,性格直率。‎ ‎②心地善良,乐于助人。‎ ‎③面对灾难,有无所畏惧的勇气和胆量。(每点2分)‎ 解析 这种题型可以从以下几个方面切入:①人物的身份、经历、教养、气质等;②人物的外貌、神态、语言、动作、心理等;③特定的社会历史背景;④作者对人物的感情倾向。人物形象结合文本用形容词概括出来。‎ 第五部分 实用类文本阅读 传记阅读 考点二十四 筛选并整合文中信息 考点名片 考点内容 所谓“信息”是就高考实用类文本而言,并不是一般意义上的信息,而是表现为文中现成的词语,或者是藏在语句里面的深层含意,或者是闪烁在字里行间的隐含的意义。所谓“筛选”即确定对象,针对题干的要求及其提供的信息,对照语言材料进行搜索阅读,查找题干信息与语言材料信息对应的信息源。所谓“整合”即调整组合所选信息,把筛选出的有用材料按要求分类,用准确的语言把加工过的信息进行总结概述。‎ 考查形式 表述题。设问方式:①作者这样说的理由是什么?②结合全文,概括×××的主要原因。③作者是从哪几个方面写×××的?请分条概括。④从文中看,哪几件事对×××的学术生涯产生了重要的影响?‎ 趋势分析 题型出现变化,全国卷可能将采用单选题、多选题、简答题的命题方式。‎ 一、阅读下面的文字,完成问题。‎ 王永志:“大总师”飞天梦 李舒亚 一户贫农家的孩子,起初想通过读书改变命运,不曾想命运却推着他的梦想越走越远,他也逐渐成长为中国高科技重大专项——载人航天工程的总设计师。中国人千年的“飞天”梦想终在他的引领下成为现实。这位总设计师就是王永志。‎ ‎1932年,王永志出生在辽宁省昌图县的老房村,他们家主要靠租种财主家的土地和外出打工为生,他通过观察富人家孩子背着书包打雪仗的行为得到启示:求学,是改变命运的唯一出路。‎ ‎1952年,王永志考入清华大学航空系,1955年开始了在莫斯科航空学院的留学岁月,1961年3月王永志婉言谢绝了莫斯科航空学院副院长克里莫夫想让他继续攻读研究生学位的挽留,登上了归国的国际列车。‎ ‎20世纪70年代后期,人到中年的王永志被任命为中国新一代战略导弹的总设计师。他形容自己成了“那个总在探索的人”,永远在研究最前沿的新型号。‎ ‎1988年底,仍停留于纸上谈兵的“长二捆”,凭着三页草图拉到了第一单大买卖。美国休斯卫星公司愿委托中方发射一颗澳大利亚卫星,条件是火箭必须在1990年6月30日前有一次成功的发射试验,否则,不仅中止合同,还要中方赔偿100万美元。‎ ‎1990年6月30日,“长二捆”‎ 火箭如期屹立在西昌卫星发射中心,等候发射。7月16日,在惊天动地的呼啸声中,“长二捆”直上云霄,一举发射成功,实际运载能力超过设计值,达到9.2吨。从此,中国火箭不仅进入国际市场,火箭运载能力还翻了近两番。‎ ‎“我们用别人的钱,赚了外汇,发了工资,还发展了高科技!”追忆起人生中这段难忘的经历,老人高兴得仿佛回到了二十多年前的发射场。‎ ‎1992年9月21日,党中央正式批准“中国载人航天工程”立项。两个月后,王永志被正式任命为中国载人航天工程的首任总设计师。‎ 尽管是自己梦寐以求的任务,但王永志坦言:“载人航天工程是我一生中遇到的最大的挑战。”作为总设计师,王永志面临的首要难题是比美俄起步晚了整40年,什么样的方案才能既符合中国国情,又能“提高综合国力和国际地位,增强民族自尊心和凝聚力”?‎ 在王永志的主持和建议下,专家们研究提出了“三步走”发展战略设想:第一步,发射载人飞船;第二步,发射空间实验室;第三步,建设舱段组合式的空间站。他的核心理念是:不能保守,又要可靠;既要充分利用后发优势,借鉴国外航天的先进经验,又不能照抄照搬,更要融合现代先进技术。‎ 整整7年,王永志带领中国载人航天队伍默默奋战,1999年“神舟一号”发射成功,中国载人航天工程才开始为人们所关注。2003年10月16日6时23分,中国第一艘载人航天飞船“神舟五号”成功返回地面,“太空勇士”杨利伟走出舱门向人们挥手致意……那一刻,王永志落泪了。他说:“在我的一生中,这样眼泪控制不住地往下流,是极少的。我最引以为豪的是,中国载人航天工程完全是靠中国人自己的力量完成的!”‎ 现在,王永志最大的愿望是培养一批中国航天领域未来的领军人物。他说:“我也是年纪越大才越体会到,一个人再能干,他能干多少年、多少事呢?”他寄语年轻人:“勤奋加机遇等于成功。不要羡慕我们这代人赶上了这么多好机会,如今事业发展得越来越大,机会天天有,关键是,你准备好了吗?如果没有准备好,机会就是别人的,不是你的。”‎ ‎(摘自《人民画报》,有删改)‎ 相关链接 ‎①“天宫一号”是中国第一个目标飞行器和空间实验室,于2011年9月29日21时16分3秒在酒泉卫星发射中心发射,飞行器全长10.4米,最大直径3.35米,由实验舱和资源舱构成。它的发射标志着中国迈入中国航天“三步走”战略的第二步第二阶段。它的总设计师是王永志院士。‎ ‎(摘自“百度百科”)‎ ‎②王永志指出,发展载人航天事业有利于推动国民经济的发展。太空有许多对人类有重要价值的资源,人类为了社会进步和生活,需要不断扩大活动领域。载人航天活动,就是人类扩展活动领域和进一步大规模开发与利用空间资源的重要手段。‎ ‎(摘自《神舟飞船总师王永志:“伯乐”钱学森指导我一生》)‎ 王永志在中国载人航天领域做出了哪些贡献?请简要概括。‎ 答:______________________________________________‎ 答案 ①在王永志的主持和建议下,专家们提出了中国载人航天“三步走”的发展战略设想;②王永志带领中国载人航天队伍,成功发射“神舟一号”“神舟五号”,中国载人航天工程走完第一步;③以王永志为总设计师的“天宫一号”成功发射,中国迈入中国航天“三步走”战略的第二步第二阶段。‎ 解析 本题考查筛选并整合文中信息的能力。这是一道信息筛选类题目,解答时首先锁定答题区间,然后从中提取关键词句,最后整合信息分条作答。通读材料可知,此题的答题区间为正文第十段、第十一段和相关链接中的材料①。第十段讲了王永志在战略方面的贡献,第十一段和相关链接中的材料①讲了王永志在实战方面的贡献,如在王永志的带领下,团队成功发射“神舟一号”“神舟五号”等。‎ 二、阅读下面的文字,完成问题。‎ 黄遵宪的外交活动 作为清朝第一任驻日使馆参赞,黄遵宪表现出很强的历史责任感,除协助公使处理外交事务外,他“既居东二年,稍稍习其文,读其书,与其士大夫交游”。黄遵宪不愿埋首经籍,主张“识时赏知今,通情贵阅世”,走经世致用之路。为了澄清过去封建士大夫对日本的糊涂观念,“遂发凡起例,创为《日本国志》一书”。该书以“史家纪述,务从实录”‎ 为指导思想,力求客观地向中国人全面、准确地介绍日本的历史及现状,“详今略古,详近略远,凡牵涉西法,尤加详备,期适用也”。显然,黄遵宪想要通过叙述日本明治维新的改革历史,为中国的改革提供借鉴。书中以叙述日本历史为经,以评论古今得失为纬,用“外史氏曰”的形式,阐发他的见解,从而把自己的改革思想糅合进日本史的叙述之中。‎ ‎1891年11月1日,黄遵宪被任命为清政府驻新加坡总领事。到任后,他详察南洋各岛情形,体察侨民疾苦,并着手改善侨胞待遇。当地英国殖民者设立的华民政务司“名为护卫华人,实则事事与华人为难”,甚至敲诈勒索。黄遵宪一面与英国殖民主义者斗争,一面将《大清律例》中有关财产各条抄出,并译成英文,要求总督交给华侨聚居地承审官“一体遵办”,以保护华侨的财产。同时,黄遵宪还提倡发展华文教育,改会贤社为图南社。他亲任社长,拟定学规,每月设定课题,鼓励南洋诸生学习中国文化,研究地方礼俗,关心民事民疾。当他卸任归国时,门生潘百禄在《送黄观察公度夫子返国》一诗中,用“遂令蛮貊文明开”“无异岭表韩公来”表达对老师的赞誉之情。‎ ‎《马关条约》签订后,准开沙市、重庆、苏州、杭州为通商口岸。中日对方进行具体交涉时,黄遵宪受委派主持苏、杭两地谈判事宜。他谈判的对手是日本著名外交家珍田舍己。当时,珍田摆出一副盛气凌人的架势,要求在苏、杭开埠,专界专管,并且蛮横地说:“奉本国政府接收专管租界之命,但求按约指地。”黄遵宪毫不示弱,不为其气势所慑。他援引《马关条约》条文,指出“新约所评,只许通商,遍查中文、日文、英文,并无许以苏州让给一地,听日本政府自行管理之语”,拒绝了珍田的无理要求,他又亲自草拟《商埠议案》,凡是《马关条约》文本语焉不详的地方,只要有利于挽回中国自主权利的,“无孔不钻”“无微不至”,从而有理有利有节地挫败了珍田舍己的嚣张气焰,在国家民族危亡的情势下为中国争了口气。‎ 黄遵宪在《上某星使论外交家尽职书》中,把他十几年间处理外交事务的经验总结为“挪展之法”“渐展之法”和“抵制之法”,其中最重要的是“抵制之法”,因为在他看来,当时清朝处于列强环视之下,帝国主义瓜分中国的不平等条约已祸害多年,在对外交涉中只图能多挽回些利权,以保国民生计,又因为在弱肉强食的竞争原则下,弱国无外交,与帝国主义的斗争就不能不讲究策略。他阐述说:“于固执己见,则诿以彼国来明我意;于争夺己权,则托于我国愿同协力;于要求己利,则谬谓两国均有利益。不斥彼之说为无理,而指为难行;不以我之说为必行,而请其酌度。”即在谈判中千万不能感情用事,把事情弄僵,贻误全局,一定要掌握好谈判分寸,应想方设法达到自己的目的,同时又不至于使对方下不了台,交不了差。黄遵宪继续阐述说:“不以彼不悦不怿而阻而不行。言语有时而互驳,而辞气终不愤激;辞色有时而受拒,而请谒终不惮烦;议论有时而改易,而主意终不游移。将之以诚恳,济之以坚贞,守之以含忍。”黄遵宪认为,凡此种种交涉手段和谈判策略,最终是要达到使“吾民受护商之益”,即保护并发展民族工商业的目的。‎ ‎(摘编自郑海麟《黄遵宪传》)‎ 下列对传记有关内容的分析和概括,不正确的一项是(  )‎ A.为了纠正封建士大夫对日本的错误观念,实现他“识时贵知今,通情贵阅世”的主张,黄遵宪一到日本,就开始了《日本国志》的编撰。‎ B.黄遵宪从新加坡卸任回国时,他的学生潘百禄在《送黄观察公度夫子返国》一诗中,把他比作唐代曾在岭南蛮荒之地传播文明的韩愈。‎ C.在中日两国关于苏州、杭州作为通商口岸的谈判中,珍田舍己要求专管按约指地,而且态度蛮横,盛气凌人。‎ D.黄遵宪根据他十几年间处理外交事务的心得,归纳出“挪展之法”“渐展之法”和“抵制之法”,从而把分的实践经验上升为外交谈判策略。‎ 答案 A 解析 本题考查筛选并整合文中信息的能力。黄遵宪在日本居住两年后,学习了日本的文字,才开始与日本人交往,开始了《日本国志》的编撰,不是一到日本就开始编辑,故A项错。‎ 三、阅读下面的文字,完成1~2题。‎ 百年彭桓武:“默默无闻”的大师 ‎1938年,从清华大学毕业的彭桓武,远赴英国爱丁堡大学理论物理系留学,并在此后很快展露出在物理学方面的天赋。在海外期间,他与海特勒等人合作的成果“HHP”理论曾轰动一时,成为当时国际物理界公认的介子理论。‎ 然而,到了1947年,在国外事业发展如日中天的彭桓武,却做出了一个让人匪夷所思的选择。当年夏天,他辞别英伦,辗转香港,回到了阔别数年的祖国,选择了条件艰苦的西部边疆,在云南大学当起了物理老师。在新中国成立前夕,他回到了自己的母校清华大学,开启了红旗下的科研和育人生涯。‎ 在清华大学,彭桓武任物理系教授,培养了黄祖洽等一批优秀的物理学家。彭桓武还在北京大学物理系讲授量子力学,并指导北京大学的研究生周光召和严肃。彭先生是我国核物理理论、中子物理理论以及核爆炸理论等多种理论的奠基人,差不多所有这方面的后来工作者,都是他直接或者间接的学生。‎ ‎1978年,彭桓武受命建设中科院理论物理研究所,并担任第一任所长。在他开放的建所思想的影响下,理论物理所很快就形成了活跃的学术氛围,提倡学科交叉融合,吸引了海内外理论物理界的众多优秀专家来所访问交流。这种开放民主的氛围一直保留至今,成为理论物理所精神的精髓之一。‎ 然而,就在理论物理所各项工作步入正轨之后,彭桓武却向中科院递交了辞呈。这是他在建所之初就许下的约定:理论物理所所长“轮流坐庄”,废除终身制,任期为两年。‎ 当年苏联撕毁与中国的合作协议,全面撤走专家,停止供给资料和设备。中央决定调彭桓武到当时的二机部北京第九研究所,顶替撤走的苏联专家,负责核武器物理研究工作。当时,正值原子弹设计的探索阶段,唯一可参考的资料就是一个苏联专家口头讲述的原子弹教学模型。“知其然而且知其所以然”,九所的科研工作就是在彭桓武这样的学术思想指导下起步的。‎ 第一颗原子弹理论设计完成后,彭桓武敏锐地意识到,要迅速组织力量向氢弹原理的探索转移。从1964年年底开始,在彭桓武等的主持下,邓稼先、周光召组织科技人员制订了关于突破氢弹原理的工作大纲。彭桓武不但是原子弹、氢弹理论设计的领导者和参与者,更是核武器科研事业学术民主之风与集体主义精神的倡导者与践行者。‎ 在研究过程中,彭桓武组织进行了各种学术讨论。无论是刚出校门的大学毕业生,还是著名科学家,没有年龄与资历的界限,大家畅所欲言,共同探讨。哪怕会上争得面红耳赤,会下还是非常团结,互相尊重。优良的学术风气是突破原子弹、氢弹原理的利器,更成为九所代代传承的宝贵精神财富。‎ 以彭桓武为首的多位科学家获得了国家自然科学奖一等奖。按国家规定,这项一等奖的唯一一枚奖章应授予名单中的第一位获奖者。可当九所的同志把奖章送去时,彭桓武却坚决谢绝,并且说:“这是集体的功勋,不应由我一个人独享。”‎ 也正是凭着这种高尚的集体主义精神,彭桓武把他获得的何梁何利基金科学与技术成就奖100万港元全部赠给参加第一颗原子弹研发、身体受过伤害或生活困难的默默无闻的30多位同事。‎ ‎2007年2月28日,彭桓武因病辞世。他的遗体器官捐献给了医院进行医学研究,他还将毕生的工资积蓄近50万元捐赠给了中科院理论物理研究所。他留下了这样一份亲笔签字、令人动容的《彭桓武遗嘱》:“丧仪从简,不举行任何会,骨灰不存任何公墓……”‎ ‎(选自《中国科学报》,有删改)‎ 相关链接 ‎①彭桓武在中子物理、辐射流体力学、凝聚态物理、爆轰物理等多个学科领域取得了重要成果,对分子结构提出过新的处理方法,在量子多体问题研究中提出了自洽场的推广理论。他是核潜艇动力方案的领导者,参加了中国第一次地下核试验的理论领导工作。他先后获得国家科技进步奖特等奖、何梁何利基金科学与技术成就奖、“两弹一星”功勋奖章。经国际天文学联合会小天体命名委员会批准,将第48798号小行星,正式命名为“彭桓武星”。……晚年的彭桓武独自一人,过着简朴的生活。‎ ‎(“百度百科”)‎ ‎②与他订交半个多世纪的挚友钱三强多次感叹:“彭桓武默默地做了许多重要工作,但很少有人知道。”他总是不断地开辟新的方向,殚精竭虑地为国家谋福祉,而当新事业后继有人时,他总是悄然引退。在彭桓武眼中,永远都没有个人名利,只有国家与集体。‎ ‎(《彭桓武:“培养物理学家的物理学家”》)‎ ‎1.彭桓武作为物理学家取得了很多成就,请就其中的三个方面进行概括。‎ 答:____________________________________________________‎ 答案 ①理论方面。在海外期间,他和他人合作的“HHP”理论成为当时国际物理界公认的介子理论;他是我国核物理理论、中子物理理论以及核爆炸理论等多种理论的奠基人。②实践方面。担任中科院理论物理研究所第一任所长,是原子弹、氢弹、地下核试验、核潜艇理论设计的领导者和参与者。③‎ 培养人才方面。在云南大学、清华大学、北京大学担任物理老师,培养了黄祖洽等一批优秀的物理学家,指导和影响了许多学生。‎ 解析 本题考查筛选并整合文中信息的能力。解答此题,需在通读全文、了解文章内容的基础之上,根据题干的要求,以“成就”为关键词,从“物理学家”这个身份的角度去搜索彭桓武的相关信息,包括理论、实践、培养人才等多个方面,然后进行分类整合,即可形成答案。‎ ‎2.彭桓武在学术研讨方面有怎样的主张?请简要分析。‎ 答:____________________________________________________‎ 答案 ①开放民主。彭桓武组织的各种学术讨论会上,大家不分年龄与资历,畅所欲言,充分体现了开放民主的研讨之风。②提倡进行学科融合。彭桓武所在的理论物理所学术氛围活跃,提倡学科交叉融合,吸引海内外理论物理界的众多优秀专家访问交流。③提倡合作精神。形成相互团结、互相尊重的科研之风,注重学术研讨中的集体智慧。‎ 解析 本题考查筛选并整合文中信息的能力。解答本题时要注意题干中“学术研讨方面”这一范围的限定。文章第四段和第八段中都讲到了彭桓武是如何开展学术研讨的,考生认真分析这两段的内容,抓住其中的关键信息如“提倡学科交叉融合”“大家畅所欲言,共同探讨”等回答即可。‎ 四、阅读下面的文字,完成问题。‎ 国货强国:津门商界奇才宋则久 ‎15岁时,宋则久来到天津义德泰绸缎庄当了学徒,32岁应邀到天津著名的敦庆隆绸缎庄担任经理,使该号营业畅旺,誉冠全津,每年的贸易额为三百万元以上。‎ 天津工商界成立工商研究会后,宋则久以其威望和热情被推举为会长,该研究会定期开办提倡国货之讲坛。1912年,宋则久发起创立了直隶国货维持会,聘请社会名流顾琅出任会长,自任副会长。为了促进国货运动的深入普及,宋则久亲自撰文,在报上连日刊登。此时的宋则久萌发了一个大胆的想法,就是自行创办一家专卖国货的百货商场,巧的是,清末官办的天津工业售品所破产,正在进行招商承办,于是,宋则久毅然决定辞去经理职务,投资接办工业售品所,创办了天津第一家综合性百货商店。‎ 宋则久生活的那个时代,“洋货”充斥全国,国货缺乏竞争力,工业售品所是块“烫手的山芋”,亲戚朋友都劝宋则久不要做“傻事”,银行也因风险太大而不肯贷款给他,他甚至连合伙人都找不到。只有敦庆隆的董事长纪锦斋在他一番长谈后,明白了他的良苦用心,不但准其辞职,还让他带走了22名业务骨干。‎ 多年后,那场谈话依然历历在目:仅以敦庆隆销售的货品来看,国人衣食住行中的衣,其上游产业几乎被日本垄断,日本此举一石三鸟:其一,凭借大量贸易顺差攫取巨额财富,进一步拉大中国与日本的差距;其二,中国货币是银本位,大量进口日货意味着白银储备外流日本,加剧中国金融动荡;其三,一举打垮中国工商业,使中国变为日本经济上的附庸。我国之贫弱,只有振兴实业之路,别无他路可走,而打开产品销路又是民族工业发展的根本道路。因此,宋则久没有知难而退,他倾尽家财,最终还是接办了工业售品所。‎ 工业售品所重新开张后,十分醒目地打出了“专卖国货”的大旗,并于1914年创办《售品所半月报》,随时印赠国货目录,向群众介绍国货产品。1915年又在所内组织音乐会,同年,为了掀起国货运动的更大声势,国货维持会在天津公园连续三天举行大会,形成人声鼎沸的盛会。1917年组织新剧团、魔术团,经常举行文艺演出活动,演出内容以宣传提倡国货为主要目的。还经常以济贫、赈灾、助学等方式向有关单位捐赠款项,以扩大影响。‎ 接办工业售品所以后,宋则久首先遇到的问题,即是货源问题。为了开辟货源,宋则久从全国各地广泛搜罗产品,采购商品的地区不断扩大,商品的范围除日用品以外,还包括各地工艺品和土特产品。至1923年,工业售品所的经营种类已经由当初的300多种增加到4800多种。‎ ‎1919年“五四”运动爆发后,天津商界罢市响应,宋则久首先响应。在成立的天津国民大会委员会、抵制日货委员会等爱国团体中,宋则久都担任了重要职务。国民政府镇压国民大会,逮捕了很多人,宋则久利用自己的影响力,把所属的爱国团体转入租界内,继续活动。警察厅厅长杨以德打电话警告他不要和学生们一起“闹”,宋则久据理力争,毫不退缩。“七七”事变后不久,天津被日本人占领。宋则久因之前的抵制活动成了日本人的“眼中钉”,不断有日本浪人和特务寻衅滋事,恫吓敲诈。即使在这种情况下,宋则久也极力维持,但是两年后,工业售品所被迫更名为“天津百货售品所”并销售洋货。日本投降后,宋则久立即恢复了专门销售国货的传统。‎ 在那个民族工商业被各种势力打压从而举步维艰的年代,爱国商人有太多的无奈。但疾风知劲草,在手脚被缚的情况下仍能为国倾尽全力,恰见真骨气。‎ ‎(有删改)‎ 相关链接 ‎①“人无精神,虽生犹死;国无精神,虽存犹亡。有精神,万事进步;无精神,万事退步。”‎ ‎(宋则久的座右铭)‎ ‎②(宋则久)在一些慈善团体均曾担任职务和捐赠款项。而宋则久生活极为简朴,一向主张勤俭节约,以身作则。一生不做寿,婚丧事不收礼、不请客。‎ ‎(《爱国商人:宋则久与天津国货售品所》)‎ ‎1.下列对材料有关内容的理解和分析,不符合原文意思的一项是(  )‎ A.日本试图借大量贸易顺差攫取巨额财富,来实现贸易侵略,以进一步拉大中国和日本的差距。‎ B.宋则久从全国各地广泛搜罗产品,除日用品外,还有各地的工艺品和土特产,但直到20世纪20年代他所经营的种类,也未上千。‎ C.“七七”事变之后,由于宋则久之前的抵日活动,不断有日本浪人和特务寻衅滋事。‎ D.宋则久在实践强国之路时,民族工业被各种势力打压而举步维艰,特别是日本侵略者设置的障碍,也给他带来了不小的困难。‎ 答案 B 解析 原文第六段说“到1923年经营种类已增至4800多种”。‎ ‎2.下列对传记有关内容的分析和概括,最恰当的两项是(  )‎ A.宋则久15岁开始当学徒,32岁便担任敦庆隆绸缎庄的经理,使该号营业畅旺,誉冠全津,充分显露了他振兴国货的宏大抱负。‎ B.宋则久在天津工商界有着极高的威望,他被推举为天津工商研究会和直隶国货维持会的会长,积极促进国货运动的深入普及。‎ C.宋则久提倡支持国货,他不顾亲朋好友的坚决反对,毅然决定辞去敦庆隆绸缎庄经理的职务,创办天津第一家综合性百货商店。‎ D.天津被日本人占领后,宋则久遭到日本侵略者的报复,迫于日本人的强力打压,宋则久的工业售品所在顽强坚持两年后无奈开始销售洋货。‎ E.“人无精神,虽生犹死;国无精神,虽存犹亡”,这既是宋则久激励、警戒自己的座右铭,也是他做事做人的真实写照。‎ 答案 ED 解析 A项,“充分显露了他振兴国货的宏大抱负”说法错误,这显露的是他商业经营的卓越才能;B项,宋则久自任直隶国货维持会的副会长,并非被推举为会长;C项,“坚决反对”表述不准确,亲朋好友只是劝他不要做“傻事”。‎ ‎3.宋则久接办工业售品所后,是如何践行国货强国这一爱国思想的?请简要概括。‎ 答:____________________________________________________‎ 答案 ①醒目地打出“专卖国货”的大旗,并采用多种方式大力宣传国货,扩大影响;②广泛开辟货源,增加商品的经营种类,采购各地工艺品和土特产品;③积极参与商界的爱国运动,全力抵制“洋货”。‎ 解析 本题考查筛选并整合文中信息的能力。解答此题,首先要明确题干要求,根据题干要求明确答题区间。题干中的关键点是“接办工业售品所后”“如何践行”“爱国思想”。由第五段中的“工业售品所重新开张后……”可知,信息要从第五段及后面的段落中筛选。仔细阅读第五段及后面的段落可知,宋则久在接办工业售品所后,主要从三个方面来践行国货强国思想:一是打出“专卖国货”的大旗,通过多种形式如创刊、组织文艺演出活动等宣传国货,制造声势,扩大影响力;二是尽心尽力办好工业售品所,解决经营中存在的问题,采购各地工艺品和土特产品;三是直接参与商界的爱国运动,抵制“洋货”。明确了这些,便不难得出答案。‎ 五、阅读下面的文字,完成问题。‎ 台湾第一“绿巨人”郑崇华 ‎①郑崇华创立台达电子的41年间,台达从一个田埂边的15人小工厂,进化为一个在全球拥有95个营业网点、41座生产工厂、55个研发中心、年销售额高达70亿美元的产业帝国。‎ ‎②事实上,在英雄辈出的台湾商界,这段创业史算不得最令人惊叹的传奇。但若论民众口碑,郑崇华却是罕有的“人人都竖大拇指”的企业家,亦被称为“最被低估的商业教父”。比如,他给台湾的大学捐赠“绿色”建筑,设立科研基金,却只字不提自己和台达,而分别以孙运璿和李国鼎的名字命名,以纪念这两位被誉为“台湾科技产业之父”的已故官员。又如,环保电影《不愿面对的真相》上映,他一口气买下几千张电影票,遍赠业界伙伴。‎ ‎③郑崇华是一个实用主义者。埋头于品质与技术的苦干作风,成为台达一次次搏击产业浪潮,在各大IT品牌厂商背后低调崛起的潜在因素。‎ ‎④一直以来,郑崇华始终恪守“不用回扣做生意”的底线。在唯利是图的“商业法则”下,高尚很可能成为高尚者的墓志铭。面对道德与生存的两难选择,郑崇华总能找到一种在底线之上达到目的的办法。当初,郑崇华希望打入飞利浦的供应体系。飞利浦的采购经理斯沃尔先生明确地告诉郑崇华,飞利浦与实力雄厚的日本东光公司合作已久,不可能再找其他供应商。但吃了闭门羹的郑崇华并没有放弃。他经常制造与斯沃尔巧遇的机会。最初,斯沃尔向郑崇华申明:“陪我喝咖啡聊天,非常欢迎,如果要谈生意就抱歉了。”不过郑崇华仍然没有灰心,他抱着交朋友的心态继续与其打交道。机会终于不期而至。飞利浦试产新产品时,要量产出货,可东光的零件却一直达不到标准。最后时刻,斯沃尔找到了郑崇华。郑崇华紧急召集工程师攻克了难题,并通宵赶工生产,如期将2000套高质量的零件交给了飞利浦,解决了其燃眉之急。台达也一举打入了飞利浦全球供应体系。‎ ‎⑤这种“站着挣钱”的底线与办法,一直被郑崇华延续到了大陆市场。当时,“谈业务时提着一箱现金来给回扣”几乎成了一种盛行的潜规则,而郑崇华始终不这样做,这也让台达失去了不少订单。‎ ‎⑥但拒绝潜规则的同时,郑崇华也有争取客户的明办法,比如邀请客户欣赏音乐会,赠送有意义的书籍给客户,把客户请到台湾参观台达总部等。而最重要的,还是充分满足客户需求。一次上海电讯展上,郑州电信局的人向郑崇华指出,台达的通讯电源质量很好,但外形难看,且没有显示电池状况的功能,里面坏了外面看不出来。得知客户的实际需求后,郑崇华立即组织研发设计团队解决问题。第二年的上海电讯展上,台达便展示出改良后的新产品,令郑州电信局的人大为惊叹,甚至向其他省市电信局的人推销台达。至2007年,台达的通讯电源销量在中国大陆及印度市场上高居第一。‎ ‎⑦郑崇华的实用主义,更直接地体现在环保上。企业界对环保有着“既是道德标准又是经营成本”的普遍纠结,而郑崇华则将环保当成了台达41年来的经营主线。‎ ‎⑧踏着产业浪潮打下市场后,郑崇华便沿着这条主线,开始研究产品的多元化。为此,他建立了一套根深叶茂、层层递进的产品研发体系。1999年台达在上海设立研发中心,郑崇华请来老朋友李泽元博士主持研发,并通过他的学术关系网,陆续与清华大学、浙江大学等八所国内高校开展校企合作。台达内部按产品品类实行事业部制,每个事业部都有自己的工程、销售和研发部门。其中的研发部门负责在现有产品线的基础上,着眼于短期或某类特殊客户进行衍生式开发。2003年台达成立总部研发中心,建立了一条从提出构想,到可行性分析,再到成立新事业部的产品开发流程,规定每年拿出营收的5%~6%作为研发费用。从2001年开始,台达每年定期举行全球科技会议,让各事业部针对前沿技术,提出简报并讨论交流,并邀请专家学者出席指导。这使得台达各个分支的研发资源逐渐整合起来,产生协同效应。这套强大的研发体系,为后来台达商业模式转型打下了坚实基础。‎ ‎⑨经过2008年的金融海啸,2009年台达营收衰退,郑崇华及其团队决定向品牌及解决方案转型。2010年,台达开始打造全新的品牌,深耕三大业务范畴,即“电源及零组件”“能源管理”和“智能绿生活”,而其核心就是“节能环保”,这是郑崇华毕生的追求。‎ ‎⑩2012年6月,76岁的郑崇华卸任董事长,让贤于专业经理人海英俊,而他打造的台达电子仍然踏着全球产业的浪潮,奔跑不息。‎ ‎(摘编自《商界》,有删改)‎ ‎1.下列对材料有关内容的理解和分析,不符合原文意思的一项是(  )‎ A.郑崇华在“不用回扣做生意”的情况下取得了巨大的商业成功。他有埋头于品质与技术的苦干作风,因而能抓住机会,高质量地完成订单,获得新的客户。‎ B.郑崇华的“实用主义”是一种务实的经营之道。他拒绝为求一时之利用回扣做生意,而是在底线之上寻找各种有效的方法来争取客户,用高质量的产品和服务来充分满足客户需求,这些做法帮助台达获取了长远的效益。‎ C.郑崇华的“实用主义”是一种埋头苦干的作风。他在风云变幻的IT行业,埋头于品质与技术,踏实苦干,迎战一次次产业浪潮,在各大IT品牌厂商背后实现低调崛起。‎ D.郑崇华的“实用主义”是杜绝华而不实,讲究节能环保。他将多种经营作为经营主线,建立科学完备的研发体系,实现产品的多元化,使台达在商业上取得巨大成功,并能在全球经济浪潮中奔跑不息。‎ 答案 D 解析 原文是“他将环保作为经营主线”。‎ ‎2.下列对传记有关内容的分析和概括,最恰当的两项是(  )‎ A.郑崇华创立台达电子41年,将台达电子由一个小工厂发展为跨国巨头,但经历过金融海啸之后,营收衰退的台达将如何转型,仍是留给继任者的一个难题。‎ B.斯沃尔尽管明确拒绝跟郑崇华谈生意,但在飞利浦试产新产品而东光公司的零件却一直不达标时,他还是想起了郑崇华,而郑崇华也及时解决了飞利浦的燃眉之急。‎ C.在潜规则盛行的商业世界里,台达的发展为我们展示了在底线之上“站着挣钱”的办法,台达通过高质量的产品来满足客户需求,从而实现了低调崛起。‎ D.台达有“不用回扣做生意”的铁规。从短期来看,这会损失触手可及的订单;但从长期着眼,这能极大地降低交易成本,甚至改善行业风气。‎ E.郑崇华从环保“既是道德标准又是经营成本”的纠结中挣脱出来,开始生产多元化的环保节能产品,使之成为企业主要的利润来源,从而实现道德与经营的统一。‎ 答案 CB 解析 A项,“营收衰退的台达将如何转型,仍是留给继任者的一个难题”错误,原文已经介绍了台达的转型方向:深耕三大业务范畴。D项,“能极大地降低交易成本,甚至改善行业风气”表述绝对,应该是“可能会降低交易成本,甚至改善行业风气”。E项,环保“既是道德标准又是经营成本”是企业界的普遍纠结,而郑崇华没有纠结于此。‎ ‎3.郑崇华建立的“产品研发体系”包括哪些方面?请简要概括。‎ 答:____________________________________________________‎ 答案 (1)请专家主持研发,与高校进行学术合作;(2)实行事业部制,各部着眼于短期或某类特殊客户进行衍生式开发;(3)成立总部研发中心,建立新产品开发流程,规定研发费用;(4)定期举行全球科技会议,交流研讨,整合研发资源。(意思对即可。不加筛选地大量抄录原文酌情扣分)‎ 解析 本题考查筛选并整合文中信息的能力。解答本题,首先要确定答题区域。本题的答题区域为第⑧段。该段一共九句话,内容较为复杂。通过逐句精读,可以确定这九句可分为如下五层:①②/③/④⑤/⑥⑦⑧/⑨。然后逐层精读,就可以提炼出各个要点了。‎ 考点二十五 理解文中重要语句的含意 考点名片 考点内容 理解文中重要语句的含意,作为现代文阅读的一个能力点,是一个常考点。从文章的内容看,不同层次的中心句是重要的句子;从结构上说,总领句、总结句和重要的过渡句是重要的句子;从表达上说,富有特色的句子以及比较含蓄而有深层意义的语句等是重要的句子。‎ 考查形式 表述题。一般提问方式:画横线(加点)的句子(或词语)在文中是什么意思?‎ 趋势分析 题型相对稳定,新课标卷仍将采用表述题的命题方式。‎ 一、阅读下面的文字,完成文后问题。‎ 寂静钱钟书 周劼人 ‎12月19日,寂寥的寒夜,清华园日晷旁,烛光隐隐,小提琴哀婉的曲调飘散在清冷的夜空,人们伫立无语,鞠躬,献上白菊。‎ 偶有路人好奇:“这是在祭奠谁?”‎ 有人低声答语:“今天是钱钟书先生辞世10周年。”‎ ‎10年前,钱钟书先生安详离世。遵钱先生遗嘱,“一切从简”,连在八宝山的告别仪式也只有短短的20分钟。“如此寂静!”钱先生的一位生前好友说。那日,清华的南北主干道上飘起了一千只纸鹤,学生们用这种方式,静静地送别他们的老学长。‎ 他的人生,本不寂静。‎ 无论是人们熟稔的《围城》,还是近乎天书的《管锥编》,都惊讶了世人,折服了学界。《管锥编》单是书证就数万条,引述涉及四千位作家、上万种著作!世人惊叹“大师风华绝代,天才卓尔不群”。‎ 然而他却又静静地坐在书斋里,照例埋头读他的书,做他的学问。图书馆内很多冷僻线装书的借书单上,只有他一人的名字。即使是身处困境,他也只是默默地埋头书本。“文革”时他被送去干校劳动改造,能看的只有寥寥几本书,但只要抱起书本来,就能兴致盎然。第一批“大赦”回京的名单中,没有钱钟书,也没有杨绛。他们夫妻二人平静地走回窝棚,杨先生说:“给咱们这样一个棚,咱们就住下,行吗?”钱先生歪着脑袋认真地想了一下,说:“没有书。”‎ ‎“文革”后,对钱钟书先生的称颂日渐声高,然而钱家的书斋内一如既往地平静。他谢绝了一切记者和学者的拜访,有人将此误读为“清高孤傲,自以为是”。‎ 他人的不解,钱先生并未在意过。杨绛先生说:“他从不侧身大师之列……他只想安安心心做学问。”‎ ‎“钱先生做学问是‘心在焉’,”清华大学一位老师说:“而我们今天这个社会上,今天这个校园里,有多少人则是‘心不在焉’。”‎ 清华大学一位博士生说,他多次读《围城》,读第三遍时忽然明白,“围城不是别人给的,正是人在日复一日的生活中为自己编织的。钱先生没有为自己修筑围城,所以,他一辈子都活得坦然,真挚”。‎ ‎10年后的清华,10年后的12月19日,依旧是纸鹤飘飞,烛光摇曳,依旧只有师生们的心照不语。不时有人向钱先生的照片投来好奇的眼光:“这是谁?”‎ 他一生淡泊,未曾想过要轰轰烈烈,但也正是在这种“寂静”中,他书写了后人无法想象也难以企及的波澜壮阔。我们往往只惊叹他“这个脑袋是怎么长的”,却总是忘了去关注他两耳不闻喧嚣事的用心苦读,以及恪守完整人格的刚毅坚卓。‎ 寂静,这是钱先生的心底所愿,不要奇怪为什么他的离世和10周年纪念都如此寂寥,这正是他的一生的格调。‎ 一位热爱他的读者说:“这个世上唯一的钱钟书走了。”是的,这个时代再也没有了钱钟书,但,是不是也因为这个时代不再寂静?‎ ‎(选自2008年12月24日《中国青年报》,有删改)‎ 文章倒数第二段加点词语“格调”的内涵丰富,请简要分析。‎ 答:________________________________________________‎ 答案 钱钟书的“格调”是“静”:在品格修养上,他始终静静地恪守着完整的人格,这样才能一生宠辱不惊;在治学风范上,他始终墨守“寂静”,这是用心苦读、潜心治学的重要条件。(如有其他分析,言之成理亦可)‎ 解析 本题考查体会重要词语的丰富含意的能力,要求学生能够从文本中筛选出能够表现“格调”内涵的事例,可以从品格修养和治学风范上进行分析。‎ 二、阅读下面的文字,完成问题。‎ 雨果奖得主刘慈欣:在深山里缔造“科幻王国”‎ ‎1985年,刘慈欣大学毕业后,被分配到山西娘子关发电厂的计算机室工作,在这个闭塞、落寞却处处回荡着历史韵味的大山中,刘慈欣对科幻文学越发痴迷。‎ ‎《宇宙坍缩》和《微观尽头》是这位“科幻迷”‎ 参加工作之初的两篇试手之作,虽然语言显得有些生涩,描写也较粗糙,但已孕育了他后来创作的小说所具有的气魄大、悬念足的风格。短篇小说写得不过瘾,刘慈欣就开始了长篇创作。1989年,他完成了《中国2185》,两年后又写出了《超新星纪元》。但由于当时科幻文学正处于寒冬,他的《超新星纪元》等作品相继投了几家出版社,得到的回复都一样:“书稿很不错,但出不了……”‎ 直到1999年,刘慈欣的《带上她的眼睛》等科幻小说在《科幻世界》杂志发表。同年,《带上她的眼睛》还荣获了第11届中国科幻“银河奖”一等奖。从此,刘慈欣的科幻作品创作一发而不可收,而且作品质量颇高。‎ ‎2006年,刘慈欣写出了长篇科幻小说《三体》(又名《地球往事》),之后几年又陆续出版了“厚重无极,气象万千”的“三体”系列《黑暗森林》《死神永生》,征服了无数的科幻迷。‎ 刘慈欣的作品承袭了古典主义科幻小说中节奏紧张、情节生动的特征,浓墨重彩地渲染了科学和自然的伟大力量,洋溢着英雄主义的情怀。‎ 在“三体”系列小说中,刘慈欣提出的“黑暗森林法则”与物理学家霍金在2010年提出的对外星人要尽量避免接触的理论不谋而合,这就是科幻的魅力。‎ ‎2015年8月,《三体》荣获第73届雨果奖最佳长篇故事奖,使刘慈欣成为首位斩获世界科幻文坛最高奖的亚洲人,开启了中国的“科幻元年”,让中国文化在国际获得“存在感”。著名计算机和物理学家普里斯科认为“中国科幻作家已在这个领域走到前头,而我们(西方)却在近年来失去了敢于思考和敢于探索的精神”。‎ 刘慈欣说自己“在生活中是一个彻底的现实主义者,只有在科幻里,才有一些理想主义”。“其实,自己的科幻之路就是一条寻找家园的路。”在短篇小说《流浪地球》的前言中,这个理性、低调、沉默寡言的理科男,罕见地以极具浪漫色彩的话来描述自己。‎ ‎“我是疯狂的技术主义者。”他的科幻理念也让他成为了科幻界的另类人物,引起了不少的争议和批评。国际科幻界很长时间里一直在走“科学反思”的道路,主流内容都是表现科学技术的负面影响,刘慈欣却越发坚守自己的阵地,正是这种坚信与守候,让刘慈欣成为了今日中国科幻界无可争议的第一人。‎ 尽管许多读者是被《三体》中对人性的思考打动,但刘慈欣认为主流文学“超级自恋”,而科幻文学就是文学“再一次睁开眼睛的努力”,它让文学与宇宙重新联结起来,从而“超越自恋”。他说,从人文科学角度讲,科幻是为应对人类的超级灾难而产生的,是其他任何文学门类无法替代的。从自然科学角度讲,科幻是为发掘科学的美和震撼而存在的。真正好的科幻作品是要把很虚幻很空灵的东西描写得很真实,像新闻报道那般真实。‎ 从深山发电厂里的普通工程师,到名震世界的“中国科幻王”,刘慈欣用他30年磨一剑的成长故事告诉我们:世间有无限可能,只要你朝着自己感兴趣的方向努力,就终会释放光芒!‎ ‎(有删改)‎ 相关链接 ‎①刘慈欣创作的《三体》,在一定程度上改变了中国科幻文学的属性。科幻文学从20世纪初进入中国那天起,就背负了“文以载道”的重责,负担了过多的社会功能。之后随着中国政权的更迭与社会流变,科幻文学开始在意识形态工具和科普工具之间摇摆。‎ ‎(《刘慈欣:当代中国科幻第一人》)‎ ‎②在将近20年的漫长时光里,他都是一个人安静地创作,难免有一种孤独感和寂寞感,就像自己独自坚守着一片根本没人在意的疆域。‎ ‎(《刘慈欣:坚守科幻文学的疆土》)‎ ‎③宇宙就是一座黑暗森林。在这片森林中,他人就是地狱,就是永恒的威胁,任何暴露自己存在的生命都将很快被消灭,这就是宇宙文明的图景。‎ ‎(百度百科“黑暗森林法则”)‎ ‎④刘慈欣的“地球往事”三部曲时间跨度相当长,内涵非常丰富,知识信息量可以说海量。整个小说就像一部人类生存史,其对于今天现实世界的借鉴意义不可小视。‎ ‎(《从刘慈欣“地球往事”三部曲谈 当代科幻小说的现实意义》)‎ 理解文中画线句子的含意。‎ ‎(1)这就是科幻的魅力。‎ 答:________________________________________________‎ ‎(2)自己的科幻之路就是一条寻找家园的路。‎ 答:________________________________________________‎ 答案 (1)刘慈欣在科幻小说中提出的“黑暗森林法则”与物理学家霍金提出的理论不谋而合,表现了科幻小说拥有深刻的思想性,极具魅力。‎ ‎(2)生活中的刘慈欣是一个现实主义者,而他在科幻小说的创作中则表现为理想主义者,他在科幻小说创作中能寻找到自己的精神家园。‎ 解析 本题考查体会重要语句的丰富含意、品味精彩的语言表达艺术的能力。解答此题,首先要把握画线句子的内容,然后联系该语句的前后文分析其含意。(1)句的内容是讲科幻的魅力,指出文中所讲的科幻的魅力即可。(2)句是作者所说的话,联系文章具体内容分析作者说这句话的原因即可。‎ 三、阅读下面的文字,完成问题。‎ 陈复礼:耄耋大师赤子情 刘伟忠 陈复礼先生是著名的世界摄影大师,与吴印成先生、郎静山先生并称摄影界“华夏三老”。陈老今年97岁高龄,从事摄影时长六十多年。他的摄影艺术成就得到了海内外文艺界的广泛认可,饮誉无数。1996年,中国摄影家协会曾专门召开“陈复礼摄影艺术研讨会”。“年方十八高大全,亦父亦师亦友一巨人”——这是当时中国摄影界给予陈复礼的集体评价。那一年,陈复礼80岁。‎ ‎1916年7月20日,陈复礼出生在广东潮安县官塘镇石湖村,其父陈必名先生饱读诗书,喜爱潮州音乐、戏曲,尤其醉心于笔墨丹青。陈复礼出生于这样的家庭,一方面具有天然的艺术基因,另一方面从小就受到传统文化的熏陶。陈复礼八岁启蒙,上了六年小学后,又读了一年私塾,读完了《论语》《孟子》《左传》等诸多古籍经典。1931年,15岁的陈复礼就读于潮州韩山“广东省立第二师范学校”,又称韩山师范,以“总评甲等”的成绩毕业。之后遭逢战乱,“过番”[注]东南亚,泰国求亲,越南靠友,后又辗转老挝、柬埔寨。直至中年移居香港,生活才算安定下来。‎ 陈复礼回忆说,到越南后,为生存奔波无暇学画,后来在一本法国杂志上看到了台湾摄影家郎静山的摄影作品,感受到摄影也可以如此富有画意,便开始对摄影着迷。画画需要安稳的环境,而摄影却比较灵活,比较适合他当时的生活状态。‎ ‎“诗情画意王摩诘,镜里丹青复礼陈。”这是广东原省委书记王匡对陈复礼的评价。王匡认为,正如唐朝诗人王维的诗句中渗透着画意一样,陈复礼的镜头下散发着浓厚的丹青气息。他将中国画运用空白和墨线来区分物体的方法融入摄影,在黑白摄影中,运用单色来表现被拍摄物体的质感和立体感,体现出中国传统绘画去粗取精、删繁就简、虚实相间、藏露结合的精华。‎ 在风光摄影彰显中国画意的基础上,陈老进一步吸收中国传统绘画艺术的配套元素,将书法、篆刻、诗歌等以题句、落印的方式融入到照片中。他的许多摄影作品,还以著名诗词或化用诗词来命名。这个时候,“中国画意”已经不足以概括他的作品风格,而只能形容为“中国气派”。‎ 高扬着“中国气派”的旗帜,沿着吸收中华传统文化精髓的道路阔步向前,陈复礼收获了他的另一大发明——“影画合璧”。所谓“影画合璧”,即在兼具中国画意、书法、诗词、篆刻等文化元素的照片上,补充中国传统绘画,绘画题材与照片题材有机融合,成为一幅新的“合璧”作品。陈复礼先后与李可染、吴作人、刘海粟、启功、古元、范曾、黄胄、李苦禅、黄永玉、程十发等20多位书画名家合作“影画合璧”作品。现在,参与合作的书画家很多已经故去,这些独特的艺术品变得更加珍贵。‎ 在早期作品中,陈复礼通过摄影镜头表现出对劳苦大众的同情、爱怜和寄望,也寄托着自己对人生和社会的深刻体悟。他的代表作之一《战争与和平》,以铁丝网中的鸽子为主角,配以布满乌云的天空背景。其子陈惊雄说:“这与父亲的经历也是紧密相关的。日本侵华战争迫使他背井离乡,日本侵入泰国迫使他辗转越南,在越南又赶上越法之战,与战争结下了不解之缘。所以通过镜头表现出来的对和平的热爱尤其强烈。”‎ 在改革开放之前,陈复礼的作品以拍摄祖国的名山大川为主,展示华夏大地的秀美风光。改革开放后,陈复礼的作品开始捕捉时代的脉搏,记录社会的变迁,渗透了浓厚的时代色彩。如果说风光摄影着重体现了陈复礼追求的“美”,那么一系列写实性较强的作品则体现了“真”和“善”‎ ‎。对改革开放中出现的一些丑恶社会现象,陈复礼也没有规避,他以镜头为武器,或夸张,或映射,进行犀利的批判。‎ 历尽沧桑的一代大师,如今终于可以静享天伦,颐养天年。但陈复礼先生对摄影艺术界的影响并没有停止发酵,就在2011年的香港苏富比春拍中,他的两幅作品《喜雨》《朝晖颂》作为中国艺术精品亮相,吸引了海内外摄影艺术界的关注。作为当代摄影中国气派的代表者,陈老的影响注定是深远的。‎ ‎(有删改)‎ ‎[注] 过番:旧时闽南人称到南洋谋生。‎ 相关链接 ‎①潮汕一向有艺术之乡的美誉。自小耳濡目染,受影响非常大。举一个例子,潮汕的屋脊,那些画,门口的那些画,尽管是农民的画、工匠的画,但都有一定的艺术功底。我自小看这些,深深理解潮汕乡土文化,在艺术创作的道路上自自然然,不知不觉跟着这条路走。‎ ‎②经过千多年来历代中国画家的刻意经营,在山水和风景创作方面,已发展到了高深的境界。所以从事风景摄影,而不考虑到中国画的创作方法,将是莫大的损失。‎ ‎(陈复礼《论中国画意与风景摄影》)‎ ‎③用闻一多先生的话来讲,诗人的使命是爱,爱自己的国家,爱自己的人民。陈复礼先生用他六十多年的影艺生涯倾诉着一位艺术家对自己祖国的无限热爱,用他的无数作品艺术地再现了时代的发展和中国社会的进步。他的作品也因此赢得了广大民众的喜爱。‎ 结合材料内容,说说你对“年方十八高大全,亦父亦师亦友一巨人”这句话的理解。‎ 答:________________________________________________‎ 答案 ①“年方十八”,是赞誉陈复礼虽年届八十,仍然创新精神不减,赤子之心长存;②“高大全”,指陈复礼的摄影艺术创作具有高境界、大气派和对生活全方位的表现;③“亦父亦师亦友一巨人”体现了陈复礼作为艺术大师的人格、学识和艺术素养的魅力。‎ 解析 本题考查体会重要语句的丰富含意、品味精彩的语言表达艺术的能力。回答此题要注意两个方面:一是说话人;二是这句话具体指陈复礼的哪些特点,如艺术创作上、为人品格上等。这句话是当时中国摄影界给予陈复礼的集体评价。分析这句话中的词语指代陈复礼哪些方面的内容,如“年方十八”,指的是陈复礼已经八十高龄,但创新精神不减;“高大全”,从后文看主要指陈复礼摄影作品的特点,比如境界、对生活的表现等;“亦父亦师亦友一巨人”,体现了陈复礼的学识、人品等。‎ 四、阅读下面的文字,完成1~3题。‎ 国学守望者 定之 旅居杭城,已有数年了。总喜欢杭州的雅静,就着一方湖水,随意游走,或独处一隅,隐于柳桂之间,发些古思,情浓之时,便在山水悠然处,吟读几行先贤的字句,聊遣幽怀。寄情山水,任意东西,这便无疑是人生的至乐了。然数月之前,自我读完由云南人民出版社出版的《姜亮夫全集》之后,此种逸兴便也尽失,随之而来的,则是沉沉的反思,或者是在惊服之余,油然而生的一种深深的自问吧。多年以来,言及国学,总觉遥不可及、难以论道,读完姜公的文集,不得不因敬佩之至而顿生惶恐。我所读到的是一位耿直的老人,持真的学人,多才的文人,大写的巨人,而个人所学之鄙陋,因此妄作此传,以示崇敬之意。‎ 姜亮夫(1902~1995),国学大师,著名的楚辞学家、敦煌学家、语言学家、历史文献学家、教育家,云南昭通人,其概与二十世纪同岁,究其一生,可谓退能独善修身,润物于无声之处,进可兼济一方,导善于众庶之间。于己可为“守身如玉德之花”,于学则是“南北归一学之宗”,堪称一代大师。‎ 姜亮夫在1921年以官费生考入成都高等师范学校,1926年,姜亮夫毕业,而就在其毕业前夕,因不满军阀横行,便在毕业典礼之上,当面指责军阀头目杨森,措辞入骨,险遭祸害。后游东南,欲北上继续求学,遂由沪入京,却遭逢一役,对其触动颇大。在《自订年谱》中,他说:“十八日(1926年3月18日)参与执政府请愿,几及于难,踏燕京大学死难女生杨肩,逾垣出,以此伤心国事,折节读书。”而对于姜亮夫来说,后来在北京为学的时日,则更令他毕生不忘。入京后,他先考入了北京师范大学研究科,后以“备取生”进入清华大学国学研究院,师从王国维、梁启超、陈寅恪、赵元任诸公。而王梁陈三公加之后来苏州拜认的章太炎先生对他的影响则是终身的。诸位先生的治学思想、为人操守、处世态度深深地渗入了姜亮夫后来的学术生活各方面。‎ 后来王国维先生自投昆明湖,这对姜亮夫触动极大。在《自订年谱》中,他言道:“1928年8月,于人世有所了悟,至悲切。初欲自裁不可,然不敢隳而自赎之念起,急去锡,日读《离骚》。请人为章曰与屈子同年。”先师的无故离去,使他猝不及防,虽时隔一年,但依稀昨日,而世间浮沉,所遇种种,更让他无法自拔,欲有一死,不难想象这对于他该是何等的创伤。而在1927年,他便有意长期从事楚辞研究,欲“以此纪念先师王静安(王国维,字静安)先生,继承先师先生(生前)治学的路子”。此后便寄情屈原沉江,致力于楚辞学研究数十年。28岁时,他便写成《屈原赋校注》一书。‎ 此外也有感于当时的社会环境。在《路漫漫兮修远——简述我的学术研究道路》一文中,姜亮夫说:“一九二七年大革命失败后,大多数知识分子对国家前途,个人的出路都感到苦闷和彷徨,我也处在十分困惑之中。加之日军侵华开始,国家处于纷乱危急之中。我想,在我国历代文人中辞章彪炳,深得忠爱之情的代表人物是屈原!他是中华民族精神的象征。为此,我走上了治楚辞,研究屈原的路子。”‎ 在楚辞学方面,姜亮夫最重大的学术建树是以《楚辞书目五种》《楚辞学论文集》《楚辞学今绎讲录》《二招校注》和以180万字的《楚辞通故》为代表的一系列楚辞学论著。其力作《楚辞通故》则被海内外学者誉为“当今研究楚辞最详尽、最有影响的巨著”。这些长期研究的成果及心得,遂奠定了他在楚辞学研究领域的宗师地位。‎ 姜亮夫治学不仅勤恳严整,平日读书也很讲究方法和步骤。他多积资料,自编工具书,且极具系统性,这无疑是他取得学术伟绩的首要方法。就《楚辞通故》而言,就是他四十年不断抄录各代楚辞学方面的零星资料,整理总结出来的,可谓皇皇巨著。姜亮夫说自己是个“钝根人”,治学就是用笨办法来做的。但正是这“笨”办法,方足显了他的大智和功底;返璞归真之美,往往便是学术追求的至高境界。‎ ‎(选自《国学守望者——姜亮夫传略》,有删改)‎ 相关链接 ‎①姜先生的说法:“我总觉得世界上不论什么地方的什么事什么理什么物,他的生成,决不单是为那个地方那个事那个理而有;通通都是为全人类而成立。历史总是为全人类而留的血痕。”姜先生以毕生努力为全人类留下了中国文化的一抹血痕,其情其意,可赞可叹。‎ ‎②姜亮夫本来是要到巴黎大学攻读博士学位,但去了以后,在巴黎的博物馆、图书馆里看到了我国早年流散到法国的敦煌文物和经卷,痛心疾首之后,放弃了攻读博士学位的机会。姜亮夫先生曾和学生谈到过自己在巴黎博物馆里抄录、拍摄中国文物的事情。先生本没有多少积蓄,又是自费留学,那段生活过得非常清苦。先生如此亏待自己,却把省吃俭用下来的钱拿去拍照。‎ ‎③1979年,年逾古稀的姜亮夫主持杭州大学中文系,事务繁难,却接到了教育部发给他的一个函,委托老先生办一个“楚辞进修班”。姜老激动得几天睡不着觉,他说:“教育部给我这么个重托,经过十年动乱,全国有水平的关于楚辞方面的师资力量更少了,楚辞教育水平下降了,为了国家的文化教育,我活着一天就要做一天的贡献。这是一个非常重要的任务,也许是我最大的一次耕耘。”‎ ‎1.下列对材料有关内容的理解和分析,不符合原文意思的一项是(  )‎ A.第一段在全文中的作用是引出传主,概括总领全文,以自己的不足反衬姜亮夫的深厚的国学功底与高尚的人格,激发读者的阅读兴趣。‎ B.传记首段说“油然而生的一种深深的自问”,作者自问实则是自责:一方面自责于自己寄情山水,任意东西,逍遥人生,并以此为乐;另一方面自责于对国学研究不深入。‎ C.关心国事,揭露军阀。面对国家于水火之中而折节读书,姜亮夫研究屈原《楚辞》;在巴黎放弃了攻读博士学位的机会而抄录、拍摄中国文物。‎ D.治学高远,倾心教育。姜亮夫认为一切事理都是为全人类而成立,精研楚辞,以毕生努力为全人类留下了中国文化。姜亮夫倾心教育,古稀之年接受重托,为培养《楚辞》方面的师资而欣喜不已。‎ 答案 C 解析 C项,列举文本内容不足以支持“揭露军阀”。‎ ‎2.下列对传记有关内容的分析和概括,最恰当的两项是(  )‎ A.姜亮夫在巴黎看到了我国早年流散的敦煌文物和经卷,痛心疾首,决意抄录、拍摄,而这番辛苦奠定了他在楚辞学研究领域的宗师地位,成为他学术人生中的重要阶段。‎ B.先师的无故离去和人世沉浮使姜亮夫悲痛欲绝。为纪念王国维先生,也为继承先师治学道路,同时有感于当时的社会,姜亮夫投入对楚辞的研究。‎ C.文章说姜亮夫是一位耿直的老人,持真的学人,多才的文人,大写的巨人,是因为他终世不倦于学术求索,虽久历坎坷,但建树卓著,治学为师,皆为世人所景仰。‎ D.姜亮夫在成都高等师范学校毕业前夕,因不满军阀横行,在毕业典礼之上,当面指责军阀头目杨森,措辞入骨,险遭祸害,这正是他“守身如玉德之花”的表现。‎ E.1926年3月18日,姜亮夫曾参与执政府请愿,几乎遇难,后来他越墙脱离险境。这件事对他触动极大,一方面使他忧患国事;另一方面也使他潜心治学,发奋读书。‎ 答案 BE 解析 A项,姜亮夫在巴黎的所作所为与他在楚辞上的宗师地位无必然联系;C项,还因为其人格高尚;D项,“守身如玉德之花”错。‎ ‎3.文中第二段说姜亮夫“进可兼济一方”,他的“进”具体表现在哪些方面?请简要分析。‎ 答:________________________________________________‎ 答案 ①治学高远:认为一切事理都是为全人类而成立,精研楚辞,以毕生努力为全人类留下了中国文化的一抹血痕。②倾心教育:姜亮夫倾心教育,古稀之年接受重托,为能够培养楚辞方面的师资力量而欣喜不已。③忠诚祖国:关心国事,揭露军阀,积极参加爱国运动;面对国家于水火之中的境况而折节读书,研究屈原、楚辞学;在巴黎放弃了攻读博士学位的机会而抄录、拍摄中国文物。‎ 解析 解答本题的关键是对“进”的理解。“进”指的是有责任意识,甘于奉献。本题的有效答题区间主要集中在第三、四、五段及“相关链接”中的内容。如“相关链接”中,①主要讲他治学高远,为全人类留下中国文化的一抹血痕;③则是写他倾心教育;②则是从忠诚祖国的角度来谈。‎ 五、阅读下面的文字,完成问题。‎ 翻译奇人许渊冲 邓郁 ‎“杨振宁1957年获诺贝尔奖,王希季是70年代长征一号火箭首射成功。我是2014年才得奖,比振宁晚了五十(多)年啊!”93岁的许渊冲鹤发白眉,声如洪钟,中气十足,讲到畅快处,还会把袖口往上撸。8月22日,中国翻译协会在外文局礼堂为获得国际翻译家联盟最高荣誉“北极光奖”的他举行了盛大的授奖仪式。作为和傅雷、钱钟书同时代的资深翻译家,许渊冲已出版了120多本译作,翻译了《楚辞》《诗经》《西厢记》《唐诗三百首》《宋词三百首》等经典,被誉为“20世纪下半叶中国典籍翻译历史上的丰碑”。与他相伴了半个世纪的夫人照君则对我说:“许先生,是一个奇人啊。”‎ 许渊冲是翻译界的“少数派”。多年来,翻译界强调译文要忠实原文。他的翻译却不拘泥于原作,讲求再创造。他认为翻译“要发挥译语优势”。这种“优势论”也成为了他在翻译界备受质疑和诟病的一点。陆谷孙、王佐良、许钧等翻译家都曾公开和他唱反调。‎ 率性、张扬的个性,让他在崇尚低调的翻译圈子里“独树一帜”。不少人读许渊冲的回忆录《逝水年华》,既觉得痛快,又感叹此人毫不自谦。他重视感情,又难说谙熟“人情”。可谓在“狂作文章信手书,一章一句真性情”。‎ 虽然在学术界备受争议,但在中国诗词的翻译成就上,许渊冲早已得到国内外公认。自1980年起,他开始致力于把唐诗、宋词、元曲译为英法韵文。已故宾州大学教授顾毓琇赞誉许译:“历代诗、词、曲译成英文,且能押韵自然,功力过人,实为有史以来第一。”但他的成功,却是从挨批斗开始的。1958年,许渊冲被打成右派。‎ 一次在烈日下挨斗,又热又累。他忽然想起毛泽东《沁园春·雪》,就默默在心里试着将其译成英文。“说来也怪,我一译诗,什么热、累、批、斗全都没了,眼里看到的仿佛只有山舞银蛇,原驰蜡象,心里想到的只是略输文采,稍逊风骚。等我把词译完,批斗会也结束了。”‎ 在回忆录中,许渊冲更是不吝笔墨地描述母校西南联大。那时候的联大可谓大师云集。闻一多讲《诗经》,刘文典讲《史通》,罗庸讲唐诗,蒲江青讲宋词,萧乾谈“创作与译诗”,卞之琳谈“写诗与译诗”……这些都奠定了许渊冲的中国传统文化和西洋文化的根基。而他毕生追求的“从心所欲,不逾矩”‎ 的翻译准则,则来自于朱光潜和钱钟书的影响。受朱光潜的熏陶,他奠定了诗译的理论基础:不但要写景,还要传情;不仅存义,而且存音。他屡次写信向钱钟书请教诗词翻译中的问题,后者都不吝回信点拨,给了许渊冲无穷的动力。‎ 北大畅春园的许家,一套70平米的简陋居室。其中最惹眼的陈设,莫过于满满当当的书架。那座透明的“北极光”奖杯即被搁在书架的最上层,不踮着脚,几乎意识不到它的存在。比起奖杯,许渊冲更亲近的是书桌上的绿格白底稿纸、放大镜,和一台看不出年代的长城台式电脑。“翻译是他一生中最重要的事。他在我心中就像是普罗米修斯,永远进行着自己的事业。”和许渊冲相交甚笃的翻译家许钧说。‎ 如今他的作息如钟表一样规律:早9点到阳台做操,早饭后翻译《莎士比亚集》,午睡后看看报纸,晚7点收看《新闻联播》,饭后接着翻译,直到深夜。有时他会半夜里坐起,打开电灯,把梦里想到的东西写下,生怕第二天忘记了。‎ 晚年的许渊冲爱读《参考消息》,常看《海峡两岸》。早年在西南联大受到的自由民主思想熏陶,现在被包裹在一颗浓烈的爱国心里。他心里时刻怀揣着让中华文化成为世界文明主流的愿望。他引用杨振宁的话:“我一生最重要的成就是帮助克服了中国人觉得自己不如人的心理。英文和法文是英美人和法国人的最强项,中国人的英法文居然可以和英法作家比美,这也可以长自己的志气,灭他人的威风了。”许渊冲已年过九旬,但精神矍铄,正如朱自清的旧诗所云:“但得夕阳无限好,何须惆怅近黄昏!”‎ ‎(节选自2014年第9期《人物周刊》,有删改)‎ 相关链接 ‎①许渊冲,北京大学教授,翻译家。被誉为“诗译英法唯一人”,1999年被提名为诺贝尔奖候选人。2010年,中国翻译协会授予他“翻译文化终身成就奖”。2014年8月2日荣获国际译联最高荣誉“北极光奖”,系首位获此殊荣的亚洲翻译家。‎ ‎②杨振宁、王希季都是许渊冲西南联大时期的同窗。‎ ‎1.下列对材料有关内容的理解和分析,不符合原文意思的一项是(  )‎ A.许渊冲93岁获奖,虽比同学杨振宁、王希季都晚很多年,但发言时激情澎湃,声如洪钟,中气十足,体现出其通过努力扬眉吐气的自豪感。‎ B.许渊冲晚年勤奋译著,坚持不懈。有时深夜也会坐起,把梦里想到的东西写下,体现其积极作为的进取精神。‎ C.许渊冲晚年常看《参考消息》《海峡两岸》,关注时事。这种心系天下的爱国情怀,充分体现其耄耋之年仍以天下为己任的担当精神。‎ D.文章结尾援引朱自清的诗句“但得夕阳无限好,何须惆怅近黄昏”,这句话充分表现了许渊冲虽年过九旬仍执着进取、坚持作为的积极乐观精神。‎ 答案 A 解析 体现其老而弥坚的乐观精神,不应是“扬眉吐气”。‎ ‎2.下列对材料有关内容的分析和概括,最恰当的两项是(  )‎ A.许渊冲的“优势论”在翻译界备受质疑和诟病,这也使得他的翻译成就直到2014年获“北极光奖”才被认可,可谓大器晚成的代表。‎ B.许渊冲把“北极光”奖杯搁在书架的最上层,不踮着脚,几乎意识不到它的存在。可见,许渊冲并不在乎这个奖,更关心他的翻译事业。‎ C.许渊冲引用杨振宁的话“我一生最重要的成就是帮助克服了中国人觉得自己不如人的心理”,既表明了对杨振宁观点的赞同,也表现了他自己强烈的民族自豪感。‎ D.许渊冲翻译成就的取得,离不开西南联大老师们的影响。他曾屡次向钱钟书当面请教翻译问题,后者更是悉心点拨,给了他无穷的动力。‎ E.“翻译是他一生中最重要的事。他在我心中就像是普罗米修斯,永远进行着自己的事业。”翻译家许钧的话说明他虽公开和许渊冲唱过反调,但并不影响他对许渊冲的认可。‎ 答案 CE 解析 A项,他的翻译成就早已得到公认;B项,“并不在乎”说法不严谨;D项,是“写信”请教。‎ ‎3.“与他相伴了半个世纪的夫人照君则对我说:‘许先生,是一个奇人啊。’”请结合材料简要概述许渊冲“奇”在何处。‎ 答:__________________________________________________‎ 答案 ①他是翻译界的“少数派”。坚持翻译“优势论”,不拘于原作,讲求再创造。②他在翻译圈里“独树一帜”。他个性率性、张扬,重视感情,又难谙人情。③虽在学术界备受争议,但他在中国诗词翻译的成就得到国内外公认。诗词曲译成英文,实为第一,被誉为“诗译英法唯一人”。④他的成功,居然是从挨批斗开始的。‎ 解析 此类题目可在通读全文的基础上筛选出相关信息,然后归纳概括。本题可从原文中筛选出如下信息:“许渊冲是翻译界的少数派。多年来,翻译界强调译文要忠实原文。他的翻译却不拘泥于原作,讲求再创造。”“率性、张扬的个性,让他在崇尚低调的翻译圈子里独树一帜。”“虽然在学术界备受争议,但在中国诗词的翻译成就上,许渊冲早已得到国内外公认”“历代诗、词、曲译成英文……实为有史以来第一”,被誉为“诗译英法唯一人”“但他的成功,却是从挨批斗开始的”。根据这些信息,考生稍作整理可得到答案。‎ 考点二十六 分析语言、文体及写作特色 考点名片 考点内容 分析传记的文体特征、语言特色等作为现代文阅读的一个能力点,是一个常考点。一般表现为考查文章的语句、结构(句段作用)和选材组材技巧。‎ 考查形式 设问方式:①画横线(加点)的句子(或词语)在文中是什么意思?②本传记在选材、组材上有何特色?③本文材料的顺序是如何安排的?④作为×××样的传记,本文有什么特点?请简要回答。‎ 趋势分析 题型相对稳定,全国卷仍将采用表述题的命题方式。‎ 一、阅读下面的文字,完成后面问题。‎ 杰克·伦敦的悲剧 ‎①杰克·伦敦(1876~1916)是美国近代著名作家。他和马克·吐温[注],也许是我国读者最熟悉的两个美国作家了。然而,同作为美国现实主义时代的现实主义作家,两个人的结局却大不一样。马克·吐温以七十五岁高龄病死在写作岗位上,实践了他的“工作是世界上最大的快乐”的信念。杰克·伦敦却在四十岁壮年之时,吞服了大量吗啡,在自己豪华的大牧场中结束了一生。‎ ‎②杰克·伦敦的童年是很不幸的。他是一个在旧金山出生的私生子,生父是一个占卜者。后来,母亲嫁给已经有十一个孩子的约翰·伦敦,继父的境况也不好。杰克·伦敦的童年在穷苦的日子中度过。十一岁他就外出打零工谋生,十四岁到一家罐头厂做工,每天工作十小时,得到一元钱,这已经是很不错的了。干了不多久,这个十四岁刚出头的孩子借了一些钱,买了一条小船,参加到偷袭私人牡蛎场的队伍中,希望用这种手段来改善穷困的处境。偷袭中他被渔场巡逻队抓获,被罚做苦工。不久,他放弃了“牡蛎海盗”的营生,当水手去远东。航海生涯,增长了见识,扩大了眼界,遍地的贫困、剥削和暴力,深深地印入杰克·伦敦还没有完全成熟的心灵中。‎ ‎③航海归来,境况并未好转。1894年,十八岁的杰克·伦敦参加了“基林军”,这是当时由平民党人领导的向华盛顿“进军”的失业者组织的一部分。这次“进军”的领导人考克西等在华盛顿以“践踏国会草坪”被捕,进军组织亦遭取缔。杰克·伦敦在退出“进军”行列之后,又继续过流浪生活,监牢、警察局成了他常进常出的地方。‎ ‎④长年的流浪没有使杰克·伦敦丧失生活的信心,他强烈地追求知识,不甘于自暴自弃。即使在飘泊无定、随时会以“流浪罪”被拘捕的困境中,书也总是他的伴侣。1896年他二十岁时,甚至还考进了加州大学。然而,大学的门毕竟不总是向穷困如杰克·伦敦这样的人敞开的。1897年他就被迫退学,同姐夫一起去阿拉斯加淘金。“黄金梦”又很快破灭,他身染重病回家。‎ ‎⑤一条条的路走不通,一件件的事碰壁,杰克·伦敦萌发了写作的愿望。他有丰富的生活经历,有满腔的对穷苦人的同情,在二十三岁(1899年)时,他的第一篇小说《给猎人》发表了,二十四岁时出版了第一个短篇小说集《狼之子》。在这些作品里,淘金工人的生活是杰克·伦敦心爱的题材。‎ ‎⑥杰克·伦敦的思想是混杂的。他读过马克思的著作,也读过黑格尔、斯宾塞、达尔文和尼采的著作。在他青年时代的作品中,人们可以感到他向资本主义社会挑战的脉搏。1907年(时年三十一岁)写的《铁蹄》,指出美国资本主义有向极权主义转变的可能性,还对法西斯主义的兴起和消灭作了有预见性的警告。我国已经有译本的《马丁·伊登》(1909年),是杰克·伦敦的代表作。这本带有自传性的小说,揭露了资本主义社会的残酷无情,对人性的蹂躏、对正义的践踏。主人公伊登依靠个人奋斗成了名,但是成名之后得到的不是欢乐,而是可怕的空虚,结果以自杀了结一生。七年后,它的作者杰克·伦敦真正走上了马丁·伊登的道路。‎ ‎⑦极端的个人主义,尼采的“超人”哲学,把杰克·伦敦带进了一个矛盾的精神世界,使他青年时期具有的向资本主义社会挑战的叛逆者的性格,逐渐消褪,变成了一个玩世不恭的花花公子。‎ ‎⑧1911年,他公开声明,他写作的目的就是为了钱。他在成名之后,得到很多的钱。他认为他有权过豪华奢侈的生活。他曾经用一大笔钱建造一条命名为“斯纳克”(一种想象中的恶兽)的游船;1913年用了十万美元(在当时是一笔惊人的钱财)以近四年时间建造一所名叫“娘居”的别墅,在落成后即将迁居的时候,忽然起火焚毁。这位已经跻身上流社会的大作家,看了看价值十万美元的废墟,摆了摆手,宣布将另建一个庄园。这时的杰克·伦敦已经陷入了不能自拔的拜金主义泥淖,为了得到更多的钱,粗制滥造,写出一些完全背离自己信念的低劣之作。他在1911年时还说过:“我如果自己能够做出选择的话,除了写一篇说明我对资产阶级世界是多么鄙视的社会主义者的文章外,我什么也不会下笔。”可是在1916年1月,他公开声明脱离自己曾经积极参与活动的美国社会党。‎ ‎⑨这位曾经饱尝人世艰辛,也曾经用自己的笔为社会底层的不幸者呼喊过的作家,随着他的成名和发财,沉沦到了极端个人主义的深渊。1916年11月22日,杰克·伦敦用自杀结束了四十年的一生,留下了鱼龙混杂的四十九部著作。‎ ‎(有改动)‎ ‎[注] 马克·吐温(1835~1910),19世纪后期美国现实主义文学的杰出代表,也是幽默大师、著名演说家。他出身贫苦,上学时就不得不打工,做过印刷所学徒,送报人和排字工,后来又在密西西比河上当水手和舵手。‎ 作为一篇评传,这篇传记最突出的写作特点是什么?请结合文章举例说明。‎ 答:___________________________________________________‎ 答案 “评”“传”结合(或夹叙夹议)。传文第②③④段在对杰克·伦敦童年、少年、青年时期的苦难艰辛的流浪生活的叙述之前都有简短的评论,点明他人生坎坷,突出他的坚韧、顽强、不自暴自弃的性格;又如第⑦段,在叙述了杰克·伦敦的丰富的阅读经历和杰出的创作成就之后,又用评论点明其性格蜕变的原因。‎ 解析 此题考查文本的文体基本特征和主要表现手法。对于这篇传记来说,最突出的特点,从很多段落的开头、结尾及文段内容便很容易看出,是叙述和评论相结合,即夹叙夹议或者“评”“传”结合。答题时,结合着②③④⑦段的写作特点来分析即可。‎ 二、阅读下面的文字,完成后面的题目。‎ 温州踪迹 陈孝全 温州市的浙江省立第十中学原系温州府学堂,创办于1902年,校舍是原来的中山书院,辛亥革命后被改为省立第十学堂。翌年,“学堂”改称“学校”。第十师范学校前身为温州师范学堂。1923年实行新学制,十中和十师合并,仍称省立第十中学,分中学部和师范部两部。前者设在仓桥的十中,而后者设在道司前的十师。学校研究部部长兼图书馆主任金嵘轩是浙江瑞安人,早年留学日本东京高等师范学校,和章太炎、陶成章等革命党人有过交往,富有民主主义思想,是浙江省著名的教育家。朱自清就是由他提议聘请来校的。‎ 在春花烂漫的三月,朱自清带着家小来到了这个位于瓯江下游的古城,先在离学校较近的大士门租了一所房子,不久因大士门失火又迁至朔门西营堂34号。这是一座老式的两进平房,前后都有院子,四周有围墙,靠大门处有两间厢房,外面一间当住室,里面一间前半间为书房。朱自清又从学校里借了一张学生自修桌,放在前方的门下。靠墙的0.66米许空隙,放了一把旧藤椅。房子的后半间便做了厨房。厢房外面有花墙把大院隔开,自成小院落,种了些花木,环境还颇幽静。‎ 在十中,朱自清的教学任务相当重,在中学部教国文,又在师范部教公民和科学概论。他教学认真,态度严肃。在课堂上他极力向学生传授新知识、播种新文学种子,还讲究教育方法,注重教学效果。当时一个学生有生动的回忆:‎ 朱先生来教国文,矮矮的,胖胖的,浓眉平额,白皙。经常提一个黑皮包,装满了书,不迟到,不早退。管教严,分数紧,课外还另有作业,不能误期,不能敷衍。‎ 同学们开始都不习惯,感觉这位老师特别啰唆多事、刻板严厉,因而对他没有好感。但日子一久,同学们对朱老师的看法起了变化:‎ 说起教书的态度和方法,真是亲切而严格,别致而善诱。那个时候,我们读和写,用的都是文言文。朱先生一上来,就鼓励我们多读多写白话文。“窗外”“书的自叙”等都是他出的作文题目,并且要我们自由命题,这对于写惯了“小楼听雨记”“说菊”之类文言文的我们来说,是思想上和文笔上的解放。‎ 朱自清还创造了特别的“作文记分法”,他要求学生在作文本首页的一边,将本学期作文题目依次写下,并注明起讫页码,另一边由他记分,首格代表90分到100分,次格为80分到90分,如此顺推下去。他每批改一篇就在应得分数格里标上记号。这样学期结束时,只要把这些记号连接起来,就出现一个升降表,作文水平的升降便一目了然了。这种记分法,大大激起学生对写作的兴趣,激励了他们的学习。学生们都喜欢听他的课,中学部、师范部各年级学生都争着要求他来上课,而朱自清只得奔波于两部之间,尽量满足学生的要求。学生们也常到他家里拜访,向他请教问题,三三两两、络绎不绝,简直是门庭若市了。‎ 有一个刚从日本回来的学生,他的父亲特地托朱自清指点。这个学生是在日本接受的教育,对国文一窍不通,于是朱自清便告诉他,文字的运用和艺术的境界是国际性的,所不同的只在使用的符号上,即文字的不同,要他在这一原则下去领会自己国家的文字。又选《辛夷集》为他讲解,花了近三个月时间,并经常和他闲聊,锻炼他的汉语能力。时间一久,两人结下了深厚的友谊。有时朱自清要他邀请一些同学到三角门外去看妙果寺的“猪头钟”,到江心寺后看古井,渡瓯江去白水,坐河船去探访头陀寺,大家相处十分欢洽。‎ ‎(选自《朱自清传》,有删改)‎ 在这篇传记中,作者主要运用了什么手法来刻画主人公朱自清?请简要分析。‎ 答:___________________________________________________‎ 答案 细节描写。比如作者详细描述了朱自清的“作文记分法”,通过这一细节体现出朱自清教学态度的严谨、认真,教学方法的独特等特点。再如文章第2段中对朱自清住处的详细描述,其用意在于体现朱自清俭朴的生活习惯和沉静的性格特点。‎ 解析 本题考查考生分析文本的文体基本特征和主要表现手法的能力。考生解题时可首先考虑描写人物的手法有哪些,如肖像描写、语言描写、动作描写、心理描写等,然后对照原文看文中描写人物主要运用的手法,并与原文对照分析。如对朱自清的教学法、教学态度,特别是“作文记分法”和他住处的详细描写,这些又如何体现他的精神品质。‎ 三、阅读下面的文字,完成下面题目。‎ 朱启钤:“被抹掉的奠基人”‎ 林天宏 ‎①2006年6月13日下午,一场大雨过后,正阳门箭楼被带着水雾的脚手架包裹得严严实实。北京旧城中轴线上的这座标志性建筑,正经历着新中国成立后规模最大的一次修缮。‎ ‎②由正阳门箭楼北望,长安街车水马龙,它与城楼左右两侧的南北长街、南北池街,一同构成了北京旧城东西、南北走向的交通要道。‎ ‎③我问同行的一个记者:“你知道改造北京旧城,使其具有现代城市雏形的第一人是谁?”“梁思成?”她答道。‎ ‎④这个答案是错误的,却并不让人意外。随着北京旧城改造不断进入媒体视野,梁思成等一批建筑学家已被大众熟知。但少有人知晓的是,从1915年起,北京已开始有计划地进行市政工程建设,正阳门箭楼、东西长安街、南北长街与南北池街,都是在时任内务部总长朱启钤的主持之下改造与打通的。‎ ‎⑤同样少有人知晓的是,1925年,25岁的美国宾夕法尼亚大学留学生梁思成,收到父亲梁启超从国内寄来北宋匠人李诫撰写的《营造法式》一书,兴趣大增,由此走上中国古代建筑研究之路。1930年,梁思成加入中国营造学社,在那里撰写了《中国建筑史》,成为建筑学一代宗师。而《营造法式》一书的发现者与中国营造学社的创始人,正是朱启钤。‎ ‎⑥“朱启钤是中国古建筑研究工作的开拓者与奠基人,没有他,就不可能在20世纪30年代出现像梁思成这样的建筑学领军人物,我们读到《中国建筑史》的年份,还不知要推迟多少年。”中国文物研究所某研究人员曾这样评价。但是,“由于历史原因,他被研究者们有意无意地抹掉了”。‎ ‎⑦朱启钤于1930年创办的中国营造学社,将他的筹划与组织才能发挥得“淋漓尽致”。他为学社请来当时最为优秀的学术精英:东北大学建筑系主任梁思成,中央大学建筑系教授刘敦祯,著名建筑师杨廷宝、赵深,史学家陈垣,地质学家李四光,考古学家李济……他还以其社会人脉,动员许多财界和政界人士加入学社,直接从经费上支持营造学社的研究工作。曾有建筑史家这样评价朱启钤:“人力、物力、财力,这些都是研究工作所必不可少的条件,能把这方方面面的人事统筹起来,是需要非凡之才能的。朱启钤以一己之力,做了今天需要整个研究所行政部门做的事。”‎ ‎⑧有数据显示,截至1937年,营造学社野外实地测绘重要古建筑达206组,因此探索出一整套研究中国古建筑的科学方法,为撰写《中国建筑史》构建了扎实的科学体系。‎ ‎⑨假若没有朱启钤,中国的古建筑研究,又会是怎样的图景?如今,斯人已逝,营造学社停办也已整整60周年。‎ ‎⑩6月13日的那场大雨,将故宫端门外西朝房冲洗得干干净净。游人如织,屋宇间却依旧透着落寞,此处正是营造学社旧址。而位于东城区赵堂子胡同3号的朱启钤故居,住着数十户人家,杂乱之中,依稀可辨当年气魄。‎ ‎(摘自2006年6月21日《中国青年报》,有删改)‎ ‎[注] 朱启钤(1872~1962),曾任全国政协委员、中央文史馆馆员。‎ 文章多处提到梁思成,这对写朱启钤有什么作用?请选一例作简要分析。‎ 答:___________________________________________________‎ 答案 文章通过对梁思成有关事迹的叙述,或直接或间接地写出了朱启钤对我国古建筑研究的重要贡献。所选事例及分析:第③段,文章写女记者认为改造北京旧城第一人是梁思成,间接说明朱启钤不为人所熟知,点明题意,引出下文。第⑤段,梁思成因《营造法式》一书而走上中国古代建筑研究之路,加入中国营造学社,在中国古建筑学界有很高的地位;而《营造法式》一书的发现者与中国营造学社的创始人就是朱启钤,这间接反映出朱启钤鲜为人知的贡献,照应题目。第⑥段,点明如果没有朱启钤,就不可能在20世纪30年代出现像梁思成这样的建筑学领军人物,直接突出朱启钤的贡献。‎ 解析 本题考查其他人物对传主的映衬作用。解答本题,首先要明确作者写梁思成是为了写朱启钤,因此分析时的落脚点必须在塑造朱启钤这一人物形象上;其次要找出文章具体写梁思成的地方,确定其中一处,分析作者在此处写梁思成对表现朱启钤这一人物形象的作用;最后还要分析在文章结构上所起的作用。‎ 四、阅读下面文章,完成后面的题目。‎ 梁启超面面观 梁启超很欣赏孔子所说的“智者不惑,仁者不忧,勇者不惧”,并以此自励,大智、大仁、大勇的梁启超为后人铭记。‎ 在近现代学术史上,梁启超的成就和影响都非常大。他留下了1400多万字的著述,在33年著述生涯中平均每年要写40多万字,他撰写《陶渊明年谱》三日而成,他用一昼夜完成《戴东原先生传》,他最有名的著作《清代学术概论》,本是为别人作序,结果一发而不可收,用15天写成一本6万字的著作……而这一切都是在他自戊戌以后的政治运动无不参与的情况下完成的!梁启超的治学领域极其广泛,他的著作涉及哲学、史学、文学、图书馆学、社会学等诸多学科。在学术研究上,他不动手则已,一动手便有极大的格局放在那里,不管能否成功。他喜于将某一件事物、某一国学术做一个通盘的打算,进行大规模的研究,永不肯安于小就,做一种狭窄专门的精密工作。‎ 梁启超在《新民说》中把敢于进取冒险作为新国民应具备的品德之一,他一生至少有三次冒险,尤以1916年只身劝广东都督龙济光反袁护国最为惊险——此前梁启超好友汤觉顿前往劝说摇摆不定的龙济光时,竟被龙济光的部下开枪打死。梁启超强压悲愤,冒险出马,与龙济光苦口婆心地谈了十几个钟头,龙济光当时表示心悦诚服,随后龙济光把他手下的军官聚集起来给梁启超开欢迎会,这帮军官个个拖枪带刀,一开始还客客气气,酒过三巡,凶相毕露。梁启超一看耐心说服已无可能,索性豁出去了,对龙济光吼道:“‎ 我单人独马,手无寸铁,跑到你千军万马里头,我本来不打算带命回去。我一来为中华民国前途求你们帮忙,二来也因为我是广东人,不愿意广东糜烂,所以我拼着一条命来换广州城里几十万人的安宁,来争全国四万万人的人格。既已到这里,自然是随你们的便,要怎样就怎样!”梁启超滔滔不绝地演说一个多钟头,声音之大就像打雷,一面说一面不停地拍桌子,把桌子上的玻璃杯震得叮当作响,一时举座皆惊,左右皆靡。梁启超这一吼,不仅救了自己一命,也逼着龙济光宣布独立。‎ 梁启超一生经历过多次大风大浪,但他成不了一个老练的政治家,这与当时的政局有关,也是他的性格使然。他特别看重趣味,曾说:“有人问我,你的人生观拿什么做根底?我便答道拿趣味做根底,凡人必常常生活于趣味之中,生活才有价值。”“我所做的事常常失败——严格的可以说没有一件不失败——然而,我总是一面失败一面做,因为我不但在成功里头感觉趣味,在失败里头也感觉趣味。”梁启超的学问涉及面广,就是因为他兴趣特别广泛。他曾对子女说:“我是学问趣味方面极广的人,我之所以不能专积有成者在此,然而我的生活内容异常丰富,能够永久保持不厌不倦的精神,亦未始不在此。我每历若干时候,趣味转过新方面,便觉得像换个新生命,如朝旭升天,如新荷出水,我自觉这种生活是极可爱的,极有价值的……”‎ 梁启超一生多变,世所共知。他的多变颇遭时人及后人的诟病。有人认为他一生所为学问除文学外都无大价值,不过于初学者有启迪之用,为他学问不能精深而感到惋惜;有人说他政治上“反复无常”——与康有为始合终分,与孙中山合作又对抗,对袁世凯先拥后反。梁启超晚年对自己的学生说,他是有中心思想和一贯主张的,绝不是望风转舵、随风而飘的投机者。郑振铎认为,梁启超最伟大之处,最足以表示他的光明磊落的人格处便是他的“善变”,他的“屡变”。他的变不过是变化的方法而已,他的宗旨、他的目的并未变,那就是爱国。‎ ‎1.下列对材料有关内容的理解和分析,不符合原文意思的一项是(  )‎ A.在学术研究上,梁启超不管能否成功,都乐于创立大格局,着力于大规模的研究,而不肯安于在一种狭窄专门的精密工作中有小成就。‎ B.梁启超的“大智”体现在:近现代学术史上,梁启超著述颇丰;在劝龙济光反袁时,力挽危局。通达人生智慧,认为人要生活于趣味之中;梁启超虽然一生多变,但绝不望风转舵。‎ C.梁启超说的“这种生活”指的是梁启超政坛上风云跌宕的生活。其好处:有助于积极思考探索;能够发现生活的可爱之处;能够正确地看待成败,有不竭的奋斗动力。‎ D.梁启超说的“这种生活”的不足:学问上,可能有多方面的收获,但难以精深;立身处世上,容易多变化,少常态,难以明白他的宗旨原则。‎ 答案 C 解析 “这种生活”指的是梁启超充满趣味的生活。‎ ‎2.下列对传记有关内容的分析和概括,最恰当的两项是(  )‎ A.梁启超自戊戌以后的政治运动无不参与,但他始终成不了一个老练的政治家,这固然与他的性格有关,但更是当时的政局使然。‎ B.汤觉顿劝龙济光反袁护国,却被龙济光部下枪杀,在这种情况下,梁启超仍只身赴虎穴,这一行动不仅体现出他的大勇,也体现出他的大仁。‎ C.梁启超因多变而学问不能精深,郑振铎虽然为此感到惋惜,但又认为“善变”“屡变”是梁启超光明磊落人格的体现,变的是方法,爱国宗旨始终未变。‎ D.文章用了多种手法刻画梁启超形象,既有侧面描写,又有动作、语言、神态描写,从多个侧面生动再现了梁启超的风貌。‎ E.梁任公的治学领域极其广泛,著作涉及哲学、文学、史学、社会学、图书馆学等诸多学科。在学术研究上,总是进行大格局、大规模的研究,永不肯于做一种狭窄的精密工作。‎ 答案 BE 解析 A项,梁启超始终成不了一个老练的政治家,原文说“这与当时的政局有关,也是他的性格使然”,选项中将政局的影响夸大了;C项,为梁启超因多变而学问不能精深感到惋惜的不是郑振铎,张冠李戴。D项,原文中没有神态描写。‎ ‎3.文中多处引用了梁启超的话,有什么好处?‎ 答:___________________________________________________‎ 答案 表现了梁启超勇敢无畏、重人生趣味的特点;有助于读者从多个角度了解梁启超的精神特质;使传记的内容更加真实可信。‎ 解析 首先在文本中找到引用梁启超话的区间,明确对人物性格特点的作用。如对龙济光吼出的话,就体现出梁启超英勇无畏的性格;倒数第二段的话,就体现了其看重人生趣味的性格,并且这一性格特点,前文就有明确的交代。同时,还要注意引用梁启超的话对文本内容、读者等的作用。‎ 五、阅读下面的文字,完成问题。‎ 梅贻琦:是“寡言君子”,也是治校大师 ‎①梅贻琦(1889~1962),字月涵。自1914年由美国伍斯特理工学院学成归国后,到清华担任教授和教务长等。他的一生只做了一件事,就是成功地执掌清华并奠定了清华的“校格”。‎ ‎②1931年12月,梅贻琦在清华大学发表就职演讲,这篇立言名篇中可圈可点者极多。他说,“办学校,尤其是办大学,应有两种目的:一是研究学术,二是造就人才”。他又说,“一个大学之所以为大学,全在于有没有好教授……所谓大学者,非谓有大楼之谓也,有大师之谓也”。能发表这样今天读来仍新鲜的“重要讲话”的人,才算是真正了解了教育的意义——梅贻琦任校长后,不到10年,清华的面貌便彻底改观,声名鹊起。‎ ‎③今天,我们可以随口列出清华很多杰出人物及其事迹,却很少提到梅贻琦。这跟他的教育风格很有关系。梅贻琦用的是“黄老之治”,这种思想推崇黄帝时期的政治清明,发扬老子的清净俭约,奉行无为而治。“黄老之治”希望人尽其才,故梅贻琦治校也信奉让每个人充分表达,只要提出有利于清华的建议,他就会颔首微笑“吾从众”;“黄老之治”奉行法度,故梅贻琦尊崇科学,科学即法则;“黄老之治”奉行无己,故梅贻琦可以忘我地去服务。他说,清华的教师出名、学术成果出名,优秀学生出名,而校长却不必出名。‎ ‎④梅贻琦的寡言少语使一般人或以为他严肃,或以为他谦和,或以为他没有主见。其实他并非不懂幽默,他的反应是才士也为之叫绝的。有人奇怪,那么多校长来来去去,为何独有他能在大师云集的清华立足,梅贻琦回答:“大家倒这个,倒那个,就是没人愿意倒梅(霉)!”他的寡言也并非没人欣赏,陈寅恪就曾说:“假使一个政府的法令,可以和梅先生说话那样谨严,那样少,那个政府就是最理想的。”‎ ‎⑤作为一名学养极厚的物理教授,他自幼熟读经史,背诵如流,曾对朋友笑言:“假如有谁背诵古经传有错漏,我可以接背任何章节。”‎ ‎⑥不少人都能回忆起梅贻琦的律己。他做清华校长时,取消了原有的一些校长特权,不再让学校为他家中工人付工资,并拒绝领取包括手纸在内的一切免费供应。1943年,梅贻琦的母亲去世,当天,同事建议他不开联大常委会。他坚持照常:“不敢以吾之戚戚,影响众人问题也。”……这样的例子举不胜举。1941年,美国空军来华抗日,急需大量翻译,梅贻琦号召联大外文系的男同学参军。1943年,他又动员所有应届四年级身体合格的男生去当美军翻译官。在此期间,梅贻琦的儿子梅祖彦虽然还不到四年级,却提前参军。女儿梅祖彤也随军做了护士。‎ ‎⑦梅贻琦的风度令人难忘。学者何兆武在《上学记》中回忆:“梅校长那时候五十好几了,可是极有绅士风度,平时总穿得很整齐,永远拿一把长长的弯把雨伞,走起路来非常稳重,甚至于拉警报的时候,周围人群乱哄哄,他还是不失仪容,安步当车慢慢地走,同时疏导学生。”‎ ‎⑧因工作繁重,梅贻琦积劳成疾,1960年被确诊罹患癌症。他终身从事教育,毫无积蓄。新竹国立清华大学为其垫付一部分治疗费,医院又酌情减少了一部分,还是不够。清华校友们商议募捐,半年间募集台币65万元。躺在病床上的梅贻琦看到凝聚着爱心的募捐记录,“阅后半晌无语,后曾流泪颔首”。‎ ‎⑨1962年5月19日,梅贻琦溘然长逝。蒋介石特颁“勋昭作育”挽额一方,副总统陈诚送的挽联是:“崇朴学以黜浮华,实大声宏,盛绩久为文苑重;树良规而垂教泽,薪传火尽,道徽犹系国人思”。梅贻琦的墓前广植梅花,刻石“梅园”,为于右任所书。清华海内外校友,每到新竹必去祭拜。当地人士每日清晨也去凭吊,十年如一日,成为新竹一景。教育家蒋梦麟曾说:“中国崛起于近代文明国家之林,(梅贻琦)厥功之伟,莫之与京(争强比胜)。”‎ ‎⑩梅贻琦之执掌清华,是清华之幸,也是中国之幸。‎ ‎(有删改)‎ ‎1.下列对材料有关内容的理解和分析,不符合原文意思的一项是(  )‎ A.治校思想先进。梅贻琦认为办学应有两种目的:一是研究学术,二是造就人才。认为大学要有大师级的教授。‎ B.梅贻琦是“治校大师”,他寡言少语,以严格果断治校,为师生服务,为让每个人充分表达建议,尊崇科学,奉行无己。‎ C.梅贻琦严于律己,甘作表率。他取消原有的一些校长特权,不肯因母亲去世耽误工作,在祖国需要的时候将儿女都送去参军。‎ D.“梅贻琦之执掌清华,是清华之幸,也是中国之幸”,“幸”表明他对清华对中国做出了巨大贡献,也为清华奠定了“校格”。‎ 答案 B 解析 以“黄老风格”治校。‎ ‎2.下列对传记有关内容的分析和概括,最恰当的两项是(  )‎ A.文章开头说“梅贻琦一生只做了一件事”,意在说梅贻琦所做的事虽然很少,但人这一生若能做成一件这样的事,足矣。‎ B.梅贻琦主张“大学者,非谓有大楼之谓也,有大师之谓也”,说明他在学校的管理和发展中不注重校舍的建设,只注重人才的会聚。‎ C.人们提到清华很少提到梅贻琦,是因为他把全部的精力都投入到治校中,于是也就疏于学问的精进。‎ D.第④段中陈寅恪的话既嘲讽了政府制定法令的多而不够谨严,又表达了对梅贻琦的寡言少语的欣赏。‎ E.传记运用平实自然的语言,撷取很多生活片断展示了梅贻琦的寡言君子和治学大师形象。‎ 答案 DE 解析 A项,原文并没有嫌梅“做事少”的意思,表述不准确;B项,“只注重”说法过于绝对;C项,“少有人提”和他的教育风格有关。‎ ‎3.文中两处画线句子皆为引用,请分析其作用。‎ 答:___________________________________________________‎ 答案 ①引用何兆武的话突出梅贻琦即使在非常时期也极具绅士风度。②引用蒋梦麟对梅贻琦的评价,突出梅贻琦在近代历史上的巨大功绩。③两处引用都从侧面丰富了传主形象,增强了传记的真实性。‎ 解析 本题考查鉴赏作品的表达技巧的能力。文中两处画线部分,一处是通过何兆武之口对梅校长的肖像、动作进行描写,另一处是教育家蒋梦麟对他的评价。可以说,这两处都是借他人之口来刻画梅校长这位传主的形象,使人物形象更加突出。所以,分析其作用,主要应从表现人物形象这个方面来作答,涉及人物的性格、功绩等。另外,由于是征引别处资料,所以也增加了本文的可信度。‎ 考点二十七 鉴赏传主形象及表现技巧 考点名片 考点内容 人物传记中都有个主要人物,这个人物就是传主。传主形象的考查,是传记类文本命题的必考点。可以说,只要考查传记,就要涉及对传主品质及性格特点的考查。传主的成功和优秀品质的形成都不是偶然的,它与个人的天赋、良好的性格、环境的影响、后天的努力等因素是分不开的,既有主观因素,又有客观因素。我们分析他们成功的原因,从而汲取自己成长的营养,是我们学习传记的主要目标,也是高考考查的主要命题方向。‎ 考查形式 设问方式:①结合作品,简要分析“×××”这一人物形象。②作品中的“×××”具有什么样的性格特点?请简要分析。③这篇文章表现了“×××”哪些优秀思想品质?④从文中看,“×××”的主要事迹(功绩)是什么?请简要论述。⑤×××事件,反映出传主怎样的性格?‎ 趋势分析 题型相对稳定,全国卷仍将采用表述题的命题方式。‎ 一、阅读下面的文字,完成1~2题。‎ 王国维“道器合一”的文史治学 周传家 ‎20世纪初,随着西学东渐,中国传统文化受到冲击,发生了翻天覆地的变革。早在1902年,梁启超就发表文章批评旧史学的弊端,大声疾呼号召“创新史学”。1919年,毛子水发表《国故与科学精神》,批评“国故”和旧史学,引起张煊的反驳,新旧两派展开激烈交锋。1923年,胡适在《国学季刊》“发刊宣言”中指出:“国故”即“国学”,涵盖“一切过去的文化历史”。他主张“打破一切的门户成见,拿历史的眼光来整统一切”,提出“研究问题,输入学理,整理国故,再造文明”的口号。‎ 王国维深受梁启超新史学观点及胡适“整理国故”口号的影响,坚持从历史着眼,主张“欲知古人必先论其世,欲知后代必先求诸古”,取得了一系列研究成果。特别是他以专门史形式呈现出来的《宋元戏曲史》,“求事物变迁之迹而明其因果”,拓展了传统历史学的广度和深度,不仅成为新史学的典范,而且是公认的中国戏曲史的开山之作,对后世产生了深远的影响。‎ 王国维治戏曲史,不仅坚持历史的观点,而且走的是“即器即道”的文化史路数,从严密的考证出发,用史料说话,绝非虚玄论道,同时也不停留于文化表象就事论事。他溯源探流,追求道器结合——于形下之器透视形上之道,又让形上之道坐实于形下之器,使戏曲这一中华元素从文化事象娓娓道来的展示中得以昭显。‎ 王国维治戏曲史,不仅有“史识”——别具只眼,更重要的是有“史胆”——良知和担当。王国维幼年即喜读《汉书》,早年的《咏史诗二十首》不局限于个体的人生感慨和悲欢心绪,总揽全局,分咏相对完整的“中国全史”,“议论新奇而正大”。处在大厦将倾、内忧外患的历史背景下,面对文物制度的崩溃、历史延裂的残酷现实,王国维“毋忘其天职,而失其独立之位置”,怀着对中国文化灭绝的深度恐惧感,通过古器物、古文字以及古史研究,说明中国文物制度的真实性,表达中国文化复兴的理想。‎ 读王氏之《咏史诗二十首》,不由得使人想起2000多年前屈原的《天问》。屈原有感于楚国的危机和忧患,叩天问地,在对有关天、地、人的某些传说质疑的过程中,体现出他的怀疑精神、思想观念及其为楚国现实政治服务的意图;王国维在《咏史诗二十首》中,面对苍茫历史“呵而问之”,同样是通过卒章显志的惯常创作模式,试图从中发现中国文化的兴衰存亡之道,为风雨飘摇的中国传统文化确立一个历史根基。这绝非迷古恋古,而是民族情怀和爱国精神的生动体现。因此,从某种意义来说,《咏史诗二十首》就是20世纪的《天问》,王国维“就是现代中国的“三闾大夫”,连他结束生命的方式也酷似屈原怀忠而自沉汨罗。‎ 王国维具有扎实深厚的国学功底和严谨科学的治学态度,他在《殷虚书契考释·后序》中指出:“我朝学术所以超绝前代者,小学而已。”他以这种具有科学性质的小学工具来衡定历史存在之真伪,重证据,重事实,不肯在没有确凿证据时乱下结论,更不会想当然地凭空捏造,欺世盗名。对于“不通字例、未习旧艺”之“俗儒鄙夫”的“肆其私臆,无所忌惮”,他深恶痛绝,讥之为“鬼魅之易画”。但他不是迂腐学者、冬烘先生,他“取地下之实物,与纸上之遗文互相释证;取异族之故书与吾国之旧籍互相补证;取外来之观念与固有之材料互相参证”,从而“移一时之风气,示来者以轨则”。这种采用古今中外结合的“古史新证”手段,对于今天的“古史研究”也颇有超越之处,因而成为王氏之学所具有的重要现代学术特征之一,开一代之学术和学风。‎ 吴梅是南派昆曲的著名学者,其曲学研究涉及度曲、制曲、曲史等众多领域,在曲学方面,钱基博先生甚至认为,吴梅代表着20世纪前半期昆曲曲学的最高成就。就治学而言,王国维和吴梅之间有许多共性,也各有其特色。两者走的均是“即器即道”、道器兼容的路子,窃以为王氏更钟情于“道”,而吴氏则偏重于“器”。‎ ‎(选自《中国社会科学报》,有删减)‎ 相关链接 ‎①王国维吸收了西方输入的文学史观念,对中国古典戏曲的发展线索进行了严密的考证和分析,开创了“戏曲史”研究的新领域。他在研究中借鉴了中西诗学中的某些思想意蕴。提出了许多合理而又富有启发性的理论观念,对中国古典戏曲的研究有理论上的重要突破。‎ ‎②《宋元戏曲史》中体现的研究方法,是王国维对戏曲研究的一个突出贡献,被人评其为“转移一时之风气,而示来者以轨则”。王国维的胞弟王哲安将其兄的研究方法概括为:乾嘉学风的严密实证和西学的逻辑推演的相互制约和影响。‎ ‎1.王国维治学的情感态度有哪些特点?请结合材料简要分析。‎ 答:_________________________________________________‎ 答案 ①良知担当:他试图发现中国文化的兴衰存亡之道,为风雨飘摇的中国传统文化确立一个历史根基。②敢于怀疑:王国维在《咏史诗二十首》中,面对苍茫历史“呵而问之”,分咏相对完整的“中国全史”。③科学严谨:他治学从严密的考证出发,用史料说话;重证据,重事实。‎ 解析 通过整体阅读,筛选和把握作者的主要观点态度。从第二、三、四段提取关键句。王国维“坚持从历史着眼”,“从严密的考证出发”,“用史料说话”,有“良知和担当”,像屈原一样有“怀疑态度”,“具有扎实深厚的国学功底和严谨科学的治学态度”,然后分条说明即可。‎ ‎2.结合材料,谈谈王国维在治学方法上有哪些独到之处。‎ 答:_________________________________________________‎ 答案 ①坚持了历史的观点:主张欲知古人必先论其世,欲知后代必先求诸古;②道器结合:主张从严密的考证出发,用史料说话,溯源探流,追求道器结合——于形下之器透视形上之道,又让形上之道坐实于形下之器;③采用古今中外结合的“古史新证”的手段:取地下之实物,与纸上之遗文互相释证;取异族之故书与吾国之旧籍互相补证;取外来之观念与固有之材料互相参证;④吸收了西学逻辑推演的方法。‎ 解析 本题考查对文本的某种特色作深度的思考和判断的能力。须在通读全文的基础上,理解题干中“独到之处”的意思,找出文中的提示语:“取得了一系列的研究成果”,“新史学的典范”,“开一代之学术和学风”等,结合上下文,即可得出答案。‎ 二、阅读下面的文字,完成文后题目。‎ 蔡元培的北大时代 黄艾禾 ‎①北大因蔡元培,从此确立其现代传统和校格。而蔡元培也因北大,成为中国伟大的教育家。‎ ‎②1916年时的北大,虽然已经改名为国立北京大学,其作为“皇家大学”的官僚气与衙门气依然浓厚。在教员中,有不少是北洋政府的官僚,这些教师即使不学无术,也受到学生巴结,巴结是为了日后自己仕途的方便。陶希圣先生对那时的北大曾有这样的回忆:“民国初年,贵族子弟仍然不少。……两院一堂是八大胡同受欢迎的重要的顾客。一堂就是北京大学——京师大学堂。”北大的这种腐败名声,蔡元培早有所闻,朋友们也劝他不要去,担心他“进去了,若不能整顿,反于自己的名声有碍”,然而,蔡元培内心里已经下定决心。实际上,“教育救国”的理念,是蔡元培自戊戌变法失败后一直坚信不移的信念。‎ ‎③1916年12月26日,蔡元培接受了北洋政府大总统黎元洪的北大校长委任状。1917年1月4日,蔡元培赴北大上任。据当时正在北大上学的顾颉刚的回忆,到任那天,校工们在门口恭恭敬敬排队向他行礼,蔡元培“脱下礼帽,郑重其事地向校工们回鞠了一个躬,这就使校工和学生们大为惊讶”。实际上,蔡元培从来也没有把北大校长一职看作是一官职,他不做官,也要求学生们不做官。‎ ‎④蔡元培1月4日到北大上任,1月11日就呈请教育部聘任陈独秀任文科学长。蔡元培与陈独秀个性完全不同,但是蔡元培看中陈独秀的,是他的新锐和毅力。一旦下定决心,就一趟趟“亲顾茅庐”。他去陈独秀的住处拜访陈时,因陈独秀习惯晚睡晚起,他就耐心地坐在门口的一只小板凳上,等待着年龄小他一轮的陈独秀醒来。‎ ‎⑤在蔡元培引进了陈独秀后,又引进了胡适进北大当教师。而胡适当时不过是个连博士学位还没拿到的毛头小伙——后来,胡适在他的纪念文章里曾提到,如果没有蔡元培,他的一生很可能会在一家二三流的报刊编辑生涯中度过。‎ ‎⑥引进陈独秀与胡适,是蔡元培“兼容并包,思想自由”的著名八字方针下的两段佳话。蔡元培决心以这八个字来塑造北大。这是他在欧洲留学期间就已埋下的心愿。在这种精神指导下,北大吸引了中国的各路学术精英。大师云集,各种文化社团风起云涌。而那种“师生间问难质疑,坐而论道的学风”,那种民主自由的风气,从那时开始形成,成为北大异于其他大学、吸引后来一代又一代学子的独特传统。‎ ‎⑦实行教授治校,也是蔡元培在国外留学期间感悟到的。在蔡元培任北大校长期间建立起教授会、评议会,各科学长由教授会公举等举措,都是对德国大学管理方法的仿效。据时任北大教授的沈尹默的回忆,当时的评议会,由全体教授推举,约五人中选一人。凡校中章程规律,都要经评议会同意。‎ ‎⑧讲到蔡元培与北京大学,就必然讲到“五四运动”。在蔡元培的支持下,以北京大学为中心的波澜壮阔的新文化运动,为“五四运动”孕育了丰厚的文化背景,而“五四运动”‎ 本身,也直接与蔡元培相关。1919年5月4日。在游行队伍从红楼出发之前,蔡元培曾在出口处挡了一下。他表示大家有什么要求,他可以代表同学们向政府提出来。但在激愤之下,学生们不肯。‎ ‎⑨1919年5月4日当晚,蔡元培对学生们说:“你们今天所做的事情我全知道了,我寄以相当的同情。”话一出,全场欢声雷动。蔡又说:“我是全校之主,我自当尽营救学生之责……我保证在三天之内,把被捕同学营救回来。”‎ ‎⑩在社会的强大舆论压力下,被捕学生于5月7日被保释出狱,而蔡元培却在5月8日向政府提出辞呈。第二天,就悄然去了天津,后来回了浙江老家。‎ ‎⑪1923年之后的蔡元培,人或在海外,或在国内办大学院和中央研究院等,已不再管理北大校务,但是,北大却从此确立了现代传统和校格,而蔡元培也因北大成为伟大的教育家。‎ ‎⑫毛泽东曾赞誉蔡元培先生为“学界泰斗,人世楷模”。‎ ‎(重庆《新华日报》1940年3月8日)‎ 依据本文内容,分析本文哪些方面体现了蔡元培的“学界泰斗,人世楷模”的特征,请简要论述。‎ 答:_________________________________________________‎ 答案 (1)他有高尚的道德情操。如他不做官,也要求学生们不做官;愤然辞职,不与腐朽的统治者同流合污等。(2)他尊重学术,尊重人才,胸襟博大。如引进陈独秀与胡适等不同风格的人才。(3)他热爱学生。如想方设法营救被捕的学生,对学生一片赤子之心等。(4)他博学,学贯中西,善于借他山之石,来塑造北大。‎ 解析 本题考查概括传主的精神品质的能力。我们可以从主要事件中概括:从他不做官,也要求学生们不做官,愤然辞职,不与腐朽的统治者同流合污等事件看出“他有人格魅力,有高尚的道德情操”;从他引进陈独秀与胡适等不同风格的人才看出他“尊重学术,尊重人才”,写引进陈独秀时,通过对蔡元培细节性的行为描写,如“他去陈的住处拜访陈时,因陈习惯晚睡晚起,他就耐心地坐在门口的一只小板凳上,等待着年龄小他一轮的陈独秀醒来”,表现了蔡元培求贤若渴的诚意;从他想方设法营救被捕的学生,对学生一片赤子之心等看出他“热爱学生,对学生负责任”;从实行教授治校,看出他“善于借他山之石,来塑造北大”。‎ 三、阅读下面的文字,完成1~2题。‎ 百年杨绛亦芳华 ‎100年前,诞生了多位大师级人物。作为其中唯一的女性,杨绛先生以其博学、才情、恬淡、低调被称为是“这凡俗人间的一丝光亮”。杨绛先生的曾祖父、祖父秉性正直,父亲杨荫杭是晚清留日学生,曾做过胡适的老师。杨绛5岁开蒙,1919年其父辞职南归。1928年夏,杨绛考入苏州的东吴大学,是班上的“笔杆子”,中英文俱佳。后来她考上了清华研究院。‎ 当时清华大学研究院还鼓励研究生跨系选修课程。杨绛出于文学创作的需要,选修了中文系的写作课,授课老师是朱自清教授,杨绛的文学创作是从朱自清的课上开始的。写于1933年的《收脚印》是杨绛的处女作,这篇作品收录于1994年出版的《杨绛散文》集中,杨绛在其《附记》中写道:“这是我在朱自清先生班上的第一篇课卷,承朱先生称许,送给《大公报·文艺副刊》,成为我第一篇发表的写作。留志感念。”‎ ‎1935年,新婚的杨绛随丈夫钱钟书远赴英国,到牛津求学。抗日战争全面爆发之后,杨先生和钱先生“为国为家,都十分着急”,1938年回国。钱先生只身赴昆明西南联大任教,杨绛则带着女儿回到上海,住在被日军包围“孤岛”一样上海的租界里。功底深厚,出手不凡,创作的第一部话剧《称心如意》于1943年春天正式公演。随着《称心如意》的成功,杨绛一鼓作气接连创作了喜剧《弄真成假》《游戏人间》,悲剧《风絮》。‎ ‎1949年上海解放前夕,杨绛、钱钟书和许多爱国的知识分子一样,拒绝了国民党的拉拢,不去台湾。当年5月上海解放,杨绛、钱钟书已接到清华大学的聘函,任清华大学外文系教授。‎ 之后便经历了种种斗争,历尽劫难。值得一提的是,即便动荡年月,杨绛也没有放弃学术研究,通晓英、法两国语言的杨绛,近60岁时,从零开始学习西班牙语,并翻译了《堂·吉诃德》。‎ 从《洗澡》《干校六记》,到2003年出版回忆一家三口数十年风雨生活的《我们仨》,96岁成书《走到人生边上》,杨绛的作品日益受到读者的喜爱。而历经百年沧桑,杨先生的近照却依然如三联书店出的那套《钱钟书集》中夫妇二人的合照一般素雅恬淡。‎ 杨绛与钱钟书相识在1932年春天,丁香、紫藤盛开的清华园古月堂前。刚入读清华大学研究院外国文学专业的杨绛虽然个头不高,但面容白皙清秀,身材窈窕,性格温婉和蔼,人又聪明大方,自然深受男生的爱慕。很多当时清华的人都说:“杨绛进入清华大学时,才貌冠群芳,男生欲求之当偶者70余人,谑者戏称为七十二煞。”‎ 或许是天意,她在等待一个人——钱钟书。‎ 据杨绛回忆,初见钱钟书时,他穿着一件青布大褂,一双毛布底鞋,戴一副老式大眼镜,机智幽默,侃侃而谈。两人一见如故,发现相互之间有着说不完的话题。文学上的共同爱好和追求,性格上的互相吸引,心灵的默契交融,这一切使他们一见钟情。钱钟书与杨绛相爱了,并从此相携走过风风雨雨的一生。‎ 杨绛甘做“灶下婢”,辅佐夫君全力创作《围城》,是20世纪40年代文坛的一则佳话。“女佣因家乡生活好转要回去。我不勉强她,也不另觅女佣,只把她的工作自己兼任了。劈柴生火烧饭洗衣等等我是外行,经常给煤烟染成花脸,或熏得满眼是泪,或给滚油烫出泡来,或切破手指。可是我急切要看钟书写《围城》(他已把题目和主要内容和我讲过),做灶下婢也心甘情愿。”‎ ‎《围城》是1944年动笔,1946年完成的。钱钟书在《围城》序言中说:“这本书整整写了两年。两年里忧世伤生,屡想中止。由于杨绛女士不断的督促,替我挡了许多事,省出时间来,得以锱铢积累地写完。照例这本书该献给她。”‎ 经历了这样的姻缘,杨绛对现代人的婚姻有着自己的看法:“我是一位老人,净说些老话。对于时代,我是落伍者,没有什么良言贡献给现代婚姻。只是在物质至上的时代潮流下,想提醒年轻的朋友,男女结合最最重要的是感情,双方互相理解的程度,理解深才能互相欣赏吸引、支持和鼓励,两情相悦。我以为,夫妻间最重要的是朋友关系,即使不能做知心的朋友,也该是能做得伴侣的朋友或互相尊重的伴侣。门当户对及其他,并不重要。”‎ 杨绛先生富有“老派”中国知识分子的学术精神和人格魅力,在历经风风雨雨后,她仍然温柔敦厚、哀而不伤。‎ ‎“我觉得在艰难忧患中最能依恃的品质,是肯吃苦。因为艰苦孕育智慧;没有经过艰难困苦,不知道人生的道路多么坎坷。有了亲身经验,才能变得聪明能干。”走过一个世纪沧桑的杨绛近日在回答记者提问时说:“我的向上之气来自信仰,对文化的信仰,对人性的信赖。总之,有信念,就像老百姓说的:有念想。”‎ 在许多朋友眼里,杨绛生活异常俭朴、为人低调。她的寓所没有进行过任何装修,水泥地面,非常过时的柜子、桌子,老旧的样式,始终安之若素,室内没有昂贵的摆设,只是浓浓的书卷气。杨绛说:“我家没有书房,只有一间起居室兼工作室,也充客厅,但每间屋子里有书柜,有书桌,所以随处都是书房。”‎ 如今,杨绛已是期颐之年,却仍笔耕不辍,每天伏案工作。杨绛一直非常低调,有一年新著出版,出版社有意请她“出山”,召开作品研讨会,宣传一下作品。对此,杨绛谢绝出席。‎ 闭门谢客的杨绛过着一个普通老人的生活,耳朵有些背,视力也下降了。她曾对记者说:“最大的问题就是打扰特别多,尤其是电话太多,我真担心自己的时间是不是就这样被消耗掉。还是请大家给我留些时间吧,那样我写些文章出来,大家看到后就权当写给大家的一封信吧。”‎ ‎(有删改)‎ 相关链接 ‎①杨绛,江苏无锡人,中国著名的作家,戏剧家、翻译家。‎ ‎②她坚忍于知识分子的良知与操守,她坚贞于伟大女性的关怀与慈爱,她固守于中国传统文化的淡泊与坚韧,杨绛的内心是坚硬的,又是柔软的。(搜狐网)‎ ‎③“最贤的妻,最才的女”,“赠予杨季康(杨绛本名),绝无仅有的结合了各不相容的三者:妻子、情人、朋友”。(钱钟书评)‎ ‎1.结合全文,请概括杨绛先生具有哪些优秀品质,并简要分析。‎ 答:_________________________________________________‎ 答案 ①博学有才,文学功底浑厚。出身书香门第,家庭熏陶,名师指教;5岁启蒙,不断上进求学;出手不凡,发表多篇极有影响的作品,如《收脚印》《称心如意》《走在人生边缘上》等。②淡泊功名,生活低调俭朴。婚姻只求相知,不求富贵;拒绝出席作品研讨会,为自己吹嘘;闭门谢客,包括记者采访;生活简单朴素,只愿过普通人的生活。③意志坚强、感情忠贞,有强烈的爱国情感,富有“老派”‎ 中国知识分子的学术精神和人格魅力。抗战全面爆发,毅然回国;拒绝国民党的拉拢,留在大陆;历尽风雨,仍然温柔敦厚、哀而不伤。期颐之年,仍笔耕不辍。(答到了爱情方面,酌情给分)‎ 解析 此题考查对传主性格特点的分析,注意抓住传主的典型事例分析,分条回答。此题可以从杨绛的生活为人、工作成就、爱国情感等方面结合文本分析,概括为以下几点:博学有才,文学功底浑厚;淡泊功名,生活低调俭朴;意志坚强、感情忠贞,具有强烈的爱国情感,富有“老派”中国知识分子的学术精神和人格魅力。然后再逐条分析。‎ ‎2.杨绛先生说她的“向上之气来自信仰”,具体表现在哪些方面,并简要分析。‎ 答:_________________________________________________‎ 答案 ①在知识、文化、学术(创作)上,孜孜不倦地追求。早年时期,不断求学,甚至到外国求学;抗战时期,仍然不断创作;动荡时期,仍学习西班牙语,翻译《堂·吉诃德》;晚年时期,仍然笔耕不辍。②爱国品质和乐观态度。抗战时期,先国家之急,后家庭生活,夫妻分居两地,为国家作贡献;动荡时期,历尽劫难,哀而不伤。③生活上,能吃苦,低调俭朴。包下所有家务劳动,她的寓所简陋,家具陈旧,只愿过平静的、普通人的生活。(只讲实例,不给分)‎ 解析 此题属于概括内容要点题,筛选杨绛的典型事件进行分析,回答时要先概括再结合文中事件进行具体分析。比如:①早年时期,不断求学,甚至到外国求学;抗战时期,仍然不断创作;动荡时期,仍学习西班牙语,翻译《堂·吉诃德》;晚年时期,仍然笔耕不辍。可以说这是一个在知识、文化、学术(创作)上,孜孜不倦地追求的人。②抗战时期,以国家为先,面对命运上的困难挫折,不会低沉。③她的家居简陋陈旧,生活上,能吃苦,低调俭朴。‎ 四、阅读下面的文字,完成问题。‎ 岭南画派第二代大师司徒奇 刘丽琴 司徒奇出身于广东开平的一个书香世家,父亲司徒枚是知名诗人,有“开平才子”之称。司徒枚设馆授徒,以诗、书、画教授儿孙,司徒奇幼年时曾取其父所藏书画临摹。他16岁考入广州市市立美术学校西洋画科,两年后转入上海中华艺术大学,所作油画《艺人之妻》以第一名入选当时第一次全国美术展览会,名声渐露。1929年,他回广州主办烈风美术学校等,黎雄才曾在此期间向司徒奇学习素描。‎ 司徒奇与关山月、黎雄才一起被称为“春睡三老”,并有《春睡三友》作品集行于世,与赵少昂、黎雄才、关山月、杨善深并称“岭南五老”。民国时期广东“新”“旧”画派之论争,在报刊上长年“笔战”,影响巨大。司徒奇虽习西洋画,但对国画饶有兴趣,他认为艺术应往前看,中国画应不断革新,遂提笔投稿连写48天,声援“岭南画派”的锐意创新。高剑父对其才华、勇气倍加欣赏,邀他加入春睡画院,攻习国画,并认为他“学有根底,才气横溢,如能致力于国画,定能成名,自开家派”。司徒奇就这样“带艺投师”高剑父。‎ 司徒奇跟高剑父的其他学生不同,艺术上面的领域更宽。他原来学西画,曾经还创办过烈风美术学校,那么从西画切入到中国画,艺术上的宽度优于他人;在写实方面,其造型能力比直接学国画的强。另外,在日本侵占广州以后,高剑父去了澳门躲避战火,在澳门他提出过“新文人画派”。‎ 而在澳门这段时间,司徒奇也一直跟随高剑父,是第一个实践、倡导“新文人画”的“岭南画派”弟子。司徒乃钟回忆父亲生前曾讲过与高剑父在澳门期间的一次深谈,他问高剑父:“所有画派都有兴衰起落,‘岭南画派’未来会是什么派?”高剑父沉思很久后答道:“是‘新文人画派’。”这次谈话直接影响了司徒奇后半生的艺术探索,此后他学取徐熙、宋光宝、居巢、居廉、高剑父等绘画大师之法而扩展,兼及对石涛、八大山人等历代文人画的研习,终成花鸟画一派。在画展中便不乏司徒奇后期那些大写意的老辣笔墨,而隐藏在其作品背后的中国传统文化,父辈引用的诗歌、家学素养和文化底蕴,都对他的艺术推波助澜,“如果说‘岭南画派’是画家的画,而我父亲不同于关、黎,他偏重文人写意,继承了‘新文人画’这个概念,而且在20世纪60年代已经写得很灿烂了。”司徒乃钟说。‎ 如今谈到“岭南画派”,人们往往以“二高一陈”为中心,以“关黎”并举,业界对司徒奇也缺乏关注。对此,司徒乃钟很平和地解释:“新中国成立后,对艺术家的宣传,内地和港澳各有不同,以前很封闭,现在才融汇在了一起,所以没有听过我爸爸也是很自然的事情。他们每个人的画都是自己的自画像,是每个人的际遇。”‎ 司徒奇别号“苍城”,司徒乃钟介绍了这个别号的由来:苍城原为广东开平一个小镇的名称,曾做过开平的县城,而司徒奇也在苍城小学担任过校长职务,所以别人尊称他为“苍城先生”。他觉得很好,以“苍城”为别号,后来更成立了苍城画院、苍城画会。‎ 自20世纪50年代起,司徒奇长期生活在港澳,一生都担任美术教职,培育英才。后来移民加拿大,仍孜孜不倦地宣扬中国文化和艺术,七八十岁高龄仍到处写生,用传统笔墨写异国山水。司徒乃钟说:“每逢下雪,别人都是开车回家,而我们则欢呼雀跃,去写生啰!加拿大的雪景、落基山的风景、北美印第安人的村落、雪山、冰河,皆能入画。到了晚年,父亲的画很淡雅,没有火气,以无法胜有法。”‎ ‎(选自《广州日报》,有删改)‎ ‎1.下列对材料有关内容的理解和分析,不符合原文意思的一项是(  )‎ A.司徒奇习西洋画时,以油画《艺人之妻》开始名声渐露。拜高剑父为师后,他从西画切入到中国画,艺术上的宽度优于他人;在写实方面,其造型能力比直接学国画的强。‎ B.接触“新文人画”后,司徒奇偏重文人写意,在大写意的老辣笔墨背后,隐藏着中国传统文化,终成花鸟画一派。‎ C.“每个人的画都是自己的自画像”是指每位画家都会在绘画中倾尽自己的情感和心血;“每个人的际遇”指的是不同的人有不同的经历。‎ D.绘画是一种艺术,在追求探索的过程中,每一位画家都会形成自己独特的风格。他们的作品,会因为这种风格而成为自己的标志,表现个人发展的历程。‎ 答案 C 解析 “每个人的画都是自己的自画像”,是指每位画家都会通过自己的画展示个人独特的风格。‎ ‎2.下列对传记有关内容的分析和概括,最恰当的两项是(  )‎ A.司徒奇出身于书香世家,有“开平才子”之称。他幼年时曾取其父所藏书画临摹,因作品《艺人之妻》名声渐露。‎ B.司徒奇在绘画界很有名气,是“春睡三老”之一,也是“岭南五老”之一,曾参与“新”“旧”画派之论争。‎ C.司徒奇后半生在艺术上不断探索,学取各位绘画大师之法而扩展,并对历代文人画进行研习,终成花鸟画一派。‎ D.在司徒乃钟看来,人们对司徒奇缺乏关注,是因为新中国成立后,内地比较封闭,对艺术家的宣传不如港澳。‎ E.司徒奇别号“苍城”,是因为他曾在苍城小学担任过校长职务,后来他更在苍城成立了苍城画院、苍城画会。‎ 答案 BC 解析 A项,张冠李戴,有“开平才子”之称的是他的父亲司徒枚。D项,“对艺术家的宣传不如港澳”理解不准确,文中说“对艺术家的宣传,内地和港澳各有不同”。E项,司徒奇“成立了苍城画院、苍城画会”是在他以“苍城”为别号之后。‎ ‎3.文章多处通过司徒乃钟之口来介绍其父司徒奇,这样写有什么好处?‎ 答:_________________________________________________‎ 答案 ①司徒奇的作品有深厚的文化底蕴,这一点,从他的儿子司徒乃钟做了解更全面,也更深刻。②从司徒乃钟的角度来写司徒奇,他谈到的每一件事情、每一个问题,都能够以真实可感的情境表现出来。③这样写,可以增加文章的纪实性,充分体现传记类作品的特点。‎ 解析 文章介绍司徒奇,却多处从他的儿子的角度安排写作内容,这样,整个文章就显得真实可感,纪实性更强,也符合传记类文本的特点。‎ 考点二十八 探究文本所反映的人生价值和时代精神 考点名片 考点内容 相对于文学类文本阅读,实用类文本阅读侧重于探讨文本反映的人生价值和时代精神,探究文本中的疑点和难点,并提出自己的见解。“文本反映的人生价值”‎ 应该包括传主本身体现出来的和传者的情感态度所反映出来的人生价值。“文本反映出来的时代精神”则是指文本体现出来的它所产生的那个时代的精神风貌,另一方面也包括文本本身与现在社会的精神之间的相通之处。‎ 考查形式 设问方式:①对此你认为×××有没有道理?请谈谈你的看法。②×××给你什么样的启示?请结合全文,谈谈你的看法。③在×××看来××,请结合全文和社会现实,谈谈你的看法。④×××在现在有何意义?请联系现实谈谈自己的看法。‎ 趋势分析 题型相对稳定,全国卷仍将采用表述题的命题方式。‎ 一、阅读下面的文字,完成1~2题。‎ 徐光宪:他的世界“祖国最大”‎ 王庆环 ‎5月6日,“稀土之父”徐光宪遗体告别仪式在北京八宝山殡仪馆东礼堂举行。‎ 他是我国著名化学家和教育家,是国家最高科学技术奖获得者、中国科学院院士、中国稀土化学的奠基人之一……斯人已逝,精神长存。告别的时刻,感念先生作为一代科学巨匠走过的一生,人们发现,他留给科学世界的宝贵财富是如此厚重与绵长。‎ ‎“科学没有国界,但是科学家有祖国。”正是这一信念,支持徐光宪在朝鲜战争爆发后,放弃留在美国的优厚待遇和科研前景,放弃妻子高小霞一两年后可能得到的博士学位,毅然回国。‎ 回顾徐光宪先生的一生,爱国,不仅是他个人崇高的情感,更成为他科学工作的最高准绳。他常说“科学研究应该时刻关注国家目标”,国家需要你改变方向时,科学家应该服从国家需要。他曾在科学研究中四次改变科研方向,先是从量子化学转到络合物化学,再转入核燃料萃取方向,接着是稀土分离,最后又回到量子化学方向。‎ 这种情怀,让徐光宪对基础研究和应用研究之间的关系有着深刻的理解和把握。一方面,他强调要思考纯科学研究如何衍生出应用价值,考虑其远期的应用背景,使基础研究的成果尽可能应用到实际中去,产生社会和经济效益。另一方面,他又特别强调基础研究,他常说,没有基础研究,引进国外的技术需要付出很大的代价,而且自己的基础研究不行,就没能力消化,更谈不上创新。“他这么做,完全站在‘祖国最大’的高度上,非常了不起,所以我们看他,觉得他就是黄河,就是泰山,就是北斗。”北京大学党委书记朱善璐说。‎ 徐光宪生前经常讲他年轻时一段“塞翁失马”的经历。1939年夏,徐光宪从宁波高级工业职业学校土木科毕业,有机会获聘成为昆明铁路局的铁路练习工程员,铁路局派来一个人,把徐光宪及其他7名同学带到上海,并安排他们住进一家小旅馆,谁知这个人却带着8个人的路费趁着兵荒马乱跑了。没办法,徐光宪只能投奔在上海当初中教师的哥哥,得以有机会备考上海交大。每次回忆这段往事,他都会感慨“塞翁失马,焉知非福”。‎ 从生活小事中也能看出哲学意义的徐光宪,在科学道路上有着非常宽广的学术视域,十分关注化学学科的基本问题,对科学史、传统文化均有所思考,其中哲学思考更是贯穿其一生的科研工作。‎ 徐光宪是新中国成立后最早对化学哲学相关问题进行讨论的自然科学家之一,结合马克思主义哲学和恩格斯的自然辩证法等经典理论,他在20世纪50年代对“物质和运动”进行了考察,其成果属于中国化学哲学最早问世的一批成果。‎ 科学发展史也是徐光宪感兴趣的领域,他善于从科学史的重大发现中来获取科学研究中的科学方法,他的许多分类学与方法学思想即从科学史当中获取灵感。他的报告,让人有醍醐灌顶之感。‎ 徐光宪自美国留学回国后,一直在北京大学工作,既从事科学研究,也从事教学工作,两项工作都做得十分成功。他常说,作为教师,讲课比天大。徐光宪上课从不迟到,他曾说:“我从学生的脸上,看到他们对我的讲课是满意的,我就感到很高兴,很幸福。”‎ 就像他做科学研究一样,他的教育教学工作从来都不“单纯”。‎ 正是因为意识到逻辑思维训练与科学创新的重要性,他编写的讲义与教材,都特别注重习题练习与训练,课前五分钟的习题测验,是他上课的特色。他希望学生做完习题,能够在潜移默化中学到科学的思想方法和研究方法,用学到的知识分析和解决问题。他的课程一般都是与科研需要结合起来的,在教学中引入自己研究的新成果,介绍研究领域所需的基本理论和知识,并及时反映国际学术界的最新研究成果。‎ 徐光宪非常关心青少年的科学教育,1999年8月北京青少年科技俱乐部初建时,他是俱乐部第一批学术指导中心导师。“那时徐老年近八旬,亲自做讲座、答疑,让我们尽情享受了科学的魅力与活力,那场景,仿佛就在昨天……”已经毕业多年的俱乐部老会员闻讯来到灵堂祭奠,并留下了这样的文字。‎ ‎“我对老师最突出的感受是他的为人,他特别乐于助人。物质上,捐钱捐物的事情做得真不少,帮助困难职工和学生;精神上,凡是向他求教的人,无论是不是他的学生,他都热心帮助。他对年轻人特别好,担负责任,给予帮助,让年轻人放手去干,成长起来。还有,他无论做什么都坚持不懈,遇到困难用几年的时间去解决也不怕。”黎乐民院士说。‎ ‎(摘自新华网,有删改)‎ 相关链接 ‎①在20多年前,国外稀土行业的跨国企业,被迫经历过一次巨变,他们称这次巨变为“中国冲击”。中国终于实现了由稀土资源大国向稀土生产大国、稀土出口大国的转变。为此,徐光宪被称作“稀土界的袁隆平”。‎ ‎(余玮《徐光宪:举重若重的“稀土之父”》)‎ ‎②“科学家中有两种人,一种是‘工匠’,还有一种是‘大师’。前者的目光局限在具体的研究中,而后者则研究科学的哲学层面。徐光宪就是后者的境界。”徐光宪的学生严纯华院士说。‎ ‎(马荣真、彭广舟《“中国稀土之父”徐光宪 逝世:一生“扎根”稀土》)‎ ‎1.徐光宪被称为“稀土之父”,可是文中并没有突出他在稀土方面的贡献,原因是什么?‎ 答:________________________________________________‎ 答案 ①徐光宪多次改变研究方向,在每个领域都有杰出成就,稀土方面的成就只是其中之一,不必单独介绍。②从文章标题中的“他的世界‘祖国最大’”可以看出,文章的主要内容是讲徐光宪的精神世界的,没有必要突出他在稀土方面的成就。③徐光宪在稀土方面的成就巨大,其他报道已很多,不必细说。‎ 解析 本题考查理解文章内容的能力。解答本题,首先要从文中找传主在研究方面的内容。文中说徐光宪多次改变研究方向,可知在稀土方面的成就只是他成就的一部分,不必特意突出,由此可得出第一点答案。从作者的写作意图的角度分析,主要侧重于他的精神世界,可得出第二点答案。徐光宪既然被称为“稀土之父”,则其成就早已尽人皆知,由此可得出第三点答案。‎ ‎2.徐光宪“留给科学世界的宝贵财富是如此厚重与绵长”,他留下了哪些宝贵财富?请结合文本谈谈你的看法。‎ 答:________________________________________________‎ 答案 ①以祖国需要为研究方向的爱国精神。他四次改变科研方向,都是因为国家的需要。②在科学研究中引入哲学思考的思想。哲学思考贯穿其一生的科研工作,他在科学道路上有着非常宽广的学术视域。③注重培养学生科学的思想方法和研究方法的经验。他编写的教材和讲义,注重训练学生的逻辑思维和创新精神。④良好的科学家风范。他乐于助人,勇于负责,坚持不懈,这些品格都影响着同事与学生。‎ 解析 本题考查探究能力。本题题干中的关键词是“宝贵财富”,从原文第二段的“斯人已逝,精神长存”和文章主要内容可知,这里的“宝贵财富”主要是指人物精神方面的财富。从前到后逐一梳理文章内容,筛选出相关信息,从中提炼出答案要点即可。‎ 二、阅读下面的文字,完成问题。‎ 克罗齐的求索 事业成功的种子往往播撒在苦难的土壤之中。1883年,卡萨米西奥拉大地震夺去了贝尼代托·克罗齐的双亲和妹妹的生命,被埋在废墟下的他侥幸得救。当时他17岁,温馨的生活在暴虐的自然面前顷刻灰飞烟灭。这场灾难给青年时代的克罗齐造成了巨大的心理创伤,也使他在人生之旅和学术之途上更为关注自我心灵的充实和人类社会的精神景况。‎ ‎1886年克罗齐辍学,开始自主学习和独立研究。他一度潜心于他的家乡那不勒斯以及欧洲的历史研究,期望能在喜欢的学术事业中求得情感的解脱。期间,他结合自己从未放弃的美学志趣,把历史与艺术联系起来进行思考。他发现,历史与艺术都以人为焦点,着力于表现独特而个别的事实,不关心抽象概念或一般法则。艺术不是世俗理解中的娱乐手段,它创造的是一个可能的事实;历史则提供实有的真实,表现为真实发生过的一切。‎ ‎1902年,克罗齐出版了影响深远的《美学》,为现代美学做出了开拓性贡献。中国美学家朱光潜在《文艺心理学》中对克罗齐的美学思想曾作过专章介绍。意大利哲学家金蒂雷赞赏克罗齐不仅发现了想象的规律,而且引入了心灵发展的概念。在共同的学术兴趣和理论探讨中,克罗齐和金蒂雷的友谊也由此得到发展。在《美学》中,克罗齐重点对过去时代的“错误的美学”,尤其是“自然主义美学”和“唯理主义美学”进行批驳,肯定了艺术的“心灵性”和“非逻辑性”等特点,把艺术和美学从对道德与理性的依附中解放出来。‎ 然而,学术上的创获并未使克罗齐摆脱椎心的痛苦和深刻的孤独。在潜意识里,他渴望公众对他的工作有所关注。1903年他开始创办涉及文学、历史与哲学的文化刊物——《批评》。克罗齐在《自传》中曾说:“创办《批评》杂志标志着我的生活中一个新时期的开始。”他主要负责文学与历史,前期曾邀请金蒂雷负责哲学。在长达42年的办刊过程中,克罗齐渐渐达到了他所期待的理论家和实践者之间协调统一的理想。1920年6月至1921年7月,克罗齐担任意大利教育部长,发起了意大利现代史上著名的教育改革运动。‎ 在进行社会活动的同时,克罗齐不断修正完善他的理论学说。他在一次交流活动中说:“正如每一个对自己所做事情感到要负责的人一样,我喜欢反复思索我写过的并发表过的那些东西,喜欢经常回味和审查它们,以求看一看是否可以发现一些不妥之处,是否可以进一步加以展开……人不是生来就什么都会的。这就是说,他所懂得的事情只是慢慢才学会的;似乎还应当加上一句——人也不是死后才会的。因此,人往往要像前几年那不勒斯的一位外科医生所做的那样,他也是我在参议院的一位同事,有一次做手术时他突然感到不舒服,觉得自己要死了,于是把手术刀交给一位同行,说请你来干下去。”‎ 克罗齐不仅勤奋钻研,而且注重研究方法。方法是抵达学术高原的重要途径和手段。在办刊前期,克罗齐完成了“心灵哲学”的系统阐发工作。在他看来,历史是人的创造,体现为人的活动,而人的活动要受心灵的支配,因此把握历史的关键在于把握心灵活动。哲学不过是历史学的方法论,追求美、真、益、善的心灵哲学,作为对心灵活动形式与规律的研究,其意义在于为创造历史和阐释历史提供方法论的依据。‎ 在克罗齐心中,意大利民族始终具有美丽的形象。它从遥远的古罗马走来,创造过文艺复兴的灿烂文化。但20世纪20年代,意大利的历史脚步踏上“迷途”。身处残酷的历史情境,克罗齐怀着对心灵自由和精神解放的执着坚守,坚持以著述和主编的刊物抵抗法西斯专制主义。1923年,金蒂雷加入了法西斯党。克罗齐以巨大的道义勇气,与宣扬极权主义理论的金蒂雷展开针锋相对的公开论战,彼此持续几十年的友谊宣告破裂。1925年,克罗齐发表了著名的《反法西斯知识分子宣言》,反对恐怖政策,并征集到数百名知识分子的签名。他的寓所在1926年几次遭到法西斯党徒的洗劫。1944年4月,金蒂雷被击毙于佛罗伦萨街头,克罗齐闻讯,曾长久地倚在寓所的门框上,遥望北方深邃的蓝天……‎ ‎“我将死于工作。”这是晚年的克罗齐在他人问及自己健康状况时一句平静的回答。创造性的努力、自由的激情以及深刻的公民责任感,既是意大利民族性格的基本素质,亦折射出克罗齐的人格理想。‎ ‎(摘自张敏《克罗齐美学论稿》,有删改)‎ ‎“人不是生来就什么都会的。”克罗齐在其后又加上一句:“人也不是死后才会的。”如何理解克罗齐这一拓展的内涵?请结合原文简析。‎ 答:________________________________________________‎ 答案 克罗齐的拓展不仅强调人获取知识靠后天的学习,而且强调了这种学习的紧迫性,学习、追求要贯穿生命整个过程,体现了克罗齐勤勉探究、不断自我完善、积极进取的精神品质。文中克罗齐从青年开始自主学习,持之以恒,珍惜在世时光,提升生命质量,用自己的一生实现了对这句话的拓展。(答出拓展的内涵,结合原文分析,意思答对即可)‎ 解析 考查评价文本的主要观点和基本倾向。“人不是生来就什么都会的”与“人也不是死后才会的”两句话说法貌似不同,实质相同,都强调了后天学习的重要性。文中克罗齐从辍学开始到出版影响深远的《美学》再到后来的从事社会活动等等,无不体现了这一观点。答案结合文中对克罗齐成功的记述即可得出。‎ 三、阅读下面的文字,完成1~2题。‎ 徐乾清:清水徐徐洒乾坤 ‎1月9日上午11点15分,我国著名水利专家、中国工程院院士徐乾清,静静地走了,悄然离开了他牵挂一生的水利事业。‎ 徐乾清的离去,带给人们的是无尽的哀思,而他心系水利,为我国大江大河治理倾注的大量心血,也深深感染着每一位水利工作者。‎ 徐乾清1925年出生于陕西城固,汉中人。汉水自西向东横贯于汉中盆地,建于秦汉的众多水利工程和灌区,真实反映了汉中盆地稻作农业的繁荣景象。‎ ‎1949年9月初,23岁的徐乾清从国立上海交通大学毕业后,被选派到泰州苏北行署水利处,开始了其一生的水利生涯。‎ 此后,从苏北到上海再辗转到北京中央水利部,他当过工程技术人员,担任过专家工作室技术组组长并兼任苏联专家组组长的助手,在水利部科学技术委员会工作过……到后来担任水利电力部计划司副司长、水利部副总工程师。‎ ‎60余年的水利生涯,从长江到黄河,从松花江、辽河到淮河、海河、珠江流域,徐乾清跑遍了祖国的各大江河湖泊,推动参与了全国主要江河流域规划的编制、修订、审查工作,参与了长江三峡、南水北调等重大水利建设项目的论证和审查工作,先后担任“黄河治理”“长江防洪”“西北水资源开发利用和生态环境保护”等国家科研项目的专家组长,为我国大江大河治理倾注了大量心血。‎ ‎20世纪80年代以来,徐乾清几次从宏观角度指出我国水资源短缺给社会经济发展及生态环境保护带来的严重影响,提出了明确的对策。当“八五”攻关即将结束之时,徐乾清又敏锐地将视野投向了水资源短缺、生态脆弱的西北地区,提出要尽快开展西北地区生态需水研究。在他的积极倡导下,“西北水资源合理利用及生态保护研究”被列为国家“九五”科技攻关重点项目。该项目的及时设立和成果产出,为20世纪末我国西部大开发战略的实施提供了重要的科技支撑。‎ 了解徐乾清的人都知道,他一生出席的大小会议无数,每次话都很少,却句句铿锵有力。无论是“赞成”还是“反对”,事先他都会进行充分的论证。“徐院士的科学作风十分严谨,从他嘴里说出来的每句话、每个词都非常准确。他告诫我们,对任何事情,哪怕是一个数据都不能含含糊糊。”中国水利水电科学研究院副院长胡春宏对此深有感触。‎ 徐乾清做人做事求真务实,在全国水利系统是出了名的。徐老总结他一生的治水经验,希望能将其精华传授给后人,而这精华,就是要实事求是,要尊重科学,要遵循客观规律,只有这样,才能真正做好治水安邦的大业。‎ 对年轻学者的请教,徐乾清是从来不吝啬的,但他也有自己的原则。平时,经常有年轻的水利科研人员在评职称、报奖等方面请徐乾清帮助推荐,而他立了三个“门规”:一是被推荐的人及其所做的工作必须是他所熟悉的;二是草拟的材料必须提前送去,他认真看完及时通知对方结果,并注明“可以签”“做修改后再签”“能否请其他专家推荐”等意见;三是凡他签过字的材料,必须给一份复印件留存。‎ ‎“徐老开会发言语气十分平和,且都围绕着指出问题、提出建议展开,从不以专家身份凌驾于别人之上。”同时有两个深切感受:一是“实”,徐乾清向来没有空话和套话;二是“有收获”,言之有物,从不漫无边际。‎ 大家都说,徐乾清一生把名利看得很淡很淡。徐乾清一辈子不要奖,谁给他个荣誉证书,他都要着急。直到2009年9月,被评为“全国离退休干部先进个人”,这也是他平生唯一接受的个人荣誉。‎ ‎“我只能算是一个平庸的水利科技工作者,根本够不上专家的称号。这一生辛勤劳累有余而业绩成效甚微,不能不说是终生遗憾。”徐乾清在日记里这样概括自己的一生,“但我尽了我的一切能力,做了我力所能及的工作,无负农民给我的饭,工人供我的衣和生活用品;一生未做对不起社会和周围同志和亲朋好友的事。大概还算是一个可以问心无愧地度过这一生的普通劳动者。”‎ 斯人远去,魂系水利。徐乾清走了,而他为我国大江大河治理倾注的大量心血,将永留史册。‎ ‎1.常言道“做事先做人”,当代大禹徐乾清是怎样的一个人?请结合材料概括分析。‎ 答:________________________________________________‎ 答案 ①爱国、责任感强。一生献身祖国水利事业,为祖国大江大河治理倾注大量心血。②科学作风严谨。说话从不含糊,遇事及下结论都要事先充分论证。③求真务实勤奋。治水遵循规律,说话从不浮夸。一直奋战在水利一线,考查、实践、记录,为国家水利事业做出巨大贡献。④‎ 讲原则。对年轻学者的请教不吝啬,有原则性,立下三个“门规”。⑤淡泊名利。“全国离退休干部先进个人”,是他平生唯一接受的个人荣誉。⑥平和谦逊。从不以专家身份凌驾于别人之上,说自己只是一个普通劳动者。‎ 解析 解答本题,应根据文本中徐乾清的言行举止及议论性评价性的语句加以概括作答。例如,“一生献身祖国水利事业,为祖国大江大河治理倾注大量心血”“说话数据从不含糊,遇事及下结论都要事先充分论证”表现了徐乾清的爱国,责任感强,科学作风严谨。以此类推,再从求真务实勤奋、讲原则、淡泊名利、谦逊平和等方面分析作答即可。‎ ‎2.徐乾清总结自己的人生“辛勤劳累有余而业绩成效甚微,不能不说是终生遗憾”,请结合材料谈谈这句话给你哪些启示。‎ 答:________________________________________________‎ 答案 这是徐老的谦虚及对自己高标准要求的体现。①我们要像徐老一样一生勤奋,勤奋努力就一定能做出更多的成绩。②对自己所取得的成绩要不知足。徐老深爱自己的祖国和水利事业,希望能为国家做出更多的贡献,所以不满足于已有成绩。③我们要像徐老一样谦逊朴实。徐老已经取得了很大的成绩,却从不居功自傲,80岁高龄依然为祖国的水利事业继续发挥着自己的作用。如果我们能够做到谦虚平和、勤奋刻苦、永不满足,我们的人生也会精彩。(答案应扣住原文,言之有理即可)‎ 解析 这是开放性的探究题,解答本题,应先正确理解“辛勤劳累有余而业绩成效甚微,不能不说是终生遗憾”这句话的含意,根据对文本中徐乾清的精神品质的理解可知,这句话表现出了他谦逊、勤奋、不满足已有成绩的精神品质。然后联系自己的实际,从谦虚、勤奋、不满足于成绩等方面阐述徐乾清的精神品质对我们的启发与影响,只要能自圆其说,言之有理即可。‎ 四、阅读下面的文字,完成问题。‎ 李尚师:修史·铸鉴·报国 邢兆远 李建斌 ‎1942年出生的李尚师,饱历世间沧桑。血与火,痛与苦,烧灼着他的心,更让他悟出一个理:个人与国运血脉相连,只有国家强大,百姓方可安居乐业;只有治国有方,人民才能过上幸福生活。‎ ‎1985年,任中学教师的李尚师的人生出现了转折,当时山西师院的“晋国史研究室”缺少人手,本族叔叔李孟存教授邀他一起编写《晋国史》。而自幼就崇拜“报国修史”的李尚师,此时不禁热血沸腾,下决心步入“漫漫”的修史路,为国家尽一介草民之力。‎ 为搜集史料,李尚师走遍了晋国古地,晋侯墓地、陶寺遗址、尧都、天马——曲村遗址、晋献侯墓的车马坑、晋国铸铜遗址、侯马盟书出土遗址、牛村晋国宫殿遗址等都出现了他的身影。‎ 为考察传说中的晋国大宗之都——翼,李尚师曾住在翼城南梁农家,骑着自行车奔波于翔山之下、浍河之滨,观其山如大鹏翱翔之状,察其遍地灰层、绳纹陶片遗存比比皆是,从而确定“翼”乃南梁故城。‎ 李尚师千方百计地参加各种研讨会,与史学界的朋友交流信息,追踪最新研究成果。北大教授、著名史学家李伯谦感慨地说:“李尚师对文献史料的重视和掌握,是很少有人能够匹敌的。百万字的手写书稿,装满整整一大箱子,仅写一遍、改一遍,再誊抄一遍,工作量可想而知,个中清苦甘甜,唯有自知,他完全可以称得上‘中国当代民间修史第一人’。”‎ 研究晋国史,疑点、难点很多。比如晋国始君叔虞所封的唐究竟在何地?历史就有七说,其中东汉班固、郑玄等认为唐在今太原,后世在今太原晋源区建有晋祠,更使“太原晋阳说”不胫而走,家喻户晓。‎ 面对诸多大相径庭的观点,李尚师的大脑在梳理着一个个信息团,一遍又一遍地翻阅着诸多古籍文献,查找考古资料,实地观察地形,厘清各种说法的偏差,毅然选定“临汾盆地说”,并确定唐国地域覆盖范围。‎ 挑战与创新的激情燃烧着李尚师的心,他不断审视权威史书:《春秋》只有1.8万字,由于只记事件结果,不记事件的经过和背景,过于简略,用语隐晦;《史记》以文学笔法写史,有许多地方“失实”。他将《左传》与《春秋》《国语》等文献中的记载进行梳理、比较,将众多历史事件和人物加以互相印证,厘清了主线脉络和史实。谈及此,李尚师说,修史之人不能人云亦云,要有自己独立之思想,敢于突破禁区,超越前人。‎ ‎2011年,李尚师开始编撰《治国方略史鉴》。他俯瞰五千年漫漫长史,在揭开一层层面纱后发现,源远流长的儒家,提倡“德治”“仁政”“中庸”‎ ‎,使国家和谐,社会安定,然而其以宗法制为核心,讲等级,守旧保守,不易于社会变革,造成社会进步缓慢。法家崇尚法治,树立法律至高无上的权威性,建立一个有法可依、执法必严、违法必究的规范有序的社会。其缺点是,由于手段强硬,滥用之,则容易激化矛盾,甚至造成社会动乱。‎ 在《治国方略史鉴》中,李尚师厘清了中国历史上治国理政思想的主线脉络:尧舜时期始创“明刑弼教”“德主刑辅”,于孔、孟时期发展成熟;与此同时,法家学派在晋国孕育、发展,然后儒、法思想通过晋人卜子夏在晋(魏)发生碰撞、融合,并形成了“儒法兼容”思想。战国末期,荀况在“儒法兼容”思想的基础上,将儒家的“礼制”与法家的“法治”有机融合起来,形成了“礼法合治”“德主刑辅”的思想,转变形成历时两千年的治国思想的主线。‎ ‎(选自《光明日报》,有删改)‎ ‎1.下列对材料有关内容的理解和分析,不符合原文意思的一项是(  )‎ A.李尚师通过研究五千年中国历史,开始编撰《治国方略史鉴》的主线是“礼法合治”“德主刑辅”的思想,核心是宗法制。‎ B.儒家提倡“德治”“仁政”“中庸”,法家崇尚法治,两者各有优点与缺点,在互相碰撞融合的过程中,最终形成了“儒法兼容”的思想体系。‎ C.具体阐述儒家和法家时,通过两种思想的对比,表现出各自的长处与不足,突出了两种思想的本质特征。‎ D.孟世凯对李尚师在学术研究方面取得的成就作整体概括,宫长为则从李尚师的学术地位进行评价,从侧面突出传主李尚师的贡献及其在学术界的地位。‎ 答案 A 解析 “核心是宗法制”不对,以宗法制为核心的是儒家。‎ ‎2.下列对传记有关内容的分析和概括,最恰当的两项是(  )‎ A.饱经世间沧桑的李尚师,在血与火,痛与苦的经历中,体会到个人生活境遇与国家命运的关系,因此走上了漫漫修史路。‎ B.为考察传说中的晋国大宗之都——翼,李尚师进行了大量的实地考察,分析了大量的历史遗存,从而确定“翼”乃是南梁故城。‎ C.修史过程中,李尚师重视文献史料,写一遍,改一遍,再誊抄一遍,形成了百万字的手写书稿,其中艰辛只有自己知道。‎ D.在考查晋国始君叔虞所封的唐究竟在何地时,李尚师推翻了历史上的七说,理清各种偏差,确定了唐国地域覆盖范围。‎ E.李尚师突破禁区,超越前人,将《左传》《春秋》《国语》等中的记载进行梳理、比较,从而理清了主线脉络和史实。‎ 答案 EB 解析 A项,“因此走上了漫漫修史路”说法不严密,这只是他走上这条道路的一个前提。C项,“形成了百万字的手写书稿”错误,是对“百万字的手写稿”“写一遍改一遍再誊抄一遍”。D项“推翻了历史上的七说”,文中无依据,另外文章说,他最终选定的临汾盆地说也应该属于七说之一。‎ ‎3.李尚师被称为“中国当代民间修史第一人”,他的事迹体现了一种怎样的生命情怀?请就此谈谈你的认识。‎ 答:________________________________________________‎ 答案 这是一种对社会、对历史负责任的生命情怀。历史是一面镜子,能够让我们在反思中完善自己,提高自己。李尚师修史,进一步厘清了历史的真实性,让人们更好地认识历史,并在认识历史中总结经验教训,也为当代人治国与修身带来了思想上的启迪。而在修史的过程中,他也实现了自己的人生价值,实现了社会价值与个人价值的统一。(意思对即可)‎ 解析 题目要求对李尚师的生命情怀进行探究。一个人的生命情怀,往往与整个社会联系在一起。具体分析时,可以从社会与个人的角度,对其修史这一行为进行全面、客观的思考,揭示李尚师修史的时代意义,表达对人生价值的深刻思考。‎ ‎ [3年高考真题集训]‎ 一、[2016·全国卷Ⅰ]阅读下面的文字,完成1~4题。‎ 寻找属于自己的句子 ‎1942年夏,陈忠实出生在陕西农村。上中学时,陈忠实读赵树理的《三里湾》和柳青的《创业史》,得到滋养,萌发了文学梦。也许是好事多磨,1962年高中毕业后,他未能如愿上大学读中文系。这个20岁的青年,常常一个人坐在家乡的灞河边,想着文学,想着寻找属于自己的句子。‎ 三年之后,陈忠实的散文《夜过流沙沟》在1965年3月8日的《西安晚报》文艺副刊上发表,他的文学生涯由此正式开始。但直到1979年小说《信任》获得全国优秀短篇小说奖,他才确立了文学上的自信。他感觉自己不再是一个文学爱好者和业余作者了。是年9月25日,他加入中国作家协会。又一个三年之后,陈忠实40岁,他的第一个短篇小说集《乡村》出版,赢得“小柳青”的名声,工作单位也换成陕西省作家协会。他终于是一名专业作家了。‎ 随着年岁的增长和时代的变化,陈忠实越来越觉得要从赵树理、柳青的文学中剥离出来。他将这个愿望写进了小说《蓝袍先生》中。小说写于1985年,一个认知作者的标志性年份。这年的最后10天,他随中国作家代表团出访泰国。第一次走出国门的陈忠实特意置办了一套质地不错的西装。当他第一次穿上西装打上领带站在穿衣镜前的时候,脑海里浮现出刚完成的小说的主人公蓝袍先生。蓝袍先生多年以来一直穿着蓝色长袍,受到同学讥笑以后才脱下蓝袍,换上“列宁装”。陈忠实认为那是摆脱封建残余桎梏、获得精神解放的象征。脱下穿了几十年的中山装、换上西装的那一刻,他切实意识到自己就是蓝袍先生。‎ ‎1985年的泰国之行让陈忠实深受刺激,他联想起家乡人自嘲的称呼。相比那些见多识广的城市人,他们把自己称作“乡棒”。游逛在曼谷的超市大楼,看着五颜六色、各式各样的服装,作家觉得眼花缭乱。那一刻,他觉得不仅自己是“乡棒”,教他观察服装的北京作家郑万隆也是“乡棒”。面对世界,1985年的中国人大都是“乡棒”。他痛感自己需要从什么地方剥离出来,将自己彻底打开,不仅要在生活上打开自己,更重要的是要在思想上打开自己。‎ 在剥离的愿望中,陈忠实认识到必须写一部史诗般的长篇小说,才能在文学上确立自己。这时,各种新近阅读过的长篇小说萦绕心头,作家备感困惑,又备受启发。马尔克斯《百年孤独》的结构像网一样迷幻,王蒙《活动变人形》的结构自然随意,却俨然大手笔,张炜《古船》的结构完全不同,有一种精心设计的刻意……而结构背后似乎还有更深的东西。陈忠实最终发现,不是作家先别出心裁弄出一个新颖骇俗的结构来,而是先要有对人物的深刻体验。寻找到能够充分描写人物独特的生活和生命体验的恰当途径,结构方式自然就出现了。恰巧此时兴起的“文化心理结构”学说给了他决定性的影响。他相信,人的心理结构主要是由理念支撑的,而结构一旦形成,就会决定一个人的思想、道德和行为,决定一个人性格的内核。如果心理结构受到社会冲击,人就将遭遇深层的痛苦,乃至毁灭。陈忠实感到自己终于从信奉多年的“典型性格”说中剥离出来,仿佛悟得天机,茅塞顿开。多年以后,作家回忆往事,认为自己就是在1985年开始重建自我,争取实现对生活的独特发现和独立表述的。‎ 陈忠实后来寻找到了什么是人所共知的,1992年开始在《当代》杂志连载的长篇小说《白鹿原》已经成为我们的文学经典,他在中国当代文坛的位置也随之奠定。此后,功成名就的作家继续在文学的园地里辛勤耕耘,寻找属于自己的句子。‎ ‎2016年春天,陈忠实走了,属于陈忠实的句子永留人间。‎ ‎(摘编自陈忠实《寻找属于自己的句子》、‎ 李清霞《陈忠实年表》等)‎ 相关链接 ‎①‎ 陈忠实的《白鹿原》是上世纪90年代中国长篇小说创作的重要收获之一,能够反映那一时期小说艺术所达到的最高水平。把这部作品放在整个20世纪中国文学的大格局里考量,无论就其思想容量还是就其审美境界而言,都有其独特的、无可取代的地位。即使与当代世界小说创作中的那些著名作品比,《白鹿原》也应该说是独树一帜的。‎ ‎(何西来《关于〈白鹿原〉及其评论》)‎ ‎②陈忠实常讲,创作到了一定阶段,不一定是拼生活,拼艺术,而是拼人格。好一个拼人格!这正是作家自身博大的人格魅力的反映。这就不难理解他最终被公认为描摹巨大民族悲剧的圣手,成为当代中国文学的大家之一。‎ ‎(李满星《陈忠实:回首六十五载风雨人生》)‎ ‎1.下列对材料有关内容的分析和概括,最恰当的两项是(  )‎ A.赵树理《三里湾》和柳青《创业史》是陈忠实最初的文学营养,使他萌发了文学梦,后来则成为他创作上必须突破的对象。‎ B.小说《信任》获得全国优秀短篇小说奖,使陈忠实在文学上确立了自信心。这是他从业余作者走向专业作家的重要转折。‎ C.陈忠实认为“面对世界,1985年的中国人大都是‘乡棒’”,这与其说是他的一种觉悟,不如说是他受刺激后的错误判断。‎ D.陈忠实善于学习前人并感知时代,不仅拼生活、拼艺术,而且拼人格,不断地提升思想境界,获得对人和生命的独特理解。‎ E.从发表第一篇作品到被人称为“小柳青”,再到被人称为“当代中国文学的大家”,陈忠实的整个文学生涯可谓一帆风顺。‎ 答案 DA 答D给3分,答A给2分,答B给1分;答C、E不给分。‎ 解析 B项,张冠李戴,由原文可知,陈忠实从业余作者走向专业作家的重要转折应是他的第一个短篇小说集《乡村》出版,工作单位换成陕西作协。“这是他从业余作者走向专业作家的重要转折”说法不准确,文中说“他感觉自己不再是一个文学爱好者和业余作者了”,这只是他个人的感觉。C项,于文无据,相关信息在原文第4段,文中只是说“1985年的泰国之行让陈忠实深受刺激”,并没有说他做出了“错误判断”;结合文意可知,那应该是他受刺激后的一种觉悟。E项,“陈忠实的整个文学生涯可谓一帆风顺”推断错误,陈忠实在创作生涯中也有困惑的时候,不能绝对地说是“一帆风顺”。‎ ‎2.为什么说1985年是认知陈忠实的标志性年份?请结合材料简要概括。‎ 答:_________________________________________________‎ 答案 ①他意识到要像自己笔下的蓝袍先生一样接受时代的变化,在生活和思想上打开自己;②他认识到必须写出史诗般的长篇小说,才能在文学上确立自己的位置;③他认为自己是在1985年开始重建自我,产生对生活的独特理解和表述的。‎ 解析 首先在原文中找到“1985年是认知陈忠实的标志性年份”这一信息所在之处;然后找到相应的答案所在范围(第3、4、5段);最后筛选出相关信息,并用自己的语言将其整合起来。‎ ‎3.文中认为“属于陈忠实的句子永留人间”,为什么?请结合材料简要分析。‎ 答:_________________________________________________‎ 答案 ①他的小说艺术达到了当时的最高水平;②他的文学作品的思想容量和审美境界在20世纪中国是无可取代的;③他的作品是当代世界文学中独树一帜的文学经典。‎ 解析 认真审题,理解句子。解答本题,应将“属于陈忠实的句子永留人间”拆开来理解:“属于陈忠实的句子”是什么句子?“属于陈忠实的句子”为什么能“永留人间”?结合材料,围绕这两个问题进行简要分析即可,答案要点主要从相关链接中提炼。‎ ‎4.陈忠实的“剥离”和“寻找”是什么关系?有哪些表现?请结合材料详细说明。‎ 答:_________________________________________________‎ 答案 “剥离”和“寻找”是辩证关系。剥离的结果带来寻找的可能,而寻找的冲动激发剥离的愿望。①从赵树理和柳青的文学中剥离,寻找到马尔克斯、王蒙等新的文学营养;②从中山装所代表的时代精神中剥离,寻找到西装所代表的面对世界的契机;③从“典型性格”说中剥离,寻找到“文化心理结构”学说;④从自身已有的文学成就中剥离,寻找到新的文学高度,写出了文学巨著。‎ 解析 第一问要求阐述“剥离”和“寻找”的关系,首先要通读全文,弄清陈忠实要“剥离”什么,即要“从赵树理、柳青的文学中剥离出来”“摆脱封建残余桎梏”“从信奉多年的‘典型性格’‎ 说中剥离出来”;进而弄清他要通过“寻找”获得什么,即“获得精神解放”“在思想上打开自己”“重建自我”。然后分析两者之间的关系,即相辅相成、辩证统一。回答第二问“有哪些表现”,则要纵观全文,从重点段落中筛选出相关内容并加以概括,如第3段的开头句,第4段的结尾句和第5段后半部分及相关链接。要言之有理有据。‎ 二、[2016·全国卷Ⅱ]阅读下面的文字,完成1~4题。‎ 吴文俊的数学世界 吴文俊小学时成绩平平,也没有显示出独特的数学才华,初中时数学甚至得过零分,高中时最喜欢的是物理而非数学,但他从小就对读书有浓厚兴趣,初中时国文成绩一直不错。尽管高三时物理得了满分,但教物理的赵贻经老师却看出了他的数学潜力,力荐他入数学系。正始中学决定,吴文俊必须报考数学系,才能得到每年一百块大洋的奖学金,加之他父母又不放心独子离开上海,吴文俊就进了上海交大数学系。所谓“知之不如好之,好之不如乐之”,吴文俊向来是以兴趣为先导来读书的。因为他对物理有兴趣,甚至一度想要转系。是大三时教数学的武崇林老师帮助他摆脱了专业上的困惑,使他认识到数学的巨大魅力。‎ ‎1940年,吴文俊从交大毕业,先后在育英中学、培真中学担任数学教员,直到1946年见到了影响他一生的恩师陈省身,他才由一个普通的中学数学老师成为数学研究所的专业研究员。对于吴文俊的数学研究,他的学生高小山总结说:“吴先生做拓扑研究,一下子就能抓住核心问题,为代数拓扑学的兴起作出了影响深远的贡献。他从事机器定理证明也是这样,极其敏锐地看出了信息时代数学的发展趋势,他的研究受到中国古代数学的启发,汲取了中国传统数学的养分。使用吴先生的方法,几乎所有数学定理的证明,都可以由计算机来完成,从而让人类把精力放到更加宏观的层面上去思考问题。”‎ 对吴文俊来说,虽然最初选择数学是被动的,但综观其一生,数学已逐渐成为他生命的一部分。从事数学研究,吴文俊特别强调数学思维。他说:“要创新,就要独立思考,就不能总是跟着人家亦步亦趋,当然开始的时候参考借鉴也是必要的。牛顿就说过,他之所以获得成功,因为他站在巨人的肩膀上,才能看得远。所以不能忽略学习,可是除了学习之外,还要能够独立思考,这是创新的必要条件。现在摆在中国面前的是,数学就要靠下一代、下下代在创新方面取得巨大成功,中华民族才可以得到复兴。”吴文俊自己的经历就是很好的例子。他在数学上的一系列成就,特别是他运用机械化思想来考察数学,发现了数学的不同侧面,并建立了新的模式,这全得益于他的独辟蹊径。‎ 对我国的数学基础教育,吴文俊也颇有心得。我国中学生多次在国际奥数竞赛中获奖,被当作我国数学教育成功的证明,但吴文俊更赞同丘成桐的观点:“奥数应该是一种建立在兴趣之上的研究性、高层次学习,中国的奥数学习过分关注海量题目,直接与考试、竞赛挂钩,对系统学习数学不利。作为基础学科,应着重引导学习的兴趣,不应当过分追求功利。”吴文俊同样清醒地认识到:“竞赛获奖固然可贵,但也不能看得过重。因为它不能代表学生对数学的深度理解,也不能有效地训练数学思维。”他认为,数学教育更重要的是培养数学的思维方式。‎ 有人曾揶揄数学家迂腐,吴文俊不但不迂腐,而且兴趣广泛,内心充满童趣。他说:“我是个想怎样就怎样的人,想玩就玩,想工作了就会安安静静地工作,从不多想。”他喜欢看电影、读历史小说,也喜欢看围棋比赛。老伴说他“贪玩”,他却说:“读历史书籍、看历史影片,帮助了我的学术研究;看围棋比赛,更培养了我的全局观念和战略眼光。”‎ 吴文俊37岁时就获得了国家自然科学一等奖,四十多年后,他再次获得国家最高科技奖。如此长的学术生命,在数学界是非常罕见的。当记者提出疑问时,吴文俊反问道:“我为什么不能保持这么长的学术生命?”在他看来,学术生命是能够终生保持的,很多人做不到,那是他们自己的问题,应该自我反省。他特别强调研究数学要下扎实的功夫。他说:“外国许多数学家,尽管有的我非常佩服,可是我并不认同他们靠所谓巧思妙想研究数学的方法。应该根据客观实际具体分析,一切以事实为主。这是我主要的想法。”‎ ‎(摘编自柯琳娟《吴文俊传》)‎ 相关链接 ‎①1974年,吴文俊转向中国数学史研究,从中得到启发,开创了具有中国传统数学特点的数学机械化之路。他提出的“吴方法”,继承和发扬了中国古代数学基于“计算”的传统,与通常基于逻辑的方法根本不同,首次实现了高效的几何定理自动证明。国际机器证明研究领域的权威人物S.穆尔说:“在吴文俊之前,机械化的几何定理证明处于黑暗时期,而吴的工作给整个领域带来光明。”‎ ‎(黄婷、邱德胜《数学大师:华罗庚、陈省身、吴文俊》)‎ ‎②一般说来,吴教授的工作,都是独辟蹊径,不袭前人,富有创造性的。‎ ‎(陈省身为吴文俊颁发杰出科学家奖时的评语)‎ ‎1.下列对材料有关内容的分析和概括,最恰当的两项是(  )‎ A.在上海交大读书期间,吴文俊因为对数学不感兴趣,曾一度想转到物理系,后来遇见一位高明的数学老师武崇林,他才打消了转系念头。‎ B.吴文俊清楚地看到信息时代数学的发展趋势,受到中国古代数学的启发,提出了用计算机实现数学定理证明的方法,作出了影响深远的贡献。‎ C.吴文俊能够清醒地认识到中国数学研究领域存在的主要问题,期待着未来的中国数学家开拓创新,取得巨大成就,从而实现中华民族的复兴。‎ D.外国不少数学家只靠巧思妙想研究数学,尽管名气很大,吴文俊却并不认同他们的研究成果,而是坚持用自己以客观为主的方法研究数学。‎ E.吴文俊在拓扑学、机器定理证明、数学机械化等领域都取得了很多独创性成果,获得了国际数学界同行的高度认可与评价。‎ 答案 EB 答E给3分,答B给2分,答A给1分;答C、D不给分。‎ 解析 A项,“他才打消了转系念头”过于绝对,文中表述的是“是大三时教数学的武崇林老师帮助他摆脱了专业上的困惑,使他认识到数学的巨大魅力”。C项,过分拔高了人物形象。D项,不认同的是“他们靠所谓巧思妙想研究数学的方法”,而不是“不认同他们的研究成果”。‎ ‎2.吴文俊的数学研究为什么能够取得创造性成果?请结合材料简要分析。‎ 答:_________________________________________________‎ 答案 ①不蹈袭前人,不盲从权威,能够独辟蹊径;②具有扎实功底、全局观念和战略眼光,善于抓住事物的本质;③学术视野广阔,注重人文修养。‎ 解析 本题考查筛选、分析信息的能力。可以从吴文俊扎实的数学功底、创新的品质和深厚的人文素养三个角度思考。‎ ‎3.对我国的数学基础教育,吴文俊有哪些心得?请结合材料简要概括。‎ 答:_________________________________________________‎ 答案 ①基础教育应着重引导学生深入学习、探究的兴趣;②数学教育要有利于系统学习和深入理解数学,而不是海量题目训练和追求竞赛获奖;③现行奥数教学方法太功利,且无法引导学生深入理解和训练数学思维。‎ 解析 本题考查整合、概括文中信息的能力。首先,要找到答题区间,文本第4段开头“对我国的数学基础教育,吴文俊也颇有心得”,说明答案就在这一段。其次,认真审读本段内容,划分层次,筛选关键词句,组织答案。‎ ‎4.作为一位杰出的数学家,吴文俊对物理学、文学艺术等也有广泛的兴趣。请结合材料,就兴趣广泛与专业研究的关系进行分析。‎ 答:_________________________________________________‎ 答案 ①吴文俊广泛的阅读面,为日后的专业研究奠定了基础,也有助于科学与人文交融理念的形成;②物理与数学本来就关系密切,吴文俊对物理的兴趣,为他的数学研究提供了便利条件;③吴文俊兴趣广泛,视野开阔,使他的思维活跃,能够融会贯通,富有创造性;④吴文俊富有生活情趣,心胸开阔,能够保持罕见长久的学术生命。‎ 解析 本题考查对文本的探究能力。探究时,要到文中找到吴文俊兴趣广泛的事例,然后分析这些事例与专业研究之间的关系。关键信息散布全文,应认真筛选。‎ 三、[2015·全国卷Ⅰ]阅读下面的文字,完成1~4题。‎ 朱东润自传 ‎1896年我出生在江苏泰兴一个失业店员的家庭,早年生活艰苦,所受的教育也存在着一定的波折。21岁我到梧州担任广西第二中学的外语教师,23岁调任南通师范学校教师。‎ ‎1929年4月间,我到武汉大学担任外语讲师,从此我就成为大学教师。那时武汉大学的文学院长是闻一多教授,他看到中文系的教师实在太复杂,总想来一些变动。用近年的说法,这叫作掺沙子。我的命运是作为沙子而到中文系开课的。‎ 大约是1939年吧,一所内迁的大学的中文系在学年开始,出现了传记研究这一个课,其下注明本年开韩柳文。传记文学也好,韩柳文学也不妨,但是怎么会在传记研究这个总题下面开韩柳文呢?在当时的大学里,出现的怪事不少,可是这一项多少和我的兴趣有关,这就决定了我对于传记文学献身的意图。‎ ‎《四库全书总目》有传记类,指出《晏子春秋》为传之祖,《孔子三朝记》为记之祖,这是三百年前的看法,现在用不上了。有人说《史记》《汉书》为传记之祖,这个也用不上。《史》《汉》有互见法,对于一个人的评价,常常需要通读全书多卷,才能得其大略。可是在传记文学里,一个传主只有一本书,必须在这本书里把对他的评价全部交代。‎ 是不是古人所作的传、行状、神道碑这一类的作品对于近代传记文学的写作有什么帮助呢?也不尽然。古代文人的这类作品,主要是对于死者的歌颂,对于近代传记文学是没有什么用处的。这些作品,毕竟不是传记文学。‎ 除了史家和文人的作品以外,是不是还有值得提出的呢?有的,这便是所谓别传。别传的名称,可能不是作者的自称而是后人认为有别于正史,因此称为“别传”。有些简单一些,也可称为传叙。这类作品写得都很生动,没有那些阿谀奉承之辞,而且是信笔直书,对于传主的错误和缺陷,都是全部奉陈。‎ 是不是可以从国外吸收传记文学的写作方法呢?当然可以,而且有此必要。但是不能没有一个抉择。罗马时代的勃路塔克是最好的了,但是他的时代和我们相去太远,而且他的那部大作,所着重的是相互比较而很少对于传主的刻画,因此我们只能看到一个大略而看不到入情入理的细致的分析。‎ 英国的《约翰逊博士传》是传记文学中的不朽名作,英国人把它推重到极高的地位。这部书的细致是到了一个登峰造极的地位,但是的确也难免有些琐碎。而且由于约翰逊并不处于当时的政治中心,其人也并不能代表英国的一般人物,所以这部作品不是我们必须模仿的范本。‎ 是不是我国已经翻译过来的《维多利亚女王传》可以作为范本呢?应当说是可以,由于作者着墨无多,处处显得“颊上三毫”的风神。可是中国文人相传的做法,正是走的一样的道路,所以无论近代人怎么推崇这部作品,总还不免令人有“穿新鞋走老路”的戒心。‎ 国内外的作品读过一些,也读过法国评论家莫洛亚的传记文学理论,是不是对于传记文学就算有些认识呢?不算,在自己没有动手创作之前,就不能算是认识。‎ 这时是1940年左右,中国正在艰苦抗战,我只身独处,住在四川乐山的郊区,每周得进城到学校上课,生活也很艰苦。家乡已经陷落了,妻室儿女,一家八口,正在死亡线上挣扎。我决心把研读的各种传记作为范本,自己也写出一本来。我写谁呢?我考虑了好久,最后决定写明代的张居正。第一,因为他能把一个充满内忧外患的国家拯救出来,为垂亡的明王朝延长了七十年的寿命。第二,因为他不顾个人的安危和世人的唾骂,终于完成历史赋予他的使命。他不是没有缺点的,但是无论他有多大的缺点,他是唯一能够拯救那个时代的人物。‎ ‎(有删改)‎ 相关链接 ‎①自传和传人,本是性质类似的著述,除了因为作者立场的不同,因而有必要的区别以外,原来没有很大的差异。但是在西洋文学里,常会发生分类的麻烦。我们则传叙二字连用指明同类的文学。同时因为古代的用法,传人曰传,自叙曰叙,这种分别的观念,是一种原有的观念,所以传叙文学,包括叙、传在内,丝毫不感觉牵强。‎ ‎(朱东润《关于传叙文学的几个名词》)‎ ‎②朱先生确是有儒家风度的学者,一身正气,因此他所选择的传主对象,差不多都是关心国计民生的有为之士。他强调关切现实,拯救危亡,尊崇气节与品格。这都是可以理解的。‎ ‎(傅璇琮《理性的思索和情感的倾注 ‎——读朱东润先生史传文学随想》)‎ ‎1.下列对材料有关内容的分析和概括,最恰当的两项是(  )‎ A.当年有所大学的中文系开传记研究课,课程内容却是韩愈、柳宗元的古文,朱东润就是因为这件事决心献身传记文学的研究。‎ B.“我的命运是作为沙子而到中文系开课的”,这样的表述与其说写出了自己过去的经历,不如说反映了朱东润写自传时的心态。‎ C.朱东润虽然认可国外的传记文学,但却担心“穿新鞋走老路”,因此拒绝把近代人推崇的《维多利亚女王传》作为写作范本。‎ D.出于自己的现实关怀来选择传主,是朱东润传记文学创作的一贯原则。有学者总体上对此表示理解,但在态度上略有保留。‎ E.朱东润虽然认为“传叙文学”的说法更加科学,但为了避免常会发生的分类麻烦,还是在自传中采用了“传记文学”的说法。‎ 答案 DB 答D给3分,答B给2分,答A给1分;答C、E不给分。‎ 解析 A项,朱东润“决心献身传记文学的研究”,除了因为“这件事”,还因为这是他的兴趣爱好。C项,“拒绝”说法不当,文中说“应当说是可以”。E项,“为了避免常会发生的分类麻烦,还是在自传中采用了‘传记文学’的说法”错误,文中无此信息。‎ ‎2.朱东润的传记文学观是如何形成的?请结合材料简要分析。‎ 答:_________________________________________________‎ 答案 ①广泛阅读古今中外的传记作品,如《史记》《汉书》《约翰逊博士传》《维多利亚女王传》等,并比较它们的异同;②深入研究传记文学理论,辨析不同概念,如阅读莫洛亚的传记文学理论,分辨史传、别传、自传、传叙文学等;③进行传记文学写作实践,如给张居正写传。‎ 解析 题目要求对朱东润传记文学观形成的过程进行分析,比较容易得到答案,因为整个文章是围绕这一问题展开的。对全文进行相关信息的筛选、整合即可。朱东润对一些传记作品的阅读与比较,对传记文学理论的研究,进行的传记文学的创作实践等,都应该成为本题的筛选点。‎ ‎3.作为带有学术性质的自传,本文有什么特点?请简要回答。‎ 答:_________________________________________________‎ 答案 ①偏重学术经历,主要写自己的传记文学观及其形成过程;②写生平与写学术二者交融,呈现学术背后的家国情怀;③行文平易自然,穿插使用口语,就像和老朋友闲谈一样。‎ 解析 题目要求从“带有学术性质的自传”的角度出发,分析文章的特点。显然,文章以作者自身的经历为依托,介绍了作者对传记文学的深入理解,而学术背后的家国情怀也得以呈现。从语言上看,本文语言朴素自然,偶用口语,如“总想来一些变动”。‎ ‎4.朱东润认为传记文学作品应如何刻画和评价传主?你是否同意他的观点?请结合材料说明理由。‎ 答:_________________________________________________‎ 答案 第一问:①应该入情入理地细致刻画传主的个性。如果只重比较,就看不清传主的个性,而要是像《维多利亚女王传》那样就不够细致,像《约翰逊博士传》那样细致则难免琐碎。②应该信笔直书,全面评价传主的优缺点。要是像有些古代文人的作品那样只是歌颂死者,就不是传记文学。‎ 第二问:观点一,同意。①只有入情入理地细致刻画传主的个性,才能给人深刻的印象,且具有可读性;②人无完人,只有全面评价传主的优缺点,才能给读者一个完整的人物形象。‎ 观点二,不同意。①细致刻画个性需要史料支撑,如果史料不足而仍然强调这一点,就会导致不够客观,显得矫揉造作;②追求全面评价传主的优缺点,不能有效凸显传主的个性。‎ 解析 题目要求对朱东润刻画和评价传主的主张进行分析探究。第一问要回答朱东润的具体主张是什么,这就需要考生围绕“刻画和评价传主”这一点从文中筛选出关键信息,如“没有那些阿谀奉承之辞,而且是信笔直书,对于传主的错误和缺陷,都是全部奉陈”。第二问具有开放性,则可以仁者见仁,智者见智,但需要从材料中寻找理论依据。‎ 四、[2014·全国卷Ⅰ]阅读下面的文字,完成1~4题。‎ 科学巨人玻尔 ‎1927年,第五届索尔维物理学会议在布鲁塞尔召开,激烈的辩论很快就变成了一场爱因斯坦与玻尔之间的“决斗”。这场辩论在三年后的第六届索尔维会议上战火再续,玻尔获得胜利,他所代表的哥本哈根学派因此获得了大多数物理学家的认同,他们对量子力学的解释也被奉为正统解释。这次辩论就是著名的“爱因斯坦—玻尔论战”,有人称之为物理学史上的“巅峰对决”。‎ 爱因斯坦和玻尔这两位科学巨人的背后,是现代物理学的两大基础理论——相对论和量子力学。他们的争论旷日持久,几乎所有理论物理学家都被吸引并参与进来,乐此不疲。尽管两人的科学理论和思想观点始终没能调和,但他们却结下了长达数十年的友谊。玻尔高度评价他与爱因斯坦的学术之争,认为它是自己“许多新思想产生的源泉”。爱因斯坦也称赞说:“很少有谁像玻尔那样,对隐秘的事物具有如此敏锐的直觉,同时又兼有如此强有力的批判能力。他是我们时代科学领域伟大的发现者之一。”‎ 与爱因斯坦更个性化的独自研究不同,玻尔周围聚集着许多杰出的理论物理学家。他不但有革新的勇气,更是一位伟大的伯乐。他为量子物理学培养和组织了一支创新发展的队伍,人们称之为“哥本哈根学派”。后来的诺贝尔物理学奖获得者玻恩、海森伯、泡利以及狄拉克等都曾是其主要成员。‎ 哥本哈根学派活动的大本营就是哥本哈根理论物理研究所。该所是玻尔在1917年申请,并于1921年正式成立的。他以著名科学家的身份为研究所作担保,筹集了大量资金。在任所长的40年间,他以特有的人格魅力,吸引了世界各地的青年才俊,使研究所成为当时全世界最重要、最活跃的量子力学研究中心。这里先后培养了600多名物理学家。玻尔使这个科学家群体中的每个个体的力量发挥到极致,形成了以集体讨论和自由探索为特征的研究风格。他还经常在此举办非公开的小型年会,邀请各国著名的物理学家出席,相互学习,启发交流。这里没有论资排辈,只有挑战与争鸣,形成了富有激情和活力、不断进取的学术精神,人们誉之为“哥本哈根精神”,这种精神至今仍在科学研究领域受到推崇。量子力学每前进一步,或多或少都与这个学派科学家的合作研究有关。可以说,玻尔领导的哥本哈根学派具备了一个科学学派应有的优秀特质。‎ 希特勒上台后,玻尔以访问德国为名,暗地调查德国科学家的安全情况,然后设法把可能受到迫害的犹太科学家转移到安全地方。他还积极创立和参加丹麦救援组织,尽力帮助逃到哥本哈根的科学家与其他难民。‎ 德国纳粹控制丹麦后,玻尔起初留在国内,与抗敌组织保持密切联系。他一贯的不合作态度,令纳粹非常恼火。1943年玻尔受到纳粹分子的威胁,他冒险出逃,历尽艰险,辗转到达美国。在美期间,为抗击法西斯,他曾参加原子弹的研制工作。在研制过程中,他就考虑到这一研究成果对未来世界的影响,并曾多次接触英美首脑,建议他们及早与苏联达成控制原子武器的协议,但没有成功。‎ 二战后,玻尔积极倡导和实施国际间的科学合作。1957年,美国福特基金会将第一届“原子为了和平”奖授予玻尔,以表彰他“在全世界迫切需要的原则上,以友好的精神进行科学探索,在和平利用原子能以满足人类需要方面作出了榜样”。‎ ‎(摘编自邹丽焱《玻尔传》)‎ 相关链接 ‎①玻尔(1885~1962),丹麦物理学家。在普朗克量子假说和卢瑟福原子行星模型的基础上,于1913年提出氢原子结构和氢光谱的初步理论。稍后,又提出“对应原理”。对量子论和量子力学的建立起了重要作用。1927年又提出互补原理。在原子核反应理论、解释重核裂变现象等方面,也有重要贡献。获1922年诺贝尔物理学奖。‎ ‎(摘自《辞海》第六版)‎ ‎②1918年,玻尔的老师卢瑟福邀请他赴英国工作,他在回信中说:“虽然哥本哈根大学在财力、人员、能力和实验室管理上,都达不到英国的水平,但我立志尽力帮助丹麦发展自己的物理学研究工作……我的职责是在这里尽我的全部力量。”‎ ‎(摘自戈革《玻尔集》)‎ ‎1.下列对材料有关内容的分析和概括,最恰当的两项是(  )‎ A.爱因斯坦与玻尔在争鸣中惺惺相惜,爱因斯坦高度评价玻尔的贡献,玻尔也感念爱因斯坦的支持,他们之间建立了长久的友谊。‎ B.玻尔以自己创办的研究所为平台,通过邀请各国科学家前来交流学习,使团队的成员能有机会博采众长,不断发展量子力学理论。‎ C.玻尔敏锐察觉到纳粹将要对犹太人实施迫害,及时转移了大批犹太科学家,后来还亲自参加了丹麦的抗敌组织,反对纳粹暴行。‎ D.玻尔不但有科学家的直觉,也不乏政治家的远见。他预感到核武器的危害,试图尽力说服各大国首脑达成禁止使用核武器的协议。‎ E.玻尔致力于维护世界和平,为科学技术的国际间合作及和平利用原子能作出了卓越贡献,并获得了 ‎“原子为了和平”奖。‎ 答案 BE 答B给3分,答E给2分,答A给1分;答C、D不给分。‎ 解析 A项,“玻尔也感念爱因斯坦的支持”在选文中并无明显的表述。C项,“玻尔敏锐察觉到纳粹将要对犹太人实施迫害,及时转移了大批犹太科学家”与原文不符;“参加了丹麦的抗敌组织”错误,原文为“参加丹麦救援组织”“与抗敌组织保持密切联系”。D项,“试图尽力说服各大国首脑达成禁止使用核武器的协议”中的“禁止使用”应为“控制”;另外,“各大国”的表述也欠准确,原文为“英美首脑”。‎ ‎2.为什么爱因斯坦和玻尔的论战被称为物理学史上的“巅峰对决”?请结合材料简述原因。‎ 答:_________________________________________________‎ 答案 ①从成员上看,论战双方都是当时物理学界的代表人物;②从内容上看,辩论涉及现代物理学两大基础理论——相对论和量子力学;③从影响上看,辩论带动了整个理论物理界的学术争鸣。‎ 解析 这道简答题可以从选文的第一和第二自然段中去寻找:①巅峰对决的双方;②辩论涉及的内容;③对决造成的影响。‎ ‎3.文中说:“玻尔领导的哥本哈根学派具备了一个科学学派应有的优秀特质。”请结合材料,具体分析哥本哈根学派有哪些“优秀特质”。‎ 答:_________________________________________________‎ 答案 ①拥有站在学术前沿的核心领导人物;②有堪称骨干的科学家群体;③创造了独特的学术精神。‎ 解析 可从选文的第三、四自然段中概括总结:①什么样的领导人物;②学派的水平;③学派的宗旨。‎ ‎4.玻尔“特有的人格魅力”表现在哪些方面?请结合材料谈谈你的看法。‎ 答:_________________________________________________‎ 答案 ①追求真理,在学术之争中胸怀坦荡,不掺杂个人恩怨;②以赤子之心帮助祖国发展物理学研究;③慧眼识才,吸引了大批青年科学家,并为他们提供发展的平台;④有人道主义关怀,积极营救受纳粹迫害的科学家。‎ 解析 概括玻尔的人格魅力,要从全文着眼,筛选所有评述玻尔的内容,提炼出适合题目要求的内容,如:玻尔对学术真理的追求、对自己祖国的物理学研究、吸引大批青年科学家,还有营救受迫害的科学家,然后条理化,言简意赅地表达出来。‎ ‎[2年全国模拟重组]‎ 一、[2017·曲靖一中模拟]阅读下面的文字,完成1~3题。‎ 千秋家国梦,百年翰墨韵 耿蕾 蒋思豫先生在百岁时写下诗句以为纪念:“少年贫笈弃家乡,煮鹤焚琴六月霜。唾面自干腰不折,尘颜含笑看洛桑。”他还写有“士气峥嵘焉可侮,骨头如鼓作铜声”等联句。其诗其辞,也是老人百年人生、风骨铮铮的生动写照。‎ ‎1914年的春天,中国大地风起云涌。宜兴和桥,秀美的江南古镇。蒋思豫,就出生在这里。‎ 蒋姓是宜兴的大姓,蒋思豫的祖辈显赫不凡。他的外祖父徐致靖,是赫赫有名的戊戌“维新元老”,官至二品礼部右侍郎,著名的“二王”书法家。他的远房堂舅便是徐悲鸿。蒋思豫的父亲也是饱读诗书、精于书法。蒋思豫从小便酷爱书法,读小学时临摹柳公权的书法习作便已名列学校第一。‎ 故园与家族给予的人文影响,让蒋思豫拥有了不凡的眼界和气度,赋予他一颗处变不惊的心。‎ ‎6岁那年,父亲病逝,家道中落,小学未毕业的蒋思豫只能辍学跟随大姐去常州生活。‎ 转眼十余年过去,当年的稚童已长成清俊青年。18岁时,蒋思豫独自一人去上海谋生。他当过小报校对,学过铸字技术。1933年,蒋思豫迎来了他人生的最为重要的转折点——进入复旦大学求学。当时的年轻人都有着一腔爱国之心。1937年全面抗战爆发后,受过良好教育的蒋思豫加入了救国行列,先后在武汉、太原等抗战前线工作,还曾与周恩来、连战的父亲连震东在一个大院共事。‎ 抗战期间,蒋思豫还身兼《中国青年》编辑,并在《益世报》等担任记者和特约撰稿人,亲身经历了台儿庄战役和武汉保卫战。有一次,他与同事一起赴抗日前线采访,正遇日军进攻,一片枪林弹雨,同事只因稍微抬高了头,立刻被削去半个脑袋。还有一次在重庆工作时,遇到日军飞机空袭,因为来不及躲避,两位同事被炸身亡,他们倒下的地方与蒋思豫只相距数米。“一寸山河一寸血,十万青年十万军。”‎ 那时候,蒋思豫的心中便存有一个和平与宁静的梦,哪怕完成这个梦要耗费一生,他也在所不惜、追求不止。‎ 当年在复旦学习期间,他非常景仰于右任先生的道德文章和书法成就,将于右任编著的《标准草书》随身带了近十年,临习不断,立志端正书品,传承右公书法心得。‎ 思豫的品行、书艺得到于右任的认可,经于右任外甥周伯敏的推荐,他有幸成为于右任先生的入室弟子。1947年7月的一天,在南京于右任长子于望德府邸中,33岁的蒋思豫恭恭敬敬地向这位著名的“当代草圣”,磕头行拜师礼。于先生忙拉他起身说:“行个鞠躬礼就行啦!”就这样,蒋思豫得到右公的悉心指导,并渐渐形成了朴茂厚实、简洁凝练、运转持重又大气磅礴的草书风格。‎ ‎1976年,蒋思豫回到夫人的故乡——宁波镇海,过起了隐居的生活。在这近40年的时光里,蒋思豫将对书法的热爱发挥得淋漓尽致,与书法为伴的宁静岁月,让老人感到无比幸福。他经常对子女们说:“中国书画艺术源远流长、博大精深,是中华民族的文化瑰宝和巨大精神财富,延绵数千年的中国书法决不能在吾辈手上中断。”‎ 蒋思豫一生研习“于体”,其草书直追于公笔韵。他还写下洋洋万言的《论于右任标准草书》一至三论,为“于体”书法传承做出了理论贡献。他的篆隶各体,也均有不俗功力:篆书质朴凝练、格调高古;隶书结字工稳、平实古雅。胸襟的开阔与心气的平和,在老人的书法作品中得到了充分的体现。‎ 蒋老气度儒雅,言笑洒脱,又喜以诗言志,创作了不少反映晚年心声的诗歌:“六十年来多祸患,苦中有乐却亦难。九秩望颐老未死,晚向夕阳看落山。”念及故乡宜兴,他曾写下诗:“一别故乡八十年,家园早碎址难辨。近邻亲友去何处?只有屺山屹巍然。”‎ 蒋思豫在艺术上认真计较,但为人处世上却随遇而安、与世无争。在镇海区的一个老小区巷子里,一间二室一厅六十平方米的旧楼房,就是蒋思豫与夫人徐敏蕾的住所。家什是旧样子,墙面有些泛黄,门板十分轻薄,有一些房间仅用旧棉布当门帘。但眼前这两位老人的风采却让这间简陋狭小的屋子增添了活力与温馨。‎ 蒋思豫的人生在沧桑百年中历经坎坷,始终陪伴老先生的是他最钟爱的笔墨。唯有这件最珍贵的东西,没有在千般风雨中丢失。可以说,书法是蒋思豫一生的主调。‎ 相关链接 ‎①在蒋老住宅的门楣上,刻有“窭隐”二字。“窭”是贫穷之意。蒋思豫笑呵呵地说:“我是一个被历史遗弃,或说是被淘汰了的‘闲云野鹤’,一生无所作为的糟老头。”跌宕起伏的人生经历,至尊至贱的世事体验,蒋思豫对人生有着常人难以企及的旷达洒脱。他说:“不开心是一天,开心也是一天。何不笑看烟霞,落得一身轻松呢。”‎ ‎(王宣民《蒋思豫:百岁翰墨写春秋》)‎ ‎②百岁,是一个无上的命令。这年龄包含着生命对于时间的尊严,包含着历史对于个体的温情,包含着专业对于社会的颜面,包含着大地对于生灵的褒奖。一位书法家能享百年之寿,而笔墨未停,又可展示中国书法艺术的健康个体。蒋思豫先生正是一个书法艺术的持久实践者。‎ ‎(余秋雨《蒋思豫百岁书法集序》)‎ ‎1.下列对材料有关内容的分析和概括,不正确的一项是(  )‎ A.蒋思豫非常景仰于右任先生,练习他的书法,并经由于右任外甥周伯敏的推荐,成为先生的入室弟子。‎ B.蒋思豫在艺术上认真计较,使得书法成就颇高,篆书质朴凝练,格调高古,隶书结字工稳,平实古雅。‎ C.传记再现了蒋思豫先生百年的沧桑历程,他虽历经风雨,但始终没有丢弃钟爱的笔墨,是一个书法艺术的持久实践者。‎ D.本文语言清新自然,明快流畅,并大量引用蒋思豫先生的诗歌,增加了文学色彩,使语言在质朴之余更添典雅。‎ 答案 D 解析 D项,本文语言无“清新”的特点,文中也没有大量引用蒋老的诗歌。‎ ‎2.文章结尾说“书法是蒋思豫一生的主调”,这一说法在文中有哪些体现?请简要概括。‎ 答:_________________________________________________‎ 答案 ①自小受家庭熏陶,酷爱书法,小学时期,书法习作就名列学校第一。②‎ 复旦学习期间景仰于右任的书法成就,临习草书不断。③33岁时向于右任拜师,一生研习“于体”,逐渐形成自己的书法风格。④花甲之年回到夫人故乡,此后几十年一直与书法为伴,持久实践,将对书法的热爱发挥得淋漓尽致。‎ 解析 这是一道局部信息筛选的题目,注意找准区间,选取关键的词句分条作答,问题关键句“书法是蒋思豫一生的主调”,该题的答题区间在文章三、八、十一段,注意对文段进行划分,即在锁定“块”的基础上,分出“条”来抽取答案信息,提取关键词语加以总结,形成答案。‎ ‎3.蒋思豫先生有哪些人格魅力?请结合材料谈谈你的认识。‎ 答:_________________________________________________‎ 答案 ①真挚的爱国情怀。抗战时积极加入救国行列,身兼数职为国效力,心存和平之梦。②胸襟开阔,旷达洒脱,为人处事随遇而安,与世无争。蒋思豫自嘲是一个“一生无所作为的糟老头”,认为对待人生“何不笑看烟霞,落得一身轻松”。他与夫人住的旧楼房,家具老样,墙面泛黄,房屋简陋、狭小,却充满活力与温馨。③对自己崇尚实践的书法艺术认真计较。热爱书法,端正书评,书法功力不俗,在实践和理论两方面,为书法传承做出了突出贡献。‎ 解析 分析人物形象首先应结合着文中叙述性的语句,从身份、地位、修养、气质等总体上把握小说人物形象特点,然后根据小说中描写这个人物的言行举止的语句以及作者的议论或者作者借作品中其他人物对他的评价的语句直接提取或概括即可。本题可从“抗战时积极加入救国行列,身兼数职为国效力,心存和平之梦”;蒋思豫自嘲是一个“一生无所作为的糟老头”,认为对待人生“何不笑看烟霞,落得一身轻松”;他与夫人住的旧楼房,家具老样,墙面泛黄,房屋简陋、狭小,却充满活力与温馨;“热爱书法,端正书评,书法功力不俗,在实践和理论两方面,为书法传承做出了突出贡献”等情节来分析人物形象。‎ 二、[2017·新余质量检测]阅读下面的文字,完成1~3题。‎ 吴宇森的坚守 贺晓闽 我整理吴宇森先生历年导演的四十四部片单时,发现他出品的数量和心智的增长几乎同步。从1973年第一部影片《过客》开始,到1982年《八彩林亚珍》的近十年时间里,他拍了十多部讨好市场的喜剧片,却一直没有进入事业的拐点。其实,从教会学校走出来的吴宇森,最喜欢的是好莱坞的黑帮片、西部片以及黄金时期的歌舞片,这些电影的共同点是画面充满动感,故事跌宕起伏。而拍摄喜剧片,并不是他的追求,而且饱受争议。分析当年商业营销的影响,吴宇森慢慢明白,确定自我风格,才是对理念的一份坚持。‎ 回到现实生活中,吴宇森的内心似乎被照进了光,光无法言说,但通过挽留,可以多留在心中,使作品的光影与情感的温度融合。四年之后,吴宇森决定进行一场冒险赌博,他选用了一群被称作“票房毒药”的男人,如周润发、张国荣等,执导了《英雄本色》。该片在徐克的帮助下大获成功。这次冒险是值得的,大投资、大制作、大场面,从此奠定了吴宇森暴力美学的电影风格,让他把多年来在影坛积郁已久的失意与强烈的表现力浸透到每一个画面,其酣畅如行云流水般的剪接,在今天看来也是无可挑剔的。这不仅成为他的人生丰碑,也成为香港的电影丰碑。那年吴宇森四十岁,正是一个男人的青壮年,生命的春天和事业的春天一起到来。‎ 冒险未必就是实实在在的危险。它牵涉到人的认知模式,也牵涉到人的诸多观念,比如自信、决断。央视记者李小萌在《面对面》栏目中采访吴宇森时说,在《英雄本色》创作现场,吴宇森豪气毕现,他是进行了一场赌博,并且赢了,他和“票房毒药”的男人们征服了命运,相互迸发出了无比耀眼的光芒。我想说,证明自信的不仅需要实践,更需要时间。后来,吴宇森的影片让人们认识到,他的电影每次获得殊荣,都是内心历练的结果。他知道,达摩克里斯之剑依旧高悬头顶。‎ ‎1989年的《喋血双雄》也是吴宇森的重要代表作。在影片中,他用极为自信的手法让暴力世界中弥漫出无法言语的浪漫,华丽而控制自如的镜头、精练的对白以及舞蹈般的枪战动作都令人为之倾倒。特别是片尾在教堂里的生死决战:烛光映出的圣母像,背影中掠过的白鸽,一对最不应成为朋友的生死之交,共同执行着自己心目中的正义。正如吴宇森所说:“这是我最想用镜头来表达的东西,哲学式的电影,骑士般的角色,都是我心中的理想。”如果说《喋血双雄》让吴宇森把握了好莱坞动作片的命脉,那么《变脸》则让吴宇森在片中找到了东西方世界共同崇尚的那种“人道主义”和“家庭观念”。这部被认为“最具吴宇森风格”的作品,同时也为吴宇森带来全美华裔艺术基金会颁发的最高荣誉——“金环奖”,让他成为继李小龙、成龙之后,进入好莱坞的第三位华人明星。‎ 吴宇森的电影好看,他所讲述的故事不远不近,有爱有恨。不管是功夫片还是谍战片,都可以明显感觉到他的表达,感觉到一个活生生的人,和他背负的真实生活。这些人里,有庞大机构里的大人物,也有机构压榨下的弱小者,还有为了某些利益冒险的暴力者、潜伏者。但这些人的故事,能让人产生共鸣,我们从中可以看到,家庭、亲情、变迁,一切都被结合在一起,那么自然而然,像我们身边的故事。所以他的粉丝中,白领、军官甚至科学工作者的比例明显上升,因为,这不仅是电影。‎ ‎(选自《人物传记》,有删改)‎ ‎1.下列对这篇小说有关内容的分析和概括,不恰当的一项是(  )‎ A.本文在语言上很有特色,比如用意蕴丰富的语句表现吴宇森的坚守品格以及吴宇森执导的电影的风格特征等。‎ B.文章开篇写吴宇森执导了十多部喜剧片,其目的是说明吴宇森还没有进入事业的拐点。‎ C.吴宇森执导的电影《英雄本色》成功奠定了他在导演界的重要地位,也使得他以后的导演生涯十分顺利,从此吴宇森的心里也比较轻松了。‎ D.《喋血双雄》体现了好莱坞动作片的特点,如浪漫的暴力世界、精练的人物语言、舞蹈般的枪战动作以及华丽而控制自如的镜头等。‎ 答案 C 解析 C项“吴宇森的心里也比较轻松了”欠妥,文中说“他知道,达摩克利斯之剑依然高悬头顶”。‎ ‎2.文章第三段引用央视记者的话有什么作用?请简要分析。‎ 答:_________________________________________________‎ 答案 ①丰富了文章内容,增强了文章的说服力。②说明了影片《英雄本色》的成功,佐证了作者在上文中的观点,说明吴宇森的冒险是值得的。③侧面表现了吴宇森具有冒险精神,敢于同命运抗争。‎ 解析 这是一道典型的作用题,可从内容和结构两方面作答。内容上,要分析央视记者的话塑造人物形象、表达主旨等方面的作用;结构上,可分析央视记者的话与上下文的关系等。‎ ‎3.吴宇森执导的电影受到观众欢迎的具体原因有哪些?‎ 答:_________________________________________________‎ 答案 ①吴宇森执导的电影中,暴力世界中充满浪漫,镜头华丽而控制自如,对白精练,枪战动作如舞蹈一样具有观赏性。②电影故事体现了人道主义和家庭观念。③电影让人感觉到活生生的人物和他们背负的真实生活,能让观众产生共鸣。‎ 解析 此题的答题区间为后两段,要注意文中有关吴宇森执导的电影特点的表述,从其电影的画面、对白、表达主题、给观众的感受等方面入手分析原因,再用简洁的语言概括出来即可。‎ 三、[2017·鹰潭模拟]阅读下面的文字,完成1~3题。‎ 房玄龄,生而为宰相 初唐的天空,星光灿烂。‎ 在后人看来,这些光芒属于魏征、王珪、尉迟敬德、李世……一个个激情张扬的人。他们思考、磋商、争辩,挺直了脊梁建功立业,做人成事。‎ 然而一个被很多人忽视的问题是:这么多个性鲜明的人,最容易骄傲不服气,碰撞成灾难性的内斗和党争。‎ 他们太需要一个没有个性、没有激情的人,来做稳压器。‎ 这个人,就是房玄龄。他总是笑眯眯地洞明世事,又总是不吭声地隐忍委屈。皇亲攻击过他,重臣萧踽攻击过他,第一宠将尉迟敬德也攻击过他,眼看都要变成朝里的“出气筒”了,可他还是和没事人一样。他简直天生是给李世民做宰相的。‎ 有一次房玄龄得了重病,一个口齿轻佻的小官吏开玩笑道:“宰相小病去探访有好处,如果病得快要死了,去探访也就没什么用了。”有人将这话挑唆到房玄龄那里。房玄龄对此的反应是——见到那个随众来探访自己的小官吏时,笑着调侃一句:“你都肯来看我,那我一时半会儿还死不了啊……”‎ 房玄龄还是出名的“怕皇帝”者。急躁任性的李世民,也和大臣们一样,经常拿房玄龄当“出气筒”,越用越顺手。房玄龄照例不争辩、不顶撞,逆来顺受,道歉了事。有人因此讥讽老房“没骨气”‎ ‎,尤其是他身边还有傲骨铮铮的魏征作对比。其实房玄龄心里很明白,李世民是个多么骄傲的人,成天被魏征骂来骂去又不能还嘴,难免心理不平衡。那我老房做做自我牺牲吧,皇帝把气撒在我身上,总比他攒多了委屈以致决策失误要好。于是我们就看到了一次又一次心照不宣的表演:皇帝为鸡毛蒜皮的小事大发雷霆痛骂宰相,剥官落爵叫他回家待罪。房玄龄也就乖乖回家,平静地告诉家人“把房子打扫打扫,待会儿皇帝要过来接我回去”。而李世民果真就来接了,两人携手上车回宫,谁都不多说什么,好像发脾气那一幕从来就没发生过。‎ 那些君臣的脾气,终于在性情温和、通达睿智的房玄龄手里。揉捏成方向一致、和衷共济的“合力”了。‎ 但“老好人”房玄龄的本事绝不止于此,他有着令人惊叹的办事效率和实干能力。李世民和魏征等人通过争辩讨论定下了大政方针。房玄龄就和他选拔的一批实用性人才一起埋头苦干,去实现贞观的宏图大业。‎ 贞观时代,朝廷全部官员只有六百多人,房玄龄做了22年的宰相,总是日复一日处理着繁杂的日常行政事务,让一个高度精简的行政机构,发挥出强大的作用,支撑起贞观之治的日常大局。以致于李世民有时会抱怨他太过细致,警告说,身为宰相应该只管大事,把那些小事丢给下属就好。可房玄龄还是继续“我行我素”地“琐碎”着。‎ 当时尚书省下辖的各部里,工作最繁琐、最被士人看不起的,是管理财政预算和账目的“度支司”,一度无人肯任其职。房玄龄竟以宰相之尊自任“度支郎中”,亲自把守着大唐国库。‎ 这22年的殚精竭虑,比起早年辅佐李世民征战天下、决战玄武门,更值得记录在房玄龄一生功业的榜首。诚如后世史学家所公认的那样:作为宰相,他开国有功,却从不突出自己;王珪、魏征以谏诤闻名,他就竭尽全力给他们提供机会;李靖、李世善于带兵,他就在后方做好后勤支持;每一个官员,他都能让他们在贞观时代淋漓尽致地展示才华。他没有一项拿得出手的“政绩工程”,然而天下人都知道,这个国家少不了他,世民也一样知道。‎ 人生终是难免一死。贞观二十二年,房玄龄走到古来稀的71岁。在那个时代里,他已经是活得很长的老人了,高龄带来的疾病难以克服,房玄龄的身体越来越差。而此时,比房玄龄年轻许多的皇帝也已重病缠身,那太过绚烂的一生迅速地消耗掉了李世民的寿命。重病中的李世民,让人把房玄龄抬到殿中,在御座之侧放下,君臣对坐流泪,竟是相视无言。回首他们在渭北秋风之中的初遇,时光已经流逝了32年。‎ 一段浩浩汤汤的贞观时代,回荡着各种各样的旋律。人们立德,立言,立功,以丰富多彩的形式把自己独一无二的声音,铭刻在这个时代上。“贞观”因此而朝气蓬勃、气象万千。但贞观的第一宰相房玄龄,却在洪亮之声中沉默着,埋头忙于记录、整理、实施……他从来都没有自己的声音,他的声音就是“贞观”。‎ ‎1.下列对传记有关内容的分析和概括,不正确的一项是(  )‎ A.文中叙述一个口齿轻佻的小官吏开房玄龄玩笑的事件,从房玄龄的回话中可以看出房玄龄的大度、有气量。‎ B.房玄龄身为重臣,几十年来,没有做过大事,处理的都是一些琐碎的事情,以致君王李世民抱怨,警告他。‎ C.本文注重细微处写人。从小官吏开房玄龄的玩笑、被君王遣回家、君臣年高体衰时惺惺相惜等方面来突出传主形象,以小见大,可谓匠心独运。‎ D.行文叙评结合,以叙为主。既用鲜明的事例反映人物形象,又通过作者的评述补充人物形象,使人物形象鲜明丰满。‎ 答案 B 解析 B项,“没有做过大事”不准确,只是文本没有写其所做大事。‎ ‎2.文中传主是房玄龄,但是开头却写了魏征、王珪等人,有何用意,请做分析。‎ 答:_________________________________________________‎ 答案 写了他们的激情张扬、骄傲、个性鲜明,①为反衬房玄龄的沉稳、低调大度的性情;②为下文写房玄龄张本;③营造房玄龄作为宰相处理政事的人文环境,突出他的能力和性格。‎ 解析 人物传记中,经常在文章开头写传主前,先写一些其他人物,这主要是为了陪衬主要人物,用他们的性格陪衬传主的性格,用他们的形象衬托传主的形象,他们为传主构造了一个人物活动的人文环境,可以从这几个方面来解答。‎ ‎3.这篇人物传记,作者是如何刻画人物形象的?这样写有什么好处?请结合文章作简要分析。‎ 答:_________________________________________________‎ 答案 ①本文通过典型场景或典型事例或细节描写或对比或以小见大等手法刻画人物形象。②‎ 选择典型事例凸显人物个性;把人物放置在具体的场景中,借助描写和对比烘托等手法刻画人物,使传主形象更加丰满。(或选择细节描写或对比或以小见大等手法结合文本分析,言之成理即可)‎ 解析 第六段中,文中通过叙述一个口齿轻佻的小官吏开房玄龄玩笑的事件,从房玄龄的回话中可以看出房玄龄的大度、有气量。第二、三、四段中,通过典型场景来反映房玄龄的沉稳、低调大度。通过典型场景或典型事例或细节描写或对比或以小见大等手法刻画人物形象。这样写的好处从对刻画人物形象的个性,使人物形象更加丰满的角度解答。‎ 四、[2017·河南六市联考]阅读下面的文字,完成1~3题。‎ 一代人师蔡元培 刘继兴 被毛泽东誉为“学界泰斗,人世楷模”的蔡元培先生,是中国近现代著名的民主斗士和教育家,尤其是他创造的北大辉煌,历来为人所敬仰。‎ 他上任不久,就向全校发表演说,倡导教育救国论,号召学生们踏实地研究学问,不要追求当官。同时在管理中革故鼎新,将北京大学原来的那一套封建腐朽的条例扫进了历史的垃圾堆,使北大成为鲁迅所说的“常为新的改进的运动的先锋”。‎ 蔡元培深知,要振兴一所大学,仅靠思想与制度是远远不够的,必须注重人本,师资才是最关键的。他被任命为北大校长的当日,就力劝陈独秀出任北大文科学长。考虑到出任文科学长需要一定资历方能通过,蔡元培甚至为陈杜撰了履历,使他顺利地到北大任教。梁漱溟投考北大未被录取,他在《东方杂志》上发表了一篇讲佛教哲学的文章《究元决疑论》,蔡元培看了认为是“一家之言”,就破格请他来北大任教,讲印度哲学。这一年,梁漱溟年仅24岁。‎ 蔡元培执掌北大之后,强调自己的治校方针是:“依世界各大学通例,循思想自由原则,取兼容并包主义。”在蔡先生这种办学方针的指引下,那时北大不但聘请“左派”和激进派人士李大钊、陈独秀当教授,请西服革履的章士钊、胡适当教授,还聘身穿马褂、拖着一条长辫的复辟派人物辜鸿铭来教英国文学,喜谈怪论的国学家黄侃,甚至连赞助袁世凯称帝和筹安会发起人之一的刘师培,也登上了北大教坛。特别是“性博士”张竞生,被封建卫道士辱骂为“四大文妖”之一,在半封闭半蒙昧的中国,张竞生的言论绝对算得上耸人听闻,惊世骇俗,也被蔡元培延揽来北大讲“美的人生观”。‎ 蔡元培当北大校长期间做得最骇人听闻的事是开放女禁。那时有一个勇敢的女生王兰向蔡先生请求入学,蔡元培就让她到北大当了旁听生。这件事当时轰动了全北大、全北京。此后招生时,就允许女生和男生一样地应考了。这是男女同校的开始,后来各大学都兼收女生了。‎ 学术上的争论则更多。蔡元培均能很好地处理各方的关系,连最顽固的、拖着一条长辫的教授辜鸿铭也对蔡校长深为钦佩。1917年,蔡元培出版了《石头记索隐》一书,提出《红楼梦》是一部“政治小说”的概念。由于恰逢五四时期,人们“反对满清”的情怀甚重,因此这本书在当时得到了广泛传播。1921年,胡适发表《红楼梦考证》,矛头直指蔡元培。他毫不客气地指出蔡的索隐是牵强附会的“大笨伯猜笨谜”的方法。胡适为了推翻蔡元培的观点,曾到处寻找录有曹雪芹身世的《四松堂集》这本书。就在他求而不得、心灰意冷、近乎绝望的时候,蔡元培却托朋友为他借到了此书。胡适根据书中的史料记载,更加充分地证明了自己关于《红楼梦》是“曹雪芹自述”的说法。在蔡元培的影响与治理下,北大容纳了各派的学说和思想,空气新鲜得很。特别是教师们对于学术争论的态度,都颇具大师风范。如有一次钱玄同在讲课,对面教室里黄侃也在讲课。黄侃大骂钱玄同的观点如何如何荒谬,不合古训;而钱玄同则毫不在乎这些,你讲你的,我讲我的。‎ 蔡元培的两袖清风,也是有口皆碑的。1935年9月7日,蒋梦麟等联名给蔡元培祝寿,提出要为一生清廉直至晚年仍全家租赁房屋连藏书的地方都没有的老校长造一所“可以住家藏书的屋”。后来这个计划虽然由于抗战全面爆发而未能付诸实现,但反映了北大师生对蔡元培的深深敬爱之情。‎ ‎1940年3月5日,蔡元培在香港病逝。蒋介石在重庆主持公祭,红都延安则举行各界追悼大会。毛泽东在唁电中称其为“学界泰斗,人世楷模”;蒋梦麟的挽联是“大德垂后世,中国一完人”。捧着一颗心来,不带半根草去,这就是蔡元培一生的真实写照。‎ 伟哉蔡元培!‎ ‎(选自《天津日报》中《兼容并包 雅量似海:一代人师蔡元培》一文,有删减)‎ ‎1.下列对材料有关内容的理解和分析,不符合原文意思的一项是(  )‎ A.不拘一格,选用良师。只要是师才,蔡先生不拘一格,加以选用。不合文科学长要求的陈独秀和喜谈怪论的国学家都被请来任教。‎ B.兼容并包,思想自由。蔡元培执掌北大之后,包容各家观点人物,无论持激进思想还是复辟思想,都能在北大教坛寻得一处园地。‎ C.号召学生研究学问,改变北大学风。在蔡元培的影响和治理下,北大容纳了各种学说和思想,教师们对于学术的争论,很有大师风范。‎ D.北大开放女禁,主张男女同校,被辱骂为“文妖”的张竞生也登上北大讲坛,甚至于未当北大学生的梁漱溟也被破格请去讲学,因此鲁迅赞誉北大是“常为新的改进的运动先锋”。‎ 答案 C 解析 北大容纳各种思想,教师们学术自由,并不能说明“号召学生研究学问,改变北大学风”的观点。‎ ‎2.下列对传记有关内容的分析和概括,最恰当的两项是(  )‎ A.蔡元培病逝后,毛泽东和蒋介石分别在延安和重庆举行了追悼大会,充分说明了蔡元培先生在中国知识界的影响。‎ B.在北大,即使两个老师之间学术观点大相径庭,甚至面对课堂上有辱骂对方的现象,大家也见怪不怪,相安无事。‎ C.蔡元培为胡适借书一例,体现了蔡元培身体力行“思想自由,兼容并包”的方针,以身垂范,雅量高致,绝不以权压人。‎ D.蔡先生求贤若渴,唯才是举,且不拘一格。为了让陈独秀能顺利地到北大任教,他甚至不惜违背原则,亲自为陈杜撰了履历。‎ E.24岁的梁漱溟在北大这个舞台上尽情施展,终成了举世闻名的大哲学家。可以说,北大成就了梁漱溟,梁漱溟也成就了北大。‎ 答案 CD 解析 E项,“北大成就了梁漱溟,梁漱溟也成就了北大”不够严谨。A项,文中只提到毛泽东的挽联,并未提到毛泽东举行了追悼大会。B项,原文只是提到了老师间学术观点的争论,并非是辱骂对方。‎ ‎3.蔡元培先生除了作为独树一帜的教育家、大学者值得我们敬仰外,还有哪些值得我们学习和仿效的品质?请结合材料具体分析。‎ 答:_________________________________________________‎ 答案 ①为理想而奋斗的精神。捧一颗心来,不带半根草去。②管理的智慧。上任伊始,从思想、制度、师资引进等方面,为管理好北大奠定良好的基础。③为人的雅量。即使是对学术思想相左的胡适,蔡先生也不遗余力地帮他寻找支撑观点的书籍。蔡先生的雅量也影响了北大对于学术争论的态度。④两袖清风的节操。一生清廉直至晚年仍全家租赁房屋连藏书的地方都没有。⑤开放的思想。开放女禁;不同思想可以在北大得到包容。‎ 解析 此题考查学生个性化阅读文本、提出自己见解的基本能力。这是一道开放性的探究题,解答此题,要首先明确自己的看法,然后根据自己的观点,结合着文本的具体内容,联系现实生活,阐述自己的理解与看法,只要能言之有理即可。‎ 五、[2017·河南六市联考]阅读下面的文字,完成1~3题。‎ 钱钟书生命中的杨绛 黄薇 ‎①1928年,杨绛17岁,考入苏州东吴大学。1932年初,杨绛去了清华当借读生,结识钱钟书。1935年7月13日,钱钟书与杨绛结婚。随后钱钟书考取了中英庚款留学奖学金,杨绛毫不犹豫中断清华学业,陪丈夫远赴英法游学。1937年,上海沦陷,第二年,两人携女回国。钱钟书在清华谋得一教职,到昆明的西南联大上课,而杨绛留在上海,任了一年母校振华女中的校长。‎ ‎②钱钟书从昆明回上海后想写《围城》,杨绛甘做“灶下婢”,辅佐夫君全力搞创作,闲时尝试写了部四幕剧《称心如意》,第二年《称心如意》在金都大戏院上演时“引来阵阵喝彩声”,一鸣惊人,她所署的笔名 ‎“杨绛”也就此叫开。‎ ‎③新中国成立后,杨绛翻译的47万字的法国小说《吉尔·布拉斯》,受到朱光潜的高度称赞:我国散文(小说)翻译“杨绛最好”。‎ ‎④新中国成立后至清华任教,她带着钱钟书主动拜访沈从文和张兆和,愿意修好两家关系,因为钱钟书曾作文讽刺沈从文收集假古董。杨绛的沉稳周到,是痴气十足的钱钟书与外界打交道的一道润滑剂。家有贤妻,无疑是钱钟书成就事业的最有力支持。1946年,短篇小说集《人·兽·鬼》出版后,在自留的样书上,钱钟书为妻子写下这样无匹的情话:“赠予杨季康,绝无仅有的结合了各不相容的三者:妻子、情人、朋友。”‎ ‎⑤1958年,47岁的杨绛,利用大会小会间隙,开始自学西班牙语,打算从原文翻译《堂·吉诃德》。译稿历经“文革”的摧残,“被没收、丢弃在废纸堆里”,最后“九死一生”,逃过劫难。1978年4月译的《堂·吉诃德》出版并畅销。1978年6月,西班牙国王访问中国,邓小平把这本翻译书作为国礼送给西班牙贵宾。1986年10月,西班牙国王专门奖给75岁的杨绛一枚“智慧国王阿方索十世十字勋章”,以表彰她的贡献。‎ ‎⑥1989年,《围城》将要搬上荧屏前,杨绛边读剧本,边逐段写出修改意见。而出现在每集片头的那段著名的旁白——“围在城里的想逃出来,城外的人想冲出去。人生的愿望大都如此”被无数人时常引用,实际上就出自杨绛之手。钱钟书说,“实获我心”。‎ ‎⑦从1994年开始,钱钟书住进医院,缠绵病榻,全靠杨绛一人悉心照料。不久,女儿钱瑗也病重住院,与钱钟书相隔大半个北京城,当时八十多岁的杨绛来回奔波。1997年,被杨绛称为“我平生唯一杰作”的爱女钱瑗去世。一年后,钱钟书临终,一眼未合好,杨绛附他耳边说:“你放心,有我呐!”内心之沉稳和强大,令人肃然起敬。‎ ‎⑧2001年,杨绛参加了清华大学“好读书奖捐赠仪式”。这是她和钱钟书在病榻前商量好的,将二人全部作品著作权中因作品使用而获得的财产收益捐赠母校,以资助那些爱好读书的贫寒子弟。奖学金不用他们个人的名字。镜头前,90岁的杨绛站起来,用清脆明亮的声音讲述自己对清华校训“自强不息,厚德载物”的理解。‎ ‎⑨2003年,《我们仨》出版问世,这本书写尽了她对丈夫和女儿最深切绵长的怀念。时隔四年,96岁高龄的杨绛又意想不到地推出一本散文集《走到人生边上》,探讨人生的价值和灵魂的去向。走到人生的边上,她愈战愈勇,唯愿“死者如生,生者无愧”——钱钟书留下的几麻袋天书般的手稿与中外文笔记,多达7万余页,也被杨绛接手过来,陆续整理得井井有条。‎ ‎⑩2016年5月25日,杨绛在北京协和医院病逝,享年105岁。‎ ‎(选自《时代人物》,2016年6期,有删改)‎ 相关链接 ‎①我今年一百岁,已经走到了人生的边缘……我快“回家”了。我得洗净这一百岁沾染的污秽回家……细想至此,我心静如水,我该平静地迎接每一天,过好每一天,准备回家。‎ ‎(杨绛《走到人生边上》)‎ ‎②整个20世纪,中国文学界再没有一对像他俩这样才华高而作品精、晚年同享盛名的幸福夫妻了。‎ ‎(夏志清,文学评论家)‎ ‎③从杨绛涉足文坛开始,她坚持写作时心灵的自由表达,抒发自我、关注个体,坚持知识分子的独立姿态,这种创作态度代表着一类知识分子清高孤傲和明哲保身的价值取向。‎ ‎(《杨绛:淡定背后的风华》)‎ ‎④在多重因素的共同影响下,杨绛早已成为一道文化景观,然而景观往往遮蔽真义。如果粗暴地将杨绛简化成一个标签、一个传奇,那是对知识尊严的亵渎。‎ ‎(《纪念作为知识分子的杨绛》)‎ ‎1.下列对材料有关内容的理解和分析,不符合原文意思的一项是(  )‎ A.杨绛以“杨绛”为笔名创作四幕剧《称心如意》,得到好评。翻译法国小说《吉尔·布拉斯》、西班牙小说《堂·吉诃德》等。出版了散文《我们仨》和《走在人生边上》等。‎ B.杨绛47岁开始自学西班牙语,准备从原文翻译《堂·吉诃德》,体现了杨绛终身学习的思想。‎ C.《堂·吉诃德》的译稿经历文革,命运多舛,最后终于逃过一劫,获得出版。说明翻译《堂·吉诃德》的环境极其恶劣。‎ D.《堂·吉诃德》翻译本作为国礼赠给西班牙国王,西班牙国王授予杨绛“智慧国王阿方索十世十字勋章”,说明杨绛在国外很有名声。‎ 答案 D 解析 说明杨绛这本书翻译得很好。‎ ‎2.下列对材料有关内容的分析和概括,最恰当的两项是(  )‎ A.钱钟书考取中英庚款留学奖学金后,杨绛中断在清华大学的学习,陪丈夫到英国、法国游学并深入学习英语、法语,这为她后来熟练翻译外国文学作品奠定了基础。‎ B.钱钟书认为杨绛是自己的“妻子、情人、朋友”,评价十分高,而夏志清所说的二人是一对才华高、作品精并在晚年同享盛名的夫妻,其原因就在于杨绛的这种身份定位。‎ C.在钱钟书、钱瑗住院期间,杨绛要走大半个北京城分别照顾他们。在他们去世后,杨绛用《我们仨》表达对他们的怀念之情,由此可见杨绛老人坚定的性格和内心的强大。‎ D.《坐在人生的边上》表达了杨绛看淡生死和净化灵魂的思想,反映了老年的杨绛对心如止水、平静地度过每一天的追求,说明此时的杨绛已达到完全净化自己的目的。‎ E.如果忽略杨绛是知识分子,忽视她在翻译领域等文学创作方面的贡献,只津津乐道杨绛是钱钟书的妻子或者杨绛活到了105岁等内容,就是对知识尊严的亵渎。‎ 答案 EC 解析 A项,“深入学习英语、法语”于文无据,原文中没有说杨绛此时深入学习英语和法语;后来杨绛翻译外国作品时还学习了西班牙语。B项,“原因就在于杨绛的这种身份定位”错误,钱钟书的说法表达的是对杨绛的感谢之情,而杨绛的文学作品与钱钟书对杨绛的称赞没有关系。D项,“反映了老年的杨绛对心如止水、平静地度过每一天的追求,说明此时的杨绛已达到完全净化自己”错误,此时的杨绛已经过着心如止水的生活,不是“追求”,“完全净化自己”文中没有任何根据。‎ ‎3.杨绛被称为“钱钟书生命中的杨绛”,除了她是钱钟书的爱人外,一定还有其内在原因。请结合材料具体分析。‎ 答:_________________________________________________‎ 答案 ①甘于奉献,帮助丈夫创作。钱钟书准备写小说《围城》时,杨绛甘愿做“灶下婢”;《围城》将要搬上荧屏时,杨绛提出修改意见,写出片头的旁白。②襟怀坦白,协调疏通关系。在清华任教期间,杨绛带着钱钟书主动与沈从文和解。③勇敢无畏,整理丈夫遗稿。百岁高龄时,把写得如天书一般的钱钟书的遗稿整理得井井有条。④低调做人,捐资帮助学生。实现钱钟书生前遗愿,把二人全部作品著作权中因作品使用而获得的财产收益捐赠母校,并且奖学金不用二人的名字。‎ 解析 本题考查发掘文本所反映的人生价值和时代精神的能力。杨绛被称为“钱钟书生命中的杨绛”,一定是从杨绛在钱钟书生命中起到了巨大的作用来考虑的。具体在文本介绍中,可以包括学术研究、生活、为人等不同角度。第一段写“杨绛毫不犹豫中断清华学业,陪丈夫远赴英法游学”,从此杨绛进入钱钟书的生命里,生死相随。第二段,为了支持钱钟书写《围城》,杨绛甘心做他的“灶下婢”,全力支持。第四段,杨绛的沉稳周到,是痴气十足的钱钟书与外界打交道的一道润滑剂。由此恢复了和沈从文的关系。第六段写杨绛帮助钱钟书修改剧本。第七段写两个人共同的处世作风,把著作权捐赠给母校资助贫寒学子。‎ 滚动提升训练(七)‎ ‎  时间:50分钟   满分:58分 一、语言文字运用(20分)‎ ‎1.[2017·成都龙泉一中月考]下列加点成语使用正确的一项是(3分)(  )‎ ‎①遇到问题,他总喜欢一个人琢磨,不向领导汇报,往往误事。不过,这是工作中出现的小问题,不足为训,看一个人应该看主流。‎ ‎②在社会责任上,有两种经济学观点可谓南辕北辙:一种认为股东实现组织利益最大化是企业的天职,另一种观点认为企业必须对社会负责。‎ ‎③他性格内向,一涉及个人生活问题就讳莫如深,弄得我们不知如何是好。‎ ‎④自己再也躲不过雪,无论我蜷缩在屋子里,还是远在冬天的不为人知的另一个地方,纷纷扬扬的雪,都会落在我正经历的一段岁月里。‎ ‎⑤短小精悍的吕厚民,1950年被调到中南海,专门给毛主席和其他中央领导照相,开始了前后12年不平凡的人生历程。‎ ‎⑥2015年,民航进入多事之秋,空难事件一个接一个,一时间,民航人草木皆兵,民航安全形势更是如履薄冰。‎ A.①④⑥ B.③④⑤‎ C.②④⑤ D.②④⑥‎ 答案 B 解析 ①不足为训:不能当作典范或法则。望文生义。②南辕北辙:心里想往南去,却驾着车往北走,比喻行动和目的相反,对象不当。③讳莫如深:将事情尽量深藏隐瞒,使外人不知,正确。④纷纷扬扬:形容雪、花、叶飘洒得多而杂乱,正确。⑤短小精悍:a.形容人身材矮小而精明强干;b.形容文章、戏剧等篇幅短而有力。选项取a的意思,是说人,故正确。⑥如履薄冰:形容非常谨慎、小心翼翼的样子。句中说“民航安全形势”,对象不当。‎ ‎2.[2017·名师原创卷]填入下面文段空白处的词语,最恰当的一组是(3分)(  )‎ ‎__①__你对“共享经济”一知半解,__②__你有必要了解一下现在流行的打车软件。一个软件平台,把私家车、出租车都整合起来了。它给大家的出行带来了便利__③__全新的体验。__④__,我们会发现一个商业的基本事实:无共享,不经济。美国硅谷预测家认为,在不久的将来,可能你要做的事情就是:在网站上挑一个工作任务,__⑤__不用朝九晚五地工作。__⑥__工作之外,你的吃穿住行,全部可以实现共享。‎ ‎①‎ ‎②‎ ‎③‎ ‎④‎ ‎⑤‎ ‎⑥‎ A 因 因而 或 所以 然而 除 B 如果 那么 和 于是 但是 除了 C 因为 所以 且 于是 而且 除非 D 假如 那么 或 故而 虽然 除了 答案 B 解析 解答此题,首先要通读语段,理解句意,摸清语段中存在的逻辑关系,然后逐空进行分析,运用排除法确定答案。分析可知,文段第一句话是假设句,前一个分句假定“你对‘共享经济’一知半解”,后一个分句是针对这一假设给出的建议。据此可排除A、C两项。“便利”和“全新的体验”是并列关系,应选“和”。据此排除D项。‎ ‎3.[2017·名师原创卷]下列各句中,没有语病的一句是(3分)(  )‎ A.郭聃洋入选第五批“长白慧谷”英才计划,这意味着他研发了新型益生菌共生体制剂项目将获得数额可观的资金支持,在较短的时间内产值可达上千万元。‎ B.位于韩国首尔的东大门市场是亚洲规模最大的批发市场之一,也是中国代购商的首选之地,这里服饰、玩具、化妆品、首饰等一应俱全。‎ C.朝鲜族舞剧《阿里郎花》两场演出门票很快销售一空,其实在去北京公演之前,这部舞剧就已经在吉林很火了,人们被它的感人情节和艺术魅力深深打动。‎ D.面对如雨后春笋般崛起的林立高楼,山东省青岛市李沧区却选择了做“减法”:减出更多的城市空间,让城区的格局“疏密得当”,多一些宜居的幸福感。‎ 答案 C 解析 A项,结构混乱,可将“他研发了”改为“他研发的”。B项,不合逻辑,“服饰”包含“首饰”,二者不能并列。D项,成分残缺,可在“多一些宜居的幸福感”前加“让市民”。‎ ‎4.[2017·绵阳丰谷中学诊断]在下面一段文字中横线处补写恰当的语句,使整段文字语意完整连贯,内容贴切,逻辑严密。每处不超过15个字。(5分)‎ ‎  __①__。17世纪法国思想家伏尔泰说过,时间是个谜:“最长又最短,最快又最慢,最能分割又最宽广,最不受重视又最宝贵,渺小与伟大都在时间中诞生。”这一串充满哲理的话,__②__。例如,上课的时候,总觉得时间很长,过得很慢;放假的时候,却又觉得时间很短,过得很快……是啊,__③__。‎ 答:①________________________________________________‎ ‎②____________________________________________________‎ ‎③____________________________________________________‎ 答案 ①时间让人感到神秘莫测 ②我们在日常生活中也能感受到 ③时间真的太神秘了 解析 开头有引起下文或统领全段的作用,根据文意可联想到关键词“神秘莫测”“奇妙”等;中间要搭起一块跳板,把前后脱节的地方衔接起来,根据文意应从身边的实际生活着眼;结尾最好能够“一石二鸟”,既能收束文段,又与开头照应。本题答案不强求统一,只要基本符合要求即可。‎ ‎5.[2017·宜昌一中测试]下面是关于理想的一个框架图,请你把它写成一段话,要求内容完整,表述准确,语言连贯,不超过70个字。(6分)‎ 答:_________________________________________________‎ 答案 我们树立理想的前提是要保证它科学合理,并且要具体可实施。在实现理想的过程中,我们需要踏实,坚韧不拔,同时还要具备良好的心态。‎ 解析 图文转换的题型主要有以下几种:描述内容,拟标题,品寓意。本题要求的是描述内容,要求做到客观、全面,即把所呈现内容按照一定的逻辑关系说明。应按照从下往上的顺序进行说明,先是实现理想的条件包括两个方面,一是前提,二是过程;再根据事物的逻辑关系来陈述,前提条件是什么,过程有哪些。‎ 二、文学类文本阅读(14分)‎ ‎6.[2017·鹤壁统考]阅读下面的文字,完成(1)~(3)题。(14分)‎ 教授 ‎[美]兰斯顿·休斯 七点整,一辆大型轿车在布克蒂华盛顿旅馆前面停了下来,一个穿制服的白人司机从车里走了出来,向大门走去,打算向服务台询问有没有一位名叫布朗的黑人教授住在这里。其实这位教授正坐在门厅里。‎ 司机一进来,教授就迎了过去。“是钱得勒先生的汽车吗?”他带点踌躇地问道。“对,先生,”白人司机对这位穿戴整齐、身材矮小的黑人说,“您是布朗博士吗?”“我就是。”教授微笑着稍稍欠身说。司机给布朗博士打开了临街的大门,他们上了车。汽车驶过一条长长的黑人街。坐在轿车里的教授对于黑人区的大街总是呈现同样的低劣的、讨厌的景象感到很遗憾。教授移开了目光不再去看这种又贫穷又邋遢的典型黑人大街的令人不愉快的情景。‎ 钱得勒一家在中西部和南方都很有势力,是美国巨富之一。现在这位钱得勒先生和他的夫人,就要邀请布朗教授到他们的住宅来吃晚饭。‎ 教授坐在这辆又宽大又暖和的轿车里,在美丽的林荫大道上迅速地前进,黑人贫民区已经远远地落在他们后面了。‎ 教授叹了口气,望着汽车外边宽广的草地和好看的住宅,这些住宅里面住的都是白人。过了一会儿,汽车转进一条郊区道路,只见一些吊着常春藤的围墙,修得很整齐的灌木,还有黄杨木等,这些都表明这里不仅仅有住宅,还有庞大的庄园。不久,汽车穿过充满了喷泉和树木的园林,开到了一座像旅馆一样大的私人住宅前。一盏挂在高大门廊上的巨大吊灯,散发出柔和的光线,直照在轿车镀有铬合金的黑色车身上。白人司机跳下车来,恭恭敬敬地为黑人教授把车门打开。男管家领着教授走进一间大客厅。‎ 这时钱得勒先生和他的夫人走上前来,一一同教授握了手。他们又介绍了另一位将和他们一起进晚餐的客人布尔威克博士。‎ ‎“很高兴认识您,”布尔威克博士说,“我跟您一样,也是社会学家。”‎ 男管家送来一大银壶雪利酒。他们都坐下来,开始有礼貌地谈了起来,布尔威克博士开始问起布朗教授的那本书——《偏见的社会学》,问他认为黑人问题是否能有解决的一天。‎ 布朗博士亲切地说:“我们是在前进。”这句话是他经常说的,虽然他老是感到自己是在说谎。‎ 布尔威克博士说:“没有一所为黑人办的大学——虽然这儿的人口有将近百分之四十是黑人。我们又不能把他们当作本校学生收下。这个在目前是不可能的,太不幸了。”‎ ‎“确实,确实。”钱得勒先生说,“但是,等你们黑人有了发展——我们希望他们将来有所发展,我们委员会计划帮助他们发展——那时候他们的系将由像您这样的人来主持。”‎ ‎“您说得很对。”布朗博士圆滑地答应着,他想到他们会捐赠一个社会学的讲座,他自己将成为讲座的第一任教授,也许每年可以收入一万元。但是他的脑子里忘不了刚才坐车经过的黑人区大街,满街的肮脏、悲惨景象,还有那热水总是冰冷的只住黑人的旅馆,那个他讲课的黑人教堂,以及其他所有隔离黑人、歧视黑人的待遇——从来就没有对黑人有过平等的待遇!‎ ‎“饭已经好了。”男管家说。‎ 他们站起来,走进餐厅,边喝汤边轻松地交谈。‎ ‎“您的《偏见的社会学》,”布尔威克博士说,“在结束处有一段话是我非常赞成的。”‎ 布朗博士不住地点着他的黑脑袋,心里想的却是有一年一万元的收入,夏天他可以把他的一家带到南美洲去,在那儿他们就可以有三个月的时间不当黑人。‎ 钱得勒说:“现在我们再谈谈您的学院——大学吧,我相信您称它为大学。要使它成为第一流的大学,您需要的是?”‎ ‎“我们需要,”布朗博士说起来了。眼前来了黄金机会,这工作可能变成一年一万元的职位,他本人可以有足够的钱,至少能带领他的一家到南美洲去旅行,在那里他们就可以不当黑人了。“我们需要,钱得勒先生。”‎ 布朗博士的小学院所需要的东西在钱得勒夫妇的眼里都算不上一回事。布朗博士这种明智的、稳健的提出要求的方式使钱得勒夫妇的慈善心肠大为高兴。在适当的时候布朗博士起身要走了。汽车叫来了,他与布尔威克博士和钱得勒夫妇一一握手。这些白人对布朗博士感到满意,于是他们就鞠躬送别。‎ ‎(有删改)‎ ‎(1)下列对这篇小说思想内容与艺术特色的分析和鉴赏,不恰当的两项是(4分)(  )‎ A.小说中的钱得勒夫妇对黑人布朗教授极其尊重,因而专门派了一位穿制服的司机准时到达旅馆去接布朗教授用晚餐。‎ B.布朗教授坐在车上经过黑人区的时候,看到黑人区的景象感到很遗憾,这是因为他自责没有为黑人种族争取应有的权力和地位。‎ C.小说在塑造布朗教授这个人物形象时,揭示了他的自私、虚伪和软弱,让读者在冷冷的轻蔑中又分明感受到一种沉重。‎ D.小说通过语言描写从正面突出了钱得勒先生的形象特点,又通过对其居住环境的描写从侧面说明了钱得勒先生的身份和地位。‎ E.这篇小说里没有大肆渲染的夸张,没有反差强烈的脸谱,更没有令人捧腹的喜剧效果,在冷静客观的叙事中蕴含着深刻的主旨。‎ 答案 AB 解析 A项从小说开头来看,钱得勒夫妇对布朗教授是有一定尊重的,但从全文来看,钱得勒夫妇内心对黑人还是存在歧视的。B项“这是因为他自责没有为黑人种族争取应有的权力和地位”说法错误,布朗教授此时并没有自责心理,而是对黑人区的景象感到厌恶。‎ ‎(2)小说的第二段是如何刻画布朗这个人物形象的?请简要分析。(4分)‎ 答:_________________________________________________‎ 答案 ①“司机一进来,教授就迎了过去”运用动作描写,表现布朗教授行动快捷,折射出他迫不及待地主动趋迎的心态。②“‘是钱得勒先生的汽车吗?’他带点踌躇地问道”运用语言描写,表现布朗教授语言犹豫,揭示了他面对白人时的胆怯和自卑。③通过对人物一“迎”一“问”的简单勾勒,透视出布朗教授被遮掩着的整个灵魂,既简单又含蓄,既直观形象又内蕴丰富。‎ 解析 解答此题,可根据刻画人物形象的基本方法,结合具体内容作答。小说在刻画布朗教授时,就分别采用了语言描写、心理描写和动作描写等描写方法来塑造布朗教授的人物形象。例如,“司机一进来,教授就迎了过去”运用动作描写,表现布朗教授行动快捷,折射出他迫不及待地主动趋迎的心态。以此类推,再结合语句从语言描写、心理描写等方面作答即可。‎ ‎(3)小说中运用了对比手法,请指出三处并简要分析。(6分)‎ 答:_________________________________________________‎ 答案 ①黑人和白人居住环境的对比。“又贫穷又邋遢的典型黑人大街”,与白人那边“宽广的草地和好看的住宅”构成对比,显示出不同种族在物质形态上的强烈鲜明的反差。②布朗教授的头衔和地位的对比。布朗是教授,和布尔威克博士一样是社会学家,但布朗在他们面前却唯唯诺诺。他的“教授”“社会学家”的头衔,与他在这个社会所处的可怜地位构成对比,丰富了人物形象,揭示了社会现状。③布朗的身份与他的表现的对比。布朗教授是黑人社会学家,本应为黑人谋福利,但他却自私自利,在与钱得勒等人交谈时一心想着“一万元”等私利。‎ 解析 解答此题,可先根据对小说内容的感知与理解,从文本中找出三处运用对比手法的语句,然后再结合着对比的一般性作用分析作答。例如,“又贫穷又邋遢的典型黑人大街”,与白人那边“宽广的草地和好看的住宅”构成对比,显示出不同种族在物质形态上的强烈鲜明的反差;还有布朗教授头衔和地位、身份和表现的对比,再结合原文进行分析。‎ 三、实用类文本阅读(24分)‎ ‎7.[2017·成都外国语学校一诊]阅读下面的文字,完成(1)~(3)题。(12分)‎ 在全球反法西斯格局中重新评价中国抗战 文汇报:您把二战中的中国称为“被遗忘的盟友”,可见在西方的二战研究中,中国战场的作用是被低估的。请您谈谈西方学界目前如何看待中国在二战中的作用。‎ 米特(英国牛津大学中国研究中心主任、现代中国政治与历史学教授):中国的抗日战争在整个第二次世界大战中所起到的作用不容忽视,但是在西方的二战语境中,中国抗日战场的重要性至今仍然是被忽视的。这种湮没不闻的现状很大程度上是拜冷战所赐。中国与苏联曾经同处一个社会主义阵营,因此,在1949年以后,西方的研究对于回顾二战期间的中国历史并未表现出多大的热忱。此外,很长一段时间内,西方学者来中国学习研究和搜集中国抗战史料的困难比较大,与中国学者合作的可能性更是微乎其微。‎ 文汇报:您本人如何评价中国战场在二战中的地位?‎ 米特:中国在二战期间的亚洲战场所起到的作用尤其重要。回到70多年前,如果中国在1938年被全面侵占,那么整个亚洲可能就不得不屈服于日本的铁蹄了。由于中国军民的浴血奋战,大量日本军队被牵制在中国战场,日本占领整个亚洲的企图因此无法实现。当然,中国的抗战也是整个二战的一部分,所以美国的协助对中国最终获胜也起到了重要作用,否则中国也不可能在1945年获得最后的胜利。‎ 文汇报:把中国的抗战历史放到整个二战的背景中来审视,中国抗战如何影响了整个二战的战局?‎ 米特:我认为中国战场最重要的贡献在于:在没有任何外界援助,也没有任何战争同盟可供借力的情况下,中国军民能在漫长艰苦的岁月中坚持不懈地独立抵抗日本侵略者。‎ 文汇报:根据您掌握的档案,中国的抗战为世界反法西斯战争做出的牺牲有多大?‎ 米特:从以往历史学家的研究中可知,负责任地估计,至少1400万中国人在战争中丧生、8000万中国人流离失所。‎ 文汇报:您为何认为1937年7月7日“卢沟桥事变”是二战起点,而不是普遍认为的1939年德国入侵波兰。这一论断的依据何在?‎ 米特:‎ 由于中国在二战中的重要作用尚未被西方充分认识,大多数历史学家倾向于将二战起始点标识为1939年。但是,如果我们将中国的抗战纳入整个二战的视野中来考虑,我想,随着中国抗战历史逐渐走出被遮蔽的研究现状,越来越多的人应该会认同我的看法,把“卢沟桥事变”作为二战的起点。‎ 文汇报:我们查阅您关于中国抗日战争的研究成果,您多次在文章中谈到,中国取得抗日战争的胜利,这对中国历史而言是个重要起点。请问这个“起点”开启了什么?‎ 米特:中国对日抗战所取得的胜利,极大改变了中国:中国的社会问题开始得到关注;在这段同仇敌忾的经历中,中国人(尽管不是每个中国人)的“国家统一”意识极大增强;抗战经历奠定了日后中国共产党的执政基础。总之,抗日战争重构了中国的社会与政治状况。‎ 文汇报:您认为现代中国的起点是什么时候?如何理解八年抗战锤炼出了一个现代中国?‎ 米特:所谓现代中国的起点并不是一个确切的年份,鸦片战争对中国而言的确有标志意义,但是中国的现代性其实并不仅仅由鸦片战争来指示,而是有不少其他标志,包括鸦片战争之前很长一段时期的经济发展和政府行为合理化的努力等。‎ 而抗日战争确实刺激了中国在更多领域的现代化,包括科技的发展,也对中国后来的发展意义重大。‎ 文汇报:您认为尽管中国是“被遗忘的盟友”,但是二战对中国后来历史的持续改变非常深刻,比对其他国家都深刻。这点如何理解?‎ 米特:我认为,中国在抗战胜利后,在政治方面的急遽变化是反抗外侮和抗战期间社会变革的迫切要求的产物。‎ 文汇报:您认为,对普通中国人而言,经过二战的洗礼,“国家”的概念比之前有了很大的加强。这是为什么?‎ 米特:抗日战争是一个漫长而艰难的过程,战争期间,民族感情并非无论何时、无论何地都能被激发。但是,在很多领域,比如教育界,由于对日作战的氛围,由于中华民族到了最危险的时刻,民族主义尤为激荡。此外,中国在医疗健康、难民救济和政府机构等各方面都有长足进步,整个抗日战争的过程促使政府和民众之间的联系变得紧密。‎ ‎(选自《文汇报》,有删改)‎ ‎(1)下列对文章有关内容的分析和概括,不正确的一项是(3分)(  )‎ A.记者在采访时首先提到米特把二战中的中国称为“被遗忘的盟友”的问题,原因是在西方的二战研究中,中国战场的作用是被低估的,而米特是坚持中国是二战盟友观点的人。‎ B.从米特对中国在抗战中起到的主要作用以及美国对中国的协助对中国最终取得抗战胜利起到重要作用等观点分析,米特是在全球反法西斯格局中重新评价中国的抗战。‎ C.中国抗战为世界反法西斯战争做出了巨大的贡献,但因为中国在二战中的重要作用还没有被西方充分认识,所以大多数历史学家倾向于二战起始点的标志是1939年。‎ D.记者在对米特提问时很注重引而不发的技巧,即只问相关的问题,而不更多地解说相关的背景,让米特根据自己的知识去判断询问的目的,然后表达出自己的独特见解。‎ 答案 D 解析 D项,说法错误,记者的提问十分注重相关的背景介绍,比如第一个问题,先表述米特的观点,然后询问西方对中国在二战中的作用的看法,目的很明确,没有让对方猜测问题。‎ ‎(2)超级大国的“冷战”为何让中国长期成为“被遗忘的盟友”?请结合文本简要分析。(4分)‎ 答:_________________________________________________‎ 答案 ①西方研究回顾二战期间的中国历史没有很大热忱。②西方学者来中国学习、搜集中国抗战史料的困难比较大。③相当长一段时间中,西方学者没有与中国学者合作研究中国抗战的可能性。‎ 解析 回答问题应注意题干的提示“冷战”,可以从“冷战”给中西方交流带来的阻隔的角度筛选相关信息,可以发现,因为“冷战”,西方国家的研究者很少涉及对中国抗战的研究,同时中西方在中国抗战研究上没有更多的交流,也没有任何的合作等。‎ ‎(3)米特在这次访谈中就中国在抗战中的贡献以及抗战对中国的作用方面阐释了哪些观点?请结合文本简要概括。(5分)‎ 答:_________________________________________________‎ 答案 ①中国抗战最重要的贡献在于坚持不懈独立抵抗日本侵略者,在中国战场牵制了大量日军。从“卢沟桥事变”‎ 开始,中国军民浴血奋战,坚决抗日,把大量日本军队牵制在中国战场,粉碎了日本占领整个亚洲的企图。②中国抗战锻造出了一个现代的中国,这是因为长期的抗战刺激了中国在更多领域的现代化,比如科技的发展等,这对中国后来的发展有着重大的意义。③抗日战争重构中国的社会与政治状况。中国抗日战争的胜利是中国历史的重要起点,增强了“国家统一”的意识,奠定了中国共产党执政的基础。④抗日战争过程促使政府和民众之间的联系变得紧密。经过二战的洗礼,中国人对“国家”的概念加强,民族感情被激发。‎ 解析 回答问题,注重把握题干的具体要求。题干提出两个方面的内容:①中国在抗战中的贡献,②抗战对中国的作用。回答问题,注重把握米特的回答内容:米特的第一个观点是中国抗战在二战中起到重要作用,中国军民浴血奋战,牵制了大量的日本军队,中国最大的贡献就是坚持不懈地抗击日本侵略者;米特的第二个观点是中国的抗战构成了中国目前的社会以及奠定了共产党的领导,刺激了中国在更多领域的现代化,让中国的政府和民众联系更加紧密。‎ ‎8.[2017·成都外国语学校一诊]阅读下面的文字,完成(1)~(3)题。(12分)‎ 梵高:情之热烈 心之悲悯 丰子恺 梵高幼年的生活,只是亲近田园的自然。稍长,他就欢喜研究博物,常常离开了父母弟妹,独自赴田野中采集植物,或捕鸟类虫类,归家制成博物标本。他的性质从小是阴郁的,沉默而寡言笑。他的身材矮短,背脊稍向前屈。头发带红色,额上多皱纹,颜貌古朴,显然是一个富于冥想而内心深刻的人。八岁的时候,有一天他向一个美术家的助手索得一块黏土,拿来塑成一只小象,手法非常精巧,犹如学过雕塑的一样。又有一天看见一只花猫跳上庭中的苹果树,就在纸上描出它的活泼的姿态,笔致非常灵动。然而他这成绩不是技术的产物,乃热情的产物。他向来不习雕塑与绘画,一旦心有所感,形象就会得心应手地产出。他一生并未受过正式的绘画的基本教育,他的杰作都是随感兴而产出的。这特点从小就已显露。‎ 梵高有一颗强烈的宗教心。他在比利时的时候,已曾把所见的不能忘怀的情状,用真挚的态度描表为绘画。作业中的坑夫、小舍前的男子、拾石炭屑的女子……题材类似米勒而盛情比米勒更为激烈。‎ 自从父亲带他从比利时归家之后,他的身体虽然离开了那苦难的群众,然而他们的悲惨的幻象时时显现在他的眼前。“用绘画来表现!”这念头崛起在他的胸中。从此他把自己的全身奉献于绘画——其精神完全与从前的奉献于苦难的群众一样热烈。‎ 但父母亲不欢喜他描画,画布与颜料的钱也没有着落!在父母看来这不过是浪费。因为他的作画,完全不晓得迎合俗众的心理,完全抛却利害得失的念头,只晓得追求他的“真实”。家庭的圆满的和爱,从此有了缺陷,终于使梵高不能再留在家中了。‎ 梵高飘然地逃到海牙,先投宿于他的从妹家里,后又栖身逆旅,依靠乃弟和父亲的接济勉强维持生活。海牙是荷兰最特别的一座都市,在全欧中也是最富于原始趣味的地方。梵高用他的犀利的眼光,赏识其地的风物,又批判来自诸画家的艺术。他对于米勒尤富有崇敬之情,因为他与米勒同是用了宗教的敬虔的态度而进于画家的生涯的。‎ 他就作一幅素描,题名为《悲哀》。画中所描的,是一个病弱的女子把脸孔隐在两手中,俯伏在膝上哭泣。枯草一般的黑发垂在她的颈与肩的没有光泽的皮肤上。阳春已到人间,她的脚下有嫩草萌芽着,她的旁边有果树开花着;然而在她如同不见,只管哭泣,叹息。‎ 这画的模特儿,是一个抚育五个无父的小儿的伶仃孤苦的母亲。她每天叩访画家的画室,为他们当模特儿,拿所得的钱来养活六条性命。梵高有一次雇用她,听她诉说了她的长而悲惨的故事,热情的宗教徒兼画家十二分地感动,决意要为她减却一点苦恼。他慷慨地允许这女子,从明天起,由他担负她家六口的生活费。于是,梵高又将陷入像在比利时时候的穷困了。‎ 父亲得知了这消息,亲自来海牙探望儿子的情形。就和在比利时一样,带了他一同归家。‎ 新迁居的小村是织工所集居的村落,富有特别的诗趣。低矮的房屋,宽广的农场,夹道的胡桃树。有质朴的村人、园丁、农夫,劳动者们憧憧来往于其间。‎ 这些于梵高都是美妙的题材。每天朝出晚归,没有片刻的休息。为了采择画题,不问路的远近。普通人所欢喜的事物,往往是他的好题材。贫苦的人、拙陋的人,在他反觉得可爱。他能用同他们一样的朴素的调子,和他们攀交,请他们做模特儿。他对于模特儿,不但从丰报酬,遇到小孩子或老人的时候,又备糖果、烟草,亲切地款待他们。‎ 他的作品源源地产出。他所选的题材,第一是劳动者。他对于劳动者一向有好感;从比利时归来以后,这好尚更深,疲劳的人、忧愁的人、病苦的人,一直不离开他的脑际了。他的初期的作品,大部分是劳动者生活的深刻的表现。一切上流社会的人物,他都不描。因为他的心一向不被牵引向上流社会的方面。又因为他的经验范围限于下层阶级,对于下层生活的风俗、习惯、气质,都有彻底的理解。他用肉迫的态度来描写他们。不但表现他们的外部的形态,同时又描出他们的内部的生命。‎ ‎(选自《光明日报》,有删节)‎ ‎(1)下列对材料有关内容的理解和分析,不符合原文意思的一项是(3分)(  )‎ A.第一段运用类比和比喻的手法,以充满感情的笔触揭示了梵高狂放的性格、执着的信念和对生命的热情等精神特质,表达了作者对梵高的热爱与敬仰之情,奠定了全文淡淡的悲凉的感情基调。‎ B.想象力和创造力强。八岁时,用黏土塑小象,手法非常精巧,犹如学过雕塑的一样;看见花猫跳上果树,描画其姿态,笔致非常灵动。不习雕塑与绘画,心有所感,形象就会得心应手地产出。‎ C.善于思考,见解独到。在海牙,批判其他画家的艺术;吃饭时,思考画中细节;谈论文学的话题时见解高明,能使他人倾听;以下层社会为绘画题材,既能表现其外部形态,又能表现其内部生命。‎ D.走进“草根”的心灵深处。他乐于和底层的人们相处,并亲切地和他们交流,热情地招待他们,在画作中不但表现他们的外形,还描出他们内部的生命。‎ 答案 A 解析 奠定了全文深沉而热烈的感情基调。‎ ‎(2)下列对传记有关内容的分析和概括,最恰当的两项是(4分)(  )‎ A.父亲两次在梵高陷入穷困之时带他一同回家:第一次是他在比利时时,第二次是他在海牙担负起落难模特儿一家六口的生活费时。‎ B.梵高和米勒都是用宗教敬虔的态度作画,并且两人的作画题材相似;梵高对米勒怀有崇敬之情,而且对绘画投入的盛情更激烈。‎ C.梵高是一个极富美术天赋的人,八岁时就能以非常精巧的手法用黏土塑小象,用非常灵动的笔致描出花猫跳树的活泼姿态。‎ D.梵高的作品都是表现下层劳动者生活的,他的生活经验范围也仅限于下层社会,他对下层生活的风俗、习惯、气质都有彻底的理解。‎ E.梵高作画只晓得追求“真实”,完全不晓得迎合俗众的心理,再加上父母亲也不支持他的绘画,所以曾一度画布和颜料的钱都没着落。‎ 答案 AB 解析 C项中“极富美术天赋”无中生有,文中只提到“富于冥想而内心深刻”。D项中“都是表现下层劳动者生活的”错误,原文是“他的初期的作品,大部分是劳动者生活的深刻的表现”。E项中因果关系搞错了,应将“所以”提到“再加上”的位置并删去“再加上”三字。‎ ‎(3)请简要概括梵高在绘画领域能取得成就的原因。(5分)‎ 答:_________________________________________________‎ 答案 ①对绘画充满兴趣和热情。他从小喜爱绘画,并对创作充满激情,为了绘画不辞辛劳。②将生活敏感度付诸绘画,态度真挚,用心描绘。③坚持自己的爱好并为之不懈努力。家人不喜欢他绘画,绘画所需的物质条件也不充足,但在这样的情况下,他仍旧坚持自己的爱好。‎ 解析 解答该题,可以从原文提取信息并概括,如抓住文中的“他这成绩不是技术的产物,乃热情的产物。他向来不习雕塑与绘画,一旦心有所感,形象就会得心应手地产出。他一生并未受过正式的绘画的基本教育,他的杰作都是随感兴而产出的。这特点从小就已显露”“用真挚的态度描表为绘画”“但父母亲不欢喜他描画,画布与颜料的钱也没有着落……家庭的圆满的和爱,从此有了缺陷,终于使梵高不能再留在家中了”等内容概括,分点作答,意思对即可。‎ 第六部分 写作 学案一 审题之准——慧眼识真谛 一、材料作文审题五法 ‎1.关键把握法 审题立意,贵在既有深度又有高度,要做到这一点,就要善于抓住材料中的一些关键词句(“文眼”)进行反复推敲,理解意蕴,从而找到立意的突破口。‎ ‎【边练边悟】‎ ‎1.阅读下面一则材料,确定作文立意。‎ 高尔基《花——致M.A.彼什科夫》:“要是你在任何时候,任何地方,自己一生留给人们的都只是美好的东西——鲜花、思想、对你的非常好的回忆——那你的生活将会是轻松愉快的。那时候,你会感觉到所有的人都需要你,这种感觉会使你成为一个心灵丰富的人,要知道给永远比拿愉快。”‎ 这则材料的立意:___________________________________‎ 答案精析 这则材料的关键语句是:“要知道给永远比拿愉快。”若抓住这一关键语句,便可以提炼出观点:“人生的价值在于奉献,而不在于索取。”拟题时可用“奉献与索取”“给比拿愉快”等。‎ ‎2.主旨领悟法 一个写作者,若能够不为表象所迷惑,进而能够透过现象深入问题本质,便有希望步入事物的核心地带,其文章的思想、观点便会给人以启迪。‎ ‎【边练边悟】‎ ‎2.阅读下面的寓言《乌鸦喝水》,确定作文立意。‎ 一只乌鸦口渴极了,很想喝水。它看见了一只水壶就飞过去。壶里水很浅,乌鸦伸长了脖子也喝不到。后来它想了一个办法,用嘴衔起小石子,扔到水壶里,一颗颗小石子沉到水壶底,水渐渐升起来了。“啊,”乌鸦快活地说,“我喝到水了。”‎ 这则材料的立意:___________________________‎ 答案精析 这则寓言故事,透过乌鸦喝水的现象,解释蕴含的本质。它告诉我们,遇到困难时不要退缩,要动脑筋,想办法,然后努力不懈地去克服困难,终究会实现自己的愿望。我们可以取其本质,作为文章的论点。‎ ‎3.由果溯因法 任何事物的产生、变化和发展,都有其内在或外在的原因。新材料作文的审题,关键是准确地理解材料要义。而准确地理解材料的寓意、要义的最好方法是:按照“何人—何事—何果”的方式将其中心内容梳理出来,然后由果溯因,即可找到合适的写作角度。‎ ‎【边练边悟】‎ ‎3.阅读下面一则材料,确定作文立意。‎ 动物园里来了一位哲学教授,向动物们传授哲学。哲学教授讲了好多空洞的理论,接着说:“任何事物都必须从基础做起,就如任何建筑都必须从底层筑起。”有一只青蛙听得不耐烦了,就向教授发问:“请问教授,真的所有建筑都必须从底层筑起吗?”哲学教授瞄了青蛙一眼:“当然!井底之蛙!”青蛙反击说:“正因为是井底之蛙,我才问你——难道打井也从底层做起吗?”哲学教授哑口无言。动物们纷纷说:“是啊,即使是井底之蛙,他也有自己独特的见解,难道不是吗?”‎ 何人:_____________________________________________‎ 何事:_____________________________________________‎ 结果:_____________________________________________‎ 原因:_____________________________________________‎ 立意:_____________________________________________‎ 答案精析 何人:哲学教授;‎ 何事:说“任何事物都必须从基础做起,就如任何建筑都必须从底层做起”;‎ 结果:被一只青蛙驳得哑口无言;‎ 原因:(1)理论与实践脱离;(2)具体问题不能具体分析;(3)说话过于绝对。‎ 立意:(1)理论要与实践相结合;(2)具体问题要具体分析;(3)看问题不能太过绝对。‎ 也可以从青蛙的角度立意。‎ 何物:一只青蛙;‎ 何事:“请问教授,真的所有建筑都必须从底层筑起吗?”‎ 结果:把哲学教授驳倒;‎ 原因:(1)勇于质疑;(2)敢于挑战权威。‎ 立意:(1)要敢于质疑;(2)要敢于挑战权威。‎ ‎4.辩证拓展法 所谓辩证拓展,就是要利用辩证思维去展开思路,立意谋篇。辩证拓展,通常要运用以下观点:‎ 一是发展的观点。它要求我们分析问题时,应看到万事万物都在运动、发展、变化之中,论证不能绝对化。二是联系的观点。分析事物时,我们要从它与周围的千丝万缕的联系中找到问题的本质。三是一分为二的观点。它要求我们分析问题时,要正反通观、全面认识。‎ ‎【边练边悟】‎ ‎4.阅读下面一则材料,分别用“发展的观点”“联系的观点”“一分为二的观点”确定作文立意。‎ 材料 一个五年级的小学生,从别人那里偷了一个玩具。‎ ‎(1)用发展的观点分析,立意为:____________________________‎ ‎(2)用联系的观点分析,立意为:____________________________‎ ‎(3)用一分为二的观点分析,立意为:________________________‎ 答案精析 (1)用“发展的观点”分析这则材料,就要看到小学生以后的变化,可有两个写作角度:‎ ‎①从小偷针,长大偷金(坏习惯→坏行为→坏结果);‎ ‎②(经教育及时改正)将来可能成才(浪子回头金不换,今天的坏习惯并不意味着明天的堕落)。‎ ‎(2)用“联系的观点”分析这则材料,就需要多研究一下这个小学生的周围环境,从他坏习惯的形成与周围环境的联系上展开思路,探求小学生“偷”的深刻原因:‎ ‎①从小学生坏习惯看其家庭环境,父母是子女的榜样;‎ ‎②从小学生坏习惯看学校教育,学校应重视“养成”教育;‎ ‎③从小学生坏习惯看社会风气,社会是大课堂。‎ 由此揭示“责任在谁”的问题,进而强调学校、家庭和社会“三位一体”的教育的重要性。‎ ‎(3)用“一分为二”的观点分析材料,就要求我们在分析这个小学生“偷”的“不光彩”的行为时,应看到其另一方面,不能将其“一棍子打死”。‎ 一次偷不代表其本质坏。(平时,他可能团结同学,乐于助人,尊敬老师)‎ ‎5.多向发散法 新材料作文大都可以多角度立意。一般来说,材料中不同的人、事、物就是不同的立意角度,甚至主要的人、事也有多角度立意。当然,立意可以多角度,但我们写进文中的却只能有一个,因而,我们要从主要人物、主要事件角度立意,找出最有利于发挥,且易出彩的立意来书写成文。‎ ‎【边练边悟】‎ ‎5.阅读下面一则材料,试从多角度进行审题立意。‎ 有个四五岁的男孩,平时总爱反着穿袜子,父亲有时帮他把袜子好好地穿上,他还要自己脱下来,再翻个面穿上去。有一次父亲生气地问他:“你为什么总要反着穿呢?线头露在外面,多难看!”没想到男孩竟理直气壮地说:“袜子是我在穿,不是穿给别人看的,线头在里面,会使我的脚不舒服,我当然要把袜子翻过来!”‎ ‎(1)儿子角度:‎ ‎①_____________________________________________________‎ ‎②_____________________________________________________‎ ‎③_____________________________________________________‎ ‎(2)父亲角度:‎ ‎①_____________________________________________________‎ ‎②_____________________________________________________‎ ‎③_____________________________________________________‎ ‎(3)综合角度:‎ ‎_______________________________________________________‎ 答案精析 (1)儿子角度(关键词:自知、适合自己、坚持自己的做法、亲身体验)‎ 与儿子角度相关的关键语句是“袜子是我在穿,不是穿给别人看的,线头在里面,会使我的脚不舒服,我当然要把袜子翻过来!”这应该是本次作文立意的关键句。审题的核心要素是如何看待自我行为,“自己”一词为立意的关键点。从另一个角度看,材料还有一个如何看待他人与外物的角度,这就给学生留下了辩证思维的空间。‎ 解读此关键语句,可有以下立意:‎ ‎①为自己而活(不必活在别人的眼光下);‎ ‎②坚持自己的正确判断;‎ ‎③勇于打破常规,选择适合自己的;‎ ‎④只要适合自己,就应该被尊重;‎ ‎⑤坚持符合自己发展的做法;‎ ‎⑥人生如人穿袜子,苦乐自知;‎ ‎⑦别人只是观众;‎ ‎⑧快乐的感受因人而异;‎ ‎⑨实践出真知,行不行要靠亲身体验(实践是检验真理的唯一标准);‎ ‎⑩人要根据需要去控制物,而不能成为物的奴隶(物的价值在为人所用,而非为了好看)。‎ ‎(2)父亲角度(关键词:别人的感受、合乎习惯常规、站在孩子的角度)‎ 与父亲角度相关的关键语句是“有一次父亲生气地问他:‘你为什么总要反着穿呢?线头露在外面,多难看!’”父亲想引导孩子照常规穿袜子,孩子却不听从,父亲生气也是人之常情。但父亲也不能不尊重孩子的感受,不能一味要求孩子按惯例做事。毕竟,孩子的做法是建立在真实感受上的,看似违反常规,却不违反常理,对孩子的成长没有什么不好的影响,作为父亲有必要予以尊重,这对培养孩子的个性很重要。孩子的童真不应该被扼杀,只要孩子的举动不违背常理,只要适合其成长轨迹,都应该被理解、尊重,甚至引导、鼓励。因此,面对孩子的另类表现,家长怎样处理就是一门成长教育的高深学问。‎ 解读此关键语句,可有以下立意:‎ ‎①教育孩子时要尊重孩子的真实感受,因势利导;‎ ‎②人要努力适应、遵守社会规范(我们的行为举止要合乎习惯,顺乎常规);‎ ‎③要多考虑他人感受。‎ ‎(3)综合角度 人既要保有本真,尊重自我,又要努力适应社会规范。‎ 二、漫画式作文写作五法 ‎1.观察画面 仔细地读图,细心审阅。不放过任何一个细小的地方。‎ 观察细节包括画中人或物的情态动作、提示语中隐含的信息、人物所处的环境、与人物密切相关的“道具”等等;漫画中的人物形象有其特殊性(一般表现为夸张的动作、表情、语言),漫画的夸张之处、荒诞之处往往是漫画的真正寓意所在。要注意透过其独特的手法解读其现实意义。‎ ‎2.展开联想 要学会找事物之间的联系。一幅漫画往往由几部分组成,要把各个部分或者部分与整体联系起来看。有标题的(一般都有标题)不要看到一点就草草而作,从而漏掉了主要信息,要作具体分析,才能明白漫画的意义。要展开联想,如果通过联想,能把漫画引起的问题解答了,就具备了对漫画的解读能力。找出其讽刺、颂扬的对象或行为。‎ ‎3.把握寓意 不管写哪一类漫画作文,把握寓意都是第一关。一是要理解画面意义,正如阅读文字材料先要读懂文字的表现意义一样;二是看看漫画有哪一方面的隐含着的意义。漫画的画面意义与现实生活是有直接联系的。联系生活实际,揣摩漫画的画中之话,话外之音。‎ 要注意:画面的形象主体不一定是讽刺或颂扬的对象,要学会由“实”及“虚”,由表及里,然后挖掘隐含信息,进一步提炼概括画面所揭示的主题。‎ ‎4.确定立意 确定写作的立意点,不是泛泛而议,面面俱到,而是抓住一“点”,深入钻研。材料的标题对理解漫画寓意起导向和引领功能,特别是对一幅漫画可能表达多种意思的更是有着提示和画龙点睛的作用。画中字:画中人物的语言或是人物的“心灵独白”等,往往字数简约但意味深长,对正确领会漫画寓意很有帮助。‎ 要注意:一幅漫画的寓意可以多角度理解,在审读的时候理解的角度固然越多越好,以便打开思路;但在确定写作内容的时候,就必须选取某一个角度而“不及其余”了。否则,文章内容驳杂,头绪众多,是不利于表达的。‎ ‎5.构思成文 根据题目要求,结合主体,综合构思成文。联系生活实际,跳出画面,由此及彼地加以联想与引申,在画面意义的基础上联想到另一层新的内容,然后展开写作。‎ ‎【边练边悟】‎ ‎6.下面一幅漫画引发了你怎样的思考或联想?请根据你的思考或联想确定立意。‎ 春的景象,只有乍寒、乍暖、忽晴、忽雨是实际而明确的。此外虽有春的美景,但都隐约模糊,要仔细探寻,才可依稀仿佛地见到,这就是所谓“寻春”罢?有的说“春在卖花声里”,有的说“春在梨花”,又有的说“红杏枝头春意闹”……‎ 其实人生何尝不是如此?我们苦苦寻觅的人生之春,究竟在哪里呢?‎ ‎(1)观察画面,你从中看到了:‎ ‎_______________________________________________________‎ ‎_______________________________________________________‎ ‎_______________________________________________________‎ ‎(2)通过观察画面,你想到了:‎ ‎_______________________________________________________‎ ‎_______________________________________________________‎ ‎_______________________________________________________‎ ‎(3)把握漫画寓意,确立作文立意为:‎ ‎_______________________________________________________‎ ‎_______________________________________________________‎ ‎_______________________________________________________‎ 答案精析 (1)画面里有三个人物,两个在一起聊天的男子,一个卖花的女子;一条小狗随着卖花女;栏杆内外有发青的柳树、青草。画面的左上角有一排字:春在卖花声里。‎ ‎(2)按理说,图中既然已有“发青的柳树、青草”,说明春天已经来了,可为什么,漫画作者却说“春在卖花声里”呢?材料中也说,“春……实际而明确的”“春的美景隐约模糊,要仔细探寻,才可依稀仿佛地见到”,这就是所谓“寻春”,春在哪里?由“实”及“虚”,由表及里,由此引发对“探寻人生春天”的思考。‎ ‎(3)立意点基本可以确定,即“探寻人生春天”,人生的春天是怎样的春天,人生的春天在哪里,人生的春天如何探寻。命题意图旨在引导考生正确的人生观、世界观、价值观。这个春天可以是奋斗的,成功的(哪怕是失败后的反思),快乐的(哪怕是苦难后对人生的正视),感恩(感动)的,可以是一个人的、民族(复兴)的乃至人类(和平)的等等。‎ 三、材料作文审题四清 ‎1.理清对象 有些材料可能会涉及两个甚至两个以上对象,这几个对象之间如果无明显的主次之分,从理论上来说,每一个对象都可以提炼出至少一个观点。‎ ‎2.分清主次 有些材料可能会涉及几个对象,但这几个对象并不是并列的关系,其中有主次之分,那么,我们在审题时就应该分清主次,从主要对象入手进行分析,而不能是次要对象,否则有可能出现偏题现象。‎ ‎3.辨清关系 有些材料可能会涉及几个对象,而且这几个对象之间存在着一定的内在联系,审题时一定要辨析清楚这几个对象之间可能存在的关系。‎ ‎4.辨清含义 有些材料运用比喻或蕴含哲理,审题时我们首先应该认真分析,仔细揣摩,从而揭示出材料所蕴含的意义或道理,并以此作为立意的根据。‎ ‎【文题】 [2016·全国卷Ⅰ]阅读下面的漫画材料,根据要求写一篇不少于800字的文章。‎ 要求:结合材料的内容和寓意,选好角度,确定立意,明确文体,自拟标题;不要套作,不得抄袭。‎ 材料解读 漫画类材料作文,说简单也不简单,说难也不难。说简单,是因为只要把握了漫画的正确的、核心的寓意,作文的观点也就迎刃而“立”了。说难,是因为漫画的寓意并不容易把握,特别是漫画的各种符号所代表的精神寓意。‎ 对于这幅漫画,需要注意以下细节:‎ 第一,该漫画其实是由两部分构成,每一部分是由两个人物组成。第一部分是,一个孩子拿着100分的成绩单,眉开眼笑,脸上有爸爸妈妈亲吻过的痕迹;另一个孩子拿着55分的成绩单,愁眉苦脸,脸上有印迹清楚的巴掌印。第二部分正好相反,第一个孩子拿着98分的成绩单,愁眉苦脸,脸上有印迹清楚的巴掌印,第二个孩子拿着61分的成绩单,眉开眼笑,脸上有爸爸妈妈亲吻的痕迹。其中,四个分数、两个唇印、两个巴掌印所代表的意义,需要考生首先把握。‎ 第二,该漫画的两个部分是一个整体,而且两部分之间构成了对比关系,这就需要考生综合两个部分的信息,切不可人为割裂二者之间的关系。也就是说,第一个孩子的100分与98分之间,第二个孩子的55分与61分之间,形成了某种纵向的对比。‎ 第三,该漫画的每一个部分的两个孩子之间,其实也形成了某种对比。即第一个孩子的100分与第二个孩子的55分,第一个孩子的98分与第二个孩子的61分之间,也存在某种内在的对比关系。这四个分数并非漫画的作者随意设置的,而是有着某种含意,需要考生去把握。‎ 立意分析 这道作文题要求考生“结合材料的内容与寓意”进行写作。所谓材料的内容,就是漫画本身的外在表现,即画面本身的内容,而材料的寓意,则是漫画所反映出来的某种精神理念或现实观念。题目要求“明确文体”,其隐含的意思是文体不限,但一旦选择了某种文体,就应该按照该文体的形式规范进行写作。在这些要求的前提下,本材料作文可以从以下几个方面进行分析:‎ 第一,四个显眼的分数对比,巴掌印与唇印的对比,可以得出一个结论:对孩子的教育,切不可“唯分数至上”‎ ‎。这是大部分考生都能想到的一个观点。漫画中的人物显然是义务教育阶段的小学生,对于小学生的教育,很多家长眼里只有考试成绩与分数,对于孩子其他方面的表现例如身心健康、实际能力等,采取忽视的态度。这就是一个残酷的现实,漫画的首要意义就在于用夸张的形式批判这种现实。考生可以从这些符号入手,解读符号所代表的意义,总结出观点,并且结合现实进行分析。‎ 第二,如果再仔细分析这四个分数的话,我们还可以得出一个结论,即“标准化的分数”不仅限制了孩子的自由,同时也给家长带来了心理扭曲。在0~100的分数区间,有几个分数的标志,它们分别代表不同的知识与能力的等级,例如优秀的等级、良好的等级、合格的等级等等。其中最重要的两个标志,一是100分,这是满分,不仅意味着优秀,而且意味着完美无缺,是家长心目中最完美的综合实力的体现。第二个就是60分,这是合格能力的最低标准,60分及格,这是所有学生都知道的常识。那我们是不是可以得出这样一个结论:第一个孩子之所以愁眉苦脸,挨了家长的巴掌,正是因为偏离了“最高标准”;第二个孩子之所以喜笑颜开,得到家长的赞赏,正是因为达到了“最低标准”。从这个现象出发,我们也可以反思分数标准与实际的等级含义之间的关系,二者之间并不存在必然的逻辑因果关系。‎ 第三,如果我们着眼于同一个孩子的分数的对比的话,我们还可以得出一个结论:即现实生活中,特别是在学生的学习中,存在着一种“唯进步主义”的倾向。对于这种倾向,需要我们辩证看待。第二个孩子第一次考试得了55分,第二次考试得了61分,两次相比,成绩存在着明显的进步,面对这种进步,家长和孩子都开心,这无可厚非;对于第一个孩子,第一次考试得了100分,第二次考试得了98分,或许家长和孩子都不开心的一个原因是成绩退步了。或者说,家长真正恼火的并不是孩子没有拿到满分,而是与上一次考试相比,孩子的成绩退步了。于是一个问题出现了,当一个孩子的成绩已经处于高位时,不论是进步还是保持都是一件很困难的事情。这就要求我们能够辩证看待孩子成绩的进退,特别是在一个不能改变的大环境的前提下,用平常心和理智来看待成绩的进退,无疑是正确的做法。‎ 第四,如果我们着眼于分数与巴掌印、唇印的对比分析,是不是还可以这样分析:当一个孩子取得了满分或者不及格的分数,家长的做法就是简单的表扬或批评,甚至使用暴力手段进行批评;当一个孩子取得进步或者成绩退步时,家长的管理手段也是如此。也就是说,目前很多家长在教育孩子时,只会使用简单的表扬或粗暴的批评等教育手段,或者说,这些都不能算是真正的教育手段。这幅漫画其实也是在批判家长在教育孩子时,不能使用真正的教育方法,不能真正与孩子进行心灵沟通,而总是运用这些感性的手段。也就是说,这幅漫画从侧面反映出了当前家庭教育中存在的一些问题。‎ 第五,当我们着眼于孩子的表情与分数之间的对比时,我们也可以进行如下分析:悲观主义者往往只能看到自己的不足之处,当自己退步时,看不到自己的真正实力;乐观主义者往往能够看到自己的长处,当自己进步时,即使这种进步依然没有达到很高的层次,却依然坦然面对。这也启示我们,要善于培养积极乐观的心态,让孩子在一个宽松自由的环境中健康成长。‎ 佳作欣赏 勿以点滴沉浮论英雄 ‎“爱其子,择师而教之。”今人之于古人,更为注重孩子的教育。然而,不少父母把重视化为对于点滴成绩的纠结,平日里的“天之骄子”因点滴失误而被“由爱转恨”者有之,一直的“差生”因点滴进步而顿被“捧在手心”者亦有之。父母的态度因一点点分数改变而“一百八十度”大转弯,这是对待成绩过于主观、生硬的行为。‎ 分数的沉浮本就有多重原因,孩子的掌握、思维方式、老师出题的难度,甚至于个人的运气皆可改变成绩的高低。仅以“这次比上次高了几分”来界定孩子是否认真学习是不客观的、表面的。高分学生的父母看不到孩子一直以来的勤奋与不缀,低分学生的父母不能认识到孩子的能力与水平。单次成绩的沉浮即界定英雄狗熊、评定父母心中的“宝”与“草”,难以认识到孩子的水平,终会使孩子深陷那红色的分数中,不清醒且不理智。‎ ‎《礼记》有言“父之爱子,乃生而行之乎。”父母的爱本是无条件,而现今因分数一点点退步或增加均可改变父母的爱,则此爱易流于表面而不触之心。孩子退步了两分即挨了巴掌,或许便是错罚了试题变难仍是班级第一的他。孩子学习如同苦行僧在路上踱步前进,这一路上,父母的关爱与理解若如“春风潜入夜,润物细无声”一般,会让前路阳光明媚。而若父母的爱变得“有偿”,须用分数进步来赢得,只会让孩子战战兢兢,时刻背负“优生的包袱”或者“差生的重担”。‎ 古人云:“爱其子,则为之计深远。”‎ 父母看待孩子的眼光不应局限于那小小的分数。考试是对孩子学习的评估,并非是对孩子的评估。父母看待孩子分数的眼光也不应局限于一隅,而应用整体的、比较的眼光来看待,这样才能知道成绩背后那点滴浮沉的真正意义。‎ 父母的“巴掌与吻”是可影响孩子一生的。曾几何时武汉神童惊人跳级、学业有成,而父母惯于其优异,容不得半点失误或退步,他终顶不住压力而厌学,“泯然众人矣”。‎ 成绩那点滴浮沉并不能直接界定孩子的“好坏”“优差”,也不能作为孩子赢得父母的爱的筹码。只有看到孩子真正的实力,给予及时支持,方能在无涯学海上为他撑起一叶小舟,助他学成至彼岸。‎ 出彩理由 本文是考场中一篇精彩的议论文。首先,作者能准确地解读漫画的寓意。题目“点滴浮沉”四字用词非常精妙,体现了该考生对漫画的全面评价。全文紧紧围绕着“点滴浮沉中的成绩”所存在的问题展开议论。文章思路清晰,从分析“只看分数浮沉”的错误观点给孩子造成的危害,进而能高屋建瓴地提出教育的本质应在于看“浮沉分数”背后的真正意义,体现了考生极强的逻辑思维和思辨能力。其次,文章最为精妙之处,还在其语言文字的亦雅亦俗,畅达准确,分析的严谨冷静远远超出了一般的考生,极富说服力。成语的运用顺手拈来,比喻的句子生动形象,“父母之爱子,则为之计深远”等经典语句的引用,更能表现出该考生丰富的知识储备和比较深厚的文化底蕴,以及驾驭素材的综合能力。‎ ‎1.阅读下面的材料,给出准确的立意。‎ 材料 一只蜗牛,很想做成一番惊天动地的事业,开始它想东游泰山,一直爬到山顶,可一计算,要实现这个目标,至少需要3000年时间,只好忍痛放弃这个打算。后来它又想爬到长江边上,看一看奔腾的江水,可一计算,要实现这个目标,至少也需要3000年时间,蜗牛知道自己生命短暂,不禁十分悲哀,于是什么也不肯做,最终死在草丛中。‎ 立意:‎ ‎_______________________________________________________‎ ‎_______________________________________________________‎ 答案精析 (示例)①立志、选定目标时要符合自身实际,不能好高骛远。②做事要脚踏实地,从平凡的小事入手来实现自己的生命价值。‎ 审题:这是一则寓言故事,首先概括主要情节:主人公蜗牛想干什么?它干了没有?最终是什么结果?‎ 分析材料,明确:蜗牛想干惊天动地的事,可因为两个目标都太遥远,使宏伟的计划流产。放弃了大目标后的蜗牛十分悲哀,什么也不肯做,一蹶不振,最后死于草丛。‎ 造成蜗牛悲剧的原因是什么?①蜗牛选定的目标太遥远了,以他的能力根本无法实现。②蜗牛放弃了大的目标后,什么也不肯做,最终一事无成,死在草丛中。‎ 从蜗牛联系到人,就可以理解故事的寓意了。‎ ‎2.阅读下面的文字,根据要求写一篇不少于800字的文章。‎ 玫瑰说:“我只有在春天开花!”‎ 日日春说:“我开花的每一天都是春天!”‎ 要求:(1)自选角度;(2)自定立意;(3)除诗歌外,文体不限;(4)不得脱离材料内容及含意的范围作文。‎ 立意分析 审题:在审读上面的材料时,首先要审明材料蕴含的观点、态度。“我只有在春天开花!”中的“春天”,代表美好的时期,“只有在春天”隐含着与“春天”相对的“夏天、秋天和冬天”。既然此话中的“春天”代表美好的时期,那么,“夏秋冬”就代表“不美”的时期。分析到此,“玫瑰花”的生活态度就显露出来了:在美好的时候开放,而在不美的时候就不开放。换句话说,像玫瑰花这样的人只有在优越的条件下才能快乐地、幸福地生活,否则,就会陷入痛苦之中。而“我开花的每一天都是春天”是说,只要“我”开花,外部环境不管是春天,还是夏天、秋天或冬天,在“我”看来,它都是美好的。说得更明白些,像日日春这样的人,只要他生活着,不管生活怎样,但在他看来,一切都是合理的、美好的,无论在什么情况下,都不会“以物喜,以己悲”‎ ‎。通过分析,两种不同的人生观就明确地显现了出来:消极、悲观、低沉和积极、乐观、向上。两种人生观形成了鲜明的对比,在对比中,我们就能够很容易地看出命题人的观点,即赞成什么,反对什么。由命题人的观点态度可以进一步推论:人生观决定人生命运。以这两个不同角度的观点为基础,可以引出很多话题,如微笑、态度、心态、命运、乐观与悲观、幸福与痛苦、心态与生活等等。‎ 佳作欣赏 谁打开你的窗 一片花瓣飘进来,落在你的手边,落寞的你嗅了嗅。“哦,春天来了,是玫瑰吗?玫瑰只在春天开花。”‎ ‎“不,是日日春,一种很美的小花,不太选择季节,总是很积极很美好地开放着。她开的每一天都是春天。”我说。‎ ‎“池塘里的冰该化了吧?柳树是不是也发芽了?我闻到了油菜花的香味,听妈妈说油菜花是金黄色的,我想知道金黄色是不是给人温暖的颜色……,”你好像说不下去了,声音哽咽起来。“金黄色就像你发卡上的蝴蝶一样美丽,也像你衣服上的毛毛熊一样让人温暖。”我安慰着,幸好你看不见我心疼你的眼,要不你一定会哭出来。‎ 你摸索着打开窗,做出望的姿态,你说你想看看春天的颜色,嗅嗅春天的气息。我知道你的渴望,可是更多的时候我跟你一样保持沉默,我不知道该说什么,任何的语言描摹对你都是一种残忍的折磨。风铃响了,那是阿姨送给你的生日礼物。你让我告诉你风铃的颜色,于是我为你描述着风铃是天空一样透明的蓝色,我安慰你风铃会带你入梦,你终于忍不住哭了,你说:“我的梦里一片漆黑,找不到出口,就是一场噩梦。”我落荒而逃,像一个想为人开窗却打坏了玻璃的孩子。‎ 一年之后,又是一个春天。我去看你,你好像变了一个人,开朗热情。你说:“你走之后,我在窗前种下了日日春,日日春开花的每一天都是春天,现在我也在我的春天里开放着。”我注意到你手中的盲文书,原来你上学了。蓝风铃随微风飘动,快乐的音符奏着轻灵的音乐。我看到你在书的封面上写道:如果可以,我要变成天空中的一抹云彩,我把风铃捎给小窗,让她为你我歌唱;如果可以,我要变成大海中的一条小鱼,我把浪花推向沙滩,让沙滩变成诗行;如果可以,我不选择玫瑰,我要变成日日春,让每一天都绽放我的美丽……我的眼里噙满泪水,你的窗户已经打开。‎ ‎“真美,”我第一次不是安慰而是惊喜地望着你说,“真美,你用春天般美好的心灵看到的比许多人用眼睛看到的更多更美。”‎ ‎“是谁为你打开窗?”我问。‎ ‎“是日日春带给我春天!”你回答。‎ 出彩理由 本文构思新颖,立意准确,文笔细腻流畅,感情真挚自然。通过两个具有朦胧意境的画面描写,来向我们展示乐观进取的日日春所带给盲姑娘的积极影响,从而也启发我们每个人都应该打开自己的心窗,让乐观与进取永驻心田!‎ 学案二 扣题之紧——行文常记钩联 一、扣题的要求 ‎1.扣题句一定要有 一篇文章中,必须要有点明主题或话题的词语或句子,用意就在于提示阅卷老师你没有跑题,你是一直紧扣主题或话题来行文的,即便你有几行文字偏离话题中心,但只要在关键的地方出现了点题句,老师在快速阅卷中,也会觉得你是扣题的。‎ ‎2.扣题要准确到位 表达中心的语句要突出显豁。在行文时,要用准确的语言清晰地表达出自己的思路观点。把表达中心的语句放在突出的位置,让阅卷老师一目了然,能清楚地了解作文的观点或主旨。‎ 二、扣题的类型 ‎1.字面重复明点 这是形式上的扣题,是外显的。作文内容与文题或材料要有明显的相似语句。如《甘瓜抱苦蒂》作文片段:‎ 漫步于历史的走廊,我看到了许多圣贤用自己的言行证明了这一真理。孙子膑脚,这是苦;《孙膑兵法》流传下来,造福后代,这是甘。司马迁受宫刑,这是苦;《史记》被称为“史家之绝唱,无韵之《离骚》”,这是甘。李煜国破家亡,这是苦;百千佳作留千古,这是甘。马克思生活艰难,食不饱,衣不暖,这是苦;创造马克思主义,领导工人阶级革命,这是甘。周恩来总理日理万机,这是苦;换来国家繁荣人民幸福,这是甘……历史用它自己独特的方式证明了甘与苦是相生相伴的。‎ 在这一文段中,作者援引史例,连用几个简单的判断句,用最明晰的语言明确地说明了什么是“甘”,什么是“苦”,而且,“甘与苦是相生相伴的”与题目“甘瓜抱苦蒂”极相似,点题清楚,扣题紧密。‎ ‎【边练边悟】‎ ‎1.阅读下面的习作《人之常情》的开头与结尾,分析它是如何扣题的。‎ 开头:激动是人之常情的体现,痛苦是人之常情的体现,愤怒是人之常情的体现,宽容是人之常情的体现。……人之常情就是人一生的宝贵财富。‎ 结尾:人生在世,谁都有感情,谁都会宣泄感情。世界因人之常情而更美丽,更温馨。‎ 人之常情,是人一生的宝贵财富。‎ 扣题情况分析:‎ ‎_______________________________________________________‎ ‎_______________________________________________________‎ 答案精析 习作的开头和结尾,均紧紧围绕“人之常情”来写,明确照应了标题,且前后呼应,结构严谨,浑然天成。‎ ‎2.似曾相识暗扣 这是在立意上扣题,是内隐的。作文内容与文题或材料要有本质的联系和契合,而不是貌合神离。如作文《甘瓜抱苦蒂》,我们可以在开篇这样扣题:‎ 黄山松是痛苦的,因为它的根部没有土壤,只有岩石,因此它必须十分艰辛地深入到岩石的内部去吸取有限的养分;但黄山松也是甜蜜的,因为它能“倚绝壁”,得到了人们的赞美与憧憬。‎ 珍珠贝是痛苦的,因为它要孕育珍珠就必须忍受那锥心沥血、撕心裂肺的痛苦;但珍珠贝也是甜蜜的,因为它孕育了一颗价值连城的珍珠,受到人们的青睐。‎ 取黄山松、珍珠贝两物拟人化进行类比而论证,妙!“痛苦”“甜蜜”二词不露痕迹镶嵌其中而扣题,巧!‎ ‎【边练边悟】‎ ‎2.阅读下面的习作《绿色生活》中的三个片段,分析它是如何扣题的。‎ ‎①那是一块地,一块青草地。那时的生活是绿色的,因为,有这块地。‎ ‎②总怀念那天高云淡、芳草青青的日子。曾几何时,那片绿地躺在城市的近郊,尽情地呼吸,恣意地生长,没有人能打扰它的安宁,它的欣喜,它以为可以一直这样下去,蓬勃而淡然,我也以为。‎ ‎③那片地上什么草都有,参差而杂乱,远比不上城里的景观草坪美观,但我总觉得这里多了些什么,一种自然,一种蓬勃,一种生命的本真。‎ 扣题情况分析:‎ ‎_______________________________________________________‎ ‎_______________________________________________________‎ 答案精析 第①段直接入题,“生活是绿色的”,与题目语意相同,仅表述形式不同。②③两段虽未直接点题,但已暗扣题旨描述“绿色生活”。第②段描述“绿色”,第③段揭示“绿色生活”的内涵。‎ ‎3.浓墨重彩反复扣 写作时,我们用作扣题的话语往往不止设置一两处,而是步步勾连,时时扣题,似曾相识的词语、句子反复出现,不变的是基调,变化的是细节,不断强化主题,又在变化中给人丰富生动的感受。表现在形式上,就是在作文的多处反复点题,点题的形式每次都有变化,这样才显得机智圆熟。‎ ‎【边练边悟】‎ ‎3.阅读下面的习作《心有天使,诗意人生》中的三个片段,试分析它在点题扣题方面的技巧。‎ 是谁本着一颗报恩的心,是谁守望着偏僻的山村?徐本禹,是什么驱动着你奔向贫瘠的大山?是爱的天使,是一种对社会报恩的责任感吧!……‎ 是谁奔走在田垄间行医,是谁一心拯救人民?是什么驱动着你奔走在田垄间?你无怨无悔地为人民医治,你是一个如花一般美丽,如水一样灵动的天使。李春燕!……‎ 是谁在舞台上用心灵独舞,是谁于无声处展现生命的蓬勃?邰丽华,你于无声处展现生命的蓬勃,你的“开花衽”绽放出的花儿是那么骄艳,那么动人,你心中一定有自己雕琢的天使,一个让当今中国人民感动得流泪的天使,是吗?‎ 扣题情况分析:‎ ‎_______________________________________________________‎ ‎_______________________________________________________‎ 答案精析 每一段的首尾用字面相同的句子重复标题和中心,反复点题扣题,形成排比,从而达到了突出中心的目的。‎ ‎【文题】 [2016·全国卷Ⅱ]阅读下面的材料,根据要求写一篇不少于800字的文章。‎ 语文学习关系到一个人的终身发展,社会整体的语文素养关系到国家的软实力和文化自信,对于我们中学生来说,语文素养的提升主要有三条途径:课堂有效教学、课外大量阅读、社会生活实践。‎ 请根据材料,从自己语文学习的体会出发,比较上述三条途径,阐述你的看法和理由。‎ 要求:选好角度,确定立意,明确文体,自拟标题,不要套作,不得抄袭,不得泄露个人信息。‎ 材料解读 本作文题的材料主要内容为:‎ ‎1.语文学习与语文素养 材料主要围绕语文学习和语文素养展开,谈到了个人发展、整个社会、国家的软实力和文化自信等问题。个人的语文学习可以促进个人发展和提升个人的语文素养,每个个人的语文素养的提升有助于整个社会的语文素养的提升,进而提升国家的软实力和文化自信。‎ 材料中提到的国家软实力即一个国家的文化实力。它是一国综合国力的重要体现,具体包括文化、教育、国民的心态、国民的形象、中华民族精神以及文化产品等,更甚至语言也可看作是一种文化实力。中国在实现民族振兴的道路上以经济建设为中心,过度依赖经济利益来解决问题,导致国家的文化软实力远远落后于其他国家。《人民日报》英文版2006年的一期中写道:作为一个拥有5000多年文明史的文化发源地,中国如果只出口电视机,不出口电视机播放的内容,也就是不出口中国的思想观念,那么中国就成了一个“硬件加工厂”。事实上,文化是综合国力的重要组成部分,也就是人们所说的“软实力”,它已经成为各国竞争的焦点。中国有着以和为本的价值观和依靠德行赢得尊重的传统,在文化多元化、全球化的时代有着巨大的吸引力。‎ 十八大以来,习近平总书记在多个场合谈到中国传统文化,表达了自己对传统文化、传统思想价值体系的认同与尊崇,也多次提到核心价值观和文化自信。材料将语文学习、语文素养放到了国家政策的层面。‎ ‎2.提升语文素养的途径 课堂有效教学——这一途径强调的是语文素养的提升,课堂是关键。考生要明白,课堂不仅教授学生掌握基础知识、阅读与写作的方法和技巧以及语言的综合运用等,更能让学生接受优秀的文化熏陶。但是,也应该注意到课堂教学的有限性,课堂教学是我们提升语文素养的一部分,但不是全部。‎ 课外大量阅读——书籍是人类进步的阶梯,阅读是提升语文素养的首选。书籍作为一种间接经验,可以让我们更好地、更快地感受到语言的魅力以及生活的丰富性。但考生依然要辩证地看待问题,大量的课外阅读固然能够很好地提升语言素养,但过多、太杂的书籍也会降低我们的语文素养,在这样一个自媒体的时代,我们更是要注意。‎ 社会生活实践——语文来自于生活,生活处处皆语文。语文素养的提升离不开生活,社会生活实践可以为我们提供丰富多彩的生活体验和永不枯竭的素材。如果考生从这一途径入手,同样需要明白,社会生活实践虽然是语文知识永不枯竭的源泉,但是很多时候它无规则、无技巧,单靠自我的觉悟是远远不够的。‎ 本作文题的材料要求为:‎ ‎1.请根据材料,从自己语文学习的体会出发。‎ 这就要求考生在写作时围绕材料中的核心事件就事论事。也要求考生不要泛泛而谈或一味地拔高,而是要结合自己的语文学习体会,从己入手,由小到大,理论与实践相结合。‎ ‎2.比较上述三条途径,阐述你的看法和理由。‎ ‎“比较”二字就告诉考生,这三种途径都可行,哪一个更有效呢?这就需要考生在写作时运用对比的方法作文,在对比中阐述自己的看法和理由。值得注意的是,考生在做出选择后,主要笔墨应该集中在更有效的那个途径上面,而不应把大量的笔墨放在否定论述上,否则,中心将难以突出,主题也就不够鲜明了!‎ 立意分析 根据前面的分析,我们已经明确考生在作文时必须在三种途径中进行权衡和选择,考生可以从肯定课堂有效教学,并在某种程度上否定课外大量阅读和社会生活实践来立意,也可以从肯定课外大量阅读,并在某种程度上否定课堂有效教学和社会生活实践来立意,抑或可以从肯定社会生活实践,并在某种程度上否定课堂有效教学和课外大量阅读来立意。‎ 佳作欣赏 花气袭人知昼暖 欲知四季,去山野吧,看抽芽的嫩柳,看金黄的麦穗,看累累的硕果,看白了青山的雪,眼知窗外美。‎ 欲知路远,就出发吧,走悠长的夕照小巷,走古朴的木桥,脚知漫道长。‎ 欲知文学,来人间吧,看爱恨贪嗔,看嬉笑怒骂,看王公贵胄,看布衣黔首,心知世界大。‎ 以清晨壮丽恢宏的半边云霞起兴,以赶路人脚下不停生长的风为修辞,以公交站牌前偶遇的笑脸为标点,以温馨午餐氤氲的香气为内容,以一对老夫妇互相搀扶的背影为结局,以摇尾跑来的小狗为句号,洋洋洒洒一篇以“语文”为题的文章已挥笔写就。生活就是语文,叫作“幸福里”的招牌,“爱护自然”的温馨提示,充满希冀的电话号码,无一不是语文的化身。‎ 曹雪芹写《红楼梦》,有人说他是写自己,在富贵家庭里养尊处优,一场冰冷的大雨浇灭了所有骄傲,他在破败小屋里衣衫褴褛,在萧瑟寒风中饥寒交迫,在浊酒昏灯下增删批阅,他所经历的,就是最好的素材,他用他的脚印,缀满了大观园所有人的悲欢。‎ ‎“纸上得来终觉浅,绝知此事要躬行。”生活是最好的老师,生活中的点滴汇聚,终成语文壮阔的海洋。向来喜欢语文,所以在生活中处处留心,也许是母亲一句温暖的关怀,也许是路人一句友好的提醒,也许是演讲者或激昂或抒情的言语,也许是相声演员幽默生动的段子,也许只是几个字,都可以触动灵感的源泉,目光所及之处,生活所经之事,尽是好文章。‎ 我们是尘世中蹒跚而行的赶路人,三毛说:“心若没有栖息的地方,到哪里都是在流浪。”语文便是我们心灵的栖息地,我们将生平中见过的或纤弱或雍容的花别在语文的衣襟上,我们将生活中听到的或低吟或高啸的言语缀在语文的耳后,我们将生活中嗅到的或馨香或馥郁的气息扑洒在语文的发梢上,这样,被生活悉心装扮过的语文便亭亭而立。‎ 语文教我们品味生活,生活教我们学习语文。从最初的咿呀学语,到以后的执笔写字,到后来笔下开花,随着我们一步步成长,生活向我们展示了语文更多的魅力和无法替代的重要性,如同喝一坛甘醇的老酒,越饮越醉人,在香气的熏陶下,我们情不自禁地想要再次一品佳酿。‎ 曾有诗云:“花气袭人知昼暖。”提高语文素养又何尝不是如此呢?唯有立足于生活实践的沃土,语文之花才能盛放。‎ 出彩理由 这是阅卷组评定的一篇考场标杆佳作。本文以中心突出、论证严谨、颇具文采而被评为一类卷。具体特点如下:1.主题鲜明,形散神聚。这篇议论性散文,与众不同之处在于开篇没有直接提出论点,而是通过一组排比句,将生活与文学也就是语文自然而然联系了起来,再通过层层论证,将语文与生活的关系揭示得清清楚楚,文末还在强调语文必须根植于生活。2.论据充分,论证缜密。文中用曹雪芹的事例论证生活是写作的素材,引用陆游的诗句论证“目光所及之处,生活所经之事,尽是好文章”,用身边的事例揭示语文和生活的辩证关系,论据充分,论证有力。3.手法多样,文采飞扬。文中多用排比、比喻等修辞手法,为文章增色不少。‎ ‎1.请根据提供的文字补写,要求:①紧扣观点,②形象点题。‎ 聆听生命的乐章 每天清晨,当地平线分娩出一轮红日,我迎着那热情奔放的第一缕晨光;每天夜晚,当月亮羞涩地闪动她的光华,我作别了远去的晚霞,也享受着夜幕里阒无一人的安详。我来到海边,朝晖中我漫步在松软的沙滩上,夕阳中我聆听惊涛拍岸的鸣响。(1)________。(开头)‎ ‎…………‎ 青春是美丽的,她奏响生命中最华美的乐章。但我不能长醉于满园春色中,我要感受那酷暑骄阳炙烤着我的脊梁,我要让那岩洞凛冽的寒风吹干我憔悴的面庞。春荣秋谢是自然的法则,但成功与希望奏响了生命的乐章。‎ ‎(2)____________________________。(结尾)‎ 答:(1)_________________________________________________‎ ‎_______________________________________________________‎ ‎(2)_____________________________________________________‎ ‎_______________________________________________________‎ 答案精析 (1)日月更替,潮起潮落,我仿佛在其中找到了这天地间生命的节奏,听到了世间最动人的乐章。‎ ‎(2)从不放弃,永不凋谢,任清风与暴雨在我胸中跳荡,这才是生命的最高亢的音符,最动听的乐章。‎ ‎2.请根据提供的文字补写后文,要求:①紧扣观点;②形象点题。‎ 梦想在现实中起舞 ‎…………‎ 在那个人人埋怨的时代,沈从文先生目瞩现实的残酷,却依然将那个江南小城写成了山美、水美、人美的世外桃源。现实没有湮没他对人生的希望,他用一份最原始的情感和一颗赤子之心看待这个社会,看待自己的人生。他没有沉醉于自己的幻想亦没有让现实麻木自己的心灵。‎ 张允和先生亦是一位智慧的老人,她一生经历了大富大贵,也经历了战火纷飞、十年浩劫,而她却永远保有一副悲天悯人的情怀、一颗永不衰老的童心。她那悲天悯人的情怀使她正视现实并战胜现实,而她那颗永不衰老的童心则使她在任何艰难的情况下都不放弃幻想和权利。‎ 川端康成浅浅的一句“凌晨四点钟,看到海棠花未眠”瞬间感动了多少心灵。这是梦和现实完美的结合,让那些世俗之物顷刻间土崩瓦解,让多少在现实中日渐麻木的心灵得到了温暖。‎ ‎(例后分析):‎ ‎_______________________________________________________‎ ‎_______________________________________________________‎ 答案精析 (示例)正视现实,但不委身于现实;保持幻想,但不沉溺于幻想。让梦想在现实中跳一曲酣畅淋漓的舞蹈吧!‎ ‎3.阅读下面的材料,按要求完成题目。‎ 员工对老板说:“讲到勤奋,你不如我;说到成功,我根本不能和你比,为什么呢?”‎ 老板回答说:“我曾经也很勤奋,但那是我年轻时候的事了,那时我是为老板工作,我比你现在要勤奋刻苦得多。在任何社会里,大部分人都很勤奋,但成功者极少。我想,既然大家都很勤奋,难道缺我一个吗?所以我不再埋头勤奋,而是思考着勤奋,后来就成功了。现在我是为自己工作,我的成功之道就是想方设法给更多的人提供机会,让别人勤奋,而不是要我比别人更勤奋。”‎ 所以说,在现今社会,与其默默无闻地埋头苦干,不如多动些脑子。‎ ‎(1)根据材料,给作文拟一个形象的标题。‎ 答:___________________________________________________‎ ‎(2)根据所拟标题,给全文设计三个分论点。‎ ‎①_____________________________________________________‎ ‎②_____________________________________________________‎ ‎③_____________________________________________________‎ 答案精析 (1)(示例)给勤奋插上思考的翅膀/扬思考之帆,启生命之舟/忙碌而不盲目/我想飞得更高 ‎(2)(示例)①“勤奋不一定能成功,但不勤奋就一定不能成功;②勤奋只是成功的条件之一,而不是唯一;③勤奋+思考=成功。‎ 学案三 语言之美——提高作文“颜值”‎ 一、引用诗词名句依然是“增分点”‎ ‎【边练边悟】‎ ‎1.品读下面的文段,说说这些文段在语言方面有何特点。‎ ‎(1)林徽因曾说:“真正的宁静不是避开车马喧嚣,而是在内心修篱种菊,尽管如流往事,每一天都涛声依旧。”真正的宁静是内心的平和,这与“大隐隐于市”是一样的道理。只要内心宁静,便能于车马喧嚣的繁华都市有一个宁静之所。(高考优秀作文《非宁静无以致远》)‎ 答:___________________________________________________‎ ‎_______________________________________________________‎ ‎(2)缪尔曾言:走向外界,我发现,其实是走向内心。梭罗扛把斧头,在树林间自给自足地诗意栖居。他修剪门前花树,正如他刈除日常生活中的繁文缛节,如同大自然般自由呼吸与生活。而我们呢,走向支离破碎的外界,确实发现它是我们浮躁内心的外化和映照。当自然之光不再照亮未来,人心将在黑暗中徘徊。(高考优秀作文《蝶失》)‎ 答:___________________________________________________‎ ‎_______________________________________________________‎ 答案精析 (1)引用名言+分析论证。首先根据论点的需要,引用恰当的名人名言或古诗文,然后用自己的语言进行分析阐述,搭建起引文与论点之间的桥梁,彰显深度。‎ ‎(2)精选恰如其分的引言,之后再选择历史人物,用他们的经历来印证引言,使得引用论证与举例论证水乳交融,更有利地突出中心论点。‎ 二、雕琢细节,于细微处见精神 ‎【边练边悟】‎ ‎2.品读下面的文段,说说这些文段有何特点。‎ 他的右手正奋力地鼓着风箱,简陋的炉灶上火苗跃动。一旁的小炉灶上支着一口锅,清净的汁液正烧得噼啪作响。细细听来,寰宇间仿佛只剩下火苗炙燃的嗞嗞声。轻轻一嗅,糖汁溶化的丝丝甜意直润肺腑。‎ 炒米匠望着压力表,轻轻一唤:“要炸喽!”平淡中渗出一丝威猛,把一个三四岁的小女孩吓得哇哇直哭。炒米匠嘴角轻咧,围观的大人倒是笑得前仰后合。只见他娴熟地把炉膛塞入布甲中,手中一根杵棒猛力一拨,“嘭”的一声,白烟缭绕。那孩子破涕为笑。脆脆的香气混合着糖汁软软的清甜让人心旷神怡。(高考优秀作文《薄暮依然炒米香》)‎ 答:___________________________________________________‎ ‎_______________________________________________________‎ ‎_______________________________________________________‎ 答案精析 (1)全方位刻画人物。写记叙文,要时时刻刻提醒自己,在情节发展的过程中,有必要从语言、外貌、神态、行为、心理等各方面去刻画人物,使笔下的人物立体丰满、有血有肉。比如选段就着力描写了炒米匠娴熟的动作、个性化的语言以及神态,让读者仿佛亲眼观看了他爆炒米的过程。‎ ‎(2)细化环境描写,助力作文。环境描写的作用不容小觑,但确是很多考生容易忽略的细节。汪曾祺说:“气氛即人物。”选段调动视觉与听觉,“描写火苗炙燃的嗞嗞声”以及“脆脆的香气”,渲染了爆炒米时的氛围,烘托了人物愉悦的心情,同时也写出了炒米匠给人们带来的欢乐。‎ 三、接地气,摒弃过古人的“复活节”‎ ‎【边练边悟】‎ ‎3.品读下面的文段,说说文段有何特点。‎ ‎(1)索尔仁尼琴曾说:“宇宙有多少生命,就有多少中心。”‎ 可多数人并未意识到,物质迅速膨胀,世界空前繁荣,这一排光鲜场景背后却是伤痕累累的自然。(高考优秀作文《蝶失》)‎ 答:___________________________________________________‎ ‎_______________________________________________________‎ ‎_______________________________________________________‎ ‎(2)杨绛先生是一位大家闺秀,但她一生都因悲悯之心与平凡的劳苦大众建立友谊。她写《老王》,写一名车夫的不幸,写《林奶奶》,写一名帮佣的坚忍。我相信,每个人都原有杨绛先生一样的善良,只是在一次次沟通不畅的付出爱的行动中,灰了心。(《细心呵护生命》)‎ 答:___________________________________________________‎ ‎_______________________________________________________‎ ‎_______________________________________________________‎ 答案精析 (1)开拓“海外市场”。数据显示,一部分考生已经将引用或举例的范围投向了“海外市场”,这也不失为一种避免“撞车”的好办法。文段引用索尔仁尼琴的名言,虽然他是“大咖”,但是深入了解他的考生并不多。另外,相对小众的不太知名的经典人物也可以大胆地“收为己用”。‎ ‎(2)回归课本,返璞归真。“踏破铁鞋无觅处,得来全不费工夫”,与其一天到晚抓破脑袋想着如何去积累新颖的素材,不如回归课本——教材是一座素材宝库,有待深入挖掘。文段就是选择课本上杨绛的文章作为例证,既恰如其分,又质朴清新。‎ 四、扩视野,向《花儿与少年》学漂亮句子生成法 这是我们重点要品悟的。深受广大高中生青睐的湖南卫视真人秀节目《花儿与少年》,其文案对节目的吸引度就起到了举足轻重的作用。金牌编剧吴梦知的点题文案,瞬间将节目的内涵升华。漂亮的文句常常能产生四两拨千斤的作用,不容小觑,作文亦是如此。‎ ‎【边练边悟】‎ ‎4.品读鉴赏下面的文段,说说这些文段有何特点。‎ ‎(1)从陌生人到一家人的距离,也许只是一瓦一檐,一蔬一汤,愿砺砺尘路,相亲相爱 答:___________________________________________________‎ ‎_______________________________________________________‎ ‎_______________________________________________________‎ ‎(2)我们不想长大,却一定会长大,而我们不想变老,就可以永远年轻/用一生来探求和世界的关系,不急,它不是拳击手,我们也不是橡皮泥。‎ 答:___________________________________________________‎ ‎_______________________________________________________‎ ‎_______________________________________________________‎ ‎(3)有一位流浪汉,哥伦布,五百年前年轻气盛的他,相信地球是圆的。‎ 为了寻求航海援助,一路从意大利流浪而来。‎ 就是这个曾经的“江湖骗子”,后来一生四次横渡大西洋。‎ 至今依然作为航海界的一哥,面朝新大陆的方向,遥望他曾如何拯救过欧洲,又如何在历史中,给世界搅了局。‎ 答:___________________________________________________‎ ‎_______________________________________________________‎ ‎_______________________________________________________‎ ‎(4)也许,每个人都是孤单的史努比 也许,一上场,队友就变成了对手 可是,胜利,有时候又不是因为战斗 只是因为学会了,勇敢地举起白旗和自己的手 答:___________________________________________________‎ ‎_______________________________________________________‎ ‎_______________________________________________________‎ ‎(5)就像所有恶作剧里的主角,结尾都会有一个幸福的亮点/人生里,学会和麻烦过招,它们也许就成了点赞之交。‎ 答:___________________________________________________‎ ‎_______________________________________________________‎ ‎_______________________________________________________‎ 答案精析 (1)意向叠加营造感情张力。‎ 所谓意向叠加,就是将多个意向渗透交融为一体,通常表现为将名词或名词性短语巧妙排列在一起,构成生动可感的图像,以烘托气氛、创造意境、表达情感。这是一种颇具审美效应的表达方式,它借助连接词缺位,引发解读欲望,引起审美注意,从而催生审美价值。‎ 读者必须摆脱常规思维进行跳跃式联想、想象。比如“瓦”“檐”“蔬”“汤”的组合,就高度浓缩了日常生活中一家人的生活状态,同时又给予读者更加鲜明突出的印象,其内涵也得以突破,为观众提供联想的空间和咀嚼的余地。家人之间互帮互助、互敬互爱的感情被这几个名词美妙地勾勒了出来。‎ ‎(2)巧用双关,妙趣横生。‎ 在一定的语言环境中,利用词的多义或同音的条件,有意使语句具有双重意义,言在此而意在彼,这种修辞手法叫双关。恰如其分地运用双关的修辞,可或含蓄或诙谐地“影射”另外一件事物,可以使表达更有意趣,隽永深刻。例句中“变老”“年轻”既指生理上的、外在的,但更本质地指向心理的、精神的、内在的。因而“而我们不想变老,就可以永远年轻”这句话与前句的表述形成了意思上的反差,含蓄地提醒读者去思考其中蕴含的道理。而句子从形式上来看,虽然不是完全工整的对仗句,但对照当中又有变化,错落有致,别有一番滋味。‎ ‎(3)旧瓶装新酒,素材运用见新意。‎ 所谓旧瓶,就是指旧材料、老材料,这类素材既“熟”又“俗”,很容易拉低作文的水准。因此,在备考过程中,老师一般会指导学生有针对性地准备新颖的素材,殊不知将已经掌握的素材进行改造,旧瓶装上新酒,也不失为一种另辟蹊径的创新法。旧例翻新,可以从角度以及语言方面着手。比如这段材料讲哥伦布发现新大陆的事例,本是老生常谈,但却让人耳目一新。其根源就在于从语言上玩花样,比如将哥伦布定义为“流浪汉”,又恰如其分地融入了流行词汇“一哥”等,更有反语“江湖骗子”令读者会心一笑。而对于他的历史贡献,则用了“给世界搅了局”这样活泼的评价,整个语段寓意深刻有趣,利用语言的智慧瞬间化腐朽为传奇。‎ ‎(4)押韵生和谐,和谐造气度。‎ 押韵为诗歌提供了一种声韵上的和谐,可以增强诗歌在音节、韵律上的美感。从语言认知的角度分析,前后句的押韵,能够构建内容在音韵上的关联,更易于为读者所接受。例句通过配搭“队友”“对手”“战斗”“手”等数个押韵词汇,使整个句子铿锵有力、朗朗上口,更容易引发读者的深思,同时也将以退为进、以柔克刚的道理阐发得更加含蓄。‎ ‎(5)妙用拟人兴意趣,词意错位见苦心。‎ 在议论文写作中,说理之时常常需面对一些抽象概念,如果能巧妙地利用拟人的手法,将抽象的道理以具体化的方式表达出来,可以使文章更为浅白。例句将“麻烦”拟人化,人需要不断与麻烦“过招”,这样的表达新鲜、幽默、有趣,语言更有嚼头。‎ ‎【文题】 [2016·全国卷Ⅲ]阅读下面的材料,根据要求写一篇不少于800字的文章。‎ 经历几年试验,小羽在传统工艺的基础上推陈出新,研发出一种新式花茶并获得专利。可是批量生产不久,大量假冒伪劣商品就充斥市场。小羽意识到,与其眼看着刚兴起的产业这么快就走向衰败,不如带领大家一起先把市场做规范。于是,她将工艺流程公之于众,还牵头拟定了地方标准,由当地有关部门发布推行。这些努力逐渐见效,新式花茶产业规模越来越大,小羽则集中精力率领团队不断创新,最终成为众望所归的致富带头人。‎ 要求:综合材料内容及含意,选好角度,确定立意,明确文体,自拟标题,不要套作,不得抄袭。‎ 材料解读 从材料看,促成小羽成功的因素很多,可以就某一方面立意,也可以综合立意。单从某一方面来谈小羽的成功,似乎稍嫌疏漏,但仔细考察,小羽成功的各个因素之间是互相关联的,如果考生兼顾几方面来谈,驾驭不好的话,文章难免流于疏散而不够聚焦。那怎么办呢?笔者的看法是,聚焦主要矛盾,以小羽成功的其中一个主要原因来统率其他因素。譬如,从创新的角度切入,可以统率诚信、规则、合作、共享等因素。小羽本来是靠创新而获得专利的,但是“批量生产不久”,即遇到了新矛盾——“假冒伪劣产品充斥市场”。是选择放弃任由假货泛滥,自己再另辟蹊径,还是不惜成本同制假售假者打一场持久战?从常规思维出发,小羽只能非此即彼。但是如果任选其一,对小羽而言,都是无奈的。怎么办呢?小羽做出了超出常规的举动:将工艺流程公之于众,带领大家将市场做规范。这不就是创新之中又统率了合作、共享与规则吗?“在这些努力逐渐见效后,小羽则集中精力率领团队不断创新,最终成为众望所归的致富带头人。”最后的成功还是回到创新这一主因上了。“创新—诚信—共享—规则—创新”,如此,“抓住牛鼻子”展开论证,就可以圆满完成“小羽成功主要靠的是什么?”这一写作任务了。‎ 作文材料的内涵分析如下:‎ ‎1.纵观材料,显而易见,“创新”是贯穿这则材料的关键词,也是最明确的立意方向。如上“材料解读”部分的分析,不再赘述。‎ ‎2.“批量生产不久,大量假冒伪劣产品就充斥市场”,这是小羽始料未及的,也是违背小羽初衷的,更是违反市场秩序和法律规范的,这其中暗含一个老话题,那就是:诚信和规则。‎ ‎3.“小羽意识到,与其眼看着刚兴起的产业这么快就走向衰败,不如带领大家一起先把市场做规范。”这不仅显示了主人公的合作、规范意识,更显示了她不凡的见识和胆识、专注于事业的长远眼光和社会责任感。‎ ‎4.“于是她将工艺流程公之于众,还牵头拟定了地方标准,由当地政府有关部门发布推行。”先说“她将工艺流程公之于众”,“还牵头拟定了地方标准,由当地政府有关部门发布推行”,这体现了主人公的责任担当及规则意识。‎ ‎5.“这些努力逐渐见效,新式花茶产业规模越来越大,小羽则集中精力率领团队不断创新,最终成为众望所归的致富带头人。”说明创新合作,不仅化解了矛盾,成全了别人,也成就了自己,还赢得不断创新的空间和动力,使得小羽对工艺精益求精,更好地造福社会,这不仅体现了一种“工匠精神”,还有一种社会责任感和家国情怀。‎ 立意分析 综合上述分析,从不同的角度,可以确定以下立意方向:‎ ‎1.唯有创新,才是成功的永久驱动力(创新)‎ ‎2.创新永无止境(创新)‎ ‎3.独享与共享(共享、合作)‎ ‎4.独富乐不如与众富乐(共享、合作、友善、共赢)‎ ‎5.一枝独秀不是春,万紫千红春满园(责任、担当、合作、共享)‎ ‎6.社会责任,家国情怀(责任、担当)‎ 综合立意示例:‎ ‎1.唯诚信与规则不可辜负(诚信、规则)‎ ‎2.让规则为创新保驾护航(规则、创新)‎ ‎3.远见与胸襟拓展创新之路(见识、胸襟、创新)‎ ‎4.铁肩担道义,合作谋发展(责任、合作、共享)‎ 佳作欣赏 ‎ 小享艺术,大享胸怀 科学家将自己的研究成果一览无余地公布于世,但他仍在物理界独当一面;思想者将其各自主张著书讲学百家争鸣,但它仍在传统文化中生机勃勃;企业者将其独家工艺流程花茶专利公之于众,但她最终成为众望所归的致富带头人。‎ 享受,或小享,或大享,虽只一字之差,但其映射往往有很大不同。小享的最大外化表现是一个人的“高处不胜寒”,而大享则是一群人的狂欢。小享大抵称为艺术,可以是字里行间独自思考,也可以是纵神笔锋挥洒墨痕。而大享则是一个人的分享,一群人的热情享受,大享正如小羽有分享花茶工艺、共同致富的胸怀。‎ 而要在大享中又能站得比别人高一砖,须必备一些方法,也是现在很多企业所匮乏的方法。在大大小小的企业和公司中曾流行过这么一句话,“一流创新,二流模仿,三流盲从”,反映的就是要凝聚起一个富有激情和活力的生产团队,在创新中改进工艺流程,培育工匠精神。持之以恒,脱颖而出也是大享胸怀的一种,正如小羽率领团队创新花茶工艺。‎ 大享中的分享是持续创新的保证。无论是贾思勰编集《齐民要术》,将黄河中下游的农业生产经验总结记录并分享于农民;抑或是张仲景总结中医诊断和治疗理论,结合自己的亲身实践,集成专著《伤寒杂病论》留给后世参考;是居里夫人历经千辛万苦得到镭却将研究成果公开发表,无偿把镭赠予研究治癌实验室,共享科学成果;是小羽将独特花茶工艺分享于众人。如果没有他们的分享,怎会有古代农业生产的耕作技术的总结提升因地制宜,怎有中医临床学的理论基础,怎有放射性同位素治疗癌症的新手段,又怎有花茶产业规模越来越大,一壶紫砂,一注热水,高低缓就,花茶飘香沁人心脾呢?‎ 一个人,一家企业的成功不在于占有什么,而在于分享环境下得到的前进的力量,不在无人区沼泽中独自沦陷,也不是只做自家生意导致乱相丛生。‎ 大享在分享中积极创新。分享之后,人人都是一样的基础,那么怎么才能长久繁荣?脱颖而出还得靠科技创新,制度创新,思维创新。儒家思想的因时而变,格力空调的连年创新,日本商人安藤百福的创新方便面,法国杂工乔利研制干洗剂,及如今民营资本抱团出海一路绿灯,靠的都是技术制度的创新活跃。小羽也是因不断地推陈出新,才能成为致富带头人,引领花茶企业发展。‎ 对于企业能有大享胸怀,群众一直表现出“乐见其成”的积极态度,因为它不仅可以保障生机,同时也有对假冒伪劣行为的客观整顿效果。大享对市场经济的自发性盲目性有不容小觑的纠正作用,也符合推崇的工匠精神的精神内容。‎ ‎“穷则变,变则通,通则达,达则久”,企业唯拥有大享胸怀才能不被历史的浩渺泯灭,才能持久发展。‎ 出彩理由 立意精准,时代感强。文章从小羽将自己的专利技术公之于众的方向立意,提出“乐于分享”这个关键词,并密切联系当代经济发展的现状,区分“小享”“大享”的境界之高下,立意准确高远,弘扬时代主旋律。‎ 语言灵动优美,文句表现力很强。开篇使用排比句,将科学家、思想者、企业者三个善于分享的论据进行精练,使其凝练厚重,使得后文论据的提出显得水到渠成。在论证中,企业名言、贾思勰、张仲景等典型事实论据随手拈来,古典名言的运用驾轻就熟,很见功力。‎ 立论善于思辨,彰显个性。本文能层层深入挖掘材料的“分享”和“精神”,结合时代精神,反对“小享”,倡导“大享”,体现出与众不同的思想境界,观点振聋发聩。‎ ‎1.仿照《花儿与少年》文案“我们不想长大……”(见“边练边悟”4-〈2〉)的写法,以“自由和束缚”为话题,运用双关手法,写一段语言优美的文字。‎ 答:___________________________________________________‎ ‎_______________________________________________________‎ ‎_______________________________________________________‎ ‎_______________________________________________________‎ 答案精析 (示例)我们不想被束缚,却必定活在束缚之中。而我们拒绝羁绊,就可以永远自由。正如卡夫卡在《误入世界》中说:“因为你必须穿越这片沙漠,不自由,因为无论哪条路,由于奇谜般的特点,必然令你触及这片沙漠的每一寸土地。但你的意志是自由的。”这就是说:想要穿越沙漠时,是自由的,因为可以选择穿越的道路。自由来源于可以选择,可以取舍。‎ ‎2.仿照《花儿与少年》文案“有一位流浪汉,哥伦布……”(见“边练边悟”4-〈3〉)的写法,另选一个你心目中的伟人,写一段语言优美的文字。‎ 答:___________________________________________________‎ ‎_______________________________________________________‎ ‎_______________________________________________________‎ ‎_______________________________________________________‎ 答案精析 (示例)有个不自量力的人,愤世嫉俗的他与族人一起被关在漫无边际的大铁笼里。他本可安然地做一个治病救人的医生,衣食无忧,甚至功成名就,然而他却偏偏想要用手中的笔,发出最振聋发聩的呐喊,叫醒铁笼子里沉睡或者装睡的人。有人因而诅咒他,谩骂他,甚至打击他,迫害他,当然,在幽暗的角落里,也有人暗自佩服他。当最黑暗的时刻过去,微光渐渐透进来的时候,偶有人记起,他是如何奋力呐喊,又如何在漫漫黑夜里,给中国点亮了微弱的火把。‎ ‎3.仿照《花儿与少年》文案“也许,每个人都是孤单的史努比……”(见“边练边悟”4-〈4〉)的写法,写一段激励自己的话,注意韵脚。‎ 答:___________________________________________________‎ ‎_______________________________________________________‎ ‎_______________________________________________________‎ ‎_______________________________________________________‎ 答案精析 (示例)在人生的旅途中,有些事,只能一个人做;有些关,只能一个人过;有些路,只能一个人走。所以每个人,都要学会和自己做朋友。得意的时候,敬自己一杯酒;失意的时候,摸摸自己的头;难过的时候,陪灵魂走一走;挫折的时候,劝自己别放手。唯有陪自己走过的光阴,才会真正变成属于自己的岁月。‎ ‎4.仿照《花儿与少年》文案“就像所有恶作剧里的主角……”(见“边练边悟”4-〈5〉)的写法,以“苦难”为话题,运用拟人手法,写一段语言优美的文字。‎ 答:___________________________________________________‎ ‎_______________________________________________________‎ ‎_______________________________________________________‎ ‎_______________________________________________________‎ 答案精析 (示例)苦难总是不期而至,她面目狰狞,却偏偏对你死缠烂打。她讨好你,信誓旦旦说要嫁给你,你被吓得不轻,差点得了心脏病,也唯有振奋精神,和她斗智斗勇。而当有一天,你终于摆脱了她,走向更远更阳光的未来,却在不经意间蓦然回首之时,发现她摘下面具,变成了美丽的女神。有时候,磨砺你是因为很爱你。‎ 学案四 层次之清——胸中必备模板 模板一 分点式议论文 一、并列式分解论点 ‎1.因果分析法 从“为什么”的角度,对中心论点产生的原因进行分析,提炼分论点,分论点与中心论点构成因果关系。如高考满分作文《我想握住你的手》:‎ 总论点:我想握住奋斗的手 分论点:‎ ‎①我想握住奋斗的手,因为奋斗是战胜生活挫折的法宝。‎ ‎②我想握住奋斗的手,因为奋斗是实现人生理想的基石。‎ ‎③我想握住奋斗的手,因为奋斗是充实自我素质的条件。‎ ‎【边练边悟】‎ ‎1.阅读下面的习作《探究,路漫漫其修远兮》的写作提纲,在横线处填写分论点。‎ 总论点:真正的探究性学习的推广,任重而道远。‎ 分论点①:探究之路漫漫,是因为硬件缺乏。‎ 分论点②:探究之路漫漫,__________________‎ 分论点③:探究之路漫漫,__________________‎ 答案精析 (示例)分论点②:是因为教育方式“穿新鞋走老路”‎ 分论点③:是因为唯分数至上的高考指挥棒在作祟 ‎2.概念分析法 从“是什么”的角度,阐述中心论点里关键概念的内涵,提炼分论点。如话题作文《宽容》的分论点:‎ ‎①宽容是一种博大的胸怀。‎ ‎②宽容是一种高远境界。‎ ‎③宽容是一种正直的品格。‎ ‎④宽容是一种化敌为友的智慧。‎ ‎【边练边悟】‎ ‎2.阅读下面的习作《理想是青春的光源》的写作提纲,在横线处填写分论点。‎ 总论点:理想是青春的光源。‎ 分论点①:理想是结实的鞭子,鞭策你奋勇拼搏。‎ 分论点②:______________________________________________‎ 分论点③:______________________________________________‎ 答案精析 (示例)分论点②:理想是指路的航灯,引导你安然前行。‎ 分论点③:理想是重启生命的动力,引领你走出黑暗,重铸辉煌。‎ ‎3.条件分析法 从“怎么办”的角度,把中心论点作为结果,提炼满足这一结果的“条件”来设立分论点。如作文《战胜挫折》的分论点:‎ ‎①战胜挫折,我们要有坚忍不拔的奋斗精神。(勾践)‎ ‎②战胜挫折,我们要有永不言弃的顽强品格。(林肯)‎ ‎③战胜挫折,我们要有积极乐观的人生态度。(爱迪生)‎ ‎【边练边悟】‎ ‎3.阅读下面的习作《预约精彩》的写作提纲,在横线处填写分论点。‎ 总论点:预约精彩 分论点①:道路幽暗,我用自信做明灯,预约精彩。‎ 分论点②:______________________________________________‎ 分论点③:______________________________________________‎ 答案精析 (示例)分论点②:前路荆棘,我用勤奋做刀剑,预约精彩。‎ 分论点③:路途遥远,我用毅力做马力,预约精彩。‎ ‎4.结果分析法 从“会怎样”的角度来设立分论点,主要是回答中心论点会产生的结果和效能方面的问题(意义、作用、价值、影响)。如作文《贫困也是一笔财富》的分论点:‎ ‎①贫困可以培养人的意志和毅力,而人的意志和毅力是成功的重要条件。‎ ‎②贫困能增长人的能力,人的许多能力是在困境中锻炼出来的。‎ ‎③贫困可催使有志者奋发图强,改变命运,穷则思变。‎ ‎【边练边悟】‎ ‎4.阅读下面的习作《为“青出于蓝而胜于蓝”叫好》的写作提纲,在横线处填写分论点。‎ 总论点:为“青出于蓝而胜于蓝”叫好 分论点①:_____________________________________________‎ 分论点②:_____________________________________________‎ 分论点③:_____________________________________________‎ 答案精析 (示例)分论点①:“胜于蓝”是学生努力的结果,标志着他们的发奋取得了成绩。‎ 分论点②:“胜于蓝”是老师的期望。老师不但希望学生有真才实学,更重要的还希望学生能打破前人(当然也包括自己)的学术,有所创新。‎ 分论点③:“胜于蓝”也是社会发展的需要。社会要发展,时代要前进,就需要一代更比一代强。‎ 二、递进式分解论点 即依据“是什么——为什么——怎么样”的提问思路来提炼分论点。如《生活是丰富多彩的》的分论点:‎ ‎①生活丰富多彩是指生活不应是一种模式。(是什么)‎ ‎②符合人的本性(人具有多方面的精神需要);有利于人的全面发展,有益于身心健康;有利于充分调动人的积极性,形成生动活泼的局面。(为什么)‎ ‎③要会工作,也要会休息,培养多方面的生活情趣;社会要为人的全面发展创造条件。(怎么样)‎ ‎【边练边悟】‎ ‎5.用递进式分解法为习作《谈风度》列出几个分论点。‎ 分论点①:_____________________________________________‎ 分论点②:_____________________________________________‎ 分论点③:_____________________________________________‎ 答案精析 (示例)分论点①:风度是一种美德,一种富有教养的臻于完善的气质和风采。(是什么)‎ 分论点②:有了风度,能受到别人的敬重,人与人之间增进爱心,有利于和睦相处。(为什么)‎ 分论点③:有风度,须加强道德情操的修养,摒弃粗俗言行,自觉培养良好的风度。(怎么样)‎ 三、对比式分解论点 即将两种不同的事物或同一事物的不同情况加以对照比较,从而从正反两个方面提炼分论点。如《适度的爱》:‎ ‎①爱子适度,才能使孩子走上健康发展的轨道。‎ ‎②爱子不当以至溺爱,不但不会使孩子茁壮成长,反而会阻碍孩子身心的健康发展。‎ ‎【边练边悟】‎ ‎6.用对比式分解法为习作《如何估价自己》列出两个分论点。‎ 分论点①:_____________________________________________‎ 分论点②:_____________________________________________‎ 答案精析 (示例)分论点①:正确估价自己,既看到自己的长处,又看到自己的不足,就会不断努力,不断前进。‎ 分论点②:过高地估价自己,瞧不起别人,刚愎自用,不思努力,就会停滞不前。‎ ‎【文题】 阅读下面的材料,按要求作文。‎ 你注意到了吗?装鲜牛奶的容器一般是方盒子,装矿泉水的容器一般是圆瓶子,而装酒的圆瓶子一般又装进方盒子里。方圆之用,各得其妙。正如古诗所云:“方圆虽异器,功用信俱呈。”人生也是如此,所谓“上善若水任方圆”。‎ 请根据你对材料的理解,自选一个角度,写一篇不少于800字的文章。‎ 要求:明确立意,自定文体,自拟标题;不要套作,不得抄袭。‎ 佳作欣赏 方圆之用 辩证法的实质在于对立统一,人生的智慧也恰在于此。所谓刚柔并济,所谓能屈能伸,所谓知白守黑,无不告诉我们:方圆之用,善莫大焉。‎ 入世为方,出世为圆。‎ 要说人的一生,首先遇到的问题,当属入世与出世的抉择。有英雄如曹操,对酒当歌,舳舻千里,山不厌高,求贤若渴,乱世之中尽显英雄本色,当属入世之典范。也有隐者如林逋,以梅为妻,以鹤为子,寄情山水,徘徊光影,不愧出世之楷模。‎ 然而视孟德一生,豪情虽多,闲情却少;观林逋之一世,虽名闲适,却少激越。但也有透彻的高人游刃于方圆之间,以出世之心,做入世之事,功成之时急流勇退。汉初三杰之一的张子房,运筹于帷幄之中,决胜于千里之外,官封留侯却于功成名就之时退隐山林,一入一出,方圆之间,尽显风度。‎ 痴心为方,明哲为圆。‎ 人生之理想、事业、感情,皆有痴心与明哲之二法。痴心者如尼采,醉心于自己的超人哲学,自己的热血烧死了自己,落得一个发疯的下场;明哲者更为多,芸芸众生之中多少人精于算计,深谙明哲自保之道,而历史的淘洗却隐去他们的姓名,终被岁月遗弃。‎ 尼采之痴心,使自己走向疯狂;世人之明哲,使自己陷入庸碌,却也有痴心而明哲之人如张大千,热爱艺术也不排斥世俗,懂得进退之中见风华。痴心而明哲,明哲而痴心,方圆之间,尽显智慧。‎ 激进为方,渐进为圆。‎ 再看国家民族之兴亡,亦满是方圆之道。法国大革命之狂热,带来了多少本可避免的残杀,罗伯斯庇尔的白色恐怖,更使人心惶惶。而德民主改革之不彻底,封建残余充斥整个德意志,亦是过于保守之害矣。‎ 历史证明,不论是法兰西还是德意志,方圆只取其一,流弊无穷。相反,外有英国之光荣革命,保守中蕴含激越,内有中国新民主主义革命,有进有退,能打能谈,激进与渐进,方圆之间,耐人寻味。‎ 方中有圆,圆中有方,方圆并举,刚柔并济。方圆之用,善莫大焉。‎ 出彩理由 这篇文章抓住方圆之间的对立统一关系立意,中心突出,主体部分采用横向并列式结构,借助三个分论点,分别从态度、情感、方式三个层面,借助曹操、尼采、民主革命等典型材料,论证“方圆”的重要意义。内容充实,用例丰富典型简练。其间正反说理,加强了论证的力度。最后点题升华,首尾呼应。该文扣题紧,引例典型,感情饱满,内容充实,具有很强的说服力。在表达方面,为了有力论证论点,作者除了引用名言,还大量引用典型的人物材料,语句多用长短句、排比句,增强了论证的力度。结构严谨,层次清楚,语言流畅、形象。‎ ‎1.请分别采用并列式分解论点的几种方法为作文《拆除心墙》确立总论点和分论点。‎ ‎(1)从“为什么”的角度:‎ 总论点:_______________________________________________‎ 分论点:_______________________________________________‎ ‎_______________________________________________________‎ ‎(2)从“是什么”的角度:‎ 总论点:________________________________________________‎ 分论点:_______________________________________________‎ ‎_______________________________________________________‎ ‎(3)从“怎么办”的角度:‎ 总论点:_______________________________________________‎ 分论点:_______________________________________________‎ ‎_______________________________________________________‎ ‎(4)从“会怎样”的角度:‎ 总论点:_______________________________________________‎ 分论点:_______________________________________________‎ 答案精析 (1)(总)人有适当的戒备之心是理所当然的,但对于人与人之间的心墙,我们应该坚决说:“不!”‎ ‎(分)首先,心墙阻止了人与人之间的亲密合作。其次,心墙的存在间接影响了社会风气。最后,心墙还使人看不到社会光明的一面。‎ ‎(2)(总)家庭成员、师生、朋友之间都会有隔阂,有阻挡的心墙。‎ ‎(分)子女与父母之间的鸿沟是心墙。学生与老师之间远远的距离是心墙。朋友与朋友之间深深的误解是心墙。‎ ‎(3)(总)让我们拆除心墙,体验美好。‎ ‎(分)拆除心墙,我们应该抛开嫉妒,拥抱宽容。拆除心墙,我们应该抛开自私,拥抱微笑。拆除心墙,我们应该抛开虚伪,拥抱真诚。‎ ‎(4)(总)我始终相信,推倒彼此之间的心墙,你将会看到别样的洞外天。‎ ‎(分)推倒心墙,我们可以感悟无私的亲情。推倒心墙,我们可以感知真挚的友情。推倒心墙,我们可以感受不变的爱情。‎ ‎2.阅读下面的文字,完成文后题目。‎ 心系一处,难能可贵 ‎①很多人都知道要有所作为就应该“心系一处”的道理,但现实生活中能真正做到“心系一处”的人并不多。“心系一处”,难能可贵。‎ ‎②做到“心系一处”,需要具有顽强的毅力。世界著名物理学家丁肇中先生在40岁时就获得了诺贝尔物理学奖。有人问他成功的原因,丁先生说:“与物理无关的事我从来不参与。”事实的确如此,几十年来他始终把精力集中在科学研究上,集中在探索宇宙的奥秘上。他在实验室里做实验,有时候连续四五天不睡觉,正是凭着这股韧劲,他的实验获得了成功。‎ ‎③做到“心系一处”,需要守住内心的宁静。在这个越来越繁华的世界,人们的目光能够不为五光十色的景色所吸引,的确不易。作家苏童一直以来潜心创作,心无旁骛。作品《妻妾成群》被改编成电影后,他名声大噪,上门的采访者、崇拜者络绎不绝,但是苏童很冷静,他对记者说:“门外的繁华不是我的繁华,我是过室内生活的人,以前很安静,现在更安静。”另一个作家张炜则奉行“三不主义”:不看热闹的书,不去热闹的地方,不交热闹的朋友。对于许许多多的成功者而言,最难能可贵的是他们成功后依然做到了“心系一处”。‎ ‎④“心系一处”是一种智慧。这种智慧不是一意孤行的固执,而是繁华过后的觉醒;不是缺乏思想的单纯,而是一种去繁就简的境界。当你处在人生的低谷,“心系一处”能让你学会坚持,带给你重振雄风的希望;当你处在人生的峰巅,“心系一处”能让你保持清醒,带给你再创辉煌的动力。‎ ‎(1)本文的中心论点是什么?‎ 答:___________________________________________________‎ ‎(2)第②段以丁肇中先生为例证明了什么道理?‎ 答:___________________________________________________‎ ‎(3)第③段论述的观点是什么?‎ 答:___________________________________________________‎ ‎(4)第④段论述的观点是什么?‎ 答:___________________________________________________‎ 答案精析 (1)心系一处,难能可贵。‎ ‎(2)做到“心系一处”,需要具有顽强的毅力。‎ ‎(3)做到“心系一处”,需要守住内心的宁静。‎ ‎(4)“心系一处”是一种智慧。‎ 模板二 层进式议论文 一、是什么,为什么,怎么办 按照“提出问题—分析问题—解决问题”的思路安排论证结构,即围绕中心论点回答三个问题:是什么,为什么,怎么办。‎ 例如,写作《谈坚持》时,我们可以列出下面的本论提纲。‎ 坚持是什么:坚持是对极限的挑战,是对心血和汗水的慷慨挥洒,是对理想的执着,是不到长城不止步的豪迈。‎ 为什么要坚持:绳锯木断,水滴石穿,成功往往就诞生在再坚持一下的努力之中。‎ 坚持需要什么:坚持需要韧性,需要耐得住寂寞,更需要知其不可为而为之的大智大勇。‎ 怎样做到坚持:坚持不是固守,更不是画地为牢。‎ 结论:远方的诱惑是我们之所以忙碌,之所以奋斗,之所以拼搏之所在。当我们遇到困难,遭受挫折,当我们汗流浃背,精疲力竭的时候,我们应该在心中默念一声:再坚持一下。‎ ‎【边练边悟】‎ ‎1.下面是一位同学的习作,阅读后请梳理出这篇文章的结构提纲。‎ 让梦想在现实中起舞 仰望星空,那似乎没有纤瑕的星辰在银河中闪耀,它带给我们无限的遐想,那不染纤尘的星空里,放飞了多少人美丽的梦想!飞上星星的人知道,那里像地球一样,有灰尘也有石渣,于是他们失去了对幻想的渴望。我们虽不能一味沉溺于自己的幻想之中,却也不能让自由飞翔的思想湮没在无情的现实里。‎ 阮籍目睹世间的浑噩不堪和好友的身首异处,借醉酒逃避现实,他的一生一直在逃避、逃避、逃避,却终因一篇《为郑冲对晋王笺》被人唾弃。嵇康则完全生活在现实之中,不肯向生活做出任何妥协,最终以一曲《广陵散》而成为绝响。其实人生由阮籍的醉酒向前一步便是嵇康的《广陵散》,人生由嵇康的《广陵散》向后退一步便是阮籍的醉酒,殊途同归者的境遇竟是如此迥异。若是两人各向中间迈出一步,将幻想与现实稍加中和,也许就不会落得生者隐入迷幻,死者融入苍穹,只留给后人无尽的怅惘。‎ 我们如何才能让仰望星空的人了解现实,又如何才能让飞上星星的人保留梦想呢?‎ 在那个人人埋怨的时代,沈从文先生目睹现实的残酷,却依然将那个江南小城写成了山美、水美、人美的世外桃源,现实没有湮没他对人生的希望,他用一份最原始的情感和一颗赤子之心看待这个社会,看待自己的人生。他没有沉醉于自己的幻想,亦没有让现实麻木自己的心灵。‎ 张允和先生亦是一位智慧的老人,她一生经历了大富大贵,也经历了战火纷飞,十年浩劫,而她却永远保有一副悲天悯人的情怀,一颗永不衰老的童心。她那悲天悯人的情怀使她正视现实并战胜现实,而她那颗永不衰老的童心则使她在任何艰难的情况下都不放弃幻想的权利。‎ 川端康成浅浅的一句“凌晨四点钟,看到海棠花未眠”,瞬间感动了多少心灵,这是梦和现实最完美的结合。让那些世俗之物顷刻间土崩瓦解,让多少在现实中日渐麻木的心灵得到了温暖。‎ 正视现实,但不委身于现实,保持幻想,但不沉溺于幻想,让梦想在现实中去跳一曲酣畅淋漓的舞蹈!‎ 结构提纲:‎ ‎_______________________________________________________‎ ‎_______________________________________________________‎ ‎_______________________________________________________‎ ‎_______________________________________________________‎ 答案精析 立论:提出问题——不能让梦想湮没于无情的现实中。‎ 分析问题——正反对比“为什么”。(反:阮籍醉酒。正:嵇康《广陵散》。结论:梦想与现实中和便无人生怅惘。)‎ 本论:解决问题——举例论证“怎么办”。(沈从文:用最原始的情感看待现实社会。张允和:正视现实,不放弃梦想。川端康成:梦想与现实完美结合。)‎ 结论:重申论点。‎ 二、摆现象,析危害,挖根源,指办法 就是针对某些不好的现象,分析其危害,挖掘其产生的根源,指出解决问题的办法。即“摆现象—析危害—挖根源—指办法”的格式。‎ 例如:根据下面的材料,自选角度,自拟题目,写一篇不少于800字的议论文。‎ 一位台湾女作家,在她的散文中,写了这样的句子:“初夏的日子,阳光改变了空气的气味。”一位大陆评论家说:“这样写很好,用的是美文笔法,善用通感,讲究炼字,刻意经营。”我看了不禁哑然失笑,因为我的女儿4岁时就“通感”过。冬天里我时常晒被子,晚上睡觉时女儿说:“妈妈,我最喜欢睡晒过的被子,里面有太阳的味道。”撇开母女亲情不说,平心而论,我觉得女儿的话说得比散文家更生动些。记得一位大画家说过:“每一个孩子都是艺术家。”不知什么缘故,有不少孩子长大了反而变蠢了呢!‎ 佳作欣赏 自然才是美 一位台湾女作家在她的散文中这样写道:“初夏的日子,阳光改变了空气的气味。”无独有偶,一个小女孩也这样告诉她的妈妈:“我最喜欢睡晒过的被子,里面有太阳的味道。”小女孩的话和女作家的话意思相近,可是小女孩的话却要生动得多。究其原因,是因为小女孩是非常自然地说出这句话的。她说话的时候,无须考虑讲究语言的修辞;女作家则不然,她要刻意雕琢语言,而过分的雕琢反失却了自然。可见,自然才是美。‎ 当今青年学子中,像那位台湾女作家那样刻意为文的并不少,其作品矫揉造作有余,清新自然不足。尤其在华丽文风盛行的风气下,不少人竞相追求一种飘浮、朦胧的意境。结果,朦胧过度,反成晦涩,令人不知所云。我曾在《大学生》上看见过这样一首诗:“明月高悬/孤星冷照/离开了你/我的世界/只剩下/一根冷冷的弦。”这就是典型的“为赋新词强说愁”。除了把读者弄得一头雾水之外,还给读者心中平添一股怒气:这也叫诗?!‎ 追求纯朴自然是我国诗文家的传统。李白曾经说过:“清水出芙蓉,天然去雕饰。”可见他也是不赞同刻意为文的。不只李白,还有清代的袁枚,他主张“性灵说”,也认为文章贵在自然,不能有空泛的话。中国近代有“才女”之称的张爱玲,她的小说《金锁记》就由于过分讲究技巧而招致许多文学家的批评。著名文学家傅雷在《论张爱玲的小说》中这样写道:“技巧将是张女士的最大的敌人。”看来,刻意雕饰是写不出好作品的,文章贵在自然。‎ 当然,我们提倡写文章要自然,并不是要大家不注重语言的锤炼,而是要反对刻意为文;诚心诚意希望青年学子们多写一些自然、清新的文章。‎ 出彩理由 本文抓住材料中的关键语句之一——“我觉得女儿的话说得比散文家更生动些”,从文风角度立论,联系当前青年学生文风不正的实际,有感而发,有的放矢,针对性强。文章首先以台湾女作家与小女孩的话进行对比分析,揭示原因,推出论点;然后联系当今青年学生文风不正的现象,强调端正文风的必要性;再以李白、袁枚、张爱玲的正反事例论证追求纯朴自然是我国诗文家的传统,强调端正文风的重要性;最后特别说明“提倡写文章要自然,并不是要大家不注重语言的锤炼,而是要反对刻意为文”,是“诚心诚意希望青年学子们多写一些自然、清新的文章”,在辩证说理的同时,照应了观点,使文章结构浑然一体。‎ ‎【边练边悟】‎ ‎2.下面是一位同学的习作,阅读后请梳理出这篇文章的层进式结构提纲。‎ 学会礼让 荷兰不知名古镇德拉赫腾镇没有红绿灯,人们用眼神和手势来交流达成默契,素不相识的驾驶者会在路口相让,“汽车让自行车,自行车让行人”成为人们默认的规则,他们的礼让创造了交通奇迹。‎ 荷兰古镇德拉赫腾镇的“无红绿灯”现象,使我不禁想到我国许多地方的“无视红绿灯”现象。‎ 在我国大小城市,红绿灯、电子警察遍布交叉路口和要道,可谓设施完备,然而时常可见高速奔驰的汽车乱闯红灯。如果两车在小巷“狭路相逢”,“虎视眈眈”的现象并不鲜见,你不让我,我不让你,有时甚至恶语相向,拳脚相加。最有讽刺意味的是,在我国,“人让自行车,自行车让汽车”倒成为人们“默认的规则”。‎ 不懂礼让的行为贻害无穷,使我们愧对“文明古国”的称号。‎ ‎“请自觉排队”“请不要大声喧哗”……这种仅以简体中文标出的警示牌,正在中国人出境游的主要目的地——法国、德国、日本、泰国、新加坡等地频现。当大批中国游客走向世界各地时,“中国人”却成了不文明和粗鲁的代名词。‎ ‎“不懂礼让”现象的出现,根源何在?‎ 一个有文明素养的人才懂得礼让,礼让的本质是对人的体贴和尊重。不懂礼让,其实质就是以自我为中心,不懂得为对方着想,不懂得体贴对方、尊重对方。很多人满脑子都是“孔方兄”,早已将古人“人有礼则安,无礼则危”的“遗训”抛在脑后。十字路口,加大油门高速奔驰,为的是赢得时间去追求自己的金钱、名誉和地位;在追求功利的路上,“狭路相逢”,必定是一场好斗……‎ 由此可见,缺乏文明素养,不懂得体贴人和尊重人,过于追求功名是造成“不懂礼让”的根源。‎ 在这样的情形之下,只有使国人认识到文明礼让的重要性,大力培养国民的文明素养,中华民族这个古老的礼仪之邦才能重新焕发出新的光彩。‎ 礼让是中华民族的优良传统,战国时蔺相如对廉颇一让再让,终于让出了“将相和”‎ ‎,成为千古美谈。古人曰:“爱人者,人恒爱之;敬人者,人恒敬之。”可见礼让的作用是无与伦比的。礼让是人与人交往的缓冲带,可以减少许多不必要的冲突;礼让又是人与人交往的润滑剂,它可以减轻摩擦,化解紧张的关系。‎ 学会礼让,对于我们每个人来说只是举手之劳,于己、于人、于社会有百利而无一害,我们又何乐而不为呢?一个宽松的人际关系需要我们自己去创造,一个祥和的文明环境需要每个人都学会礼让。只要人人都学会礼让,祖国处处都能变成美好的德拉赫腾古镇。‎ 结构提纲:‎ ‎_______________________________________________________‎ ‎_______________________________________________________‎ ‎_______________________________________________________‎ ‎_______________________________________________________‎ ‎_______________________________________________________‎ 答案精析 (1)引述材料,表明角度。(1~2段)‎ ‎(2)摆现象——我国大小城市不懂礼让的行为。(第3段)‎ ‎(3)析危害——愧称“文明古国”,成了不文明和粗鲁的代名词。(4~5段)‎ ‎(4)挖根源——缺乏文明素养,不懂得体贴人和尊重人,过于追求功名。(6~8段)‎ ‎(5)指办法——明确礼让的重要性,大力培养国民文明素养。(第9段)‎ ‎(6)总结:明己见,提号召——学会礼让。(10~11段)‎ ‎【文题】 阅读下面的材料,根据要求写一篇不少于800字的文章。‎ 近年来,素有“语林啄木鸟”之称的《咬文嚼字》杂志开设专栏,为当代著名作家的作品挑错,发现其中确有一些语言文字和文史知识差错。对此,这些作家纷纷表示理解,并积极回应。中国作协主席铁凝诚恳地感谢读者对她的作品“咬文嚼字”;莫言在被“咬”之后,也表达了自己的谢意,他表示,请别人挑错,可能是消除谬误的好办法。‎ 要求:①选准角度,自定立意;②自拟题目;③除诗歌外,文体不限,文体特征鲜明;④不要脱离材料内容及含意的范围。‎ 佳作欣赏 感谢挑错 近年来,素有“语林啄木鸟”之称的《咬文嚼字》杂志开设专栏为当代作家挑错。对此,作协主席铁凝诚恳致谢,莫言在被“咬”后也表示:请别人挑错,可能是消除谬误的好办法。‎ 挑错,一个似乎让人厌恶的字眼,在著名作家的眼中竟成了值得感谢的行为。感谢挑错,他铲去错误的毒瘤,让真理之花绽放,让跋涉者离卓越的山峰更近一步。‎ 感谢挑错,是一种谦卑的姿态,更是一种大智慧。京剧名旦梅兰芳先生,在一次演出结束后,于众人喝彩声中听见一老人大喊:“不好!”散场后,梅先生特意将老人请至后台:“说吾孬者,是吾师也。请您指教。”老人深受感动,提出中肯的改正建议。从那以后,梅先生每到此地演出,必请老人前来,面对老人毫不客气的挑错,梅先生不觉丢脸怨恨,反而拜老人为师。‎ 正是因为感谢挑错,改进不足,梅先生登上京剧舞台的巅峰,成为一代传奇。‎ ‎“人非圣贤,孰能无过?”即使是大家学者,也要经历由无知开始的学习过程。感谢挑错,才能认清不足,至真至善。章太炎先生出身于医学世家,自恃医技高明,直至一次自己生病久治不愈,经名医挑错才知道自己的药方不管用。从此苦心钻研,在医学领域终有建树。‎ 假如没有那次生病后名医挑错,章太炎恐怕意识不到自己的不足,发奋学成又从何谈起?感谢挑错,成就大师之学。‎ 感谢挑错,是一种态度,更是一种行动,闻过则喜,主动找人挑错,更难能可贵。日本原一平,由一个小职员成为“推销大师”,正是因为他每周请朋友、客户吃饭,让他们为自己的言谈举止挑错,不断改进,终于创造了行业神话。从现在做起,感谢挑错,才能把握成功的契机。‎ 为学时,感谢挑错可以消除谬误,凸显真理;为人时,感谢挑错可以彰显风度,追求卓越。人生路途中,正是那些挑错的声音将我们的道路照亮,让我们少走弯路,直奔成功。‎ 面对“咬文嚼字”的挑错,铁凝、莫言不但不气不恼反而心怀感恩,相比之下,孔庆东在微博骂人被告仍不改正,则失了知识分子的精神。感谢挑错,作家们为当今社会树立榜样,是对真理的尊重,更是对谦逊人格的展示。‎ 感谢挑错,它是成功路上的垫脚石,让我们循着真理之路攀上卓越之巅。‎ 出彩理由 一是立意精准,素材丰富。考生匠心独运,题目是“感谢挑错”,短短四字,既概括了文章主旨,更彰显了作者概括材料的能力。本文素材丰富,选取了三个正面事例,梅兰芳先生的事例详尽且形象生动,简洁而富有镜头感,让人印象真切;章太炎先生的事例叙议结合,要言不烦,注重假设说理,事例论证效果好;原一平的事例运用了因果论证方式,先果后因,突出了文章核心观点的论证力度;反面事例孔庆东微博事件则略写,形成集束式正反对比的论证力量。‎ 二是逐层递进,章法有序。“感谢挑错”,这种提取材料关键词组合而成的观点直抵材料的核心内涵,立意准确,切题;且在论述中形成了从“一种姿态”到“一种行动”的层进式的说理。“感谢挑错”的姿态与智慧已是难能可贵,易于除谬守正;“感谢挑错”的态度与行动更显非凡风釆,更能体现追求卓越的精神。层层深入,使得行文论证由平面化走向了立体纵深化。‎ 三是善于扣题,且行文质朴中透着生动形象。文中,“感谢挑错”一词多次出现,显示了强烈的扣题意识。行文总的来看,质朴刚健,但不乏比喻句、反问句、感叹句等。如“铲去错误的毒瘤,让真理之花绽放”“成功路上的垫脚石”等比喻贴切形象,也给文章添彩不少。‎ ‎1.请以“教养”为题,在确立中心论点后,写出层递式结构提纲。‎ 结构提纲:‎ ‎_______________________________________________________‎ ‎_______________________________________________________‎ ‎_______________________________________________________‎ ‎_______________________________________________________‎ ‎_______________________________________________________‎ 答案精析 (示例)引论:教养是人才素质的重要组成部分,是成功的基石。‎ 本论:①列举当今社会种种缺乏教养的表现。②剖析原因:a.他们不知道教养在成功中的重要作用。b.他们不知道教养在国家文明建设中的地位。‎ 结论:教养是一种高贵的气质,它可以令你在人群中发出非凡的光芒。照应前文。‎ ‎2.阅读下面的文字,根据要求写一篇不少于800字的文章。‎ ‎(1)习近平同志指出,是否具有担当精神,是否能够忠诚履责、尽心尽责、勇于担责,这是检验每一个领导干部身上是否真正体现了共产党人先进性和纯洁性的重要方面。作为党员特别是领导干部,敢于担当,既是党和人民事业的要求,也是共产党人应该具备的政治品格。‎ ‎(2)国务院总理李克强在自己的首场“总理记者会”上给新一届政府“约法三章”:一是政府性的楼堂馆所一律不得新建;二是财政供养的人员只减不增;三是公费接待、公费出国、公费购车只减不增。“约法三章”用实际行动践行着“己正,才能正人”,不仅表明了新一届政府的反腐决心,更彰显了新政府的担当。‎ 要求:选好角度,确定立意,明确文体,自拟标题;不要脱离材料内容及含意的范围作文,不要套作,不得抄袭。‎ 立意分析 审题:这道作文命题由两则材料组成,二者是有机的整体,不可分割开来。由多则材料组成的题目,一般有两种情况,一是材料之间是统一关系,互相支持,互相印证,共同说明某个问题,反映某种规律,揭示某种观点,审题立意时关键是要抓住它们的共性。二是材料之间是对立关系,互不包容,甚至相互排斥,审题的方法是寻求互补,运用“求和”‎ 的方法归纳出材料的基本观点,进而立意。这道作文命题属于前者。解读本题,要仔细研究材料提供的信息,从中寻找或提炼关键词,求同存异,准确立意行文。材料(1)中,习近平同志主要强调的是“忠诚履责、尽心尽责、勇于担责”,其关键词是“担当”;材料(2)中,李克强总理的“约法三章”主要包含了“反腐”和“担当”两层意思。将这两则材料综合比对,求同存异,它们的共性自然是“担当”了。由此,可以以“担当”为中心进行立意谋篇,如“担当是一种美德”“勇于担当,善于担当,是人生的至高境界”“生命因担当而精彩”等。‎ 解读本题还必须注意两点:一是不可就事论事,只在“领导干部”或“政府”身上做文章,空话大话满天飞,而应该由点到面、由此及彼展开联想。联系实际特别是对生活实际进行深入论证,使文章更加贴近现实,具有更广泛的指导意义。二是严防偏题跑题,不能断章取义,抓住“干扰信息”不放,围绕“腐败”立意行文——这是对材料的错误理解。‎ 佳作欣赏 勇于担当 走向成功 鲁迅先生说过:“自古以来就有拼命实干的人,有舍身求法的人,有为民请命的人,他们是中国的脊梁。”‎ 孙中山先生也曾说:“唯我辈既以担当中国改革发展为己任,虽石烂海枯,而此身尚存,此心不死。”‎ 先贤的教导犹在耳畔,我们要时刻牢记:一个人只有敢于担当,才能走向成功;只有敢于担当,才会被历史铭记。‎ 翻开历史画卷,那些可歌可泣的英雄人物,他们身上无不闪耀着“担当”的光彩。‎ 苏武敢于担当,才会不为匈奴的威逼利诱所动,成为后世楷模;林则徐敢于担当,才会有虎门销烟的壮举,成为民族英雄;詹天佑敢于担当,才能成功修筑京张铁路,让中国人扬眉吐气。正是这么多勇于承担历史重任的巨人的存在,中华民族才能不断发展,一步步走到今天,以高昂的姿态屹立于世界民族之林。‎ 可能你会说,敢于担当是那些名人伟人的事,与我们平民百姓没有关系。其实,每个人都身负担当之责,“担当”体现在生活的方方面面。‎ 敢于承认自己的错误并承担后果是担当。巴金老人晚年在《随想录》中忏悔了自己曾做过的错事、说过的假话。他敢于担当,为国家为人民树立了敢于说真话的榜样。他的担当,实现了他“化作泥土,留在人们温暖的脚印里”的愿望。人无完人,敢于揭自己的“疮疤”,才是明智的人,才是健康的人。‎ 做好自己的本职工作,不因个人利益或各种困难而轻言放弃是担当。一个平凡的公交车司机,驾驶途中遭遇“飞来”异物导致肝脏破裂,他强忍剧痛,缓缓停稳车,安全疏散乘客,挽救了24名乘客的生命,自己却因抢救无效而与世长辞。他就是杭州“最美司机”吴斌。在生死关头,他不顾个人的生命安危,勇于担当,尽职尽责,让我们永远记住了他。‎ 自觉承担社会责任是担当。在汶川、玉树发生地震时,人们众志成城,出名的抑或普通的,都在为社会尽一份力量。正是有了这些乐于担当的人,我们的社会才能和谐、稳定地发展。‎ 诸如此类的事情,难道说跟我们没有关系吗?‎ 个人的成长需要担当,社会的发展更离不开担当。敢于担当,人生才会多一份精彩;敢于担当,才能拥有事业的辉煌;敢于担当,才能勇做中国的脊梁!‎ 模板三 议论性散文模板 一、议论性散文的特点 议论文的框架,散文化的笔法,二者合一,就是议论性散文。形象美和理性美,是议论性散文最本质的文体特征。在高考中议论性散文比较受欢迎,一是议论便于联系话题,二是散文形式较灵活、自由,便于广泛地选取材料。‎ 二、议论性散文的写法 ‎1.入题:在散文化的描写中,用议论提纲挈领,引领全文,或者由新颖、形象的比喻入题,直接扣题。如文章《生命如大河》的开头:‎ 如果生命有形色,它一定奔涌如大河,挟卷一切入海,奔流不返,它以劈山穿石的伟力流动,在我们的叹惋中奔去,雄伟而决绝。所以勤勉者说:生无所息。‎ 如果生命有形色,它一定奔流如大河,穿越千里,穿越岁月,生命在月光下奔流,在平原上涌动,累了,倦了,便暂停下匆匆脚步,于是有了湖泊的美丽与宁谧。所以,睿智者说:生有所息。‎ 人生,便是这二者的统一。‎ ‎【边练边悟】‎ ‎1.下面是高考优秀作文《学会历史般的旁观》的开头,试分析这样开篇的特色。‎ 在蝶的眼中,花是天使,因为花给予她生命的甘露;在花的眼中,蜂是挚友,因为蜂给予她生命的延续。然而在蝶眼中,蜂不过是埋头苦干的笨蛋;在蜂眼中,蝶不过是游戏花间的浪子。‎ 答:___________________________________________________‎ ‎_______________________________________________________‎ ‎_______________________________________________________‎ 答案精析 文章开头赋予蝶、蜂、花以人的性情,连物都会带着情感的眼光来评价、认知事物,更何况是情感丰富的人呢?通过生动贴切的拟人手法,将话题的内在含意巧妙点出,不仅唤起了人们对美好事物的相关联想,更体现出了作者的睿智。‎ ‎2.主体:多角度选材,内容丰富,思路清晰,用议论的方式画龙点睛。‎ 与议论文一样,散文要求内容充实,选取2~3个具体事例或者描写片段体现主题,并穿插议论深化对事物的认识。‎ ‎【边练边悟】‎ ‎2.阅读下面的高考优秀作文《一步与一生》,试分析其主体部分的特色。‎ 一步与一生 路的两旁氤氲着乳白色的雾气,前面是一条悠长悠长的古道,迷离神奇。‎ 我准备沿着这条充满神秘的古道去寻找自己的幸福人生,一束极细的光芒从遥远的天际投射过来,抚摸着那历经悠悠岁月的古道刻痕,它闪耀着夺目的光芒。‎ 一串串脚印散落在岁月的路途中,我要去寻找决定人一生的那一步,用心去发现一个刻满幸福的出口。‎ 我渐行渐远,我渐进渐远……‎ 终于,我发现路在此时分成了两个岔口,两扇高大的门摆在面前,它们的上面镌刻着“一步与一生”,苍劲的字体中流淌着充满理性的溪流。‎ 我打开其中一扇门,发现上面雕刻着许许多多的小字,我仔细凝视,仿佛时光在这里轮回翻转……‎ ‎“曾经的庄子从这里走过,为了摆脱一个屈从权势的社会,他推开了这扇门,然后决绝地跨了过去,做了一棵在夜里独自守望月亮的树。”‎ ‎“曾经的五柳先生从这里走过,为了摒弃官场的丑恶,他驾着破旧的马车从这扇门驶过,然后‘采菊东篱下,悠然见南山’。”‎ ‎“曾经的东坡先生从这里走过,为了遗忘痛苦的失意,他穿着粗衣淡定地走了过去,在荒凉的黄州‘倚杖听江声’,任凭人生的失意而高唱‘大江东去,浪淘尽’。”‎ 我品读着这曾经的故事,路旁的竹林也在歌唱,我要把它们谱写成一曲沧桑而美丽的歌谣。‎ 我又走到另外一个路口仰望另一扇门,它上面同样刻满了密密麻麻的小字。‎ ‎“西楚霸王项羽面对刘邦的追兵从这里逃走,忆起昔日的鸿门宴,不禁哀叹‘时不利兮骓不逝,骓不逝兮可奈何,虞兮虞兮奈若何?’”‎ ‎“残忍阴险的秦桧从这里步入罪恶的深渊,杀岳飞,贬忠良,他的一生必定是让世人唾弃的一生,他将永远地被钉在历史的耻辱柱上。”‎ ‎“居心叵测的陈水扁从这里走上了分裂国家的道路,嘴角邪恶的笑容干枯了他丑恶的灵魂。”‎ 我不想再读下去,抚摸眼前的这扇门,沧桑而厚重。历史本没错!错的是他们洞开了本不应该洞开的大门,跨出了他们本不应该跨出的一步!‎ 枫竹林飘荡着幸福的歌谣,我回眸最初的那扇门,它闪着幽蓝幽蓝的光,我毫不犹豫地走了进去,迎接我期待的黄金般的人生!‎ 答:___________________________________________________‎ ‎_______________________________________________________‎ ‎_______________________________________________________‎ 答案精析 在主体部分,作者引入了庄子、五柳先生、东坡先生以及西楚霸王项羽、奸佞秦桧、小丑陈水扁六个人物,但并不嫌繁琐,因为作者将这六个人物的事例分为两组,构成对比,既突出了中心,又有变化。前一组事例,用“曾经的……从这里走过”这一句式引出,具有整饬之美。后一组事例,则用“……从这里……”引出,既整齐,又与前面的一组句式不同,加上内容的区别,使文章整饬中有变化,不显得板滞沉闷,而是富于灵动变化。‎ ‎3.结尾:结尾是文章前后呼应、结构完整的重要一环,是文章的重点。人们对文章总体印象的好坏,文章的结尾部分起着至关重要的作用。好的结尾,能使文章产生意味深长、余音绕梁的效果。如作文《走进生活》精彩的结尾:‎ 无论是宁愿受骗也不愿意有人身陷痛苦的球手还是整夜不眠的父亲,他们都用行动说出了生活中最美的语言:仁爱与宽容。生活如花篮,需要用爱来装饰;生活如树苗,需要用爱来灌溉。而生活中的爱则恰如在心灵中怒放的玫瑰,散发着诱人的芬芳。让我们走进生活,一同感受人间花海般的爱。‎ ‎【边练边悟】‎ ‎3.下面是张晓风《劫后》的结尾,试分析这一语段的特色。‎ 于是学会了为阳光感谢——因为阴晦并非不可能。学会了为平静而索味的日子感谢——因为风暴并非不可能。学会了为粗食淡饭感谢——因为饥饿并非不可能。甚至学会了为一张狰狞的面目感谢——因为有一天,我们中间不知谁便要失去这十分脆弱的肉体。‎ 答:___________________________________________________‎ ‎_______________________________________________________‎ ‎_______________________________________________________‎ 答案精析 此段由四个句子组成。前三个呈现并列关系,每个句子前后分句主要内容呈相对关系。第四句是在此基础上的递进,表达出作者“学会感谢”的主题。这段话适合写“感谢生活”“走进生活”“生活”“生命”“宽容”等话题的作文。‎ ‎【文题】 见本部分“学案一”本栏目[2016·全国卷Ⅰ]‎ 佳作欣赏 标准,你为何如此重要?‎ 当我们拿着满分试卷回家后,家长的亲吻源源不断;但当拿到一张不及格试卷时,得到的只是一个巴掌。然而我满心欢喜拿着差两分就满分的试卷回家时结果却跟不及格试卷结果一样。心里不禁生出一个疑问:标准,你真的如此重要吗?‎ 我想说:不!‎ 不可否认,我们都想要一个达到标准的试卷,对于家长们来说更是如此。然而,一张达到标准的试卷就能说明我们真的全部理解了知识,全都掌握了知识吗?诚然,或许家长们只是希望自己的孩子能取得很好的成绩,这确实是每一个父母都有的心理,但是,达到了一个标准便是亲吻,低于这个标准便是巴掌的行为真的合理吗?难道一张98分的试卷不及一张61分的及格卷?只因98分没有得到满分这个标准便是巴掌,只因61分满足了及格这个标准便亲吻。那我们学生以后要怎么做呢?‎ 若家长坚持这种做法,那学生会不会在发现自己不能达到满分而只能达到高分,却想得到家长们的表扬,选择放弃高分而只是达到及格呢?那以后的学生是不是只会安于现状,达到及格便可?因为高分还不如一个及格标准,所以学生们会从此安于现状止步不前,不会想取得高分,那有些知识便不再去理会。我想这并不是家长们的初衷。所以,标准也没有如此重要。‎ 正如当代诗人郭小川说:“但愿每次回忆,都对生活不感负疚。”希望家长们也明白,标准不能衡量一切,因而,有时我们也大可不必太注重标准,让我们对生活不感负疚,就像麦克阿瑟所言:“当生活都用标准来衡量时,那社会便不再有温情和喜悦。”‎ 标准确实可以衡量一些东西,但它只是一个标志性的东西,并不能代表一切。钱钟书在美国留学期间,他的导师并不用标准考试成绩来衡量一个人的付出,而是结合他平时的表现综合思考,从而对学生有一个更客观的定论,学生也会不断努力向前奋进。因此,钱钟书一生最敬佩的人便是他的导师,钱钟书取得如此的成就也是受其影响。试想一下,倘若这位导师也像漫画中的家长般,如此注重一个标准,结果又会如何?还会有如此成就的钱钟书吗?他还会成为钱钟书最敬佩的人吗?‎ 危害如此,痛心也在此。‎ 反观当下,多少人追名逐利,只为一纸文凭和所谓的标准,争得头破血流,最后也一无所获。为何不把标准看淡些,正如汪国真所说:“我不去想是否能够成功,既然选择了远方,便只顾风雨兼程。”‎ 这红尘,太污太苦太锦簇;这人情,总浮总疏总麻木。愿我们所有人都能把标准看淡些,然后去追逐诗意和远方,不留负疚。‎ 出彩理由 此文开头采用对比手法阐释漫画寓意,接着就势点题,亮出中心论点,与一般作文的开头迥乎不同。论述的时候有侧重,重在论述标准的危害:“从此安于现状止步不前,不会想取得高分,那有些知识便不再去理会。”‎ 此外,这篇议论文逻辑严密而又富有文采,具备议论性散文的特质,如考生在引用汪国真诗句后,用“这红尘,太污太苦太锦簇;这人情,总浮总疏总麻木。愿我们所有人都能把标准看淡些,然后去追逐诗意和远方,不留负疚”收束全文,呼应前文诗句,又让文采芬芳四溢。‎ 考生在开头提出自己的观点后,并没有一味否定标准,考生认为标准确实可以衡量一些东西,但它只是一个标志性的东西,并不能代表一切,这样观点就极为辩证和深刻。‎ ‎1.阅读下面这篇高考优秀作文,思考一下它体现了议论性散文的哪些特征。‎ 天光甚美,造化有情 ‎(一)面对 面对生活斯芬克斯式的笑脸,我们有太多的抉择。只有用真、善、美的心灵标准去选择的人,才体现出俄狄浦斯式的睿智;然而用一颗丑陋自私的心去描绘生活画谱的人,则表现出了堂·吉诃德式的幼稚。‎ ‎(二)和亲中的新娘 曾记否,杜甫笔下“环珮空归夜月魂”的王昭君,在当时政局动荡的情况下,毅然把自己送上了和亲的道路,告别了汉宫的朱墙碧瓦,把自己的后半生定在阴山敕勒川下,只为匈奴民族从此能更和平地在边界上牧羊放马,只为换得汉朝边塞上数十年的安定。面对荣华,面对人生,面对国家,昭君用自己的一生做了这一次选择,这是那些深宫中“缦立远视,而望幸焉”的贵妇们所不能做出的选择。昭君将被当作那金色的图腾,被人们供奉于心灵的顶端。‎ ‎(三)岳飞的选择 历史,被悲壮打磨得锃亮,辉煌了岳飞的一生。他不与秦桧同流合污,驰骋“八千里路云和月”中,然而无奈于十二道金牌的催逼,最终被害于风波亭。面对恶势力的威逼,他选择了国家,这是心灵的选择,虽然是要付出代价的,然而他无悔,他用自己的生命谱写了一曲高昂的旋律,射在历史的回音壁上,筝音四射。‎ ‎(四)天光甚美,造化有情 ‎“石蕴玉而山辉,水怀珠而川媚。”当真、善、美的情操已融入你的心中时,你也就有了极致的心境,当莎士比亚笔下的《威尼斯商人》中安东尼奥用生命作代价去借钱帮助好友时,当《巴黎圣母院》中敲钟人卡西莫多用金子般的心去帮助别人时,当《悲惨世界》中冉阿让追求光明与真理时,他们都选择了生命中最可贵的东西——人性的美,真理早已在他们心中留下了烙印,永不泯灭地存于世人心中了。‎ ‎《圣经》“马太福音”中耶稣告诉弟子要用真、善、美去衡量做人的标准。是的,只有真与美的交汇,美与仁的交融,灵与魂的合奏,才是真正的人生心声。‎ 天光甚美,造化有情。面对生活中的种种选择,我们掬一捧心灵的泉水,细细咀嚼其中的甘甜,读王昭君,读岳飞,读安东尼奥,读耶稣,读他们面对每一次困难做出的选择,读他们那高尚的灵魂,就是在酿造你的人生。‎ 诗曰:我思故我在,我在故我思。‎ 我曰:因真、善、美,故我在,我在,故我思。‎ 答:___________________________________________________‎ ‎_______________________________________________________‎ ‎_______________________________________________________‎ ‎_______________________________________________________‎ 答案精析 这是一篇文采四溢、构思精巧的议论性散文。该考生紧扣“心灵的选择”,首先,站在历史的高度,选用和亲使者王昭君和抗金英雄岳飞两个典型人物,并重点剖析他们在重要的历史关头所做的心灵的选择,以抒情的笔调揭示出英雄人物的正确选择来自他们高尚的心灵的观点。其次,该考生又从文学大师的笔下选用安东尼奥、卡西莫多、冉阿让这些文学形象,剖析他们在关键时刻选择的是生命中最可贵的东西——人性美。这样写作,既有点上的深入,又有面上的扩展,从而使文章内容紧扣话题,不蔓不枝,并且丰富而深刻。‎ 另外,本文极讲究结构形式的安排,四个小标题各领起一部分,而各部分的前后顺序又极有议论文的逻辑性,“昭君的选择”和“岳飞的选择”是点上的深入剖析,写文学大师笔下的文学形象是面上的铺叙,它们从不同的层次、不同的侧面,证明了“正确选择来自高尚的灵魂”的观点。这是本文引起阅卷老师好感的重要因素。‎ 研读本文,考生可从中发现写议论性散文的一些重要技巧:立意紧扣话题并写出高度和深度,讲究形式安排并注意体现出议论的逻辑性,拓展思维空间并展示个性才华。当然,本文在语言运用方面,在“引经据典”方面,也颇有一定的功夫,显示了该考生良好的语言和文学素养,也值得借鉴。‎ ‎2.阅读下面的材料,写一篇不少于800字的议论性散文。‎ 一个人可以走得很快,但不可能走得很远,只有一群人才能走得更远。(非洲谚语)‎ 在这人世间有些路是非要单独一个人去面对,单独一个人去跋涉,路再长、再远,夜再黑、再暗也得独自默默地走下去。(席慕容)‎ 立意分析 审题:非洲谚语强调“但”后面的内容,即一个人不可能走得很远,一群人才能走得更远。它告诉人们:做事要有合作、团队意识,众人智慧胜于个体力量,成功来自集体的团结、协作努力。席慕容的话强调个体走自己的路的重要性和必要性,人生之路有些地方可以依靠别人,但有些路必须自己单独去走、去体验,即使路上有坎坷、挫折,也要坚持走下去,因为,这是人生必走之路,是不可缺少、无法替代的成长之路。‎ 两则材料说法各有侧重,可以把两则材料综合起来思考,也可以立足一则,兼顾另一则思考,但不能完全把两则材料对立起来看待、理解。‎ 佳作欣赏 行路人生 我在天光初透的草舍醒来,只有我一个人。我踌躇着,不知是在这里等待抑或独自上路。我面对着苍茫荒野,大声道:“如果你想走得快,一个人走;如果你想走得远,一群人走。那么你告诉我,该孤独上路还是结伴而行?”荒原里一片岑寂。猎猎风声似要撕魂裂魄。‎ ‎“你的心意我无从知晓。但我势必选择孤独上路。”你的声音响起,震得我惊喜交加。我溯着你的声音之源,忆起你笔下祥林嫂的凄惨与抗争。‎ 你说,凡是一人有了新主张,得了赞同,是促其前进的;得了反对,是促其奋斗的。若呐喊于生人前,而生人不语,既非赞同,也无反对,这是怎样的悲哀啊!你于是以之为寂寞。我了解你的抗争。在那样一个黑暗年代,你以笔为戟,独自上路,竟生生地把黑暗撕出一片光明来。国人在光亮中惊醒。你走得比世人都快了些,你路上的寂寞亦如夜夜搓揉灵魂的猛兽。你的孤独只被你在拈笔一笑中了然。你用笔,用鲜血,用担当一切的勇气,为世人引路。你身后的一群人,也因此走得更快,更快。我对着荒野,微笑着说:“先生,你是勇者。”‎ ‎“当众人齐集河畔高声歌唱生活,我必定返回空无一人的山峦。”我坐在地上,听着海子微微的吟诵。‎ 你说,我不愿与他人在熙攘中前行,我宁愿独自踏上心灵之路。无关快慢,无关远近,只愿收割路的幽深,面朝大海,春暖花开。我了解你这种“只可自怡悦,不堪持赠君”的心境。你在一曲排箫中走上自己的路途。一如那篱落菊花、梅妻鹤子。你以身奏的梵音,是诱人入梦的,只有执着于心灵之路的人才梦得进去。清茶布衣,麝香酴醾,乐得出世客的逍遥。‎ 荒野里回声渐起,从“考盘在涧,硕人之宽。独寤寐言,永矢弗谖”到“从听世人权似火,不能烧得卧云心”;从“众人之途,当为远行”到“我与世界一同上路”。我在一片响声中释然。‎ 行路人生,总须有些甘愿领受寂寞独自开路的个人,他们走得快,经受的磨难也更多,但他们不仅觅得本心,也为后来的结伴行人开辟出小道。一群人相互扶持,势必走得更快更远。正是这样一群人,垒起了人类文明的金字塔。‎ 我拾起背包,决定孤独上路。‎ 出彩理由 这是一篇议论性散文,文章脉络清晰,开头写了自己的踌躇:“不知是在这里等待抑或独自上路。”中间用第二人称写了鲁迅和海子的启示,结尾再写“我拾起背包,决定孤独上路”。文章紧紧扣住题目“一个人走”与“一群人走”的关系来写。文章既展现了作者广阔的阅读面,作者对鲁迅、对海子都有深入的了解,对他们的作品、对《诗经》等经典也有独到的体会;文章还展现了作者深厚的语言功底,比如“在那样一个黑暗年代,你以笔为戟,独自上路,竟生生地把黑暗撕出一片光明来”“你走得比世人都快了些,你路上的寂寞亦如夜夜搓揉灵魂的猛兽”等语句就非常形象生动,非常有感染力。‎ 模板四 记叙文模板 一、写人有个性 描写人物个性要鲜明,应百人百面目,千人千形象。写人的文学,若抓不住人物的个性,千人一面,千篇一腔,谁知道你是在写谁呢?要抓住个性记述人物,应做到以下几点:‎ ‎1.抓特征,以形传神 每个人都有自己的性格特征,在生活中人与人千差万别,千人千样,描写时要善于发现和抓住对象与众不同的独特之处。且不说外貌、语言、动作,就是性格看来相似,实际也有很大的差别。‎ ‎【边练边悟】‎ ‎1.阅读下面的精彩文段,分析鲁提辖是如何给人深刻印象的。‎ ‎……鲁达再入一步,踏住胸脯,提起那醋钵儿大小拳头,看着这郑屠道:“洒家始投老种经略相公,做到关西五路廉访使,也不枉了叫作‘镇关西’!你是个卖肉的操刀屠户,狗一般的人,也叫作‘镇关西’!你如何强骗了金翠莲?”扑的只一拳,正打在鼻子上,打得鲜血迸流,鼻子歪在半边,却便似开了个油酱铺,咸的、酸的、辣的一发都滚出来。郑屠挣不起来,那把尖刀也丢在一边,口里只叫:“打得好!”鲁达骂道:“直娘贼!还敢应口!”提起拳头来就眼眶际眉梢只一拳,打得眼棱缝裂,乌珠迸出,也似开了个彩帛铺,红的、黑的、紫的都绽将出来。两边看的人惧怕鲁提辖,谁敢向前来劝。郑屠当不过,讨饶。鲁达喝道:“咄!你是个破落户!若只和俺硬到底,洒家倒饶了你!你如今对俺讨饶,洒家偏不饶你!”又只一拳,太阳上正着,却似做了一个全堂水陆的道场,磬儿、钹儿、铙儿一齐响。鲁达看时,只见郑屠挺在地上,口里只有出的气,没了入的气,动弹不得。‎ 鲁提辖假意道:“你这厮诈死,洒家再打!”只见面皮渐渐的变了。鲁达寻思道:“俺只指望痛打这厮一顿,不想三拳真个打死了他。洒家须吃官司,又没人送饭,不如及早撒开。”拔步便走,回头指着郑屠尸道:“你诈死,洒家和你慢慢理会!”一头骂,一头大踏步去了。‎ 答:___________________________________________________‎ ‎_______________________________________________________‎ 答案精析 鲁达拳打镇关西写得十分有个性,绘声绘色,与众不同,三拳打出三个样。正由于作者对鲁达这三拳描绘得特征显露,个性鲜明,因而给人以深刻的印象,数百年来广为流传。‎ ‎2.诉心声,提示思想性格 人物思想性格的塑造离不开内心世界的描写。一是直接描写人物的内心活动,即直接的心理刻画,写人物怎么想,怎么感觉;二是间接描写,就是借助人物的外部表现如语言、动作、肖像来反映人物的内心,间接进行心理描写,不能说一些浮泛的空话,要能把内心深处的精妙倾诉出来,使人物的思想性格得以深刻地表现。‎ ‎【边练边悟】‎ ‎2.写“我要对妈妈说句心里话”时,不少同学都只是笼统地写妈妈的爱,或者就写我病了妈妈冒雨送我上医院,半夜为我盖被子,下雨天来学校为我送伞……缺乏细节和心理描写,显得十分平常。请你摒弃常态,选取几个典型的细节来和母亲倾诉心声吧。‎ 答:___________________________________________________‎ ‎_______________________________________________________‎ ‎_______________________________________________________‎ ‎_______________________________________________________‎ 答案精析 (示例)妈妈,你曾经用手指在我的手心画了一个太阳,对我说:“当你感到困惑时,打开手心,看看这个太阳,就能战胜困难,排除困惑。”当时我用一双清澈的眼睛看着你,问:“为什么?”但你没有回答,只是用你的手掌紧紧握住了我的小手。我只觉得手上有一股热流,很温暖,你我相对而笑。现在的我终于明白,那手心的太阳是你用爱画的,是你给我的爱。每当我感到沮丧、困惑时,打开手心我就能看到那个你给我画的太阳,就有了战胜困难的力量。妈妈,我时时不敢忘记你给我的那一份爱啊!‎ ‎3.炼语言,突出个性特点 言为心声,准确而逼真地写出人物的语言,能生动地表现人物的思想性格。语言描写要切合人物的身份,要个性化,否则难以表现内心世界。老舍在《我怎样学习语言》中说:“对话就是人物的性格等的自我介绍。”对话巧妙,则无须描写人物的模样,就能使读者好像目睹了说话的那些人。鲁迅的《聪明人和傻子和奴才》,通篇是对话描写,通过对话,聪明人、傻子、奴才这三种人的思想性格活脱脱地展现在读者面前。‎ 人物语言要简洁,拖泥带水、冗长空洞是大忌;人物语言要个性化,因为语言是人物内心世界的流露,千人一腔,没有个性,也是大忌。教师有教师的语言,学生有学生的语言,工人有工人的语言,农民有农民的语言,每个人有每个人的性格,众人一个腔一个调,用词人云亦云,绝不可能把人物写活。‎ ‎【边练边悟】‎ ‎3.阅读下面的精彩文段,分析王熙凤是如何给人深刻印象的。‎ 一语未了,只听后院中有人笑声,说:“我来迟了,不曾迎接远客!”黛玉纳罕道:“这些人个个皆敛声屏气,恭肃严整如此,这来者系谁,这样放诞无礼?”心下想时,只见一群媳妇丫鬟围拥着一个人从后房门进来。这个人打扮与众姑娘不同,彩绣辉煌,恍若神妃仙子:头上戴着金丝八宝攒珠髻,绾着朝阳五凤挂珠钗;项上带着赤金盘螭璎珞圈;裙边系着豆绿宫绦,双衡比目玫瑰佩;身上穿着缕金百蝶穿花大红洋缎窄裉袄,外罩五彩刻丝石青银鼠褂;下着翡翠撒花洋绉裙。一双丹凤三角眼,两弯柳叶吊梢眉,身量苗条,体格风骚,粉面含春威不露,丹唇未启笑先闻。‎ 答:___________________________________________________‎ ‎_______________________________________________________‎ ‎_______________________________________________________‎ ‎_______________________________________________________‎ 答案精析 《林黛玉进贾府》一文中描写人物技法特别高超,王熙凤出场的文字尤其脍炙人口,语言描写也极为精彩。未见其人,先闻其声。以黛玉所思,贾府严肃的气氛从侧面烘托王熙凤的泼辣、豪放;同时在这样严肃的氛围里,王熙凤可以这样说话,也能够体现出王熙凤的位高权重及深得贾母喜爱。随后作者对其进行细致的刻画,由头到脚,使一个工于心计、身量苗条、体格风骚的少妇形象跃然纸上。‎ ‎4.给行动,描细节,形神兼备 要表现人物鲜明的个性,须重视细节的描写,借一斑以窥全豹,细节虽小,但作用不小,它在刻画人物形象时常起传神作用。为此,选择“一斑”要别具匠心,要确实能反映“全豹”,反映人物的思想性格和精神面貌,服从人物塑造的需要,服从主题表达的需要。‎ ‎【边练边悟】‎ ‎4.有位同学写命题作文《我的歌》时,写了一首“用父亲的血为词、我的泪为曲”谱写而成的歌。阅读下面他对“背尸匠”父亲的刻画部分,试分析他是如何写出父亲的个性的。‎ 在我一而再、再而三的要求下,父亲才从老家来到我的新房。上楼时,父亲显得十分吃力,我伸出手准备拉他一把,谁知他倏地把手缩了回去,只笑着说了一句:“不必了。”‎ 到新房时,我高兴地打开门,对父亲说:“瞧!”并随手拿了一双拖鞋让父亲换上,父亲刚准备脱鞋,随即又停了下来,用袖子揉了揉眼说道:“我站在门口看看就心满意足了。”‎ 答:___________________________________________________‎ ‎_______________________________________________________‎ ‎_______________________________________________________‎ ‎_______________________________________________________‎ 答案精析 作者在对“背尸匠”父亲的刻画上,用足了细节描写:“倏地把手缩了回去”是因为父亲怕把手上背尸体的“晦气”传给儿子;“刚准备脱鞋”是父亲下意识的行为,也是他欣喜之情的自然流露,“随即又停了下来”是因为父亲怕把自己身上的“晦气”带进儿子的新房;“用袖子揉了揉眼”是因为眼中有老泪溢出。这是“欣慰”之泪,儿子终于让自己“在一村老小面前可以抬头做一回人了”(文中内容);这也是“辛酸”之泪,为了这一天,自己背了一辈子尸体,这一天等得也实在太久了。正是如此生动、细腻的描写,才使人物形象霎时丰满起来,不仅传情,而且传神。‎ 二、叙事有波澜 文似看山不喜平。在记叙性文学作品中,最吸引我们的,通常不是人物本身,而是故事存在的悬念以及曲折性。‎ ‎1.设一个突转 记叙事件时,顺着一个方向铺陈渲染,把读者的注意力和情感愿望吸引到这个方向发展的一种可能性上去,层层推向高潮;达到顶点时,笔锋陡然一转,通过另一种结局的突然揭晓,掀起波澜,使读者对事件意义的理解推向一个新的高度。用此方法构思情节,在结构上可形成奇峰突起、江河陡转之势。如《项链》《最后的常春藤叶》都使用了这种技巧。‎ ‎【边练边悟】‎ ‎5.阅读下面这篇小小说《让座》,分析其在情节方面的特点。‎ 让座 我站在无轨电车里,身旁有个老大娘,两只手扶着椅背也是站着。旁边的座位上却大模大样地坐着一个15岁上下的小伙子。使劲盯着窗外,仿佛生平头一回见到这辆无轨电车沿途经过的街道似的。‎ 我开口对老大娘说话了,其实是说给那个麻木不仁的小伙子听的。‎ ‎“唉,现在的年轻人可真缺乏教养!”‎ ‎“说得是啊,说得是啊,”老大娘点了点头,“就是没有教养嘛!”‎ ‎“学校里是怎样教他们的!”‎ ‎“说得就是嘛,学校里是怎么教他们的!”老大娘同意我的说法。‎ ‎“大概他们的父母也是这种没教养的人。”我瞪着那小伙子说,可他却无动于衷。‎ ‎“有什么样的爸爸,就有什么样的儿子。”老大娘叹着气说。‎ ‎“真不像话!年轻力壮的小伙子坐着,却让老太太站在那里!”我的声音已经很高了。‎ 老大娘一会儿看着我,一会儿又看着那个小伙子。‎ ‎“喂,你这小青年,”我终于忍不住碰他的肩膀,“说的就是你哪,还不给老人让座!”‎ ‎“你拽这孩子干什么!”老大娘突然冲我嚷起来,“你回家去教育自己的孩子好了,我的孙子你可别管!”‎ 答:___________________________________________________‎ ‎_______________________________________________________‎ ‎_______________________________________________________‎ ‎_______________________________________________________‎ 答案精析 这篇小小说的主要特点是顺势层层铺垫,结尾“情节突转”,令人拍案叫绝!一般而言,读者以为小说的主旨在鞭挞年轻人缺乏教养,呼唤传统美德的回归;读到最后,才知主要是讽喻老太太那种“叶公好龙”的滑稽,对社会公德的缺失深恶痛绝,但对自己的儿孙却纵容娇惯。前面,“我”每表示不满甚至愤慨,老太太总有附和,当“我”‎ 最后高声正面指斥那个年轻人时,有一句对老太太神情的描写,这是一处藏得很好的近伏,这也表明了一个道理——伏笔宜藏,妙在伏得不注意不显露。这样,最后的转折才能陡峭而不突兀。‎ ‎2.留一点悬念 扣人心弦的悬念可使读者产生追根究底的阅读冲动,收到“踏破铁鞋无觅处,得来全不费功夫”的神奇的构思效果。如“先闻其声后见其人”“犹抱琵琶半遮面”“一切都在情理之中,一切又都在意料之外”等等,都会让读者产生一种非看下去不可的强烈冲动,从而获得引人入胜的效果。‎ ‎【边练边悟】‎ ‎6.阅读下面这篇记叙文的开头,分析其特点和好处。‎ 我的视线,这么久以来都不得不为一把蓝色的雨伞所牵绊……(《记一把蓝色的雨伞》)‎ 答:___________________________________________________‎ ‎_______________________________________________________‎ ‎_______________________________________________________‎ ‎_______________________________________________________‎ 答案精析 本文的开头言简意赅,但是又颇有意韵。“我的视线,这么久以来都不得不为一把蓝色的雨伞所牵绊……”为什么会这样?作者没有明说,而是用了一个省略号,巧妙地设置了悬念。这自然而然地激发起了读者关注的兴趣,让人不禁思考:这把蓝色的雨伞在作者身上到底会发生怎样的故事?这样的开头,无疑会起到先声夺人的效果。‎ ‎3.来一点对比 对比(衬托)法可使人辨真伪,明是非,识好坏,促人冷静地思考,做出正确的判断。如《三国演义》“赤壁之战”一节,除了以刻画鲁肃的老实反衬诸葛亮的机智外,更以描写周瑜的聪明机智正面衬托诸葛亮的计高一筹。这样正反映衬的交互运用,不仅可以使诸葛亮的形象显得更加突出,还可以使行文灵活,富有动态感。‎ ‎【边练边悟】‎ ‎7.阅读下面这篇小小说《捎》,分析其在情节方面的特点。‎ 捎 他要出国的消息不胫而走。回到家里,屋里早坐满了人。弟弟兴奋地说:“哥,给我捎台收录机!”妹妹是那么激动:“哥,给我捎一套高级化妆品!”他问丈母娘:“妈,您捎什么?”丈母娘嘴都快咧到耳朵根底下,道:“不捎别的,捎台洗衣机就行!”当他问自己的母亲要捎点儿什么时,母亲眼里闪着晶莹的泪光:“不管咋着,把你捎回来就中!”‎ 答:___________________________________________________‎ ‎_______________________________________________________‎ 答案精析 文章起笔处看似平静,却为后面的陡转做了充分的铺垫,结尾处母亲的回答增强了文章的情感震撼力。‎ ‎4.用一点抑扬 抑扬手法的运用,有欲扬先抑法和欲抑先扬法。运用抑扬手法,可以使文章显得波澜起伏,避免了行文的平庸呆板,单调乏味。这种明抑实扬、欲扬先抑的表现形式,可使作品含蓄而奔放,能让人产生柳暗花明的情致。‎ ‎【边练边悟】‎ ‎8.阅读下面的小小说《求职始末》,试分析其精妙的记叙手法。‎ 求职始末 大学毕业才两年的舒平下岗了。‎ 为了生活,舒平不得不四处奔波,终于一家有朋友的公司答应他去应聘。在复试时,他与一位业务经理因意见上的分歧而发生了争执,他那桀骜的个性使他的言语犀利而偏激,令经理十分尴尬,结果他落聘了。‎ 舒平连遭重创,心灰意冷。想到年迈的父母正巴巴地盼着享儿子的福,想到相恋两年的女友因自己境况不佳而常闹分手,想到自己才华横溢而无处施展,他绝望了,踏上了一条不归路……‎ 恰在此时,那家公司的朋友找到他,将他从死神身边拽了回来,同时带给他一张聘请书。原来那位经理经过冷静的思考,认为舒平是个不可多得的人才,决定高薪聘用他。舒平躺在病床上,通过电话与家人分享自己的喜悦。‎ 第三周星期一,舒平早早地来到公司门前,却见一张通告赫然映入眼帘:“鉴于舒平先生心理承受能力太差,本公司决定予以解聘……”‎ 答:___________________________________________________‎ ‎_______________________________________________________‎ 答案精析 文章采用了抑扬交错的手法,把故事记叙得波澜起伏,引人入胜,令人瞠目。‎ 三、画龙有点睛 画龙点睛,即记叙文“夹叙夹议”的手法。“画龙”,是指通过叙述、描写刻画人物、事物或景物;“点睛”,是在叙述、描写之前或之后的关键地方,用精辟的词句点明主题,从而使内容更加生动有力。一篇记叙文,如果没有一两句光芒四射、哲理闪烁的点睛之笔,文章就会平淡无力。‎ ‎【边练边悟】‎ ‎9.习作《生活是一首歌》,作者在叙述生活中的“苦”处之后,写下下面的语句,请简要分析这样结尾好不好。‎ 但是我知道,“生活是一条路,怎能没有坑坑洼洼”“生活是一杯酒,怎能没有酸甜苦辣”。我有什么理由怨叹生活的不公呢?‎ 答:___________________________________________________‎ ‎_______________________________________________________‎ 答案精析 这段精辟的议论,借用电视剧《篱笆·女人和狗》插曲中的歌词,将自己对生活的认识由感性升华到理性,深刻认识到生活是充满矛盾的,需要人们去搏击。‎ ‎【文题】 根据以下材料,写一篇不少于800字的文章。‎ 智慧是一种经验,一种能力,一种境界……‎ 如同大自然一样,智慧也有其自身的景象。‎ 要求:选好角度,确定立意,明确文体,自拟标题;不要脱离材料内容及含意的范围作文,不要套作,不得抄袭。‎ 佳作欣赏 王大爷的早餐店 我住在一个有些年头的小区里。住在老小区的好处就是——你知道小区里哪个店铺里的东西好。‎ 我知道王大爷卖的早饭最好吃!‎ 王大爷的小店里总是挤满了很多人。远远的只能看到雾气把小屋子填得满满的,溢到外面来。也分不清朦胧中哪些是人,哪些是桌。‎ ‎“王大爷!老样子来一份儿!”‎ 进门的人大多要喊这么一句,就像打招呼一样点一份早饭,然后坐下来跟经常碰面的“早饭友”开心地交谈。等到王大爷把东西稳稳地放在他面前,他就开始边吹边吃,满脸幸福的样子。不消几分钟,他就要向在座的告别——“你们慢吃”,再给王大爷来个预订:“我明儿还来啊,您忙!”然后快步走了——去上班。‎ 王大爷性格特别好,讲话和气还很爱聊天。笑眯眯的,我甚至觉得他的皱纹都特别可爱!他的店面小,装修也比不上别处的快餐店,桌椅因长年的油灰显得发黑,墙也因水汽好些地方脱了墙皮。但是不知道为什么大家都爱在这儿喝粥吃茶叶蛋。好像有了王大爷在,早饭就该好吃,卫生就能保证似的。事实也的确如此!‎ 吃客们有周末不肯做饭的,也跑来吃早饭。这时就少不了王大爷爽朗的笑声了。‎ ‎“王大爷,店面不租大点我们要站外面吃饭啦!”‎ ‎“哈哈,我给你们搬个小凳坐外边吃!哈哈……钱全给你们做粥吃了,哪有闲钱啊?”‎ 对,王大爷的粥里头有肉,价格还不贵哩!难怪,难怪!‎ ‎“那你提个五毛一块的价,我们还能不来啊?”‎ ‎“‎ 不行不行,都是好多年在这儿吃的老邻居了!我老王别的不敢说,良心还是有的啊。多亏你们养着我哩!哈哈,我老头子亏得有你们陪着解解闷呢。”‎ ‎“那大妈呢?”我这个心直口快的傻孩子没忍住就问了出来。我发现周围似乎过分的安静。我也看到王大爷似乎笑容僵了下来。‎ ‎“你大妈啊,陪着我呢!可不就是她派你们来陪陪我嘛。我这就够啦!有吃有喝,有人说话,干啥不满足?想这想那,争来争去的人还没我快活哩。小妮子你知道不,人要学会知足!你往后长大了还像他们来我这小店陪大爷说话,我也知足啊,哈哈,你说我这傻不傻?”‎ 我听了,眼睛忍不住眨巴了两下,多亏店里雾气重!“这粥真烫!”我大声地说,“我以后得等粥冷点再喝!”‎ 周围的人都笑了,王大爷也笑了,这种没有任何做作的笑声真好听。‎ 王大爷,你一点也不傻,你就是心里敞亮才过得好啊!我不管长多大,都会时常来看你的,陪你说说话,就像这些陪你的人一样!我心里想。‎ 周末的早上总是让人放松的,阵阵谈笑声从街角这家小小的早餐店里传出……‎ 出彩理由 本文写小区里一位卖早餐的老人。老人多年来支撑一间简陋的早餐铺子,全心全意地为邻居们服务。文章写到后来忽然点出这位老汉早已失去老伴,贫寒而又孤寂。他没有消沉,而是用精心制作早餐来“团结”邻居们的欢声笑语和质朴真情,他活得有滋有味。这种“圆通”和“豁达”,乃是“活”用小经验、小能力而展示出来的小智慧,富于启示,全文也就静悄悄地与“智慧”水乳交融了。‎ ‎1.阅读下面这篇高考优秀作文,分析其开头和结尾有何妙处。‎ 一碗大米饭 幸福如一抹浅浅的雾,淡淡地飘在天涯,飘在村东的路口,与那个破旧的屋里的大米饭的香味融合,弥漫在我的心窝。为了它,我曾等了一个夏天,等了一个童年。而此刻,它竟像一笛箫音,轻轻回响在我心灵的深处……‎ 小时候,家里很穷。穷人的天空在记忆中是灰白色的,而穷人总是有一双哀伤的眼睛,那是在他的孩子们咕噜咕噜地喝稀饭的时候。‎ 那年,外婆生病了,为了外婆,母亲借来了一升大米,细粮在当时是很奢侈的东西。每天早晨,母亲蒸一碗白稠稠的大米饭,然后一口一口给外婆喂下。每到此时,我就蹲在墙角咽口水。孝敬的定义就是把好东西留给外婆吃吗?那时,我常常这样想,直到有一天……‎ 那天,天气晴朗得可爱。早上,照例是一碗大米饭和我们的稀饭,而外婆因身体不舒服没有起来吃饭,母亲急着去地里,临走时叮嘱我喂外婆。望着端过饭碗的瘦瘦的我,外婆说:“乖囡,外婆不想吃,你去吃了吧,啊?”外婆和蔼地抚着我的头。“哎!”我满心欢喜,忙不迭地端走了那碗我梦寐以求的大米饭。但就在这时候,一种莫名的感觉涌上心头,简单的思维告诉我:“这是外婆的饭,我不能吃!”就这样,我痴痴地盯着那碗大米饭,窗外的阳光,小鸟鸣啾,全都变成了一碗碗的大米饭,在我眼前晃动。有好几次,我都差点忍不住了,一碗大米饭的诱惑力当时真是不亚于吸引一根针的那块大磁铁,而它又仿佛一个女妖,美丽地谄媚地笑着,想要一口把我吸进去似的。而我就端坐在那儿,在吃与不吃之间徘徊,耳边不断地响着一句话:外婆的饭,我不能吃……‎ 门“吱”地被母亲推开了,我扭过头去,看着母亲:“娘,外婆的大米饭我没吃。”我的嘴一撇,想哭。母亲怔怔地看着,突然像明白了什么,紧紧地抱着我,大滴大滴的泪水滴在我的脖子上。那一刻,我知道,我做对了;那一刻,我好像突然长大了。因为我终于可以承载着母亲的眼泪了。窗外的阳光斜射进来,暖暖地照在我和母亲的脸上,屋里一片淡淡红色,在这静谧中,我尽情地分享着母亲的泪光……‎ 那一年,我8岁。小小的我,像是经受了一次灵魂的洗礼,没错,就是这样的。我清楚地记得,外婆去世的那天晚上,我梦见外婆就站在我面前,手里捧着一碗香喷喷的大米饭……‎ 答:___________________________________________________‎ ‎_______________________________________________________‎ ‎_______________________________________________________‎ 答案精析 这是一篇感人至深的记叙散文。饱含深沉的情思,产生了震撼人心的艺术效果。开篇“凤头”,连用两个形象的比喻,朦胧悠扬中进入回忆,引领下文。“豹尾”结束,“我梦见外婆就站在我面前,手里捧着一碗香喷喷的大米饭……”韵味悠长,令人潸然泪下,又照应了文章的开头。‎ ‎2.阅读下面的文字,根据要求作文。‎ 材料一 谁年轻的时候没有过梦想,谁年少的时候没有过榜样?好的偶像能够成就一代人的志向、理想和美好的精神家园,而不好的偶像则会误人志向、毁人前程。在引导崇拜偶像时,我们要大力呼唤和推出“英雄偶像”,逐渐将那些“反面偶像”逐出舞台。‎ 材料二 影视歌曲明星,总是引领着一个时代的流行潮流。无论是昙花一现的辉煌还是常青藤般的经久不衰,他们带给人们的震撼与激情,是可以穿透岁月和记忆的。每个人的成长过程中都有过类似的偶像情结,只不过因所处的时代不同,意义与价值观也随之大相径庭。‎ 请根据你对以上材料的理解和体会,选准角度,写一篇作文。要求:自选文体,自拟标题,不少于800字。‎ 立意分析 (1)审题立意:此作文题由两则材料构成,即我们平常所说的多材料作文。多材料作文一般由两则或两则以上的材料构成,比单则材料要复杂,不但需要弄清每则材料的大旨,还需要从整体上掌控材料与材料之间的关系。材料之间的关系主要有两类:一是几则材料并列,互为补充,对同一问题作多层面的描画与论说;二是对立统一,对同一问题提供相反相成的事例。‎ 仔细阅读这两则材料,我们发现,材料一所表述的内含是:好偶像与坏偶像对于人所起的不同的作用以及如何引导偶像崇拜的问题;材料二所表述的内含则是:每个人成长中都有偶像情结,且这个情结是随着时代的变化而变化的。‎ 通过对这两则材料的分析与对比,我们可以看出这两则材料都是围绕“偶像”来说的,其内容上互为补充,共同阐述了偶像情结的内涵以及如何崇拜偶像的问题。写作时,我们如能紧扣“偶像”来写,并突出“情结”一词,就会很容易地写出符合要求的作文来。‎ ‎(2)写作提示:“偶像”在很久以前就已经存在,“偶像情结”也是一个既年轻又古老的话题。仔细一想,我们便会发现这个文题所蕴含的思想底蕴与文化内涵极为深远。在这个经济飞速发展、信息化与个性化并存的年代,几乎每一个人都会有自我的“择偶”标准,有人敬仰革命领袖毛泽东,有人欣赏文坛巨匠高尔基,有人崇拜篮球之王乔丹,有人迷恋影视歌曲明星刘德华等。选择偶像,关键在于背后的精神风貌与价值取向,这是同学们要着重思考的问题。‎ 具体写作时,我们可以用提供的材料打开自己的思路。比如材料中说“谁年轻的时候没有过梦想,谁年少的时候没有过榜样?”“每个人的成长过程中都有过类似的偶像情结”,我们可以从这些话语中展开联想和想象的双翅,进而在古今中外的现实或文学艺术之中寻找创作的灵感,像屈原、李白、李清照、苏轼、鲁迅、海子以及秦皇汉武、唐宗宋祖等这些对中国文化或中国历史有着深远影响的作家、学者、英雄豪杰,也可以是艺术世界中的人或物。如维纳斯雕像、《老人与海》中的桑提亚哥、《简·爱》中的简·爱、《复活》中的玛丝洛娃等,都可以进入到你的视野。这样,写作素材一下子就多了起来。‎ 有了素材之后,我们就要精心构思了——写出“亏我想得出来”的作文。如下面的一些写作角度就比较好:(1)偶像情结折射出时代和大众文化的烙印;(2)偶像崇拜是人们精神的一种寄托;(3)偶像的言行举止会影响一代人或一个时期,不可小视;(4)崇拜偶像不能迷失了自己;(5)偶像能起到榜样的作用,成为偶像,需要巨大的人格魅力;(6)青少年更容易成为偶像的崇拜者,成为偶像也需付出努力和艰辛;(7)偶像是一把双刃剑等。‎ 另外,需要向同学们提醒的一点是,文章表现手法要力求新颖,别具匠心。如果仅仅拘泥于材料中所给的信息,是不大可能写出好文章来的,需要我们思接千载,神游万里!可以用心灵游历的方式寻找偶像,在寻找过程中,表达自己的心得和感受;也可以用偶像独白的形式或“陌生化”手法来揭示“伪偶像”的真实世界等。‎ 佳作欣赏 这,不是错误 郑愁予说,我达达的马蹄声是个美丽的错误,我不是归人,是个过客。我说偶像情结并不是错误,因为每个偶像身上都有值得我们瞩目的闪光点。‎ 在《东风破》中悲伤,在《龙拳》中奋起,并不是所有的潮流都一无是处。有人说,假如老一代艺术家是一颗小恒星,那么那些新生代明星充其量是一颗流星,闪亮是刹那间的,过后,只是灰烬。姑且相信这种说法,可流星不也有它的耀眼之处吗?至少,它燃烧过!‎ 不错,无数的男生女生为“超女超男”们疯狂,在大人眼中,这些另类幼稚的小小青年又算个什么?又有什么值得万千年轻人为之东奔西跑?也许他们更以为这无数的痴迷粉丝们只不过是一只只没有头脑的小雀,因为内心空虚和精神无处寄托而去追星。可是他们未曾想到,其实我们也有自己的主张!‎ 难道不是吗?周杰伦会创作,会演戏,会唱歌,有能力有孝心,这样优秀的人谁不喜欢?把他视为偶像,以此来激励自己,难道有错吗?‎ 更何况,纵观历史,古今中外,哪一个成功者的内心深处没有自己的偶像?‎ 苏轼说,家父就是我的偶像。或许得益于偶像的熏陶,我们才能读到他“大江东去,浪淘尽,千古风流人物”的人生豪迈,才能看到他“十年生死两茫茫”的真情忧伤,才能领略到他的豪放与婉约、洒脱与细腻的一个完整的人!‎ 三毛说,曹雪芹是我的偶像。或许正是由于对偶像的崇拜与追逐,才成就了《撒哈拉的故事》,由此我们领略到了一个奇女子在大漠中的爱情故事。‎ 陈景润说,歌德巴赫就是我的偶像。或许缘于偶像的激励,才有了“1+1=2”的完美证明,才有了一代数学鬼才的横空出世。‎ 牛顿说,前人都是我的偶像。正是因为有了前人的知识铺垫,才有了“牛顿定律”的产生,才让我们听到了那句“我之所以看得比别人更远,是因为我站在巨人的肩膀上”的至理名言。‎ 更或者说,苍鹰把蓝天视为偶像,因此才能搏击长空;鱼儿把大海作为偶像,因此才能畅游海洋!‎ 由此看来,偶像情结并非一无是处。或许,正是偶像孝心的感召,让平凡的人群中出现了谢延信;正是偶像拼搏精神的振奋,让我们有了航天英雄杨利伟;正是偶像对人生的执着,NBA赛场上杀出了一个姚明!‎ 偶像情结,好像并无错误!‎ 猎豹把风视为偶像,因此成就了它的风驰之速。‎ 嫩芽把巨树视为偶像,因此成就了它的伟岸身姿。‎ 所以,偶像情结,不是错误!‎ 出彩理由 这是一篇很有特色的议论文佳作。文章结构层次分明,语言优美。作者从郑愁予的《错误》谈起,引出自己的观点。行文中对偶像情结进行深入剖析,层层推进,谈崇拜偶像的作用,且广征博引,论据充足,论证有力。结尾采用排比句式结构语言,深化主旨。另外,本文在审题方面做得也很好。小作者抓住了“偶像情结”来谈偶像对于人的作用,特别是第五段“哪一个成功者的内心深处没有自己的偶像”的反问句,既引人深思,又自然地开启了下文。下文中的苏轼、三毛、陈景润的事例无不说明偶像对于人成长的指引作用,这样就鲜明地表现出“所以,偶像情结,不是错误!”的主题。‎ 学案五 2018年高考作文备考策略 一、素养方面 ‎1.端正思想态度 教育部明确要求高考要体现“一点四面”的基本原则,“一点”就是要在高考当中体现“立德树人”的导向,“四面”就是要在高考当中体现“核心价值、传统文化、依法治国、创新精神”四个方向。因此,备考2018年高考的同学们,首先应端正自己的思想态度:热爱祖国,关注社会,关爱他人,遵纪守法,修身养德,热爱并传承、弘扬传统文化,勤奋钻研,积极向上。考生应本着“立志做有理想、有道德、有文化、有纪律的人,立志为人民作贡献,为祖国作贡献,为人类作贡献”的远大理想,全力以赴,投入学习。‎ ‎2.夯实阅读积累 ‎“腹有诗书气自华”,高考作文备考,考生必须高度重视夯实自己的阅读积累。首先要读好、用好语文课本(包括必修与选修课本),充分用好教材选文中的素材,这是基础。其次,要有计划地进行课外拓展阅读,可以选择一些报刊杂志,诸如《读者》《青年文摘》《中国青年报》《南方周末》等,多阅读时文、美文,以开阔视野、陶冶性情;也可利用网络、电视、手机等多种平台或工具,进行数字阅读,及时关注社会时事热点、磨砺思想。此外,也要有选择地阅读一些人文经典名著。配合摘抄、随笔、读后感写作等,以丰厚阅读积淀。‎ 二、思维方面 ‎1.审辩性思维 审辩性思维是指逻辑清晰严密的思考。高考作文命题尽管在文体选择上给考生提供了自由选择的空间;但要注意的是,大多数材料作文试题更适合于论述类文的写作;有的作文试题还直接规定写成议论文,如2016年北京卷的“‘老腔’何以令人震撼”,还有浙江卷,也直接限定写成论述类文章。议论文写作,更侧重检测考生审辩性思维能力。考生在备考过程中,要重视训练审辩性思维。《中庸》有言:“博学之,审问之,慎思之,明辨之,笃行之。”这其中的“审问、博学、明辨”,即所谓的“审辩性思维”,要求学习者勇于独立思考、大胆质疑、小心求证,思要慎,辨要明。这就要求学习者具有开阔的视野,广博的知识,学思结合,知行合一。‎ ‎2.形象思维 又称艺术思维,是运用集中概括的方法,对感知的事物形象(表象)进行加工改造,借助完整具体的形象以揭示事物的本质,解释生活、表现生活的一种思维活动和方式。形象思维在作文的过程中占重要的位置,尤其是写记叙性、描写性的文章,形象思维能使反映的对象更本质化、更个性化。2016年北京卷的“神奇的书签”,要求写成记叙文,而且题干规定“展开想象”“有细节,有描写”,这些都指向了对考生形象思维的检测。复习备考中,也要做一些以记叙、描写为主要表达方式的文章写作训练。例如写人叙事、状物言志、借景抒情、扩写、改写等记叙类文章的训练。‎ 三、技能方面 高考作文复习备考,审题立意与构思谋篇是重要环节。“意犹帅也,无帅之兵,谓之乌合。”审题立意,是关键的技能。下面拟以2016年9道高考作文试题为例,分别解说各种类型特点的材料及命题作文的快速审题立意的技巧。‎ ‎(一)材料作文的审题 ‎1.从相异点入手,追因溯源 有时材料中呈现的现象、事实、问题、观点等相互之间存在相异点,审题立意则应从这些相异点入手,追因溯源,寻找立意切入点。‎ ‎[2016·全国卷Ⅰ]阅读下面的漫画材料,根据要求写一篇不少于800字的文章。‎ 要求:结合材料的内容和寓意,选好角度,确定立意,明确文体,自拟标题;不要套作,不得抄袭。‎ 思路点拨 ‎ 漫画中呈现的是获得不同考分的学生受到的奖惩。这一组四格漫画,有对比,有变化,考生应在对比与动态变化中寻找立意的突破口。获得满分与获得61分的,都获得了香吻,不同层次的分数获得一样的奖赏;得98分的与得55分的,将近满分的与快要及格的,却都获得了掌掴。何以如此?家长的评价标准及其动机使然。唯分数论英雄,自然不可取。陟罚臧否,是教育的艺术。奖与惩,终极目标是为了促进孩子的发展与进步。孩子学业成绩不可能永远是持续前进的,不可能永远满分,起伏变化,是必然的,而若仅以分数高低来实施奖惩,视野未免狭窄。由于接受能力、环境影响等多种因素的制约,孩子成绩暂时落后,未能及格,也不可只是施予简单、粗暴的体罚,就可了事;而应细水长流、从长计议。当然,还可以运用类比联想,由教育领域内的依考试成绩高低实施奖赏的现象,类比联想到其他方面的类似现象与事实,比如讨论对于员工、人才的激励机制,或讨论评价标准单一化的弊端及解决对策等问题。‎ ‎2.从矛盾处入手,深入挖掘 有时材料中给出相互矛盾的观点,那么审题时则可从矛盾处入手,深入挖掘,探究双方观点的合理性、片面性及互补性,在辩证分析中,引申出自己所写文章的主旨。‎ ‎[2016·江苏高考]根据以下材料,选取角度,自拟题目,写一篇不少于800字的文章;文体不限,诗歌除外。‎ 俗话说,有话则长,无话则短。有人却说,有话则短,无话则长——别人已说的我不必再说,别人无话可说处我也许有话要说。有时这是个性的彰显,有时则是创新意识的闪现。‎ 思路点拨 这则材料先引用俗语“有话则长,无话则短”,再引出一个对立的观点“有话则短,无话则长”,并予以简要解释,进而引出关键句“有时这是个性的彰显,有时则是创新意识的闪现”。审题立意时应扣住二者的矛盾关系来深入探究,确定立意。“个性”与“创新”是材料中的两个核心关键词,具有宽泛的内涵,考生在吃透材料中的矛盾对立的两句俗语含意的基础上,将“个性”与“创新”赋予具体的内涵。立意参考如下:无话则长,有话则短——对于前人尚未涉足的空白领域,要勇于探索、发现、创新;对于大家所熟知的,不简单重复,这种不浪费的理性值得点赞。有话则长,无话则短——实事求是,不虚夸,务本真,这种诚实谦和的个性值得褒扬;等等。‎ ‎3.从空白处入手,发散思维 有的材料在叙述事实或现象,或者在引导语表述时,会给考生留出空白,让考生依据自己的理解、体验与思考进行扩展、补充,从而形成自己的观点与想法。考生应从空白处入手,应采用化虚为实,化大为小,化抽象为具体等思维策略,来确定立意。‎ ‎[2016·山东高考]阅读下面的材料,根据自己的感悟和联想,写一篇不少于800字的文章。‎ 行囊已经备好,开始一段新的旅程。路途漫漫,翻检行囊会发现,有的东西很快用到了,有的暂时用不上,有的想用而未曾准备,有的会一直伴随我们走向远方……‎ 要求:①选准角度,自定立意;②自拟题目;③除诗歌外,文体不限;④文体特征鲜明。‎ 思路点拨 材料中的关键词是“行囊”与“旅程(路途)”,立意构思可以从“行囊”的隐含意入手,化虚为实,赋予其具体的意义所指;并从“行囊”与“旅程”的关系这一角度来立意。根据材料中的引导语“有的……有的……有的……有的……”的提示,考生应将“有的”之所指具体化,形成所写文章的话题及主旨。这道作文试题立意空间是开阔的。参考立意如下:(1)“书到用时方恨少”,要不断储备知识的行囊,才能少走弯路;(2)对“真、善、美”的追求,是人生旅途不可或缺的行囊;(3)“读万卷书,行万里路”,人生旅途中应该准备经典的行囊,与名著相伴随,充实自我的人生;(4)未来岁月不可期的东西多矣,准备行囊要未雨绸缪;(5)顺时而动,与时俱进,要及时更新、补充人生行囊中的装备;(6)诚信(或乐观、恒心等)是人生旅途的重要行囊,没有它,就难以致远。‎ ‎4.从因果处入手,探究本源 有的材料所述的是一个含有多重因果联系的事件,考生应梳理出事件的来龙去脉,抓住因果关系来审题立意。可以由果溯因,也可由因及果,从而确定立意。‎ ‎[2016·全国卷Ⅲ]阅读下面的材料,根据要求写一篇不少于800字的文章。‎ 历经几年试验,小羽在传统工艺的基础上推陈出新,研发出一种新式花茶并获得专利。可是批量生产不久,大量假冒伪劣产品就充斥市场。小羽意识到,与其眼看着刚兴起的产业这么快就走向衰败,不如带领大家一起先把市场做规范。于是,她将工艺流程公之于众,还牵头拟定了地方标准,由当地政府有关部门发布推行。这些努力逐渐见效,新式花茶产业规模越来越大,小羽则集中精力率领团队不断创新,最终成为众望所归的致富带头人。‎ 要求:综合材料内容及含意,选好角度,确定立意,明确文体,自拟标题;不要套作,不得抄袭。‎ 思路点拨 这则材料所述事件大抵包含以下四个层次:(1)小羽改进、创新传统工艺,研发新式花茶;(2)新式花茶批量生产,大量假冒伪劣产品充斥市场;(3)公布工艺流程,拟定地方标准,带领大家规范市场;(4)新式花茶产业规模扩大,集中精力率领团队创新,成为致富带头人。这四个层次存在因果关系。这些因果关系中关涉“创新”“规范”“个体”“团队”“合作”“分享”等明示的或隐性的关键词。小羽何以最终成为众望所归的致富带头人?至少有以下几方面的原因:(1)敏锐的创新意识;(2)精明随机应变的市场头脑;(3)自觉的市场规范(标准、规则)意识;(4)合作意识与团队精神;(5)分享与共赢;(6)协作精神与带头(领袖)意识;(7)革新观念引领发展;(8)“风物长宜放眼量”,要有宽广的胸怀与高远的眼界;等等。通过因果关系的梳理,找出材料中潜在的关系型话题,进而在相互关系中确定立意,形成所写文章的主旨。‎ ‎5.从关键句入手,纵深开掘 材料中往往有提示材料主旨内涵、暗示立意思维指向的关键句,这些关键句或以议论句,或以材料中人物的对话等形式呈现,考生在审读材料时,应在整体把握和感知材料内涵的基础上,紧紧抓住关键句,纵深开掘,确定立意。‎ ‎[2016·全国卷Ⅱ]阅读下面的材料,根据要求写一篇不少于800字的文章。‎ 语文学习关系到一个人的终身发展,社会整体的语文素养关系到国家的软实力和文化自信。对于我们中学生来说,语文素养的提升主要有三条途径:课堂有效教学、课外大量阅读、社会生活实践。‎ 请根据材料,从自己语文学习的体会出发,比较上述三条途径,阐述你的看法和理由。‎ 要求:选好角度,确定立意,明确文体,自拟标题,不要套作,不得抄袭,不得泄露个人信息。‎ 思路点拨 这道作文试题的引导语由两句话构成。前一句是说语文学习、语文素养的重要性。后一句是说中学生语文素养提升的三条途径。关键句在于后者。题干要求考生“从自己语文学习的体会出发,比较上述三条途径,阐述你的看法和理由”。考生应针对“课堂有效教学”“课外大量阅读”“社会生活实践”这三条途径进行比较分析,结合自己的学习经历、体会来谈。题干标明“明确文体”,没有限定文体,考生在文体选择上有充分的自由度,但文体要鲜明。题干中有“比较上述三条途径”和“阐述你的看法和理由”的写作任务指令,考生立意时不可忽视,要对上述三者在语文学习中的地位进行必要的权衡、比较,从而形成自己的看法。立意参考如下:(1)“课内打基础,课外求发展”,语文学习,课堂有效教学是基础,课外大量阅读与社会生活实践是补充;(2)“读万卷书,行万里路”,单靠课堂教学,语文素养难以提升,大量的课外阅读与丰富的社会生活实践,是提升语文素养的不二法门;(3)“读书破万卷,下笔如有神”,提高语文素养的捷径是大量的课外阅读,要好读书、读好书、书读好,才能夯实语文素养;(4)“语文学习的外延与生活相等”,要学好语文,应当自觉在社会生活实践中运用语文;等等。‎ ‎6.从限定处入手,锁定焦点 对于“材料+话题”型的材料作文试题,考生除了要认真审读材料引导语之外,还应对话题短语进行语法分析,从中心语与修饰或限定语的关系入手,发现立意的突破点,从而确定所写文章的主旨。‎ ‎[2016·天津高考]请根据下面的材料,写一篇文章。‎ 在阅读方式多元化的今天,你可以通过手机、电脑等电子设备,在宽广无垠的网络空间中汲取知识;你可以借助多媒体技术,“悦读”有形有色、有声有像的中外名著;你也可以继续手捧传统的纸质书本,享受在墨海书香中与古圣今贤对话的乐趣……‎ 当代青年渴求新知,眼界开阔,个性鲜明,在阅读方式的选择上不拘一格。请围绕自己的阅读方式,结合个人的体验和思考,谈谈“我的青春阅读”。‎ 要求:①自选角度,自拟标题;②文体不限(诗歌除外),文体特征鲜明;③不少于800字;④不得抄袭,不得套作。‎ 思路点拨 与2015年高考作文题“范儿”一样,2016年天津卷的作文试题延续了“生活化”“通俗化”的风格,不在审题上为难学生,让学生有话可说。题干中给出了写作话题“我的青春阅读”,要求考生结合个人的体验和思考来谈。“我的青春阅读”是一个偏正结构的名词短语,其中心词是“阅读”,其限定语有二:“我的”“青春”。前者要求考生从自己的切身体验与思考出发;后者则从时间与性质上做了限定,即所谈的是当代青年的阅读,具有“眼界开阔、个性鲜明”等特点,而且“青春”这个词还隐含有象征意义,即充满生机的、朝气蓬勃的。根据材料引导语中“多元化的阅读方式”这一提示,“阅读方式”‎ 是立意与选材的聚焦点。考生可以选择自己最感兴趣的体验与感受最深的、最有话可说的某种阅读方式来写;也可以针对各种阅读方式的优劣,展开思辨性的议论,发表自己的看法。下列立意可供参考:(1)与时俱进,学会科学的多媒体阅读(网络阅读),开拓视野,丰富生活;(2)青春阅读我做主,年轻人应该学会运用多种阅读方式,兼收并蓄,提升阅读素养;(3)亲近纸质阅读,悦纳经典名著,让青春更绚丽;(4)警惕碎片化的数字阅读,亲近高雅的纸质阅读;等等。‎ ‎7.从问题处入手,给出解答 有的作文材料中呈现的是现实世界的具体问题,那么考生应当针对此问题,经由自己的深入思考、权衡、判断,给出针对性的解答。‎ ‎[2016·浙江高考]阅读下面文字,根据要求作文。‎ 网上购物,视频聊天,线上娱乐,已成为当下很多人生活中不可或缺的一部分。‎ 业内人士指出,不远的将来,我们只需在家里安装VR(虚拟现实)设备,便可以足不出户地穿梭于各个虚拟场景:时而在商店的衣帽间里试穿新衣,时而在诊室里与医生面对面交流,时而在足球场上观看比赛,时而化身为新闻事件的“现场目击者”……‎ 当虚拟世界中的“虚拟”越来越成为现实世界中的“现实”时,是选择拥抱这个新世界,还是刻意远离,或者与它保持适当距离?‎ 对材料提出的问题,你有怎样的思考?写一篇论述类文章。‎ 注意:①角度自选;立意自定。②标题自拟。③不少于800字。④不得抄袭套作。‎ 思路点拨 该作文试题具有鲜明的现代感,非常时尚。虚拟世界里的“虚拟”与现实世界里的“现实”,存在着既对立又统一的关系。对材料提出的问题,考生应深入思考、辩证分析。写作切入角度至少有以下几个方面:(1)“虚拟”世界利弊共存,对于“虚拟”世界潜在的“疏离现实世界”的弊端,应当保持警惕;(2)“虚拟”世界为当代人的生活提供了极大的便利,应当善用“虚拟”技术,改善人类生活;(3)“虚拟世界”并非万应灵丹,与它保持适度距离,方能更好地享受“现实世界”的美好;(4)“虚拟世界”是当代科技发展的趋势,远离并非良策,应当以趋利避害的心态迎应。‎ 题干要求写成“论述类文章”,考生可以采用立论、驳论或破立结合的方式来写,针对材料中的问题,予以明确具体的回答。‎ ‎8.从现象探本质,深入思考 有的作文材料中呈现的是社会生活现象,那么考生应当从现象入手去探寻本质。可以通过针对现象的多角度发问(诸如“是什么”“为什么”“怎么样”“结果如何”等)展开发散思维,从而选择合宜的写作切入点,表达自己的观点。‎ ‎[2016·上海高考]随着现代社会的发展,人们的生活更容易进入大众视野,评价他人生活变得越来越常见,这些评价对人与社会的影响也越来越大。人们对“评价他人的生活”这种现象的看法不尽相同,请写一篇文章,谈谈你对这种现象的思考。‎ 要求:(1)自拟题目;(2)不少于800字。‎ 思路点拨 与2015年二元对立的关系型命题形式不同,2016年上海卷作文试题给出的是社会现象,要求考生进行思考、评析。随着现代社会互联网络、智能手机、微博、微信等的普及,人们对彼此生活的介入越来越快捷与频繁,评价他人生活也日趋活跃与多元。这些评价对个人与社会会产生重要影响。人们对“评价他人的生活”这种现象的看法是见仁见智,各有不同。写作者要写出自己对这种现象的思考。这就要求考生运用透过现象看本质的哲学原理来立意构思。如何快速立意?可以围绕“评价他人的生活”这一现象展开追问:(1)该怎样评价他人的生活?应基于怎样的立场与标准?(2)评价他人的生活,可能会产生什么样的结果或影响?(3)为什么要评价他人的生活?基于什么样的动机?(4)对他人生活的不一样评价,反映了怎样的社会问题?(5)从“评价他人的生活”中,评价者自身可以获得什么?……运用基于现象的不同角度的发散思维,拓宽写作切入点的选择空间。‎ ‎(二)命题作文的审题 相较于近年来一统天下的材料作文题型而言,北京卷的命题作文给人以一枝独秀的感觉。命题作文题型的存续,也值得大家关注。命题作文通常有完全命题型和半命题型两类,命题作文题型相比材料作文而言,限制性增加了,开放性减少了。但北京卷2016年的两道命题作文,却给人以浓厚的书卷味与语文味。作文命题有严格的限制,但又给考生深广的自由发挥空间。‎ ‎[2016·北京高考]从下面两个题目中任选一题,按要求作答。不少于700字。‎ ‎①《白鹿原上奏响一支老腔》记述老腔的演出每每“撼人胸腑”,令人有一种“酣畅淋漓”的感觉。某种意义上,可以说“老腔”已超越其艺术形式本身,成为了一种象征。‎ 请以“‘老腔’何以令人震撼”为题,写一篇议论文。‎ 要求:从老腔的魅力说开去,不要局限于陈忠实散文的内容,观点明确,论据充分,论证合理。‎ 思路点拨 这道作文试题延续了2015年的作文试题(“深入灵魂的热爱”),直接关联试卷内现代文文学类文本阅读而延伸命制作文试题的思路,体现了“读写迁移、共生写作”的理念。试题由一段引导语和一个给定的标题构成。引导语先对试卷内文学类阅读文本《白鹿原上奏响一支老腔》的内容作了简要概述,突出演出的“撼人胸腑”与“酣畅淋漓”的艺术效果,然后再泛化指出,“‘老腔’已超越其艺术形式本身,成为了一种象征”,这就提醒考生要展开相关联想,挖掘“老腔”背后的象征性蕴含。审题立意时,应结合引导语的提示,锁定“‘老腔’何以令人震撼”这一标题,着眼于“何以令人震撼”这一关键,展开说理。题干“要求”中指明“从老腔的魅力说开去,不要局限于陈忠实散文的内容”,这为考生提供了开阔的论述空间。可以由表及里进行深入探究,“老腔”之所以令人震撼,盖因其艺术具有浓烈的乡土气息,或因其具有深厚的文化内涵,或因其体现了当地民族的价值认同。甚至,可以用泛化思维,由点及面,由个别到一般,由“老腔”这一乡土艺术印象谈到其他类似的、传统的、经典的、民族文化的传承与保护的问题。‎ ‎②书签,与书相伴,形式多样。设想你有这样一枚神奇的书签:它能与你交流,还能助你实现读书的愿望……你与它之间会发生什么故事呢?‎ 请展开想象,以“神奇的书签”为题,写一篇记叙文。‎ 要求:表现爱读书、读好书的主题;有细节,有描写。‎ 思路点拨 这道作文试题也是由一段引导语与一个作文标题构成。引导语对关键词“书签”的作用、形式,进而设想“你有这样一枚神奇的书签”——能与人交流,帮助人实现读书的愿望,然后提出设问:你与它之间会发生什么故事?这道作文试题将书签拟人化,要求考生“展开想象”,以“神奇的书签”为题写一篇记叙文,表达爱读书、读好书的主题。写作时,应注意关键词之前的修饰语——“神奇”,写作记叙文,应以具体情节与细节来表现“书签”的神奇之处,而这就需要考生激活自己的想象力,写出能体现“我与书签”关系的具体、形象的场景与生动、传神的细节,从而表现爱读书、读好书的这一主旨。‎ 总之,备考2018年高考作文,考生应从素养、思维、技能三个层面做好充分准备,把握高考作文命题的大趋势,领会考试大纲的要求,扎实掌握教材知识,努力夯实阅读积淀,不断磨炼思维,熟练驾驭审题、立意、谋篇、构段、语言修辞、润色修改等各项常规技能,为取得优秀的作文成绩奠定坚实的基础。‎ 第七部分 冷考点预防考查训练 考点二十九 字音 考点名片 考点内容 主要考查常见多音字、形声字、形近字、同音字字音的识记和辨析。‎ 考查形式 ‎①注音辨正误(给出加点字的读音,辨析所给读音的正误);②分组辨异同(未注音,辨析加点字读音的异同)。‎ 趋势分析 ‎①重点考查多音字、同音字、形声字的读音。②一般以注音辨正误或分组辨异同的形式单纯考查字音。③北京卷结合现代文阅读综合考查字音、字形。‎ ‎1.下列词语中加点字读音完全相同的一组是(  )‎ A.绯闻 斐然 缠绵悱恻 蜚短流长 B.应允 楹联 义愤填膺 脱颖而出 C.馈赠 磨蹭 减员增效 爱憎分明 D.寂寞 蓦然 厉兵秣马 相濡以沫 答案 D 解析 A项,“蜚”读fēi,其他三个读fěi。B项,“应”和“膺”读yīng,“楹”读yíng,“颖”读yǐng。C项,“赠”读zèng,“蹭”读cèng,“增”和“憎”读zēng。D项,都读mò。‎ ‎2.下列词语中加点字读音完全相同的一组是(  )‎ A.鸟喙 贿赂 诲人不倦 风雨如晦 B.估量 辜负 蛊惑人心 沽名钓誉 C.挣脱 狰狞 铮铮铁骨 云蒸霞蔚 D.附和 凫水 覆水难收 循环往复 答案 A 解析 A项,均读huì。B项“蛊”读gǔ,其余的读ɡū;C项“挣”读zhèng,其余的读zhēnɡ;D项“凫”读fú,其余的读fù。‎ ‎3.下列词语中加点字读音都不相同的一组是(  )‎ A.汲取 积极 负笈从师 岌岌可危 B.自刭 颈椎 泾渭分明 毛孔痉挛 C.觊觎 觇视 面面相觑 循规蹈矩 D.荟萃 杂烩 脍炙人口 市侩嘴脸 答案 C 解析 C项,依次读为jì,chān,qù,guī。‎ ‎4.下列词语中加点字,读音全都不相同的一组是(  )‎ A.跻身 侪辈 剂量 济济一堂 光风霁月 B.入殓 眼睑 验证 横征暴敛 勤俭节约 C.禅让 阐释 忌惮 箪食壶浆 殚精竭虑 D.悲怆 寒伧 创伤 沧海桑田 踉踉跄跄 答案 D 解析 D项,依次读为chuànɡ、chen、chuānɡ、cānɡ、qiànɡ。A项,“剂”与“霁”音同,都读jì;B项,“睑”与“俭”音同,都读jiǎn;C项,“箪”与“殚”音同,都读dān。‎ ‎5.下列加点字的注音,全都相同的一组是(  )‎ A.煎熬 翩跹 信笺 歼击机 草菅人命 B.渎职 赎罪 案牍 水牛犊 穷兵黩武 C.对峙 窒息 挚友 滞纳金 栉风沐雨 D.偏僻 裨益 辅弼 庇护权 刚愎自用 答案 C 解析 A项,“跹”读xiān,其余读jiān;B项,“赎”读shú,其余读dú;C项,都读zhì;D项,“僻”读pì,其余读bì。‎ ‎6.下列词语中加点字读音完全相同的一组是(  )‎ A.胜迹 圣经 剩余 盛情难却 千乘之国 B.横事 衡量 道行 横行霸道 恒河沙数 C.识趣 实录 拾级 食古不化 拾人牙慧 D.闻达 紊乱 稳练 刎颈之交 纹丝不动 答案 A 解析 A项,均读shèng;B项,依次读hèng,héng,heng,héng,héng;C项,依次读shí,shí,shè,shí,shí;D项,依次读wén,wěn,wěn,wěn,wén。‎ ‎7.下列词语中加点字的读音,全都不相同的一组是(  )‎ A.阻击 沮丧 阻止 含英咀华 越俎代庖 B.端详 揣测 湍急 苟延残喘 惴惴不安 C.估价 枯槁 训诂 怙恶不悛 沽名钓誉 D.嗔怪 谨慎 瞋视 文思缜密 镇定自若 答案 B 解析 A项,“沮”读jǔ,“咀”读jǔ;C项,“估”读gū,“沽”读gū;D项,“嗔”读chēn,“瞋”读chēn。‎ ‎8.下列词语中加点字的读音,完全相同的一组是(  )‎ A.诡谲 倔强 崛起 一蹶不振 攫为己有 B.贿赂 绿林 露面 戮力同心 烧杀掳掠 C.妊娠 绅士 辰时 扪参历井 莘莘学子 D.箴言 狰狞 甄别 文思缜密 百福骈臻 答案 A 解析 A项,全为jué;B项,“赂、绿、戮”读lù,“露”读lòu,“掳”读lǔ;C项,“辰”读chén,其余均为shēn;D项,“箴、甄、臻”读zhēn,“狰”读zhēng,“缜”读zhěn。‎ ‎9.下列词语中加点的字,每对读音都不相同的一项是(  )‎ A.噩耗/好学 曝晒/曝光 落后/落枕 B.轨迹/滚雷 洋溢/谥号 烙印/炮烙 C.淀粉/惦记 悲怆/沧海 瓜蔓/蔓延 D.附和/荷载 凯旋/觊觎 恐吓/吓人 答案 B 解析 B项,guǐ/gǔn,yì/shì,lào/luò。A项,hào,pù/bào,luò/lào;C项,diàn,chuàng/cāng,wàn/màn;D项,hè,kǎi/jì,hè/xià。‎ ‎10.下列词语中加点的字,每对读音都不相同的一组是(  )‎ A.泥沙/泥古不化 拾级/拾金不昧 载誉/载歌载舞 B.涤纶/羽扇纶巾 弱冠/冠冕堂皇 臭气/乳臭未干 C.纤维/纤尘不染 薄饼/妄自菲薄 横祸/横生枝节 D.屏障/屏气凝神 臧否/否极泰来 艾草/自怨自艾 答案 B 解析 A项,ní/nì,shè/shí,zài;B项,lún/guān,guàn/guān,chòu/xiù;C项,xiān,báo/bó,hèng/héng;D项,píng/bǐng,pǐ,ài/yì。‎ ‎11.下列词语中加点的字,每对读音都不相同的一组是(  )‎ A.吭声/引吭高歌 埋怨/埋头苦干 炮烙/如法炮制 B.折本/不折不扣 蛮横/飞来横祸 和面/和衷共济 C.落枕/丢三落四 复辟/鞭辟入里 咀嚼/咬文嚼字 D.乘船/千乘之国 阿谀/刚正不阿 积累/果实累累 答案 C 解析 A项,kēng/háng,mán/mái,páo;B项,shé/zhé,hèng,huó/hé;C项,lào/là,bì/pì,jué/jiáo;D项,chéng/shèng,ē,lěi/léi。‎ ‎12.下列词语中加点的字,每对读音都不相同的一组是(  )‎ A.蹊跷/蹊径 暴戾/风声鹤唳 泥淖/风姿绰约 B.果脯/胸脯 弓弩/驽马十驾 隽秀/隽语箴言 C.宫阙/商榷 弱冠/沐猴而冠 怅惘/为虎作伥 D.伫立/贮存 聆听/高屋建瓴 辟邪/鞭辟入里 答案 B 解析 A项,“暴戾/风声鹤唳”二者读音相同,读lì。故可排除A项。C项,“宫阙/商榷”二者读音相同,读què;“弱冠/沐猴而冠”二者读音相同,读guàn。故排除C项。D项,“伫立/贮存”二者读音相同,应读zhù;“聆听/高屋建瓴”二者均读líng。故排除D项。选B。‎ ‎13.下列词语中加点的字,每对读音都不相同的一项是(  )‎ A.媲美/譬如 犄角/脊梁骨 砥砺/舐犊情深 B.竣工/皴裂 悼词/倒春寒 眼睑/秋敛冬藏 C.编纂/篡权 上颌/余额宝 孪生/鸾凤和鸣 D.悖理/荸荠 圈养/入场券 硝酸/削足适履 答案 D 解析 A项,读音依次为:pì,jī/jǐ,dǐ/shì;B项,读音依次是:jùn/cūn,dào,jiǎn/liǎn;C项,读音依次是:zuǎn/cuàn,hé/é,luán;D项,读音依次是:bèi/bí,juàn/quàn,xiāo/xuē。‎ ‎14.下列加点字的读音与所给读音完全相同的一组是(  )‎ A.劲jìng 劲敌 遒劲 疾风劲草 刚劲挺拔 B.泊bó 停泊 漂泊 淡泊名利 水泊梁山 C.应yīng 应届 应允 应声而落 应有尽有 D.鲜xiǎn 鲜见 鲜有 寡廉鲜耻 屡见不鲜 答案 A 解析 B项,“水泊梁山”中的“泊”应读pō。C项,“应声而落”中的“应”应读yìng。D项,“屡见不鲜”中的“鲜”应读xiān。‎ ‎15.下列词语中加点字的读音全部正确的一组是(  )‎ A.粗糙(cāo) 卓越(zhuó) 肖像(xiào) 侮辱(wǔ)‎ B.不妨(fāng) 街坊(fāng) 脂肪(fáng) 妨碍(fǎng)‎ C.载体(zǎi) 载重(zài) 角逐(jué) 角力(jiǎo)‎ D.哈达(hǎ) 哈哈镜(hā) 混乱(hùn) 混水摸鱼(hùn)‎ 答案 A 解析 B项,“不妨”的“妨”应读“fáng”,“坊”应读“fang”,“妨碍”的“妨”应读“fáng”;C项,“载体”的“载”应读“zài”,“角力”的“角”应读“jué”;D项,“混水摸鱼”的“混”同“浑”读“hún”。‎ ‎16.下列词语中加点字的读音有错的一组是(  )‎ A.漱口(shù)       色厉内荏(rěn)‎ 浣纱(huàn) 否极泰来(pǐ)‎ B.鹰隼(sǔn) 瓦刀(wà)‎ 铁锨(xiān) 胼手胝足(dǐ)‎ C.褫夺(chǐ) 踟蹰(chú)‎ 彳亍(chù) 踯躅(zhí)‎ D.压轴戏(zhòu) 囹圄(yǔ)‎ 垄沟(lǒng) 涎皮赖脸(xián)‎ 答案 B 解析 B项,“胝”应读“zhī”。‎ ‎17.下列词语中加点字的读音有错误的一组是(  )‎ A.拮据(jié) 绥靖(suí)‎ 饥荒(huang) 髋骨(kuān)‎ B.碉堡(bǎo) 堡子(bǔ)‎ 十里堡(bǎo) 烘焙(bèi)‎ C.俾众周知(bǐ) 裨将(pí)‎ 裨益(bì) 髀肉复生(bì)‎ D.框框(kuàng) 粗犷(guǎng)‎ 眼眶(kuàng) 沉疴(kē)‎ 答案 B 解析 B项,“十里堡”的“堡”应读“pù”。‎ ‎18.下列词语中加点字的读音全都正确的一组是(  )‎ A.坍(tān)塌 牵强(qiǎng) 锐不可当(dāng)‎ B.岿(kuī)然 缫(cháo)丝 为虎作伥(chāng)‎ C.长吁(xū)短叹 浸渍(zé) 瞠(chēng)目结舌 D.聒(guō)噪 酗(xiōng)酒 户枢不蠹(dù)‎ 答案 A 解析 B项,“缫”读sāo;C项,“渍”读zì;D项,“酗”读xù。‎ ‎19.下列词语中加点字的读音全都正确的一组是(  )‎ A.装帧(zhēn) 拙劣(zhuō)  风靡一时(mí)‎ B.甲胄(zhòu) 拯救(zhénɡ) 封妻荫子(yìn)‎ C.靡费(mí)  饮马(yǐn)  怪石嶙峋(xún)‎ D.脖颈(ɡěnɡ) 拎起(līn)  顺蔓摸瓜(wàn)‎ 答案 D 解析 A项,靡mǐ,B项,拯zhěng,C项,饮yìn。‎ ‎20.下列词语中加点字的读音全都正确的一组是(  )‎ A.纾难(shū) 炮制(pào) 大腹便便(pián)‎ B.屏息(píng) 遒劲(qiú) 未雨绸缪(móu)‎ C.愆伏(qiān) 沙碛(qì) 强词夺理(qiǎng)‎ D.讥诮(qiào) 嵌入(qiàn) 方枘圆凿(nà)‎ 答案 C 解析 A项,“炮”读“páo”;B项,“屏”读“bǐng”;D项,“枘”读“ruì”。‎ ‎[3年高考真题集训]‎ ‎1.[2016·天津高考]下列词语中,加点字的字音和字形全都正确的一组是 A.惬(qiè)意 撕(sī)杀 狩(shòu)猎 金榜题(tí)名 B.折(zhé)本 角(jué)逐 按(ān)装 舐(shì)犊情深 C.筵(yán)席 偌(ruò)大 着(zháo)陆 前倨(jù)后恭 D.岿(kuī)然 毗(pí)邻 装帧(zhēn) 噤(jìn)若寒蝉 答案 D 解析 本题考查字音和字形。A项,撕—厮。B项,折本(shé),按—安。C项,着陆(zhuó)。‎ ‎2.[2016·浙江高考]下列词语中,加点字的注音全都正确的一项是(  )‎ A.煲汤(bāo) 恫吓(dòng)‎ 脐带血(jì) 整齐划一(huà)‎ B.古刹(chà) 衣钵(bō)‎ 挑大梁(tiǎo) 言为心声(wèi)‎ C.掣肘(chè) 卤味(lǔ)‎ 处女座(chǔ) 寅吃卯粮(yín)‎ D.笃定(dǔ) 痤疮(cuó)‎ 病恹恹(yāng) 血气方刚(xuè)‎ 答案 C 解析 A项,脐带血(qí);B项,言为心声(wéi);D项,病恹恹(yān)。‎ ‎3.[2015·四川高考]下列词语中加点字的读音全部正确的一项是(  )‎ A.暂时(zàn) 埋怨(mái)‎ 谆谆告诫(zhūn) 引吭高歌(hánɡ)‎ B.豆豉(chǐ) 踝骨(huái)‎ 踉踉跄跄(cānɡ) 按图索骥(jì)‎ C.梗概(ɡěn) 删改(shān)‎ 炊烟袅袅(niǎo) 明眸皓齿(móu)‎ D.搁浅(ɡē) 解剖(pōu)‎ 鬼鬼祟祟(suì) 不屑一顾(xiè)‎ 答案 D 解析 A项,埋怨(mán);B项,踉踉跄跄(qiànɡ);C项,梗概(ɡěnɡ)。‎ ‎4.[2015·浙江高考]下列词语中加点字注音全部正确的一项是(  )‎ A.纠葛(gé) 瓜蔓(màn)‎ 牛皮癣(xuǎn) 为(wèi)虎作伥 B.惬(qiè)意 觊(jì)觎 蒙(mēng)蒙亮 扺(zhǐ)掌而谈 C.谄(chǎn)媚 压轴(zhóu)‎ 一溜(liù)烟 间不容发(fà)‎ D.豆豉(chǐ) 箴(zhēn)言 轧(zhá)马路 开门揖(yī)盗 答案 B 解析 A项,“蔓”应读“wàn”;C项“轴”应读“zhòu”;D项,“轧”应读“yà”。‎ ‎5.[2015·天津高考]下列词语中加点字的读音全都正确的一组是(  )‎ A.寂寥(liáo) 雾霾(mái)‎ 瞋(chēng)目 潜(qián)移默化 B.氛(fēn)围 吝啬(sè)‎ 熹(xī)微 束(shù)之高阁 C.发酵(jiào) 徘徊(huái)‎ 滂(pāng)沱 叱咤(chà)风云 D.模(mó)板 怯(qiè)懦 签(qiān)署 断壁颓垣(yuán)‎ 答案 B 解析 A项,瞋(chēn)目;C项,叱咤(zhà)风云;D项,模(mú)板。‎ ‎6.[2015·广东高考]下列词语中加点的字,每对读音都不相同的一组是(  )‎ A.棱角/菱形 窒息/对峙 稽首/稽查 B.侥幸/阻挠 绚烂/驯服 称职/职称 C.塑料/朔风 叫嚣/发酵 本末倒置/倒行逆施 D.延伸/筵席 瓦砾/罹难 挑三拣四/挑拨离间 答案 B 解析 B项,jiǎo/náo,xuàn/xùn,chèn/chēnɡ;A项,lénɡ/línɡ,zhì,qǐ/jī;C项,sù/shuò,xiāo/jiào,dào;D项,yán,lì/lí,tiāo/tiǎo。‎ ‎[2年全国模拟重组]‎ ‎1.[2017·山东师大附中模拟]下列加点词语的读音全都正确的一项是(  )‎ A.打烊(yàng) 给予(jǐ)‎ 芝麻糊(hù) 混水摸鱼(hún)‎ B.卡壳(qiǎ) 处暑(chù)‎ 入场券(quàn) 虚与委蛇(wēi)‎ C.坊间(fāng) 眩晕(yùn)‎ 扎小辫(zā) 萎靡不振(mí)‎ D.棕榈(lǘ) 名媛(yuàn)‎ 一场雨(chǎng) 一哄而散(hòng)‎ 答案 A 解析 B项,“处”读chǔ;C项,“靡”读mǐ;D项,“场”读cháng。‎ ‎2.[2017·浙江名校联考]下列词语中,加点字的读音全都正确的一项是(  )‎ A.瘙痒(sào) 豇豆(ɡānɡ)‎ 捋虎须(luō) 出头露面(lòu)‎ B.怪癖(pǐ) 说服(shuō)‎ 好莱坞(wù) 戏谑之言(xuè)‎ C.札记(zhá) 蹩脚(bié)‎ 潜意识(qiǎn) 剑拔弩张(nǔ)‎ D.皴裂(cūn) 槟榔(bīn)‎ 喝倒彩(hè) 里应外合(yìnɡ)‎ 答案 B 解析 A项,“豇”读jiāng;C项,“潜”读qián;D项,“槟”读bīng。‎ ‎3.[2016·日照二模]下列词语中,加点字的注音全都正确的一项是(  )‎ A.木讷(nè) 熟稔(rěn)‎ 角(jiǎo)斗士 纵横捭(bǎi)阖 B.感喟(kuì) 屏(bǐng)息 紧箍(gū)咒 拈(niān)轻怕重 C.挟(xiá)持 聒(guō)噪 白炽(chì)灯 绿(lù)林好汉 D.书箧(qiè) 顺遂(suì)‎ 胳(gā)肢窝 扣人心弦(xuán)‎ 答案 B 解析 A项,角斗士(jué)。C项,挟持(xié)。D项,扣人心弦(xián)。‎ ‎4.[2017·天津联考]下列词语中加点字的读音完全相同的一组是(  )‎ A.嗔怒 谨慎 缜密思考 瞋目而视 B.疏浚 唆使 怙恶不悛 逡巡不前 C.哽咽 梗概 骨鲠在喉 绠短汲深 D.谐音 揩油 偕同前往 万世楷模 答案 C 解析 A项,“嗔、瞋”读chēn,“慎”读shèn,“缜”读zhěn;B项,“浚”读jùn,“唆”读suō,“悛”读quān,“逡”读qūn;D项,“谐、偕”读xié,“揩”读kāi,“楷”读kǎi。‎ ‎5.[2017·绍兴一中模拟]下列词语中加点的字,每对读音都不相同的一项是(  )‎ A.处方/处理 锁钥/钥匙 矩形/规矩 轧钢/轧账 B.瓜蔓/蔓延 躯壳/贝壳 翘首/翘楚 拓片/开拓 C.坊间/磨坊 连累/累及 稍息/稍许 应届/应允 D.挨边/挨整 泡桐/泡影 解元/解数 跂望/跂行 答案 D 解析 A项,chǔ,yuè/yào,jǔ/ju,zhá/gá;B项,wàn/màn,qiào/ké,qiáo,tà/tuò;C项,fāng/fáng,lei/lěi,shào/shāo,yīng;D项,āi/ái,pāo/pào,jiè/xiè,qǐ/qí。‎ ‎6.[2016·丽水模拟]下列每对加点字的读音都不相同的一组是(  )‎ A.模样/模范 打哈欠/点头哈腰 牵强/强词夺理 B.应允/应变 独角戏/钩心斗角 奢靡/所向披靡 C.殷红/殷实 翘尾巴/翘首以待 碑帖/俯首帖耳 D.累及/累次 梁山泊/淡泊明志 屏障/敛声屏气 答案 C 解析 C项,yān/yīn,qiào/qiáo,tiè/tiē;A项,mú/mó,hā,qiǎng;B项,yīng/yìng,jiǎo,mí/mǐ;D项,lěi,pō/bó,píng/bǐng。‎ ‎7.[2017·天津质检]下列词语中加点的字,读音全部正确的一项是(  )‎ A.睥睨(pì) 敕造(chì)‎ 不吱声(zī) 绿林好汉(lù)‎ B.果脯(fǔ) 句读(dòu)‎ 节骨眼(jiē) 掎角之势(yǐ)‎ C.腈纶(jīng) 胴体(tóng)‎ 脖颈儿(jǐng) 浩浩汤汤(shāng)‎ D.眼睑(jiǎn) 薅草(hāo)‎ 氽丸子(cuān) 心广体胖(pàng)‎ 答案 A 解析 B项,掎角之势(jǐ);C项,胴体(dòng),脖颈儿(gěng);D项,心广体胖(pán)。‎ ‎8.[2017·青岛模拟]下列词语中加点的字,读音全都正确的一组是(  )‎ A.弄堂(nòng) 悄然(qiǎo)‎ 星宿(xiù) 牝鸡司晨(pìn)‎ B.侪辈(chái) 歼灭(jiān)‎ 冯河(píng) 饮鸩止渴(zhèn)‎ C.倥偬(zǒng) 舌苔(tái)‎ 巨擘(bò) 强人所难(qiǎng)‎ D.觊觎(jì) 靓妆(liàng)‎ 皴裂(cūn) 不容置喙(huì)‎ 答案 B 解析 A项,弄堂(lòng);C项,舌苔(tāi);D项,靓妆(jìng)。‎ ‎9.[2017·浙江模拟]下列词语中,加点字的注音全部正确的一项是(  )‎ A.鱼鳔(biào) 荨(qián)麻 脖颈(gěng) 哈(hā)巴狗 B.窨(yìng)井 弓弩(nǔ)‎ 桔(jié)梗 油坊(fáng)‎ C.唠(láo)叨 扼腕(wàn)‎ 款识(zhì) 悄(qiǎo)寂 D.结束(sù) 神祇(qí)‎ 啰(luō)唆 刽(guì)子手 答案 C 解析 A项,哈(hǎ)巴狗;B项,窨(yìn)井;D项,结束(shù)。‎ ‎10.[2017·烟台质检]下列加点字的读音完全正确的一项是(  )‎ A.和(huò)稀泥 骨(gú)气 土坯(pī) 踮脚(diǎn)‎ B.船坞(wū) 间(jiàn)隔 祓(fú)除 搀扶(chān)‎ C.呶呶(náo)不休 骨髓(suǐ)‎ 阿(ē)胶 酽(yàn)茶 D.扒(pá)窗而入 堰塞(sè)湖 矗(chù)立 豆豉(chǐ)‎ 答案 C 解析 A项,“骨”读gǔ;B项,“坞”读wù;D项,“扒”读bā。‎ ‎11.[2017·温州测试]下列词语中,加点字的注音全都正确的一项是(  )‎ A.瞭望(liǎo) 挫折(cuò)‎ 抹布(mā) 谬误(miù)‎ B.涮洗(shuā) 佳境(jiā)‎ 纷呈(chénɡ) 敞篷(chǎnɡ)‎ C.陆军(lù) 鞠躬(jū)‎ 称心(chēnɡ) 致残(zhì)‎ D.航程(hánɡ) 瞪眼(dènɡ)‎ 讳称(huì) 协调(xié)‎ 答案 D 解析 A项,瞭望(liào);B项,涮洗(shuàn);C项,称心(chèn)。‎ ‎12.[2017·山东检测]下列词语中,加点字的注音全都正确的一项是(  )‎ A.结束(shù) 恐吓(hè)‎ 青蒿素(hāo) 流水淙淙(cóng)‎ B.扒窃(pá) 怫然(fú)‎ 炮羊肉(bāo) 叱咤风云(chà)‎ C.轻佻(tiāo) 赧然(nǎn)‎ 闷葫芦(mēn) 模棱两可(léng)‎ D.纰漏(pī) 摈弃(bìng)‎ 文件夹(jiā) 喁喁私语(yú)‎ 答案 A 解析 B项,“咤”应读zhà;C项,“闷”应读mèn;D项,“摈”应读bìn。‎ ‎13.[2016·杭州模拟]下列各组词语中加点的字,读音都不相同的一组是(  )‎ A.创伤/重创敌军 投奔/奔走相告 蒙骗/蒙混过关 B.别扭/别开生面 旋风/旋转乾坤 专横/横行霸道 C.谙熟/黯然失色 缜密/半嗔半笑 炮烙/刨根问底 D.渲染/煊赫一时 罢黜/相形见绌 辑录/缉拿归案 答案 B 解析 B项,biè/bié,xuàn/xuán,hèng/héng;A项,chuāng,bèn/bēn,mēng/méng;C项,ān/àn,zhěn/chēn,páo;D项,xuàn/xuān,chù,jí/jī。‎ ‎14.[2016·天津质检]下列词语中加点字的读音,有错误的一组是(  )‎ A.牛虻(méng) 氓隶(méng)‎ 魍魉(wǎng) 欺罔(wǎng)‎ B.草菅人命(jiān) 绾结(wǎn)‎ 猪倌(guān) 管理(guǎn)‎ C.枢纽(shū) 抠门(kōu)‎ 沤肥(òu) 伛偻(gōu)‎ D.慑服(shè) 蹑手蹑脚(niè)‎ 镊子(niè) 嗫嚅(niè)‎ 答案 C 解析 C项,“伛”应读“yǔ”。‎ ‎15.[2017·济南一中检测]下列词语中,加点字的注音全都正确的一项是(  )‎ A.盛(chéng)饭 噬啮(niè)‎ 典型(xín) 沐猴而冠(guàn)‎ B.歼灭(qiān) 水泵(bèng)‎ 血淋淋(xiě) 晕头转向(zhuàn)‎ C.偌大(nuò) 铩羽(shā)‎ 压轴戏(zhòu) 模棱两可(léng)‎ D.肇事(zhào) 棕榈(lǘ)‎ 露马脚(lòu) 锱铢必较(zī)‎ 答案 D 解析 A项,“型”读xínɡ;B项,“歼”读jiān;C项,“偌”读ruò。‎ 考点三十 字形 考点名片 考点内容 主要考查在理解词义、句意的基础上辨析字形的能力。以辨析形近字、同音字的字形为主,只考别字,不考错字。‎ 考查形式 ‎①词中辨正误;②句中辨正误;③综合考查字音、字形。选择题,3分。‎ 趋势分析 ‎①重点考查辨析形近字、同(近)音字的字形。②多以词中辨正误的形式单纯考查字形。③浙江卷以句中辨正误的形式单纯考查字形。④北京卷结合现代文阅读综合考查字音、字形。‎ ‎1.下列各组词语中,有错别字的一组是(  )‎ A.变换/变幻 处世/处事 渊源/源远流长 B.爆发/暴发 法制/法治 融汇/融汇贯通 C.装潢/装帧 琐碎/琐细 谋取/牟取暴利 D.拖拉/拖沓 挑拨/挑唆 激流/急流勇退 答案 B 解析 “融汇贯通”应为“融会贯通”。‎ ‎2.下列各组词语中没有错别字的一项是(  )‎ A.立竿见影 无尚光荣 余勇可贾 怨天尤人 B.振聋发聩 弥天大谎 委曲求全 省吃俭用 C.沧海一粟 坐收鱼利 屡见不鲜 金榜题名 D.与时俱进 自负盈亏 耳熟能详 欢渡良宵 答案 B 解析 A项,尚—上;C项,鱼—渔;D项,渡—度。‎ ‎3.下列词语中没有错别字的一组是(  )‎ A.斑斓 贪脏枉法 好高骛远 孽根祸胎 B.包涵 默然消魂 萍水相逢 棉里藏针 C.遐想 披星戴月 拭目以待 恍然大悟 D.呕歌 义愤填膺 貌和神离 湮没无闻 答案 C 解析 A项,脏—赃;B项,消—销,棉—绵;D项,呕—讴,和—合。‎ ‎4.下列词语中,有两个错别字的一组是(  )‎ A.靓丽 哈密瓜 反躬自问 过目成颂 B.卸妆 明信片 开源截流 玲珑剔透 C.账目 至高点 优柔寡断 过尤不及 D.涵养 亲合力 人才汇萃 嘻笑怒骂 答案 C 解析 A项,颂—诵;B项,截—节;C项,至—制,尤—犹;D项,合—和,汇—荟,嘻—嬉。‎ ‎5.下列词语中有两个错别字的一组是(  )‎ A.宣泄 明信片 重峦迭嶂 再接再厉 B.文身 挖墙角 悬梁刺股 计日成功 C.凑和 水蒸汽 犄角之势 平心而论 D.简练 水龙头 一如继往 三缄其口 答案 B 解析 A项,“重峦迭嶂”应为“重峦叠嶂”;B项,“挖墙角”应为“挖墙脚”,“计日成功”应为“计日程功”;C项,“凑和”应为“凑合”,“水蒸汽”应为“水蒸气”,“犄角之势”应为“掎角之势”;D项,“一如继往”应为“一如既往”。‎ ‎6.下列词语中没有错别字的一组是(  )‎ A.既使 连锁店 察颜观色 言之无文,行而不远 B.坐落 炒鱿鱼 青山绿水 一言既出,驷马难追 C.苍桑 下马威 渡过难关 防人之口,胜于防川 D.影碟 歌午厅 偃旗息鼓 桃李不言,下自成溪 答案 B 解析 A项,既—即;C项,苍—沧,胜—甚;D项,午—舞,溪—蹊。‎ ‎7.下列词语中,没有错别字的一组是(  )‎ A.磋商 举步为艰 仗势欺人 自鸣不凡 B.誊写 却之不恭 断章取意 博闻强识 C.肄业 唾手可得 应接不暇 恣意妄为 D.推诿 亦步亦趋 层峦叠嶂 头晕目炫 答案 C 解析 A项,举步维艰、自命不凡;B项,断章取义;D项,头晕目眩。‎ ‎8.下列词语中没有错别字的一组是(  )‎ A.坐垫 练习簿 蒜苔 蛛丝马迹 B.省份 凑份子 谜团 做贼心虚 C.颂读 螺旋桨 神采 节目精彩 D.暖气 哈密瓜 蛰伏 经济版块 答案 B 解析 A项,苔—薹;C项,颂—诵;D项,版—板。‎ ‎9.下列各组词语中没有错别字的一组是(  )‎ A.铩羽而归 坐阵指挥 画地为牢 鞭辟入里 B.严惩不怠 相辅相成 栉风沐雨 势在必行 C.改弦更张 岿然不动 休戚与共 长年累月 D.精兵简政 出类拔萃 一如既往 披沙捡金 答案 C 解析 A项,坐镇指挥;B项,严惩不贷;D项,披沙拣金。‎ ‎10.下列各组词语中,没有错别字的一项是(  )‎ A.惯例 老皇历 墨守成规 魔高一尺,道高一丈 B.安详 协奏曲 面面具到 翻手为云,复手为雨 C.学籍 洽谈会 风弛电掣 盛名之下,其实难符 D.彩票 唱双簧 针贬时弊 城门失火,殃及池鱼 答案 A 解析 B项,具—俱,复—覆;C项,弛—驰,符—副;D项,贬—砭。‎ ‎11.下列各组词语中,错别字最多的一组是(  )‎ A.脉搏 装钉 各行其事 一张一驰,文武之道 B.烦燥 诡辩 莫名其妙 万事具备,只欠东风 C.提纲 讴歌 彪炳千秋 千里之堤,溃于蚁穴 D.起讫 凌晨 两全齐美 差之毫厘,缪以千里 答案 A 解析 A项,钉—订,事—是,驰—弛;B项,燥—躁,具—俱;C项,全部正确;D项,齐—其,缪—谬。‎ ‎12.下列词语中错别字最多的一组是(  )‎ A.远眺 戴手饰 别出新裁 出奇不意 B.撕杀 挖墙脚 涣然冰释 礼上往来 C.躁热 哈蜜瓜 老生长谈 道貌黯然 D.嘉话 主旋律 食不裹腹 金壁辉煌 答案 C 解析 A项,戴首饰、别出心裁、出其不意;B项,厮杀、礼尚往来;C项,燥热、哈密瓜、老生常谈、道貌岸然;D项,佳话、食不果腹、金碧辉煌。‎ ‎13.下列词语中,没有错别字的一组是(  )‎ A.旋律 天籁 协奏曲 休止符 余音缭绕 不绝如缕 B.文采 脚本 座右铭 俱乐部 鸿篇巨制 异曲同功 C.履职 禀报 蝇投利 传帮带 弊绝风清 惠民工程 D.机率 调剂 孺子牛 气垫船 顾名思义 一如既往 答案 A 解析 B项,功—工。C项,投—头。D项,机—几。‎ ‎14.下列各句中,没有错别字的一项是(  )‎ A.在青山绿水中拥有一套自己的居所,历来是人们的梦想,而西溪自古就是隐逸之地,被文人视为人间静土、世外桃源。‎ B.借助煽情的大结局,暴笑喜剧《爱情公寓第三季》完满落幕,超高人气继续领跑热播榜的同时,也顺势带火《爱情公寓第一季》。‎ C.一年一度的“阳光助学”已近尾声,企业与公众双翼齐飞,“微公益”助学也风生水起,接下来让我们一起进入“公益医保”时间,用一分一厘来聚沙成塔。‎ D.都市快报全新推出“微博爱心拍卖”单元,在这里,无论是慷慨的竞拍出价还是简单的转发,只要动动鼠标,片刻之间你就完成了爱心的传递与会聚。‎ 答案 C 解析 A项,静—净;B项,暴—爆;D项,会—汇。‎ ‎15.下列句子中没有错别字的一项是(  )‎ A.素有“云中牧女”之称的艾菲尔铁塔虽然经历了一百多年的苍桑,但仍以她那拔地耸云的英姿迎送天下客。‎ B.简单并可预料的故事情节、眼花缭乱的视觉效果、很少需要用字幕的单字对白,成为好莱坞影片走向世界的“通行证”。‎ C.在最伟大的人物中间,巴尔扎克是名列前矛者;在最优秀的人物中间,巴尔扎克是佼佼者之一。‎ D.“一片甲骨惊天下”,百余年前因发现甲骨文而振惊世界的殷墟,再次为世界瞩目。申遗成功的消息传来,河南安阳沉浸在喜悦和兴奋之中。‎ 答案 B 解析 A项,应为“沧桑”;C项,应为“名列前茅”;D项,应为“震惊”。‎ ‎16.下列句子中有两个错别字的一项是(  )‎ A.夕阳映照下的江水波光粼粼,宛如一串光彩夺目的项链,挂在大桥颀长的脖子上,让人流恋忘返。‎ B.尽管此文个别词句仍有锤练余地,但瑕不掩瑜,作者在文中表现出的不俗文才仍搏得了人们的青睐。‎ C.管理部门人手匮乏导致执法时捉襟见肘,使不法分子有机可乘,铤而走险,盗猎行为已呈蔓延之势。‎ D.在灿若星辰的作家中,我尤其尊崇鲁迅先生,因为他的作品针贬时弊,振聋发聩,唤醒了民众麻木的灵魂。‎ 答案 B 解析 B项,“练”为“炼”,“搏”为“博”;A项,“恋”为“连”;D项,“贬”为“砭”。‎ ‎17.下列各句中没有错别字的一项是(  )‎ A.他用原形分析的方法,用文本细读的姿态,读出了很多人眼中没有的《红楼梦》,写出的论文高屋建瓴,大气磅礴。‎ B.那一张张发黄的照片,承载着历史遗迹,能给人带来无限的回忆和遐想,让人浮想连翩,追忆逝去的青葱岁月。‎ C.他老道地说:“具有这种锐利眼光,能够看清事实真相的人,可以任意支配整个世界及其知识财富,拥有多彩的人生。”‎ D.孩子大了,要到外面历练,于是,十二岁的他告别了连绵不断的绿色山峦和淙淙流淌的溪涧,来到了喧闹的城市。‎ 答案 D 解析 A项,原形—原型;B项,浮想连翩—浮想联翩;C项,老道—老到。‎ ‎18.下列句子中有两个错别字的一项是(  )‎ A.近年来,有不少海外学子相继学成归来,报效桑梓,他们在我国对外开放和经济建设中已经暂露头角。‎ B.在选购古物时,请不要匆忙洽购,要多看、多揣摩,特别要注意古物的年份,艺术性和器形。‎ C.浓郁的情韵和深刻的哲理在文章中水乳交溶,增强了文章的艺术魅力,强烈地振撼着读者的心灵。‎ D.维护法律的尊严,要从司法部门抓起,个别营私午弊,循情枉法者,要坚决从执法队伍中清除出去,绝不辜息。‎ 答案 C 解析 A项,暂—崭;C项,溶—融,振—震;D项,午—舞,循—徇,辜—姑。‎ ‎19.下列各句中,没有错别字的一项是(  )‎ A.中国传统的耻感文化包括人的个性休养和整个社会的道德评价机制,具体表现为向内和向外两个层面。‎ B.我的故乡没有繁华酥骨的都会,没有静谧侵肌的湖泊,没有悲剧般幽深奇诡的城堡,没有绿得能熔化你所有思绪的大森林。‎ C.另外六只显然是正在发育的小狼也从柳树丛中跑了出来,它们喜气洋洋地摇着尾巴,嬉戏着搅在一起。‎ D.明末清初牙雕“题扇”仕女,略显头大于身,明显受到明万历时期板画的影响,身材过度修长而显现出大头的仕女,彰显出明代工艺的特征。‎ 答案 C 解析 A项,休养—修养;B项,熔化—融化;D项,板画—版画。‎ ‎20.下列各句中,没有错别字的一项是(  )‎ A.南浔区从人民大会堂捧回了联合国教科文组织非物质文化遗产名录项目证书,标志着南浔古老的蚕桑习俗文化正式挤身世界级文化殿堂。‎ B.雷军说在45岁生日当天他收到了两份大礼,一份是360投资结盟酷派,另一份是华为发布荣耀6plus,“这个行业的竞争恐怖得一踏糊涂”。‎ C.一个人着装时不拘礼节可能表示你对交流沟通的另一方的漠视,肮脏的牛仔裤与笔挺气派的西装会给人以截然不同的印象。‎ D.记者从省文联获悉,由文化部、中国文联、中国美协共同主办的“第十二届全国美术作品展览暨中国美术创作奖、获奖题名作品展”15日在中国美术馆开幕。‎ 答案 C 解析 题干问的是“没有错别字的一项”,关键词“没有”,选项中C项没有错别字,其他选项中应该修改为:A项,挤身—跻身;B项,一踏糊涂—一塌糊涂;D项,题名—提名。‎ ‎[3年高考真题集训]‎ ‎1.[2016·浙江高考]下列各句中,没有错别字的一项是(  )‎ A.当前,文艺创作最突出的问题是浮燥,急功近利,粗制滥造,不仅是对文艺的一种伤害,也是对社会精神生活的一种伤害。‎ B.电视剧播出前,剧组为聚人气而做密集宣传,虽无可厚非,也应把握尺度;低俗的噱头或许能暂时搏得关注,但终究不会提升电视剧本身的价值。‎ C.史铁生、霍金或许抱怨过不公的命运,却并不曾在这个飞扬拔扈的对手面前认输,他们拼尽全力与对手掰手腕,直至打败对手,取得胜利。‎ D.影片《荒野猎人》中,“小李子”扮演的不再是西装革履、风度翩翩的潇洒绅士,而是蓬头垢面、茹毛饮血,与自然鏖战的拓荒英雄。‎ 答案 D 解析 A项,浮燥—浮躁;B项,搏得—博得;C项,飞扬拔扈—飞扬跋扈。‎ ‎2.[2015·四川高考]下列词语中,没有错别字的一项是(  )‎ A.妨碍 功夫片 钟灵毓秀 管中窥豹,可见一斑 B.梳妆 吊胃口 瞠目结舌 文武之道,一张一驰 C.辐射 入场券 循章摘句 风声鹤唳,草木皆兵 D.蜚然 直辖市 秘而不宣 城门失火,殃及池鱼 答案 A 解析 B项,“驰”应为“弛”;C项,“循”应为“寻”;D项,“蜚”应为“斐”。‎ ‎3.[2015·浙江高考]下列各句中,没有错别字的一项是(  )‎ A.风电属于绿色清洁能源,行业主管部门和相关企业不能墨守成规,应该把握机遇,发挥我们幅原辽阔、风能资源丰富的优势,大力发展风电。‎ B.许多造诣远不能与他媲美的人早已声名雀起,他却仍然不急不躁,保持着艺术家应有的淡泊与执着,相信自己终究会跻身真正的大师行列。‎ C.为了抑制城市机动车数量的快速膨胀,某市实施限牌新政,规定参与摇号竞价的申请人必须持有驾照,这一门槛绊住了7万多人。‎ D.活根吸水与花茎泡水养出来的花,乍看似无二致,但一段时间后命运迥异:一个让你忍不住精心浇灌,另一个新鲜过后被弃若蔽屣。‎ 答案 C 解析 此题考查字形的识记能力。解答此题,可用比较排除法。在理解句意的基础上,可以先排除固定词语、常用词语,然后,结合词语意思来辨别词语搭配是否正确。A项,“幅原”应为“幅员”,幅:宽度;员:周围。B项,“声名雀起”应为“声名鹊起”。D项,“蔽屣”应为“敝屣”。敝:破旧,破烂。‎ ‎4.[2015·天津高考]下列词语中没有错别字的一组是(  )‎ A.透彻 频律 攻坚战 振聋发聩 B.通谍 竞聘 节骨眼 锋芒毕露 C.精悍 杂糅 识时务 礼尚往来 D.坐标 博取 辨证法 大相径庭 答案 C 解析 A项,频率;B项,通牒;D项,辩证法。‎ ‎[2年全国模拟重组]‎ ‎1.[2017·甘肃调研]下列字词书写正确的一项是(  )‎ A.苍琼 酩酊大醉 咗证 吗啡 B.燥热 懵懂不知 琥粕 孽障 C.搏弈 按步就班 执牛耳 呼啸 D.嬉笑 嗷嗷待哺 双面人 热忱 答案 D 解析 A项,苍穹、佐证;B项,琥珀;C项,博弈、按部就班。‎ ‎2.[2017·山东师大附中模拟]下列词语中,没有错别字的一项是(  )‎ A.精粹 头炷香 拾人牙惠 精诚所至,金石为开 B.凑合 挖墙脚 名缰利锁 城门失火,殃及池鱼 C.通牒 警戒线 死皮癞脸 风声鹤唳,草木皆兵 D.吐槽 冷不妨 额手称庆 前事不忘,后事之师 答案 B 解析 A项,惠—慧;C项,癞—赖;D项,妨—防。‎ ‎3.[2016·天津十二区联考]下列词语中没有错别字的一组是(  )‎ A.装璜 紧箍咒 焦阳似火 钉是钉,铆是铆 B.流弊 玻璃钢 革故顶新 言者无罪,闻者足诫 C.旌旆 绊脚石 要言不烦 明修栈道,暗度陈仓 D.气概 高材生 再劫难逃 万事俱备,只欠东风 答案 C 解析 A项,装潢、骄阳似火;B项,革故鼎新、闻者足戒;D项,在劫难逃。‎ ‎4.[2016·烟台模拟]下列词语中,没有错别字的一组是(  )‎ A.销魂 脱口秀 义愤填赝 蓬生麻中,不扶自直 B.馨香 洽谈会 舐犊情深 静若处子,动若逃兔 C.决对 和事佬 鳞次栉比 一夫当关,万夫莫开 D.梦魇 阴凉地 釜底抽薪 冯唐易老,李广难封 答案 D 解析 A项,“义愤填赝”应为“义愤填膺”;B项,“动若逃兔”应为“动若脱兔”;C项,“决对”应为“绝对”。‎ ‎5.[2016·北师大二附中模拟]下列词语中,没有错别字的一项是(  )‎ A.箭镞 激扬文字 苇编三绝 多行不义必自毙 B.爆涨 令人心仪 面黄饥瘦 画虎不成反类犬 C.博弈 不测之渊 大笔如椽 识时务者为俊杰 D.斩获 集液成裘 云销雨霁 解铃还须系铃人 答案 C 解析 A项,苇—韦;B项,爆—暴,饥—肌;D项,液—腋。‎ ‎6.[2016·天津南开中学模拟]下列汉字书写完全正确的一项是(  )‎ A.寂寞 杀风景 涣然一新 邯郸学步 B.诀别 挖墙脚 怀瑾握瑜 适得其反 C.弩钝 掉书袋 怒发冲冠 磋砣岁月 D.憧憬 跑龙套 根深蒂固 坐壁上观 答案 B 解析 A项,焕然一新;C项,驽钝、蹉跎岁月;D项,作壁上观。‎ ‎7.[2017·宁波十校联考]下列词语中,没有错别字的一组是(  )‎ A.冀望 建档 捷足先登 宁缺毋烂 B.抵御 修葺 玲珑剔透 信马由缰 C.壁垒 赎职 心驰神往 视如寇仇 D.缄默 疏浚 得垄望蜀 望风响应 答案 B 解析 A项,烂—滥;C项,赎—渎;D项,垄—陇。‎ ‎8.[2016·和平区质检]下列词语中,没有错别字的一组是(  )‎ A.膏梁 青涩 雍容华贵 豆寇年华 B.缴纳 戍边 平心而论 得鱼忘筌 C.桀骜 羁旅 磨肩接踵 励精图治 D.袅娜 覆盖 开城布公 呕心沥血 答案 B 解析 A项,膏梁—膏粱,豆寇年华—豆蔻年华。C项,磨肩接踵—摩肩接踵。D项,开城布公—开诚布公。‎ ‎9.[2016·浙江五校联考]下列词语中没有错别字的一组是(  )‎ A.谛造 矫键 四不像 解铃还须系铃人 B.渲泄 内敛 炒鱿鱼 曾经苍海难为水 C.贸然 气慨 流水帐 兵来将挡水来土淹 D.脉搏 笼络 跆拳道 文武之道一张一弛 答案 D 解析 A项,缔造、矫健。B项,宣泄、沧海。C项,气概、流水账、水来土掩(堰)。‎ ‎10.[2016·山东模拟]下列词语中没有错别字的一组是(  )‎ A.景仰 利欲熏心 汪洋咨肆 一言以蔽之 B.蜂拥 贸然行事 戛然而止 万变不离其宗 C.松弛 骨鲠在喉 察言观色 防民之口,胜于防川 D.排斥 渡日如年 即景生情 顾左右而言他 答案 B 解析 A项,汪洋恣肆;C项,防民之口,甚于防川;D项,度日如年。‎ ‎11.[2016·浙江模拟]下列各句中,没有错别字的一项是(  )‎ A.中国汉字听写大会,一个寓教于乐的传播创意,一场风靡全国的知识游戏,它激发了公众对传统文化的热情,也激活了传统在当下的生机。‎ B.人类文明其实有它脆弱的一面,洪水、瘟疫、战火都能轻易损毁它。然后我们在残存的基础上再弥补、修复,历代主要也靠书籍传递的人文星火来跨越精神天堑。‎ C.医疗骗子把生命当成生意,不惜重金占领搜索引擎入口,以精心炮制的虚假宣传为诱饵,大事吹嘘“神奇技术”与“惊人疗效”,句句戳中患者痛点,使患者甘愿压上身家性命“赌一把”。‎ D.唐山市市长曾多次召开会议安排布署关停工作,通过政务调度微信群实时关注进度,协调解决问题;常务副市长多次到一线督导,组织多部门采取综合措施倒逼企业加快关停进度。‎ 答案 A 解析 B项,薪火;C项,大肆;D项,部署。‎ ‎12.[2017·绍兴一中模拟]下列各句中,没有错别字的一项是(  )‎ A.安慰、揉腿、喂饭、伸手去接呕吐物……这些体贴入微、让人动容的举动,就发生在60多岁的老俩口带着80多岁的老师去看海途中的航班上。‎ B.听到这个令人抖然一惊的好消息,他不由得深深感慨:看来先前的一切担忧都是庸人自扰、平添烦恼罢了。‎ C.杯中的水,池中的水,湖中的水,它们最容易达到平静状态。人生中,不少人追求平静,希望能像平静的水那样,四平八稳,优哉悠哉。‎ D.物质日益强大而膨胀,而精神的萎缩状态正在加剧,在这种“内忧外患”的夹击之下,我们被裹挟其中,身不由己,身心交瘁。‎ 答案 D 解析 A项,老两口;B项,陡然一惊;C项,优哉游哉。‎ ‎13.[2017·浙江模拟]下列各句中,没有错别字的一项是(  )‎ A.这些官员都有幸福和睦的家庭,然而,遗憾的是父母、妻儿都未能使其幡然醒悟。最终,他们触犯党纪国法,追悔莫及,留给家庭永远的伤害。‎ B.“职业教育活动周”的设立,目的是要在全社会弘扬劳动光荣、技能宝贵、创造伟大的时代风尚,形成“崇尚一技之长、不唯学力凭能力”的良好氛围。‎ C.对唐宋诗的评价,往往因个人爱好的不同而大相径庭,其实两个朝代的不同风格的诗歌,不应该用一种故化的标准去评价。‎ D.昨天,广元青川上演了一场真实版的“人熊大战”,面对突如奇来的袭击,老人一跃而起将狗熊扑倒,赤手空拳与其在地上厮打近10分钟。‎ 答案 A 解析 B项,学力—学历;C项,故化—固化;D项,突如奇来—突如其来。‎ ‎14.[2016·浙江五校联考]下列各句中,没有错别字的一项是(  )‎ A.每个党政组织,每个领导干部都必须服从和遵守宪法法律,不能把党员职务当作个人谋权徇私的挡剑牌。‎ B.记者从省文联获悉,由文化部、中国文联、中国美协共同主办的“第十三届全国美术作品展览暨中国美术创作奖、获奖题名作品展”4月15日在中国美术馆开幕。‎ C.3月21日,由贵州省委宣传部、贵州省旅游局共同主办的“山地公园省·多彩贵州风”2016全国旅游推广活动在杭州盛大启动,邀请四方宾朋去贵州作客。‎ D.仰视戏台中间的“鸡笼顶”藻井,用了数百个花板榫接成一个金灿灿的旋涡,藻井四角是四个代表福祉的变形蝙蝠。‎ 答案 D 解析 A项,挡剑牌—挡箭牌;B项,题名—提名;C项,作客—做客。‎ ‎15.[2017·杭州七校联考]下列各句中,没有错别字的一项是(  )‎ A.经过大动作的反赌扫黑行动,扭曲、丑恶的足坛歪风邪气被重拳击破,盘根错结的利益交换被连根拔起,中国足球回到健康发展的轨道。‎ B.《狼图腾》在中国上映后获得了票房口碑双丰收,在蒙古国上映的首日,电影票便告售罄,在意大利也非常火爆,仅次于好莱坞动画大片《疯狂外星人》以及《灰姑娘》。‎ C.瑞典皇家科学院12日宣布,将2015年诺贝尔经济学奖授于经济学家安格斯·迪顿,以表彰他在消费、贫穷与福利方面的研究贡献。‎ D.作品的伟大与否,和作品究竟写了些什么并不会有必然的关联。但是,任何一个时代都必定要呼唤自己的主弦律作品。它是时代和历史前进的天赋使命。‎ 答案 B 解析 A项,盘根错结—盘根错节。盘:弯曲;错:交错;节:枝节。树根盘绕,枝节交错。比喻事情复杂,纠缠不清。C项,授于—授予,授于是传授某种技艺或者知识给特定的人;授予,指给某人某单位奖章奖励的一种动作形式。D项,主弦律—主旋律。‎ 考点三十一 标点符号 考点名片 考点内容 重点考查5种点号(顿号、逗号、分号、冒号、问号)、5种标号(书名号、括号、引号、破折号、省略号)。‎ 考查形式 ‎①以句中辨正误的形式综合考查多种标点,选择题,3分。②作文中也要求正确使用标点符号。‎ 趋势分析 仍将重点考查5种点号(顿号、逗号、分号、冒号、问号)、5种标号(书名号、括号、引号、破折号、省略号)。①少数省市高考题考查该考点。②以句中辨正误的形式考查标点使用正误。‎ ‎1.下列句子中,标点符号运用正确的一项是(  )‎ A.近日,教育部发布通知,提出高校要建立弹性学制,允许在校学生休学创业。(12月11日《京华时报》)‎ B.作为世界第二大经济体的中国,在“互联网外交”中自信地展示实力,与世界各极实现“沟通中互信,合作中共赢。”‎ C.光明日报新闻报道策划部、文艺部和光明日报出版社共同主办的《第一届光明微博微小说大赛》,于3月1日开始征稿。‎ D.古人说:“芭蕉分绿上窗纱”,眼光未免太低,只是阶前窗下的所见而已。若登楼眺望,芭蕉便落在眼底,应见“梧桐分绿上窗纱”了。‎ 答案 A 解析 B项,句号放在引号外;C项,书名号应为双引号;D项,冒号应去掉。‎ ‎2.下列各句中,标点符号使用正确的一项是(  )‎ A.嘉峪关曾是中西往来的要塞,明朝时人们须持有“关照”方可出入嘉峪关。“关照”是出入关塞的凭证:“关”本意为门闩,引申为关塞,“照”是公文、证件。‎ B.“开阳”就是战国时期赵国代郡的“安阳邑”,《史记·赵世家》也记载了“赵主父武灵王封长子章为代郡安阳君(武灵王自号为主父)”的史实。‎ C.短短几年间,宁波农村幼儿园到底发生了怎样的变化?如何实现了华丽转身?带着这样的疑问,记者近日走访了宁波的一些农村幼儿园。‎ D.近年来,东北地区耕地遭受污染情况严重,主要原因是农田中化肥、农药、农膜超量使用、养殖场畜禽粪便随意处理、大型企业废水、废渣随意排放。‎ 答案 C 解析 A项,第二个“关塞”后的逗号应改为分号,因这两个句子是并列的分句,并列分句之间一般用分号;B项,括号及里面内容应在“赵主父”后,解释谁就紧跟着放在谁的后面;D项,第三个和第四个顿号应改为逗号,因为这两个和前边不属于同一层次。‎ ‎3.下列各句中,标点符号使用正确的一项是(  )‎ A.养殖业的发展要结合市场需求,更要结合国情。国外的养殖方式:大规模、产业化自然是好处很多,但也不能照搬。‎ B.各国展示的最新科技成果,包括太阳能技术、资源循环技术、固体废弃物无害化、减量化、资源化处理技术等。‎ C.《孟子》里有各种圣人,表现各不相同,但有一点相同,即“行一不义,杀一不辜而得天下,皆不为也”(《孟子·公孙丑上》)。‎ D.我不知道这一实验到底能不能得出最终的结论?但我一定要坚持下去,直到把问题搞个水落石出,尽管我面前困难重重。‎ 答案 C 解析 A项,冒号应改为破折号;B项,第一、二个顿号应为逗号;D项,问号应为逗号。‎ ‎4.下列语句中,标点符号使用正确的一项是(  )‎ A.领导者要深入了解国情,要有“如履薄冰,如临深渊”‎ ‎(《诗经·小雅》)的自觉,牢记人民的利益高于一切。‎ B.袁阔成先生博采众长,吸收话剧、戏曲、相声……等艺术形式之长,用民族语言诠释了评书的艺术真谛。‎ C.鲁迅先生所写的:“寄意寒星荃不察,我以我血荐轩辕”的誓言,道出了家与国休戚与共、个人命运与民族兴亡息息相关的忧患意识。‎ D.古人说文章是“经国之大业,不朽之盛事,”共产党人读书作文,更要把目光投向干事创业、治国理政的大境界。‎ 答案 A 解析 B项,“省略号”与“等”不能连用,删其一。C项,“写的”后面的“冒号”去掉。D项,“盛事”后的“逗号”放在“引号”外面。‎ ‎5.下列各句中,标点符号使用正确的一项是(  )‎ A.关于什么是大数据技术?业界并无标准答案,但一般而言,可以将它理解为对海量数据的计算和加工,其核心是预测。‎ B.中纪委监察部网站刊登了题为“高压惩腐无禁区无特区无盲区”的评论员文章,称国企将成为惩治腐败的重要“战场”。‎ C.只要你点开关于“双十一”抽奖调查的网页,网络账户里的钱很快就被骗子盗去了——“双十一”已成为木马病毒青睐的对象。‎ D.《国家中长期教育改革和发展规划纲要(2010~2020年)》提出,要逐步形成分类考试、综合评价、多元录取的招考制度。‎ 答案 D 解析 A项,问号应改为逗号;B项,引号应改为书名号;C项,破折号改为冒号。‎ ‎6.下列句子中,标点符号运用正确的一项是(  )‎ A.环境保护部部长周生贤表示,要进一步加大环境执法力度,严管“两高一资”行业(高污染、高能耗、资源消耗型),集中开展钢铁、涉砷行业专项检查,巩固饮用水源保护区集中整治成果。‎ B.作为世界第二大经济体的中国,在“互联网外交”中自信地展示实力,与世界各极实现“沟通中互信,合作中共赢。”‎ C.昨天,教育部发布通知,提出高校要建立弹性学制,允许在校学生休学创业。‎ D.季羡林先生曾说:“如果翻译的是不需要的垃圾,翻译再多有什么意义?至于‘翻译强国’的标准是什么、怎么定、谁来评?都说不准。”‎ 答案 C 解析 A项,括号解释的内容为“两高一资”,应把括号及其内容移到“两高一资”的后边;B项,句号放在引号外;D项,“谁来评”后面的问号改为逗号。‎ ‎7.下列各句中,标点符号使用恰当的一项是(  )‎ A.2016年“十一”期间,湖北318家景点门票降价,云南全省景点门票打八折,山东曲阜推出“背《论语》免费游三孔”活动……今年假期,各地景区门票不再是“涨声”一片。‎ B.新疆将用三至五年时间,力争在冶金、化工、纺织、农副产品加工、旅游和物流等领域组建五、六个具有一定规模和实力的“百亿”企业集团。‎ C.许多消费者面对种类繁多的食品发愁:怎么知道究竟是不是转基因,生活中有无可能完全避免转基因,那些在网上流传甚广的鉴别转基因食品的方法正确吗?‎ D.考生赶考,请携带您的有效身份证件和准考证(“居民身份证”“军官证”“文职干部证”或“军校学员证”),若无上述证件,将无法进入考场。‎ 答案 A 解析 B项,“五”与“六”间的顿号删去。C项,连续发问用问号,两个逗号改为问号。D项,括号及其内容应放在“身份证件”之后。‎ ‎8.下列各句中标点符号使用正确的一项是(  )‎ A.要写出好诗,就不得不在这两方面注意:一方面要做诗人人格的涵养,另一方面要做诗的艺术训练,写出自然优美的音节,协和适当的词句。‎ B.八月初五是唐玄宗的生日,被定名为“千秋节”(又称“千秋金鉴节”),到这一天全国都铸造镜子,当作礼物送人以庆祝长寿。‎ C.谁都知道“蜀锦”是指四川成都织造的花锦,可是“蜀锦”究竟是个什么样子?在历史发展中,每个时代花样有什么特征?还少有人认真注意过。‎ D.有个经济史料《范子计然》,曾道及当时山东盛产的锦绣价值,“齐细绣文,上等匹值二万,中值一万,下值五千。”‎ 答案 B 解析 A项,“涵养”后面的逗号改为分号,“音节”后面的逗号改为顿号。C项,两个问号改为逗号。D项,后引号中的句号放在外面。‎ ‎9.下列各句中,标点符号使用正确的一项是(  )‎ A.近日,大范围雾霾再次笼罩我国北方一些地区,记者调查发现,十面“霾”伏下,体育活动开不开展,怎么开展,学生、家长和学校都很困惑。‎ B.《国家中长期教育改革和发展规划纲要》(2010~2020年)(简称《教育规划纲要》)在多处提出,政府及其部门要树立服务意识,改进管理方式。‎ C.一段时间以来,有的干部深入基层蹲点调研,真实掌握第一手资料,有的与群众同吃同住同劳动,获得了与坐办公室完全不一样的体验,有的询问群众的急难之事,给出时限做出承诺。‎ D.在微博上,无论你是机构、组织,还是名人,都同普通用户一样拥有完全平等的发言权,“人人都有麦克风,”在这里得到了最好的诠释。‎ 答案 A 解析 B项,“(2010~2020年)”应放在书名号内。C项,“资料”“体验”后都应该用分号。D项,“人人都有麦克风,”逗号放引号外。‎ ‎10.下列各句中,标点符号没有错误的一项是(  )‎ A.羊年春晚将贯彻节俭办春晚的原则,勤俭节约、不砸钱、控制舞美和舞台规模、严格预算和成本、运用好虚拟技术,力求内涵的丰富。‎ B.作为广大“跑友”们一年一度的节日,北京马拉松昨日开跑。不巧的是,今年的北京马拉松遭遇“霾伏”。既然遇到雾霾,比赛为什么不延期?‎ C.航天飞行控制中心总工程师周建亮告诉记者,对测控而言,“最关键的阶段就是再入返回段,最大的难点和亮点就是采用‘半弹道跳跃式再入’方式返回地球。”‎ D.近年,随着低龄留学热兴起,越来越多的小留学生走出国门,家长却不知道娃娃在外适应吗?学校能让人放心吗?低龄留学,哪些国家可陪读?‎ 答案 B 解析 A项,顿号改为逗号;C项,句号移到引号外;D项,三处问号均使用错误。‎ ‎11.下列语句中,标点符号使用正确的一项是(  )‎ A.北京麦当劳成为具有中国特色的“美国文化”,为了吸引更多的家庭来聚餐,北京麦当劳把《欢聚麦当劳,共享家庭乐》作为他们的广告主题之一。‎ B.记不清都是在地坛的哪些角落里了,我一连几小时专心致志地想关于死的事,也以同样的耐心和方式想过我为什么要出生?‎ C.鲁迅对中国青年寄予了很大的希望,“愿中国青年都摆脱冷气,只是向上走。”(《热风·随感录四十一》)于是写下了这篇题为“死水”的文章。‎ D.我已经爱过,恨过,欢笑过,哭泣过,体味过,彻悟过……细细想来,便知晴日多于阴雨,收获多于劳作。‎ 答案 D 解析 A项,书名号改为引号;B项,问号改为句号;C项,第一个句号去掉,在括号后加上逗号。‎ ‎12.下列各句中,标点符号使用正确的一项是(  )‎ A.去年到今年频发的高官贪腐案为人们敲响了警钟,这些官员案发前后境遇的巨大落差再次提醒我们——“不义而富且贵,于我如浮云。”‎ B.目前大学毕业生就业存在一种奇怪的现象:一方面很多学生毕业后找不到工作,一方面很多民营以及西部边远地区招不到需要的工作人员,出现这种现象的原因之一在于大学毕业生没有树立正确的择业观。‎ C.温庭筠以前,诗人存词甚少,到温庭筠才“能逐管弦之音为侧艳之词”(《旧唐书·温庭筠传》)。(《词在唐五代和两宋的发展过程及其流派》)‎ D.曼德拉,论经历,他不是唯一曾受牢狱之灾的政客;论权力,比他执政国土面积大,比他掌权时间长,比他拥有更绝对权威的人物,更不在少数。‎ 答案 C 解析 A项,句号应放在引号之后;B项,第二个逗号改为句号;D项,第四、第五个逗号改为顿号。‎ ‎13.下列语句中,标点符号使用正确的一项是(  )‎ A.业内人士表示,目前房地产市场已经进入由“量减”向“价跌”转变的关键时期,调控政策需要继续坚持,再次发力,以保证房价回到合理区间。‎ B.《法制日报》记者经深入调查后发现,毒豆芽在沈阳乃至全国来说,并不是一个“新生事物”,而是一个由来已久的老问题。‎ C.从国家伦理看,正如歌曲中所唱的:“家是最小国,国是千万家”,每个成员退离休后应得到平等回报。‎ D.由于一些院校的不负责任,很多考生都有报名2小时、候考5小时但考试只用几分钟的“不爽”经历,这让考生及家长们不住地抱怨:千里奔波而来就是为了排队吗,我们不遭这个罪了行不行?‎ 答案 A 解析 B项,“毒豆芽”应加引号;C项,冒号应去掉;D项,最后一个逗号应改为问号。‎ ‎14.下列各句中的标点符号使用正确的一项是(  )‎ A.11岁时,他给白城子一家地主老张家放牛;13岁,用他自己的话来说:“官升了一级”,给老张家放马了。‎ B.动物的游戏行为成为行为研究中最有争议的领域。争议的焦点,是动物为什么要进行游戏?‎ C.我想,首先是不管三七二十一,“拿来”!但是,如果反对这宅子的旧主人,怕给他污染了,徘徊不敢走进门,是孱头;勃然大怒,放一把火烧光,算是保存自己的清白,则是混蛋。‎ D.这孩子看上去有十五、六岁的样子,本应该读书的他却早早地挑起生活的重担。‎ 答案 C 解析 A项,冒号用错;B项,该句整体上是陈述语气,最后的问号改为句号;D项,顿号去掉。‎ ‎15.下列句子中标点符号使用正确的一项是(  )‎ A.面对世界首富比尔·盖茨,为什么人们如此“吝啬”仇富心理?而对他充满了崇敬和敬佩?‎ B.经过10年磨炼,逐渐形成由张瑞锋、祝勇和庞培、宁肯为代表的新散文写作群体,产生了诸如《算术题》《牙》《旧宫殿》《收藏》等一批独创性的作品。‎ C.2月15日,全国人大委员会新闻发言人指出:本次两会将对驻会代表严格要求,不允许各省市代表自带厨师、自带食品、自带加工原料(2月16日《娱乐信息报》报道)。‎ D.图书馆里的学生几乎清一色地都看英语四六级、托福、雅思、计算机、经济、法律……等应用类书籍,大家都忙于考研、考证、考公务员、考出国英语。‎ 答案 B 解析 A项,第一个问号改为逗号;C项,最后的句号应放在上括号之前;D项,去掉省略号。‎ ‎[3年高考真题集训]‎ ‎1.[2016·山东高考]阅读下面文段,完成问题。‎ 黟县的西递和宏村,拥有蜚声海内外的徽派建筑群。两村背依青山,清流抱村穿户,数百幢明清时期的民居静静伫立。高大奇伟的马头墙有骄傲睥睨的表情、跌宕飞扬的韵致灰白的屋壁被时间涂画出斑驳的线条。李白的“黟县小桃源,烟霞百里间。地多灵草木,人尚古衣冠”,道出了这里山水风物的幽美、民风人情的淳厚从容。要真正领略徽派建筑之美,该是在西递村在都市的喧哗之外,西递向我们呈现了一种宁静质朴的民间生活。从远处望去,西递是一片线条简洁的黑瓦铺成的屋顶和高大的白墙,黑白相间错落有致。迈入老屋你会发现,这些老屋内部的繁复精致与外部的简洁纯粹形成鲜明的对照,徽派建筑中著名的三雕木雕、砖雕、石雕在这里体现得淋漓尽致。‎ 文中画线处的标点,使用错误的一项是(  )‎ A.① B.②‎ C.③ D.④‎ 答案 A 解析 根据顿号的使用方法,如果并列词语中还有并列词语,大的并列词语要用逗号。本题中“有骄傲睥睨的表情”和“跌宕飞扬的韵致”是并列关系,中间已经使用顿号,那么“高大奇伟的马头墙有骄傲睥睨的表情、跌宕飞扬的韵致”和“灰白的屋壁被时间涂画出斑驳的线条”之间就不能再使用顿号,而应用逗号。‎ ‎2.[2015·福建高考]下列语句中,标点符号使用不正确的一项是(  )‎ A.在远走他乡、辗转天涯时,他才明白为什么那些远离家乡的人们会那么怀念故乡?‎ B.中国传统文化重视人生哲学,儒家坚持以修身为本,追求的是“齐家、治国、平天下”。‎ C.建立现代科学的三大基石是理论、实验和数学(包括计算、统计与建立在抽象模型基础上的演绎推理)。‎ D.2012年开始实施的新《标点符号用法》,我们要怎样贯彻:通知各校自行学习?组织骨干教师来培训?‎ 答案 A 解析 A项,是陈述句,不是疑问句,不应用问号,而应用句号。‎ ‎3.[2014·重庆高考]下列选项中,标点符号使用不正确的一项是(  )‎ A.中国重庆——德国杜伊斯堡,2011年开通的渝新欧铁路,横跨欧亚6国,为建立新丝绸之路经济带打下了基础。‎ B.“玉兔”号月球车登月后需要一条“被子”御寒和一个“闹钟”唤醒,承担这两项任务的是它的供电系统——太阳翼。‎ C.小王正在专心致志地看央视的《舌尖上的中国Ⅱ·家常》。叫她,不应;碰她,不理。她仿佛要从家常百味中体会人生百味。‎ D.她对着妈妈跺着脚大喊:“我的话你听见没有呀?干嘛要做那么多事呀?你知道不知道我不需要你做那么多、管那么多呀?”‎ 答案 A 解析 A项,破折号改为连接号,因为句中是连接两个地名。‎ ‎[2年全国模拟重组]‎ ‎1.[2017·绍兴一中模拟]下列各句中,标点符号使用正确的一项是(  )‎ A.阿拉法特是否因中毒而死?如果是的话,投毒者是以色列情报人员?还是巴勒斯坦反对派人士?有的疑问可通过尸检解答,而有的则可能成为永远的谜。‎ B.龙胜梯田的色彩像梯田本身一样层次分明:春天,银光闪烁;夏天,青翠点染;秋天,金黄耀眼;冬天,雪白盈野,一年四季可谓色彩迥异,各有神韵。‎ C.本来约好下午两点钟碰面,可我等了二、三个小时,他才来。‎ D.走进宁夏石嘴山市,眼前花草斗艳,绿树成荫,群鸟飞翔,湖面波光粼粼……令人难以想象,这是一座因煤而建、依煤而兴的现代重工业城市。‎ 答案 D 解析 A项,第二个问号应为逗号。B项,“雪白盈野”‎ 后的逗号应为句号。C项,概数中间不能用顿号。‎ ‎2.[2017·甘肃调研]下列各句使用的标点符号,完全正确的一项是(  )‎ A.“我们希望探索出一种‘全透明模式’,让捐款人知道自己的钱怎么用的?帮助了谁?每一笔钱都清清楚楚、有据可查。”马宁宇说。‎ B.演到哪里、观众追到哪里的话剧《立秋》,应观众要求反复播出的电视剧《亮剑》,清新明快、广为传唱的歌曲“我和你”……这些文艺作品先后荣获“五个一工程”奖。‎ C.卫生部的调查数据显示,中国平均每年每人要“挂8瓶水”,远远高于国际上2.5瓶~3.3瓶的水平,我国俨然已成“吊瓶大国”。‎ D.专家认为,我国文化改革发展面临两个目标:即总体战略目标和为实现总体战略目标打下坚实基础而制定的到2020年要实现的目标,目标清晰,催人奋进。‎ 答案 C 解析 A项,无疑而问,将问号改为逗号。B项,歌曲名应用书名号,应为《我和你》。D项,将第二个逗号改为句号,冒号一管到底,而“目标清晰,催人奋进”不属于两个目标的内容。‎ ‎3.[2016·德州一中模拟]下列各句子中,标点符号使用正确的一项是(  )‎ A.先秦两汉历史散文内容丰富,形式多样:有编年体的《左传》,有国别体的《国语》《战国策》,有纪传体的《史记》《汉书》等。‎ B.“怎么啦,你?”他问道,“你到底买?还是不买呢?你说呀!”‎ C.这次校团委组织的“为贫困生献爱心”活动所募集的捐款,据不完全统计:高三年级捐了一千元,高二年级捐了九百元,而高一年级则多达一千八百元,同学们捐款十分踊跃。‎ D.这种办法的收获是:学生有了自由甚至兴趣,在读写方面可以自己调节,不至于累得要死;教师呢,也可以轻松一些,不至于长年背诵“良辰美景奈何天,赏心乐事谁家院。”‎ 答案 A 解析 B项,“你到底买”后面的问号改为逗号;C项,“一千八百元”后用句号;D项,句号移到引号的外面。‎ ‎4.[2016·德州一中模拟]下列各句中,标点符号使用不正确的一项是(  )‎ A.“冰冻三尺,非一日之寒。”做好一件事情往往需要一个长期的积累过程,写作更是“路漫漫其修远兮。”‎ B.仙女湖,美丽的湖,神话的湖,充满诗情画意的湖,她像一个淡妆素抹的少女,含情脉脉地注视着来来往往的游客。‎ C.河南“天价过路费案”的偷逃过路费数额由原来的368万元减至49万元,这个结果是公检法部门自身作为使然,还是媒体和公众的舆论使然?‎ D.先秦历史散文内容丰富,形式多样:有编年体的《左传》,有国别体的《国语》《战国策》,有纪传体的《史记》《汉书》等。‎ 答案 A 解析 第二个句号移到引号后面。‎ ‎5.[2016·衢州质检]下列各句中标点符号使用正确的一项是(  )‎ A.诈骗犯有一种特长,讲究“适销对路”:你迷信鬼神,他就以鬼神为饵,你迷信权利,他就以权利相诱。‎ B.“留住济南”图片展深深吸引了观众。那小巷,那泉水,那绿草青苔,已经成为难以寻觅的影子——真可谓“梦忆深深深几许,一街一巷总关情。”‎ C.金,原指一切金属,如“金就砺则利”(《荀子·劝学》),现在专指黄金。‎ D.当我让脚步停留在林间的幽静里,抬眼望去,往日的绿色,忽然显得那么厚重斑斓。是被秋风所感染?还是绿色已经疲惫?是个性的宣泄?还是世界本身的灿烂?‎ 答案 C 解析 A项,“为饵”后的逗号,改用分号;B项,引号内的句号移到引号外;D项,“秋风所感染”“个性的宣泄”后面的问号均改为逗号。‎ ‎6.[2016·天津八校联考]下列各句中,标点符号使用正确的一项是(  )‎ A.不久前,美国加州大学神经生理学家汉斯·特贝、哈佛大学社会生物学家斯塔·阿特曼等提出一种引人注目的新假说——《学习说》。‎ B.古人多喜欢采用自然界的植物、动物作为意象来表达情感,《诗经·小雅·采薇》里的“昔我往矣,杨柳依依。”是我国最早用柳来渲染别情的诗句。‎ C.“就是——”她走近两步,放低了声音,极秘密似的切切地说,“一个人死了之后,究竟有没有魂灵的?”‎ D.清华大学培育出的人才数不胜数,让人不禁要问:是否这方土地养育了这批杰出的人才,是否清华园给予他们灵感、启迪?‎ 答案 C 解析 A项,“学习说”只是一种观点,不是一本书,不能用书名号,应改为引号。B项,“杨柳依依”后的句号应删去。D项,第二个逗号应改为问号。‎ ‎7.[2016·青岛检测]下列句子中,标点符号使用符合规范的一项是(  )‎ A.《新民丛报》虽然名为“报”,其实却是期刊,是梁启超等人于1902年在日本横滨创办的,曾产生过较大影响。‎ B.“雷锋精神”已经成为中华民族精神的重要内容,它体现了中华民族的传统美德,内涵丰富、意蕴深刻,是一面永不褪色、永放光芒的心灵旗帜。‎ C.对新建造纸厂,规定严格的污染物排限值,对现有造纸厂,设置两年半的达标排放过渡期,给企业一定时间进行改造。‎ D.他表示,要进一步加大环境执法力度,严管“两高一资”行业(高污染、高能耗、资源消耗型),集中开展钢铁、涉砷行业专项检查,巩固饮用水源保护区集中整治成果。‎ 答案 A 解析 B项,第一个顿号应为逗号,主谓短语做并列谓语。C项,第二个逗号改成分号。D项,括号内容为对“两高一资”的解释,故应把括号及其内容移到“两高一资”的后面。‎ ‎8.[2016·江西质检]下列各句中,标点符号使用正确的一项是(  )‎ A.香港中美聚焦网4月11日刊登题为《习近平的反腐运动进入关键阶段》一文,作者为美国克莱尔蒙特麦克纳学院教授裴敏欣。‎ B.北京阅读季针对“没时间”这一现象,特设立“走进北京阅读季,马上读书吧”的宣传口号。借古人读书“三上”的典故(马上、枕上、厕上),提醒人们阅读不拖延、常读久受益。‎ C.“千呼万唤始出来”的《舌尖上的中国》第二季今晚终于被端上电视荧屏的餐桌,“舌尖2”比第一季制作费高了30%,在内容上延续了第一季的主题——“探讨人与食物的关系”。‎ D.黄休复《益州名画录》把画分为四格——即逸格、神格、妙格、能格,他认为逸格为第一位,并声言:“画之逸格,最难其俦。”‎ 答案 C 解析 A项,书名号改为双引号;B项,括号放在“三上”后面;D项,破折号改为逗号,“能格”后的逗号改为句号。‎ ‎9.[2017·北京模拟]下列各项中,标点符号的使用符合规范的一项是(  )‎ A.近日一条有关“市售豆浆机、果汁机中的电机用的是工业润滑油”的帖子被广泛转载,豆浆机“有毒”,究竟是谣言?还是“揭黑”?‎ B.唐“开元杂报”是开元年间发行的邸报,该报系雕版印刷,质量不是很高。此报本身原无报名,因其发行是在开元年间,后人便称其为“开元杂报”。‎ C.这里选取几个亲历者,他们中有参加救援的消防界人士,有现场播报“9·11”事件的新闻主播,有在世贸大厦办公的公司职员……等等。‎ D.此次活动由中纪委、中联部、外交部协同江苏省纪委组织举办,范围涵盖了中国东部两省六市——这些地方在反腐倡廉方面进行的探索将通过国内外媒体的报道一一亮相世界。‎ 答案 D 解析 A项,“究竟是谣言?还是‘揭黑’?”属于选择性问句,前一个问号应改用逗号。B项,“开元杂报”是报名,第一套引号应改为书名号。C项,已经用了省略号,就不能再用“等等”,可删除“等等”和后面的句号。‎ 考点三十二 散文阅读 考点名片 考点内容 分析作品结构,概括作品主题;分析作品体裁的基本特征和表现手法;体会重要语句的含意,品味语言表达艺术;赏析作品的内涵,领悟作品的艺术魅力;从不同角度和层面发掘作品的意蕴;对作品进行个性化阅读和有创意的解读。‎ 考查形式 课标卷一般设置3个题目,题目类型有:多项选择题、简答题和论述题。‎ 趋势分析 ‎2018年应该不会发生变化。‎ 一、阅读下面的文字,完成1~3题。‎ 石砌的史书——阿斯哈图 张抗抗 ‎“阿斯哈图”是什么意思?阿斯哈图是蒙语,汉译为“险峻的山峰”。阿斯哈图在哪里?阿斯哈图在天边。去阿斯哈图怎样走?从赤峰启程至克什克腾旗经棚镇,然后再往北行三小时。若是夏季,那将是一次绿色的旅程。把绿色走到尽头,耸立的大山阻断了去路。大山拔地而起,如同草原剽悍的巨人卫队。换车上山,峰回路转,扬起一路烟尘。山脚是一层层茂密的白桦原始次森林,沿途可见灌木草坡交替,已是一派高原风光。抬头仰望,山顶嶙峋的巨石轮廓,似乎遥不可及。‎ 是什么原因让你走那么远的路,去阿斯哈图?是因为克什克腾国家地质公园独一无二的“冰川石林”。‎ 世上有很多冰川遗址和怪石奇林呵,为什么非要去阿斯哈图呢?我喜欢它那种宏伟的霸气、磅礴的气势和气度,那种不可一世的傲慢、遗世独立的尊严。在我曾经走过的地方,那些秀美的石林都太精致太诡谲了。‎ 阿斯哈图的山峰终于呈现在你眼前的时候,穿着什么颜色的袍子?我无法辨别它的颜色,因为它始终在不断的变化之中。灿灿斜阳直射之下,它是暖金色;背阴处却是中性的灰褐;远远的剪影是冷冷的黑;走到近前细细观摩,越发觉得它的调性难以确定,色块互相渗透融合,一抹赭红、一层青灰、一团麻黄、一片蓝绿;当它们混合在一起,就构成了斑驳沉着的杂色,似一座巨大的露天矿藏。‎ 阿斯哈图的岩石究竟是什么形状,能让人如此震撼?我无法描绘它的形状,因为每一座山岩的姿态,从每一个不同的角度望去,都会变成另一种样子。通常,它们会被牵强地解释成各种世俗的物体,被赋予某些象征性的意义,比如塔、鹰或是情侣。但在我看来,阿斯哈图是一座史前古城堡的遗址,高几十米的城墙巍然矗立,陡峭的烽火台依然坚硬,石砌的通道在荒草中依稀可辨,奇巧牢固的防守工事潜藏在拐角的暗处……那是一个消失了的巨人王国,山峦间每一道高不可及的断垣残壁上,都遗留着当年的巨人营造城堡的痕迹。若是从这一座砖墙走向另一座石壁,要经过开阔辽远的山梁与谷地。在夕阳下眺望周边5平方公里范围内的四处散落的城堡废墟,我确信巨人王国是曾经存在过的。唯因其巨,而不堪其重。‎ 那么你见到阿斯哈图石林城堡中的巨人脚印了吗?我见到山坡上以完整的巨石铺就的巨人卧榻。我看到山谷中岩石上烙刻的巨人手纹。在荒凉的城堡石壁下,开满金红浅紫的野花,每一片战栗在风中的花瓣,都残留着远古的气息。但我最终被阿斯哈图慑服,仍是因为阿斯哈图山巅上那些神奇的花岗石,每一块或方或扁,就像用锋利的刀斧削凿后打磨而成。岁月流逝风雨剥蚀,它们被挤压成棱角浑圆的石块石板,边缘清晰、线条流畅、厚薄均匀,然后一层一层地重叠,整整齐齐地码放,犹如一页页巨型厚纸,最后被装订成了一本本重量级的大书,随意地摞在山峦的高地上。我第一眼仰视它们的那个瞬间,有一种打开翻阅的冲动。我想这石头的书页里定是藏有文字的,每一页都有葱郁的白桦树林、烂漫的山花作为插图,这是一座用石头史书垒砌的城堡,深邃的岩缝里刻录着历史的沧桑。阿斯哈图,原来是一部巨人的史书。史书未著一字而尽得风流。‎ 你走不出阿斯哈图了,这一座巨大的地质博物馆,真解不开你的种种迷思吗?回望阿斯哈图,我看见巨石峰尖上的冰山漂砾插入云层,将绚丽的晚霞温柔地撕裂。‎ 想象着远古时期冰封雪盖的阿斯哈图,怎样在微弱的暖意中渐渐苏醒;高原隆升,顶开了巨大的冰盖,雪层崩塌;冰川融水,刨蚀浸蚀拔蚀冲蚀,终至水滴石穿水落石出。克什克腾的阿斯哈图石林,是冰川馈赠给人类的珍贵遗产。如此惊心动魄、波澜壮阔的大手笔,超越了人类已有的文字记录。‎ 后记:‎ 阿斯哈图纠正了我们以往对草原的肤浅认识。其实,没有巨人也没有巨人王国,只有地壳运动所留下的那部石砌史书,从此将被人们一次次用目光抚摸,然后,记住。‎ ‎(选自《张抗抗散文》,有删改)‎ ‎1.下列对作品有关内容和艺术特色的鉴赏,最恰当的一项是(  )‎ A.文中第一段巧妙设置三个问题,答案诗意浪漫,使读者在困惑和迷茫中揭开阿斯哈图的神秘面纱。‎ B.草原深处的阿斯哈图呈现出了令人感到惊异的地貌特征,冰川石林在平坦的草原地带显得格外突出。‎ C.画线句子运用了比喻、拟人的修辞手法,生动形象地描摹了阿斯哈图巨石蔚然高耸的形象特征。‎ D.阿斯哈图,原来是一部巨人的史书,作者从中读出了大自然的鬼斧神工,读出了石林的执着悲壮。‎ 答案 B 解析 A项,不是“困惑和迷茫”,而是“好奇和憧憬”。C项,没有运用比喻的修辞手法。D项,“读出了石林的执着悲壮”是夸大其词,文中没有具体体现。‎ ‎2.作品在写法上有哪些明显特点?分别有什么好处?请简要分析。‎ 答:___________________________________________________‎ ‎_______________________________________________________‎ ‎_______________________________________________________‎ 答案 ①以阿斯哈图为线索,进行了全方位的介绍,好处是线索分明、逻辑清晰;②以对话形式展开,一问一答,好处是主题集中,连贯晓畅;③描写与议论相结合,好处是由表及里,既得其形,亦得其神。‎ 解析 这是一篇典型的写景散文。首先,找到作品的写作对象和线索。本文以阿斯哈图为中心,写了其命名、地理位置、特征、人文内涵等方面的内容。其次,在表现形式上主要采用了问答对话,这也是散文中常见的一种手法,可从第1~7自然段开头第一句均是问句看出。最后就是表达方式的运用,本文主要是以描写、议论相结合的方式展开的。‎ ‎3.从内容上看,删去尾段后记部分,作品更显简洁,作者为什么还要增加尾段后记部分?结合全文,谈谈你的看法。‎ 答:___________________________________________________‎ ‎_______________________________________________________‎ ‎_______________________________________________________‎ 答案 ①艺术结构上,以“石砌史书”照应标题,同时总结全文,使结构严谨;②情感表现上,强调“抚摸,然后,记住”,再次强调阿斯哈图给人的震撼,升华情感;③思想内容上,纠正“对草原的肤浅认识”,深化了阿斯哈图的人文内涵,凸显作品主题。(其他答案言之成理亦可)‎ 解析 回答本题,可从作品艺术结构、情感表现、思想内容上进行分析,然后再结合文本内容概括。需要注意的是:探究此类题目,答案需要言之有据,能自圆其说。‎ 二、阅读下面的文字,完成1~3题。‎ 音乐 赵丽宏 ‎①深夜。无月,无风。带木栅栏的小窗外,合欢树高大的树冠犹如张开着巨臂的人影,纹丝不动,贴在墨一般深蓝的天幕上。一颗暗淡的星星孤独地挂在树梢上,像凝固在黑色人影上的一粒冰珠,冷峻而肃穆。‎ ‎②静。静得使人想到死亡。思绪的河流也因之枯涸,没有涟漪,没有飞溅的水花,没有鱼儿轻盈的穿梭……只有自己沉闷的呼吸,沉闷得像岩石,像龟裂的土地,像无法推动的铁门。难熬的寂静。‎ ‎③‎ 这时,突然有一种极轻微的声音从远处飘来,仿佛有一个小提琴手将弓轻轻地落到E弦上,又轻轻地拉了一下。这过程是那么短促,我还没有来得及品味其中的韵律,声音已经在夜空里消失。世界复又静寂。在我的小草屋里,这响动却留下了回声,一遍又一遍,委婉沉着地回荡着,回荡成一段优美的旋律,优美中蕴含着淡淡的忧伤,也流淌着梦幻一般的欣喜。眼前恍惚有形象出现——一个黑衣少女,伫立在月光下拉一把金黄色的小提琴,曲子是即兴的,纤手操持着轻巧的弓,在四根银弦上自由自在地跳跃滑行,音符奇妙地从弓弦下飘起来,变成一阵晶莹的旋风,先是绕着少女打转,少女黑色的长裙在旋风中翩然起舞,旋风缓缓移动,所达之处,一片星光闪烁。‎ ‎④渐渐地,我也在这旋风的笼罩之中了。我仿佛走进了一个辉煌的音乐厅,无数熟悉的旋律在我耳畔光芒四射地响起来。钢琴沉静地弹着巴赫,长笛优雅地吹着莫扎特,交响乐队大气磅礴地合奏着贝多芬……也有洞箫和琵琶,娓娓叙说着古老的中国故事……‎ ‎⑤终于,一切都消失了,万籁俱寂,只剩下我坐在木窗下发呆。窗外,合欢树的黑影被镀上一层亮晶晶的银边。月亮已经悄悄升起……‎ ‎⑥以上的经验,距今已有二十多年,那时我孤身一人住在荒僻乡野的一间小草屋里,度过了无数寂静的长夜。静夜中突然出现的那种声音,其实是附近的人家在开门,破旧的木门被拉动时,门臼常常发出尖厉的摩擦声,从远处听起来,这尖厉的声音便显得悠扬而奇妙,使我生出很多不切实际的幻想。门臼的转动和美妙的音乐,两者毫不相干,把它们联系在一起,似乎很荒唐,然而却又是那么自然。‎ ‎⑦我曾经请一位作曲家对音乐下一个定义,他几乎是不假思索地答道:“什么是真正的音乐?音乐是人类的爱和智慧的升华,是人类对理想的憧憬和呼唤。”他的回答使我沉思了很久。这回答当然不错,可是用这样的定义来解释其他艺术,譬如绘画和舞蹈,似乎也未尝不可。但音乐毕竟不同于其他艺术。音乐把人类复杂微妙的感情和曲折丰富的经验化成了无形的音符,在冥冥之中回响,它们抚摸、叩动、撞击甚至撕扯着你的灵魂,使浮躁的心灵恢复宁静,使干涸的心田变得湿润,也可以让平静的心灵掀起奇妙的波澜。音乐对听者毫无要求,它们只是在空间鸣响,而你却可以使这鸣响变成翅膀,安插到你自己的心头,然后展翅翱翔,飞向你所向往的境界……音乐是自由的,又是无所不在的。有什么记忆能比对音乐的记忆更为深刻,更为顽强,更为恒久呢?‎ ‎⑧我想起了最近欣赏的一场交响音乐会。指挥这场音乐会的是一位个子矮小、性格文静的中年人,当他站到庞大的乐队前面,不慌不忙地举起指挥棒时,就像一个骄傲而威严的大将军面对着他的千军万马……‎ ‎⑨乐队演奏的是瓦格纳的歌剧《唐豪赛》序曲。小小指挥棒挑出了惊天动地的声音。然而我的眼前既未走来朝圣的信徒,也没有舞出妩媚妖娆的仙姑,那位在盛宴上放歌豪饮的英雄更是无影无踪。我似乎又走到了二十年前我常常走的一道高耸的江堤上。灰色的浓云低低地压在我的头顶,眼前是浩瀚无际的长江入海口,浑黄的江水在云天下起伏翻滚,发出低沉的咆哮,巨大的浪头互相推挤着,成群结队向我扑来。巨浪一个接一个轰然打到堤壁上,又被撞成水花和白雾,飘扬到空中,飞溅到我的身上。我的整个身心逐渐湿润了,清凉了,郁积在心底的忧愁和烦闷在轰鸣的涛声中化成了轻烟,化成了白色的鸥鸟,振抖着翅膀翔舞在水天之间。浓重的铅云开裂了,露出了隙缝,一道阳光从隙缝中射进来,射在起伏的水面,波浪又把阳光反映到空中。我是在一片光明的包围之中了……‎ ‎(有删改)‎ ‎1.下列对作品有关内容和艺术特色的鉴赏,最恰当的一项是(  )‎ A.文章回忆和音乐有关的经历,思考音乐独特的价值,描写和议论结合,想象与联想并用,诗意地渲染出音乐的魅力。‎ B.住在荒僻乡野的作者听见了附近人家的开门声,误以为是小提琴的声响,进而产生一系列关于音乐的美好幻想,这也算是美丽的错误。‎ C.第③段“音符奇妙地从弓弦下飘起来,变成一阵晶莹的旋风”,运用通感手法化无形为有形,表现了音乐的灵动和飘逸之美。‎ D.文章从二十年前的感受写起,最后又联想起二十年前漫步的江堤,前后呼应,由虚到实,表现出音乐给人的强烈而深刻的震撼。‎ 答案 C 解析 A项除了描写和议论,还有抒情;B项“误以为是小提琴的声响”不妥,从原文可知作者知道这是开门声;D项“由虚到实”不当,二者都有虚有实。‎ ‎2.文章开头两段有关深夜的描写有什么作用?‎ 答:___________________________________________________‎ ‎_______________________________________________________‎ 答案 ①写出了作者所处环境的荒僻,营造出寂静沉闷的环境氛围,突出作者内心的孤寂和苦闷;②为后文出现的细微声音及关于音乐的联想作铺垫,前后对比,表现了音乐的美好和对人心灵的抚慰。‎ 解析 回答本题,主要应考虑散文中描写的作用。散文中开头运用环境描写,可以渲染和烘托气氛,突出人物形象或中心;优美的自然环境描写,还可以打动读者的心,使读者产生愉悦感,增强文章的感染力。具体回答时应结合文本具体分析。‎ ‎3.结合文章,回答下面的问题。‎ ‎(1)从文中看,作者认为音乐“不同于其他艺术”的特点是什么?‎ 答:___________________________________________________‎ ‎_______________________________________________________‎ 答案 ①音乐能通过无形的音符表现人类复杂的情感和体验;②音乐能通过声响撞击人的灵魂,抚慰人的心灵;③音乐无所不在,听者可以根据自己的需要展开自由的想象。‎ 解析 本题主要考查理解文意与筛选信息的能力。首先应找到答题区间,即文章的第⑦段,“音乐把人类复杂微妙的感情和曲折丰富的经验化成了无形的音符……你却可以使这鸣响变成翅膀,安插到你自己的心头,然后展翅翱翔,飞向你所向往的境界”,然后分层筛选概括即可。‎ ‎(2)文中第⑤段画线处写道:“窗外,合欢树的黑影被镀上一层亮晶晶的银边。”这样写有什么好处?请简要赏析。‎ 答:___________________________________________________‎ ‎_______________________________________________________‎ 答案 ①运用比喻修辞,形象地描绘了月亮升起后合欢树的美丽画面,更具意境美。②呼应开头,并借景象的变化,表现作者在想象音乐后,由孤寂苦闷变得宁静平和的心理状态。‎ 解析 本题主要考查体会重要语句的丰富含意及其作用的能力。解答此题的关键是进入文本,采用瞻前顾后法。具体回答时首先应明确技法,然后结合文本阐释意义及作用。‎ 三、阅读下面的文字,完成1~3题。‎ 人生 ‎[英国]戴维·赫伯特·劳伦斯 世界的开端和末日之间出现了人。人既不是创世者又不是被创者,但他是创造的核心。一方面,他拥有产生一切创造物的根本未知数;另一方面,又拥有整个已创造的宇宙,甚至拥有那个有极限的精神世界。人在喧闹、不完善和未雕琢的状态下诞生,是个婴儿、幼孩,一个既不成熟,又未定型的产物。他生来的目的是要变得完善,以至最后臻于完美,成为纯洁的生灵,就像白天和黑夜之间的星星,披露着另一个世界,一个没有起源亦没有末日的世界。‎ 人不可能创造自己,任何时候也不可能。他只能委身于创世主,屈从于创造一切的根本未知数。每时每刻,我们都像一种均衡的火焰从这个根本的未知数中释放出来。我们不能自我容纳,也不能自我完成,每时每刻我们都从未知中衍生出来。‎ 这就是我们人类的最高真理。我们的一切知识都基于这个根本的真理。我们是从基本的求知中衍生出来的。看我的手和脚:在这个已创造的宇宙中,我就止于这些肢体。但谁能看见我的内核,我的源泉,我从原始创造力中脱颖出来的内核和源泉?然而,每时每刻我在我心灵的烛芯上燃烧,纯洁而超然,就像那在蜡烛上闪耀的火苗,均衡而稳健,犹如肉体被点燃,燃烧于初始未知的冥冥黑暗与来世最后的黑暗之间。‎ 我们永远不会知道什么是起源,永远不会知道我们怎样才具有目前的形状和存在。但我们可能知道那生动的未知——让我们感受到的未知是怎样通过精神和肉体的通道进入我们体内的。谁来了?我们半夜听见在门外的是什么?谁敲门了?谁又敲了一下?谁打开了那令人痛苦的大门?然后,注意,在我们体内出现了新的东西,我们眨眨眼睛,却看不见。我们高举以往理喻之灯,用我们已有的知识之光照亮了这个陌生人。然后,我们终于接受了这个新来者,他成了我们当中的一员。‎ 那么,未知又是怎么进入我的呢?未知所以能进入,就因为在我活着时,我从来不封闭自己,从不把自己孤立起来。我只不过是通过创造的辉煌转换,把一种未知传导为另一种未知的火焰。我只不过是通过完美存在的变形,把我起源的未知传递给我末日的未知罢了。‎ 起先,我的精神惴惴不安,坐卧不宁。深更半夜时,它听到了从远处传来的脚步声。谁来了?呵,让新来者进来吧,让他进来吧。在精神方面,我一直很孤独,没有活力。我等待新来者。我的精神却悲伤得要命,十分惧怕新来的那个人。但同时,也有一种紧张的期待。我期待一次访问,一个新来者。我聆听着,我在精神里聆听着。我匆忙打开门。啊哈,门外没有人。我必须耐心地等待,一直等到那个陌生人。一切都由不得我,一切都不会自己发生。想到此,我抑制住自己的不耐烦,学着去等待,去观察。‎ 终于,在我的渴望和困乏之中,门开了,门外站着那个陌生人。啊,到底来了!啊,多快活!我身上有了新的创造,啊,多美啊!啊,快乐中的快乐!我从未知中产生,又增加了新的未知。我心里充满了快乐和力量的源泉。我成了存在的一种新的成就,创造的一种新的满足,一种新的玫瑰,地球上新的天堂。‎ 这就是我们诞生的故事,除此之外,别无他路。我就像森林边上的一座小房子。从森林的未知的黑暗之中,在起源的永恒的黑夜里,那创造的幽灵正悄悄地朝我走来。我必须保持自己窗前的光闪闪发亮,否则那幽灵又怎么看得见我的屋子?如果我的屋子处在睡眠或令人害怕的黑暗中,天使便会从房子边上走过。‎ 我害怕那朝我走来、富有创造力的陌生的未知吗?我怕,但只是以一种痛苦和无言的快乐而害怕。我怕那死神无形的黑手把我拖进黑暗,一朵朵地摘取我生命之树上的花朵,使之进入我来世的未知之中吗?我怕,但只是以一种报复和奇特的满足而害怕。因为这是我最后的满足,一朵朵地被摘取,一生都是如此,直至最终纳入未知的终端——我的末日。‎ ‎(有删改)‎ ‎1.下列对作品有关内容和艺术特色的鉴赏,最恰当的一项是(  )‎ A.这是一篇思想深刻的哲理散文,通篇采用第一人称,现身说法,娓娓道来,平易近人,亲切感人,毫无居高临下的说教感。‎ B.人不能创造世界,却是创造的核心;人虽然并不具有可塑性,但是每时每刻都会从未知中衍生出一个新的自我。‎ C.人在任何时候都不可能创造自己,只能屈从于未知,不过我们可能知道那些未知,要保持清醒的头脑和自我追求的激情。‎ D.文章多用比喻和象征,如“每时每刻我在我心灵的烛芯上燃烧”,“烛芯”比喻人生,“燃烧”象征思考,语言生动形象。‎ 答案 C 解析 A项,“通篇采用第一人称”和“现身说法”不当,文章不是通篇采用第一人称,也不是在讲述自己的事情。B项,“人虽然并不具有可塑性”与文意不符。D项,“心灵的烛芯”为一种特殊的比喻,如“思想的潮水”“思念的苦酒”等,定语为比喻的本体,中心词为喻体,“心灵的烛芯”即“心灵这根烛芯”。“燃烧”象征生命的蓬勃。‎ ‎2.请用简洁的文字概括文章的基本思路。‎ 答:___________________________________________________‎ ‎_______________________________________________________‎ 答案 文章先通过前四段阐述人生的每时每刻都从未知中衍生出来的道理,再由第5至8自然段说明人生怎样从未知中衍生,即未知怎样进入自我,最后结尾段指出人生不应惧怕任何未知。‎ 解析 作答本题,要善于抓住能体现文章思路结构的语句。如“这就是我们人类的最高真理”,从此句可知前面的内容是阐述“最高真理”的;再如“那么,未知又是怎么进入我的呢”“这就是我们诞生的故事”,据此可知两句之间的部分说明了人生是怎样从未知中衍生的。‎ ‎3.请赏析文中画线句的精妙之处。‎ 答:___________________________________________________‎ ‎_______________________________________________________‎ ‎_______________________________________________________‎ 答案 画线句用形象描绘的手法展现了人接受新事物时的情形,“远处传来的脚步声”象征未知的新事物,“精神惴惴不安,坐卧不宁”表现人在接触未知的新事物前的不安、紧张心理,连续两句“进来吧”则表现了人对未知的新事物的渴望之情。用第一人称,化抽象说理为形象描绘,易于为读者接受。‎ 解析 ‎ 首先要把这句话放在全段甚至全篇之中,理解其含义,然后明确其所用的表达技巧。就含义而言,画线句旨在表现人在接受新事物时的矛盾心理。就表达技巧而言,画线句则是将抽象道理形象化。将这两方面综合整理,即可得出答案。‎ ‎[3年高考真题集训]‎ 一、[2016·天津高考]阅读下面的文章,完成1~4题。‎ 在母语的屋檐下 彭程 少年时代的伙伴自大洋彼岸归来探亲,多年未见,把盏竟夜长谈。我们聊到故乡种种情形,特别谈到了家乡方言,兴之所至,后来两人干脆用家乡话谈起来。‎ 本来以为这么多年不使用,很多方言都已忘记,不料却在此时鲜明地复活了。恍惚中,甚至忆起了听到这些话时的具体情境,眼前浮现出了说话人的模样。友人感慨:真过瘾。‎ 在一种语言中浸润得深入长久,才有资格进入它的内部,感知它的种种微妙和玄奥,那些羽毛上的光色一样的波动,青瓷上的釉彩一般的韵味。几乎只有母语,我们从牙牙学语时就亲吻的语言,才应允我们做到这一点。‎ 关于母语,英文里的一个说法,最有情感温度,也最能准确地贴近本质:mother tongue。直译就是“妈妈的舌头”。从妈妈舌头上发出的声音,是生命降临时听到的最初的声音,浸润着爱的声音。多么深邃动人的诗意!在母语的呼唤、吟唱和诵读中,我们张开眼睛,看到万物,理解生活,认识生命。‎ 诗作为浓缩提炼过的语言,是语言的极致。它可以作为标尺,衡量一个人对一种语言熟悉和理解的程度。“眼看他起高楼,眼看他宴宾客,眼看他楼坍了”,说的是世事沧桑,人生无常。“而今识尽愁滋味,欲说还休。欲说还休,却道天凉好个秋”,说的是心绪流转,昨日迢遥。没有历史文化为之打底,没有人生经历作为铺垫,就难以深入地感受和理解其间的沉痛和哀伤,无奈和迷茫。它们宜于意会,难以言传。‎ 每一种语言都连接着一种文化,通向一种共同的记忆。文化有着自己的基因,被封存在作为载体和符号的特有的语言中。仿佛一千零一夜的故事中,阿里巴巴的山洞里,藏着稀世的珍宝。‎ ‎“芝麻开门吧!”咒语念起,山洞石门訇然敞开,堆积的珠宝浮光跃彩。‎ 但洞察和把握一种语言的奥秘,不需要咒语。时间是最重要的条件。在一种语言中沉浸得足够久了,自然就会了解其精妙。有如窖藏老酒,被时光层层堆叠,然后醇香。瓜熟蒂落,风生水起,到了一定的时候,语言中的神秘和魅惑,次第显影。音调的升降平仄中,笔画的横竖撇捺里,有花朵摇曳的姿态,水波被风吹拂出的纹路,阳光下明媚的笑容,暗夜里隐忍的啜泣。‎ 对绝大多数人来说,只有母语,才有这样的魅力和魄力,承担和覆盖。日升月落,春秋代序;昼夜不舍的流水,亘古沉默的荒野;鹰隼呼啸着射向天空,羊群蠕动成地上的云团;一颗从眼角滑落的泪珠有怎样的哀怨,一声自喉咙迸发的呐喊有怎样的愤懑。一切,都被母语捕捉和绾结,表达和诉说。‎ 我骄傲于自己母语的强大的生命力。五千年的漫长历史,灾祸连绵,兵燹不绝,而一个个方块汉字,就是一块块砖石,当它们排列衔接时,便仿佛垒砌了一个广阔而坚固的壁垒,牢牢守卫了一种古老的文化,庇护了一代代呼吸沐浴着它的气息的亿兆的灵魂。‎ 童年在农村度过。记事不久的年龄,有一年夏天,大人在睡午觉,我独自走出屋门到外面玩,追着一只蹦蹦跳跳的兔子,不小心走远了,一直走进村外一片茂密的树林中,迷路了,害怕得大哭。但四周没有人听到,只好在林子里乱走。过了好久,终于从树干的缝隙间,望见了村头一户人家的屋檐。‎ 一颗悬空的心倏地落地了。‎ 对于长期漂泊在外的人,母语熟悉的音调,带给他的正应该是这样的一种返归家园之感。一个汉语的子民,寄居他乡,母语便是故乡的方言土语;置身异国,母语便是方块的中文汉字。“官秩加身应谬得,乡音到耳是真归”,故乡的语言,母语的最为具体直观的形式,甚至关联到了存在的确凿感。‎ 因为时时相与,反而熟视无睹。就像对于一尾悠然游弋的鱼儿,水的环抱和裹挟是自然而然的,不需要去意识和诘问的。但一当因某种缘故离开了那个环境,就会感受到置身盛夏沙漠中般的窒息。被拘禁于全然陌生的语言中,一个人也仿佛涸辙之鲋,最渴望母语的濡沫。那亲切的音节声调,是一股直透心底的清凉水流。‎ 每一种语言的子民们,在自己母语的河流中,泅渡,游憩,俯仰,沉醉,吟咏,创造出灿烂的文化,并经由翻译传播,成为说着不同语言的人们共同的精神财富。以诗歌为证,《鲁拜集》中波斯大诗人伽亚谟及时行乐的咏叹,和《古诗十九首》里汉代中国人生命短暂的感喟,贯穿了相通的哲学追问;中世纪的意大利,彼特拉克对心上人劳拉的十四行诗倾诉,和晚唐洛阳城里,李商隐写给不知名恋人的无题七律,或者隽永清新,或者宛转迷离,各有一种入骨的缠绵。让不同的语言彼此尊重,在交流中使各自的美质得到彰显和分享。‎ 热爱来自母亲的舌尖上的声音,应该被视为是一个人的职责,他的伦理的基点。他可以走向天高地阔,但母语是他的出发地,是他不断向前伸延的生命坐标轴线上,那一处不变的原点。‎ ‎(原载《光明日报》,有删节)‎ ‎1.作者回忆童年迷路的经历,在文中有什么作用?‎ 答:___________________________________________________‎ ‎_______________________________________________________‎ 答案 ①内容上,用孩子迷路比喻游子离开母语,强调母语给人带来的庇护感和安全感。②结构上,呼应题目“屋檐”,引出下面的议论。‎ 解析 解答本题,应从内容和结构两方面入手。文章第11段描写了作者童年迷路的经历,从内容上来看,这段文字明写孩子迷路,害怕,望见屋檐而心安,实际上比喻漂泊在外的游子离开母语后的不安,强调母语能带给人庇护感与安全感;从结构上来看,该段文字写孩子最终“望见了村头一户人家的屋檐”,呼应了文章标题中的“屋檐”,并以此为喻,引出了下文的议论,呼唤人们回归“母语的屋檐”。‎ ‎2.赏析文中画线的文字。‎ 答:___________________________________________________‎ ‎_______________________________________________________‎ 答案 运用比喻、对比等手法,揭示出人和母语之间生死难离的关系,使事理具象化,生动形象。‎ 解析 赏析句子可从修辞手法入手。从画线句涉及的“就像”“鱼”与“水”“人”与“母语”这些关键信息中,我们可以得出画线句使用了比喻、对比的手法。然后指出手法的作用,并结合文章内容来分析。‎ ‎3.作者深情地诠释了母语的多重意义,请结合全文加以概括。‎ 答:___________________________________________________‎ ‎_______________________________________________________‎ 答案 ①母语可以拉近彼此的关系。②母语最早打通人与世界的联系。③母语可以自由地抒情状物。④母语包蕴文化基因,守卫民族文化。⑤母语给人以家的归宿感。⑥各民族用自己的母语创造了人类共同的精神财富。‎ 解析 “结合全文加以概括”给我们的信息是:通读全文,对文中重点语段、关键语句进行概括。“母语的多重意义”中的“多重”表明不止两三种,结合全文一一总结归纳。‎ ‎4.下列对文章的理解与分析,不正确的两项是(  )‎ A.文中引用英语mother tongue,是为了引出“妈妈的舌头”这一形象说法,强调母语的温馨可亲。‎ B.作者用“羽毛上的光色一样的波动”“青瓷上的釉彩一般的韵味”来形容母语的微妙和玄奥,是说母语宜于意会,难以言传。‎ C.文中引用阿里巴巴的故事,旨在说明封存在语言中的文化基因如珠宝般珍贵。‎ D.文中列举“昼夜不舍的流水”“亘古沉默的荒野”“一颗从眼角滑落的泪珠”等意象,意在说明,只有用母语才能准确言说它们的内在情韵。‎ E.文章融记叙、议论、抒情为一体,引经据典,华美而不失厚重,有较深的文化意蕴。‎ 答案 BC 解析 B项,“母语宜于意会,难以言传”无中生有。C项,原文引用阿里巴巴的故事,旨在说明“文化有着自己的基因,被封存在作为载体和符号的特有的语言中”,而非说明“封存在语言中的文化基因如珠宝般珍贵”。‎ 二、[2015·湖北高考]阅读下面的文章,完成1~4题。‎ 头脑中的旅行 彭程 ‎①对当代人来讲,旅行是一件平淡无奇的事情,但在古代,技术落后,交通不便,旅行经常和冒险联系在一起,另外还要有相当的经济实力作为后盾,因此,旅行对于很多人来说并非易事。那时候的一些人尤其是文人,愿望难以满足,只好经常借助于幻想,在头脑中旅行。文人许多是贫穷而兼病弱,却拥有敏锐的感受力和丰富的想象力,现实生活中的阻碍反而进一步激发起他们的热情。一幅图画,书里一段并不起眼的描绘,都能够成为点燃他们灵感的火种。借助无限的想象,他们能够生动地描绘出一个地方的景色氛围,读来有身临其境之感。‎ ‎②法国诗人波德莱尔就突出地体现了这样一种才华。他的不少篇章,都表达了对于远方的向往。远方,始终是一个充满魅力和诱惑的巨大泉眼,汩汩涌流出诗意和美。波德莱尔的女友有着一半非洲血统,据说正是她周身所散发出的异域气息令他痴迷,她的秀发,她的一颦一笑,都让他恍惚感受到了遥远的、另外一个大陆的奇异魅力。他有一首散文诗《头发中的半球》,其中有这样的描绘:‎ 你的头发蕴藏着一个完整的梦,充满了船帆和桅杆的梦;它也包藏着大海,海上的季风把我带到那些迷人的地方,那里的太阳显得更蓝更深,那里的大气充满果实、树叶和人类肌肤的香味。‎ ‎③从这些文字中,你能强烈地感觉到诗人感受力的灵敏和丰盈,视觉、嗅觉等都在全方位地、酣畅地敞开着,借助于一些要素,他生动地描绘出遥远地方的风光气氛,栩栩如生。而这一幅幅巨大的、声色流溢的画面,最终是靠着强大的想象力来加以拼接、连缀和粘合的。‎ ‎④终其一生,波德莱尔都被港口、轮船、铁路、火车以及酒店客房所吸引,因为这些都连接着远方,通向另外的生活。因为很难真正具备出行的条件,波德莱尔更多的是从想象中获得满足。‎ ‎⑤获得诺贝尔文学奖的俄罗斯作家蒲宁,也是一位善于运用想象力的大师。在自传体长篇小说《阿尔谢尼耶夫的一生》中,他回忆了自己在俄罗斯腹地的一个庄园里度过的童年时代。在漫长寒冷的冬夜,《鲁滨逊漂流记》等书里的插图,让他想象遥远的热带。狭窄的独木船、拿着弓箭和长矛的光身子的人、椰子树林,都让他感到甜蜜和陶醉,产生了一种身临其境的幻觉:“我不但看到,而且以自己的整个身子感觉到了那么多干燥的炎热,那么多阳光!”以至于多年后,他有机会来到那些地方时,心中浮现的第一感觉就是:对,对,所有这一切正如我三十年前首次“看到”的那样!‎ ‎⑥拥有这样一种强大的想象能力,堪称是生命中获得的宝贵奖赏。它打通了一条连接诗和美的道路。‎ ‎⑦以上种种都表明,一个善感的灵魂,可以创造出怎样的奇迹。这是一些具有异禀的人,能够通过一棵树想象一片森林,借助一片贝壳想象一片大海。一些零散寒碜的线头布片,到了他们手中,竟被拼接成一幅色彩斑斓的织锦。读这样的作品,与其说是观赏作者借助于想象而描绘出的风景,不如说是欣赏灵魂的奇观。这样的灵魂正是艺术的摇篮和息壤。‎ ‎⑧当然,我们都是凡夫俗子,不具备那样卓越的才华。不过,从他们的这种嗜好中,还是可以悟出一些有益的东西。虽然如今旅行成本大大降低,但一个人的时间、精力、财力等,永远是处于一种短缺的状态。相对去过的地方而言,更多的地方是去不成的,这样,就不妨退而求其次,借助想象的力量来作为一种弥补。‎ ‎⑨在这个意义上,我们有必要向那些杰出作家学习,培养和丰富自己的想象力,努力使自己变得细腻善感:欣赏一泓碧蓝的山涧溪水的图片,仿佛感觉到浸入脚底的丝丝寒凉;目光流连于画面上一间江南小城临水的茶楼,似乎嗅到一缕明前龙井的清香。对于气氛、情调的细腻感知和把握,才堪称旅行最重要的收获。‎ ‎⑩如今技术的快速进步,为这种想象的旅行提供了极好的帮助,鼠标轻轻一点,你可以从白雪皑皑的北极冰原,到花木葳蕤的热带海岛。瞩目于图片,充分调动想象力,把感受的旋钮调到最高档,庶几可以获得几分真切的、身临其境般的体验。‎ ‎⑪当然,对于这种替代的旅行,你尽可以不以为然,但我只需要用一句话来辩护:人生奄忽,步履真正踏及的地方,能有几处?‎ ‎(本文有删改)‎ ‎1.下列对文章内容的分析和概括,正确的两项是(  )‎ A.作者认为文人多与贫穷相伴,这使得他们对生活有着更加深刻的体验,从而拥有了敏锐的感受力和丰富的想象力,蒲宁就是明证。‎ B.文章运用古今对照的手法,从文学写作延伸到普通人的日常生活,说明以丰富的心灵展开对未知的想象是普通人培养审美化的生活态度的必要手段。‎ C.“头脑中的旅行”‎ 不是才华横溢的作家的专利,普通人通过自身的努力,变得细腻善感,同样可以在这种替代性的旅行里获得身临其境般的体验。‎ D.第⑨段中“目光流连于画面上一间江南小城临水的茶楼,似乎嗅到一缕明前龙井的清香”一句,运用了比拟的修辞手法,从视觉与嗅觉的角度展开联想。‎ E.今天,科技的发达既使现实的旅行更加便利,也为想象的旅行提供了帮助,瞩目于网上的各种图片,在头脑中旅行,我们可以“抵达”世界的各个角落。‎ 答案 CE 解析 A项,“从而拥有了……想象力”错误,强加因果。B项,“以丰富的心灵展开对未知的想象是普通人培养审美化的生活态度的必要手段”于文无据,文中也未使用“古今对照的手法”。D项,“比拟”错误,应是通感,“目光流连……茶楼”却似乎“嗅到……清香”,用嗅觉来写视觉。‎ ‎2.文章第②段引用波德莱尔散文诗《头发中的半球》的片段,有何作用?请简要分析。‎ 答:___________________________________________________‎ ‎_______________________________________________________‎ 答案 ①以具体例子说明想象的旅行打开了诗人通向远方的道路,诗人虽然不能亲临其境,却能从丰富的想象中获得满足。②生动地表现了诗人的才华——即使身不能至,也能够以超凡的想象力和敏锐的感受力生动传神地描绘出远方的风景,呈现出富有诗意、流光溢彩的画面,给人以强烈的感染。‎ 解析 本题是考查句段和引用的作用,其中应更侧重于引用,因为题干中问的是“引用波德莱尔散文诗《头发中的半球》的片段”,而不是直接说第②段波德莱尔散文诗《头发中的半球》的片段,可见这里是一个暗示。结合文本可总结两点,一是想的作用,二是表现了诗人的才华。然后再分析说明即可。‎ ‎3.作者为什么说“读这样的作品,与其说是观赏作者借助于想象而描绘出的风景,不如说是欣赏灵魂的奇观”?请简要分析。‎ 答:___________________________________________________‎ ‎_______________________________________________________‎ 答案 ①因为借助想象描绘出的风景并不完全是现实的复现,而是作家的艺术创造,其中蕴含了作家连通诗和美的生命感受。②作家之所以能够创造出这样的艺术世界,是源于其善感的灵魂、丰富的心灵。杰出作家想象中的旅行实乃心灵的探寻,他们描绘的动人风景,映现的正是作家心灵世界的奇景。‎ 解析 本题既考查学生理解句子的能力,又考查学生筛选信息和概括信息的能力。首先要分析题干中的句子,理解其含意;其次再回归文本,找到这句话;最后在上下文按照由近及远的方式搜索,联系上下文分析句子的内涵。‎ ‎4.与现实中的旅行相比,“头脑中的旅行”是一种替代性的旅行,它可以满足人们对远方的向往吗?请结合文章内容和个人生活体验,谈谈你的看法。‎ 答:___________________________________________________‎ ‎_______________________________________________________‎ 答案 (示例一)头脑中的旅行足以满足我们对远方的向往。①现实生活中的各种束缚不可避免,我们未必有条件、有能力踏遍万水千山;②头脑中的旅行能超越现实旅行的局限,我们凭借丰富的想象力和敏锐的感受力可以领会旅行的真正精神,领略远方的精彩;③我们向往远方,实质上是为了摆脱平庸的现实,获得别样的生活体验,精神的漫游可以拓展生活中的诗意空间,令平凡的现实生活变得丰富多彩。‎ ‎(示例二)现实中的旅行与头脑中的旅行各有优势,互相不可替代,二者互补,能够更好地满足我们对远方的向往。①百闻不如一见,条件具备时,我们不妨踏上旅途,去欣赏远方的自然风光与人文景观;②如果不具备实地旅行的条件,大可借助想象,在心灵世界中构建“第二自然”,感受远方的魅力;③现实有限而想象无垠,头脑中的旅行能够使我们超越现实,突破束缚。真实与想象交融,头脑中的旅行就能为眼前的风景增色添彩,令现实的旅行更加富有意趣,从而更好地满足我们对远方的期待和向往,同时也使我们的想象力得到丰富和提升。‎ ‎(示例三)头脑中的旅行无法满足人们对远方的向往。①现实中的旅行能够把我们带到真实的远方,在现场获得直接体验,而头脑中的旅行毕竟是一种替代性的旅行,想象出的远方再美好也不真实,不能提供我们关于远方的准确的认识。②‎ 一些细腻微妙的感受与体验只有身临其境才能获得,不可能通过头脑中的旅行得到。远方风景里动人的韵致,不亲眼目睹,不亲身感受,就永远无法真切地体会到。远方,如果只能想象,不能亲至,将是巨大的遗憾。③头脑中的旅行能否满足对远方的向往,与人的感受力和想象力密切相关。如果一个人没有善感的心灵,缺乏想象力,就无法想象充满魅力的远方,也就不能满足他对远方的向往。‎ 解析 此题考查学生对作品进行个性化阅读和有创意的解读的能力。答题时要注意审题,一审题干出题的方向,二审我们需要答什么,三审答题的模式。本题为开放性题目,可以结合文本与个人生活体验自由发挥,但要逻辑严密,有理有据。‎ 三、[2015·四川高考]阅读下面的文章,完成1~4题。‎ 太湖碎锦 范烟桥 ‎①太湖,用文人的套语来形容,是“三万六千顷、七十二峰”。民间则说“八百里太湖跨三州”。不经过实测,这样笼统地画出一个轮廓,只能给人们一种山明水秀、浩瀚无际的想象。至于它有什么诗情画意,要费一点时间实地去观察、探索,才能领会。‎ ‎②我从不同的角度,看太湖的部分画面,就感到有不同的胜概。洞庭东山、西山是太湖里两个主峰。东山周围五十余里,山势并不陡峭,土壤又滋润,经劳动人民世世代代辛苦经营,已成了丰产地区。山下坡田,种植各种水稻,是秋熟的主要农作物。夏熟是三麦和油菜,还有豆类和蔬菜瓜果。他们更有园艺的丰富经验,梅、杏、桃、李……多得数说不尽。枇杷、杨梅和洞庭红(橘名)名闻远近。随着春夏秋冬,它们先后开花结果,春天果然是“姹紫嫣红开遍”,夏天、秋天、冬天,也是各有烂漫绚丽的景色。说是“美尽东南”,并不夸张。从观赏说,四时皆宜;从生产说,那就是取之不尽、用之不竭的天然资源。江南的许多淡水鱼,这里样样都有。朝出暮归的千百艘大小渔船,点缀湖光水色中,渔民们勤劳、勇敢,征服自然,利用自然。‎ ‎③西山和东山隔着东太湖,东山最高峰——莫釐,和西山最高峰——缥缈遥遥相对,同为七十二峰的领袖。西山也是丰产地区,同是“花果山”,东山所有的名花嘉果,西山都繁生着。从东山坐独具风格的小艇——龙飞快,驶入东太湖,莫釐峰头,云气滃然如蒸。别的不知名的远近诸山,时隐时现,好似给烟波吞吐着,山色因明暗而浓淡不一。船家果然有眼明手快的本领,坐在船里的我,到湖心时常为颠簸震荡而惊心动魄。正因为如此,而愈觉山水奇丽得来不易的乐趣。兀立在东山、西山之间的石公山,则是以玲珑秀逸的姿态吸引着人们。小艇乘风破浪而去,到了山下,显然可见四围的山石,经过千万年的冲刷,有了“皱、瘦、透”的美姿,早给鉴赏者陆续凿去了,苏州园林里的太湖石,都是取于石公一带的石山。因此,石公山像斧削过,没有了山脚,正如一块翡翠放在一个玻璃盘里。‎ ‎④假使从苏州直接到西山,出蠡口,就展开了图画,山更多,湖更大,变幻就更多。王鏊“山与人相见,天将水共浮”,冯善“震泽春浮涨碧漪,净涵天影漾玻璃”,能把湖山之胜,描绘得恰到好处。道书上所说的第九洞天——林屋,就在西山。到了里面,石壁嶙峋如雕塑,是洞庭一奇。这里有许多神话,和山农们闲谈,妄言妄听,也增添了些兴趣。而西边的消夏湾,更附会着西施的种种传说。山湾柔顺的湖水,浅而澄清,可以游泳。有着荷花、菱叶,清风徐来,颇有凉意,确是夏天避暑的好去处。到了包山寺,才窥见缥缈峰突起在丛林杂树之上。近观不如远眺之美,大凡山水之胜,都有这个境界。有了山,有了水,才见得山的灵秀,水的空明。太湖就以此特饶奇胜。‎ ‎⑤太湖还有四个画面,和洞庭东山、西山合起来,差不多得见其全貌。一是从湖州到无锡的一段水程,在群山断续中经过,前后左右可以看到云峦起伏,似乎它们都有动态,与人游戏。一是从无锡到宜兴,数十分钟的汽车行程,在湖边掠过,太湖平铺在车外,远山几抹,可望而不可即。一是无锡的鼋头渚,割取了太湖的一角,经过人力的整理,有着怪石突兀、惊涛汹涌的奇趣。不仅有色,而且有声。夕阳将下,余晖照映湖面,金光璀璨,不可名状。一是苏州光福的石壁,也是太湖的一角,更见得静止处,已不是空阔浩渺的光景。而即小见大,可以使人有更多的推想。‎ ‎⑥阴、晴、风、雨、云、雾,固然使山水多变,适逢其会,逸趣横生。便是朝曦、夜月下特有的湖光山色,也是可遇而不可求的。古今诗人画师,尽管灵思妙想,摄取片断到诗画里,有着他们的杰作,还是概括提炼。我更无能,凭我接触到的,写了些体味,或许有三言两语,能引起到过太湖者的同情,作会心的微笑。毕竟是“尝鼎一脔”,太湖实在是描写不尽,描写难工的。‎ ‎(有删改)‎ ‎1.下列对文章有关内容的理解和赏析,不正确的两项是(  )‎ A.作者对太湖东山、西山及太湖在阴、晴、风、雨、云、雾下的变幻作了详细的描绘,表现了太湖山水的多变与逸趣横生。‎ B.文章引用形容太湖的套语、诗句,谈及苏州园林、道书记载和神话传说,增添了太湖的人文色彩和传奇性。‎ C.文章用“一块翡翠放在一个玻璃盘里”,比喻四围山石被削取的石公山兀立湖面这一湖山相映的景致,极为生动形象。‎ D.第⑤段写太湖的四幅画面,都运用想象和移步换景的方法,结合色彩、声音等变化,表现不同时空之下的太湖奇景。‎ E.本文写太湖美景,详略有致,既有整体勾勒,又有局部描写,自然景色与审美体验有机融合,韵味丰富。‎ 答案 AD 解析 A项,文中并没有对“太湖在阴、晴、风、雨、云、雾下的变幻”进行详细的描绘。D项,“都运用想象和移步换景的方法”错误。‎ ‎2.文章第②段写了东山一带哪些“胜概”?表达了作者怎样的思想感情?请简要概括。‎ 答:___________________________________________________‎ ‎_______________________________________________________‎ 答案 ①文章第②段写了三方面“胜概”:自然之美,物产之丰,劳作之美。②表达了作者对祖国大好河山的热爱和赞美之情,歌颂了劳动人民的勤劳、勇敢,认识到劳动创造美和人民群众改造自然的伟大力量。‎ 解析 回答第一问,应聚焦第②段。“胜概”即美景、美好的境界,“哪些”暗示考生应分类概括。第②段中有两个关键句:“从观赏说,四时皆宜;从生产说,那就是取之不尽、用之不竭的天然资源”“渔民们勤劳、勇敢,征服自然,利用自然”。抓住这些具有概括性、提示性内容的语句,结合上下文分析,即可概括出“胜概”中的三美。回答第二问,应聚焦作者“情感”。既然“胜概”呈现,则爱此景、赞此景之情自然流出,喜爱、礼赞之情兼具,认知、崇尚之情并生。‎ ‎3.结合全文,分析标题“太湖碎锦”的内涵和作用。‎ 答:___________________________________________________‎ ‎_______________________________________________________‎ 答案 ①“碎锦”用比喻手法,形象生动地表现了太湖景色的丰富多彩,表达了作者对太湖的喜爱和赞美。②具有统摄全篇和联结全文的作用。因太湖辽阔,作者只选取有代表性的几处景色来描绘,并通过这些“碎锦”的组合,呈现太湖的全貌。③作者以“碎锦”表明其文只能算是对太湖美景的部分呈现,既表现了他的谦虚,也暗含了对描写难尽的太湖美景的赞赏。‎ 解析 一般说来,标题的作用可从内容主题、文章结构、表达技巧等方面考虑。就本文来说,标题“太湖碎锦”有修辞的运用,要指出修辞手法及表达效果;再结合文意分析,可知这一标题还有统领全篇、勾连结构、凸显情感等作用。如此把握,答案便会更为周全。‎ ‎4.文中说:“近观不如远眺之美,大凡山水之胜,都有这个境界。”你是否赞同这个说法?结合本文和生活实际,谈谈你的思考。‎ 答:___________________________________________________‎ 答案 (示例一)赞同。近观和远眺是两种不同的审美视角。远眺比近观更能领略自然景物的整体特征和独特风貌。如文中东山莫釐峰“云气滃然如蒸”的迷蒙奇幻之美,正为远眺所得。现实生活也是如此,近观事物,固然能体验到细节之美,但很难统观全局之胜。因此,只有立足高远,面向未来,才能凭高视远,达到人生更高境界。‎ ‎(示例二)不赞同。远眺只能让人获得事物的概貌,近观才能让人发现和体察细微之美。本文作者若仅远眺而不近观,就无法领略石公山“像斧削过,没有了山脚”的奇景。现实生活也是如此,要想领悟到近处的美与人生的当下意趣,就要立足眼前风景和现实人生。‎ 解析 探究题有意蕴探究、体验探究、手法探究、取舍探究之分,本题属于取舍类探究题目。考生应在整体把握文本的基础上,立足文本,联系实际,发表见解。第一步,务必表明观点:“赞同”或“不赞同”‎ ‎。第二步,结合文本和生活实际,写出思考感悟,若赞同“近观不如远眺之美”之说,可联系文本、现实加以解说;若赞同“远眺不如近观之美”之说,也应找到充分理由进行说明。只要言之成理即可。‎ ‎[2年全国模拟重组]‎ 一、[2017·四川模拟]阅读下面文字,完成1~3题。‎ 滕王阁之殇 程维 如果以青云谱为视角,在南昌从文化意义上能与之相对的只有滕王阁。从地标意义上说,滕王阁之于南昌,相当于埃菲尔铁塔之于巴黎。‎ 为什么?谁都知道现在的滕王阁是一堆钢筋水泥,是一个空壳,然而它的存在价值却在于它具有使世人对已往文化的发生或流逝产生某种追忆的价值。也就是说它尚能帮助人们在想象中完成对往昔文化的重返与凭吊,从而令它行使着不仅是纪念碑的意义。‎ 当然没有必要将南昌家门口的滕王阁,与远在法国巴黎的埃菲尔铁塔相提并论,其实没有可比性。埃菲尔铁塔(法语:La Tour Eiffel)是一座1889年建成的位于巴黎战神广场上的镂空结构铁塔,高300米,天线高24米,总高324米。埃菲尔铁塔得名于设计它的桥梁工程师古斯塔夫·埃菲尔。铁塔设计新颖独特,是世界建筑史上的杰作,因而它是巴黎的一个重要景点和突出标志。‎ 滕王阁是中国农耕文明相对发达时期的产物,始建于唐永徽四年(公元653年),为唐高祖李渊之子李元婴任洪州都督时所创建。李元婴出生于帝王之家,受到宫廷生活熏陶。“工书画,妙音律,喜蝴蝶,选芳渚游,乘青雀舸,极亭榭歌舞之盛。”(明陈文烛《重修滕王阁记》)据史书记载,永徽三年(公元652年),李元婴迁苏州刺史,调任洪州都督时,从苏州带来一班歌舞乐伎,终日在都督府里盛宴歌舞。后来又临江建此楼阁为别居,实乃歌舞之地。因李元婴在贞观年间曾被封于山东省滕州,故为滕王,且于滕州筑一阁楼名“滕王阁”,后滕王李元婴调任江南洪州,又筑豪阁,仍冠名“滕王阁”,此阁便是后人所熟知的滕王阁。滕王阁因“初唐四杰”之首王勃的一篇骈文《秋日登洪府滕王阁饯别序》(简称《滕王阁序》)而得以名贯古今,誉满天下。历史上的滕王阁先后重建共达29次,屡毁屡建,今日之滕王阁为1989年重建。‎ 滕王阁似乎有一种超强的叙事功能,无论阁存阁毁,它仍然在叙事,在水天相接的空白里虚构着忧郁的辉煌。如果恰巧有孤鹜划过,那或许是神的笔在书写无字之书。‎ 也就是说,南昌滕王阁的历史有1300多年,巴黎埃菲尔铁塔的历史还不如它的零头。但是现存的滕王阁建于1989年,仅仅存世21年,勉勉强强也只相当于埃菲尔铁塔的另一个零头。所以如果他们之间按存世的年头来形成一种对话,将是无比奇妙,且充满玩味的。谁都可以高谁一头,谁也都比对方矮一截,里面充满反讽与悖论。然而,它们确确实实都是名胜,纵向看滕王阁远比埃菲尔铁塔的时间要长,横向看必须承认埃菲尔铁塔在全世界的名声远比滕王阁广,其游人来自世界各地,他们去巴黎的理由之一就是登埃菲尔铁塔,这是必须身体力行的,除此,他们与该塔不会发生任何关系。因此,人们会把埃菲尔铁塔当作幽会的情人。而滕王阁则不能,其历史虽久,但远没有世界闻名,不太可能会令远在非洲的人也想来南昌登滕王阁。其知名范围大都限在华人以内,前提是他们读过唐人王勃的《滕王阁序》。‎ 从中不难看出中国传统文化输出能力存在的普遍性缺陷。而且,纵然有人慕名来了,他们面对的是一座落成才只21年的钢筋水泥仿古建筑,还是滕王阁1300多年的历史?反之埃菲尔铁塔只向人们提供一次性消费的游历经验,其存世价值便可宣告完成。‎ ‎(有删改)‎ ‎1.下列对作品有关内容和艺术特色的鉴赏,最恰当的一项是(  )‎ A.作者认为南昌滕王阁与巴黎埃菲尔铁塔相比,除建筑风格外,其他都没有可比性。‎ B.埃菲尔铁塔得名于设计它的桥梁工程师古斯塔夫·埃菲尔,所以全世界的人慕名而来,它是法国的一个重要景点和标志。‎ C.历史上的滕王阁先后重建共达29次,屡毁屡建,今日之滕王阁为1989年重建,因此现在的滕王阁其实并非1300年前的建筑。‎ D.滕王阁有一种超强的叙事功能,即使被毁坏,它也仍然在叙事,在水天相接的空白里虚构着忧郁的辉煌。‎ 答案 C 解析 A项,“除建筑风格外”于文无据;B项,全世界的人慕名而来的原因,并没有埃菲尔铁塔的命名这一项;D项,“滕王阁有一种超强的叙事功能”错,原文是说“滕王阁似乎有一种超强的叙事功能”。‎ ‎2.作者认为滕王阁的文化意义是什么?‎ 答:___________________________________________________‎ ‎_______________________________________________________‎ ‎_______________________________________________________‎ 答案 滕王阁现在虽只是一个空壳,但具有使世人对已往文化的发生或流逝产生某种追忆的价值;帮助人们在想象中完成对往昔文化的重返与凭吊。这使它行使着不仅是纪念碑的意义。‎ 解析 本题考查对文章内容理解的能力。本题的有效区间主要集中在第二段,根据第二段的内容进行概括,就可以得出两个答案要点。‎ ‎3.作者认为滕王阁和埃菲尔铁塔相比有什么缺陷?你认为中国传统文化应该如何进行输出?请简要分析。‎ 答:___________________________________________________‎ ‎_______________________________________________________‎ ‎_______________________________________________________‎ 答案 滕王阁虽然比埃菲尔铁塔历史长,但是知名度却比不上埃菲尔铁塔,因为埃菲尔铁塔能够吸引全世界的游客慕名而来。可以对中国传统文化进行广泛宣传;还可以学习、借鉴国外好的方式。‎ 解析 本题考查对文章内容的理解和对问题探究的能力。第一个问题可以从文本中找出相关内容,根据文章倒数第二段的内容进行概括,即可得出埃菲尔铁塔的优势在于其远播世界的声名。而中国传统文化的输出问题,具有一定的开放性,言之成理即可。‎ 二、[2016·衡水检测]阅读下面的文字,完成1~3题。‎ 泸沽湖 ‎①“泸沽湖有多么美?”人们总爱这么问。‎ ‎②只能这么说:面对泸沽湖,任何一个傻瓜用一种傻瓜相机,顺手这么一按——行了,这张照片准能参加全国影展,说不定还能得奖!‎ ‎③这就是泸沽湖。‎ ‎④“神仙姿态”“天上人间”这一类的华丽辞藻你都可以给它,它当之无愧也不稀罕。像所有遗世独立的无名风景区一样,它们的美是孤傲的,是拒绝世俗的探访的,因为它们的美原本是由于世俗社会的遗忘和冷漠而造成的。九寨沟也好,张家界也好,喀纳斯湖也好,现在都著名了。泸沽湖也会著名的,但是,随着著名而来的是什么呢?有时候真希望制定一个参观这类风景区的法,规定只有高教养的人才有资格参观。不然,这类风景区很快会荡然无存。‎ ‎⑤泸沽湖是摩梭人的伊甸园。‎ ‎⑥它是人类遥远的母系社会时期遗失在今天这个世界上的一粒最后的种子,一册字迹不清的孤本,一部有关远古先民们的电影,一滴保存到了本世纪末的单纯水珠儿。‎ ‎⑦我不想多说自然,自然风光如若没有人和其他一些生命,便是浑然无觉的。开辟鸿蒙,盘古创造了世界。作为先祖和先知,他临终看到的是一个生机无限的世界。但是盘古没有料到,仅仅只过了3000年,人口已经膨胀得使地球的自转减速,也使整个地球变得乖僻起来。‎ ‎⑧他留给我们的那些伟大的本钱,已经快花光了。‎ ‎⑨而泸沽湖,恰是当初不小心遗落在某个角落的一枚。它还存在,是因为人们在忙乱中没能注意到它,就像一个人偶然从过去的裤子口袋里翻出了一枚钱币。‎ ‎⑩咦?他看着它,奇怪而又欣喜。‎ ‎⑪他会用它赶快去酒馆再喝一杯酒吗?‎ ‎⑫没准儿。‎ ‎⑬但是摩梭人不会花它。摩梭人也许没有经济头脑,但绝对有生命意识。他们视这枚遗落的金币为自己的族徽,为自己生存的依据;他们虽然没有导弹而只有弓弩,却学会了和谐地与泸沽湖共存。‎ ‎⑭摩梭人!‎ ‎⑮当夜晚来临,当清澈的湖水轻轻舔着沙滩,款款涌向村落;当村落较为空旷的一角点燃了篝火,火光和笛的声音首先召唤了青年男女,然后是小孩,然后是老人……‎ ‎⑯舞蹈就自然而然地跳起来了,摩梭人!‎ ‎⑰男子多为高大健美的,穿着类似藏民的袍子,戴着硬边的礼帽,有些像牛仔。女子多是美丽轻盈的,穿了镶边的长裙,像一排舞蹈演员似的整齐。‎ ‎⑱男子和女子很自然地拉起手来,臂挽着臂;他们很自然,看起来心理活动单纯而健康。没有什么男女授受不亲的影子,也没有理论上赞美劳伦斯实际上受困于朱熹的矛盾。他们非常坦然,因坦然而纯洁无邪;我们却有些窘迫,因窘迫而证明我们想得太多。这不太好,可是我们正是因为念头复杂才进化成“文明人”的呀!这时候,我们或多或少会发现自己回不去了,想使自己单纯已经十分困难,甚至是不可能了。‎ ‎⑲但是摩梭人舞蹈起来了!‎ ‎⑳一支短笛引领着的舞蹈,高大的青年男子在前面,姑娘们拖曳着白裙随着舞步整齐地抖动;男子和女子挽着手臂有节奏地摇起又落下;男子的腿长而有力,踏跺和旋扭如同一群骏马的腿不停地运动着,节奏强烈,充满着粗犷单纯的力度;女子的面部表情是一种平静的微笑,是含蓄的陶醉,她们间歇着唱歌和呼喊;所有的女子都天生拥有金属般的嗓音和唱歌的天才,这一切都在夜色的掩护和篝火的照耀下,得以凸现,显得神秘,显得光彩迷人!‎ 围绕着火又被水围绕,水和火这一对不相容的美丽的矛盾,就这样和谐地共存了。‎ 至于“歌星”这样的词,我想会使摩梭人惊讶不解的。在他们看来,每一个人都会唱歌,就像每一只百灵鸟都会唱歌一样自然。另外,我们现在使用得很多的名词和概念,我想也会令他们费解。譬如“离婚”这个词吧,我猜他们会这么问:“就是他和她的阿助(摩梭人语,朋友。)不好了吧?”‎ “对,是这么回事儿。”‎ “那她的阿助就不会再来她家了。”‎ “这……一言难尽!”‎ 究竟是我们文明呢还是他们文明?我们进步了3000年怎么还没走到人类的起点呢?摩梭人可能什么主义都没有,什么古典主义,什么浪漫主义,什么超现实主义,他们都不懂。但是都比我们像一些,唯独我们,正搁浅在昨天与明天的礁区,拼命挣扎。‎ 摩梭人,小心啊!‎ 泸沽湖——最后的伊甸园。‎ 这湖畔派兼高蹈派的舞蹈就这么跳着,精力充沛不狂不躁地舞蹈着,欢唱着,直到深夜。‎ 连续好多个夜晚,我的耳朵里总是升起那短笛的声音,若隐若现;从很远的地方飘来,在很近的地方响着。‎ ‎(有删改)‎ ‎1.下列对作品有关内容和艺术特色的鉴赏,最恰当的一项是(  )‎ A.文章开篇描写泸沽湖的自然美,只需一个傻瓜用一种傻瓜相机顺手这么一按,照片准能参加全国影展,有吸引读者急于阅读下文的作用。‎ B.所有遗世独立的无名风景区一旦声名鹊起,世俗社会的探访,很快会使风景区荡然无存。泸沽湖被人发现后,她的美将会消失。‎ C.第⑥段用排比兼夸张的修辞手法,充分地表现了泸沽湖的弥足珍贵。‎ D.第⑨~⑫段作者用一个形象的比喻,再现了世人发现泸沽湖后的奇怪而欣喜的心态,也表现出作者隐隐的担忧——泸沽湖会被人糟蹋掉。‎ 答案 D 解析 A项,文章开篇虚写泸沽湖的自然美,不是实写。B项,“泸沽湖被人发现后,她的美将会消失”‎ 说法欠妥。文意应为缺乏教养的探访者将有可能破坏泸沽湖的生态,从而失去她的美;C项,“排比兼夸张的修辞手法”应是“排比兼比喻的修辞手法”。‎ ‎2.第⑦段首句“我不想多说自然,自然风光如若没有人和其他一些生命,便是浑然无觉的”在文中有着重要的作用,请作简要分析。‎ 答:___________________________________________________‎ ‎_______________________________________________________‎ ‎_______________________________________________________‎ 答案 在结构上承上启下:收住上文所写泸沽湖自然之美的笔墨,转入对泸沽湖人的活动的叙写,使文章文脉连贯,展开自然。‎ 解析 本题考查分析文章结构的能力。本题设问第⑦段首句在文中的重要作用,我们可以从结构、内容(情感)、主题等方面入手分析。通常来说,处于文章中间的句子,往往有过渡(或承上启下)、为下文某些内容作铺垫、与前文某些内容相呼应等作用,具体要结合原文进行思考分析。另外,内容方面,我们要指出该句本身写出了什么、强调了什么内容等。主题方面的作用,常常出现在一些处于关键位置(如开头、结尾)的句子上,如点明题旨、强化主题等,作答时注意要点出“什么主题”。‎ ‎3.作者认为“泸沽湖是摩梭人的伊甸园”,文中哪些地方体现了这一点?请简要分析。‎ 答:___________________________________________________‎ ‎_______________________________________________________‎ ‎_______________________________________________________‎ 答案 作者从自然风光和人的活动两个方面表现了泸沽湖是摩梭人的伊甸园。先写其纯洁的自然风光:“任何一个傻瓜用一种傻瓜相机,顺手这么一按”,“照片准能参加全国影展”;再将泸沽湖与九寨沟、张家界、喀纳斯湖等著名风景区相提并论,凸显其自然之美。再从摩梭人的人情美的角度去表现:珍爱家园泸沽湖,学会了和谐地与泸沽湖共存(人与自然和谐相处);人与人和谐相处(青年男女纯情相处,男女老少雍雍穆穆);能歌善舞;自然(真正自由)的婚姻方式等。‎ 解析 本题考查筛选并整合文中信息的能力。题干中所说的“泸沽湖是摩梭人的伊甸园”,实际上可以视为本文的“文眼”,全文内容就是围绕这句话展开的。审题要深入思考,题目设问“文中哪些地方体现了这一点”,实际上问的就是“文章从哪几个方面来体现这一点的”。抓住“伊甸园”这个关键词,细致梳理作者的思路,可以发现,以第⑦段为过渡段,文章前后大体上分别写了泸沽湖的自然风景和人文风情。这样梳理清楚了,答案就不难得出。‎ 三、[2017·四川联考]阅读下面的文章,完成1~3题。‎ 紫茉莉 林清玄 ‎①我对那些接着时序变换姿势,或者是在时间的转移中定时开合,或者受到外力触动而立即反应的植物,总是保持着好奇和喜悦的心情。‎ ‎②那种在园子里的向日葵或是乡间小道边的太阳花,是什么力量让它们随着太阳转动呢?难道只是对光线的一种敏感?像平铺在水池的睡莲,白天它摆出了最优美的姿势,为何在夜晚偏偏睡成一个害羞的球状?而昙花正好和睡莲相反,它总是要等到夜深人静的时候,才张开笑颜,放出芬芳。夜来香、桂花、七里香,总是愈黑夜之际愈能品味它们的幽香。还有含羞草和捕虫草,它们一受到摇动,就像一个含羞的姑娘默默地颔首。还有冬虫夏草,明明冬天是一只虫,夏天却又变成一株草。‎ ‎③在生物书里我们都能找到解释这些植物变异的一个经过实验的理由,这些理由对我却都是不足的。我相信在冥冥中,一定有一些精神层面是我们无法找到的,在精神层面中说不定这些植物都有一颗看不见的心。能够改变姿势和容颜的植物,和我关系最密切的是紫茉莉花。‎ ‎④我童年的家后面有一大片未经人工垦殖的土地,经常开着美丽的花朵,有幸运草的黄色或红色小花,有银合欢黄或白的圆形花,有各种颜色的牵牛花,秋天一到,还开满了随风摇曳的芦苇花……就在这些各种形色的花朵中,到处都夹生着紫色的小茉莉花。‎ ‎⑤紫茉莉是乡间最平凡的野花,它们整片整片地丛生着,貌不惊人,在万绿中却别有一番姿色。在乡间,紫茉莉的名字是“煮饭花”‎ ‎,因为它在有露珠的早晨,或者白日中天的正午,或者是星满天空的黑夜都紧紧闭着;只有一段短短的时间开放,就是在黄昏夕阳将下的时候,农家结束了一天的劳作,炊烟袅袅升起的时候,才像突然舒解了满怀心事,快乐地开放出来。每一个农家妇女都在这个时间下厨做饭,所以它被称为“煮饭花”。‎ ‎⑥这种一两年或多年生的草本植物,生命力非常强盛,繁殖力特强,如果在野地里种一株紫茉莉,隔一年,满地都是紫茉莉花了;它的花期也很长,从春天开始一直开到秋天,因此一株紫茉莉一年可以开多少花,是任何人都数不清的。‎ ‎⑦最可惜的是,它一天只在黄昏时候盛开,但这也是它最令人喜爱的地方。曾有植物学家称它是“农业社会的计时器”,当它开放之际,乡下的孩子都知道,夕阳将要下山,天边将会飞来满空的红霞。‎ ‎⑧我幼年的时候,时常和兄弟们在屋后的荒地上玩耍,当我们看到紫茉莉一开,就知道回家吃晚饭的时间到了。母亲让我们到外面玩耍,也时常叮咛:“看到煮饭花盛开,就要回家了。”我们遵守着母亲的话,经常每天看到紫茉莉开花才踩着夕阳下的小路回家。巧的是,我们回到家,天就黑了。‎ ‎⑨从小,我就有点痴,弄不懂紫茉莉为什么一定要选在黄昏开,多次坐着看满地含苞待放的紫茉莉,看它如何慢慢地撑开花瓣,出来看夕阳的景色。问过母亲,她说:“煮饭花是一个好玩的孩子,玩到黑夜迷了路变成的,它要告诉你们这些野孩子,不要玩到天黑才回家。”‎ ‎⑩母亲的话很美,但是我不信,我总认为紫茉莉一定和人一样是喜欢好景的,在人世间又有什么比黄昏的景色更好呢?因此它选择了黄昏。‎ ‎⑪紫茉莉是我童年里很重要的一种花卉,因此我在花盆里种了一棵。它长得很好,可惜在都市里,它恐怕因为看不见田野上黄昏的好景,几乎整日都开放着。在我盆里的紫茉莉可能经过市声的无情洗礼,已经忘记了它祖先对黄昏彩霞最好的选择了。‎ ‎⑫我每天看到自己种植的紫茉莉,都悲哀地想着,不仅是都市的人们容易遗失自己的心,连植物的心也在不知不觉中迷失了。‎ ‎——一九八二年九月八日 ‎(有改动)‎ ‎1.下列对作品有关内容和艺术特色的鉴赏,最恰当的一项是(  )‎ A.作者两次写到母亲提醒“我”看到煮饭花开就回家这一内容,从侧面突出紫茉莉生命力旺盛的特点,也照应了上文紫茉莉得名“煮饭花”的由来。‎ B.第④段写出了“我”家后院那片土地花朵常开的美景,这些内容与第⑩段紫茉莉选择在黄昏开放形成了呼应。‎ C.童年记忆中的紫茉莉是美妙的,但在都市里的紫茉莉,可能是经过了城市无情的洗礼,几乎整日地开放着。‎ D.本文围绕黄昏开放的紫茉莉展开,旨在表达对童年快乐生活的怀念和对在艰难的乡村环境中母爱的赞美。‎ 答案 C 解析 A项,选定内容从侧面突出紫茉莉只在黄昏时开放的特点;B项,上下文不能形成呼应;D项,对文章主旨的理解不当,主旨应该是我们要善于思考生活,保持自己本色和本真,活出人类应该有的尊严。‎ ‎2.作者在第③段中说“和我关系最密切的是紫茉莉花”,作者和紫茉莉之间有什么样的密切关系?请根据文意概括作答。‎ 答:___________________________________________________‎ ‎_______________________________________________________‎ ‎_______________________________________________________‎ 答案 ①“我”童年时生活的乡村环境有许多紫茉莉;②紫茉莉是童年时提醒“我”回家的天然“计时器”;③紫茉莉为“我”的童年生活增添了快乐和温馨;④“我”对紫茉莉始终保持着好奇和喜悦的心情。‎ 解析 本题考查理解和分析重要语句的含意的能力。通读全文,分条概括出“我”与紫茉莉的相互联系即可。‎ ‎3.文章最后一段说:“我每天看到自己种植的紫茉莉,都悲哀地想着,不仅是都市的人们容易遗失自己的心,连植物的心也在不知不觉中迷失了。”‎ 请结合本文和下面的材料谈谈你对这句话的思考。要求:观点明确,阐述合理,有说服力。‎ 所以在(洛阳)这阴冷的四月里,(即便是牡丹花节)奇迹不会发生。任凭游人扫兴和诅咒,牡丹依然安之若素。它不苟且不俯就不妥协不媚俗,它遵循自己的花期自己的规律,它有权利为自己选择每年一度的盛大节日。(张抗抗《牡丹的拒绝》)‎ 人只不过是一根苇草,是自然界最脆弱的东西,但他是一根能思想的苇草。……我们的全部尊严就在于思想。‎ ‎(帕斯卡《人是一根能思想的苇草》)‎ 答:___________________________________________________‎ ‎_______________________________________________________‎ ‎_______________________________________________________‎ ‎_______________________________________________________‎ 答案 这句话借紫茉莉花在都市里忘记了它祖先对黄昏彩霞的选择这一事,告诫人们在都市生活的喧嚣中要保持本真,不要迷失了自我。就像四月的洛阳的牡丹,不为迎合游人的到来而开放,保持了自己应有的开放的节奏,显示出了非同寻常的个性,保持了自己应有的本色,让人赞赏。在生活日趋城市化的今天,人类不能因为远离乡村,远离自然,被城市的喧嚣淹没,就缺少了对生活、生命的思考。我们要善于思考生活,保持自己的本色和本真,活出人类应有的尊严。‎ 解析 本题考查探讨作者创作意图和运用材料证明观点的能力。首先要有对原句句意的理解,然后对两则材料进行分析,分别扣住“要保持本真,不要迷失了自我”作分析。‎ 考点三十三 新闻、报告、科普类阅读 考点名片 考点内容 考查新闻、访谈、科普文章阅读,一般涉及文体特征,写作手法,内容概括分析。‎ 考查形式 主观题和客观题。①请概括出这篇访谈中,×××的主要观点。②围绕×××,×××先生从哪几个方面做出了倡导?③阅读材料,概括说明他们各自传递的信息有哪些异同。‎ 趋势分析 新闻的内容概括、手法特点、访谈技巧等。‎ 一、阅读下面的文字,完成1~3题。‎ 专访刘慈欣:他的创作和生活 科幻小说《三体》的英文版已经推出,出版方托尔出版公司(Tor Books)曾出版过多部获得星云奖和雨果奖的重要作品。中国内地的长篇科幻小说在欧美出版,是非常新鲜的现象。对作者刘慈欣的采访,可谓恰逢其时,也自然由此开始。‎ 财新记者:《三体》系列一共有三册,是一个很宏大的系列。故事最后的发展,与你最初的预设是否一致?‎ 刘慈欣:当然,作为一个业余作者,我肯定是完全想好再去创作一本书的。但是,在想的过程中变化太多了,肯定和最初的想法完全不一样。不说完全不一样,也有很大的差异,但是变化了多少是无法统计的。这么长的作品,构思肯定经过了一定的演变过程,这中间会出现各种改变。‎ 财新记者:在写作的过程中,你会跟别人讨论所写的内容吗?‎ 刘慈欣:绝对没有。以我的习惯,在创作过程中我绝对不会和别人有任何讨论。有的作家写一段给朋友看一段,然后征求意见,这对我来说是不可想象的。完全没有交流,100%都是我自己在做这个事情。‎ 财新记者:很多人认为,《三体》达到了很高的高度,你会不会担心自己很难超越这个成就?现在写作时会不会有一些压力?‎ 刘慈欣:应该是有的。难超越,一来是书本身的影响;第二,也有个重要因素,就是任何一本书的成功,特别是类型文学的成功,除了书本身以外还有很多机遇的因素。机遇这个因素是可遇不可求的,不可能每本书的机遇都那么好。这也是它难以超越的一个原因吧。‎ 财新记者:成为优秀的科幻作者需要哪些条件呢?‎ 刘慈欣:优秀的作家、作者可遇不可求。成为优秀作者的条件很难量化,优秀作者最有可能在特定文学体裁繁荣的时期诞生。如果科幻市场比较繁荣,就会吸引越来越多的作者投身于科幻创作,诞生大师、有影响力的作家的可能性自然会加大。如果整个文学体裁不繁荣,很冷清,虽然不是完全没可能在冷清的文学体裁里诞生大师,但是可能性很小。其实,一部有影响力的作品,就像金字塔的塔尖,底下有大量普通的作品做基础。否则,有影响力的作品只能是一个特例,成功很难重复。‎ 财新记者:那目前中国科幻文学处于什么样的阶段?‎ 刘慈欣:我们还处在一个积累阶段,作品数量还是太少。科幻作家的数量更少,长期写科幻作品的就这么几个人。虽然,发表过科幻小说的作者数量并不少,《科幻世界》上新面孔就常常出现,但大多发表一两篇后就销声匿迹了,长期在科幻文学里面存在的作者数量非常少。主流文学中,中国作协会员有万人左右,而科幻小说的长期作者也就二三十个,数量太少。‎ 财新记者:现在恰逢毕业季,你在年轻的时候,有没有一些至今都觉得有意思的事情,甚至在创作小说时提供了灵感?‎ 刘慈欣:我是一个离开学校30多年的人了,作为科幻小说作者,也不是太怀旧的人。所以,我对在学校时的记忆都有些模糊了。前几天,我回了一次母校,和同学们交流,才感受到学校的生活。但是,我仍然觉得,现在看到的学校生活和我当年很难联系起来。所以,学校生活对我的作品基本没有什么影响。‎ 财新记者:和现在的年轻人沟通时,你觉得会有所谓代沟吗?‎ 刘慈欣:这要从两方面来说:肯定会有代沟,现在新媒体、网络技术发展得这么快,时代不同,人们接触到的信息更多,生活环境变化很快,代沟肯定是存在的。但是,我和年轻人的共同之处肯定多于差异。科幻作品的读者一般都是年轻人,《三体》这本书在年轻人之间产生了很大影响,说明我们之间的共同之处还是很多的,能够互相理解。‎ ‎(有删改)‎ ‎1.下列对材料有关内容的理解和分析,不符合原文意思的一项是(  )‎ A.刘慈欣认为如果科幻市场比较繁荣,就会吸引越来越多的作者投身于科幻创作,诞生大师的可能性就大;反之,可能性就小。‎ B.在刘慈欣看来,一部有影响力的作品不是一蹴而就的,作家只有在创作上有大量的经验积累,才能厚积薄发。‎ C.科幻文学的发展,需要大量长期坚持科幻文学创作的科幻作家。当前科幻作者数量非常少,这样,就使这一文体缺少了基础,优秀的作品便很难出现。‎ D.中国的科幻小说尚处于起步阶段,要想使中国科幻文学获得进一步发展,需要培养更多的创作者,也要求创作者能够坚持创作并了解主要的阅读对象——年轻人的心理。‎ 答案 C 解析 C项,当前作者数量并不少,只是长期坚持的非常少。‎ ‎2.下列对文章有关内容的分析和概括,最恰当的两项是(  )‎ A.刘慈欣是科幻小说作家,科幻小说《三体》的英文版获得了星云奖和雨果奖,即将由托尔出版公司出版。‎ B.《三体》是一个很宏大的系列,作者刘慈欣虽然是完全想好再去创作的,但因为想的过程中变化太多了,结果肯定和最初的想法有很大的差异。‎ C.刘慈欣在整个创作过程中都不会和别人讨论他所写的内容,而其他作家则是写一段就给朋友看一段,然后征求意见。‎ D.很多人认为,《三体》达到了很高的高度,刘慈欣也担心自己超越不了这个成就,因此,在写作时会有很大的压力。‎ E.刘慈欣作为一个离开学校30多年的人,因为不是太怀旧,所以年轻时候的一些事情,对他的小说创作基本没有什么影响。‎ 答案 BE 解析 A项,“获得了星云奖和雨果奖”‎ 错,文中并没有说《三体》的英文版获得了这两个奖项,只是说《三体》英文版的出版方曾出版过多部获得星云奖和雨果奖的重要作品。C项,“其他作家”错,原文说的是“有的作家”。D项,“很大的压力”不准确,原文说压力“应该是有的”,但不代表压力很大。‎ ‎3.阅读全文,概括《三体》这一科幻作品的特点。‎ 答:___________________________________________________‎ ‎_______________________________________________________‎ ‎_______________________________________________________‎ 答案 ①规模宏大,构思充分;②达到了很难超越的高度;③符合年轻人的心理。‎ 解析 本题主要考查筛选并整合文中信息的能力。题目要求在阅读全文的基础上概括《三体》这一科幻作品的特点。首先,明确整个访谈有哪几个部分是就这一作品本身而言的。其次,对这几个层次分别进行分析,找出能够体现这一作品特点的内容。最后,对关键信息进行整合概括,就可以得出正确答案。需要注意的是,刘慈欣在谈其他问题时也可能会谈到《三体》及其特点,考生要善于分析刘慈欣的话中隐含的信息。‎ 二、阅读下面的新闻调查,完成1~3题。‎ 八成贫困生相信上大学可改变命运 这些天,各地高校的录取通知书陆续发放。但是,一纸喜报换来的并不都是如愿以偿的笑脸。‎ ‎6月19日夜,山西省榆社县西马乡新村43岁的陈东生(化名)服毒自杀,原因是他“不堪承受儿子读大学的经济重负”。‎ ‎8天之后,同样的悲剧再次上演。因为女儿的高考分数达到了二本录取分数线,自己又无力凑够学费,山西省翼城县唐兴镇南官庄村农民李海明在家中上吊自尽。‎ 为何悲剧总在收获的季节里重演?高考贫困生的生活状况如何?他们更需要哪些帮助?‎ ‎5月10日~6月20日,在中国青少年发展基金会的监督和指导下,北京锐智阳光信息咨询有限公司对高考贫困生进行了一次走访调查。本次调查覆盖北京、安徽、湖南、广西、云南、山西、四川等11个省(区、市)的16所高中,共回收贫困高考生问卷322份,高中教师和校领导问卷89份。‎ 调查显示,82.3%的贫困高考生家庭贫困的主要原因是教育支出大。其中,平均每年每个贫困高考生家庭的子女教育支出在6780元左右,高于平均家庭总收入(4756元),超过2/3的家庭入不敷出,有81.0%的家庭总收入尚不足以支付子女的教育费用。‎ 在农村,因子女教育花费大而造成家庭致贫、返贫的事早已不是新闻了。江西省赣州市某村的一名考生家长说,以前指望孩子通过读书跳出农门,现在发现从小学、中学到大学,学费一路看涨,不少家庭供到半路,就揭不开锅了。村里初中毕业就外出打工赚钱的孩子越来越多了。‎ 在调查涉及的地区中,贫困生约占32.7%,西部的比例明显高于东部。他们当中,品学兼优者占52.2%。调查发现,11.5%的贫困生在高中期间曾经辍学,12.7%的人因交不起学费可能放弃上大学。‎ 调查中,62.8%的高中教师认为,能否支付大学第一年学费往往决定着贫困生能否圆大学梦。而从实际情况来看,六成贫困高考生表示自己不能凑齐大学第一年的学费。‎ 与之形成鲜明对照的是,82.5%的贫困生认为自己能考上二本以上院校,80.3%的贫困生认为上大学可以改变命运。‎ ‎“我听说大学里有助学贷款,还有奖学金。我先贷款,再挣奖学金,假期还能想办法打工赚钱。”家住沂蒙山区的张慧今年9月马上就要成为山东大学的学生了,头一年的5000元学费是父母东拼西凑筹集的。对于未来,她充满了信心:“已经考上了,再难都要熬过去,往后就能替家里分忧了。”‎ 和张慧一样,由学校提供的助学贷款已经被33.8%的贫困高考生认为是最有效的资助方式。相比之下,最贫困的那部分人(25.5%),明显对社会经济资助更为渴望。‎ 值得关注的是,虽然近些年来政府相关部门不断提高助学贷款的总量,但却无法满足同样逐年增长的贷款需求。有数据显示,截至2005年8月,我国普通本专科在校生数约1562万人,其中贫困大学生405万人,约占学生总数的26%。截至2005年12月底,在全部大学生中,获得助学贷款者的比例仅占14.6%——这说明,约有178万贫困大学生未能取得助学贷款。‎ 本次调查发现,有些人是主动放弃贷款和社会资助的。家住安徽省固镇县马林村洪庄、就读于固镇一中高三10班的张莹莹在调查中说:“不想申请社会资助是不想被别人看不起,我会尽量靠自己的能力和亲戚的支持完成学业的。”‎ 面对上大学后将面临的经济困难,94.7%的人表示“进入大学后将争取获得奖学金”,82.2%的人“希望通过勤工助学获得一定收入来补贴生活费”。还有部分学生表示,大学期间要省吃俭用甚至假期不回家,以降低生活开支。‎ ‎(选自《中国青年报》)‎ ‎1.结合文中调查,简要概括高考贫困生的生活状况。‎ 答:___________________________________________________‎ ‎_______________________________________________________‎ 答案 ①多数家庭总收入不足以支付子女的教育费用,不能凑齐大学第一年的学费;②多数贫困生品学兼优,认为上大学可以改变命运;③多数贫困生渴望得到社会经济资助,对未来充满信心。‎ 解析 关于“高考贫困生的生活状况”的调查,在文章第六至十五段,第六、七两段主要讲高考贫困生的家庭经济状况,第九段对高中教师的调查从侧面反映高考贫困生的家庭经济状况;第八、十两段主要反映贫困生的学习状况及对上大学的认识;第十一、十二、十五段主要反映贫困生的心理渴望与对前途的信心。所以,答案可从三方面概括。‎ ‎2.文章开头的悲剧令人痛心,结合全文分析,如何避免悲剧在收获的季节里重演。‎ 答:___________________________________________________‎ ‎_______________________________________________________‎ ‎_______________________________________________________‎ 答案 ①贫困生家庭要正视困难,做生活的强者;②政府要最大量地提供助学贷款;③完善社会救助机制,维护受助者的自尊。‎ 解析 这两起悲剧的发生,一是因为“不堪承受儿子读大学的经济重负”;二是因为自己无力凑够女儿的学费。悲剧是可以避免的,文中第十一段张慧及其父母的做法给了贫困生家庭一个启示:正视困难,勇敢面对困难,做生活的强者。第十二、十三段“助学贷款的效果”提示政府要最大量地提供助学贷款。第十四段有些人主动放弃贷款和社会资助的原因提示要完善社会救助机制,维护受助者的自尊;第十五段也提示贫困生要自立自强。‎ ‎3.本文在新闻调查情况的表达上有什么特点?这样表达有什么效果?‎ 答:___________________________________________________‎ ‎_______________________________________________________‎ 答案 主要使用百分数表达调查的情况,这样便于直观地看出每种调查情况涉及的人数比例和某一问题存在的普遍程度。‎ 解析 从全文看,主要使用百分数表达调查情况。如调查“高考生家庭贫困的主要原因”时,82.3%的贫困高考生家庭贫困的主要原因是教育支出大,用“82.3%”很直观地表达出“教育支出大”是绝大多数贫困高考生家庭贫困的主要原因,其他调查也和这个调查一样。‎ 三、阅读下面的文字,完成1~3题。‎ 矮象和硕鼠 方舟子 提起庞然大物,我们往往会想到象。象是现存最大的陆地动物,最大的非洲丛林象的身高能超过4米,体重能达到12吨。象刚生下来时的体重通常就有120千克。它们是如此庞大,以至于我们习惯充满敬畏地称之为“大象”。‎ 但是有一种象的形象很难让人给它冠以“大”字,它们成年时身高还不到1米,体重大约只有100千克,和猪的大小差不多——猪鼻子插葱有时的确是可以装象的。这样的矮象一定很可爱,可惜它们只生活在更新世,已在大约1万年前灭绝了。‎ 矮象并不是大象的祖先,在它们之前大象早已进化出来了。相反,矮象是从大象进化来的。这些矮象的化石是在地中海的西西里岛和马耳他岛发现的。在地中海的其他岛屿,也有类似的矮象化石出土。大约600万年前,地中海的海水被部分或全部蒸发了,生活在大陆的大象得以迁移到这些岛屿。后来随着海平面的上升,这些大象的后代与世隔绝,在岛上进化成了猪一般的矮象。‎ 为什么大象的后代变成了“猪”呢?在回答这个问题之前,我们需要先问一下,为什么大象要把身体 弄得那么庞大呢?原因很简单,身体庞大是它们保护自己的法宝。当大象的身体大到连老虎、狮子也望而生畏时,大象就没了天敌,可以自由自在地吃草。但是身体庞大也意味着要消耗大量的食物,一头成年大象每天要吃一两百千克的植物。而一个大象群体为了避免近亲繁殖,通常有几百头大象,它们消耗的食物量大得惊人。‎ 在地中海的岛屿上,没有大型的肉食动物需要抵御,大象身体庞大的优势消失了,劣势暴露了出来。在岛上,资源非常有限,没有足够的食物来维持一个大象群体,身体大的象饿死,身体小的象由于需要的食物量也少,反而容易生存下来。要维持一个500头大象的群体,可能至少需要几千平方千米的空间,但是要维持一个同等数量的矮象群体,有几十平方千米的地盘可能就足够了。这样,在自然选择的作用下,大象的身体就变得越来越小。‎ 不仅大象如此,其他大型食草动物到岛上繁衍后,由于没有天敌,也由于缺乏食物,它们后代的身体也会逐渐变小。‎ 但是对俗称老鼠的啮齿动物来说,情形却正好相反。老鼠体型小的原因,是为了避免被天敌吃掉,因为身体小容易躲藏。到了岛上以后,没有那么多天敌,甚至根本就没有了天敌,没有必要再躲躲藏藏,身体小的劣势就暴露出来了,因为身体太小,热量容易散失,而且能吃的食物种类有限,比如粗一点的植物就咬不动。对老鼠来说,身体大一些反而更有优势。在岛上生活的啮齿动物的身体通常要比在大陆生活的大很多。例如弗洛里斯岛上原来生活着一种窟山鼠(已灭绝),和猫一样大。‎ 不过,和在乌拉圭发现的一种豚鼠相比,窟山鼠就是小巫了:这种乌拉圭豚鼠比牛还大,体长3米,高1.5米,重达1吨,比矮象还要大得多。幸运的是这种可怕的硕鼠生活在200~400万年前。在当时,巴拿马地峡还未出现,南美洲和北美洲还是分离的,南美洲实际上是一个大岛,北美洲的剑齿虎等天敌还不能过去享用这些硕鼠。现在还生活在南美洲的花背豚鼠是硕鼠的近亲,而花背豚鼠的体长只有80厘米,体重只有15千克,这在现在的啮齿动物中已经算是大的了。‎ 当大型的哺乳动物迁移到岛屿,它们的身体通常会逐渐矮化,而小型的哺乳动物则相反,它们的身体会逐渐巨化,这种现象被称为“岛屿法则”。这个进化速度会非常快。对88种岛屿哺乳动物的调查表明,它们的进化速度要比大陆哺乳动物快两倍。‎ 人类的文化进化其实也存在类似的“岛屿法则”。对人类社会来说,“岛屿”并不一定是地理意义上的。开放的岛国可以包容世界,自我封闭的大陆国家却是文化的岛屿。一个国家如果闭关锁国成了世界的孤岛,那么由于缺乏来自外部的竞争压力和文化交流,也会很快地导致伟人矮化,鼠辈巨化。这样的国家,会一时成为“矮象”和“硕鼠”的天堂。但是不论是可爱的矮象还是可怕的硕鼠,终究还是都灭绝了。‎ ‎(选自《中国青年报》,有删改)‎ ‎1.下列对这篇科普文章有关内容的分析与概括,正确的一项是(  )‎ A.历史上曾经有过一种象,成年时身高还不到1米,体重大约只有100千克,因此它是一种实质上不能称为象的象。‎ B.矮象曾经生活在地中海的西西里岛、马耳他岛以及其他许多岛屿,这是由那里被发现的大量矮象化石推断出来的。‎ C.动物进化受地理环境影响巨大,如同种动物在大陆和岛屿上进化的结果迥异,而所有动物在岛屿上的进化结果则趋于一致。‎ D.本文通过对矮象与硕鼠的进化背景的分析,让我们看到了环境对进化所起到的作用,即环境的改变一定会使生命体发生变化。‎ 答案 B 解析 A项,“它是一种实质上不能称为象的象”错误,它实质上还是一种象。C项,“所有动物在岛屿上的进化结果则趋于一致”错误,如通常象在岛屿上会变小,啮齿动物在岛屿上会变大。D项,夸大了环境的作用,太绝对。‎ ‎2.本文的语言除了准确严谨之外,还具有什么特色?请结合文中加点的内容简要分析。‎ 答:___________________________________________________‎ ‎_______________________________________________________‎ ‎_______________________________________________________‎ 答案 生动有趣。如“猪鼻子插葱有时的确是可以装象的”,活用歇后语,表现矮象之矮小;“猪”运用了比喻的修辞手法,形象地突出了矮象之矮小;“弄得”口语化,通俗易懂,大象的身体大小本是自然选择的结果,而这样说好像结果是大象自己选择的一样,耐人寻味。‎ 解析 该题考查分析文本的基本特征的能力。科普文的语言特色,一般来说都是准确严谨的,而本文的语言还十分生动。明确了这一点,还要结合具体内容进行分点分析。‎ ‎3.“岛屿法则”对我们有着怎样的启示?请结合我国一直坚持实行对外开放政策的现实谈谈你的理解。‎ 答:___________________________________________________‎ ‎_______________________________________________________‎ ‎_______________________________________________________‎ ‎_______________________________________________________‎ 答案 要想生存,必须开放。一个国家如果不对外开放,就会缺乏来自外部的竞争压力和文化交流,而缺乏来自外部的竞争压力和文化交流,就会盲目自信,导致经济和文化建设缺乏动力和创新,轻则影响社会主义现代化建设的进程,重则导致国家的灭亡。我国一直坚持实行对外开放政策就是避开“岛屿法则”的正确做法。‎ 解析 该题考查从不同的角度和层面发掘文本反映的民族心理和人文精神的能力。作答本题时,首先要结合文本内容,明确“岛屿法则”的具体内涵,然后将其上升到国家层面,联系我国的开放政策进行分析。‎ ‎[3年高考真题集训]‎ ‎[2015·广东高考]阅读下面的文字,完成1~3题。‎ 种子萌芽从汲取水分开始,种子首先膨胀,胚部接着生长。①发芽就像分娩,一旦启动就是个不可逆的过程,因此发芽的时机攸关生死,种子早已演化出各式各样的手段找对时机。杨树和柳树的种子寿命极短,若没有在散播后几个小时内找到湿泥地,就会死亡。热带雨林中,许多树木结的大型种子如果没有在几周内发芽,就会腐烂。对这些物种来说,要掌握正确的发芽时机,得先掌握正确的结子时机,因为种子一旦成熟,保存期限是很短的。‎ 一年中,若季节气候对种子来说太干燥或太寒冷,不利于发芽,种子就进入休眠,等待不适宜的气候过去。‎ 如果将植物行为以精巧度衡量,扁蓄的发芽行为只有初级程度。扁蓄是长在耕地和花园中的杂草,种子先天为休眠型,未经冬天不会发芽。土壤中的种子在冬季感受到低温,打破休眠状态,预备好在春天来临、土壤回暖时发芽。没有在五月发芽的种子,又逐渐进入休眠状态,静待另一波寒意来临,准备好再次发芽。扁蓄的行为其实相当合理,称为初级程度好像不太公平,更何况相似的植物还不少,许多杂草也有依季节循环的休眠期和发芽期。‎ 另一种相当常见的杂草则要比扁蓄技高一筹,那就是羊腿藜。羊腿藜在年初结不休眠的种子,产季时则结休眠的种子。有了年初生产的种子,这年的羊腿藜就有更多子代,后来所结的种子则安然保存至明春。‎ 一年生植物的种子发芽机制就更精巧了,这些植物会打赌冬天冷不冷。许多冬季一年生植物,像雀麦草,都是在秋天发芽,这样做的风险是冬天的寒霜很可能冻死幼苗。但如果存活下来,就有了回报,因为比起其他春天才发芽的种子,这些幼苗抢先一步发育成长,能长得比较大,结的种子也多得多。冬季一年生植物也懂得分散风险,让部分种子在冬季休眠、春天发芽。比起熬过冬天的幼苗,这些春天发芽的种子长出的植株比较小、结的种子也比较少,但是存活的几率通常大一些。由于某几年的冬天比较冷,某几年比较不冷,所以有时不休眠种子长得好,有时休眠种子胜算大。‎ 目前为止所谈的发芽行为,精巧程度都还不及格。种子还能从其他线索得到更精确的讯息,以决定什么时候发芽。许多像莴苣一类的种子,对光很敏感,在黑暗中不会发芽,即使温度和湿度都很恰当。这样的机制可以避免那些被埋得很深的、没有机会到达土壤表层的种子发芽。而只要一丝阳光,就能让种子自休眠中苏醒。替菜圃翻土会让阳光传递讯息给土壤中许多对光敏感的杂草种子,让它们知道接近土壤表层了。‎ 很多种子的招数更了不起。植物可借由远古演化而来的光感受体(即光敏素分子)感知光线。光敏素分子有两种形式,可彼此转换。一种称为Pr,吸收红光后转换成另一种形式,称为Pfr,Pfr吸收远红光后再转回Pr型分子。Pr对Pfr的比率是由接收多少这两种不同波长的光而定。这种Pr对Pfr的比率提供了地方环境的讯息,对植物来说极为重要。②未被遮蔽的日光,红光/远红光的比率为1,然而,阳光穿过树叶后,大部分红光为树叶吸收,这时比率远小于1。植物透过Pr对Pfr的比率能侦测到光的改变。由此,植物利用光敏素察觉邻近植物的位置,调整自己的生长以避开邻近植物。同样,种子如果暴露在透过叶子照射的阳光中,就不发芽。与其在其他植物的遮蔽下发芽、长出难以存活的幼苗,还不如维持休眠状态。‎ 光敏素有个缺点,就是在黑暗环境中无用武之地。因此只有位于土壤表层或接近表层的种子能加以利用,侦测竞争对手是否环伺在侧。不过还有另一种线索,能让深埋土中的种子知道上方有没有一线生机。土壤表层的植物,例如草类,就像一层隔绝物,调节土壤内种子感受温度的范围。光裸的土壤表层没有这层隔绝物,所以底下的种子会感觉到强烈的温度起伏,许多种子就从这种温度起伏中得知土壤表面是光裸的。如果感受到温度起伏,种子就会在春天发芽;而如果感受到的温度很平稳,没有高低起伏,种子就不发芽。‎ ‎(选自[英]乔纳森·西尔弗顿著、徐嘉妍 译《种子的故事》,商务印书馆,2014年,有删改)‎ ‎1.简要分析文中画线部分的语言特色。‎ ‎①发芽就像分娩,一旦启动就是个不可逆的过程,因此发芽的时机攸关生死,种子早已演化出各式各样的手段找对时机。‎ 答:___________________________________________________‎ ‎_______________________________________________________‎ ‎②未被遮蔽的日光,红光/远红光的比率为1,然而,阳光穿过树叶后,大部分红光为树叶吸收,这时比率远小于1。‎ 答:___________________________________________________‎ ‎_______________________________________________________‎ 答案 ①用分娩的不可逆转和攸关生死来比拟种子选择发芽时机的重要性。语言形象生动,令人印象深刻。②使用专业术语和数据来说明日光被遮蔽前后红光/远红光的比率变化。语言平实准确,令人信服。(意思答对即可)‎ 解析 回答句子的语言特色时,应抓住该句子的表现手法、语言风格等,并结合语境进行分析。第①句运用比拟的修辞手法,形象生动地说明了种子选择发芽时机的重要性;第②句用语专业,且列举了相关数据,其中的“大部分”“远小于”等使文章语言准确严谨。‎ ‎2.作者认为有些种子发芽行为的精巧程度还不及格。请写出这些种子的发芽行为。‎ 答:___________________________________________________‎ ‎_______________________________________________________‎ 答案 ①扁蓄种子感受到冬季的低温才结束休眠,在春天发芽。②羊腿藜分别结不休眠的种子和休眠的种子,它们各选择不同的时机发芽。③许多冬季一年生植物的种子选择全部在秋天发芽。④有的冬季一年生植物的种子部分在秋天发芽,部分在冬季休眠、春天发芽。(意思答对即可,只要言之有理,均可酌情给分)‎ 解析 答题时应抓住题目对应的第六段“目前为止所谈的发芽行为,精巧程度都还不及格”这句话,对前文所列举的种子的发芽行为进行归纳总结。‎ ‎3.翻土可以除掉土壤表层的杂草,但是翻动过的土壤有时反而长出更多的杂草。请用文中观点解释这种现象。‎ 答:___________________________________________________‎ ‎_______________________________________________________‎ 答案 ①翻动土壤后有些种子到达离土壤表层较近的地方,那些对光敏感的杂草种子能感知到光而发芽。②翻动土壤除去了土壤表层的隔绝物,深埋土中的杂草种子会感受到强烈的温度起伏而在春天发芽。‎ 解析 解答本题时,应明确答题区间为文章最后三段。翻土会让阳光传递讯息给土壤中许多对光敏感的杂草种子,除掉土壤表层的隔绝物也影响杂草种子对温度起伏的感知,这都可能促使杂草种子发芽。‎ ‎[2年全国模拟重组]‎ 一、[2017·湖南长郡中学模拟]阅读下面的文字,完成1~3题。‎ 阅读是一个民族持久的功课 ‎——国家新闻出版广电总局原副局长邬书林访谈录 记者:“全民阅读”今天已经成为一股热潮。从2006年中宣部等八部委提出开展全民阅读活动至今已近10年。今天回过头来看,您认为我们做得怎么样?‎ 邬书林:我认为现在态势很好,中央重视、社会认同、民众参与、活动健全,经过10年努力,全民阅读有了非常坚实的基础。现在全国31个省700多个市都已开展了形式多样、民众广泛参与的阅读活动。阅读不仅仅是个人的事,阅读已经走进学校、社区、军营、农村甚至高墙。江苏、湖北、深圳、辽宁等地先后完成了全民阅读的地方立法工作,确定了全民阅读在建设学习型社会中的法律地位,把全民阅读上升为法律法规,我认为这是一个标志性的重大进展。‎ 对一个民族而言,阅读是一项长久的文化工作,绝不是一时的。如何把全民阅读作为一项基础性的长期性的工作持之以恒地开展下去,需要好好总结前10年的经验。‎ 记者:您认为有哪些经验值得总结?‎ 邬书林:把全民阅读作为中华民族提升整体国民素质的重要工作长期不懈地抓下去,需要总结过去10年的经验。我认为,首先要提高认识,要对阅读的功能、意义,阅读的本质给予进一步的认识。阅读的本质是通过阅读去借鉴、吸收和掌握前人已有的知识、经验与智慧。现在有很多人认为阅读主要是用来提升修养,我觉得这不是阅读行为的本质。阅读的本质是要获取知识,在获取知识的基础之上有了生产生活科学研究的能力,同时再提高自己的精神境界。这两者并行不悖。‎ 今天我们要很好地用阅读这个工具来提升国民素质,用全人类的图书把中华民族武装起来,为中华民族伟大复兴打下坚实的文化基础、知识基础。‎ 记者:对于全民阅读的重要意义,现在社会和民众已有共识,那么国家、政府层面还需要做好哪些工作?‎ 邬书林:我们不是一般地提倡全民阅读,而是要把全民阅读持久深入、实打实地推进下去,这就需要做很多工作。这几年的两会,我都联合政协委员上交关于尽快出台《全民阅读促进条例》的提案,以法律法规的形式将推动全民阅读工作纳入法制化轨道。各级政府及相关部门应进一步加大公共图书馆和农家书屋等基础设施的建设。另外,养成阅读习惯是一门科学,0到6岁是阅读的启蒙阶段,使儿童通过阅读养成阅读兴趣,从阅读中找到快乐;6到12岁小学教育阶段是非常重要的环节,使孩子基本掌握阅读的方法;12到18岁是巩固阅读习惯,提升阅读水平的关键阶段。打好这三个基础,终身阅读才有可能,活到老、学到老才有坚实的阅读习惯基础。专家学者要为整个国家提升阅读水平做好表率,同时提供丰富的精神食粮。‎ 记者:在今年的全国两会上,李克强总理说到中国人阅读量不足有些国家的1/10。从近年调查情况来看,国民阅读率虽然在上升,但速度比较慢。您如何看待中国国民阅读率低的问题?‎ 邬书林:对这个问题要实事求是地看。新中国成立初期,我们国家的文盲率是80%,改革开放初期我国文盲率是34%,最近这次人口普查统计显示文盲率是4%。解决阅读问题首先要解决文盲,我国的文盲问题才刚刚解决,国民阅读率不可能有多高。我坚信孩子的阅读率会越来越高。其次现有的阅读率的统计指标还要进一步完善。要把图书馆的数据和其他形式的阅读数据也纳入其中。最后我国的国民阅读率的确亟待提升。‎ 不过,现在信息技术的进步,特别是移动互联网技术为我们的阅读提供了非常好的工具,通过手机、平板电脑等设备阅读的人数大增。我最近反复强调这个观点,不要以为通过手机阅读就不是阅读,不要以为通过电子阅读器阅读就不好。相反,新技术极大提升了阅读效能。根据美国所做的一项统计,网络时代科学家的阅读量比原来增长了25%,普通民众也增加了5%。因为不需要跑图书馆了,买书便宜了,看书更方便了。所以,既要重视传统阅读,更要关注现在信息技术条件下的阅读,两者并行不悖。我相信,随着阅读意识的提高,阅读设施的完善,阅读氛围的浓厚,我国国民阅读率一定会赶上先进国家的。‎ ‎(选自《人民日报》,有删改)‎ ‎1.下列对文章有关内容的理解和概括,不正确的一项是(  )‎ A.邬书林认为全民阅读经过10年的努力有了坚实的基础,无论是国家重视程度,还是全民参与程度,都有了很大的进步。‎ B.邬书林在总结经验时主要从对阅读本质的理解和认识上做出了总结,认为阅读主要是用来提升人的修养的。‎ C.记者由李克强总理在两会上说的中国人阅读量不足有些国家的1/10切入问题,准确恰当,并且突出了时效性和针对性。‎ D.邬书林认为移动互联网技术改变了人们的阅读方式,新技术极大提升了阅读效能,随着信息技术条件下的阅读的进一步发展,我国国民阅读率一定会赶上先进国家的。‎ 答案 B 解析 本题考查理解文意、筛选并整合文中信息的能力。B项,“认为阅读主要是用来提升人的修养的”理解错误,原文中邬书林说“阅读的本质是通过阅读去借鉴、吸收和掌握前人已有的知识、经验与智慧。现在有很多人认为阅读主要是用来提升修养,我觉得这不是阅读行为的本质”。‎ ‎2.邬书林认为要提倡全民阅读,需要做好哪些工作?‎ 答:___________________________________________________‎ ‎_______________________________________________________‎ ‎_______________________________________________________‎ 答案 ①国家立法,以法律法规的形式将推动全民阅读工作纳入法制化轨道;②各级政府应进一步加大基础设施投入,创造全民阅读条件;③遵循阅读规律,养成阅读习惯;④专家学者要做好阅读表率并为大众提供丰富的精神食粮。‎ 解析 本题考查理解文意,筛选并整合文中信息的能力。解答此题首先要确定答题区域,题目所问“需要做好哪些工作”,其实就是记者提的第三个问题,那么答题区域就是邬书林针对这个问题的回答。在这段话中“打好这三个基础”前的三句话即是分别从国家、各级政府、全民三个角度提出的做法,另外不要忽略本段最后针对“专家学者”提出的建议。‎ ‎3.邬书林认为中国国民阅读率低的原因是什么?请从个体角度谈谈如何提升阅读率。‎ 答:___________________________________________________‎ ‎_______________________________________________________‎ ‎_______________________________________________________‎ 答案 原因:①历史原因。我国一直存在文盲问题,而解决这一问题经历了较长的过程,国民阅读率不可能有多高。②统计指标还不完善。图书馆的数据和其他形式的阅读数据尚未纳入其中。‎ 做法:①提高阅读意识,认识到阅读的重要性。要充分意识到阅读不仅能获取知识,而且能提高自己的精神境界,提升国民素质。②注重多种方式的阅读。既要重视传统的纸质阅读,更要关注信息技术条件下的电子阅读。③遵循阅读规律,制定阅读计划,努力做到终身阅读。‎ 解析 本题考查探究文本中某些问题、提出自己的见解的能力。第一问,可从文中总结归纳,倒数第二段邬书林的回答中前两点即是国民阅读率低的原因。第二问,可结合文本相关信息和自身实际,文中第五段和最后一段都谈到国民阅读的经验和方法,分别从阅读意识和阅读方式的角度概括即可。‎ 二、[2016·福建模拟]阅读下面的文字,完成1~3题。‎ 假课改应付不了考查能力的高考 李新玲 今年北京地区高考分数每个批次均上涨几十分,一本录取比例由于指标缩减而略有下降。在这样的情况下,北京东城区考生的高考成绩和上线率均好于去年,一本上线率连续3年保持六成以上,本科上线率连续3年保持九成以上。东城区教师研修中心的课改专家发出感叹:“这归功于扎实的课改。面对考查能力的高考,假课改行不通!”‎ 改变课程内容“繁、难、偏、旧”的现状,减轻学生负担,推行素质教育,是我国第八次课程改革纲要明确提出的要求,也是社会公众对新课改寄予的期望。但是在很多地方,教师反映新课改后教学难度加大,学生课业负担加重。也出现过一些怪现象,比如一个地区或者学校推行素质教育或者教学改革,往往是有头没尾,或者是在高考这一关上败下阵来。在课改专家看来,这是因为教师没有领会新课改的精神实质,是走偏了的“课改”。‎ 今年是北京市高中实行新课改的第6个年头,而义务教育的课改也正好走过了12年,完成了基础教育的一个完整批次,今年的考生都是从小学一年级开始就进入了新课程改革的实践。所谓“假课改”‎ 是指有的学校虽然进行了课改,但课程形式、教学方法仍没有真正改变,老师还是课堂的主角,填鸭式满堂灌,大量作业重复训练。‎ 东城区教师研修中心常务副主任郑克强介绍,在东城区已经基本上找不到传统课堂,启发式、讨论式课堂比比皆是,不仅是高考的9大科目,音、体、美等学科也在探索自主课堂。‎ 有一所这几年高考成绩和社会声誉提高较快的学校,它创建了“五步自主高效课堂”,就是要求学生在学习新知识之前,在家完成“自学案”,之后在课堂上分为五步走:第一步课堂上组内交流讨论,第二步按小组进行答案讲解,第三步同学相互提问讲解,第四步教师进行进一步的讲解,第五步完成“检测案”。‎ 这种教学模式最大的改变在于,将传统的教师教学为主改为学生在家自学为主,这在一定程度上,锻炼了学生的自学能力和语言表达能力以及相互合作解决问题的能力。为了鼓励学生发言,还制定了分数奖励制度,每次发言加1分,每次上台展示加2分。分数由组长记录,放学之前交给班长,由班长进行每个小组的分数统计,评出优秀小组。‎ 研修中心对该校教师教案进行抽查发现,这所学校的教师每堂课讲课的时间都不会超过20分钟,其余是学生提问题或者师生共同讨论时间。这套教学改革方案的成效在高考中得到体现,在今年的高考中,这所学校近八成的学生总分超过600分,创历史新高,一本上线率连续3年突破99%。‎ 北京朝阳区教委近日也对辖区各校高考成绩进行了统计与分析,无论是一本上线率还是其他分数段比例,今年均是历年来最高的,有8所学校本科上线率达到100%。他们发现,不管是这8所学校还是其他高考成绩有所提升的学校,它们都是课改有尝试、有作为、有成果的学校。‎ 在北京各区总结课改成效与高考关系的同时,山西一所一直坚持“超课改”的学校——山西运城市新绛中学也传来好消息。一本、二本上线率均比去年有所提升,这已经是这所学校连续4年一本二本上线率上升。起点就是2008年山西开始的高中课改,“问题解决式”学习是新绛中学课改的精华。‎ 从实行课改以来,这所学校的升学率一直在提升,前两年每年提升10%,2011年的高考二本上线率一下子提升了30%,2012年和今年,一直在延续上升的势头。‎ 新绛中学校长宁致义认为,这是坚持真正课改,坚持把新课改的目标“自主、探究、合作”落实到每节课,落实到每个教师和学生身上的结果。‎ ‎(选自《中国青年报》,有删改)‎ ‎1.下列对文章有关内容的理解和概括,正确的一项是(  )‎ A.北京市朝阳区教委对所辖各高中高考成绩进行统计与分析发现,今年辖区内所有的高中高考成绩都有所提升,升学率都有所提高,这些成绩说明了新课改的成果。‎ B.“五步自主高效课堂”将学生预习、小组讨论、同学相互提问、老师课堂讲解等内容放在课堂上分为五步走,课堂变得更加民主,学生自主学习能力不断提高。‎ C.因为坚持课改,北京东城区各学校、山西运城的新绛中学都取得了连续三年一本、二本上线率上升的好成绩,特别是新绛中学,四年来升学率一直上升。‎ D.实行新课改的学校,为保证课改的深入进行,积极进行各种有益的探索,比如进行“问题解决式”学习方法、启发式和讨论式课堂教学模式的探究等。‎ 答案 D 解析 A项,原文“不管是这8所学校还是其他高考成绩有所提升的学校”说明高考成绩提高和升学率提高的不是朝阳区所有的高中,而是部分。B项,课堂的“五步走”中不包括“学生预习”这一内容。C项,“北京东城区各学校”“取得了连续三年一本、二本上线率上升的好成绩”于文无据,文中只是说,北京东城区“一本上线率连续3年保持六成以上,本科上线率连续3年保持九成以上”。‎ ‎2.实行新课改之前的传统教学有哪些弊端?请简要概括。‎ 答:___________________________________________________‎ ‎_______________________________________________________‎ ‎_______________________________________________________‎ 答案 ①课程内容“繁、难、偏、旧”,学生课业负担重。②课堂教学以教师为主,方式以填鸭式满堂灌为主,辅以大量作业重复训练。③学生在课堂上是被动学习,不能主动探索学习内容。④不能培养学生的语言能力、合作能力等。‎ 解析 本题考查筛选并整合文中信息的能力。回答问题时,要找到答案区间,筛选文章中与新课改形成对比的传统教育的表现。第2、3、5、6段是用对比的手法,写新课改前后的课堂教学内容、教学方式、老师表现、学生表现等方面的区别。‎ ‎3.北京市新课改取得成功的具体原因有哪些?请结合文章进行探究。‎ 答:___________________________________________________‎ ‎_______________________________________________________‎ ‎_______________________________________________________‎ 答案 ①深刻领会新课改的精神实质。北京市高中在课改方面能够深刻理解改革的实质,在不加大教学难度、学生的课业负担的情况下,提高了学生的知识水平和多方面的能力。一方面,强化对学生自学能力的培养。“五步自主高效课堂”从独立完成“自学案”开始,到课堂的讨论、提问、完成“检测案”等都凸显了对学生自学能力的培养。另一方面,改变教师角色。教师不再满堂灌,而是把时间还给学生,让学生成为课堂的主体。②贵在坚持。北京市高中实行新课改已经有6年,坚持走课改的道路,不走形式,坚持实实在在的课堂改革。‎ 解析 本题考查探究能力。回答问题时应注重分析北京市在实行新课改时的具体做法和具体做法所体现出来的课改精神,同时注重分析与之对比的未进行课改的做法,以此把握北京市新课改体现出的学校、课堂、学生等有关方面的特点,从而探究课改取得成功的原因。‎ 三、[2017·内蒙古质检]阅读下面的文字,完成1~3题。‎ 池莉专访:在生活中修行 ‎“车、房,所有这些面上的东西我都不热衷。”2007年的春天,池莉在酒店大堂一边看手机短信,一边果断总结。这个终日白描物质的女人,自己却在恰当的时机看穿逃离,大约有十余年的时间,池莉不开自己的作品研讨会,不请著名人物作序或者写书评,拒绝电视,关闭博客。她在武汉市郊安家落户,自己种菜自己吃,满足于劳动后一身大汗,享受着与人世两不相争的快活:“上午我在阅读以赛亚·柏林的书,下午我在菜地里干农活。当家家户户炊烟升起的时候,我倚靠在篱笆上休息。”‎ 在生活中修行 中国新闻周刊:你小说中的市井描写和你的清静生活差别很大,仿佛是一个人一边在茶馆说书,一边在庙里修禅。这种转换会让你觉得不适吗?‎ 池莉:没有。如果不这样就不是我了。我就是需要这样,我一直都很静,而且还一直在刻意地修炼自己内心的平静。海必须很深很静,才可以在表面翻腾出各种浪花,很浅的水都是泡沫。作家要目光清澈,首先物质欲望要很少。‎ 中国新闻周刊:你觉得自己达到了理想状态吗?‎ 池莉:修身养性不是一天是一辈子的事情。而且修炼得越到位,感觉越好。中国式样的生活是从政治到社会到人事,到城市的每个角落,是一个非常复杂的综合状态。作家看得越深越透越好,写得越像越客观越好。‎ 中国新闻周刊:除了阅读,你通过什么途径来观察这种复杂生活呢?‎ 池莉:不需要什么刻意的途径啊!我就在生活。生活对我来说无处不在。不是你不出去就没有生活。对于一个修炼到一定层次的人来说,光是看和思想,就足可以达到生活的最深处;而一些生活现象与场景,在写作中就是技术性问题,很简单了。‎ 中国新闻周刊:你会担心被时代淘汰吗?‎ 池莉:任何时代都有不被淘汰的作家,也都会有作家被淘汰。作家和社会的变化是没有直接的关系的。关键在于作家自己。最好的作家无须把所有的职业都做完了再来写作。他把握的是人类的灵魂和精神状态。‎ 幸福感与成败无关 中国新闻周刊:作家出版社1989年出版的文学新星丛书——《烦恼人生》里,你在前言中介绍,我只剩下了一个愿望:好好过日子。这个愿望如今变化与否?‎ 池莉:我在80年代后期发愿,所谓好好过日子,就是要创建自己的生活方式。学会远离世俗热闹,断绝名利诱惑,让自己的文学感觉更敏锐,思想更深远和明澈,更有能力理解真实的众生并写好每一个文字。‎ ‎20年前人年轻,名利愿望很强烈,不过同时也已经感觉和意识到了浮云遮日的可怕,那时候我要求自己开始“好好过日子”,也就是好好开始个人的修炼,目标是争取十年还有读者。感谢上苍!我的愿望基本实现。不过,我还在坚持修炼和进步。‎ 中国新闻周刊:对于你来说幸福的元素除了写作,除了女儿,还有什么?‎ 池莉:现在我对幸福的要求,不落在什么具体的元素上。不需要外部的任何形式来给予我幸福感。甚至它不具体地落在孩子、落在写作身上,更不落在名利上头。只要有心情,有能力而且有时间做我自己一心想做的事情,我觉得都很幸福,和事情的成败没有关系。‎ 中国新闻周刊:现在的你,为何而写作?为谁写作?‎ 池莉:现在的我,与从前的我一样:为自己天生的热爱而写作,为神交的读者而写作,为我母语的众生而写作。‎ 知天命之年的池莉看上去大方得体,衣服颜色搭配及装扮和谐,头发一丝不苟,还有隐约的脂粉香,更像是一名外企女高管,精力充沛,不讲废话,谨慎而知进退。张爱玲在《我看苏青》里总结,驻颜有术的女人总是:一、身体相当好;二、生活安定;三、心里不安定。因为不是死心塌地,所以时时注意到自己的体格容貌,知道当心。‎ ‎《池莉影记》里有一张她上大学时的照片,连衣裙的棉布是她费好大劲才买到的,款式是自己设计的,并且为此制作了一条项链,巧妙遮住了一小块疤痕。如今的池莉仍然善于修饰,保养得当,甚至连声音都很年轻,包括语气和声调的起伏,略熟以后,她的诚恳和天真就像泉水里的泡泡一串串冒出来,但绝不水花四溅。‎ 采访中,她拒绝摄影师近距离拍照,她自嘲不上相,“所有的摄影师都不喜欢拍我,没有轮廓”。她认为自己最好的照片都是家常的,因为只有家里人才懂得怎么拍她。她做得一手好菜,女儿赞扬作家妈妈的手艺,总是能够让自己胃口大开。‎ ‎1.下列对材料有关内容的理解和分析,不符合原文意思的一项是(  )‎ A.开篇以语言描写刻画人物,以池莉本人的话来表达她对物质生活的态度,一个不在乎物质奢华而追求生活本质的人物形象跃然纸上。‎ B.本文大多通过池莉本人的讲述来表现她对于物质生活的态度及其当下的生活方式,让读者感觉到真实且亲切可靠的作家生活。‎ C.池莉认为生活无处不在,从政治到社会到人事,再到城市的每个角落,是一个非常复杂的综合状态,正好为自己提供了一个最后的平台,自己就是需要在生活中修行,因此一直在刻意地修炼自己内心的平静。‎ D.自古文人很注重自然和生活汲取,注重修行。池莉崇尚老庄的自然美学观,又是生活劳动在田园之中,很自然地,田园生活便成了她的审美对象化。‎ 答案 D 解析 池莉认为修身养性是一辈子的事情,修炼得越到位,感觉越好;崇尚老庄的自然美学观的是陶渊明一类的隐士。‎ ‎2.下列对文章有关内容的分析和概括,最恰当的两项是(  )‎ A.作为作家的池莉从始至终坚持“为自己天生的热爱而写作,为神交的读者而写作,为我母语的众生而写作”。‎ B.池莉对于复杂生活的观察不需要任何途径,因为生活对她而言无处不在,光是看和思想就足够了。‎ C.池莉的幸福不落在孩子、写作身上,更不落在名利头上。她强调只要有心情做自己一心想做的事情,就觉得幸福。‎ D.作为一篇人物专访(或人物通讯),本文全方位多角度地概括了池莉最主要的特点,即在生活中修行。‎ E.在写作上,本文大多通过语言描写来表现池莉对于物质生活的态度以及她当下的生活方式,让读者感觉真实可靠。‎ 答案 AE 解析 B项“不需要任何途径”绝对化;C项“池莉的幸福不落在孩子、写作身上”不符合文意,原文是“甚至它不具体地落在孩子、落在写作身上”;D项“全方位多角度”不准确。‎ ‎3.即使不认识传主,通过作者的叙述你也会在脑海里产生一个“立体”的池莉形象,请结合文本简要分析。‎ 答:___________________________________________________‎ ‎_______________________________________________________‎ 答案 ①淡泊名利,纯粹宁静;②不看重物质,追求精神的富有;③随性自然,享受清净自由;④注重修身养性;⑤始终坚持为热爱而写作的理念,有普世情怀;⑥生活中,为人诚恳谨慎,精神充沛,注重形象,厨艺精湛等。‎ 解析 答题时首先要求审清题干,确定试题的类型是局部信息筛选还是综合信息筛选,局部信息筛选要对对应的段落进行划分,提取关键词语;综合信息筛选注意对局部或全文内容的概括,重点注意各段的段首和段尾的句子。本题作答时,主要从传主精神、性情、写作理念和生活方面进行总结和分析。‎ 四、[2016·石家庄二中模拟]阅读下面的文字,完成1~4题。‎ 悬在空中的河流 我们都知道,依靠浮力,船能够在水中漂浮;依靠浮力,羽毛能够随风飘浮,然而,你听说过飞机在几万米的高空,突遇“空中河流”,竟然发生了高空漂浮现象吗?‎ ‎1982年4月的一天,在冲绳岛美军那霸空军基地,有5架当时最为先进的F16战斗机升空,在1.2万米的高空作编队变化演习。当时晴空万里,西太平洋的上空能见度非常高,可以说是高空飞行演习的最好天气。‎ 突然间,飞机在雷达屏幕上排成“人”字形不动了!在场的所有人都被这突然的变化惊呆了。当时的指挥员菲尔德上校认为,一定是雷达系统出了毛病,急令雷达兵仔细检查。经过检查,所有的雷达运转都是正常的。也就在此时,无线电波传来了呼叫之声,飞行队队长报告:“报告指挥官,不好了,飞机的发动机突然间全部熄火了,我们的飞机像漂浮在河水之中。四周都是水,飞机已经不受我们控制了……”话尚未说完信号就中断了,接着雷达荧屏上的飞机也消失了。‎ 菲尔德上校目瞪口呆地坐在椅子上,这位有丰富飞行经验的空军上校突然向周围的人冒出一句话:“不!绝不可能!在1万多米的高空,滴水成冰,哪来的河水?”‎ 最令人费解的是,这几架演习飞机当时在高空突然熄火,并未下滑,而是被一股无形的力量托住,处于高空漂浮的状态。从驾驶员的报告中可以看出,飞机遇上了离奇的“空中河流”,漂浮在高空的水中。‎ 这几架正在演习的战斗机就这样离奇地消失了。‎ ‎“空中河流”的怪事早在第二次世界大战中就曾经发生过。‎ ‎1943年5月,美国和日本的海空军在太平洋南部各岛屿之间展开激烈的拉网战。美军的空军英雄——曾击落过多架日军零式战斗机的戈巴得里上尉,驾驶着一架侦察机在所罗门群岛一带海域侦察日军联合舰队的动向。‎ 当戈巴得里上尉驾机飞行到大巴里尔岛的海岸线一带时,飞机突然在高空中停滞不前,一个发动机熄火,仅靠另一个发动机继续飞行着。接着飞机开始后退,被一股无形的力量推动着,像漂浮在水面上一样。戈巴得里上尉惊恐万分,他认为是日军发明了最新式秘密武器,慌忙将高空所遇的情况报告给了美军基地指挥部。指挥官命令他马上设法返航。好在戈巴得里上尉驾驶技巧娴熟,他立即掉转头来返回了基地。‎ 经过检查,这架飞机像受了重创,机翼开裂,机身和发动机均受到损伤。当时高空气流稳定性良好,无任何雷雨,四周能见度又高,那么,是什么力量使飞机不能前进反而后退呢?机翼、机身、发动机为什么会遭到如此损伤呢?‎ 唯一的解释是,飞机突遇了高空强气流形成的“空中河流”,发生了高空漂浮现象。‎ 近几年来,美国的气象学家们运用计算机进行模拟分析后认为,形成所谓“空中河流”的原因是空中的大气急流因地球的运转而产生的磁场变化引起的冷热交叉,形成了一股如汹涌波涛般的气流,这股强大的气流如同水流一样威力无比。“空中河流”能够使具有强大动力的飞机倒行逆驶,或者停滞不前。同时,它又具有强大的浮力,使飞机如同漂浮在水面上一样,还会因局部缺氧而使飞机的发动机熄火。‎ 美国气象学家史蒂罗迈教授认为,如果所谓“空中河流”的特异现象确实存在,那么科学界应花大本钱、下大功夫来揭示其奥秘,因为如果此奥秘被揭开,那么人类将大大受益。如今地球上最缺少的是淡水,由于缺水,大量的良田变成沙漠。如果能运用上“空中河流”的水,那么将会使沙漠变良田,永无旱灾之忧,将会使地球成为绿色的田园……‎ 美国的军事科学家们对此高空特异现象也很重视,他们曾建议美国政府拨专款供科学家们研究。他们认为,谁掌握了“空中河流”‎ 的奥秘,谁就能在空中绝对称霸,再也不怕什么远程战略轰炸机、洲际弹道导弹和超现代化高速战斗机了。因为它能使一切空中飞行物体停止运行、改变航向直到坠毁,所以美国政府再也不用花上千亿美元去研制导弹防御系统了。‎ ‎(选自《知识窗》,有删改)‎ ‎1.下列对文章有关内容的分析和概括,最恰当的两项是(  )‎ A.菲尔德认为,在1万多米的高空不可能有河水存在,所以面对F16战斗机在雷达荧屏上消失的情况,他不相信飞机真失踪了。‎ B.从驾驶员的报告看,5架F16战斗机在高空突然熄火,被无形的力量托住而没有下滑,所以才会遇上离奇的“空中河流”。‎ C.尽管当前的气象科学十分发达,但面对离奇的高空漂浮现象,气象学家们还是无法做出具有说服力的圆满解释。‎ D.在高空强大气流的作用下,戈巴得里的飞机机翼开裂,机身和发动机均有损伤,戈巴得里幸亏经验丰富才成功返航。‎ E.美国政府花大本钱支持对“空中河流”奥秘的研究,渴望早点揭开奥秘,一方面造福人类,另一方面取得空中绝对霸权。‎ 答案 CD 解析 A项,“他不相信飞机真失踪了”表述错误。由原文可知,菲尔德不相信的是“空中河流”的说法,而不是飞机失踪的事实。B项,“所以才会遇上”表述错误。前后句之间没有因果关系。E项,“美国政府花大本钱支持”表述错误。原文中花大本钱研究仍处于建议阶段,并非既成事实。‎ ‎2.简要分析文章第1自然段在文中的作用。‎ 答:___________________________________________________‎ ‎_______________________________________________________‎ 答案 ①以生活中常见的浮力现象引出对“空中河流”这一奇异现象的介绍。②用浅显易懂的原理初步解释高空漂浮现象存在的可能性,为下文作铺垫。③总领全文内容。‎ 解析 本题考查赏析文章结构特点的能力。首段在文中的作用从内容和结构两个方面回答,结合文本进行分析;另外,首段通常有为下文作铺垫和总领全文的作用。‎ ‎3.作者为什么要详细介绍美国战斗机离奇消失和“二战”中美军侦察机遭遇的怪事?‎ 答:___________________________________________________‎ ‎_______________________________________________________‎ 答案 ①将“空中河流”的现象具体化,让人容易理解。②为下文科学家们的种种解释提供事实依据。③说明“空中河流”现象虽然离奇但不是个例,而是普遍真实的存在。‎ ‎4.请结合文章探究“悬在空中的河流”这一标题的意蕴。‎ 答:___________________________________________________‎ ‎_______________________________________________________‎ ‎_______________________________________________________‎ 答案 ①“空中河流”现象是一种极为离奇的自然现象。②科学界对这一离奇现象的解释仍停留在猜想的层面,莫衷一是。‎ 五、[2017·广东模拟]阅读下面的文字,完成1~4题。‎ 排球不是美国女排姑娘的全部梦想 南方周末:郎指导,你觉得作为美国女排主教练,是你改变了美国队还是美国队改变了你?‎ 郎平:不同的文化使得执教美国队和执教中国队有很大不同。美国人认为我给美国队注入了更多的亚洲元素,比如在打法上更细腻,作风更严谨。但从我内心来说,应该说是美国队很大程度上改变了我。‎ 美国队队员文化程度都很高,并且学习速度很快,我很尊重她们思考问题的方式。美国姑娘们很放松的心态,为兴趣而打球的状态影响了我,让我绷得很紧的神经也放松不少。她们似乎更享受生活的丰富多彩,排球不是她们生活的全部梦想。‎ 刚开始我对她们的心态不很理解,因为我一直是挺较劲地在打球,每一分都在拼,是一种非抓住胜利不可的心态。当我作为一个外来者,浸入了这一种有历史而又被大多数人认同的国家体育文化中,我想使劲改变她们,但是她们的文化势能比我更茁大,所以只能换个角度去理解她们,所以说美国队改变了我。现在我可以欣赏她们这种非职业的轻松心态。就像弹簧的力量来自于能保持柔韧的弹性一样,我不得不说,美国姑娘在随性自然的心态中,处理排球时富于创造性,常有神来之笔。‎ 南方周末:在美国做主教练的经历让你学到了什么?‎ 郎平:国外的训练科学性更强。他们更注重对人体结构细致入微的研究,比如身体哪一块肌肉如何调节,计算多少热量能承担多大的运动量,更细致精确,确实让我更新了思维。‎ 从心态上说,美国的优势在于放松。我们对体育比赛的追求更多是只允许成功,不允许失败,导致过度紧张。而美国文化在乎的是参与精神,是你有没有尽自己的最大努力。只要发挥了你的能力和潜力,你就是胜利者,就能获得承认,而不是在心理上将自己逼到无路可退的死角。他们在胜负问题上更能保持平常心,输球了很简单,就是重新再来。‎ 南方周末:这些观念你是从一开始就适应,还是有磨合期?‎ 郎平:我带美国队开始时,说实话非常不适应。因为在国内,教练有比较大的权威,队伍非常整齐,各项训练配套都很有保障。但在美国完全不是这样,她们更强调个人的兴趣,难免显得自由松散。即使打奥运会,我作为国家队教练,就连想把全体运动员集中起来都非常困难——她们在各州训练,有各自的安排,你也很难按计划调度她们。‎ 北京奥运会开始前2个多月,我的队员还是不齐。有个主力去做手术,奥运会也参加不了;还有两个主力是伤员,到大奖赛时能不能打都不知道。哎,愁得慌。这种实际条件逼得我抛掉依赖计划的习惯,考验我随时随地根据队伍的变化调整训练思路。你不能指望游刃有余的深思熟虑,没这个条件。现实条件随时在变化,不到最后一分钟你没法作决定。‎ 我们经历了很长的磨合期。她们都很有想法,强调自己的状态。有时我批评她们比赛不到位,她们会用“不在状态”来回应。当时比较郁闷,在中国比赛,你很难以“不在状态”来作为成绩差的借口,状态好不好你都要拿下。你不允许自己放松,全国人民的眼睛都盯着你,强大的国家荣誉感需要你的成功去支撑,不能动不动就闹情绪。‎ 南方周末:美国之前,你在意大利职业队曾执教6年。‎ 郎平:意大利和美国打球很不一样。意大利以职业联赛为主,比赛的细则,球员和教练的责任和义务都很明确,大家心照不宣,必须尊重商业合同。在意大利打比赛,主要是商业压力。教练也就是个打工的角色,得听老板的。有时候会非常累心,商业利益是残酷的,没有太多人情味。你必须适应。当时女儿在美国,我想跟她呆在一起,就离开了意大利。‎ 南方周末:你的个人经历正好体验了三种不同的体育体制。‎ 郎平:中国、意大利和美国正好代表三种不同的体育体制。如果你要轻松型的模式,自由发挥你的兴趣,享受排球的乐趣,美国模式最适合你;如果你要通过高强度的职业联赛来训练,意大利的模式更能练兵;如果要拿世界冠军,中国的体制比较好,经济上、人才、时间都更有保障。‎ ‎(选自《南方周末》郎平访谈录)‎ ‎1.在美国执教的经历,使郎平在哪些方面有了改变?‎ 答:___________________________________________________‎ ‎_______________________________________________________‎ ‎_______________________________________________________‎ 答案 打球时要有放松的心态,在对待胜负问题上保持一颗平常心;训练科学性强,更新了她的思维;抛掉依赖计划的习惯,学会根据现实条件变化调整训练思路;尊重队员,尽量减轻他们的心理压力,以民主平和的态度对待他们。(答出其中三点即可)‎ 解析 本题考查筛选并整合文中信息的能力。答题时审清题干,锁定区位,概括文意作答。找到“在美国执教的经历,使郎平的改变”分条概括。‎ ‎2.为什么郎平说“排球不是美国女排姑娘的全部梦想”呢?‎ 答:___________________________________________________‎ ‎_______________________________________________________‎ ‎_______________________________________________________‎ 答案 美国女排姑娘打球时心态放松,是出于自己的兴趣爱好去打球,享受着排球的乐趣,不会为了责任去打球。只要发挥了自己的能力和潜力,就是胜利者。她们似乎更享受生活的丰富多彩,所以郎平说“排球不是美国女排姑娘的全部梦想”。‎ 解析 本题考查筛选并整合文中信息的能力。分析题干隐含的信息,“排球不是美国姑娘的全部梦想”,抓住“全部梦想”分析。‎ ‎3.郎平体验了三种不同的体育体制。请简要概括各有什么特点。‎ 答:___________________________________________________‎ ‎_______________________________________________________‎ ‎_______________________________________________________‎ 答案 第一种,中国模式;特点:经济上、人才、时间都更有保障,有强大的国家荣誉感,但缺少个人自由度。第二种,美国模式;特点:轻松型的模式,自由发挥你的兴趣。第三种,意大利模式;特点:通过高强度的职业联赛来训练,商业利益是残酷的,没有太多人情味。‎ 解析 答题时注意审清题干,然后锁定区位,概括文意作答。此题是综合筛选的题目,要立足原文,找到“三种体育制度的特点”,分条概括。‎ ‎4.在美国女排队员比赛不到位时,她们会用“不在状态”来回应。你对这种用“不在状态”来作为成绩差的借口的做法,持何种态度?并说明理由。‎ 答:___________________________________________________‎ ‎_______________________________________________________‎ ‎_______________________________________________________‎ 答案 这种做法是可以理解的。美国的体育文化和中国不同,她们不像中国队员那样不能以“不在状态”来作为成绩差的借口。在中国强大的国家荣誉感需要你的成功去支撑。美国队员身上没有那么重的责任感,她们更重视个人的兴趣和生活的丰富多彩,重视个体的生命质量。这种轻松的心态虽然显得自由散漫,但是更利于发挥她们的潜能,发挥她们的创造力。所以我们不能以中国的文化观念去面对美国人,如果站在美国文化心态的基础上去看待美国队员的做法,就会感觉她们做法的合理性。(如果从反面作答,只要言之成理即可)‎ 解析 这是一道探究的题目,既涉及文章的内容,又要拓展,首先立足文本谈谈用“不在状态”来作为成绩差的借口的做法的认识,用文中的事例进行说明,然后谈自己的认识。‎ 滚动提升训练(八)‎ ‎  时间:45分钟   满分:55分 一、语言文字运用(29分)‎ ‎1.[2017·天津南开中学模拟]下列各组词语中加点字的读音,全都不相同的一组是(3分)(  )‎ A.纯粹 淬火 翠绿 鞠躬尽瘁 出类拔萃 B.沮丧 诅咒 狙击 含英咀华 减租减息 C.抠门 讴歌 怄气 呕心沥血 并驾齐驱 D.茎叶 径庭 痉挛 经年累月 泾渭分明 答案 C 解析 A项,都读作“cuì”;B项,读音分别为“jǔ、zǔ、jū、jǔ、zū”;C项,读音分别为“kōu、ōu、òu、ǒu、qū”;D项,读音分别为“jīng、jìng、jìng、jīng、jīng”。‎ ‎2.[2017·温州十校联考]下列词语中没有错别字的一组是(3分)(  )‎ A.由于缺乏有效的监管,互联网上色情反动等负面的信息屡见不鲜。很多辨别是非能力较弱的青少年因此而荒费学业,成为“电子海洛因”的吸食者。‎ B.如果仅为了商业利益,将民俗改头换面欺骗消费者,使民俗失去了原本的风貌,就会遗害匪浅。对这种“伪民俗”现象,我们一定要正本清源。‎ C.外交部副部长李辉在19日约见俄驻华大使时表示,中方对“新星”号货船沉没事件发生感到震惊,并对俄方的推脱态度表示强烈不满。‎ D.气象部门的统计表明,近来中国华南地区连续降水偏少,旱情漫延粤桂琼三省——广东、广西两省局部地区已出现严重旱情,海南也发生全省性干旱。‎ 答案 C 解析 A项,荒废;B项,贻害匪浅;D项,蔓延。‎ ‎3.[2017·莱芜质检]下列语句中标点符号使用恰当的一项是(3分)(  )‎ A.“冰冻三尺,非一日之寒。”做好一件事往往需要一个长期的积累过程,写作更是“路漫漫其修远兮”。‎ B.对于外来投资,应该是你投资,我欢迎;你盈利,我收税;你违法,我查处;你破产,我清算,这才是正常的。‎ C.谁也不能突出,如果你想比别人多动、多干、多拥有,那就甭指望有好日子过。这是平庸社会敬奉的一条多么可怕的生存法则!‎ D.我告诉他们:“是众多的爱(父母之爱,手足之爱,爱人之爱,甚至许多陌生人之爱),让我活得有勇气,有希望。”‎ 答案 A 解析 B项,“这”之前的逗号不当。C项,顿号应为逗号。D项,括号应为破折号。‎ ‎4.下列各句中,加点成语的使用,不恰当的两项是(3分)(  )‎ A.虽然最初并不相信自己获得了助学金,但由于电话那头的骗子言之凿凿,加上所谓教育局的文件,信息闭塞的徐玉玉最终成了骗子的猎物。‎ B.马帮在茶马古道上已传承千年,在中国大西南的土地上,马帮的赶马人用他们自己的方式走过了山高水低,走过了春夏秋冬。‎ C.只要上下一致,勠力同心,站在新的历史起点上,全省人民必定打赢精准扶贫攻坚战,确保贫困地区、贫困群众与全省人民一道同步迈入全面小康。‎ D.降央卓玛演唱了8首歌曲,这些草原歌曲各有特色,曲尽其妙,洋溢着浓郁的亲情、友情,让歌迷们尽情地回味草原风情。‎ E.构建基于互联网的创新人才培养平台是一项系统工程,不可能一蹴而就,需要我们摒弃那些限制和阻碍创新人才培养的僵化模式、陈旧思维和不良风气。‎ 答案 BD 解析 A项,言之凿凿:形容说话有真凭实据,而不是空泛的用语。使用正确。B项,山高水低:指意外发生的不幸事情,多指死亡。望文生义。此处应为“险山恶水”。C项,勠力同心:指齐心合力,团结一致。使用正确。D项,曲尽其妙:指把其中微妙之处委婉细致地充分表达出来,形容表达能力很强。在句中不能用来陈述“歌曲”。E项,一蹴而就:指踏一步就成功,比喻事情轻而易举,一下子就成功。使用正确。‎ ‎5.[2017·河南模拟]填入下面文段空白处的词语,最恰当的一组是(3分)(  )‎ 让孩子选择未来,__①__不是放手不管不问,__②__在既有的经验和孩子的意愿之间,寻找最恰当的平衡点,并__③__给孩子更多的尊重。自主选择,__④__一种意愿,__⑤__一种能力,__⑥__这种能力呼应了时代的要求。‎ ‎①‎ ‎②‎ ‎③‎ ‎④‎ ‎⑤‎ ‎⑥‎ A ‎/‎ 要 尽力 不仅是 也是 因为 B 并 而是 尽可能 是 更是 ‎/‎ C 也 ‎/‎ 一定要 不但是 同样是 所以 D 虽然 而是 尽可能 是 ‎/‎ 当然 答案 B 解析 本题考查语言表达连贯、准确的能力。②空,根据上文中的关联词语“不是”可知,要选择“‎ 而是”,排除A、C两项;⑤空,根据语意“自主选择是意愿”“自主选择是能力”可知,两者是递进关系,所以选择“更是”,排除D项。‎ ‎6.[2017·珠海模拟]下列各句中,没有语病的一句是(3分)(  )‎ A.通过深化改革,大家更加清醒地认识到,只有全面深化改革,才是解决我市发展面临的一系列突出矛盾和挑战、实现经济持续健康发展的根本途径。‎ B.今年初刚建成的“幸福村居”,每天都可接待游客数十桌,实现收入3000元以上,每年预计可为集体经济创收50多万元。‎ C.“民生工程”最终变成“形象工程”,数以亿计的投资打了水漂,造成巨大财政浪费不说,更严重的是,搭进了当地政府的公信力。‎ D.就我个人而言,我非常喜欢《香河》这部作品反映出作家深厚的生活积累和饱满的创作感情。‎ 答案 C 解析 A项,“解决我市发展面临的一系列突出矛盾和挑战”中,“解决……挑战”搭配不当,删去“和挑战”。(正确的搭配是“应对挑战”。)B项,语序不当。“每年预计可为集体经济创收50多万元”语序不当,应为:预计每年可为集体经济创收50多万元。D项,“我非常喜欢《香河》这部作品反映出作家深厚的生活积累和饱满的创作感情”句式杂糅,它包含了两句话:我非常喜欢《香河》这部作品,它反映出(了)作家深厚的生活积累和饱满的创作感情。也可增加一个虚词来改变句子的结构:我非常喜欢《香河》这部作品所反映出的作家深厚的生活积累和饱满的创作感情。‎ ‎7.[2017·四川模拟]在下面一段文字横线处补写恰当的语句,使整段文字语意连贯,内容贴切,逻辑严密。每处不超过15字。(5分)‎ 电视台身负公共文化服务的职责,__①__,而应该传播社会正能量。现在的许多电视节目,尤其是综艺节目粗制滥造,不仅缺少原创性,而且__②__。比如,有的明星毫不掩饰对金钱的崇拜,有的明星频频利用桃色新闻吸引人的眼球,还有的明星不顾伦理道德在节目上乱认“干爹”“干妈”。因此,许多家长认为__③__,还不如让孩子看动画片。‎ 答:①__________________________________________‎ ‎②______________________________________________‎ ‎③______________________________________________‎ 答案 ①不应该传播负面内容 ②给青少年带来了极大的消极影响 ③与其让孩子们观看综艺节目(言之有理即可)‎ 解析 这是一道根据文意填空的题目,答题时注意前后文的照应,以及语意的提示,如第①空注意后文中的“传播社会正能量”,第②空注意后面的“金钱的崇拜”等,第③空注意后文中的“还不如”。‎ ‎8.认真观察下图,按要求回答问题。(6分)‎ ‎(1)用简练的语言描绘画面内容,不超过20个字。‎ 答:___________________________________________________‎ ‎_______________________________________________________‎ ‎(2)请为该图配一句话。要求:语言表达准确、鲜明、生动,不超过20个字。‎ 答:___________________________________________________‎ ‎_______________________________________________________‎ 答案 (1)在长长的跑道上,一只蜗牛向着阳光前进。‎ ‎(2)只要坚持,梦想必将实现。‎ 解析 本题考查图文转换及语言表达准确、鲜明、生动的能力。第(1)问,要客观描述画面内容,不要加自己的想象和联想。从画面来看,主要涉及阳光、跑道和蜗牛,所以描绘时要把这三者作为重点。第(2)问,为图配一句话,要注意突出画面的内涵,并且要注意语言的准确、鲜明、生动。‎ 二、文学类文本阅读(14分)‎ 阅读下面的文字,完成9~11题。‎ 夜遇阿克哈巴河 王族 怎么说呢,看到阿克哈巴河的那一刻,我的第一个感觉是,它不是一条河。‎ 天慢慢地黑了。夜幕完全拉开的时候,一抬头就看见了挂在天边的月亮,新疆的地广,所以,经常能看见天上一边挂着太阳,一边挂着月亮。要是在白天,月亮就只能悄悄地在天上挂着,一般不会被轻易发现;而一旦太阳落山,天刚麻麻黑,你总能看见远处的天边先亮了起来,不一会儿,那片光亮越来越大,一直涌到你的眼前。‎ 此时,阿克哈巴河上游被一片月光照白。那片月光慢慢向下,几乎是顺着河道在移动。我看到在月光的移动中,河水变得更白了,由于月光在动,河水似乎也在向下汹涌,这种汹涌是一团白光的涌动,越来越快,似乎已经倾泻起来。‎ 月光顺着河道从我面前移动过去。在越过我的时候,我看见河水的内层被照亮,很深,也很厚重。月光移动过去之后,河面只有一层淡淡的亮光,仍让人觉得阿克哈巴河不是一条河,而是别的什么。‎ 这时候,一位哈萨克族牧民骑着马,一边往这边走,一边唱着歌。空旷的夜晚忽然有了他的歌声,宁静和孤独一下子就被打破了。他走到我跟前,从马上跳下来,愣愣地望着月光中的阿克哈巴河。我觉得他有点奇怪,怎么忽然瞅着阿克哈巴河就发起了呆。过了一会儿,他表情非常复杂地看了我一下,然后转过身去,准备牵马离去。‎ ‎“哎,佳克斯——”我不知道为什么忽然想和他说几句话,就使用了这句哈萨克语,叫了他一声。他听到我的叫声后,停了下来,准备去牵马的那只手在半空中犹豫了一下,还是收了回去。他走到我跟前,也像我一样说了一句“哎,佳克斯——”。他的声音很有磁性。‎ 我们两个人都不说话,临河而立,望着月光中的阿克哈巴河,长久地沉默着。此时的阿克哈巴河河面仍旧是一片铁青,我仍然感觉它不是一条河。这时候,我发现他的右手上有血。再仔细一看,他的那只手正在流血,一滴一滴的鲜血从指缝里流出,滴在沙土中。此时月光正亮,因而他的那只手看上去黑乎乎的,可以肯定已经有大量的血流了出来。“你的手?你……”他把手伸到我跟前。我看见他的手心扎着一根筷子粗的骆驼刺。他把手翻过来,他的手让我触目惊心,那根骆驼刺刺穿了他的掌心,又露出手背两三寸。我知道紧挨着阿克哈巴河的山坡上,到处都长着骆驼刺。骆驼刺较之于其他沙漠植物,似乎有着钢铁铸就的枝叶,其枝坚硬无比,其叶锋利如刃,人和动物一旦碰到骆驼刺,必然会被划破皮肤,如果碰得重了,则会被刺入肉中。‎ ‎“你这是怎么回事?”“刚才,我的马看见阿克哈巴河被月光照亮,就狂跑起来,我不小心跌落在地上,这根骆驼刺就钻进了我的手心。”“疼不疼?”“有一点点。”我扭头去看犯下错误的那匹马,它仍然在出神地望着阿克哈巴河。看它的样子,它很想向着阿克哈巴河一跃而入,但拴在它脖子上的那根缰绳却被它的主人紧紧地抓在手中。‎ ‎“我本来想在河水中把手上的血洗掉,但一看见阿克哈巴河,我从来都没有发现它在月光中会是这样。我不洗了。”说完,他翻身上马,两腿用力一夹马腹,那匹马就奔腾而起,驰向远处。不一会儿,远处传来了他的歌声。我知道,此时他跟刚才来阿克哈巴河边时一样,正高声唱着歌。而那些鲜血,伴着歌声,正从他的指缝里一滴一滴地落入沙漠。‎ 文章写到这里,我才记起,当时他面部的颜色和阿克哈巴河一样,都是铁青色的。‎ ‎9.下列对作品有关内容和艺术特色的鉴赏,最恰当的一项是(4分)(  )‎ A.开头说“看到阿克哈巴河的那一刻,我的第一个感觉是,它不是一条河”,是因为作者看到阿克哈巴河后产生错觉。‎ B.文章第三、四段中,对月光的描写很有意味。作者描写了月光和河水的动态,月光之“动”与河水之“动”相映成趣,有一种别样的美感。‎ C.文中多处写到了马,牧民说“我的马看见阿克哈巴河被月光照亮,就狂跑起来”,直接表现了阿克哈巴河的奇异之美。‎ D.写牧民“当时他面部的颜色……都是铁青色的”,突出表现了新疆的哈萨克族人受当地自然风物的影响,脸色都为铁青色。‎ 答案 B 解析 A项,“是因为作者看到阿克哈巴河后产生错觉”一句错,作者不是产生了错觉,而是有意这么说,强调河的奇异之美,灵性十足。C项,文中多处写到的马,侧面烘托了阿克哈巴河的奇异之美。D项,“受当地自然风物的影响”的说法于文无据,另外,“脸色都为铁青色”的说法武断,不合实际。‎ ‎10.作者用较多篇幅写牧民“手上的血”,那鲜血“一滴一滴地落入沙漠”,这样写有何用意?(4分)‎ 答:___________________________________________________‎ ‎_______________________________________________________‎ ‎_______________________________________________________‎ 答案 ①写他被骆驼刺刺伤了手,血流下来,他却并不太在意,突出了他的性格特点:坚强、刚毅。②写他在受伤后,仍被月光下的阿克哈巴河吸引,被阿克哈巴河的美震撼,突出了自然之美和他内心对美的崇敬。‎ 解析 本题考查鉴赏文学作品的形象和表达技巧的能力。解答此题,首先一定要在原文中找出相关的文字,对之进行仔细分析。通过分析,我们可以确定描写鲜血“一滴一滴地落入沙漠”等文字主要是细节描写,分析这一细节描写的意图,可联系上下文。通常来说,细节描写用来写人,目的当然主要是刻画人物形象,而刻画人物形象最终是为表现主题服务,表达作者的某种主张或寄托某种情感。我们可以往这些方面考虑。本文中写血“一滴一滴地落入沙漠”,如果是一般人,可能会惊慌,会急着去医治,但牧民没有这样,表现出他性格坚强,毫不在意伤口;如果没有阿克哈巴河的美吸引他,他也会赶快回去医治的,但他却停下来领略河之美,表现出他被河之美震撼,以及内心对美的崇敬。‎ ‎11.联系全文,简要分析阿克哈巴河在文中的作用。(6分)‎ 答:___________________________________________________‎ ‎_______________________________________________________‎ ‎_______________________________________________________‎ 答案 全文以阿克哈巴河为线索,将人、事、景融为一个有机的整体,使景美和人美相互衬托,将新疆广大土地的神奇,哈萨克族牧民的热情粗犷以及牧民受伤后的坚强、勇毅串联起来。作者描写阿克哈巴河,不仅写出了阿克哈巴河巨大的美,而且通过阿克哈巴河表现了新疆大自然的美丽神奇,更是通过阿克哈巴河表现新疆各族人民坚强、刚毅的性格和博大的内心美。‎ 解析 本题综合考查鉴赏作品形象、分析作品的结构的能力。本题设问“分析阿克哈巴河在文中的作用”,这是一种常见的题型,即“分析某一物件(事物)在文中的作用”。考虑这一类问题,可以从结构、内容(情感)、主题等方面入手。通常来说,该物件作为线索的作用、物件的象征作用以及作为抒情的载体(或情感的寄托,可以是作者的,也可是文中人物的)的作用等三方面的作用是非常常见的,值得注意。因此,考生可从这几个方面出发来作答本题。‎ 三、实用类文本阅读(12分)‎ 阅读下面的文字,完成12~14题。‎ 而我却今天才知道他的存在 柴静 我们这些知道李政道、钱学森、钱三强、王淦昌的人,原本都应该知道他——他是他们的老师。‎ 李政道大二的时候,是他破格选送去美国,当时李政道才19岁,穿着短裤去办护照,办公的人员都不相信“怎么会是个儿童?”李政道后来说“他决定了我的命运”。华罗庚是初中生,是他让其在清华算学系任职,又送去英国深造,华罗庚说“我一生得他爱护无尽”。‎ 那是战乱烽火时代,但后来的重要科学发展所依仗的这些人,是他在那时满地焦土上栽下的桃李。‎ ‎——可是我为什么不知道他?‎ 他生在上海,父亲是旧式文人,让他从小读经史子集。他幼年已经以君子“慎独”‎ 之道要求自己,修身自省,对跟朋友之间“因小故而致割席”之事也写在笔下:“一时之忿,至今思之,犹有隐痛。”‎ 他讷于言,但一生都保持温润如玉的君子之风。1915年,他在清华上学的时候,成立清华校史上的第一个学生团体——科学会。1918年,他留学美国,后来在哈佛读博士,导师是诺贝尔物理奖获得者布里奇曼。他的第一个研究课题,是用X射线短波极限法精确测定基本作用量子h值。实验结果,在美国《科学院院报》和《光学学会学报上》发表,很快被国际科学界公认为当时最精确的h值。这一数值被国际物理学界沿用达16年之久。这一年他23岁。‎ 他27岁回国,于清华执教,执教之严也是出名的,他的课给李政道的分数只是83。他允许这学生不听自己的课,“因为你看的参考书比我的更高明”,但是“你的实验做的不认真,要扣去25分”。‎ 他去世后多年,亲人发现他一直留着当年的那三张答卷,写在泛黄的昆明土纸上。他是清华物理系主任,这对他自己来说其实是一种牺牲,相当于要放弃了自己的专业研究来作行政的工作。因为他把聘任第一流学者到清华任教列为头等大事。从1926~1937年,他先后为物理系和理学院聘来了熊庆来、吴有训、萨本栋、张子高、黄子卿、周培源、赵忠尧、任之恭等一批学者。‎ 吴有训还只不过是刚到校的普通教师,资历年纪都不如他,他把吴有训的工资定得比自己还高,1934年,他引荐吴有训接替自己的物理系主任一职。四年后,他力主吴有训接替自己的理学院院长一职,那时他正当盛年。冯秉铨毕业的时候,他对他们说:“我教书不好,对不住你们。可是有一点对得住你们的就是,我请来教你们的先生个个都比我强。”‎ 他在1929年又组建了清华理学院,其中包括算学、物理、化学、生物、心理、地学6系。‎ 他说凡是出人才的地方,必然是科学文化最盛行、科学土壤最肥沃、科学气氛最浓厚之地。比如欧洲的哥廷根、慕尼黑和美国的芝加哥等。‎ 中国科学研究停滞数千年,第一次有了这滚热得烫手的雄心:“除造就科学致用人才外,尚谋树立一研究科学之中心,以求中国之学术独立。”‎ ‎1929年,他在一篇叫《中国科学界之过去、现在和将来》的文章里说“有人怀疑中国民族不适宜研究科学,我觉得这些论调都没有根据。中国在最近期内方明白研究科学的重要,我们还没有经过长时期的试验,还不能说我们缺少研究科学的能力。惟有希望大家共同努力去做学研究,五十年后再下断语。诸君要知道,没有自然科学的民族,决不能在现代立脚得住。”‎ 他终身未娶,唯与学生亲厚,当中有一人叫熊大缜。‎ ‎1938年,熊突然对他说要去冀中抗日。他明知这学生在河北没有依靠相熟的人,又没有政治经验,但是国难当头,他只能送他去,熊走后,他曾“约有十余天,神思郁郁,心绪茫然,每日只能静坐室中,读些英文小说,自求镇定下来。”‎ 他唯一能安慰的一点,是他能够帮着自己的学生在后方搜购一些雷管、炸药等军用物资。‎ 看这书时,我才知道,曾经炸碎日军机车车头的TNT药性地雷,是来自熊所在的“技术研究社”的制造,而不是我们小时看的电影《地雷战》中由农民土法制成。新中国成立后他仍然当过一段清华的一把手,一直到1951年。‎ 八十年过去了,他在空白处栽种的一切,让我这样的后代得以生活在一个浓荫蔽头的世界上,而我却今天才知道叶企孙先生的存在。‎ ‎(摘编自柴静《而我却今天才知道他的存在》)‎ 相关链接 ‎①1992年5月,叶企孙的老学生和老同事185人捐赠了几万元,成立“叶企孙奖”基金,以奖励品学兼优的学生。1992年,海内外127位知名学者联名向清华大学呼吁为叶企孙建立铜像,1995年在清华大学校园,一代名师叶企孙的铜像立了起来。‎ ‎②晚年,叶企孙还作了许多科学史的研究,他的一些科学史研究论文迄今还是研究科学史的样板。‎ ‎12.下列对材料有关内容的理解和分析,不符合原文意思的一项是(3分)(  )‎ A.本文记述传主时先从他的学生李政道、华罗庚说起。这种欲扬先抑的写法,是为了突出传主的形象。‎ B.献身科学,积极探索,研究成果被国际物理学界沿用多年。晚年研究科学史,一些科学史研究论文迄今还是研究科学史的样板。‎ C.从他研究出被国际物理界沿用16年的h值,以及到了晚年仍然做科学史研究,可分析出他献身科学、不断追求探索的精神。‎ D.从他跟学生说“我教书不好,对不住你们。可是有一点对得住你们的就是,我请来教你们的先生个个都比我强”。说明他谦虚、真诚。‎ 答案 A 解析 李政道、华罗庚是举世闻名的科学家,而他们的恩师叶企孙先生却不为人知,这是设置悬念,由此引出下文,启人思考,并不是欲扬先抑的写法。‎ ‎13.下列对传记有关内容的分析和概括,最恰当的两项是(4分)(  )‎ A.叶企孙先生出身于书香世家,自幼学习经史子集,以君子“慎独”之道要求自己,修身自省,始终保持温润如玉的君子之风。‎ B.在那战乱烽火时代,叶企孙先生不拘一格选拔培养人才,为中国科学事业的发展奠定了基础,体现了他独到的眼光与开阔的胸襟。‎ C.担任清华物理系主任期间,叶企孙先生把刚到校教书,资历年纪都不如他的普通教师吴有训的工资定得比自己还高,后来又力主吴有训接替自己的理学院院长一职,这对他自己来说其实是一种牺牲。‎ D.抗战期间自己的得意门生熊大缜投身革命队伍,叶企孙先生积极支持他报效祖国的行动。他还想方设法在后方搜购军用物资,师生同心协力为反抗日本侵略者做出了贡献。‎ E.文章开头就说我们“原本都应该知道他”,中间部分又说到“可是我为什么不知道他”,最后写道“而我却今天才知道叶企孙先生的存在”,旨在体现叶企孙先生默默耕耘、无私奉献的精神。‎ 答案 BD 解析 A项,“出身于书香世家”说法不当,原文中只说他父亲是旧式文人;C项,“这对他自己来说其实是一种牺牲”属于曲解文意,原文的表述是“他是清华物理系主任,这对他自己来说其实是一种牺牲,相当于要放弃了自己的专业研究来作行政的工作”;E项,“旨在体现……”表述有误,重点强调了“我”以前不知道他很是遗憾,以及对他不被人知道的原因的反思。‎ ‎14.在《中国科学界之过去、现在和将来》一文中,先生说“没有自然科学的民族,决不能在现代立脚得住”。请结合材料、联系现实,谈谈你对这句话的理解。(5分)‎ 答:___________________________________________________‎ ‎_______________________________________________________‎ 答案 ①这是先生对我们民族苦难深重的历史与现实的深刻审视之后的觉悟。叶企孙先生生活的时代中国可谓内忧外患:外有列强侵略,遍地烽火;内则积贫积弱,被动挨打。先生痛感科学不兴,国家不强,民族难立。故有此深刻的认识。②这也是先生对中国科学界之同人的激励与鞭策。在有人对我们民族研究科学的能力持怀疑态度之时,先生并未轻言放弃,而是希望同人充分认识到自然科学的重要性,努力钻研,以实际行动证明中华民族是有无穷创造力的民族,中华民族能够自立于世界民族之林。③这更是先生及几代科学家科学强国的美好愿景。在完成我们民族文化的伟大复兴,实现中国梦的今天,科学强国、实干兴邦已成共识。‎ 解析 本题考查探究文本中的某些问题,提出自己的见解的能力。解题时,首先要理解“没有自然科学的民族,决不能在现代立脚得住”这句话的含意(结合当时的社会现实,以及作为中国科学界的领袖面临的处境),自然科学的成功对于当时中国的作用与影响,以及对于提升自身的抗御能力,提升国家的形象等,然后联系现实,做到从文本出发,有理有据。‎